You are on page 1of 1304

A-PDF Merger DEMO : Purchase from www.A-PDF.

com to remove the watermark

Anglo-Chinese Junior College


Physics Preliminary Examination
Higher 2

PHYSICS 9646/01
Paper 1 Multiple Choice 31 Aug 2010
1 hour 15 minutes
Additional Materials: Multiple Choice Answer Sheet

READ THESE INSTRUCTIONS FIRST

Write in soft pencil.


Do not use staples, paper clips, highlighters, glue or correction fluid.
Write your Name and Index number in the answer sheet provided.

There are forty questions in this section. Answer all questions. For each question there are
four possible answers A, B, C and D.
Choose the one you consider correct and circle your choice in soft pencil on the separate
Answer Sheet.

Read the instructions on the Answer sheet very carefully.

Each correct answer will score one mark. A mark will not be deducted for a wrong answer.
Any rough working should be done in this Question Paper.

This paper consists of 20 printed pages


2

DATA AND FORMULAE


Data
speed of light in free space, c = 3.00 × 108 m s−1
permeability of free space, μo = 4π × 10−7 H m−1
permittivity of free space, εo = 8.85 × 10−12 F m−1
(1/(36π)) × 10-9 F m-1
elementary charge, e = 1.60 × 10−19 C
the Planck constant, h = 6.63 × 10−34 J s
unified atomic mass constant, u = 1.66 × 10−27 kg
rest mass of electron, me = 9.11 × 10−31 kg
rest mass of proton, mp = 1.67 × 10−27 kg
molar gas constant, R = 8.31 J K−1 mol−1
the Avogadro constant, NA = 6.02 × 1023 mol−1
the Boltzmann constant, k = 1.38 × 10−23 J K−1
gravitational constant, G = 6.67 × 10−11 N m2 kg−2
acceleration of free fall, g = 9.81 m s−2

Formulae
uniformly accelerated motion, s = ut + 1
2
at 2
2
v = u 2 + 2as
work done on/by a gas, W = p ΔV
hydrostatic pressure, p = ρgh
Gm
gravitational potential, φ = −
r
displacement of particle in s.h.m., x = xo sin ωt
velocity of particle in s.h.m., v = vo cos ωt
= ± ω xo2 − x 2
3
mean kinetic energy of a molecule of an ideal gas E = kT
2
resistors in series, R = R1 + R2 + …
resistors in parallel, 1/R = 1/R1 + 1/R2 + …
Q
electric potential, V =
4πε o r
alternating current/voltage, x = xo sin ωt
transmission coefficient, T ∝ exp(−2kd)
8 π 2 m (U − E )
where k =
h2
radioactive decay, x = xo exp(−λt)
0.693
decay constant, λ = t1
2

ACJC 2010 H2 9646 Prelim Exam P1


3

1 Which of the following SI units can be expressed in exactly two base SI units?

A coulomb
B tesla
C newton
D hertz

2 The viscosity of a fluid, μ, can be determined by measuring the terminal velocity, vT, of
a sphere when it descends in the fluid. The fluid has a density ρf while the sphere has
a density ρs and a diameter of d. The viscosity can then be calculated by the formula

5( ρs ρf )
μ= d2
9v T

The values measured are


vT = (1.60 ± 0.04) m s−1
ρs = (2700 ± 20) kg m−3
ρf = (900 ± 10) kg m−3
d = (20.0 ± 0.4) mm

What is the percentage uncertainty in the value of μ?

A 6.2 % B 7.1 % C 8.2 % D 8.4 %

3 A car is travelling at a velocity of 24 m s-1 due west initially. At a later time, it is seen
travelling at a velocity of 10 m s-1 due south.

Given that the direction North N, points vertically upwards, which of the following vector
R represents the change in velocity of the car?

A B C D
N N N N

R R
R R

ACJC 2010 H2 9646 Prelim Exam P1 [Turn over


4

4 Which displacement-time graph best represents the motion of a falling sphere, the
initial acceleration of which eventually reduces until it begins to travel at constant
terminal velocity?

A B

C D

5 A motorist travelling at 10 m s–1 can bring his car to rest in a braking distance of 10 m.
In what distance could he bring the car to rest from a speed of 30 m s–1 using the same
braking force?

A 17 m B 30 m C 52 m D 90 m

6 Two equal masses travel towards each other on a frictionless air track at speeds of 60
cm s–1 and 30 cm s–1. They stick together on impact.

What is the speed of the masses after impact?

A 15 cm s–1
B 20 cm s–1
C 30 cm s–1
D 45 cm s–1

ACJC 2010 H2 9646 Prelim Exam P1


5

7 In 2001, the leaning tower of Pisa was stabilised by a bundle of steel cable anchored to
the ground as shown by the simplified sketch below.

Tower

Steel Cable
ground

The weight of the tower is W, the tension in the cable is T and the reaction from the
ground is R.

Which vector triangle best represents the forces acting on the tower?

A B
T T

W R R W

C D
T T

R W W R

8 The area of the horizontal cross-section of a barge is 97 m2 and the sides of the barge
are vertical. The flat bottom of the barge is 70 cm under water when it is loaded with
2.0 x 104 kg of cargo.

How deep would the bottom of the barge below the water surface when the cargo is
unloaded off the barge? Density of sea water is 1030 kg m-3.

A 20 cm B 50 cm
C 70 cm D 90 cm

ACJC 2010 H2 9646 Prelim Exam P1 [Turn over


6

9 The moon remains in its orbit around the Earth rather than falls to the Earth because

A it is also attracted by the gravitational forces from the sun and other planets
B the net force on the Moon is zero
C the gravitational force exerted by the Earth on the moon provides a net force
that provides the Moon’s centripetal acceleration.
D the magnitude of the gravitational force from the Earth is too small to cause any
appreciable acceleration of the Moon

10 A person of weight 500 N does a bungee jump using an elastic rope of unstretched
length 40 m and having a spring constant k equal to 50 N m-1. During the initial fall
there is a transfer of energy from gravitational potential energy to kinetic energy and
elastic potential energy. The person falls through a distance of 80 m before beginning
to move upwards.

Which set of graphs correctly represent the variation of the three energies?

A B
50 50

45 45

40 40

35 35
Energy / kJ

Energy / kJ

30 30

25 25

20 20

15 15

10 10

5 5

0 0
0 10 20 30 40 50 60 70 80 90 100 0 10 20 30 40 50 60 70 80 90 100

Distance of fall / m Distance of fall / m

C D
50 50

45 45

40 40

35 35
Energy / kJ

Energy / kJ

30 30

25 25

20 20

15 15

10 10

5 5

0 0
0 10 20 30 40 50 60 70 80 90 100 0 10 20 30 40 50 60 70 80 90 100

Distance of fall / m Distance of fall / m

ACJC 2010 H2 9646 Prelim Exam P1


7

11 A satellite orbits the Earth 200 km above its surface. The satellite's acceleration
towards the centre of the Earth is 9.2 m s−2 and the radius of the Earth is 6400 km. The
speed of the satellite is

A 246 km s−1 B 7.79 km s−1 C 7.67 km s−1 D 1.36 km s−1

12 The gravitational field strength at a point P on the Earth's surface is numerically equal
to

A the acceleration of free fall at P.


B the change in potential energy per unit distance from P.
C the force acting on any body placed at P
D the work done in bringing unit mass from infinity to P

13 Planet Z has a mass of 8.0 × 10 24 kg and a radius of 6.2 × 10 7 m. Neglecting air


resistance, the energy required to lift a mass of 10 kg from its surface into outer space
is

A 46.1 × 10 6 J
B 64.1 × 10 6 J
C 86.1 × 10 6 J
D 96.1 × 10 6 J

14 The escape speed of a nitrogen molecule at the Earth's surface is 0.90 × 10 4 m s-1.
What is the escape speed at a height 0.30 RE above the Earth's surface, where RE is
the radius of the Earth?

A 0.49 × 10 4 m s-1
B 0.59 × 10 4 m s-1
C 0.69 × 10 4 m s-1
D 0.79 × 10 4 m s-1

ACJC 2010 H2 9646 Prelim Exam P1 [Turn over


8

15 The graphs in Fig. 15 show how the displacement x, velocity v and the acceleration a
of a body vary with time t when it is oscillating with simple harmonic motion.
What is the value of T?

Fig. 15
π 2π π 2π
A s B s C s D s
9 9 3 3

16 Because of air resistance, the amplitude of oscillation of a simple pendulum decays


exponentially with time. How does the total energy of the pendulum vary with time?

A It varies sinusoidally with time at the same frequency as that of the pendulum
B It varies sinusoidally with time at a frequency twice that of the pendulum
C It decreases at a steady rate
D It decreases exponentially with time

17 A constant power supply is used to melt 1 kg. of ice, to heat the water produced, and
finally to turn all the water to steam.

Specific heat capacity of water = 4 x 103 J kg-1 K-1


Specific latent heat of fusion of ice = 3 x 105 J kg-1
Specific latent heat of vaporization of water = 2 x 106 J kg-1

Which graph in Fig.17, best shows how the thermodynamic temperature T varies with
time t for this sequence?

A B C D

Fig. 17

ACJC 2010 H2 9646 Prelim Exam P1


9

18 Which statement about internal energy is correct?

A The internal energy of a system can be increased without transfer of


energy by heating
B The internal energy of a system depends only on its temperature
C When the internal energy of a system is increased, its temperature
always rises.
D When two systems have the same internal energy, they must be at the
same temperature.

19 The diagram below shows the relation between the pressure and the volume of
the gas in an engine for one cycle of operation of the engine.

Work is done by the gas in the engine during

A Q→R only.
B Q→R and R→S.
C S→T only.
D S→T and Q→R.

ACJC 2010 H2 9646 Prelim Exam P1 [Turn over


10

20 The diagram shows the snapshot of a transverse wave at a particular instant. The
wave is traveling to the right. The frequency of the wave is 12.5 Hz.

Q P

At the instant shown the displacement is zero at the point P.


What is the shortest time to elapse before the displacement is zero at point Q?

A 0.01 s B 0.02 s C 0.03 s D 0.07 s

21 A sound wave of frequency 400 Hz is traveling in a gas at a speed of 320 ms-1.

What is the phase difference between two points 0.1 m apart in the direction of travel?

π π 2π 4π
A rad B rad C rad D rad
4 2 5 5

22 Two coherent monochromatic waves of equal amplitude are brought together to form
an interference pattern on a screen. Which of the following graphs could represent the
variation of intensity with position (x) across the pattern of fringes.

A B

C D

ACJC 2010 H2 9646 Prelim Exam P1


11

23 Fig 23 shows the formation of the first order spectrum when parallel rays of
monochromatic light fall perpendicularly on a non-uniform spacing diffraction grating
PQR. For the part of the grating between P and Q, the angle of deviation θ is constant,
whilst for that between Q and R, θ decreases.
diffraction grating
R
θ

x Q
Angle of deviation, θ
decreases
P

Incident rays Diffracted parallel


rays

Fig 23

Which diagram best shows how the grating interval d varies with distance x, the
distance from P?

A B
d d

P Q R x Q x
P R
C D

d d

P Q R x P Q R x

ACJC 2010 H2 9646 Prelim Exam P1 [Turn over


12

24 In the figure shown, a point charge +Q is placed at X and another point charge -2Q is
placed at Y.

X Z Y
+Q - 2Q
r

Which of the following correctly gives the magnitude of the electric field strength and
1
electric potential at point Z, a distance of r from X assuming they are isolated
4
charges.

Electric field strength Electric potential


44Q 5Q
A
9πε 0 r 2 3πε 0 r
44Q Q
B
9πε 0 r 2 3πε 0 r
28Q Q
C
9πε 0 r 2 3πε 0 r
28Q 5Q
D
9πε 0 r 2 3πε 0 r

25 The figure shows a uniform electric field.

-60V -40V -20V 0V 20V

X Y direction of motion of the


charge

A charge of -5 µC placed at point X is projected horizontally towards Y. Which one of


the following statements is correct?

A The electric field is directed from X to Y and there is a loss of kinetic energy as
the charge moves from X to Y
B The electric field is directed from Y to X and there is a loss of kinetic energy as
the charge moves from X to Y
C The electric field is directed from X to Y and there is a gain of kinetic energy as
the charge moves from X to Y
D The electric field is directed from Y to X and there is a gain of kinetic energy as
the charge moves from X to Y

ACJC 2010 H2 9646 Prelim Exam P1


13

26 The graph below shows the variation with current I of the potential difference V across
a filament lamp.

I/mA 2.0

1.5

1.0

0.5

0.0
0.0 0.2 0.4 0.6 0.8 1.0 1.2
V/V

The resistance of the lamp when I = 1.5 mA is

A 950 Ω
B 400 Ω
C 0.40 Ω
D 0.0025 Ω

ACJC 2010 H2 9646 Prelim Exam P1 [Turn over


14

27 In the circuit below, the battery has negligible internal resistance. Three identical lamps
L, M and N having the same resistance are connected as shown.

The filament of lamp N breaks. Which one of the following shows the subsequent
changes to the brightness of lamp L and lamp M?

Lamp L Lamp M
A stays the same Decreases
B increases stays the same
C increases Decreases
D decreases Increases

28 The diagram shows a light-dependent resistor (LDR) and a thermistor (with a negative
temperature coefficient of resistance) forming a potential divider.

Under which set of conditions will the potential difference across the thermistor have
the greatest value?

Light intensity Temperature


A Low Low
B High Low
C Low High
D High High

ACJC 2010 H2 9646 Prelim Exam P1


15

29 In the potentiometer circuit below, the moveable contact is placed at N on the bare wire
XY, such that the galvanometer shows zero deflection.

The resistance of the variable resistor is now decreased.

What is the effect of this decrease on the potential difference across the wire XY and
on the position of the moveable contact for zero deflection?

Potential Difference across XY Position of moveable contact


A Increases Nearer to X
B Increases Nearer to Y
C Decreases Nearer to X
D Decreases Nearer to Y

30 A proton beam of velocity 2.00 × 107 m s-1 enters a velocity selector with a plate
separation of 2.00 cm, and has a magnetic flux density of 1.5 T directed out-of-
plane of the paper.
If the protons pass through the velocity selector undeflected, what would be the
direction and magnitude of the electric field? You may ignore any relativistic effects
and assume no electrical breakdown occurs.

proton beam

direction magnitude

A downwards 6.00 x 105 N C-1

B upwards 6.00 x 105 N C-1

C downwards 3.00 x 107 N C-1

D upwards 3.00 x 107 N C-1

ACJC 2010 H2 9646 Prelim Exam P1 [Turn over


16

31 A compass is placed above a wire. The compass needle is seen to point toward
the north. When a current is made to flow through the wire, the needle deflects to
point along the northwest direction.

The orientation of the wire and direction of the current are

orientation of wire direction of current

A north-south south to north

B north-south north to south

C east-west west to east

D east-west east to west

32 A current balance is used to measure the magnetic flux density B of a electromagnet. The
side PQ of a current balance is inserted inside a large electromagnet. The direction of
magnetic field is as shown in Fig 32. Length of PQ is L. PQ and RS are d1 and d2
respectively from the pivot. A load of mass m is placed along side RS. Take acceleration
due to gravity to be g.

The direction and magnitude of the current along PQ are

P
S
B

Q R

Fig 32

direction magnitude

m d2
A from P to Q
B L d1

m g d2
B from P to Q
B L d1

m d2
C from Q to P
B L d1

m g d2
D from Q to P
B L d1

ACJC 2010 H2 9646 Prelim Exam P1


17

33 The diagram below shows two concentric loops in the same plane.

outer loop

inner loop

The variation of the magnitude of the current in the inner loop varies with time as
shown below and it is flowing clockwise.

current

time

The effect on the induced current in the outer loop is such that it is

A increasing in the anticlockwise direction


B constant in the anticlockwise direction
C constant in the clockwise direction
D decreasing in the clockwise direction

34 A lamp of resistance R is connected in series to a source of alternating voltage. The


r.m.s. value of the voltage is 20 V. The variation with time t of the power P dissipated in
the light bulb is shown below

The best estimate for the value of the peak current of the filament of the lamp is

A 2.5 A B 2.5 2 A C 5A D 5 2 A

ACJC 2010 H2 9646 Prelim Exam P1 [Turn over


18

35 An ideal transformer has a primary coil with Np turns and a secondary coil with Ns
turns. An alternating voltage supply of frequency f and average power of Po is
connected to the primary coil.

Which of the following correctly gives the frequency and average power in the
secondary coil?

Frequency Average Power


Ns Ns
A f Po
Np Np
Ns
B f Po
Np
Ns
C f Po
Np

D f Po

36 Which one of the following shows the correct sequence of events in a helium-neon laser?

A Neon atoms excite helium atoms to a metastable state. Excited helium atoms
undergo stimulated emission of red light to a lower energy state followed by
spontaneous emission to ground state.
B Neon atoms excite helium atoms to a higher energy state. Excited helium atoms
undergo spontaneous emission to a metastable state followed by stimulated
emission of red light to ground state.
C Helium atoms excite neon atoms to a metastable state. Excited neon atoms
undergo stimulated emission of red light to a lower energy state followed by
spontaneous emission to ground state.
D Helium atoms excite neon atoms to a higher energy state. Excited neon atoms
undergo spontaneous emission to a metastable state followed by stimulated
emission of red light to ground state.

37 Which of the following statements below is NOT true regarding an intrinsic semiconductor?

A The total current flow is the sum of both ‘hole’ and ‘electron’ currents.

B In a p–n junction, free electrons near the junction in the n–type material diffuse
across the junction into the p–type material. Diffusion occurs because of different
thermal agitation of atoms in the n–type and p–type material.
C Metals and semiconductors have different electrical properties because
semiconductors have a small energy gap.
D In an intrinsic semiconductor, the valence band is completely filled and the
conduction band is empty at room temperature.

ACJC 2010 H2 9646 Prelim Exam P1


19

38 The following graph shows the spectrum of X-rays emitted from an X-ray tube.

intensity

wavelength

If the potential difference between the target and cathode is increased, which one of the
following combinations represents a possible change in minimum wavelength, and the
wavelengths of the peaks?

minimum wavelength wavelengths of the peaks

A decrease increase

B decrease remain the same

C increase increase

D increase remain the same

39 The graph represents the decay of a newly prepared sample of radioactive nuclide X to
a stable nuclide Y. The half-life of X is τ. The growth curve for Y intersects the decay
curve for X after time T.

What is the time T ?


τ ⎛τ ⎞
A B ln⎜ ⎟ C τ D 2τ
2 ⎝2⎠

ACJC 2010 H2 9646 Prelim Exam P1 [Turn over


20

40 The graph shows how the binding energy per nucleon of a nucleon of a nucleus varies
with nucleon number, A.

Which of the following statements is not true?

A Energy is released in nuclear fission reactions from nuclei in region P.


B Nuclei in region Q are more stable than nuclei in region R.
C Nuclear fusion reactions bring nuclei closer to region Q.
D The binding energy per nucleon increases most significantly at lower nucleon
numbers.

ACJC 2010 H2 9646 Prelim Exam P1


1

Anglo-Chinese Junior College


Physics Preliminary Examination
Higher 2

CANDIDATE
CLASS
NAME

CENTRE INDEX
S
NUMBER NUMBER

PHYSICS 9646/02
Paper 2 Structured Questions
20 Aug 2010
Candidates answer on the Question Paper. 1 hour 45 minutes
No Additional Materials are required

READ THESE INSTRUCTIONS FIRST

Write your Name and Index number in the spaces on all the work you hand in.
Write in dark blue or black pen.
You may use a soft pencil for any diagrams, graphs or rough working.
Do not use staples, paper clips, highlighters, glue or correction fluid.

Section A
Answer all questions.
It is recommended that you spend about 1 hour 15 minutes on this section

Section B
Answer Question 8. For Examiners’ use
It is recommended that you spend about 30 minutes on this section only
1 / 6
At the end of the examination, fasten all your work securely together.
The number of marks is given in brackets [ ] at the end of each 2 / 9
question or part question. 3 / 7
4 / 7
5 / 7
6 / 6
7 / 18
8 / 12
Total / 72

This paper consists of 20 printed pages


2

DATA AND FORMULAE


Data
speed of light in free space, c = 3.00 × 108 m s−1
permeability of free space, μo = 4π × 10−7 H m−1
permittivity of free space, εo = 8.85 × 10−12 F m−1
(1/(36π)) × 10−9 F m−1
elementary charge, e = 1.60 × 10−19 C
the Planck constant, h = 6.63 × 10−34 J s
unified atomic mass constant, u = 1.66 × 10−27 kg
rest mass of electron, me = 9.11 × 10−31 kg
rest mass of proton, mp = 1.67 × 10−27 kg
molar gas constant, R = 8.31 J K−1 mol−1
the Avogadro constant, NA = 6.02 × 1023 mol−1
the Boltzmann constant, k = 1.38 × 10−23 J K−1
gravitational constant, G = 6.67 × 10−11 N m2 kg−2
acceleration of free fall, g = 9.81 m s−2

Formulae
uniformly accelerated motion, s = ut + 1
2
at 2
2
v = u 2 + 2as
work done on/by a gas, W = p ΔV
hydrostatic pressure, p = ρgh
Gm
gravitational potential, φ = −
r
displacement of particle in s.h.m., x = xo sin ωt
velocity of particle in s.h.m., v = vo cos ωt
= ± ω xo2 − x 2
mean kinetic energy of a molecule of an 3
ideal gas
E = kT
2
resistors in series, R = R1 + R2 + …
resistors in parallel, 1/R = 1/R1 + 1/R2 + …
Q
electric potential, V =
4πε o r
alternating current/voltage, x = xo sin ωt
transmission coefficient, T ∝ exp(−2kd)
8 π 2 m (U − E )
where k =
h2
radioactive decay, x = xo exp(−λt)
0.693
decay constant, λ = t1
2

2010 ACJC H2 9646/2


3 For
Examiner’s
Section A Use

Answer all questions


It is recommended that you spend about 1 hour 15 minutes on this section.

1 (a) Give reasonable estimates of the following quantities. In each case, give your
answer in an SI unit.

(i) The volume of the 2010 official World Cup soccer ball.

volume = m3 [1]

(ii) The kinetic energy of an olympic sprinter near the ending point of a 100-
metre dash.

Kinetic energy = J [1]

(iii) The density of the head of a human being.

density = kg m-3 [1]

(b) An experiment is conducted to determine the density of cooking oil, which floats
on water. Three sets of results are obtained from the experiment. The unit for
density is kg m−3.

A B C
810 1500 740
800 1490 870
805 1495 790

(i) Which set of results is precise but inaccurate? Explain your reasoning.

[2]

(ii) Assuming the measurement of volume has been done correctly, suggest
one possible source of experimental error that causes the aforementioned
set of results inaccurate.

[1]

2010 ACJC H2 9646/2 [Turn over


For
4 Examiner’s
Use
2 (a) A mass oscillates on the end of a spring in simple harmonic motion. The graph of
the acceleration a of the mass against its displacement x from its equilibrium
position is shown in Fig 2.1

Fig. 2.1

(i) Explain how the graph shows that the object is oscillating in simple
harmonic motion.

[2]

(ii) Show that the period of oscillation of the particle is about 0.40 s.

[2]

(iii) The mass is released when it is at a displacement x = + 0.050 m. Draw a


fully labeled graph on the axes of Fig. 2.2 of the displacement of the mass
until t = 1.0 s.

Fig 2.2

[2]

2010 ACJC H2 9646/2


5 For
Examiner’s
Use
(b) In order to check the speed of a camera shutter, the camera was used to
photograph the bob of a simple pendulum moving in front of a horizontal scale.
The extreme positions of the bob were at 600 mm and 700 mm marks. The
photograph showed that while the shutter was open the bob moved from the 650
mm mark to the 675 mm mark. If the period of the pendulum is 2 s, find the time
interval during which the shutter remained open.

[3]

3 A large horseshoe magnet produces a uniform magnetic field of flux density B between
its poles. Outside the region of the poles, the flux density is zero. The magnet is placed
on a top-pan balance and a stiff wire XY is situated between its poles, as shown in
Fig 3.1 below.

Fig 3.1

The wire XY is horizontal and normal to the magnetic field. The length of wire between
the poles is 4.4 cm. A direct current of magnitude 2.6 A is passed through the wire in
the direction from X to Y. The reading on the top-pan balance increases by 2.3 g.

(a) Deduce the direction of the force acting on wire XY.

Direction is [2]

(b) Deduce the polarity of the pole P of the magnet.

Polarity of P is [1]
2010 ACJC H2 9646/2 [Turn over
For
6 Examiner’s
Use
(c) Draw the resultant magnetic field lines acting on the wire XY within the poles of
the magnet on Fig 3.2 which shows the current flowing into the page.

Fig 3.2
[2]

(d) Calculate the magnetic flux density between the poles.

Magnetic flux density = …………. T [2]

2010 ACJC H2 9646/2


7 For
Examiner’s
4 You are tasked to investigates how the current through a 6.0 V filament lamp varies as Use
the potential difference across it is changed up to 6.0 V. You are supplied with the
following apparatus: a rheostat, a 9 V cell, an ammeter, voltmeter and connecting wires

(a) Draw a suitable circuit diagram for this investigation using only the above given
apparatus.

[2]

(b) (i) Use the axes below to sketch the graph showing how the current through
the lamp varies with the potential difference across it.

Current

Potential
difference
[2]

(ii) Justify the shape of the graph.

[2]
(iii) Explain how your graph shows that the resistivity of the filament is unique
at a specific temperature.

[1]

2010 ACJC H2 9646/2 [Turn over


For
8 Examiner’s
Use

5  (a) Fig 5 shows the variation of the photocurrent I with the potential of the anode with
respect to the cathode V, in the photoelectric experiment.  
I/nA

Fig 5

- Vs 0 V1 V/V

Suggest possible reasons for the following observations as seen from Fig 5.

(i) no photocurrent is detected for values of V lower than − Vs

(ii) increasing photocurrent for values of V between − Vs and 0 V.

(iii) saturation current was not achieved immediately when V became greater
than 0 V.

[3]

(b) An orbiting satellite can become charged by the photoelectric effect when
sunlight ejects electrons from its outer surface. Satellites must be designed to
minimise such charging. Suppose a satellite is coated with platinum, a metal with
a very large work function of 5.32 eV.

(i) Determine the longest wavelength of incident sunlight that can eject an
electron from platinum which has a work function energy of 5.32 eV.

λ = ……………… m [2]

(ii) The incident photon has a particle like nature. Determine the momentum of the
photon.

p = ……………… Ns [2]

2010 ACJC H2 9646/2


9 For
Examiner’s
Use
6 (a) The uncertainty in the measurement of the momentum, Δp of a bullet and an
electron are 2 x 10−3 kg m s−1 and 2.7 x 10−32 kg m s−1 when they have the
same speed of 300 m s-1 respectively.

(i) The Heisenberg position-momentum uncertainty principle can be stated


as follows:
h
ΔxΔp ≥
2
where Δx and Δp represents the uncertainty in the position and
momentum respectively.

Hence determine the uncertainty of locating the position of the bullet and
electron using the Heiserberg Uncertainty Principle.

Uncertainty in position for bullet


Uncertainty in position for electron [1]

(ii) Discuss the implication of your answers found in (i).

[1]

(b) The shaded region in Fig 6 shows the coulomb potential barrier as seen by the
alpha particle during its decay in polonium-212 and also the wave functions of
the alpha particle in regions I, II and III respectively.

Region I: Potential energy of


alpha particle due
to presence of
Nuclear and
Coulomb force

I Region II:Potential energy of


alpha particles
II when it is still within
the atom but
III experiences mainly
the Coulomb
repulsive force.

Region III: Potential energy


of alpha when out
Fig 6 of atom.
Note that 1 fermi metre = 10−15 m

2010 ACJC H2 9646/2 [Turn over


For
10 Examiner’s
Use

(i) Explain the phenomenon that tells us that alpha particle have the
probability of appearing in Region III.

[1]

(ii) Assuming a rectangular potential barrier of height 26.4 MeV taken at the
peak of the coulomb barrier and width 17.9 fm as shown by the dotted
lines, the half-life is determined to be 1.5 x 107 s. However, the actual
half-life is only about 0.30 μs.

Comment on the large difference on the order of magnitude of 13 using


the concept of the transmission probability and decay constant of the
alpha particles.

[2]

(iii) State a possible improvement to the method of determining the


transmission probability to get a better estimate of the half-life.

[1]

2010 ACJC H2 9646/2


11 For
Examiner’s
−1
7 (a) The vertical electric field E in the air is normally in the region about 100 V m Use
outdoors with negative charges on the earth’s surface. Ordinarily, the
equipotentials are parallel to the surface as shown in Fig. 7.1.

+300V

+200V
E = 100 V m-1
+100V

- - - - - - - - - - - - - - - - - -
Ground

Fig. 7.1

This means that outdoors the potential at the height of your nose is about 200
volts higher than the potential at your feet.

(i) By drawing equipotential lines on Fig 7.2 explain why a man standing in the
electric field similar to that in Fig. 7.1 do not get a shock when you go out
into the street if indeed the above discussion is correct.

Fig. 7.2

2010 ACJC H2 9646/2 [Turn over


For
12 Examiner’s
Use

[2]

(ii) During fair weather, the total potential difference from the surface of the
earth to the top of the atmosphere is about 400 000 volts. A current
caused by the electric field may pass from the sky down to the earth.

If the total electric current reaching the earth’s surface at any time is very
nearly constant at 1800 amperes, show that the current-density (current
per unit normal surface area) in the air is of the order of 10−12 amperes per
metre square. (Radius of the earth is taken to be 6380 km)

[1]

(iii) Estimate the electrical power reaching the earth’s surface.

[1]

(b) With such a large current coming down, the negative charge on the earth should
take only about half an hour to discharge. It is the thunderstorm and its lightning
that provides the “batteries” that keeps charging the earth up negatively. There
are about 40 000 thunderstorms per day all over the earth. Lightning storms
carry negative charges to the earth.

Thunderstorms occur when moist, warm air near the ground becomes buoyant
and rises to form clouds. Inside the clouds are millions of small water droplets
and ice suspended in air.

We will assume that we have a cumulus cloud that is 2 km deep with a similar
diameter and that it contains the order of 50-500 million water droplets per cubic
metre as shown in Fig 7.3.

2010 ACJC H2 9646/2


13 For
Examiner’s
Use

Fig. 7.3

(i) If each of these droplet is about 10 μm in radius show that the mass of the
cloud at the lowest reasonable density of the droplets is about 1.3 x 106 kg.
(density of water is 1000 kg m-3)

[3]

(ii) Hence determine the depth of rainfall recorded at the lowest reasonable
density of droplets should the cloud release all its water in one instant.
(Assume the area covered by the rain is the same as the cloud base)

[1]

2010 ACJC H2 9646/2 [Turn over


For
14 Examiner’s
Use
(c) Collisions among the water droplets and ice particles cause them to become
charged. The higher portion of the cloud acquires a net positive charge while the
lower portion of the cloud a net negative charge. Through induction, the earth
becomes positively charged.

The electrical potential difference between the earth surface and the base of a
cloud may be about several hundred millions volts in magnitude (108 V). Dry air
breaks down and becomes ionised at 3 millions volts per metre. When there is
an impending thunderstorm, the air is not dry. At a few hundreds of thousands
volts per metre, the conditions are ripe for resistance in the air to begin breaking
down and lightning, the electrical discharge between the regions of the cloud or
between the cloud and the ground, is formed. Electrons begin to flow from the
cloud to earth through a path of least resistance. They formed a funnel called the
step leader. It creates a channel of ionised air, which is an extremely good
conductor. When the step leader reaches the ground or any tall object near the
ground, the electrons quickly discharge to earth, followed quickly by electrons
higher up in the channel and followed again by the electrons in the next higher
successive levels in the channel. This results in a spectacular return stroke from
ground to the cloud which causes the intense lightning flashes that we observed
during an electrical storm.

Fig. 7.4 Formation of “Step Leader” Fig.7.5 Return Stroke

In a typical lightning flash, exchange of charges between the clouds and the
earth may be about 20 coulombs. The time which the lightning flash is estimated
to be about 1 millisecond.

(i) Show that the energy involved in the lightning discharge is in the order of
109 joules.

[1]

2010 ACJC H2 9646/2


15 For
Examiner’s
(ii) Hence, estimate the order of magnitude of the electrical power reaching Use
the earth’s surface during a thunderstorm.

[1]

(iii) Fig. 7.3 shows the magnitudes of the equipotential surfaces over the tower
at the time of a gathering storm.

Fig. 7.3
Estimate the magnitude of the electric field near the top of the tower and
hence explain if a return stroke is likely to occur at the top of the tower.

[2]

(d) The electrical discharge, results in an explosion of the air around the lightning
channel. Thunder is the sound wave produced as a result of the compression
wave propagating through the surrounding air.

(i) Explain why the explosion of the air occur.

[2]

2010 ACJC H2 9646/2 [Turn over


For
16 Examiner’s
Use
(ii) As light travels faster than sound (3.00 x 108 m s-1 vs. 330 m s-1) you see
lightning before you hear the thunder. The rule of thumb is if you count the
seconds between seeing the flash and hearing the thunder, take the
seconds and divide it by 3, it will give the distance of the lightning from
where you are standing in km. Explain using the appropriate data.

[1]

(e) Absolute measurements of sound intensity can be expressed in W m−2. However,


the human ear has a non-linear response to the energy content of sound. Hence
a logarithmic scale is used to describe the response of the ear in terms of sound
level L which is measured in decibels, dB where
⎛ I ⎞
L = 10 log⎜⎜ ⎟⎟ dB
⎝ I0 ⎠
where Io = 10−12 W m−2 is the values for the threshold of hearing and I = the
intensity of the sound being measured.

If the thunder produces 120 dB in the decibel meter when the lightning is
2.00 km away, determine the distance of the source of the lightning if you record
a reading of 124.8 dB assuming that sound energy is distributed uniformly from
the source of the thunder.

Distance = km [3]

2010 ACJC H2 9646/2


17 For
Examiner’s
Section B Use

It is recommended that you spend about 30 minutes on this section.

8 Many homes have smoke detectors fitted to the ceilings of certain rooms to provide an
early warning of a fire. These detectors contain a weak radioactive source that ionises
the air between two metal plates. See Fig. 8.

Fig 8

A low voltage battery in the detector causes the ions to move. This produces a very small
ionisation current in a circuit containing the battery and the plates. Any reduction in this
ionisation current due to smoke is detected and an alarm sounds.

An airline company wishes to install some of these smoke detectors in its aircraft and
needs to know if changes in air pressure will affect the ionisation current.

Design a laboratory experiment to investigate how the ionisation current depends on air
pressure.

The equipment available includes the following:

Alpha Radioactive source


Beta Radioactive source
Milliammeter /microammeter
Galvanometer
Ammeter
Power Supply
Bourdon Gauge
Manometer
Pressure gauge
Vacuum pump
Source handling tool
Lead container

You should draw diagrams to show the arrangement of your apparatus. In your account you
should pay attention to

(a) the equipment you would use for the investigation,


(b) the procedure to be followed,
(c) the control of variables,
(d) any safety precautions,
(e) any precautions that you would take to improve the accuracy of the experiment.
2010 ACJC H2 9646/2 [Turn over
For
18 Examiner’s
Use
Diagram

2010 ACJC H2 9646/2


19 For
Examiner’s
Use

2010 ACJC H2 9646/2 [Turn over


For
20 Examiner’s
Use

2010 ACJC H2 9646/2


Anglo-Chinese Junior College
Physics Preliminary Examination
Higher 2

CANDIDATE
CLASS
NAME

CENTRE INDEX
S
NUMBER NUMBER

PHYSICS 9646/03
Paper 3 Longer Structured Questions 27 Aug 2010
2 hours
Candidates answer on the Question Paper.
No Additional Materials are required

READ THESE INSTRUCTIONS FIRST

Write your Name and Index number in the spaces on all the work you hand in.
Write in dark blue or black pen.
You may use a soft pencil for any diagrams, graphs or rough working.
Do not use staples, paper clips, highlighters, glue or correction fluid.

Section A
Answer all questions.

Section B
Answer any two questions.

You are advised to spend about one hour on each section For Examiners’ use
only
At the end of the examination, fasten all your work securely together. Section A
The number of marks is given in brackets [ ] at the end of each
question or part question. 1 / 10
2 / 10
3 / 10
4 / 10
Section B
5 / 20
6 / 20
7 / 20
Total / 80

This paper consists of 18 printed pages


2 For
Examiner’s
DATA AND FORMULAE Use
Data
speed of light in free space, c = 3.00 × 108 m s−1
permeability of free space, μo = 4π × 10−7 H m−1
permittivity of free space, εo = 8.85 × 10−12 F m−1
(1/(36π)) × 10−9 F m−1
elementary charge, e = 1.60 × 10−19 C
the Planck constant, h = 6.63 × 10−34 J s
unified atomic mass constant, u = 1.66 × 10−27 kg
rest mass of electron, me = 9.11 × 10−31 kg
rest mass of proton, mp = 1.67 × 10−27 kg
molar gas constant, R = 8.31 J K−1 mol−1
the Avogadro constant, NA = 6.02 × 1023 mol−1
the Boltzmann constant, k = 1.38 × 10−23 J K−1
gravitational constant, G = 6.67 × 10−11 N m2 kg−2
acceleration of free fall, g = 9.81 m s−2

Formulae
uniformly accelerated motion, s = ut + 1
2
at 2
2
v = u 2 + 2as
work done on/by a gas, W = p ΔV
hydrostatic pressure, p = ρgh
Gm
gravitational potential, φ = −
r
displacement of particle in s.h.m., x = xo sin ωt
velocity of particle in s.h.m., v = vo cos ωt
= ± ω xo2 − x 2
mean kinetic energy of a molecule of an 3
ideal gas
E = kT
2
resistors in series, R = R1 + R2 + …
resistors in parallel, 1/R = 1/R1 + 1/R2 + …
Q
electric potential, V =
4πε o r
alternating current/voltage, x = xo sin ωt
transmission coefficient, T ∝ exp(−2kd)
8 π 2 m (U − E )
where k =
h2
radioactive decay, x = xo exp(−λt)
0.693
decay constant, λ = t1
2

ACJC 2010 H2 9646 Prelim Exam P3


3 For
Examiner’s
Section A Use

Answer all the questions in the spaces provided.


It is recommended that you spend about one hour on this section.

1 David had used a shepherd's sling to defeat Goliath. A shepherd's sling is a weapon
typically used to throw an object such as a stone to a location some distance away.

Suppose a modern day sling is used to set a stone of mass 52 g in horizontal


circular motion and when the stone reaches a speed v, it is released from the sling.
The stone travels in a parabolic path and hits the ground at point G which is at a
vertical distance H below the point of release of the stone from the sling and G is at
a horizontal distance X away from the point of release of the stone as shown in
Fig 1.

v
stone
position Y
Fig 1 H

X
G

(a) State three conditions necessary for the stone to move in a horizontal circular
path with constant speed.

[3]

(b) State and explain the change if any on the radius of the path and the tension
in the string when the stone is suddenly swirled at a lower speed.

[3]

(c) Suppose the stone is released from the sling at position Y with a speed of
22.8 m s−1 at a height H, of 42.5 m above the point of impact G, find the
horizontal distance X travelled by the stone when it hits the ground at G.

X= m [4]

ACJC 2010 H2 9646 Prelim Exam P3 [Turn over


4 For
Examiner’s
2 (a) Explain what is meant by an elastic collision. Use

[2]

(b) An ideal gas is contained in a thermally insulated cylinder by means of a


piston as shown in Fig 2.1. An atom of the gas collides with the piston, as
illustrated.

Fig 2.1
The piston in (b) is lowered so that the volume of the gas is reduced.
(i) Explain the difference in the speed of an atom of ideal gas after an
elastic collision with a moving piston and with a stationary piston
(assume mass of piston much greater than mass of an atom of the gas).
and hence use the kinetic theory to explain the changes to the
temperature of the gas as the piston is lowered.

[4]

ACJC 2010 H2 9646 Prelim Exam P3


5 For
st
Examiner’s
(ii) Use the 1 law of thermodynamics to support your answer obtained in (i). Use

[3]

(c) Sketch on Fig 2.2, with the help of the two isotherms given, the variation in the
pressure of the gas in the cylinder as its volume is decreased.

Pressure

Volume
Fig 2.2

[1]

ACJC 2010 H2 9646 Prelim Exam P3 [Turn over


6 For
Examiner’s
3 A microwave oven, shown in Fig 3.1 consists of a reflective casing on the opposite Use
wall of the microwave source and a turntable at the bottom that rotates. It produces
microwave of frequency 2.45 GHz within the oven.

Control
Reflective panel
casing

Turntable Microwave source


Fig 3.1 (behind control panel)

Water molecules are electric dipoles (that is, they have one positive end and one
negative end). In the oscillating field of the microwave oven, the water molecules in
trying to align with the changing field, oscillate rapidly. Thus the water molecules in
the food get heated up and hence the food gets heated up.

(a) Explain how are standing waves formed inside the microwave oven.

[3]

(b) The turntable of the oven is removed from the oven so that oven’s content
will not rotate during heating. A wet piece of cardboard is placed flat in a
microwave oven. The cardboard is then micro-waved for a short while. Stripes
of dry regions regularly spaced apart are seen on the cardboard.

(i) Explain the formation of these dry stripes and hence deduce their
distances apart.

[3]

(ii) If the interior of the microwave oven is 30.5 cm wide, draw a amplitude-
position graph of the standing wave pattern in the oven. Mark the
positions on the graph with “D” that would produce dry patches on the
wet cardboard as mentioned.

[2]
ACJC 2010 H2 9646 Prelim Exam P3
7 For
Examiner’s
Use
(c) The turntable of the oven is now placed back in the oven. After five minutes of
cooking a dish in a microwave wave, and upon removing it, it is noticed that
several ants are inside the oven apparently unharmed by the intense
microwave radiation. Deduce why some of the ants did not die.

[1]

(d) Explain the purpose of the turntable of the microwave oven?

[1]

4 A metal spring of natural length 20.0 cm fixed to the ceiling such that the bottom end
is at a height of 30.0 cm from the ground as shown in Fig 4.

20.0 cm

30.0 cm

Fig 4

ACJC 2010 H2 9646 Prelim Exam P3 [Turn over


8 For
Examiner’s
(a) When the box of mass 5.00 kg hangs in equilibrium, the bottom end of the Use
spring is at a distance of 25.0 cm from the ground. Determine the spring
constant of the spring.

Spring constant = …………….. N m−1 [3]

(b) The box was then brought to a higher point, such that the bottom of the spring
was 40.0 cm above the ground. The box was subsequently released from rest.

(i) Calculate the speed of the box when the bottom of the spring is 30.0 cm
above the ground.

Speed = ……………… m s−1 [3]

(ii) Determine the nearest distance of the bottom of the spring from the
ground.

Distance = ……………… m [3]

(c) Explain why in practice, we expect the answer obtain in (b)(ii) to be larger.

[1]

ACJC 2010 H2 9646 Prelim Exam P3


9 For
Examiner’s
Section B Use
Answer any two questions

5 A magnet, labeled Magnet A, is suspended vertically from a fixed point by means of


a spring, as shown in Fig. 5.1.

Fig. 5.1

One end of the magnet hangs inside a circular coil of wire. The coil is connected in
series with a resistor R.

(a) The magnet is displaced vertically downwards a small distance D and then
released. Fig. 5.2 shows the variation with time t of the vertical displacement d
of Magnet A from its equilibrium position.

Displacem ent / cm
D

Tim e / s

0 0.5 1 1.5 2

-D

Fig. 5.2

ACJC 2010 H2 9646 Prelim Exam P3 [Turn over


10 For
Examiner’s
(i) Over the period t = 0.12s to t = 0.16s, the student estimates that the Use
magnetic flux density within the coil decreases from 120 mT to 30 mT.
Given the circular coil has a total of 80 turns and has a radius of 5.0 cm,
calculate the magnitude of the average emf induced in the coil during this
time.

Average emf = V [2]

(ii) Explain why the amplitude of the oscillation of Magnet A decreases with
time as shown in Fig 5.2.

[3]

(b) The student replaces Magnet A with another magnet labeled Magnet B of
exactly the same dimensions and mass. The student then repeats the
experiment as described in (a). Fig. 5.3 shows the variation with time t of the
vertical displacement d of Magnet A and Magnet B from its equilibrium position.
Displacem ent / cm
D

Magnet A
Tim e / s

Magnet B
0 0.5 1 1.5 2

-D

Magnet A Magnet B
Fig 5.3

ACJC 2010 H2 9646 Prelim Exam P3


11 For
Examiner’s
(i) Deduce from Fig 5.3 that the magnet field strength of Magnet B is Use
stronger compared to that of Magnet A.

[2]

(ii) State 2 adjustments that the student can apply to the apparatus in Fig
5.1 such that the amplitude of the oscillation using Magnet B (as shown
in Fig 5.3) could be increased.

[2]

(c) Magnet B is now replaced with the original Magnet A. The circular coil in Fig
5.1 is now removed and a flat horizontal coil is setup as shown in Fig 5.4.
When an alternating current is passed through the coil, the magnet vibrates
under forced oscillations. The graph, Fig. 5.5, shows how the amplitude of the
oscillations varies with f, the frequency of the alternating current where fo is the
natural frequency of the spring-magnet system.

Flat
horizontal
coil

Fig 5.4 Fig 5.5

ACJC 2010 H2 9646 Prelim Exam P3 [Turn over


12 For
Examiner’s
5 (c) (i) Explain why the magnet in the setup given in Fig 5.4 will undergo forced Use
oscillation.

[3]

(ii) State 2 parameters that will affect the amplitude of the forced oscillation.

[2]

(iii) Explain why the maximum amplitude of the forced oscillation occurs
when the driving frequency is at fo.

[1]

(iv) Suggest how the apparatus in Fig. 5.4 could be modified to make the
peak on Fig. 5.5 flatter, without significantly changing the frequency fo at
which the peak occurs.

[2]

ACJC 2010 H2 9646 Prelim Exam P3


13 For
Examiner’s
(d) The frequency of the alternating current is now fixed at a certain frequency. Use
The displacement time graph of the magnet is as shown in Fig 5.6.
Displacem ent / cm
2

Tim e / s
0
0 0.5 1 1.5 2

-1

-2

Fig. 5.6

The same setup in Fig 5.1 is now placed back to its original position as shown
in Fig 5.7.

Fig. 5.7

Sketch, on Fig 5.8, the new displacement time graph of the magnet’s
oscillation.

[3]
Fig. 5.8

ACJC 2010 H2 9646 Prelim Exam P3 [Turn over


14 For
Examiner’s
6 Fig 6.1 shows a spectroscope used to observe the emission line spectrum for a Use
hydrogen discharged tube subjected to a high voltage of 5000 V. The light emitted
from the discharged tube is passed through the collimator of the spectroscope and is
incident normally on a diffraction grating of 600 lines per mm.
Rotating Diffraction
table grating
Hydrogen
discharge Straight through
lamp θ position, θo

collimator

Diffracted
Telescope ray, θ1

Fig 6.1
(a) Four first order diffraction lines from the visible regions of the emission
spectrum of atomic hydrogen were observed on one side of the straight
through position. The reading of the straight through position and three of the
lines was recorded with the help of the telescope and the vernier scale on the
spectroscope. The data are as given in Fig 6.2.

Straight through position, θo = 289.3o

Line Readings, θ1 /o θ/ ο Wavelength Photon energy /J


number λ / nm
1 303.6 14.3 410.3 4.85 x 10-19
2
3 306.3 17.0 486.3 4.09 x 10-19 J
4 312.5

Fig 6.2

(i) Explain how the observation of the emission line spectrum provide
evidence of the existence of energy levels in atoms.

[3]

(ii) Explain clearly why we can use the term "atomic" given above even
though hydrogen gases exist as molecules.

[1]

(iii) Determine θ and hence the wavelength of line 4 for the spectrum given
to 4 sf and fill in your answer in Fig 6.2.

θ = …………o [2]
ACJC 2010 H2 9646 Prelim Exam P3
15 For
Examiner’s
(b) The telescope was replaced by a camera and a photograph taken of the same Use
spectrum. The emission line spectrum was superimposed on a scale shown in
Fig 6.3.

Line number

5 6 7 8 9 10 11
Scale readings / cm
Fig 6.3

(i) Fill in the line number which each line corresponds to in the boxes in
Fig 6.3. [1]

(ii) Hence complete the table given in Fig 6.4 with the corresponding line
number and the wavelength for Line number 1, 3 and 4.
0 cm on the scale coincides with the straight through position.
Line number Scale reading / cm Wavelength / nm
6.35
6.75
7.63
10.73
Fig 6.4 [1]

(iii) Plot a graph of wavelength against scale reading on Fig 6.5 and complete
Fig 6.4 for Line number 2.

[3]
Fig 6.5

ACJC 2010 H2 9646 Prelim Exam P3 [Turn over


16 For
Examiner’s
(c) (i) Determine the photon energies for Line 2 and 4. Use

Photon energies of Line 2 = ………....J


Line 4 = ……....... J [3]

(ii) Hence draw to scale a partial energy level map diagram for hydrogen
from your answer in (i) given that the the transitions are to level 2 of the
energy levels of atomic hydrogen, which belongs to the Balmer Series.
Show, and label clearly, the electron transitions responsible for the
emission lines observed.
Note that the energy value for Level 2 is − 5.44 x 10−19 J.

[4]

(d) (i) Explain why the size of each slit in the grating must be narrow.

[1]

(ii) State one advantage of having a large number of rulings (lines) on the
grating.

[1]

ACJC 2010 H2 9646 Prelim Exam P3


17 For
Examiner’s
Use
7 (a) Explain what is meant by nuclear decay being spontaneous and random in nature.

……………………………………………………………………………………………….

……………………………………………………………………………………………….

……………………………………………………………………………………………….

……………………………………………………………………………………………….

…………………………………………………………………………………………… [2]

(b) The 226Ra nucleus undergoes alpha decay according to


226 222 4
88 Ra→ 86 Rn+2 He

Particle Rest mass / u


226
88 Ra
226.0254
222 222.0176
86 Rn
4 4.0026
2 He

(i) Show that the energy released in this decay, Q, is 4.86 MeV.

[2]

(ii) This energy Q must be shared by the alpha particle and the daughter
nucleus. Use conservation of energy and momentum to show that
⎛ M ⎞
Q = K α ⎜1+ α ⎟
⎝ M ⎠
where Kα is the kinetic energy of the alpha particle,
Mα is the mass of the alpha particle, and
M is the mass of the daughter nucleus.

[3]

(iii) Hence find the kinetic energy of the alpha particle emitted in this decay
process. Comment on your answer with reference to (b)(i)

Kα = …………… MeV [3]

ACJC 2010 H2 9646 Prelim Exam P3 [Turn over


18 For
Examiner’s
Use
(c) Many radioisotopes have important industrial, medical and research applications.
One of these is 60Co, which has a half-life of 5.2 years and each 60Co nucleus
decays by emission of a beta particle (energy 0.31 MeV) and two gamma photons
(energies 1.17 MeV and 1.33 MeV respectively).

A scientist wishes to prepare a 60Co sealed source that will have an activity of at
least 37 x 1010 Bq after 30 months of use.

(i) Show that the initial minimum activity of 60Co when the scientist is preparing
the radioisotopes is 5.2 x 1011 Bq

[2]

(ii) Hence calculate the minimum initial mass in milligrams of 60Co required?

Minimum initial mass = …………. mg [4]

(iii) At what rate will the source emit energy after 30 months if its activity is
37 x 1010 Bq after 30 months of use.

Rate of energy emitted ……….. W [3]

(iv) With reference to the type and energies of the radiations emitted, suggest
which of the radiations emitted by 60Co decay could be used to detect the
uniformity of thickness of metal sheets.

………………………………………………………………………………………

………………………………………………………………………………………

……………………………………………………………………………………[1]

ACJC 2010 H2 9646 Prelim Exam P3


ACJC 2010 Prelims

H2 P1 Ans
Q1 Q2 Q3 Q4 Q5 Q6 Q7 Q8 Q9 Q10 Q11 Q12 Q13 Q14 Q15 Q16 Q17 Q18 Q19 Q20
A C B D D A D B C A B A C D D D D A B C
Q21 Q22 Q23 Q24 Q25 Q26 Q27 Q28 Q29 Q30 Q31 Q32 Q33 Q34 Q35 Q36 Q37 Q38 Q39 Q40
A A B B D B C B A D B D C B D C D B C A

H2P P2 Ans
1(a)(i) π
V= × d 3 , where d = 22 cm (accept values from18 cm to 24 cm)
6
∴V = 5.6 x 10-3 m3 (accept values from 3.0 to 7.5 x 10-3 m3 ); accept
corresponding values if alternative units are used (e.g. mm3 or m3)

1(a)(ii) 1
KE = mv 2 , where v = 10 m s-1 (accept values from 8 m s-1 to 12 m s-1) and
2
mass = 60 kg (accept values from 40 kg to 100 kg)
∴KE = 3000 J (accept values from 1900 J to 7200 J); accept corresponding
values if alternative units are used (e.g. kJ)

(iii) Density is between 800 to 1100 kg m-3

1(b)(i) The values are close to each other but very far from the expected value
which should be less than density of water (1000 kg m3).
Therefore, set B is precise but inaccurate.

1(b)(ii) - mass of beaker is included in the measurement of mass


- balance calibrated incorrectly
- negligence in zero-ing the weighing balance
(either one of the above)

2a(i) Graph is a straight line through the origin ⇒ a is directly proportional x


Graph is a straight line with negative gradient
⇒ a is in the opposite direction of x

(ii) ω2 = 250 rad s-1


Τ = 2π/ω
Τ = 0.40 s

(iii) Cosine curve


1
correct amplitude of 0.050 m and with 2 complete waves
2

(b) 2π 2π
ω= = =π rad s-1
T 2
Apply x = x0 sin ωt where x0 = 50 mm and x = 25 mm
Time interval when shuttle remains open ,
ACJC 2010 Prelims


⇒ 25 = 50 sin t
2
1
t= s ⇒ t = 0.167s
6

3 (a) force on magnet/ balance is downwards since reading increases


(so by Newton’s third law)force on the wire is upwards

(b) pole P is a north pole (using Fleming’s left hand rule) (allows ecf from (a))
(c) Egg shape field lines closer at the bottom and spread out at the top
Direction is clockwise

(d) F = B I L sin 90° and F = mg


2.3 x 10-3 x 9.8 = B x 2.6 x 4.4 x 10-2 ( g missing, then 0/2 in this mark)

∴ B = 0.20 T

4ai Draw a suitable circuit diagram for this investigation.

aii
I

V
ACJC 2010 Prelims

aiii As I and V increases, rate of atomic vibration increases


Number of free electrons remains the same, hence resistance increases

aiv ρl
Know that R = ; or mention that R depends on ρ
A
Therefore ρ must have been unique as R is unique at each temperature.

5(a)(i) Vs is (stopping potential). Electrons with max KE cannot reach the anode/cannot be
collected (as even they do not have enough energy to overcome the electrostatic
repulsive force between cathode and anode.)
(ii) (Electrons are emitted with a range of KE), hence when anode is make less negative
with respect to cathode, some electrons may be able to (overcome the electrostatic
repulsive force and) reach the anode
(iii) Saturation not achieved immediately once V is +ve because the electrons are
scattered randomly in different directions. (Hence with higher V, the path of more
electrons may be altered so that it is able to reach the anode due to the increased in
the magnitude of the electric force.)
(Saturation current is achieved at V1 when all emitted electrons are collected.)
5(b)(i) φ = 5.32 x 1.6 x 10-19 = 8.51 x 10-19 J
hc/λ = 8.51 x 10-19 J
λ= 2.34 x 10-7 m
(ii)
r
p =h/λ
r
p =2.83 x 10-27 N s (allow ecf)

6(a)(i) For bullet : Δx = 2.64 x 10-32 m (allow if given to 1 sf)


For electron Δx = 1.95 x 10-3 m (allow if given to 1 sf)
Both correct

(ii) For bullet, HUP sets no practical limit to the locating of the position of the bullet
accurately.
About 10-17 times diameter of nucleus
For electron, (the position of the electron is probabilistic.) Cannot determine its exact
position at any instant of time.
About 107 times diameter of an atom.

Explanation related to reasonable estimates on order of magnitude stated implicitly or


explicitly.

(b)(i) Quantum tunneling which is the phenomenon where the probability of the alpha particle
passing through the barrier is non-zero where classically it is unable to overcome the
barrier as it has less energy than the barrier.

(ii) The long half-life implies low decay constant which is the probability of decay per unit
time,
due to the error in the (huge) underestimate of the transmission probability of the
alpha particle.
ACJC 2010 Prelims

(iii) breaking the barrier into segments and (multiplying the successive tunneling
probabilities) or
r2
8 π 2m (U (r ) − E )
Use T proportional to exp ∫ −2 dr
r1 h2

(Note r is d in the formula) to account for the change of U with r.


(In this case r is from 9.01 to 26.90 fm

7(a)(i)

The body is a relatively good conductor. If the body is in contact with the
ground, the body and the ground will tend to make one equipotential surface.
So the body still have very nearly 0 potential difference between the head and
the feet.
(ii) 1800
Current Density =
4πr 2
1800
= (for correct substitutions)
(
4 × π × 6380 × 10 3 )
2

= 3.5 x 10-12 A m-2


Hence current density near the surface of earth is in the order of about 10-12 A
m-2
(iii) P = 4 x105 x 1800
= 720 MW (2 sf) (Accept 700 MW)
(b)(i) πd 2 H
Volume of cloud =
4
π (2 × 10 3 ) (2 × 10 3 )
2

=
4
= 6.283 x 109 m3
No. of droplets at lowest density = 50 x 106 m-3

Vol. of each droplet =


4
3
(
π 10 × 10 −6 )
3

= 4.189 x 10-15 m3
No. of droplets in the cloud = 50x106x6.283x109
= 3.14 x 1017
Total volume of water droplets = 3.14 x 1017x4.189x10-15
= 1.315 x 103 m3
ACJC 2010 Prelims

Mass of cloud = 1.315x103 ρ w


= 1.315x106 kg
(b) (ii) Volume
Rainfall =
Area
1.315 × 10 3
=
π (1 × 10 3 )
2

= 0.42 mm
(c) W = Q ×V
(i)
= 20 x 1 x 108
= 2 x 109 J
(ii) W
P = VI or P=
t
2 x109
=108 x 20 000 =
10−3
= 2 x 1012 W = 2 x 1012 W
Accept approx 109 / 10-3
(iii) dV
E=−
dr
80 × 10 3
E = (Accept 60 x 103 / 0.07)
0.07
= 1.14 x 106 V m-1
Since the electric field at the top of the tower is greater than the order of 105 (a
few hundred thousands V m-1), a return stroke is likely to occur.
(d) (i) The electrical discharge results in heating up of the atmosphere around the
lightning channel.
The sudden rise in pressure causes the air around it to expand rapidly resulting
in an explosion of the air.
(ii) Distance (m) = 330 (m s-1) x t (s)
(Assuming time taken by light is
negligible)
= 330t m

330 (3 x108 − 330)d


= t km t=
1000 9.9 x1010
t
= km
3
(e)(i) ⎛I ⎞
120 = 10 log⎜⎜ 1 ⎟⎟
⎝ I0 ⎠
⎛I ⎞
124.8 = 10 log⎜⎜ 2 ⎟⎟
⎝ I0 ⎠
⎛I ⎞ ⎛I ⎞ I1 = 1.0 W m-2 and
124.8 − 120 = 10 log⎜⎜ 2 ⎟⎟ − 10 log⎜⎜ 1 ⎟⎟
⎝ I0 ⎠ ⎝ I0 ⎠ I2 = 100.48 W m-2 found separately
ACJC 2010 Prelims

⎛I ⎞
4.8 = 10 log⎜⎜ 2 ⎟⎟
⎝ I1 ⎠
⎛I ⎞
0.48 = log⎜⎜ 2 ⎟⎟
⎝ I1 ⎠
I2
10 0.48 =
I1
1

r2
I 1 r1 = I 2 r2
2 2

2
I 2 r1
= = 10 0.48
I 1 r2 2
2
2
2
= 10 0.48
r2
r2 = 1.15 km
ACJC 2010 Prelims

.
Basic procedure
Mentioned that ionisation current and air pressure measured or
B1
State IV is P and DV is ionization current
Explain how pressure is varied using vacuum pump B1

[2]

Diagram shows
Correct electrical circuit; power supply must be shown. B1
Alpha source between electrodes, Pressure gauge/Bourdon Gauge and Vacuum pump/ air
B1
pump shown connected correctly.
[2]

Measurements
Measure ionisation current using milliammeter / microammeter / galvanometer B1
Measure air pressure using pressure sensor with datalogger or direct reading from
B1
pressure gauge / Bourdon gauge
(accepted if shown on diagram) [2]

Control of variables
Measure that activity of source is ensured to be constant by using a source with long half-
B1
life / high activity / checked by using radiation detector.
Separation of parallel metal plates is kept constant throughout the experiment B1

Position of source is kept constant wrt plates B1

[max 2]

Any further detail


Use Americium 241, 185 kBq as alpha particle emitter (a relatively pure alpha emitter) B1
Source of 185 kBq is also relatively safe to use in a school laboratory with standard basic
B1
precautions
Use high voltage (50 V to 1000 V) power supply as current is very small B1

Place source close to plates as alpha particles have short range in air B1

Electrical insulation between electrode plates and walls of metal case B1

8 sets of data is collected (4 below and 4 above atmospheric pressure) B1


Count recorded over a fixed duration of 1 or 2 minutes if using detector to check for
B1
constant activity
Source placed near to detector if using detector to check for constant activity. B1

Tap pressure gauge /Bourdon gauge when taking readings in case needle sticks B1
Plot suitable graph to verify proposed relationship (eg log I vs log P) with suggested conclusion
given B1
(Do not accept if mention plot I against P)
[max 3]

Safety

Use of source handling tool B1

Source kept in lead container when not in use B1

Do not point source at anyone / do not look directly at source B1

Use safety goggles when working with low/high pressure (Do not award if no explanations given) B1

Use safety screen in case of explosions B1

[max 1]
2010 H2 Prelims  

H2 P3 

1(a) Vertical component of tension = weight of stone


Horizontal component of tension/centripetal force is perpendicular to the
velocity or direction of motion.
Horizontal component of tension/Centripetal force/constant net force points
towards a fixed point.
Constant tension supplied to maintain constant centripetal force
Horizontal component of tension/Centripetal force/constant net force just
sufficient to provide circular motion for a given v and r
Constant energy supplied to stone (Do not accept Apply const v)

(b) H = ½ g t2

T sin θ > mv2/r when v decreases, hence at this instant r decreases.


When r decreases, θ decreases.
Hence from T cos θ > W, therefore T must decrease so that there will be
vertical equilibrium

(c) X = v t and H = ½ g t2 . Use H = ½ g t2

gX2
H= t = 2.944 s
2v2

2H
X=v
g
2 (42.5)
= (22.8) (for correct
9.81 Use X = ut
substitutions)

= 67.1 m X = 67.1 m
2010 H2 Prelims  

2a An elastic collision is a collision in which the (total) kinetic energy of system


is conserved,
total linear momentum and total energy of the system are all conserved

b(i)
m  M  v m M 
v
u   vv   V v

Before Collision  After collision 
For an elastic collision with a stationary piston, the speed of the gas remains
the same when it bounce off in the opp direction
With a piston moving down, using the fact that relative speed of approach =
rel speed of separation, its rebound speed will be greater than that earlier

Since for ideal gas, T is dependent on ave KE of molecule alone,


Higher ave speed implies higher ave KE, hence Temperature rises

(ii) Apply ∆U=Q+W where


ΔU = Increase in internal energy, Q is heat supplied to system + W is WD
on system
mentions that Q=0 and work done is done on gas(or W is +ve), hence
higher internal energy,
For ideal gas, Internal energy depends on temp alone
thus higher temperature

(iii)

Process shown correctly

 
 
2010 H2 Prelims  

 
3(a) There are standing waves produced in the microwave oven during the
cooking process because incident wave from the left gets reflected by the
reflective wall on the right and they superimpose /overlap /interfere
The conditions must be right such that the distance between the source and
wall must be integral multiples of half the wavelength of the microwave.to
form standing wave
as they have the same speed, frequency and almost the same amplitude

(b)(i) Intensity of the microwave is strongest (largest amplitudes) at the


antinodes, hence the dry regions are the regions of antinodes
Wavelength = 3.0 x 108/2.45 x 109
= 0.1224 m = 12.2 cm
Hence distance apart is 6.1 cm

(ii)

D D D D D
Standing wave pattern drawn with ends as nodes
(need not have 5 loops)
5 Ds shown
(c) So there are regions in the oven where the microwave has high amplitude
(antinodes) and there are region where the microwave has no displacement
(node). Thus ants can stay away from the regions of high amplitude which
has high heat and thus stay alive.
Understand that the ants will stay away from the position of antinodes to stay
alive.

(d) The turntable enables different parts of the food to move to the antinodes of
the standing wave and get heated up. Thus it helps heat up the food more
uniformly.

     
2010 H2 Prelims  

4a) F spring= kx

5 x 9.81 = k(0.05)
k=.981 N m-1
(No not penalize for inconsistent –ve sign that appears) – Pls annotate BOD

4b By conservation of energy,
(i) At a height 40cm, the contraction of spring is 10 cm.
Energy stored in spring = ½ (981)(0.1)2
= 4.905 J
Energy gain = Energy lost
½ mv2= mgh + 4.905
 
½ (5)v2= (5)(9.81)(0.1)+4.905 = 9.81
v= 1.98 m s-1
(ii) By conservation of energy,
Let final extension be x (measured from natural length)
Energy at natural length= Energy at lowest point Initial energies = final energies
2
9.81 = - (5)(9.81)x + ½ (981)x 4.905 =-5 (9.81)(x+0.1) + ½ (981)x2
490.5 x2 – 49.05 x – 9.81 = 0
50 x2 – 5x – 1 = 0
Hence x = -0.10 (initial released point) or + 0.20 (lowest point)
Hence lowest point is (0.30 – 0.20) = 0.10 m above the ground.

4c In reality, it is larger. There will be some energy lost due presence of air
resistance/friction in spring and thus less energy will be available for
conversion to elastic potential energy.

 
2010 H2 Prelims  

5  (a)(i)  | ε | = | - dΦ/dt |
 
= | - NA dB/dt |
    = | - (80) * (π * 0.052) * (30-120)*10-3 / (0.16-0.12) |  
    = 1.41 V  
    (ignore –ve sign)  
  (a)(ii)  When the magnet is oscillating, an emf will be induced in the coils and
thus induced current will flow since circuit is closed resulting in  
either electrical energy being dissipated or heating effect of coils
    Hence total (mechanical) energy of the magnet-spring system
decreases continuously as it is being converted into the electrical  
energy in the coil
    As amplitude of oscillations depends on amount of mechanical
 
energy present,
    Hence the amplitude decreases continuously  
     

  (b)(i)  A lower amplitude indicates that


more ME is converted into
A lower amplitude indicates that
electrical energy over each cycle,
there is greater damping /induced  
(that can only be the result of a
force,
higher induced emf, hence induced
current)
    (Since system has same frequency (Since system has same frequency
and started with same amplitude) and started with same amplitude)
and N and A are constant, higher This is due to greater induced  
induced emf must be due to higher emf/current which must be due to
rate of change of B. greater rate of change of B
    Magnet field strength of Magnet B
   
is stronger than that of Magnet A

  (b)(ii)  • Reduce the number of coil. Since | ε | = | - NA dB/dt |, a lower N will  


result in a lower induced emf, therefore higher amplitude for the  
oscillation  
• Use a wire with higher resistance. Same emf will result in a lower  
induced current  
• Use a resistor with higher resistance. Same emf will result in a lower  
induced current  
• Use coil of smaller cross-sectional area  
• Use spring of smaller k  
 
 
2010 H2 Prelims  

 
c (i) As the current pass through the coil, the coil will generate a magnetic field  
and will behave like a magnet. As the current is alternating, the polarity of
this coil will change as the direction of the current flow changes.
The magnet will then be attracted and repelled by the coil as the current
changes direction.

As the alternating current in sinusoidal, the force exerted on the magnet  


will also be sinusoidal and therefore, the magnet will be forced into a
sinusoidal motion.
Idea that coil generates an alternating(sinusoidal) field  
Idea that magnetic force created is hence also alternating  
(and its magnitude is proportional to I at that instant.)  
with the same frequency as the ac supply.  
    Hence magnet is undergoing forced oscillation.   
(ii) • Amount of damping on the driven system
• The relative values of the natural frequency of the driven system and
the value of the frequency of the external driving force
• Amplitude of the periodic driving force
• More turns used in the coil

(iii) Maximum energy transfer to driven system when driving frequency is


equal to (approaches the) natural frequency of system.
(iv) Mention that method is to increase damping on system
Any suitable method that increases damping, examples
• Attached an object of large surface area to the magnet
• submerged magnet in a viscous fluid
Alternative:
Use AC of lower amplitude
This will result in lower amplitude for all frequencies, hence flatter
peak.    
  (d)    Displacem ent / cm
 
  2
 
 
 
1
 
 
  Tim e / s

  0
  0 0.5 1 1.5 2
 
  -1
 
 
 
-2
 
 
2010 H2 Prelims  

 
6  (a)(i)  The distinct lines shows that only photons of certain frequency are emitted.   
    This took place for electron transition from  higher energy levels to a lower energy   
levels. 
    This electron transition results in the release of a photon from the atom of an   
amount of energy (E = hν) equal to the difference in energy of the electronic energy 
levels involved in the transition. 
    Hence showing the energy levels are quantised.   
  (ii)  The high voltage applied broke the bonds of the hydrogen molecules into its   
(isolated) atoms (and promote the electrons into higher energy levels.) 
       
  (iii)  θ = 23.2o   
    λ = 656.6 nm   
       
  (b)(i)  1,2,3,4 L to R   
  (ii)  1 – 410.3 nm; 3 – 486.3 nm; 4 – 656.6 nm (no ecf)   
  (iii)  All 3 points plotted correctly  

    Axes given to correct units   
    Wavelength read correctly as 435 nm (theoretical  value 434 nm)   
(accept 430 to 440 nm as read correctly from graph) 
       
  (c)(i)  E = hc/λ   
    E = 6.63 x 10‐34 x 3 x 108/λ  
(correct sub for h and c)
    E2 = 4.57 x 10‐19 J(allow ecf) 
    E4 = 3.03 x 10‐19 J   
  (credit given only if both calculations are correct) 
 
2010 H2 Prelims  

 
  (ii)     
  6 (‐0.59 x 10‐19
‐19
 J) 
5 (‐0.87 x 10‐19  J) 
  4 (‐1.35 x 10  J) 
  3 (‐2.41 x 10‐19 J) 
 
1  2  3 4
 
 
  2 (‐5.44 x 10‐19 J) 
 
 
    5 energy levels draw with decreasing spacing   
    Scale correct   
    Transitions all correct    
    Energy values all correct    
       
  (d)(i)  For significant diffraction to occur so that higher wavelength photons could be   
observed. 
Accept (so that lines further away from straight through position are observable) 
  (ii)  Brighter line spectrum / better contrast   
Any other acceptable answer 
       
       
       
 
2010 H2 Prelims  

 
7a Random
Impossible to predict when and which individual nuclide will decay
OR
There is a constant probability or decay
Or
Unable to predict which atom and how many will decay at the next instant
of time

Spontaneous: Decay is unaffected by environmental changes such as temp


/ pressure, external source of energy supplied etc.
7bi Q = (MRa − MRn − Mα )c 2

( ) (
= ( 226.0254 − 222.0176 − 4.0026 ) × 1.66 × 10−27 × 3 × 108 )
2

= 7.7688 × 10−13 J
= 4.86 MeV
ii COM: MV + MαV = 0 --------- (1)
1 1 2
COE: Q = MV 2 + MαVα --------- (2)
2 2
⎛M ⎞
From (1): V = −⎜ α ⎟Vα -------------- (3)
⎝M ⎠
Subst (3) into (2)

2
1 ⎛ ⎛ Mα ⎞ ⎞ 1 2
Q = M⎜ −⎜ ⎟Vα ⎟ + MαVα
2 ⎝ ⎝M ⎠ ⎠ 2
1 2⎛ M ⎞
= MαVα ⎜ α + 1⎟
2 ⎝M ⎠
⎛M ⎞
= K α ⎜ α + 1⎟
⎝M ⎠
 
   

iii ⎛ 4.0026 ⎞
4.86 = Kα ⎜ + 1⎟ (correct substitution shown)
⎝ 222.0176 ⎠
Kα = 4.77 MeV
The alpha particles carries away most of the energy – 98 %
7ci ⎛ ln2
-⎜

×2.5 ⎟
A0 e ⎝ 5.2 ⎠
≥ 37 × 1010
A0 ≥ 5.16(3) × 1011 Bq
A0 = 5.2 × 1011 Bq
2010 H2 Prelims  

 
ii λ = ln2 (5.2 × 365 × 24 × 60 × 60)
= 4.2268 x 10-9 s-1
A0 = λ N0
5.2 × 1011 Bq
N0 =
ln2 (5.2 × 365 × 24 × 60 × 60)
= 1.23 x 1020
m
1.23 × 1020 = 6.02 × 1023 ×
60
m = 12.3 mg
iii Energy emitted per decay = 0.31+1.17+1.33=2.81 MeV
Rate =( 2.81 x 1.6 x 10-13 J) x 37 x 1010 decay per second
= 166 mW
iv The β-decay energy is low compared to the γ-ray.

Hence the (2) strong γ-lines, could be used as a γ-ray source to check for
uniformity of the thickness of metal sheets in the industries
 
 
Name: ……………………….…………………. HT group: …………...

CATHOLIC JUNIOR COLLEGE


JC2 PRELIMINARY EXAMINATION 2010

PHYSICS 9646/ 01
Higher 2

Paper 1 Multiple Choice Questions Friday 17 September 2010


1 hour 15 minutes

Additional materials: MCQ answer sheet

READ THESE INSTRUCTIONS FIRST

Write in soft pencil.


Do not use staples, paper clips, highlighters, glue or correction fluid.
Write your name, HT group and NRIC/ Fin number on the MCQ Answer Sheet. Shade
your NRIC/ Fin number correctly on the MCQ Answer Sheet in the spaces provided.

There are 40 questions on this paper. Answer all questions. For each question there
are four possible answers A, B, C, D.
Choose the one you consider correct and record your choice in soft pencil on the
separate Answer Sheet.

Read the instructions on the Answer Sheet very carefully.

Each correct answer will score one mark. A mark will not be deducted for a wrong
answer. Any rough working should be done in this booklet.

This question paper consists of 15 printed pages.


PHYSICS DATA:

speed of light in free space, c = 3.00 x 108 m s-1


permeability of free space, μ0 = 4π x 10-7 H m-1
permittivity of free space, ε0 = 8.85 x 10-12 F m-1
≈ (1/(36π)) x 10-9 F m-1
elementary charge, e = 1.60 x 10-19 C
the Planck constant, h = 6.63 x 10-34 J s
unified atomic mass constant, u = 1.66 x 10-27 kg
rest mass of electron, me = 9.11 x 10-31 kg
rest mass of proton, mP = 1.67 x 10-27 kg
molar gas constant, R = 8.31 J K-1 mol-1
the Avogadro constant, NA = 6.02 x 1023 mol-1
the Boltzmann constant, k = 1.38 x 10-23 mol-1
gravitational constant, G = 6.67 x 10-11 N m2 kg-2
acceleration of free fall, g = 9.81 m s-2

PHYSICS FORMULAE:

uniformly accelerated motion, s = u t + ½ a t2


v2 = u2 + 2 a s
work done on / by a gas, W = p ΔV
Hydrostatic pressure P = ρgh
gravitational potential, φ = –Gm/r
Displacement of particle in s.h.m. x = x0 sin ωt
Velocity of particle in s.h.m. v = v0 cos ωt

= 2
± ω x0 − x2
resistors in series, R = R1 + R2 + ...
resistors in parallel, 1/R = 1/R1 + 1/R2 + ...
electric potential, V = Q / 4 π ε0 r
alternating current / voltage, x = x0 sin ωt
Transmission coefficient T = exp (-2kd)

8 π2 m (U – E )
where k =
h2

radioactive decay, x = x0 exp(-λt)


decay constant, λ = 0.693
t1
2

2
1. The power loss P through a resistor is found by measuring the potential difference V across the
resistor and the current I through it. The equation is given by P = VI. The voltmeter has a 4%
uncertainty and the ammeter reading has a 3% uncertainty. What is the uncertainty in the power
calculated?
A 3% B 4% C 7% D 12 %

2. A student made a series of measurements of the diameter d, of a wire using four micrometer
screw gauges A, B, C and D. The table shows the measurements taken.

If the actual diameter of the wire was 1.49 mm, which micrometer screw gauge produced a set of
readings that could be described as accurate but not precise?

micrometer screw
Readings d/ mm
gauge
A 1.49 1.46 1.52 1.50
B 1.48 1.58 1.51 1.40
C 1.35 1.37 1.42 1.42
D 1.32 1.37 1.41 1.50

3. A man stands on the edge of a cliff. He throws a stone upwards with a velocity of 19.6 m s-1 at
time t = 0. The stone reaches the top of the trajectory after 2.00 s and then falls towards the
bottom of the cliff. Air resistance is negligible.

Which row shows the correct velocity v and acceleration a of the stone at different times?

t /s v / m s-1 a / m s-2
A 2.00 0 0
B 1.00 9.81 9.81
C 5.00 -29.4 -9.81
D 3.00 9.81 -9.81

4. The sketch graph below describes the motion of a ball rebounding from a horizontal surface after
being released from a point above the surface.

The quantity represented on the y-axis is the ball’s


A Velocity C Acceleration
B Kinetic energy D Displacement

3
5. Two objects, X and Y, were dropped from rest from a tall tower on a wind-free day. In the graph
below are plotted their squared velocities as a function of their height above the ground.

From the information given in the graph and knowledge of the properties of bodies falling under
the influence of gravity, it is possible to say that the two objects

A experienced unequal viscous drag


B had different masses
C hit the ground at the same time
D were not dropped simultaneously

6. A proton (mass 1 u) travelling with velocity +0.100 c collides elastically head-on with a helium
nucleus (mass 4 u) travelling with velocity -0.050 c.

What are the velocities of each particle after the collision?


proton Helium nucleus
A +0.140 c +0.010 c
B -0.140 c +0.010 c
C +0.233 c -0.083 c
D -0.233 c +0.083 c

7. Which of the following pairs of forces is an action-reaction pair?

A Weight of a floating object and the upthrust acting on it.


B The force a ladder leaning on a smooth wall exerts on the rough floor and the normal
reaction from the floor
C The force a ladder leaning on a smooth wall exerts on the wall and the normal reaction force
from the wall
D Weight of a parachutist and the pull of the parachute on him when he is moving with
terminal velocity

4
8. The given diagram shows a column of dry air trapped by mercury in a narrow test tube.

Which graph best shows how the length l of the air column varies with the angle θ of the tube to
the vertical?
A C

B D

9. A right-angle rule hangs at rest from a peg P as shown below. It is made from a metal sheet of
uniform density. One arm is L cm long while the other is 2L cm long.

The angle θ at which it will hang is


A 8o B 14o C 42o D 76o

10. An object in a space capsule orbiting the Earth seems to be floating.

Which statement describes the forces acting on the object?

A There are no forces on the object.


B The centrifugal force on the object is equal and opposite to its weight.
C The centripetal force on the object is equal and opposite to its weight.
D The weight of the object is the only force acting on it.

5
11. A spring fixed at one end, has a mass attached to the other end. The mass bounces up and
down. It is shown in the diagram at three positions X, Y and Z.

Which line gives the kinetic, gravitational potential and elastic potential energies?

Kinetic energy Gravitational potential Elastic potential energy


energy
A zero at X maximum at X maximum at X
B maximum at Y zero at Z maximum at Y
C zero at Z zero at Z zero at X
D maximum at Y maximum at X maximum at Z

12. A space vehicle of mass m re-enters the Earth’s atmosphere at an angle θ to the horizontal.
Because of air resistance, the vehicle travels at a constant speed v.

The heat shield of the vehicle dissipates heat at a rate P, so that the mean temperature of the
vehicle remains constant.

Taking g as the relevant value of the acceleration of free fall, which expression is equal to P.

A mgv
B mgv sinθ
C 1
mv 2
2
D 1
mgv 2 sin 2 θ
2

6
13. A segment of the rollercoaster track in an amusement park is illustrated.

In this segment, the carriage descends from rest at H, moves through the vertical loop with its
top at A, and then over a hump at B. The friction between the carriage and the track may be
taken as negligible.

If the carriage is to complete the loop without leaving the track, what is the minimum value of the
height y?

A 30 m B 27 m C 25 m D 23 m

14 A comet of mass m is moving around Planet X in an elliptical orbit. It is moving with a speed v
when it is at P at a distance r from the Planet’s centre. What is the total energy of the comet at P
(Mass of Planet X is M)
P
Comet

r
Planet X

A GMm 1 B GMm 1 C GMm D GMm 1


+ mv 2 − + mv 2 −( + mv 2 )
r 2 r 2 r r 2

15. Points X and Y are two points at a distance of R and 3R from the centre of the Earth
respectively. The gravitational potential is -1.0 MJ kg-1 at point X. What is the change in potential
energy when a 30 kg mass is moved from Y to X?

A -20 MJ B 20 MJ C -40 MJ D 40 MJ

7
16. The graphs below show how the displacement x, velocity v and the acceleration a of a body vary
with time t when it is oscillating with simple harmonic motion.

What is the value of T?

A π/ 9 B 2π / 9 C 2π / 3 D 2π

17. A mass hanging from a spring suspended from the ceiling is pulled down and released. The
mass then oscillates vertically with simple harmonic motion of period T. The graph shows how
its distance from the ceiling varies with time t.
Distance from
ceiling /cm
100

30

t
0 T/4 T/2

What can be deduced from this graph?

A The amplitude of the oscillation is 70 cm C The kinetic energy is a maximum at t =


T/2
B The restoring force on the mass increases D The speed is a maximum at t = T/4
between t = 0 and t = T/4

18. A metal block X, of mass m, at 0oC comes into contact with another metal block Y, of mass 2m,
at 100 oC. Heat conduction takes place with no loss to the surroundings. The final equilibrium
temperature of the blocks is 20 oC. If the specific heat capacities of the two metals are cx and cy
respectively, then

A cx = 8 cy B cx = 4 cy C cx = 2 cy D cx = ½ cy
8
19. Air is enclosed in a cylinder by a gas-tight, frictionless piston of cross-sectional area 3.0 × 10–3
m2. When atmospheric pressure is 100 kPa, the piston settles 80 mm from the end of the
cylinder. The piston is then pulled out until it is 320 mm from the end of the cylinder and is held
there. The temperature of the air in the cylinder returns to its original value.
80 mm piston 320 mm
air
F

What is the force F required to hold the piston in its new position?

A 75 N B 100 N C 225 N D 300 N

20. Which one of the following statements is not correct?

A Boiling always occurs at a higher temperature than evaporation.


B In boiling, the most energetic molecules of the liquid escape leaving behind molecules with
lower energies while in evaporation, all molecules have the same energy.
C Evaporation occurs at any temperature but the boiling point depends on the external
pressure.
D The rates of evaporation and boiling are dependent on the surface area of the liquid.

21. A mechanical wave of frequency 300 Hz travels along a railway line at 6 km s-1. Two points on
the rail which are 250 cm apart are out of phase by

A 0 rad B π rad C π / 2 rad D π / 4 rad

22. A boy blows gently across the top of a piece of glass tubing the low end of which is closed by his
finger so that the tube gives its fundamental note of frequency, f. While blowing, he removes his
finger from the lower end. The note he then hears will have a frequency of approximately

A ¼f B ½f C 2f D 4f

23. Two loudspeakers L1 and L2, driven by a common oscillator and amplifier, are set up as shown.
As the frequency of the oscillator increases from zero, the detector at D recorded a series of
maximum and minimum signals. At what frequency is the first maximum observed?
(Speed of sound = 330 m s-1)

40 m
L1
D

9m

L2

A 165 Hz B 330 Hz C 495 Hz D 660 Hz

9
24. A sphere of weight 1.6 x 10-3 N has an electric charge of 2.0 μC. It is released from rest, in
vacuum between two parallel, vertical metal plates. The separation of the plates is 0.10 m and
the potential difference between them is 80 V. The point of release of the sphere is within the
region of uniform electric field between the plates. The arrangement is shown in the diagram.

Which path does the sphere follow after release?

+2.0 μC

25. The diagram below shows the variation of the electric potential V with the distance d along a
straight line in a particular electric field.

At which point is the magnitude of the electric field greatest?

26. Two wires P and Q, each of the same length and the same material, are connected in parallel to
a battery. The diameter of P is half that of Q.

What fraction of the total current passes through P?

A 0.20 B 0.25 C 0.33 D 0.50

10
27. The diagram shows three resistors of resistances 2 Ω, 20 Ω and 3 Ω connected in series. A
potential difference of 20 V is maintained across them. Point Q is earthed.

20 V

P Q R S
2Ω 20 Ω 3Ω

Which of the following gives the potentials at points P, Q, R and S?

Potential at
P Q R S
A 20 V 18.4 V 2.4 V 0V
B 1.6 V 0V 16 V 18.4 V
C 20 V 16 V -6V - 20 V
D 1.6 V 0V - 16 V - 18.4 V

28. A battery of e.m.f. E and internal resistance r delivers a current I through a variable resistance R.

E r

A
I

R
R is set at two different values and the corresponding currents I are measured using an ammeter
of negligible resistance.

R/Ω I/A
1.0 3.0
2.0 2.0

What is the value of e.m.f E?

A 3.0 V B 3.5 V C 4.0 V D 6.0 V

11
29. An electric heater can be represented as two resistors of resistances R1 and R2 and two
switches S1 and S2. The resistance R2 is greater than that of R1.

S1
R1

S2
R2

Which switches must be closed so that the heater produces the maximum possible power and
the minimum non-zero power?

maximum possible power minimum non-zero power


A S1 and S2 S2
B S1 and S2 S1
C S1 S2
D S2 S1

30. A bar magnet is to be placed in a non-uniform magnetic field as shown.

Which line of the table describes the subsequent motion of the magnet?

Rotation Movement
A anticlockwise to the left
B anticlockwise to the right
C clockwise to the left
D clockwise to the right

31. When an alternating current flows through a resistor of 5 Ω, heat is dissipated a rate of 20 W.
What is the peak value of this alternating current?

A 1.41 A B 2.00 A C 2.83 A D 4.00 A

12
32. A sinusoidal alternating current has period T. The r.m.s. value of the current in a resistor is I and
the mean power dissipated in the resistor is P.

Which statement is correct?

A 2π
The frequency is .
T
B The maximum power dissipated in the resistor is P 2 .
C I
The peak current is .
2
D P
The r.m.s. voltage is .
I

33. The graph shows the variation with time of the magnetic flux linking a coil.

flux

0
0 t 2t time
Which graph shows the variation with time of the e.m.f. induced in the coil?

A C
e.m.f

e.m.f

0
0
0 t 2t time
0 t 2t time
B D
e.m.f e.m.f

0 0
0 t 2t time 0 t 2t time

34. The de Broglie wavelength of a rifle bullet of mass 0.02 kg which is moving at a speed of
300 m s-1 is

A 7.3 x 10-36 m B 1.8 x 10-35 m C 1.1 x 10-34 m D 9.9 x 10-33 m

13
35. Which of the following statements is true?

A A beam of electrons directed at a vessel of cold gas can cause the formation of either an
absorption or emission line spectrum.
B A beam of white light directed at a vessel of cold gas can cause the formation of either an
absorption or emission line spectrum.
C A beam of electrons directed at a vessel of cold gas can only cause the formation of an
absorption line spectrum.
D A beam of electrons directed at a vessel of cold gas can only cause the formation of an
emission line spectrum.

36. A beam of 5.0 eV electrons strikes a potential energy barrier of height 6.5 eV and thickness
0.70 nm, at a rate equivalent to a current of 1000 A. How long would it take for an electron to be
transmitted?

A 0 B 1.03 x 10-18 s C 1.55 x 10-4 s D 6.27 x 109 s

37. One of the characteristics of laser light is that it is monochromatic. Which of the following
statement explains this characteristic?

A The excited electrons are in a metastable state.


B The system is in a state of population inversion.
C Stimulated emission causes the emitted photon and the incident photon to be of the same
phase.
D Photons of the same energy as that of the incident photons are emitted when the electrons
transit down from a higher energy level.

38. The diagram below is a representation of a diode.


p-n junction

P-type N-type

A B C D

Depletion
region

Which of the following statements is false?

A The overall charge in Region B is negative.


B When Group III atoms in region B accept electrons from neighbouring Group IV atoms, a
depletion region is formed.
C The potential barrier decreases when a positive potential is applied to region A and a
negative potential is applied to region D.
D Under forward-biased condition, electrons moves from D to A.

14
39. In an experiment to investigate the nature of the atom, a very thin gold film was bombarded with
α-particles.

Which pattern of deflection of the α-particles was observed?

A A few α-particles were deflected through angles greater than a right angle.
B All α-particles were deflected from their original path.
C No α-particle was deflected through an angle greater than a right angle.
D An interference pattern was observed.

40. A sample of pure potassium chloride is found to be radioactive due to the presence of 40K. The
sample contains 9.49 x 1019 atoms of 40K when the activity is measured to be 1600 Bq. The
half-life of the radioactive decay of 40K is

A 1.67 x 10-17 s B 5.93 x 1016 s C 4.11 x 1016 s D 1.30 years

END OF PAPER 1

15
Name: ……………………….…………………. HT group: …………...

CATHOLIC JUNIOR COLLEGE


JC2 PRELIMINARY EXAMINATION 2010

PHYSICS 9646/ 2
Higher 2

Paper 2 Structured Questions Monday 30 August 2010


1 hours 45 minutes

Candidates answer on the Question Papers provided

READ THESE INSTRUCTIONS FIRST

Write your name and tutorial group on all the work you hand in.

Write in dark blue or black pen in the spaces provided on the Question Paper.
You may use a soft pencil for any diagrams, graphs or rough working.
Do not use staple, paper clips, highlighters, glue or correction fluid.

Answer all questions.

The number of marks is given in brackets [ ] at the end of each question or part question.
Total marks for Paper 2 is 72 marks.

A total of 2 marks will be deducted for mistakes made in units and significant figures.

Paper 2 consists of 18 printed pages.

Marks
Awarded
Sec A 1 5
2 10
3 11
4 8
5 8
6 18
Sec B 7 12
Total 72
PHYSICS DATA:

speed of light in free space, c = 3.00 x 108 m s-1


permeability of free space, μ0 = 4π x 10-7 H m-1
permittivity of free space, ε0 = 8.85 x 10-12 F m-1
≈ (1/(36π)) x 10-9 F m-1
elementary charge, e = 1.60 x 10-19 C
the Planck constant, h = 6.63 x 10-34 J s
unified atomic mass constant, u = 1.66 x 10-27 kg
rest mass of electron, me = 9.11 x 10-31 kg
rest mass of proton, mP = 1.67 x 10-27 kg
molar gas constant, R = 8.31 J K-1 mol-1
the Avogadro constant, NA = 6.02 x 1023 mol-1
the Boltzmann constant, k = 1.38 x 10-23 mol-1
gravitational constant, G = 6.67 x 10-11 N m2 kg-2
acceleration of free fall, g = 9.81 m s-2

PHYSICS FORMULAE:

uniformly accelerated motion, s = u t + ½ a t2


v2 = u2 + 2 a s
work done on / by a gas, W = p ΔV
Hydrostatic pressure P = ρgh
gravitational potential, φ = –Gm/r
Displacement of particle in s.h.m. x = x0 sin ωt
Velocity of particle in s.h.m. v = v0 cos ωt

= 2
± ω x0 − x2
resistors in series, R = R1 + R2 + ...
resistors in parallel, 1/R = 1/R1 + 1/R2 + ...
electric potential, V = Q / 4 π ε0 r
alternating current / voltage, x = x0 sin ωt
Transmission coefficient T = exp (-2kd)

8 π2 m (U – E )
where k =
h2

radioactive decay, x = x0 exp(-λt)


decay constant, λ = 0.693
t1
2

2
SECTION A
ANSWER ALL QUESTIONS
IT IS RECOMMENDED THAT YOU SPEND ABOUT 1 HOUR AND 15 MINUTES ON THIS SECTION.

1 When a solid is heated, the thermal energy required is given by the expression

gain in thermal energy = mass X c X temperature rise, where c is a constant.

(a) Name the quantities in the expression that are SI base quantities.

…………………………………………………….......................................................................... [1]

(b) Express, in terms of SI base units, the units of

(i) thermal energy,

unit of thermal energy = ………………………… [2]

(ii) the constant c.

unit of c = …………………………
[2]

3
2 An aeroplane is flying horizontally at a steady speed of 67 m s-1 and an object is dropped off from the
aeroplane.

(a) Assume that the air resistance is negligible.

(i) Show that the vertical component of the velocity of the object is approximately 40 m s-1
when it has fallen 80 m.

[2]
(ii) Determine the magnitude and direction of the resultant velocity of the object at this point.

Magnitude of velocity =........................................................ m s-1

[3]
Direction of velocity =.................................................................

4
(b) In practice, air resistance acts on the object during the fall. The air resistance may be
assumed to be proportional to the square of the speed.

State and explain how the magnitude of the horizontal and vertical components of the velocity
of the object vary with time.

Horizontal component of velocity:

............……………………………………………………………………………………………....

............……………………………………………………………………………………………....

............…………………………………………………………………………………………….... [1]

Vertical component of velocity:

...........…………………………………………………………………………………………….....

...........…………………………………………………………………………………………….....

............……………………………………………………………………………………………....

............……………………………………………………………………………………………....

............……………………………………………………………………………………………....

............…………………………………………………………………………………………….... [2]

(c) Sketch and label the path of the object in Fig. 2

(i) without air resistance,

(ii) with air resistance.

Horizontal distance from the point of drop off [2]

Start of fall

vertical
distance from
the point of
drop off

Fig. 2

5
3 (a) A train of mass 2.2 x 105 kg is traveling at a speed of 20 km h-1 and it requires a power of
900 kW.

(i) Calculate the driving force exerted on the train.

driving force = …………..……………... N [2]

(ii) What additional power must be supplied if the train is to maintain at a speed of 20 km h-1
on a slope which rises 1 m for every 60 m of track?

additional power = ………………………….. W [2]

(iii) A simple pendulum hangs from the roof of one of the compartments. Calculate the
inclination of the string to the vertical as seen by a passenger as it slows down and
reaching station at a rate of 0.667 m s-2. Explain your reasoning with a suitable diagram.

angle of inclination = ………………..°

………………………………………………………………………………………………………………
………………………………………………………………………………………………………………
[3]

6
(b) A student wishes to find the volume of stone. He suspends the stone in air from a cord which is
attached to a force sensor as shown in Fig. 2.a. He then submerges the stone fully in a beaker
of water (see Fig. 2.b).

Force Force
sensor Sensor

Cord

Stone Beaker of
water

Fig. 2.a Fig. 2.b

(i) State and explain the change in the force sensor reading.

…………………………………………………………………………….......................................

................................................................................................................................................ [1]

(ii) Explain how volume of the stone can be determined from the readings of the force sensor
and the density of water.

…………………………………………………………………………….......................................

................................................................................................................................................

…………………………………………………………………………….......................................

................................................................................................................................................ [3]

7
4 A student sets up the apparatus illustrated in Fig. 4.1 in order to determine a value for the specific
latent heat of fusion of ice.

Fig 4.1

A heater is placed in the funnel, surrounded by pure melting ice. The student measures the mass
of melted ice in the beaker at regular time intervals before and after switching on the heater. The
variation with time t of the mass m of melted ice in the beaker is shown in Fig. 4.2.

Fig 4.2

During the heating process, the current is adjusted so that the readings on the ammeter and
voltmeter are constant.

8
(a) By reference to Fig. 4.2,
(i) Suggest a time at which the heater is switched on,

time = ………………….. minutes [1]

(ii) Determine the mass of ice melted in 1.0 minute

1. with the heater switched off,

mass = ……………………. kg [1]

2. with the heater switched on.

mass = ……………………. kg [1]

(b) The readings of the ammeter and the voltmeter are 5.2 A and 11.5 V respectively. Use your
answers in (a) to calculate a value for the specific latent heat of fusion of ice.

specific latent heat of fusion = ……………………….. J kg-1 [3]

(c) State and explain the effect on your calculated value for the specific latent heat of fusion if ice
taken directly from a freezer were used to replace the ice in the funnel.

…………………………………………………………………………………………………………..

…………………………………………………………………………………………………………..

…………………………………………………………………………………………………………..

………………………………………………………………………………………………………….. [2]

9
5 (a) A set of coloured lamps are designed for use with a 240 V supply. The set up have 12 lamps
connected as seen in Fig 5 below.

A B C D E F G H I J K L M
Fig. 5

240 V a.c.

However, the lamps do not light up when the set is plugged in. Therefore, a voltmeter is used
to test the circuit. For each of the following observations, identify the fault.

(i) The potential difference is zero across every lamp except EF, across which the potential
difference is 240 V.

………………………………………………………………………………………………………

…………………………………………………………………………………………………….
[1]
(ii) The potential difference between A and M is 240 V but the potential difference is zero
across every lamp.

………………………………………………………………………………………………………

……………………………………………………………………………………………………..
[1]
(b) (i) Some lamps are designed so that when the filament fails the resistance of the lamp
drops to zero. If this happens to one of the lamps in the above set up, calculate the
fractional increase in the power dissipated in each of the remaining lamps, assuming
that the resistance of these lamps does not change.

Fractional increase = _______________ [4]


(ii) What is likely to happen if failed lamps are not replaced?

…………………………………………………………………………………………………….

…………………………………………………………………………………………………….

…………………………………………………………………………………………………….

…………………………………………………………………………………………………….. [2]

10
6 Dangers associated with exposure to radiation have been recognised for many years. As a result
of these hazards, measures have been adopted to reduce exposure to radiation to as low a level
as possible. One such measure is to shield individuals from radioactive sources using radiation
absorbing materials.

Experiments have been carried out to investigate the effectiveness of materials as absorbers of
γ-ray photons. One possible experiment is illustrated in Fig. 6.1

The count rate Cx of γ-ray photons is measured for various thickness x of the absorber, together
with the count rate C0 for no absorber. Fig. 6.1 shows the variation with thickness x of the ratio
Cx/C0 for lead.

1.0

0.8

0.6
Cx/C0

0.4

0.2

0
0 2.0 4.0 6.0 8.0 10.0

x/cm

Fig. 6.1

(a) (i) What is a photon?

…………………………………………………………………………...................................
………………………………………………………………………………………………….
[1]

(ii) Suggest why it is necessary, in the experiment, to have a parallel beam of γ-radiation.

…………………………………………………………………………...................................
………………………………………………………………………………………………….
…………………………………………………………………………...................................
………………………………………………………………………………………………….
…………………………………………………………………………................................... [2]

11
(iii) State an evidence in Fig.6.1 for the fact that, theoretically, complete shielding is not
possible.

…………………………………………………………………………...................................
………………………………………………………………………………………………….
[1]

(b) Fig. 6.2 indicates that there may be an exponential decrease of the ratio Cx/ Co with
thickness x. In order to test this suggestion, a graph of ln (Cx/ Co) against x is plotted. This is
shown in Fig. 6.2
x/cm

0 2.0 4.0 6.0 8.0 10.0


0

-1.0
ln (Cx/ C0)

-2.0

-3.0

-4.0

-5.0

Fig. 6.2

(i) Show that Fig. 6.2 indicates a relationship of the form


Cx = Co e-μx,
Where μ is a constant.

[3]

(ii) The constant μ is known as the linear absorption coefficient. Use Fig. 6.2 to calculate
a value of μ for lead.

μ = …………………………….. cm -1 [2]

12
(c) The linear absorption coefficient μ has been found to depend on photon energy and on the
absorbing material itself. For γ-ray photons of one energy, μ is different for different
materials.

In order to assess absorption of γ-ray photons in matter such that the material of the
absorber does not have to be specified, a quantity known as the mass absorption coefficient
μm is calculated. μm is given by the expression

μm = μ/ρ

where ρ is the density of the absorbing material.

Values of μ for 2.75 MeV photons and of ρ for different materials are given in Fig. 6.3

material μ/ cm-1 ρ/ g cm-3 μm


Aluminium 0.095 2.70 0.035
Tin 0.267 7.28 0.037
Lead 11.3
Fig. 6.3

(i) give a consistent unit for μm,

unit of μm = …………………………. [1]


(ii) use your answer to (b) (ii) to complete the table of values for lead.

μ = …………………….. cm -1
μm = …………………………... [1]

(d) Concrete is a common building material which is sometimes used for shielding. The density
of concrete is 2.4 x 103 kg m-3.
(i) Use the information given in Fig. 6.3 to calculate an average value for μm and hence
show that the linear absorption coefficient μ for 2.75 MeV photons in concrete is
approximately 0.09 cm-1.

[2]

(ii) By reference to Fig. 6.1, calculate the approximate thickness of concrete which would
provide the same level of shielding, for 2.75 MeV photons, as thickness of 4.0 cm of
lead.

[3]
Thickness = …………………………… cm

13
(iii) Comment on your answer to (ii), making two suggestions as to why concrete maybe
used, in preference to lead, where radioactive sources of high activity are to be
shielded.

…………………………………………………………………………...................................
………………………………………………………………………………………………….
…………………………………………………………………………...................................
………………………………………………………………………………………………….
…………………………………………………………………………...................................
………………………………………………………………………………………………….
…………………………………………………………………………...................................
………………………………………………………………………………………………….
…………………………………………………………………………...................................
…………………………………………………………………………………………………. [2]

14
SECTION B

IT IS RECOMMENDED THAT YOU SPEND ABOUT 30 MINUTES ON THIS SECTION.

7 An air rifle can be used to fire small cylindrical metal pellets of about 5 mm diameter and 10
mm length at a speed of about 150 m s-1 on leaving the rifle. When an absorbent material is
placed some distance from the rifle, the pellets are observed to create a hole of a diameter
slightly larger than that of the pellet in the material and penetrate it to a depth of three or four
centimeters.

Design an experiment to investigate how the depth of penetration varies with the speed of the
pellet. Your answer should include a diagram and make particular reference to

(a) how the speed of the pellets may be measured,

(b) how the depth of penetration of the pellets into the absorbent material is to be measured,
(c) how the speed of the pellets is to be changed (assuming that the pellets leave the rifle
with a fixed speed),
(d) any safety precautions which should be taken during the experiment.

You may assume that all of the following equipment is available, together with any other
standard laboratory apparatus that would be found in a school science laboratory.

Air rifle
Analogue stopwatch
Digital stopwatch
Photogates
Electronic timer capable measuring time interval to an accuracy of 0.001s
Vernier calipers
Metre rule
Measuring tape
Sheets of absorbent material (cork)
Thin aluminium foils
Sheets of papers
Loud speaker
Microphone
Cathode ray oscilloscope
Power supply with connecting wires
Video camera
Stroboscope
[12]

15
Diagram

…………………………………………………………………………………………………………………………

…………………………………………………………………………………………………………………………

…………………………………………………………………………………………………………………………

…………………………………………………………………………………………………………………………

…………………………………………………………………………………………………………………………

…………………………………………………………………………………………………………………………

…………………………………………………………………………………………………………………………

…………………………………………………………………………………………………………………………

………………………………………………………………………………………………………………………..

…………………………………………………………………………………………………………………………

…………………………………………………………………………………………………………………………

…………………………………………………………………………………………………………………………

…………………………………………………………………………………………………………………………

…………………………………………………………………………………………………………………………

…………………………………………………………………………………………………………………………

…………………………………………………………………………………………………………………………

16
…………………………………………………………………………………………………………………………

………………………………………………………………………………………………………………………..

…………………………………………………………………………………………………………………………

…………………………………………………………………………………………………………………………

…………………………………………………………………………………………………………………………

…………………………………………………………………………………………………………………………

…………………………………………………………………………………………………………………………

…………………………………………………………………………………………………………………………

…………………………………………………………………………………………………………………………

…………………………………………………………………………………………………………………………

………………………………………………………………………………………………………………………..

…………………………………………………………………………………………………………………………

…………………………………………………………………………………………………………………………

…………………………………………………………………………………………………………………………

…………………………………………………………………………………………………………………………

…………………………………………………………………………………………………………………………

…………………………………………………………………………………………………………………………

…………………………………………………………………………………………………………………………

…………………………………………………………………………………………………………………………

………………………………………………………………………………………………………………………..

…………………………………………………………………………………………………………………………

…………………………………………………………………………………………………………………………

…………………………………………………………………………………………………………………………

…………………………………………………………………………………………………………………………

…………………………………………………………………………………………………………………………

…………………………………………………………………………………………………………………………

…………………………………………………………………………………………………………………………

…………………………………………………………………………………………………………………………

………………………………………………………………………………………………………………………..

17
…………………………………………………………………………………………………………………………

…………………………………………………………………………………………………………………………

…………………………………………………………………………………………………………………………

…………………………………………………………………………………………………………………………

…………………………………………………………………………………………………………………………

…………………………………………………………………………………………………………………………

…………………………………………………………………………………………………………………………

…………………………………………………………………………………………………………………………

………………………………………………………………………………………………………………………..

…………………………………………………………………………………………………………………………

…………………………………………………………………………………………………………………………

…………………………………………………………………………………………………………………………

…………………………………………………………………………………………………………………………

…………………………………………………………………………………………………………………………

…………………………………………………………………………………………………………………………

…………………………………………………………………………………………………………………………

…………………………………………………………………………………………………………………………

………………………………………………………………………………………………………………………..

…………………………………………………………………………………………………………………………

…………………………………………………………………………………………………………………………

…………………………………………………………………………………………………………………………

…………………………………………………………………………………………………………………………

…………………………………………………………………………………………………………………………

…………………………………………………………………………………………………………………………

…………………………………………………………………………………………………………………………

…………………………………………………………………………………………………………………………

………………………………………………………………………………………………………………………..

…………………………………………………………………………………………………………………………

…………………………………………………………………………………………………………………………

END OF PAPER 2

18
Name: ……………………….…………………. HT group: …………...

CATHOLIC JUNIOR COLLEGE


JC2 PRELIMINARY EXAMINATION 2010

PHYSICS 9646/ 3
Higher 2
Paper 3 Tuesday 14 September 2010
2 hours

READ THESE INSTRUCTIONS FIRST

There are 4 questions in Section A. Answer all questions in this section.


All essential working should be done in this booklet.
Total marks for Section A are 40 marks.

There are 3 questions in Section B. Answer 2 out of 3 questions in this section. Please
circle the 2 questions attempted on the cover page.
Write all answers to the questions on the question paper.
Write in dark blue or black pen on the writing paper provided. You may use a soft pencil for
any diagrams, graphs or rough working.
The number of marks is given in brackets [ ] at the end of each question or part question.
Total marks for Section B is 40 marks.

Do not use staples, paper clips, highlighters, glue or correction fluid.


Write your name and tutorial group on all the work you hand in.

Total marks for Section A and B is 80 marks.


A total of 2 marks will be deducted for mistakes made in units and significant figures.

This question paper consists of 20 printed pages.

AWARDED TOTAL
MARKS MARKS
SECTION A: Q1 11
Q2 10
Q3 4
Q4 15
SECTION B: Q5 20
Q6 20
Q7 20
TOTAL 80
PHYSICS DATA:

speed of light in free space, c = 3.00 x 108 m s−1


permeability of free space, μ0 = 4π x 10−7 H m−1
permittivity of free space, ε0 = 8.85 x 10−12 F m−1
= (1/(36π)) x 10-9 Fm−1
elementary charge, e = 1.60 x 10−19 C
the Planck constant, h = 6.63 x 10−34 J s
unified atomic mass constant, u = 1.66 x 10−27 kg
rest mass of electron, me = 9.11 x 10−31 kg
rest mass of proton, mp = 1.67 x 10−27 kg
molar gas constant, R = 8.31 J K−1 mol−1
the Avogadro constant, NA = 6.02 x 1023 mol−1
the Boltzmann constant, k = 1.38 x 10−23 J K−1
gravitational constant, G = 6.67 x 10−11 N m2 kg−2
acceleration of free fall, g = 9.81 m s−2

PHYSICS FORMULAE:

uniformly accelerated motion, s = ut + ½ at2


v2 = u2 + 2as
work done on/by a gas, W = p ΔV
hydrostatic pressure, p = ρgh
gravitational potential, φ = –Gm/r
displacement of particle in s.h.m. x = x0 sin ωt
velocity of particle in s.h.m. v = vo cosωt
= ±ω√(xo2 – x2)
resistors in series, R = R1 + R2 + …
resistors in parallel, 1/R = 1/R1 + 1/R2 + …
electric potential, V = Q/4πεor
alternating current/voltage, x = x0sinω t
transmission coefficient, T = exp(-2kd)
where k = 8 π 2 m(U − E )
h2
radioactive decay, x = x0 exp(- λt)
decay constant, λ = 0.693
t1
2

2
SECTION A
Answer ALL questions in this section.

1 (a) State what is meant by a field of force.

…………………………………………………………………………………………………………...…

…………………………………………………………………………………………………………...… [1]

(b) Atoms of Neon-20 are ionised by the removal of one electron from each atom. These ions are
accelerated through a potential difference of 1400 V. They are then injected into a region of
space where there are uniform electric and magnetic fields acting in right angles to the original
direction of motion of the ions. The region of magnetic and electric fields and polarities of the
electric plates are shown in Fig 1.1.

Region of uniform
electric and
+ + + + + + + + magnetic fields

Path of Neon ions

- - - - - - - -

Fig 1.1

The electric field has field strength E and the flux density of the magnetic field is B.

(i) State the charge of a Neon ion.

Charge of a Neon ion: …………………………. [1]

3
(iii) Calculate the speed of the Neon ions before they enter into the region of the electric
and magnetic fields.

Speed = …………………….. m s-1 [3]

(ii) (1) State the direction of the electric force acting on a Neon ion when it enters into the
shaded region.

Direction of electric force: ……………………. [1]

(2) Draw the direction of the magnetic field onto the shaded region in Fig 1.1 such
that the Neon ions pass through the shaded region undeflected. [1]

(iv) The electric field strength E is 6.2 x 103 V m-1. Calculate the magnitude of the magnetic
flux density so that the ions are not deflected in the region of the fields.

Magnetic flux density = …………………… T [2]

(c) The mechanism by which the Neon atoms in (b) are ionised is changed so that each atom
loses two electrons instead of just one. State the changes that occur in

(i) the speed of the ions entering the region of the electric and magnetic fields in (b).

……………………………………………………………………………………………...………

……………………………………………………………………………………………...……… [1]

(ii) the path of the ions in the two fields.

…………………………………………………………………………………………………...…

…………………………………………………………………………………………………...… [1]

4
2 (a) State what is meant by the internal energy of a system.

.......................................................................................................................................

...................................................................................................................................... [2]

(b) An ideal gas in a cylinder can be considered to undergo a cycle of changes of pressure,
volume and temperature as shown on the graph of Fig. 2.1.

7.5 x 10-3
Fig 2.1

The temperature of the gas at A and C are 623 K and 50 K respectively.

(i) Calculate the number of gas molecules in the cylinder.

Number of gas molecules = …………………………….. [2]

5
(ii) Calculate the volume of gas at C.

Volume of gas = ……………………….. m3 [2]

(iii) Calculate the net work done by the gas.

Net work done by gas = …………………….. J [2]

(iv) State with a reason the total change in internal energy of the gas when it completes a
cycle.

…………………………………………………………………………………………………….

…………………………………………………………………………………………………….

…………………………………………………………………………………………………….

……………………………………………………………………………………………………. [2]

6
3 When beryllium is bombarded with α-particles of energy 8.0 x 10-13 J, carbon atoms are produced,
together with a very penetrating radiation. A student suggested that the nuclear reaction might be

9
4 Be+ 24He→136 C + γ

(a) Explain what is meant by 136 C.

.................................................................................................................................................

…………………………………………………………………………………………………………. [1]

(b) (i) With the following information, calculate the energy released in the reaction as
suggested above.

Nuclide Mass/ u
9
Be 9.0150
4
4
He 4.0040
2
13
C 13.0075
6

Energy released = ……………………… J [2]


(ii) The energy of the penetrating radiation is found to be at least 8.8 x 10-12 J. Explain why
the student’s suggestion cannot be valid.

…………………………………………………………………………………………………….

……………………………………………………………………………………………………. [1]

7
4 (a) State what is meant by the photoelectric effect.

………….…………………………………………………………………………………………….…

……………….……………………………………………………………………………………….…

……………….……………………………………………………………………………………….… [2]

(b) A lamp is placed above a metal surface and an electron requires a minimum energy of 2eV
before it can be emitted from the metal surface.

(i) Calculate the maximum wavelength of the incident photons from the lamp.

λ = …………………………………. m [2]

(ii) State the kind of electromagnetic radiation that is emitted out from the lamp.

………………………………………………………………………………………………… [1]

(iii) The metal surface contains atoms of radius 2.0 x 10-10 m. It may be assumed that the
electron can collect energy from a circular area which has a radius equal to that of
the atom. The lamp provides energy at a rate of 0.40 W m-2 at the metal surface.
Estimate, on the basis of wave theory, the time required for an electron to collect
sufficient energy for it to be emitted from the metal.

Time required = ……………………… s [2]

(iv) Comment on your answer to (b)(iii)

…………………………………………………………………………..………………………

…………………………………………………………………………..………………………

…………………………………………………………………………..………………………

…………………………………………………………………………..………………………

…………………………………………………………………………..………………………

…………………………………………………………………………..………………………

…………………………………………………………………………..……………………… [2]

8
(c) A light dependent resistor is made of silicon which has a band gap energy Eg = 1.1 eV. Use
band theory to explain how the interaction of photons with electrons in this resistor might
change the resistivity of the material.

………………………………………………………………………………………………………….

………………………………………………………………………………………………………….

………………………………………………………………………………………………………….

………………………………………………………………………………………………………….

………………………………………………………………………………………………………….

…………………………………………………………………………………………………………. [2]

(d) The Scanning Tunnelling Microscope (STM) uses the principle of quantum tunnelling.
Briefly explain how this principle is used to map out the surface of a metal sample.

………………………………………………………………………………………………………….

………………………………………………………………………………………………………….

………………………………………………………………………………………………………….

………………………………………………………………………………………………………….

………………………………………………………………………………………………………….

………………………………………………………………………………………………………….

………………………………………………………………………………………………………….

………………………………………………………………………………………………………….

…………………………………………………………………………..…………………………..…

…………………………………………………………………………..…………………………..…

…………………………………………………………………………..…………………………..…

…………………………………………………………………………..…………………………..…

…………………………………………………………………………..…………………………..…

…………………………………………………………………………..…………………………..… [4]

END OF SECTION A

9
SECTION B
Answer 2 out of 3 questions in this section.

5 (a) Explain how an object travelling in a circle with constant speed has an acceleration. What is
the direction of this acceleration?

………..…………………………………………………………………………………………………….

………..…………………………………………………………………………………………………….

………..…………………………………………………………………………………………………….

………..…………………………………………………………………………………………………….

………..……………………………………………………………………………………………………. [3]

(b) (i) State the principle of conservation of momentum.

………..……………………………………………………………………………………………

………..……………………………………………………………………………………………

………..…………………………………………………………………………………………… [2]

(ii) A particle of mass m moving with speed v makes a head-on collision with an identical
particle which is initially at rest. Determine the subsequent motion of the particles after
they had made a completely inelastic collision.

………..……………………………………………………………………………………………

………..……………………………………………………………………………………………

………..…………………………………………………………………………………………… [2]

(iii) A bullet of mass 0.025 kg is travelling horizontally with a speed of 150 m s-1 when it
strikes the centre of a vertical face of a cubical block of mass 2.0 kg which is hanging at
rest from vertical strings. If the bullet embeds itself in the block, calculate the vertical
height risen by the block and bullet.

[4]
Height raised =…………….. m

10
(c) A railway truck of mass 22 000 kg and moving at a speed of 3 m s-1 catches up and collides
with a truck of mass 66 000 kg moving at 1 m s-1 moving in the same direction.

3 m s-1 1 m s-1
Spring buffers

22 000 kg 66 000 kg

The graph shows the speeds of the trucks before, during and after the collision.

V/ m s-1

0
0.000 0.100 0.200 time/ s

(i) Use the information in the graph to show that the collision is elastic.

[3]

(ii) Calculate the change in the total kinetic energy between the instant halfway through the
collision and after the collision. Suggest a reason for this change.

Change in Kinetic Energy = ………………………… J

……………………………………...………………………………………………………………

……………………………………...……………………………………………………………… [3]

11
(iii) Calculate the magnitude of the impulse exerted by the lighter truck on the heavier truck.

Impulse = ………………………… N s [2]

(iv) Explain whether or not this impulse changes in value if the collision remains elastic but
takes half the time.

……………………………………...………………………………………………………………

……………………………………...……………………………………………………………… [1]

12
6 (a) The circuit below in Fig 6.1 is set up, with a 20 V driver cell and Cell P of 12 V, each with
internal resistance 2 Ω.

Driver cell, e.m.f 20 V

2Ω

J
A B

4Ω

2Ω

Cell P, e.m.f 12 V

Fig 6.1

The uniform resistance wire AB is of length 100 cm, and the balanced length AJ is found to be
64 cm.

(i) Find the potential difference across the 4 Ω resistor.

Potential difference = ……………… V [2]

(ii) Calculate the resistance of the wire AB.

Resistance of AB = ……………… Ω [3]

13
(b) Define magnetic flux density and magnetic flux linkage.

……………………………………………………………………………………………………………...

……………………………………………………………………………………………………………...

……………………………………………………………………………………………………………...

……………………………………………………………………………………………………………... [3]

(c) A wire with resistance 8.66 Ω per metre and diameter 0.050 mm is used to construct an
electromagnet in the form of a hollow solenoid by winding one layer of close-packed turns
onto a plastic tube of length 200 mm and diameter 30 mm. The solenoid is connected in series
with a switch and a battery of e.m.f. 6.0 V and negligible resistance, as shown in Fig 6.2.

6.0 V

Fig 6.2

When the switch is turned on, the magnetic flux density B produced in the centre of the
solenoid is given by

B = μ0nI,

where n = number of turns per unit length.

(i) Calculate the number of turns of the solenoid.

Number of turns = ……………………… [1]

14
(ii) Calculate the resistance of the wire of the solenoid.

Resistance = ……………… Ω [2]

(iii) Calculate the maximum magnetic flux density produced by the electromagnet.

Maximum magnetic flux density = ……………… T [3]

(d) State and explain the effects on the magnetic flux density of each of the following changes.

(i) The plastic tube is filled with iron filling.

…………………………………………………………………………………………...…………

…………………………………………………………………………………………...…………

…………………………………………………………………………………………...…………

…………………………………………………………………………………………...………… [3]

(ii) Twice the length of similar wire is used so that the coil consists of two close-packed
layers.

…………………………………………………………………………………………...…………

…………………………………………………………………………………………...…………

…………………………………………………………………………………………...…………

…………………………………………………………………………………………...………… [3]

15
7 (a) (i) State a property that can be used to differentiate a transverse wave from a longitudinal
wave.

…………………………………………………………………………………………...…………

…………………………………………………………………………………………...………… [1]

(ii) State one feature of electromagnetic waves which are common across the whole
spectrum.

………………………………………………………………………………………………....…

…………………………………………….…………………………………………………...… [1]

(b) Two-source interference fringes using light can only be obtained if light from the two sources is
coherent. Explain

(i) the meaning of the term coherent.

……………………………………………………..…………………………………………...…

……………………………………………………..…………………………………………...…

…………………………………………………………………………………………...………… [2]

(ii) why, in practice, interference fringes can be seen only if light from a single source is
split into two.

…………………………………………………………………………………………………...…

…………………………………………………………………………………………………...… [2]

16
(c) In an experiment on superposition, light from a laser is incident normally on a double slit, and
the interference pattern is observed on a screen situated a distance D from the slits. The
fringe spacing x is measured for a number of different values of D and the graph is plotted, as
shown in Fig. 7.1.

x/ mm

20.0

18.0

16.0

14.0

12.0

10.0

8.0

6.0

4.0

2.0

0.5 1.0 1.5 2.0 2.5 3.0 3.5 4.0 4.5 5.0

D/m

Fig. 7.1

(i) Determine the gradient of the graph shown in Fig. 7.1.

gradient = ………………………………. [1]

17
(ii) Use your result in (c)(i) to determine the slit separation a, given that the wavelength of
the light is 600 nm.

a = ………………………… m [2]

The experiment is repeated with the frequency of the light slighty decreased. State what
will happen to

1. the interference pattern,

…………………………………………………………………………………………...…

…………………………………………………………………………………………...…

…………………………………………………………………………………………...… [1]

2. the graph shown in Fig. 7.1.

…………………………………………………………………………………………...…

…………………………………………………………………………………………...…

…………………………………………………………………………………………...… [1]

18
(d) Give reasons for the following.

(i) Sound waves and water waves can go round corners but light waves seem to travel
only in straight lines.

…………………………………………………………………………………………...…………

…………………………………………………………………………………………...…………

…………………………………………………………………………………………...………… [2]

(ii) Fig 7.2 represents a stationary sound wave in a pipe. This figure looks like a transverse
wave although sound waves are longitudinal waves.

node antinode node antinode

Fig 7.2

…………………………………………………………………………………………...…………

…………………………………………………………………………………………...…………

…………………………………………………………………………………………...………… [2]

19
(e) In a simple experiment to find out the wavelength of monochromatic red light emitted by a
laser, a fine beam of red laser light is shone through a diffraction grating as shown below in
Fig 7.3.

1.00 m
grating
laser

0.46 m

2.00 m

Fig 7.3

The diffraction grating has 300 000 rulings per meter and is set so that its plane is normal to
the incident light. Bright spots are observed at 0.46 m and 1.00 m from the central spot on a
screen, which is 2.00 m from the grating.

(i) From the first-order diffracted light, estimate the wavelength of the laser light.

λ = ………………………m [3]

(ii) State and explain an advantage of obtaining the wavelength of the laser light by using
the second-order diffracted light rather than the first-order diffracted light.

…………………………………………………………………………………………...…………

…………………………………………………………………………………………...…………

…………………………………………………………………………………………...………… [2]

END OF SECTION B
20
   
 
   
  YEAR 6 PRELIMINARY EXAMINATIONS 2010 
 
CANDIDATE
NAME

CLASS

PHYSICS 9646/01
Higher 2 27 September 2010
Paper 1 Multiple Choice Questions 1 hour 15 minutes

READ THESE INSTRUCTIONS FIRST


DO NOT TURN THIS PAGE OVER UNTIL YOU ARE TOLD TO DO SO.

Write your name and class at the top of this page.

There are forty questions in this Section. Answer all questions. For each question
there are four possible answers A, B, C and D.

Choose the one you consider correct and record your choice in soft pencil on the
separate Answer Sheet.

Each correct answer will score one mark. A mark will not be deducted for a wrong
answer.

This question paper consists of 20 printed pages (including this cover page).

 
DATA

speed of light in free space c = 3.00 × 108 m s-1


permeability of free space μo = 4π × 10-7 H m-1
permittivity of free space εo = 8.85 × 10-12 F m-1
(1/(36π)) × 10-9 F m-1
elementary charge e = 1.60 × 10-19 C
the Planck constant h = 6.63 × 10-34 Js
unified atomic mass constant u = 1.66 × 10-27 kg
rest mass of electron me = 9.11 × 10-31 kg
rest mass of proton mp = 1.67 × 10-27 kg
molar gas constant R = 8.31 J K–1 mol–1
the Avogadro constant NA = 6.02 x 1023 mol–1
the Boltzmann constant k = 1.38 x 10–23 J K–1
gravitational constant G = 6.67 × 10-11 N m2 kg-2
acceleration of free fall g = 9.81 m s−2
 
 

Dunman High School (Senior High) 9646/01/PRELIM/10  


   

 
 

FORMULAE
uniformly accelerated motion s = ut + ½at2

v2 = u2 + 2as

work done on/by a gas W = pΔV

hydrostatic pressure p = ρgh

gravitational potential Φ =

displacement of particle in s.h.m. x = xosinωt


velocity of particle in s.h.m. v = vocosωt
=
mean kinetic energy of a molecule of an ideal gas E =
resistors in series R = R1 + R2 + ……..

resistors in parallel 1/R = 1/R1 + 1/R2 + ……..

electric potential V =

alternating current/voltage x = xosinωt


transmission coefficient T exp(-2kd)

where k =

radioactive decay x = xoexp(-λt)

decay constant λ =

Dunman High School (Senior High) 9646/01/PRELIM/10 


   

 

1 The speed of transverse waves, v, in a stretched string is given by the


T
expression v = , where T is the tension of the string. The S.I. unit of the quantity µ
μ
is

A kg m-1 B kg-1 s2 m-1

C N m-2 D kg2 m-2

2 The velocity-time graph for a moving object is shown in the figure below.

Which of the following is the corresponding displacement-time graph?

   

A B

   

C D

Dunman High School (Senior High) 9646/01/PRELIM/10


[Turn over
 
   

 
3 A stone is projected with speed u from a cliff at a certain angle from the horizontal as
shown in the diagram below. The stone enters the water below at a horizontal
distance s from the side of the cliff. The diagram below shows the path of the stone
drawn to scale.

path

water surface
s
 

Assuming that air resistance is negligible, which one of the following statements is
correct?

A The distance s does not depend on the speed of projection, u, as it depends


only on the angle of projection of the stone.

B The vertical component of velocity of the stone is maximum just before the
stone hits the water.

C The velocity of the stone is instantaneously zero at the top of the stone's path.

D The acceleration of the stone reverses direction when the stone reaches the
top of its path.

4 A man is parachuting at constant speed towards the surface of the Earth. According
to Newton's third law of motion, the force that makes an action-reaction pair with the
gravitational force on the man is the

A tension in the ropes of the parachute.

B the gravitational force on the Earth due to the man.

C viscous force of the air on the man and his parachute.

D viscous force of the man and his parachute on the air.

Dunman High School (Senior High) 9646/01/PRELIM/10

                                               [Turn over
 
 

 
5 Three trolleys of equal mass m are stationary a short distance apart on a frictionless
track. The end trolley is given a constant velocity v towards the middle trolley. During
the collision between the end trolley and the middle trolley, the two stick together.
These two trolleys subsequently collide and stick to the third trolley.

What is the total loss of kinetic energy after these two collisions?

1 1 1 2
A mv 2 B mv 2 C mv 2 D mv 2
4 3 2 3

6 A forklift truck of weight W has dimensions shown in the diagram and its centre of
gravity G is midway between the front and rear axles.

x x x

front forks
G

w
front axle rear axle

When a uniform block of weight w (w < W) is placed on the front forks as shown, the
load on the rear axle

w
A decreases by
2

(W − w )
B decreases by
2

w
C increases by
2

(W − w )
D increases by
2

7 A ship has a horizontal cross-section of area 2.0 x 103 m2 and a mass of 6.6 x 106 kg
when unladen. It can be loaded safely until it sinks a further 1.00 m. If the density of
the sea-water is 1.20 x 103 kg m-3, what is the mass of cargo which the ship can
safely carry?

A 1.2 × 106 kg B 2.4 × 106 kg

C 4.8 × 106 kg D 6.6 × 106 kg

Dunman High School (Senior High) 9646/01/PRELIM/10


[Turn over
 
   

 
8 During a human heart beat, 20 g of blood is pushed into the main arties. This blood is
accelerated from a speed of 0.20 m s-1 to 0.34 m s-1. For a heart pulsing at 70 beats
per minute, the average power of the heart pump is

A 0.012 mW B 0.23 mW C 0.88 mW D 1.8 mW

 
9 The forward thrust provided by the engine of a car moving horizontally with constant
velocity of 12 m s 1 on a straight road is 500 N.

Which of the following statements is correct?

A The net force on the car is 500 N.

B The rate of work done by the engine is 6.0 kW.

C The power of the engine is zero as the car is moving at constant velocity.

D The power of the engine is 3.0 kW.


 
10 As illustrated in the figure to the right, a pendulum bob of
mass m is attached to a string which is hung from a L1  X
structure that rotates about XY at a constant angular
velocity ω. Given that the angle θ is 23.0o, L1 is 1.65 m
and L2 is 4.30 m, the number of revolutions made by the L2
pendulum in one second is θ

A 0.133 rev s-1 B 0.178 rev s-1

C 0.251 rev s-1 D 0.325 rev s-1


ω Y
 
 
11 A simple pendulum consists of a bob of mass m at the end of a light and inextensible
thread of length L. The other end of the thread is fixed at C. The bob swings through
point B with a velocity v and just reaches A, where the string is just taut.

L
A C
90°

What is the tension in the thread when the bob is at position B?

A mg B 2mg C 3mg D 4mg

Dunman High School (Senior High) 9646/01/PRELIM/10

                                               [Turn over
 
 

 
12 On the ground, the gravitational force acting on an object is 45 N. When the object is
at a height h above the ground, the gravitational force on it is 5 N. If R is the radius of
the earth, the approximate value for h is

A 2R B 3R C 4R D 5R 

13 P is a planet with centre O, as shown in the figure below. X and M are two points of
equal gravitational potential, ΦA. Y and N are two other points of equal gravitational
potential, ΦB.

Y
P
X

M
N
Which of the following statements is incorrect?

A The work done by an external agent to move a mass from Y to X is negative.

B The work done by the gravitational field to move a mass from X to N is the
same as that needed to move the same mass from M to Y.

C The work done by the gravitational field to move a mass from M to N is


positive.

φ A OY
D =
φB OX

14 Which one of the following is an effect of increased damping on a forced oscillation?

A The natural frequency of the oscillating system increases.

B The amplitude of forced oscillation at resonance increases.

C The total energy of the system remains unchanged.

D The system undergoes resonance at a lower frequency than the natural


frequency of undamped oscillation.

Dunman High School (Senior High) 9646/01/PRELIM/10


[Turn over
 
   

 

15 The figure below shows an oscillating system. The system is set to oscillate in two
different fluids, water and castor oil, which is of a higher viscosity than water.

Which graph correctly shows the variation of displacement x of the mass with time t
for both fluids?
  motion

 
fluid
 
smooth contact

x/m x/m

water oil

oil water

0 t/s 0 t/s

A B

x/m x/m
water oil

t/s t/s
0 0

oil water

C D

16 Which of the following statements about thermodynamic (Kelvin) scale is incorrect?

A Conversion between Celsius scale to Kelvin scale is T/oC = T/K + 273.15

B At zero Kelvin, the total internal energy of the system is at a minimum

C Absolute zero cannot be reached due to laws of thermodynamics

D The Kelvin scale does not depend on any thermometric properties

Dunman High School (Senior High) 9646/01/PRELIM/10

                                               [Turn over
 
 
10 
 
17 Material X of temperature TX and material Y of temperature Ty are in contact with
each other until there is no net transfer of energy between X and Y. Which of the
following statements is most correct?

A X and Y may not be in thermal equilibrium with each other

B The internal energy of X is equal to the internal energy of Y

C The specific heat capacity of X must be equal to that of Y

D The average kinetic energies of material X must be equal to that of Y

18 A sound wave of frequency 800 Hz is travelling in air at a speed of 320 m s−1. What is
the difference in phase between two points on the wave 0.10 m apart in the direction
of travel?

π π 3π
A B C D π
4 2 4
 
   

19 Which of the following effects provides direct experimental evidence that light is a
transverse, rather than longitudinal, wave motion?

A Interference patterns from the double slit experiment

B Particulate nature of light in the photoelectric effect experiment

C Polarised light from the reflection at a water surface

D Stationary microwaves obtained from microwaves emitter

20 An opaque object 10 cm wide casts a shadow when placed in a beam of light but has
little effect on a beam of sound emitted by a small source of frequency 300 Hz. This is
because

A Sound waves are longitudinal whereas light waves are transverse

B Sound waves travel much more slowly compared to light waves

C Light waves will be absorbed by the object but sound waves will not

D Sound waves have a much longer wavelength compared to light waves


 
 
 
 
 
 
 
Dunman High School (Senior High) 9646/01/PRELIM/10
[Turn over
 
   
11 
 
21 The diagram below illustrates an experimental arrangement that produces
interference fringes with a double slit.
X
Thin glass plate
S1

Monochromatic S2 Screen
light source
Y

When slit S1 is covered with a very thin plate of glass as shown,

A the separation of the fringe increases

B the separation of the fringe decreases

C the fringe pattern moves towards X

D the fringe pattern moves towards Y


 
 
22 The diagrams below show 5 different arrangements of charges around a circle with
centre O. Rank in ascending order the magnitude of the electric field strength at point
O

O O O O O

1 2 3 4 5

A 12354 B 41352 C 12534 D 43521


 
 
 
 
 
 
 
 
 
 
 
 
 

Dunman High School (Senior High) 9646/01/PRELIM/10

                                               [Turn over
 
 
12 
 
23 The diagram below is a scaled drawing showing the equi-potential lines in the region
of an electric field.

‐2
10  m 

Which point has an electric field strength of the greatest magnitude?

A N B R C T D U
 
 
24  Two cylindrical resistors, one made of copper and the other of aluminum, have the
same volume. The cross-section area and the resistivity of aluminum are both three
times that of copper. What is the magnitude of the ratio
resistance of aluminium resistor
?
resistance of copper resistor

1 1
A 9 B 3 C D
3 9

Dunman High School (Senior High) 9646/01/PRELIM/10


[Turn over
 
   
13 
 
25 The figure shows the graph of current against potential difference for two electrical
devices P and Q.

P and Q are joined in series. If the potential difference across P is 10 V, which of the
following is correct?

p.d. across Q/ V p.d. across P and Q/ V

A 5.0 10

B 5.0 15

C 15 15

D 15 25

26 Two resistors, R1 and R2 are connected in parallel. R1 has a fixed value and the value
of R2 is variable but always greater than R1. The combined resistance is

A less than R1 and decreases as R2 increases

B less than R1 and decreases as R2 decreases

C greater than R1 and increases as R2 increases

D greater than R1 and decreases as R2 increases

Dunman High School (Senior High) 9646/01/PRELIM/10

                                               [Turn over
 
 
14 
 
27 In the circuit shown, X, Y and Z are three identical lamps. Initially switch S is closed.

When switch S is opened, the brightness of

A X decreases and that of Y increases

B X increases and that of Y decreases

C X stays the same and that of Y increases

D X increases and that of Y stays the same

28 An electron moves in a circular path in a uniform magnetic field of flux density 1.0 mT.
If an α-particle of the same speed is to follow an identical path, what must be the
magnitude and direction of the magnetic field? The mass of α-particle is 7200 times
the mass of an electron

Magnitude of magnetic field/ mT Direction of magnetic field

A 3600 in opposite direction

B 1800 in opposite direction

C 1800 in same direction

D 3600 in same direction

Dunman High School (Senior High) 9646/01/PRELIM/10


[Turn over
 
   
15 
 
29 Two long straight wires X and Y are placed perpendicular to each other at a distance
d apart. A current flows out of the page in wire X. The same current flows from left to
right in wire Y. What are the directions of the forces acting on wire Y at points P and
Q due to the magnetic field produced by wire X?

Force at P Force at Q

A out of page into page

B into page out of page

C towards X away from X

D towards X towards X

30 A rectangular conductor is moving horizontally from left to right in a uniform magnetic


field perpendicular to UVWX. Which of the following pairs of points will NOT be at the
same potential?

A U and V B U and X C V and W D W and Z

Dunman High School (Senior High) 9646/01/PRELIM/10

                                               [Turn over
 
 
16 
 
31 In Fig (a) below, a beam of particles, each of charge q and mass m, is travelling at
speed v through a region in which a magnetic field B is perpendicular to an electric
field E. The beam is undeflected by the crossed electric and magnetic fields.

In Fig (b) below, the electric field is switched off. The beam is found to form an arc of
a circle of radius r.

Which one of the following expressions gives the mass m of the charged particle?

 
qBr qBv qB 2 r qB 2
A B C D
E E E rE

32 Alternating current of sinusoidal and square waveforms, as shown below, flow in turn
through a resistor.

The power dissipated is the same in each case. Given that the peak value of the
sinusoidal current is 1.0 A, what is the peak value, in A, of the square waveform?

1 1
A 2 B 2 C D
2 2

Dunman High School (Senior High) 9646/01/PRELIM/10


[Turn over
 
   
17 
 
33 A metal rod OP of length L is rotated uniformly about O in a plane perpendicular to a
uniform magnetic field of flux density B. The rate of rotation is N rev s-1. An electron e
is found to be stationary at P.

O P

What is the magnetic and electric force acting on the electron at P?

Magnetic force Electric Force

A πeLNB πeLNB

B πeLNB 2πeLNB

C 0 0

D 2πeLNB 2πeLNB

34 In the figure below, a windmill drives a 100 V generator which produces electrical
energy at a rate of 1000 W. This energy is supplied to a distant house through two
cables of total resistance 2.0 Ω. T1 and T2 are two transformers of 100% efficiency
used to step up and step down the voltage respectively.

What is the rate of energy dissipated in the cables?

A 1.0 W B 2.0 W C 20 W D 100 W

Dunman High School (Senior High) 9646/01/PRELIM/10

                                               [Turn over
 
 
18 
 
35 The figure on the right shows five energy levels of an atom,
one being much lower than the other four. E5 
Five transitions between the levels are indicated, E4 
each of which produces a photon of definite energy  
and frequency. E3 
 
Which one of the spectra below best corresponds to the E2 
set of transitions indicated?  
 
(high) Frequency (low)  
 
A  
E1 

36 Electrons emitted by a hot filament pass down a tube containing hydrogen and are
then collected by an anode which is maintained at a positive potential with respect to
the filament. The gas near the anode is found to emit monochromatic ultra-violet
radiation. Given that the energy levels of an electron in a hydrogen atom is

13.6
E= (eV), where n = 1, 2, 3…
n2

What is the wavelength of the monochromatic ultra-violet radiation?


 
A 122 nm B 102 nm C 97 nm D 91 nm

37 The scanning tunnelling microscope is able to obtain atomic-scale images of surface


mainly due to

A quantum tunnelling which is very sensitive to distance

B the atomic wide tip of the probe used for surface scanning

C the free electrons in the conduction band of the metallic probe

D the massive atoms at the tip which contributes to a smaller wavelength in


wave particle duality

Dunman High School (Senior High) 9646/01/PRELIM/10


[Turn over
 
   
19 
 
38 Which of the following is the correct band diagram for p-type and n-type
semiconductors?

p-type n-type

Conduction Conduction
Band Band
A − − − − − − − − − − 

− − − − − − − − − −
 
  Valence Band Valence Band
 
 
 
 
 
Conduction Conduction
Band Band
B − − − − − − − − − −
  − − − − − − − − − − 
 
  Valence Band Valence Band
 
 
 
 
 
Conduction Conduction
 
Band Band
C − − − − − − − − − −

Valence Band oooooooo


Valence Band

Conduction Conduction
Band Band
− − − − − − − − − − 
D
 
  oooooooo
  Valence Band
  Valence Band
 
 
Dunman High School (Senior High) 9646/01/PRELIM/10

                                               [Turn over
 
 
20 
 
39 One reaction that may be used for nuclear fusion is shown

7 2 4
(
3 Li+ 1 H → 2 2 He )+ X
What is particle X?

A an α particle B an electron

C a neutron D a proton

 
40 The diagram below shows a graph of the binding energy per nucleon for a number of
naturally occurring nuclides plotted against their mass number

27
13 Al
23 238
11Na   92 U

Which of the following statements is a correct deduction from the graph?

A Uranium (U-238) is the most stable nuclides amongst the 3 plotted

B Energy will be released if sodium (Na-23) undergoes nuclear fission

C Aluminum (Al-27) will spontaneously decay to sodium (Na-23) with alpha


particle

D Nuclear fusion between sodium (Na-23) and aluminum (Al-27) will release
energy

End of Paper

Dunman High School (Senior High) 9646/01/PRELIM/10


[Turn over
 
   
   
 
   
  YEAR 6 PRELIMINARY EXAMINATIONS 2010 
 
CANDIDATE
NAME

INDEX
CLASS
NUMBER

PHYSICS 9646/02
Higher 2 17 September 2010
Paper 2 Structured Questions 1 hour 45 minutes

READ THESE INSTRUCTIONS FIRST For Examiner’s Use


¾ DO NOT TURN THIS PAGE OVER UNTIL YOU ARE 1 / 9
TOLD TO DO SO.
2 / 7
¾ Write your name, class and index number at the top of
this page. 3 / 6

4 / 11
¾ Answer ALL questions.
5 / 7
¾ For numerical answers, all working should be shown.
6 / 20

¾ The number of marks is given in brackets [ ] at the end 7 / 12


of each question or part question.
S.F. Units

Total / 72
 

This question paper consists of 18 printed pages (including this cover page).

 

DATA
speed of light in free space c = 3.00 × 108 m s-1
permeability of free space μo = 4π × 10-7 H m-1
permittivity of free space εo = 8.85 × 10-12 F m-1
(1/(36π)) × 10-9 F m-1
elementary charge e = 1.60 × 10-19 C
the Planck constant h = 6.63 × 10-34 Js
unified atomic mass constant u = 1.66 × 10-27 kg
rest mass of electron me = 9.11 × 10-31 kg
rest mass of proton mp = 1.67 × 10-27 kg
molar gas constant R = 8.31 J K–1 mol–1
the Avogadro constant NA = 6.02 x 1023 mol–1
the Boltzmann constant k = 1.38 x 10–23 J K–1
gravitational constant G = 6.67 × 10-11 N m2 kg-2
acceleration of free fall g = 9.81 m s−2
 
 
 

Dunman High School (Senior High) 9646/02/Prelim/2010  


   

 
 

FORMULAE
uniformly accelerated motion s = ut + ½at2

v2 = u2 + 2as

work done on/by a gas W = pΔV

hydrostatic pressure p = ρgh

gravitational potential Φ =

displacement of particle in s.h.m. x = xosinωt


velocity of particle in s.h.m. v = vocosωt
=
mean kinetic energy of a molecule of an ideal gas E =
resistors in series R = R1 + R2 + ……..

resistors in parallel 1/R = 1/R1 + 1/R2 + ……..

electric potential V =

alternating current/voltage x = xosinωt


transmission coefficient T exp(-2kd)

where k =

radioactive decay x = xoexp(-λt)

decay constant λ =

Dunman High School (Senior High) 9646/02/Prelim/2010


   
For 
4  Examiner’s
 
Use 
Answer ALL questions

1. An object of mass 1.5 kg is released from a stationary hot air balloon. Fig 1 shows
how the velocity of the object varies with time.

Fig 1

(a) By considering the forces acting on the falling object, explain the shape of the
graph in the regions marked A, B and C. [3]

Dunman High School (Senior High) 9646/02/Prelim/2010  


   
For 

Examiner’s
 
Use 
(b) Using Fig 1, determine the distance fallen in the first 16 s [2]

Distance fallen in first 16 s = _______________ m

(c) (i) Using your answer from (b), calculate the change in gravitational potential
energy ∆Ep of the object that occurred during the first 16 s. [1]

∆Ep = _______________ J

(ii) Calculate the change in kinetic energy ∆Ek of the object during the same
period. [1]

∆Ek = _______________ J

(iii) Hence, determine the average resistive force acting on the object during this
period [2]

Average resistive force = _______________ N

Dunman High School (Senior High) 9646/02/Prelim/2010 [Turn over 


   
For 
6  Examiner’s
 
Use 
2. (a) Define impulse and state how it is related to the momentum of an object. [2]

(b) Fig 2 shows a steel bar being reduced in thickness as it moves between two
rollers.

Fig 2

The steel is fed into the rollers at a speed of 0.40 m s-1 and emerges at 0.50 m s-1.
The thickness of the metal is reduced from 0.30 m to 0.24 m and the width of the
metal remains a constant at 0.85 m.

(i) Show that approximately 840 kg of steel passes through the rollers in each
second given that the density of steel is 8200 kg m-3. [1]

(ii) Calculate the change in momentum of the steel that passes through the
rollers in each second. [2]

Change in momentum in 1 s = _______________ N s

Dunman High School (Senior High) 9646/02/Prelim/2010  


   
For 

Examiner’s
 
Use 
(iii) State the magnitude and direction of the force that acts on the rollers due to
the change in momentum of the steel. [2]

Magnitude = _______________ N

Direction = _________________

3. The diffraction pattern due to a single light source on a single slit is shown in the Fig
3.1 below.

source slit 

screen

Fig 3.1

(a) Explain what is meant by the term diffraction? [1]

Two additional slits were placed between the single slit and the screen as shown in
Fig 3.2 and an interference pattern is observed on the screen.

source  slits

screen
Fig 3.2

Dunman High School (Senior High) 9646/02/Prelim/2010 [Turn over 


   
For 
8  Examiner’s
 
Use 
(b) (i) Explain what is meant by term coherent? [1]

(ii) The slits are 3.5 × 10−6 m apart and the wavelength of the light is 0.65 × 10−6
m. Calculate the fringe spacing x if the screen is placed 2.5 m away from the
double slits. [2]

x = _______________ m

(iii) State and explain what will happen to the interference pattern if the single slit
is now removed. [2]

4. In Fig 4, a uniform magnetic field B pointing into the paper and a uniform electric field
E pointing upwards are applied on the left−hand side and right−hand side of y-axis
respectively.

Fig 4

Dunman High School (Senior High) 9646/02/Prelim/2010  


   
For 

Examiner’s
 
Use 
A particle of mass m and charge q enters the magnetic field with an initial velocity vo
at point P. After leaving the magnetic field, it moves into the electric field and leaves
at point S. Neglect gravitational effects.

(a) (i) What is the sign of the charge carried by the particle? [1]

(ii) Describe the motion of the particle in the two different fields. [2]

(b) Deduce an expression, in terms of m, q, B, E or vo, where appropriate, for the


time taken for the particle to travel through

(i) the magnetic field, [3]

(ii) the electric field [3]

Dunman High School (Senior High) 9646/02/Prelim/2010 [Turn over 


   
For 
10  Examiner’s
 
Use 
(c) If the velocity of the particle at point S is v, deduce an expression, in terms of m,
vo and v, for the work done on the particle by the field as the particle travel
through

(i) the magnetic field [1]

(ii) the electric field [1]

5. (a) A particular material is designed to emit photoelectrons when visible light is


incident on it. When light of wavelength 535 nm is incident on it, electrons are
emitted has a stopping potential of about 1.30 V.

(i) Calculate the energy of the photon incident onto the material. [2]

Energy of incident photon = _______________ J

(ii) Calculate the work function of the material. [2]

Work function = _______________ J

Dunman High School (Senior High) 9646/02/Prelim/2010  


   
For 
11 
Examiner’s
 
Use 

(b) The material used in (a) is most likely metallic in nature. By considering the band
theory of materials, state and explain what will happen if the experiment is
repeated for a semiconductor instead of a metal. [3]

6. Since Albert Einstein established the theoretical foundations for the laser in his 1917
paper “On the Quantum Theory of Radiation”, we saw many applications of laser in
our modern life. In 1960, the first laser was built by Theodore Maiman and he used a
method known as optical pumping to achieve population inversion. Gas lasers, such
as the Helium-Neon laser, which made use of electrical pumping soon followed.

(a) LASER is an acronym for Light Amplification by Stimulated Emission of Radiation,


explain how amplification of light may be achieved by stimulated emission of
radiation. [3]

Dunman High School (Senior High) 9646/02/Prelim/2010 [Turn over 


   
For 
12  Examiner’s
 
Use 

Lasers that cannot sustain population inversion for long periods of time may be
deployed in pulsed mode. In pulsed mode, lasing occurs for short period of time
hence the laser output is in pulses. Alternatively, the pulsed pumping can be used to
achieve a pulse laser. The graph in Fig 6.1 shows the output power of a certain pulse
laser with time.

Fig 6.1

(b) (i) State the period of the pulse laser. [1]

Period = _______________ ns

(ii) State the peak power of the laser. [1]

Peak Power = _______________ W

(iii) Estimate the total energy emitted by a single pulse. [1]

Total energy by a single pulse = _______________ J

(iv) Calculate the mean power of such a pulse laser. [2]

Mean power = _______________ W


Dunman High School (Senior High) 9646/02/Prelim/2010  
   
For 
13 
Examiner’s
 
Use 

Dunman High School (Senior High) 9646/02/Prelim/2010 [Turn over 


   
For 
14  Examiner’s
 
Use 

(c) Lasers may be focused onto an even smaller circular area with the use optical
lens in a microscope as illustrated in Fig 6.2.
 
 
Focused circle
 
 
  1000 μm  93 μm 
 
 
  Not to scale Image of the 
  focused laser 
Fig 6.2

The diameter of the laser spot and magnification of the optical lens used in
experimentation are tabulated below. The wavelength of the laser is 553 nm.

Magnification, M Laser Spot Diameter, D / μm


1× 1000
10 × 93
20 × 46
50 × 18

The relationship between magnification and laser spot diameter can be estimated
with
M× D = K
where K is a constant.

(i) Without drawing a graph, determine the value for K [3]

K = ____________________

(ii) Calculate the diameter of the laser spot at 200× magnification. [1]

Dunman High School (Senior High) 9646/02/Prelim/2010  


   
For 
15 
Examiner’s
 
Use 
Diameter = _______________ μm

(iii) Suggest if your answer in (ii) is an underestimate or overestimate. Explain


your answer clearly. [2]

(d) The laser spot is not uniform in its brightness. The graph in Fig 6.3 shows the
approximate beam profile of the same laser in (c) about its centre when focused
using an unknown lens.

Fig 6.3

The diameter of a laser spot is determined by its full width of the spot at half
maximum intensity. Estimate the magnification of the lens used. [2]

Magnification = _______________ X

Dunman High School (Senior High) 9646/02/Prelim/2010 [Turn over 


   
For 
16  Examiner’s
 
Use 
(e) Besides the laser beam profile, the wavelength of the photons emitted by the
laser may be studied. The wavelength of photons emitted by the laser is found to
vary by 1% or even less as shown in Fig 6.4.

Fig 6.4

(i) Suggest how the figure above provide evidence that there are energy level
splitting about each discrete energy levels of electrons [2]

(ii) For ultra-short pulse laser of 10−15 s, the variation in laser wavelength is even
greater than what is shown in the figure above. With appropriate calculations,
explain why this is so. [2]

Dunman High School (Senior High) 9646/02/Prelim/2010  


   
For 
17 
Examiner’s
 
Use 
7. The attenuation of a γ-ray beam is the reduction in its intensity due to its passage
through a material. One way of investigating the attenuation of a γ-ray beam is to
measure the half-value thickness. The half value thickness of a material is the
thickness that reduces the intensity of the γ-ray beam to half its original value.

Design a laboratory experiment to measure the half value thickness of steel. You may
assume that you have access to steel plates of varying thickness as well as a
radioactive source emitting both α and γ radiation and other standard Physics
laboratory equipments. [12]

Dunman High School (Senior High) 9646/02/Prelim/10  


   
For 
18  Examiner’s
 
Use 

End of Paper
Dunman High School (Senior High) 9646/02/Prelim/2010  
   
   
 
   
  YEAR 6 PRELIMINARY EXAMINATIONS 2010 
 
CANDIDATE
NAME

EXAM INDEX
CLASS
NUMBER

PHYSICS 9646/03
Higher 2 24 September 2010
Paper 3 Longer Structured Questions 2 hours

READ THESE INSTRUCTIONS FIRST For Examiner’s Use


¾ DO NOT TURN THIS PAGE OVER UNTIL YOU ARE Section A
TOLD TO DO SO.
1 / 8
¾ Write your name and exam index number at the top of
this page. 2 / 10

3 / 7
¾ There are 2 sections to this paper,
4 / 9
Section A: Structured Questions
Answer ALL questions 5 / 6

Section B: Longer Structured Questions Section B


Answer ANY 2 questions
6 / 20

¾ For numerical answers, all working should be shown. 7 / 20

8 / 20
¾ The number of marks is given in brackets [ ] at the end
of each question or part question. S.F. Units

Total / 80
 

This question paper consists of 25 printed pages (including this cover page).

 

DATA
speed of light in free space c = 3.00 × 108 m s-1
permeability of free space μo = 4π × 10-7 H m-1
permittivity of free space εo = 8.85 × 10-12 F m-1
(1/(36π)) × 10-9 F m-1
elementary charge e = 1.60 × 10-19 C
the Planck constant h = 6.63 × 10-34 Js
unified atomic mass constant u = 1.66 × 10-27 kg
rest mass of electron me = 9.11 × 10-31 kg
rest mass of proton mp = 1.67 × 10-27 kg
molar gas constant R = 8.31 J K–1 mol–1
the Avogadro constant NA = 6.02 x 1023 mol–1
the Boltzmann constant k = 1.38 x 10–23 J K–1
gravitational constant G = 6.67 × 10-11 N m2 kg-2
acceleration of free fall g = 9.81 m s−2
 
 
 

Dunman High School (Senior High) 9646/03/Prelim/2010  


   

 
 

FORMULAE
uniformly accelerated motion s = ut + ½at2

v2 = u2 + 2as

work done on/by a gas W = pΔV

hydrostatic pressure p = ρgh

gravitational potential Φ =

displacement of particle in s.h.m. x = xosinωt


velocity of particle in s.h.m. v = vocosωt
=
mean kinetic energy of a molecule of an ideal gas E =
resistors in series R = R1 + R2 + ……..

resistors in parallel 1/R = 1/R1 + 1/R2 + ……..

electric potential V =

alternating current/voltage x = xosinωt


transmission coefficient T exp(-2kd)

where k =

radioactive decay x = xoexp(-λt)

decay constant λ =

Dunman High School (Senior High) 9646/03/Prelim/2010 


   
For 
4  Examiner’s
 
Use 
Section A
Answer ALL questions in this section.

1 A satellite is a body that revolves around a planet. A satellite in a geostationary orbit


around the Earth does not appear to move relative to any fixed observer on Earth.

Mass of the Earth ME = 6.0 × 1024 kg


Radius of Earth RE = 6.4 × 106 m

(a) (i) One condition for a satellite to be in geostationary orbit is that the orbit of the
satellite must lie in the plane containing the equator. Explain why this is
necessary. [2]

(ii) Using Newton’s law of gravitation, explain why all geostationary satellites
have the same orbital radius around the earth which is independent of the
mass of the satellite. [2]

(iii) Determine the orbital radius of a geostationary satellite. [1]

Orbital radius = _______________ m

Dunman High School (Senior High) 9646/03/Prelim/2010  


   
For 

Examiner’s
 
Use 
(b) (i) Show that the kinetic energy of a satellite of mass ms in an orbital radius R
around the earth is given by the expression

[1]

(ii) Using your answer to (a)(iii) and the expression in (b)(i), determine the
minimum amount of energy required to put a satellite of mass 1500 kg into a
geostationary orbit. You may ignore the effects due to the rotation of the
Earth about its own axis. [2]

Energy required = _______________ J

Dunman High School (Senior High) 9646/03/Prelim/2010 


    [Turn over
For 
6  Examiner’s
 
Use 

2 Fig 2.1 below shows a simple vertical mass-spring system with the load oscillating in
a vertical plane. The spring has a spring constant k and the load a mass m. The load
may be considered to be in simple harmonic oscillation if the extension of the spring
does not exceed the limit of proportionality.

Vertical 
LOAD  Oscillation

Fig 2.1

By considering the forces acting on the platform and applying Newton’s second law, it
can be shown that when the spring obeys Hooke’s law and the extension of the
spring does not exceed the limit of proportionality, the acceleration of the load in the
vertical direction is given by

where x is the vertical displacement of the load measured from its equilibrium position.

(a) (i) Explain why the load may be considered to be in simple harmonic motion if
the spring obeys Hooke’s law and the extension of the spring does not
exceed the limit of proportionality. [1]

(ii) Show that the expression for the frequency of oscillation f of the vertical
mass-spring system is given by

[1]

Dunman High School (Senior High) 9646/03/Prelim/2010  


   
For 

Examiner’s
 
Use 

(b) For a system with a spring of spring constant 100 Nm-1, a load of mass 1.0 kg
and a vertical amplitude of oscillation of 0.20 m, calculate the magnitude of the
velocity of the platform as it passes the equilibrium position. [2]

Magnitude of velocity = _______________ m s-1

(c) The suspension system of a car, as shown in Fig 2.2, may be modeled as
vertical mass-spring system combined with a shock absorber that damps the
vertical oscillations of the car.

shock absorber 

spring 

Fig 2.2

When a driver of mass 90 kg steps into a car of mass 1410 kg, the vertical height
of the car above the road decreases by 2.0 cm. If the car is driven over a series
of equally spaced bumps, the amplitude of vibration becomes much larger at a
particular speed.
Dunman High School (Senior High) 9646/03/Prelim/2010 
    [Turn over
For 
8  Examiner’s
 
Use 
(i) Explain why the amplitude of vibration of the car becomes much larger at a
particular speed. [2]

(ii) Calculate the effective spring constant of the suspension system. [1]

Effective spring constant = _______________ N m-1

(iii) Using the expression given in (a)(ii) and your answer to (c)(ii), determine the
separation between the bumps if large amplitude vibrations occurs at a
speed of 18 m s-1. [3]

Separation of bumps = _______________ m

Dunman High School (Senior High) 9646/03/Prelim/2010  


   
For 

Examiner’s
 
Use 

3. Ice of 50g at temperature −15 oC is added to water at 30oC in an insulated container.

Specific heat capacity of ice = 2100 J kg−1 K−1


Specific heat capacity of water = 4200 J kg−1 K−1
Specific latent heat of fusion of water = 3.3 × 105 J kg−1

(a) Calculate the quantity of energy needed to change the ice at −15oC to water at
0oC [3]

Energy needed = _______________ J

(b) Calculate the mass of the water in the container if the lowest temperature
reached by the water is 7.5oC, assuming no heat is lost to the surroundings. [3]

Mass of water = _______________ kg

(c) State one other assumption that you have made in your calculations in (b). [1]

Dunman High School (Senior High) 9646/03/Prelim/2010 


    [Turn over
For 
10  Examiner’s
 
Use 

4. Jingwen wants to determine the internal resistance r of a cell of e.m.f. E using a


potentiometer as shown in Fig 4.
Eo

lo
P Q

E r G 

Fig 4

When switch S is open, the balance length is lo. When switch S is closed, the balance
length is l.

(a) (i) When switch S is closed, obtain an expression for V, the potential difference
across the variable resistor R in terms of the e.m.f. E and internal resistance
r of the cell [1]

(ii) Hence or otherwise, show that the internal resistance r of the cell is

[2]

Dunman High School (Senior High) 9646/03/Prelim/2010  


   
For 
11 
Examiner’s
 
Use 

(iii) She found that when R = 12 Ω, lo = 82.3 cm and l = 78.9 cm. Determine r. [1]

r = _______________ Ω

(b) She repeated the experiment with R set to (i) 4.0 Ω and (ii) 30 Ω. Explain which
value of R used would result in a more reliable determination of r. [2]

(c) The value of r can also be found by drawing a suitable straight line graph.
Explain how this can be done. [3]

 
 

Dunman High School (Senior High) 9646/03/Prelim/2010 


    [Turn over
For 
12  Examiner’s
 
Use 
 

5. A stationary radon (Rn) nucleus may decay spontaneously into a polonium (Po)
nucleus and an α-particle as shown below. It may be assumed that no γ-ray is
emitted in the process.

The rest masses of the polonium nuclei and constituents nucleons are
218
Po : 218.0090u
Proton : 1.007276u
Neutron : 1.008664u

(a) Explain what is meant by spontaneous? [1]

(b) Show that the binding energy per nucleon of Polonium is about 1.207 × 10−12 J
[3]

(c) Calculate the total kinetic energy of the decay if the binding energy per nucleon
of 222Rn is 1.201 × 10−12 J and that of 4He is 1.092 × 10−12 J. [2]

Dunman High School (Senior High) 9646/03/Prelim/2010  


   
For 
13 
Examiner’s
 
Use 
Kinetic Energy = _______________ J

Dunman High School (Senior High) 9646/03/Prelim/2010 


    [Turn over
For 
14  Examiner’s
 
Use 

Section B
Answer ANY 2 questions in this section.
6. Ink-jet printing is a non-impact, dot matrix technology in which ink droplets are
projected through an array of tiny nozzles onto paper.

In a continuous flow print head, ink-jet droplets are approximately 80 μm in diameter


and can be charged negatively as they pass through a charging ring electrode
immediately after they break away from the nozzle of the print head. Whether or not
a droplet is charged is controlled by a signal sent to the printer by a computer. An ink
droplet that becomes charged would be deflected into a gutter when it passes
through a region with a uniform electric field strength while uncharged droplets pass
through undeflected to reach the paper.

4.0 × 10‐4 m 
20 m s‐1

+ − 
5.0 × 10‐4 m 

35o
5.0 × 10‐4 m 

Gutter to collect  x
deflected ink droplets
5.0 × 10‐4 m 
Paper 

Fig 6

(a) Fig 6 shows the parallel deflecting plates with some dimensions of the ink-jet
cartridge. The ink drops falls straight through the centerline between the
deflecting plates with a constant vertical velocity of 20 m s-1. In order to land in
the center of the gutter, a charged ink droplet must leave the deflecting plates at
an angle of 350.

(i) Calculate the horizontal component of velocity as a charged ink droplet


leaves the deflecting plates. [1]

Dunman High School (Senior High) 9646/03/Prelim/2010  


   
For 
15 
Examiner’s
 
Use 
Horizontal component of velocity = _______________ m s-1

(ii) Determine the horizontal acceleration of the charged ink droplet for which it
is between the deflecting plates. [3]

Horizontal acceleration = _______________ m s-2

(iii) Determine the horizontal distance x between the centre of the gutter and the
centerline of between the deflecting plates. [3]

x = _______________ m

(b) (i) Explain the term uniform electric field strength. [2]

(ii) Sketch on Fig 6 the electric lines representing electric field in the region
between the pair of parallel deflecting plates. [2]

Dunman High School (Senior High) 9646/03/Prelim/2010 


    [Turn over
For 
16  Examiner’s
 
Use 
(iii) Each ink droplet has a mass of mass 2.9 × 10-10 kg. Using your answer to
(a)(ii), determine the electric force FE acting on the charged ink droplet while
it is between the deflecting plates. [2]

FE = _______________ N

(iv) If the charge gained by the ink droplet is -2 × 10-10 C, determine the electric
field strength E between the deflecting plates. [2]

E = _______________ N C-1

(v) Hence, calculate the potential difference V between the deflecting plates. [2]

V = _______________ V

Dunman High School (Senior High) 9646/03/Prelim/2010  


   
For 
17 
Examiner’s
 
Use 

(c) The uncharged, undeflected ink droplets travel beyond the deflecting plates
towards the paper. When a droplet hits the page, it forms a dot sticking to it.
Assuming that the printer giving print of 20 dots per mm would take 0.08 s to print
a line on a page 20.0 cm wide, calculate the average force exerted on the page
when the ink droplets strike. The ink has a density of 1100 kg m-3. [3]

Average Force = _______________ N

Dunman High School (Senior High) 9646/03/Prelim/2010 


    [Turn over
For 
18  Examiner’s
 
Use 

7. Fig 7.1 shows a ring falling from rest through a coil A carrying a constant current.

Fig 7.1

(a) (i) List two factors which will affect the magnitude of e.m.f. induced in the ring.
[2]
1.

2.

(ii) Using the appropriate laws of electromagnetism, state and explain how the
magnitude and direction (as viewed from the top) of the current in the ring
changes throughout the motion in which the ring approaches, falls through
and away from coil A. [6]

Dunman High School (Senior High) 9646/03/Prelim/2010  


   
For 
19 
Examiner’s
 
Use 

(iii) On the separate axis below, sketch graphs to show the variation of
1. The magnetic flux through the ring with position of the ring
2. The induced current in the ring with time [4]

Magnetic flux

Position

Induced current

Time

Dunman High School (Senior High) 9646/03/Prelim/2010 


    [Turn over
For 
20  Examiner’s
 
Use 
(iv) If air resistance is ignored, state and explain whether the ring falls through
the coil A with a constant acceleration. [2]

(b) The constant current source connected to coil is now replaced by a varying e.m.f.
which produces a current in the coil A as shown in Fig 7.2.

Fig 7.2

In addition, coil A is wound on one side of an iron core with coil B on the other
side as shown in Fig 7.3.

Fig 7.3

Dunman High School (Senior High) 9646/03/Prelim/2010  


   
For 
21 
Examiner’s
 
Use 

(i) Sketch a graph to show the variation of magnetic flux produced by the
current in coil A with respect to time. [1]

Magnetic flux

Time

(ii) Sketch a graph to show the variation of e.m.f. induced across coil B with
respect to time. [1]

Induced e.m.f.

Time

Dunman High School (Senior High) 9646/03/Prelim/2010 


    [Turn over
For 
22  Examiner’s
 
Use 

(iii) State and explain how the e.m.f. induced across coil B be affected by
1. the number of turns in coil A [2]

2. the number of turns in coil B [2]

Dunman High School (Senior High) 9646/03/Prelim/2010  


   
For 
23 
Examiner’s
 
Use 

8. Fig 8.1 below shows a simple model of a hydrogen atom in which a single ground
state electron stationary wave fits the radius of an atom. This model suggests that the
electron cannot exist outside this atomic sphere. The atomic radius is 3.0 × 10−10 m.

Electron stationary wave 

3.0 × 10−10

Edge of atom 

Fig 8.1

(a) (i) Explain what the electron stationary wave represents. [1] 

(ii) State what you can infer about the location of the electron in ground
state. [1]

(iii) With reference to stationary waves, explain why you can’t find the
electron within the nucleus even if the electron is in excited state. [2]

Dunman High School (Senior High) 9646/03/Prelim/2010 


    [Turn over
For 
24  Examiner’s
 
Use 
(b) (i) Show that the kinetic energy of this electron at ground state can be
written as

where h is the Planck constant and λ is the wavelength of the electron


stationary wave. [2]

(ii) Hence calculate the kinetic energy of the electron at ground state shown
in Fig 8.1. [3]

Kinetic energy = _______________ eV

(iii) Given that the potential energy of the atom is −9.55 eV at 1.5×10−10m
away from the nucleus, explain if this model leads to a stable atom. [3]

Dunman High School (Senior High) 9646/03/Prelim/2010  


   
For 
25 
Examiner’s
 
Use 
(c) Another model of the hydrogen atom is to consider the electron to be a free
particle inside an ‘atomic box’ as shown in Fig 8.2 below.

Infinite potential at 
the nucleus 
Energy 

Finite potential at 
the edge 

9.55 eV 

3.0 × 10−10 m
−10
0.3 × 10 m
Fig 8.2

(i) Sketch the electron stationary wave for ground state on the figure above. [2]

(ii) Using your sketch above, explain why it is possible for this model of the
hydrogen atom to experience spontaneous ionization. [2]

(iii) Using your answer in (b)(ii) as an approximate, explain if this model of an


atom leads to a stable atom. [3]

(iv) Suggest what could be wrong with this model. [1]

End of Paper

Dunman High School (Senior High) 9646/03/Prelim/2010 


   
Name of Student : Class :

College Section

H2 Physics 9646

C2 Preliminary Examination

Paper 1 Multiple Choice Questions

22 September 2010

Duration of Paper : 1 h 15 min

INSTRUCTIONS TO CANDIDATES

Do Not Open This Booklet Until You Are Told To Do So.

1. Write your name and class at the top of the question paper.

2. Shade your NRIC/FIN number using a 2B pencil on the optical mark sheet (OMS).

3. Check that you have 24 printed pages (including this cover page) for this question
booklet.

4. There are 40 multiple-choice questions in this paper. For each multiple-choice question,
there are four possible answers. Choose the one you consider correct and shade your
answer using a 2B pencil in the boxes provided in the OMS.

5. Answer ALL questions.

6. A data and formula list is provided on page 2.

7. The total marks for this paper is 40 marks.

8. Hand in only your OMS at the end of the test.

This booklet consists of 24 printed pages, inclusive of this page.


2010 HWA CHONG INSTITUTION (COLLEGE SECTION) C2
H2 PHYSICS

Data Formulae

1 2
speed of light in a vacuum, uniformly accelerated motion, s  ut  at
8 -1 2
c = 3.00  10 m s
v 2  u 2  2as
permeability of free space,
-7 -1
o = 4  10 H m work done on/by a gas, W  pV

permittivity of free space, hydrostatic pressure, p  gh


-12 -1
o = 8.85  10 Fm
-9 -1 GM
= (1/(36))  10 F m gravitational potential, 
r
elementary charge,
-19 displacement of particle in s.h.m., x  xo sint
e = 1.60  10 C
velocity of particle in s.h.m., v  v o cost
the Planck constant,
- 34
h = 6.63  10 Js v   x o  x 2
2

unified atomic mass constant,


-27 resistors in series, R  R1  R2  ...
u = 1.66  10 kg
resistors in parallel, 1/ R  1/ R1  1/ R2  ...
rest mass of electron,
-31
me = 9.11  10 kg Q
electric potential, V
4 o r
rest mass of proton,
-27
mp = 1.67  10 kg alternating current / voltage, x  x o sint

molar gas constant, transmission coefficient, T  exp(2kd )


-1 -1
R = 8.31 J K mol
8 2 m(U  E )
where k 
the Avogrado constant, h2
23 -1
NA = 6.02  10 mol
radioactive decay, x  x o exp(t )
the Boltzmann constant,
-23 -1 0.693
k = 1.38  10 JK decay constant, 
t1
2
gravitational constant,
-11 2 -2
G = 6.67  10 N m kg

acceleration of free fall,


-2
g = 9.81 m s

Page 2 of 24
2010 HWA CHONG INSTITUTION (COLLEGE SECTION) C2
H2 PHYSICS

1 Which of the following ratios is wrong?

A The ratio of the size of an atom’s diameter to the diameter of a nucleus is of


the order of 105: 1.
B The ratio of the mass of an electron to the mass of a proton is 1: 1836.
C The ratio of the period of a 100 kg satellite orbiting the Earth at distance R
(from the centre of the earth) to the period of a 200 kg satellite orbiting Earth
at a distance 2R is 1: 2.8.
D The ratio of the size of the energy band gap in an insulator to the band gap in
a semiconductor is of the order of 103: 1.

2 Ball 1 is launched up an inclined plane from point A with an initial speed that is the
minimum speed for it to just reach point B at the top of the plane. At the same moment
that ball 1 is launched up the plane, ball 2 is released from rest from point B. The two
balls make their first contact at a point C somewhere on the inclined plane between A
and B. What is the ratio of the distance AC to the distance BC?

Ball 2

Ball 1 B

A 1
B 2
C 3
D 4

Page 3 of 24
2010 HWA CHONG INSTITUTION (COLLEGE SECTION) C2
H2 PHYSICS

3 A particle of mass m is launched with an initial velocity v from a point P.

o
30
P Figure 3 Q

Neglecting air resistance, what is the magnitude of the change in momentum between
leaving P and arriving at Q?

A zero
1
B mv
2
C mv
D 2mv

Page 4 of 24
2010 HWA CHONG INSTITUTION (COLLEGE SECTION) C2
H2 PHYSICS

4 A ladder is placed between a vertical wall and horizontal ground. The wall and ground
can be either smooth or rough.

Wall

Ladder

Ground

The table below shows you the various combinations of roughness and smoothness
for the two surfaces.

Wall Ground

(i) Smooth Smooth

(ii) Rough Smooth

(iii) Smooth Rough

(iv) Rough Rough

For which of the options below would the ladder be in static equilibrium?

A (iv) only
B (ii) and (iv) only
C (iii) and (iv) only
D (ii), (iii) and (iv) only

Page 5 of 24
2010 HWA CHONG INSTITUTION (COLLEGE SECTION) C2
H2 PHYSICS

5 A fast moving neutron with an initial velocity u has a head-on elastic collision with a
stationary proton. After the collision, the velocity of the neutron is v and that of the
proton is w. Taking the masses of the neutron and proton to be equal, which one of
the following statements is false?

A Since collision is elastic, it shows that u + v = w.

B By considering kinetic energies of the particles, it can be shown that


u2 = v2 + w2.

C The speed of the proton after the collision is the same as that of the neutron
before the collision.

D The proton and the neutron move off in opposite directions with equal speeds.

6 A block and a sphere of equal mass m are placed on an inclined plane. If the
maximum frictional force that can exist between the block and the plane is equal to the
weight of the block, and there is no frictional force between the sphere and the plane,
what is the maximum angle θ at which the plane can be inclined before the block starts
to slip?

A 30o
B 45o
C 60 o
D 90o

Page 6 of 24
2010 HWA CHONG INSTITUTION (COLLEGE SECTION) C2
H2 PHYSICS

7 Mass X slides down a frictionless slope while mass Y falls freely downwards from the
same height. Both blocks are initially at rest and the gradient of the slope is vertical at
the starting position of X.

X Y

Which object takes longer time to fall through the same height H?

A X
B Y
C X or Y, depending on which mass is lighter
D X and Y take the same time

8 Two objects P and Q having the same volume are hung at either ends of a light
uniform rod and subsequently submerged in two different liquids X and Y respectively.
The density of X is less than that of Y. The system is balanced when a string is hung
right at the centre of the rod as shown in figure.

P Q
Liquid X Liquid Y

Which object has a smaller mass?

A P
B Q
C P and Q have the same mass
D Not enough information given

Page 7 of 24
2010 HWA CHONG INSTITUTION (COLLEGE SECTION) C2
H2 PHYSICS

9 Using a rope, delivery man Sam drags a crate of mass 10.0 kg across a rough
horizontal surface at a constant speed of 0.800 m s-1. The rope makes an angle of
30.0  with the horizontal. The average frictional force between the crate and the rough
surface is 200 N.

rope

crate
30.0
10.0 kg

rough horizontal surface

However, Sam realizes that he has to increase his speed if he is to deliver the goods
in the crate on time. Hence, he begins to accelerate uniformly at 1.00 m s-2. The angle
which the rope makes with the crate remains unchanged.
What is the instantaneous power input by Sam on the crate 2.00 seconds after he
starts to accelerate?

A 0W
B 194 W
C 588 W
D 679 W

10 A 50.0 kg man weighs himself using a bathroom scale at the equator as well as on the
pole of planet Earth. What is the difference in the two readings due to the rotation of
the earth? Assume the earth is a sphere with a radius 6400 km.

A 2.64 x 10-7 N
B 0.0338 N
C 1.69 N
D 465 N

Page 8 of 24
2010 HWA CHONG INSTITUTION (COLLEGE SECTION) C2
H2 PHYSICS

11 A simple pendulum is released from rest at A. What is the tension in the string when
the pendulum is at position B (where the string is vertical), given that the mass of the
bob is m and the length of the pendulum is L?
L

A 0.87 mg o
60
B mg A
C 1.27 mg
B
D 2 mg

12 A and B are two spheres with the same mass. They are both placed at a height R
above the surface of Earth. Sphere A is released from rest while sphere B is projected
with a very small tangential velocity v perpendicular to the radial direction of Earth.

B
A
v
R
R

Which of the following best describes the gain in kinetic energy of the two spheres in
their descent to surface of Earth? Neglect frictional forces.

A Identical
B A is larger
C B is larger
D Cannot be deduced

13 A particle performs simple harmonic motion with a maximum acceleration of 5.0 m s -2


and a maximum speed of 3.0 m s-1. What is the frequency of the particle’s oscillation?

A 0.095 Hz
B 0.27 Hz
C 0.60 Hz
D 1.7 Hz

Page 9 of 24
2010 HWA CHONG INSTITUTION (COLLEGE SECTION) C2
H2 PHYSICS

14 A hollow metal cylinder floats upright in a body of water with the bottom of the cylinder
at a depth of D below the water surface as shown in the figure below.

The cylinder is pressed further down into the water and upon release, performs simple
harmonic motion. Which of the following graphs (all drawn to scale) shows how the
upthrust U and net force F acting on the cylinder vary with d, the depth the bottom of
the cylinder below the water surface?

A B
U, F U, F

d d
0 D F 0 D F
U
U

C D U, F
U, F
D F
d d
0 D F 0
U

Page 10 of 24
2010 HWA CHONG INSTITUTION (COLLEGE SECTION) C2
H2 PHYSICS

1
15 n moles of an ideal gas has pressure p and volume V. The graph shows how
p
varies with V at a constant temperature.

1
/105 Pa1
p
0.4

0.3

0.2

0.1

0.0 V / 103 m3
0 1.0 2.0 3.0 4.0

If the number of moles of the gas is increased to 2n and the thermodynamic


temperature is reduced to one quarter of the initial temperature, which graph will be
obtained?

1
/105 Pa1
p
A B
0.4

0.3

0.2 C

0.1 D

0.0 V / 103 m3
0 1.0 2.0 3.0 4.0

Page 11 of 24
2010 HWA CHONG INSTITUTION (COLLEGE SECTION) C2
H2 PHYSICS

16 Container X contains neon gas and container Y contains argon gas. Container X has
twice the volume of container Y. The temperatures of the gases in both containers are
the same. What is the ratio of the mean kinetic energy of a neon molecule to the mean
kinetic energy of an argon molecule? [The relative atomic masses of neon and argon
are 20 and 40 respectively.]

A 0.5
B 1
C 2
D 4

17 A progressive wave travelling to the right hits a hard surface and gets reflected after
suffering a phase change of 180°. The diagram below shows the incident wave at a
particular instant of time.

Which of the following shows the corresponding reflected wave?

Page 12 of 24
2010 HWA CHONG INSTITUTION (COLLEGE SECTION) C2
H2 PHYSICS

18 A point source of sound emits energy equally in all directions at a constant rate. A
detector placed at a distance 8.0 m away registers an amplitude A. After a while, the
intensity of the source is halved. How far from the source must the detector be placed
now so that it can register the same amplitude?

A 2.0 m
B 2.8 m
C 4.0 m
D 5.7 m

19 A guitar string of length L is stretched between two fixed points P and Q and made to
vibrate transversely as shown in the figure.

A
P Q
s B
L

Two particles A and B on the string are separated by a distance s. The maximum
kinetic energies of A and B are KA and KB respectively. Which of the following gives
the correct phase difference and maximum kinetic energies of the particles?

Phase difference Maximum kinetic energy


 3s 
 2L   360
0
A KA < KB
 

 3s 
 2L   360
0
B same
 

C 180o KA < KB

D 180o same

Page 13 of 24
2010 HWA CHONG INSTITUTION (COLLEGE SECTION) C2
H2 PHYSICS

20 A beam of red light of wavelength 710 nm is incident normally on a diffraction grating.


The angular separation between the two second order maxima is 60o. What is the
resolution of the grating?

A 3.5 x 105 lines per millimetre


B 3.5 x 105 lines per metre
C 6.1 x 105 lines per millimetre
D 6.1 x 105 lines per metre

21 A tuning fork of pitch 2.4 kHz is placed near the mouth of a long cylindrical container
which is fully filled with water. Water is slowly drained from the bottom of the container.
Resonance is first heard when the water level had dropped 3.4 cm below the mouth of
the container. At which position of water below the mouth of the container will the next
resonance be heard? Assume the speed of sound in air = 3.4 x 102 m s-1.

A 7.1 cm
B 8.0 cm
C 10.2 cm
D 10.5 cm

22 Two fixed point charges, each of magnitude Q1 = -7.83 nC and Q2 = +4.63 nC, are
located at O and P respectively as shown in the figure. The distance OP is 48.2 cm.
The value of the electric potential at point A is

50.0 cm
33.0 cm

Q1 Q2
48.2 cm
O P

A -14.7 V
B +29.7 V
C +101 V
D -267 V

Page 14 of 24
2010 HWA CHONG INSTITUTION (COLLEGE SECTION) C2
H2 PHYSICS

23 The diagram shows the electric equipotential lines in a non-uniform electric field. At
which position is the electric field greatest in magnitude?

24 An electron is located midway inside the space between horizontal metal plates H and
K. A potential difference of 1000 V is applied across
H and K. + 1000 V
H
H is positively charged while K is earthed. Which one electron x
of the following graphs best represents how the
K
electric force F acting on the electron varies with the
distance x measured from plate H?

A F B F

0 x

0 x
F F
C D

0 x 0 x

Page 15 of 24
2010 HWA CHONG INSTITUTION (COLLEGE SECTION) C2
H2 PHYSICS

25 A potentiometer consists of a 1.000 m long resistance wire XY in series with a battery


of e.m.f. E1 = 9.00 V and internal resistance r = 1.42 Ω. The resistance of XY is 8.30 Ω.
Find the emf of cell E2 if the balance length l is found to be 0.745 m.

E1

X Y

E2 G

R2 = 2.00 Ω

A 0.636 V
B 2.72 V
C 5.73 V
D 9.00 V

Page 16 of 24
2010 HWA CHONG INSTITUTION (COLLEGE SECTION) C2
H2 PHYSICS

26 A thermistor R1 is connected to a battery of constant e.m.f. with negligible internal


resistance as shown in the figure.

R1

M R3
V
P Q

R2 S

Which of the following actions will cause an increase in the potential difference V
measured by the voltmeter? Assume that the voltmeter has infinite resistance.

A Increase the temperature of the thermistor with S open


B Remove the earth connection at M with S open
C Close switch S
D Decrease resistance R3 with S open.

27 A battery with internal resistance r is connected to a resistor R as shown in the figure


below. A constant current passes through R. When a charge of 30.0 C passes through
the circuit, the heat dissipated in r is 45.2 J and the heat dissipated in R is 88.3 J. Find
the e.m.f. of the battery.

A 0.148 V
r
B 1.51 V
C 2.94 V
D 4.45 V

Page 17 of 24
2010 HWA CHONG INSTITUTION (COLLEGE SECTION) C2
H2 PHYSICS

28 A coil PQRS in the shape of a rhombus, has its plane parallel to the magnetic field
lines of a uniform magnetic field, as shown in the figure.

Q
P
Uniform
I I magnetic
field
R
S

Which of the following statements is true when a current I flows in the coil?
A There are no forces on sides SP and QR.
B There are no forces on sides PQ and RS.
C The direction of force on PQ is acting out of the paper.
D The magnitude of force on PQ is smaller than the magnitude of the force on
QR.

29 A uniform metallic rod PQ of density , area of cross-section A and length x, is


suspended horizontally by two identical strings as shown below.

string
string B

P Q

A uniform magnetic field B acts throughout this region in the direction outwards from
the plane of this page and perpendicular to the length of the rod. When a certain
magnitude of current I is passed along the rod, the tensions of the supporting strings
reduce to zero. What is the magnitude and direction of the current in PQ?
Magnitude Direction

A B Q to P
 Ag

B B P to Q
 Ag

C  Ag Q to P
B

D  Ag P to Q
B

Page 18 of 24
2010 HWA CHONG INSTITUTION (COLLEGE SECTION) C2
H2 PHYSICS

30 An e.m.f. is induced in a coil placed in a changing magnetic field. The flux density B of
this field varies with time t as shown below.

At which value of t is the magnitude of the e.m.f. induced in the wire a maximum?

A 1 ms
B 2 ms
C 3 ms
D 4 ms

31 A copper bar of length L is moving to the right with a uniform speed v in a region of
uniform magnetic field of flux density B, directed perpendicularly downwards into the
paper in the figure below.

v
B

The ends of the rods are rigidly connected to a voltmeter which moves with the rod.
What is the reading on the voltmeter?

A Zero
B Non-zero reading less than BLv
C BLv
D More than BLv

Page 19 of 24
2010 HWA CHONG INSTITUTION (COLLEGE SECTION) C2
H2 PHYSICS

32 A 100% efficient transformer is connected as shown to a sinusoidal a.c. supply. What


is the reading on the ammeter?

Primary coil, Secondary coil,


100 turns 20 turns

Output:
A.C. supply 140 Ω
30 V a.c.
A

A 0.0429 A
B 0.214 A
C 1.07 A
D 2.14 A

Page 20 of 24
2010 HWA CHONG INSTITUTION (COLLEGE SECTION) C2
H2 PHYSICS

33 In a photoelectric effect experiment, a metallic surface A in an evacuated tube is


illuminated with light of wavelength 275 nm causing the emission of photo-electrons
which are collected at an adjacent electrode. The variation of photocurrent I with the
potential difference V between the metallic surface and the adjacent electrode is
shown in the diagram below.

A
B
V

The experiment is repeated by replacing metallic surface A by another metallic surface


B.

The table below lists the work functions of some elements.

Element Work Function (eV)

Sodium 2.7

Aluminum 4.3

Copper 4.7

What materials are the metallic surfaces A and B made of?

Metallic Surface A Metallic Surface B


A Copper Copper
B Copper Aluminum
C Sodium Aluminum
D Sodium Copper

Page 21 of 24
2010 HWA CHONG INSTITUTION (COLLEGE SECTION) C2
H2 PHYSICS

34 The x-ray spectrum of a metal target is shown in figure below.

Intensity

 / 10-12 m
36 60 70

Which of the following statements is correct?

A The graph shows that electrons with a range of kinetic energies are used to
bombard the target.
B The position of the peaks allow us to calculate the energy of the electrons
used to bombard the target
C The smallest wavelength detected, 36 x 10-12 m, is dependent on the target
material.
D The locations of the peaks can be used to identify the element that the target
material is made of.

35 An atom X is excited to an energy level E2 from its ground state E0 by collision with
another atom Y. Atom X is initially at rest. Which of the following gives possible energy
values of X and Y?

Kinetic energy of atom Y Kinetic energy of atom X


before collision after the collision
A less than (E2 - Eo) zero
B (E2 - Eo) non-zero
C (E2 - Eo) zero

D greater than (E2 - Eo) non-zero

Page 22 of 24
2010 HWA CHONG INSTITUTION (COLLEGE SECTION) C2
H2 PHYSICS

36 The figure below shows how the atomic energy levels for a material vary with the inter-
atomic separation.

energy

P Q R S
Inter-atomic separation

At which separation does the graph show the material possibly behaving like a
conductor and an ideal gas respectively?

conductor ideal gas


A P R
B P S
C Q R
D Q S

37 A p-n junction is formed when p-type semiconductor is joined to an n-type


semiconductor. Which of the following statements about the p-n junction is false?

A Each of the semiconductor is electrically neutral before joining.


B The p-type semiconductor becomes positively charged after joining.
C In reverse bias condition, the p-n junction can be used to rectify an alternating
current.
D In forward bias condition, the width of the depletion region decreases.

38 It is possible for an electron to annihilate a positron to produce two identical photons.


Given that the mass m of the positron is equal to the mass of the electron, what is the
minimum wavelength  of the photons produced?

h h h 2h
A B C D
mc mc 2 2mc mc 2

Page 23 of 24
2010 HWA CHONG INSTITUTION (COLLEGE SECTION) C2
H2 PHYSICS

39 A sample of a radioactive material contains 1018 atoms. The half life of the material is
2.00 days. Calculate the activity of the sample after 5.00 days.

A 1.77 x 105 disintegrations s-1


B 7.09 x 1011 disintegrations s-1
C 2.05 x 1012 disintegrations s-1
D 6.13 x 1016 disintegrations s-1

40 A radioactive source contains two materials. One has a half life of 4 days and decays
by the emission of alpha particles whilst the other has a half life of 3 days and emits
beta particles. The initial count rate is 160 Bq but when a sheet of paper is placed in
between the source and the detector, the reading drops to 96 Bq. What will be the
count rate after 12 days, without the paper present?

A 10 Bq
B 14 Bq
C 16 Bq
D 20 Bq

----- END OF PAPER -----

Page 24 of 24
College Section

H2 Physics 9646
Preliminary Examination
Paper 2 Structured Questions
3 Sept 2010

Duration of Paper : 1 h 45 min

Name of Student :
INSTRUCTIONS TO CANDIDATES

Do Not Open This Booklet Until You Are Told Class:


To Do So.

1. Write your name, class, tutor’s name and Name of Tutor :


calculator model clearly on this cover page.

2. Check that you have the correct number of Calculator model:


pages for this question booklet.

3. Answer ALL questions.


For Examiner's Use :
4. For numerical answers, all working should be 1 /9
shown.
2 /8
5. You may use a soft pencil for any diagrams,
graphs or rough working. 3 /8
6. Do not use paperclips, highlighters, glue or 4 /7
correction fluid.
5 /8
7. A data and formula list is provided on page 2.

8. You are reminded of the need for clear 6 /8


presentation and good English.
7 / 12

INFORMATION FOR CANDIDATES 8 /12

The number of marks is given in brackets [ ] at the Deductions


end of each question or part question.

Paper 2 /72

This booklet consists of 17 printed pages, inclusive of this page.


2010 HWA CHONG INSTITUTION (COLLEGE SECTION) C2
H2 PHYSICS

Data Formulae

speed of light in free space, uniformly accelerated motion, s = ut + 1 at2


8 -1
c = 3.00  10 m s 2 2
2
v = u + 2as
permeability of free space,
-7 -1
o = 4  10 H m work done on/ by a gas, W = p V

permittivity of free space, hydrostatic pressure p = gh


-12 -1
o = 8.85  10 F m
-9
 (1/(36))  10 F m
-1 gravitational potential,  = -Gm/r

elementary charge, displacement of particle in s.h.m., x = xo sin t


-19
e = 1.60  10 C
velocity of particle in s.h.m., v = vo cos t
the Planck constant, =  ( xo2  x 2 )
- 34
h = 6.63  10 J s
resistors in series, R = R1 + R2 + . . .
unified atomic mass constant,
-27
u = 1.66  10 kg resistors in parallel, 1/R = 1/R1 + 1/R2 + . . .

rest mass of electron, electric potential, V = Q/4or


-31
me = 9.11  10 kg
alternating current / voltage, x = xo sin t
rest mass of proton,
-27 transmission coefficient, T  exp(-2kd)
mp = 1.67  10 kg
2
molar gas constant, 8 m(U  E )
-1 -1 where k  2
R = 8.31 J K mol h

the Avogadro constant,


23 -1
NA = 6.02  10 mol radioactive decay, x = xo exp ( -t )

the Boltzmann constant, 0.693


k
-23
= 1.38  10 J K
-1
decay constant, 
t1
2
gravitational constant,
-11 2 -2
G = 6.67  10 N m kg

acceleration of free fall,


-2
g = 9.81 m s

Page 2 of 17
2010 HWA CHONG INSTITUTION (COLLEGE SECTION) C2
H2 PHYSICS

1. The figure below shows a juggler performing a trick called “the shower” in which three balls are
kept moving around between the two hands and through the air in the trajectory shown in
Figure 1.

Trajectory
of balls.
1.75 m

Ball 1

Ball 3

Right Left
(Throw) (Catch)
Ball 2
Figure 1

(a) Ball 1 leaves the right hand at an angle of 80.0o to the horizontal and reaches a maximum
height of 1.75 m above the level of the juggler’s hands. Show that ball 1’s initial speed of
projection must be 5.95 m s-1.

[2]

(b) Calculate how far the juggler must position his hands apart so that the ball lands on his
left hand.

distance =_______________ m [2]

Page 3 of 17
2010 HWA CHONG INSTITUTION (COLLEGE SECTION) C2
H2 PHYSICS

(c) For a fixed speed of projection, suggest two advantages for the juggler to throw the balls
at such a large angle to the horizontal

Advantage 1:

_______________________________________________________________________

_______________________________________________________________________

_____________________________________________________________________[1]

Advantage 2:

_______________________________________________________________________

_______________________________________________________________________

_____________________________________________________________________[1]

(d) When Ball 1 is just at its maximum height, the juggler throws Ball 2 up with the same
speed and in the same direction as he did Ball 1. How much time does the juggler have to
transfer Ball 3 from his left to right hand so that his left hand is available to catch Ball 1?

time = ______________ s [1]

(e) Suggest a minimum value for the horizontal distance between the two hands and explain
your answer clearly.

________________________________________________________________________

________________________________________________________________________

______________________________________________________________________[2]

Page 4 of 17
2010 HWA CHONG INSTITUTION (COLLEGE SECTION) C2
H2 PHYSICS

2. Daniel decides to have his first attempt at bungee jumping. He falls from rest from the top of a
tall cliff with an elastic rope tied to his feet. The force constant of the rope is 100 N m-1, and the
rope's unstretched length is 20.0 m. Daniel's mass is 80.0 kg. Assume that the average drag
force by the air on Daniel during his jump is 300 N, and that g = 10 m s-2.

cliff H

(a) Determine the lowest height H Daniel reaches in his jump.

H = ____________ m [3]

Page 5 of 17
2010 HWA CHONG INSTITUTION (COLLEGE SECTION) C2
H2 PHYSICS

(b) Calculate the tension in the rope at the instant when Daniel is at his lowest height.

tension = ____________ N [1]

(c) Hence, determine Daniel's acceleration at this instant.

Magnitude of acceleration = ____________ m s-2

Direction of acceleration = _________________ [3]

(d) Sketch a graph of the tension in the rope against the height which Daniel falls through,
from the time when he jumps to the instant when he is at the lowest height
[1]

Tension in
rope

0
Height Daniel falls through

Page 6 of 17
2010 HWA CHONG INSTITUTION (COLLEGE SECTION) C2
H2 PHYSICS

3. A small cube of mass m slides down along a spiraled path round a cone as shown in
Figure 3a. The path is always inclined at an angle  to the horizontal at any point.
There is a smooth wall along the outer edge of the spiraled path to prevent the cube
from falling out of the path (see Figure 3b). This wall is inclined such that it always
exerts a horizontal contact force on the cube as it spirals down. All frictional forces are
negligible.

Inner Wall

Cube
Cone

Spiraled Outer
Path Wall

Figure 3a

Spiral Path
taken by cube

cube

Figure 3b

(a) Label all the forces acting on the cube in Figure 3b. [3]

(b) Based on the answer in (a), describe the motion of the cube.

_____________________________________________________________________

_____________________________________________________________________

_____________________________________________________________________

___________________________________________________________________[2]

Page 7 of 17
2010 HWA CHONG INSTITUTION (COLLEGE SECTION) C2
H2 PHYSICS

(c) (i) State the work done by the horizontal contact force on the cube as it spirals
down the plane.

________________________________________________________________

______________________________________________________________[1]

(ii) Derive an expression for the rate of change of kinetic energy of this cube in
terms of m,  and its instantaneous speed v.

[2]

Page 8 of 17
2010 HWA CHONG INSTITUTION (COLLEGE SECTION) C2
H2 PHYSICS

4(a) Define gravitational potential at a point in a gravitational field.

___________________________________________________________________________

___________________________________________________________________________

_________________________________________________________________________[2]

(b) (i) State the expression of the gravitational potential due to a point mass m at a distance r
from it.

_____________________________________________________________________

___________________________________________________________________[1]

(ii) 4 identical masses, each of mass m, are arranged symmetrically about a light circular
ring of radius R.
m

O P Central axis
h
R Figure 4

Find the gravitational potential V at point P (at distance h from the centre O of the ring
along the central axis of the ring) as shown in Figure 4.

potential = _______________[2]

(iii) Another mass of mass M is placed at P. It is moved along the central axis of the ring
towards the centre O of the ring by an external force F. Assume that the kinetic energy
of the mass remains a constant. Discuss how the work done by F changes with
distance as the mass M approaches O from P.

_____________________________________________________________________

_____________________________________________________________________

_____________________________________________________________________

___________________________________________________________________[2]

Page 9 of 17
2010 HWA CHONG INSTITUTION (COLLEGE SECTION) C2
H2 PHYSICS

5(a) A student attempts to measure the resistivity of soil using two parallel copper plates driven into
the ground as shown in Figure 5.

E
A
K

d = 0.800 m 1.040 m

0.210 m
x = 0.900 m
0.050 m 0.050 m

Figure 5

Each copper plate has a height of 1.040 m, a width of 0.210 m and a thickness of 0.050 m.
Assume the ammeter has zero resistance and the voltmeter has infinite resistance.

The copper plates are driven to a depth of d = 0.800 m and separated by a distance x = 0.900
m. If the soil is acidic, it reacts with copper and this produces an e.m.f. When switch K is open,
the student obtained a steady voltmeter reading of +0.281 V. When switch K is closed, the
student obtained a voltmeter reading of +1.398 V and an ammeter reading of 0.31 mA.

(i) Show that the resistance of the soil between the copper electrodes is 3.6 k. [2]

Page 10 of 17
2010 HWA CHONG INSTITUTION (COLLEGE SECTION) C2
H2 PHYSICS
(ii) Hence, find the resistivity of the soil.

resistivity = ………………………. Ω m [1]

(iii) Suggest how the value in a(ii) could be measured more accurately.

_____________________________________________________________________

_____________________________________________________________________

___________________________________________________________________[1]

(b) A light bulb, marked 60 W, 120 V, is connected to an alternating power supply whose voltage V
in volts is given by V = 170 sin (100  t) where t is time in seconds.

(i) Calculate the value of the root-mean-square current in the bulb.

root-mean-square current =……………… A [2]

(ii) Sketch the graph showing the variation with time t of the power P dissipated by the light
bulb. Indicate the mean power clearly on the graph. [2]

P/W

t/s
0

Page 11 of 17
2010 HWA CHONG INSTITUTION (COLLEGE SECTION) C2
H2 PHYSICS
6. The Scanning Tunneling Microscope (STM) is a stylus-type instrument in which a sharp probe
is scanned across a sample to detect changes in the surface structure on the atomic scale.

(a) Explain using the concept of quantum tunneling, how the STM detects changes in the
topography of the surface.

_____________________________________________________________________

_____________________________________________________________________

_____________________________________________________________________

_____________________________________________________________________

_____________________________________________________________________

_____________________________________________________________________

_____________________________________________________________________

___________________________________________________________________[3]

(b) When the tip of a STM probe is set at a distance d of 1.0 x 10-10 m from the sample, its
transmission coefficient T is 0.0001. The sample has a work function energy of 4.0 eV.
8 2 m(U  E )
The transmission coefficient is given by T  e 2 kd , k  .
h2

(i) Compute k.

k = __________________ m-1 [2]

Page 12 of 17
2010 HWA CHONG INSTITUTION (COLLEGE SECTION) C2
H2 PHYSICS
(ii) Hence, find the value of d when T is 0.0002.

d = ________________ m [2]

(c) When STM is used in biological applications, it is often required to coat DNA complexes
with a conducting film. Suggest why this coating is necessary.

_____________________________________________________________________

_____________________________________________________________________

_____________________________________________________________________

___________________________________________________________________[1]

Page 13 of 17
2010 HWA CHONG INSTITUTION (COLLEGE SECTION) C2
H2 PHYSICS

7. Solids can be classified as crystalline, polymeric or amorphous. All these materials are widely
used in engineering and industry. In the question, we will look at how materials deform when
subjected to loads of varying amounts.

When a load F is applied to the end of a wire of unstretched length l and cross sectional A, it
extends by a length e. Some technical terms used in the subject of elasticity of wires include:

F
 Tensile stress = force per unit area =
A
e
 Tensile strain = extension per unit length =
l
stress Fl
 Young Modulus E = 
strain Ae

(a) Determine the base units of E.

Base units of E = ____________[2]

Page 14 of 17
2010 HWA CHONG INSTITUTION (COLLEGE SECTION) C2
H2 PHYSICS

A specimen fibre of glass has the same dimensions as a specimen of copper wire. The length
of each specimen is 1.60 m and the radius of each is 0.18 mm. Both specimens are loaded
until they break. The force-extension graphs of both specimens are shown in Figure 7.1.

18

16

14
A

12

10
force/N

8
B
6

0
0 2 4 6 8 10 12 14 16 18 20 22 24 26 28 30 32
extension/mm

Figure 7.1

(b) (i) State Hooke’s law.

_____________________________________________________________________

___________________________________________________________________[1]

(ii) Label the limit of proportionality for material A with X. [1]

(iii) Suggest, with a reason, which material is likely to be glass.

_____________________________________________________________________

_____________________________________________________________________

___________________________________________________________________[2]

Page 15 of 17
2010 HWA CHONG INSTITUTION (COLLEGE SECTION) C2
H2 PHYSICS

(c) Using the graphs and data given, determine


(i) the Young modulus of B

Young modulus E = ________________[2]

(ii) the approximate value of work done to stretch material A to its breaking point

Work = _____________ J [2]

Page 16 of 17
2010 HWA CHONG INSTITUTION (COLLEGE SECTION) C2
H2 PHYSICS
(d) In a separate experiment, three rods were set up as shown in Figure 7.2 for the loading test. All
rods are of the same dimensions as in the previous experiment. Rods X and Y are made of
material B and Rod Z is made of material A. Rod Z is at the midpoint of the support rod.

X Y

Support rod

Applied load

Figure 7.2

State, with a reason, which rod(s) will break first as the load increases.

___________________________________________________________________________

___________________________________________________________________________

_________________________________________________________________________[2]

Page 17 of 17
College Section

H2 Physics 9646
Preliminary Examination
Paper 2 Structured Questions
Planning Question
3 Sept 2010

Duration of Paper : 1 h 45 min

Name of student Class

Name of tutor Score 12

INSTRUCTIONS TO CANDIDATES

Do Not Open This Booklet Until You Are Told To Do So.

1. Write your name, class and tutor’s name clearly on this cover page.

2. Check that you have the 4 printed pages inclusive of the cover page for this question
booklet.

3. You may use a soft pencil for any diagrams, graphs or rough working.

4. Do not use paperclips, highlighters, glue or correction fluid.

5. You are reminded of the need for clear presentation and good English.

6. It is recommended that you spend about 25 minutes on this question.

Page 1 of 4
8. A bar magnet is attached to the end of a given spring which is connected to a corkboard. The
arrangement is as shown in Figure 8.1.

Corkboard taped onto brick


Current
carrying coil

brick
x
Coil is taped onto Bar
table surface magnet Screw
Figure 8.1

A current carrying coil is placed near the bar magnet. There is an attractive force on the
magnet which depends on the distance x between the end of the magnet and that of the coil.

Design a laboratory experiment to investigate how this attractive force varies with the distance
x.

You may use any standard equipment which may be found in a school or college science
laboratory together with some or all of the apparatus from the following list.

Ammeter,
Bar magnet,
Coil,
Connecting wires,
Mass holder,
Metre rule,
Oscilloscope,
Power supply units (variable output 0 – 6 V d.c.),
Pulleys,
Resistor,
Rheostat,
Set of masses,
Signal generator,
String,
Voltmeter

Your answer should contain details of

(a) the procedure to be followed including which measurements would be taken

(b) how the attractive force between the magnet and the coil would be measured

(c) how the current in the coil would be monitored

(d) any modifications you would make to the setup shown in Figure 8.1 in order to obtain a
measurable change in x. Diagrams should be clearly labelled.

Page 2 of 4
________________________________________________________________________
________________________________________________________________________
________________________________________________________________________
________________________________________________________________________
________________________________________________________________________
________________________________________________________________________
________________________________________________________________________
________________________________________________________________________
________________________________________________________________________
________________________________________________________________________
________________________________________________________________________
________________________________________________________________________
________________________________________________________________________
________________________________________________________________________
________________________________________________________________________
________________________________________________________________________
________________________________________________________________________
________________________________________________________________________

Page 3 of 4
________________________________________________________________________
________________________________________________________________________
________________________________________________________________________
________________________________________________________________________
________________________________________________________________________
________________________________________________________________________
________________________________________________________________________
________________________________________________________________________
________________________________________________________________________
________________________________________________________________________
________________________________________________________________________
________________________________________________________________________
________________________________________________________________________
________________________________________________________________________
________________________________________________________________________
________________________________________________________________________
________________________________________________________________________
________________________________________________________________________
________________________________________________________________________
________________________________________________________________________
________________________________________________________________________
________________________________________________________________________
________________________________________________________________________
________________________________________________________________________
________________________________________________________________________
________________________________________________________________________
________________________________________________________________________
________________________________________________________________________
________________________________________________________________________
________________________________________________________________________
________________________________________________________________________
________________________________________________________________________
________________________________________________________________________
________________________________________________________________________
________________________________________________________________________

Page 4 of 4
College Section
H2 Physics 9646
Preliminary Examinations
Paper 3
Section A
Longer Structured Questions

C2 14 September 2010 Duration: 2 h

INSTRUCTIONS TO CANDIDATES Name of Student :

Do Not Open This Booklet Until You Are Told To Class:


Do So.

1. Write your name, class, tutor’s name and calculator Name of Tutor :
model clearly on this cover page.

2. Check that you have the correct number of pages for Calculator model:
this question booklet.

3. Answer ALL question in Section A (40 Marks). For Examiner's Use :


Section A
4. The total marks for Section A is 40.
1 /10
5. For numerical answers, all working should be shown.
2 /10
6. You may use a soft pencil for any diagrams, graphs or
rough working 3 /10
7. Do not use paperclips, highlighters, glue or correction
4 /10
fluid.
Section B
8. A data and formula list is provided on page 2. Question No.

9. You are reminded of the need for clear presentation /20


and good English.
/20

INFORMATION FOR CANDIDATES Deductions

The number of marks is given in brackets [ ] at the end of


each question or part question. Paper 3 /80

This booklet consists of 10 printed pages, inclusive of this page.


2010 HWA CHONG INSTITUTION (COLLEGE SECTION) C2
H2 PHYSICS

Data Formulae

speed of light in free space, s = ut + 1 at


2
8 -1 uniformly accelerated motion,
c = 3.00  10 m s 2
2 2
v = u + 2as
permeability of free space,
-7 -1
o = 4  10 H m work done on/ by a gas, W = p V

permittivity of free space, hydrostatic pressure p = gh


-12 -1
o = 8.85  10 Fm
-9 -1 gravitational potential,  = -Gm/r
 (1/(36))  10 F m
displacement of particle in s.h.m., x = xo sin t
elementary charge,
-19
e = 1.60  10 C velocity of particle in s.h.m., v = vo cos t
=  ( xo 2  x 2 )
the Planck constant,
- 34
h = 6.63  10 J s resistors in series, R = R1 + R2 + . . .

unified atomic mass constant, resistors in parallel, 1/R = 1/R1 + 1/R2 + . . .


-27
u = 1.66  10 kg
electric potential, V = Q/4or
rest mass of electron,
-31 alternating current / voltage, x = xo sin t
me = 9.11  10 kg
transmission coefficient, T ∞ exp(-2kd)
rest mass of proton,
-27
mp = 1.67  10 kg 2
8π m(U - E)
where k=
molar gas constant, 2
-1 -1 h
R = 8.31 J K mol

the Avogadro constant, radioactive decay, x = xo exp ( -t )


23 -1
NA = 6.02  10 mol
0.693
the Boltzmann constant, decay constant, λ=
-23 -1 t1
k = 1.38  10 J K
2

gravitational constant,
-11 2 -2
G = 6.67  10 N m kg

acceleration of free fall,


-2
g = 9.81 m s

Page 2 of 10
2010 HWA CHONG INSTITUTION (COLLEGE SECTION) C2
H2 PHYSICS
SECTION A (40 Marks)

Answer all questions

1 Tom was driving his car on the expressway one evening when he spotted a van
travelling towards him at a high speed from a short distance away. The driver in the van
was apparently drunk and not aware that he was driving on the wrong lane.
Unfortunately, Tom was unable to stop his car on time and the two vehicles collided
eventually.

The variation of the velocity of both vehicles from the time Tom saw the oncoming van to
the time after the accident occurred is shown in the graph below.

velocity /km h-1

A B
80

Tom's car

60

40
C

20

E
0
1 2 3 4 5 6 time /s

–20
D

–40

–60 Van

Page 3 of 10
2010 HWA CHONG INSTITUTION (COLLEGE SECTION) C2
H2 PHYSICS

(a) Using information from the graph, briefly describe and explain what happened to [4]
Tom's car during the following periods of time:

AB: __________________________________________________________

__________________________________________________________

BC: __________________________________________________________

__________________________________________________________

CD: __________________________________________________________

__________________________________________________________

DE: __________________________________________________________

__________________________________________________________

(b) The mass of Tom's car is 1200 kg. Determine the average force experienced by
Tom's car during the collision.

Average force = ____________ N [2]

(c) Hence or otherwise, calculate the mass of the van.

Mass of van = ____________ kg [2]

(d) Is the collision between the two vehicles elastic or inelastic? Explain your answer. [2]

________________________________________________________________

________________________________________________________________

________________________________________________________________

Page 4 of 10
2010 HWA CHONG INSTITUTION (COLLEGE SECTION) C2
H2 PHYSICS
2 (a) State
(i) the first law of thermodynamics, [2]

_____________________________________________________________

_____________________________________________________________

_____________________________________________________________

(ii) the meaning of the term internal energy. [2]

_____________________________________________________________

_____________________________________________________________

_____________________________________________________________

(b) The diesel cycle is the thermodynamic cycle which approximates the pressure and
volume of the combustion chamber of the diesel engine, invented by Rudolph
Diesel in 1897.

An ideal gas undergoes the diesel cycle which comprises 4 processes:


Process A - adiabatic compression
Process B - isobaric heating
Process C - adiabatic expansion
Process D - isovolumetric cooling

P / 105 Pa

11
B

A
D

0.020 0.070 V / m3

(i) Calculate the work done by the gas during process B.

work = ___________ J [2]

Page 5 of 10
2010 HWA CHONG INSTITUTION (COLLEGE SECTION) C2
H2 PHYSICS

(ii) Table 2.1 is a table of energy changes during one cycle. Complete the table [4]
with appropriate values.

increase in
work done on heat supplied to
Process internal energy,
gas, W / kJ gas, Q / kJ
ΔU / kJ
A + 201
B + 74

C - 185

D - 35 - 35

Table 2.1

3 A researcher is investigating a cubic crystal with x-rays. The x-rays are incident at an
angle  with the crystal surface and the crystal has a lattice spacing a. He is looking at
reflection from parallel planes of atoms in a thin film deposit of the material, as shown in
Figure 3.1.

Constructive interference occurs when the path difference between radiation reflected
off adjacent layers is an integer product of the wavelength of the radiation.

Incident Reflected
parallel radiation
beams of
radiation

Top-most
layer of atoms
 a
Second layer
of atoms Fig. 3.1

(a) Derive an expression of the path difference, x, between the radiation reflected from
two adjacent layers of atoms, in terms of  and a.

x = ________________ [1]

Page 6 of 10
2010 HWA CHONG INSTITUTION (COLLEGE SECTION) C2
H2 PHYSICS
(b) (i) When the x-rays of wavelength 0.165 nm are used, a strong first order
maximum occurs as the beam makes an angle of  = 23.5° with the top-most
plane. Calculate the lattice spacing a.

a = ______________ m [3]

(ii) The beams indicated in Fig 3.1 depict the conditions for first order maxima. [1]
Sketch, in the same figure, the beams of incident and reflected radiations
corresponding to the second order maxima.

(c) It is suggested that electron beam can be used in place of x-rays to carry out this
diffraction experiment.

(i) Calculate the maximum wavelength of x-rays which can be used to probe the
crystal lattice.

maximum wavelength = ______________ m [2]

(ii) Hence, find the minimum speed which the electrons must possess to be
used for electron diffraction.

mininum speed = _____________ m s-1 [3]

Page 7 of 10
2010 HWA CHONG INSTITUTION (COLLEGE SECTION) C2
H2 PHYSICS

4 In 1932, Cockcroft and Walton produced nuclear disintegrations by bombarding lithium with
the high speed protons. The protons were accelerated through a potential difference of
4.00 x 105 V using a specially built high-voltage machine. Photographs of the reaction taken
in a cloud chamber show that two alpha particles were produced. The tracks were straight
and as their range was equal, the alpha particles have the same initial energy. Using the
length of the tracks, the initial energy of the alpha particles was calculated. The experimental
value of the energy agreed closely to the theoretical value, providing the earliest verification
of Einstein’s mass-energy relation.

The nuclear reaction is given by:


7
3 Li  11 H  2 4
2 He

(a) (i) Explain what is meant by 37 Li .

____________________________________________________________

____________________________________________________________
[1]

(ii) By considering the Coulomb repulsion between the lithium nucleus and the
proton, calculate the distance of closest approach between the lithium
nucleus and the accelerated proton.

Distance = _____________ m [2]

(iii) Suggest why it is possible for the proton to penetrate the Coulomb potential
barrier and interact with the lithium nucleus, producing the alpha particles.

____________________________________________________________
[1]
____________________________________________________________

Page 8 of 10
2010 HWA CHONG INSTITUTION (COLLEGE SECTION) C2
H2 PHYSICS
(b) The masses of the nuclei involved are listed below:

7
3 Li 7.0138 u

4
2 He 4.0015 u

1
1 H 1.0073 u

(i) Ignoring the kinetic energy of the proton, calculate the energy of each alpha particle.



Energy of an alpha particle = ____________ J [3]

(ii) On average, an alpha particle creates 5.0 x 103 ion pairs per mm of track in the
cloud chamber and the energy needed to produce an ion pair is 5.2 x 10-18 J.

1. Calculate the length of the track made by the alpha particle.

Length = __________ mm [1]

2. Sketch the tracks produced by the two alpha particles in figure below.

path of incident
proton
1 7
1 H 3 Li

[1]

Page 9 of 10
2010 HWA CHONG INSTITUTION (COLLEGE SECTION) C2
H2 PHYSICS
(c) In 1934, Fermi began using neutrons instead of protons to produce nuclear
disintegrations. Neutrons are generally more effective than protons or alpha particles for
this purpose. Suggest a reason why this may be so.

[1]
__________________________________________________________________

__________________________________________________________________

- End of Section A -

Page 10 of 10
College Section
H2 Physics 9646
Preliminary Examinations
Paper 3
Section B
Longer Structured Questions

C2 14 September 2010 Duration: 2 h

INSTRUCTIONS TO CANDIDATES Name of Student :

Do Not Open This Booklet Until You Are Told To Class:


Do So.

1. Write your name, class, tutor’s name and calculator Name of Tutor :
model clearly on this cover page.

2. Check that you have the correct number of pages for Calculator model:
this question booklet.

3. Answer ANY TWO questions from Section B and For Examiner's Use :
CIRCLE the questions attempted on this cover Section B
page.
CIRCLE Question No. attempted
4. The total marks for Section B is 40.

5. For numerical answers, all working should be shown. 5 /20

6. You may use a soft pencil for any diagrams, graphs or 6 /20
rough working.
7 /20
7. Do not use paperclips, highlighters, glue or correction
fluid.
Deductions
8. A data and formula list is provided on page 2.

9. You are reminded of the need for clear presentation


and good English.
Section B
INFORMATION FOR CANDIDATES Total
/40

The number of marks is given in brackets [ ] at the end of


each question or part question.

This booklet consists of 13 printed pages, inclusive of this page.


2010 HWA CHONG INSTITUTION (COLLEGE SECTION)
C2
H2 PHYSICS

Data Formulae

speed of light in free space, s = ut + 1 at


2
8 -1 uniformly accelerated motion,
c = 3.00  10 m s 2
2 2
v = u + 2as
permeability of free space,
-7 -1
o = 4  10 H m work done on/ by a gas, W = p V

permittivity of free space, hydrostatic pressure p = gh


-12 -1
o = 8.85  10 Fm
-9 -1 gravitational potential,  = -Gm/r
 (1/(36))  10 F m
displacement of particle in s.h.m., x = xo sin t
elementary charge,
-19
e = 1.60  10 C velocity of particle in s.h.m., v = vo cos t
=  ( xo 2  x 2 )
the Planck constant,
- 34
h = 6.63  10 J s resistors in series, R = R1 + R2 + . . .

unified atomic mass constant, resistors in parallel, 1/R = 1/R1 + 1/R2 + . . .


-27
u = 1.66  10 kg
electric potential, V = Q/4or
rest mass of electron,
-31 alternating current / voltage, x = xo sin t
me = 9.11  10 kg
transmission coefficient, T ∞ exp(-2kd)
rest mass of proton,
-27
mp = 1.67  10 kg 2
8π m(U - E)
where k=
molar gas constant, 2
-1 -1 h
R = 8.31 J K mol

the Avogadro constant, radioactive decay, x = xo exp ( -t )


23 -1
NA = 6.02  10 mol
0.693
the Boltzmann constant, decay constant, λ=
-23 -1 t1
k = 1.38  10 J K
2

gravitational constant,
-11 2 -2
G = 6.67  10 N m kg

acceleration of free fall,


-2
g = 9.81 m s

Page 2 of 14
2010 HWA CHONG INSTITUTION (COLLEGE SECTION)
C2
H2 PHYSICS
SECTION B (40 Marks)

Answer ANY TWO questions.


CIRCLE the question you have chosen on the cover sheet.

5 (a) Speed guns are frequently employed by traffic police to measure the speed of
vehicles for the purpose of speed limit enforcement. A radar speed gun sends a
broad, cone-shaped radiowave and gauge the speed of the target from the change in
frequency of the reflected wave from the moving target. A laser speed gun shoots a
very short pulse of infrared laser light and waits for the reflected laser light to
calculate the distance to the target. By taking many samples per second, the laser
speed gun can find the change in distance between samples and calculate the speed
of the target.

(i) Give one advantage and disadvantage of using the laser speed gun over the [2]
radar speed gun.

Advantage : ___________________________________________________

_____________________________________________________________

_____________________________________________________________

Disadvantage : _________________________________________________

_____________________________________________________________

_____________________________________________________________

(ii) A simple laser speed gun is used to investigate the motion of a small object
performing simple harmonic motion (Fig. 5.1). At its equilibrium position, the
object is 5.000 m away from the laser speed gun.

5.000 m

Fig. 5.1

One section of the data recorded by the laser speed gun is shown in the table
below.

time / ms distance / m speed / m s-1


1000 5.031 31.0
1001 5.059 28.0
1002 5.081 22.0
1003 5.095 14.0
1004 5.100 5.0
1005 5.095 -5.0
1006 5.081 -14.0
1007 5.059 -22.0
1008 5.031 -28.0
1009 Z -

Page 3 of 14
2010 HWA CHONG INSTITUTION (COLLEGE SECTION)
C2
H2 PHYSICS

1. Define simple harmonic motion. [2]

_____________________________________________________________

_____________________________________________________________

_____________________________________________________________

2. Hence, verify that x = Acos(Bt) is a possible solution for an object in SHM, [2]
where A and B are constants.

3. Calculate the amplitude and the period of the simple harmonic motion. [4]

amplitude = ____________ m
period = ____________ s

4. Hence, or otherwise, find the value of Z. [2]

Z = _______________ m

Page 4 of 14
2010 HWA CHONG INSTITUTION (COLLEGE SECTION)
C2
H2 PHYSICS
5. Calculate the number of samples taken per second by the laser speed gun. [3]
Hence explain why a high number of samples taken per unit time is
necessary for the laser speed gun to measure the speed of a moving object.

sampling rate = _____________ s-1

______________________________________________________________

______________________________________________________________

______________________________________________________________

(b) Consider a laser medium whose atoms only have two energy levels. Lasing can
occur when population inversion is achieved between the two energy levels through
optical pumping.

(i) Explain what is optical pumping. [2]

______________________________________________________________

______________________________________________________________

______________________________________________________________

______________________________________________________________

______________________________________________________________

(ii) Explain why it is very difficult to achieve population inversion using optical [3]
pumping in the 2 levels laser medium.

______________________________________________________________

______________________________________________________________

______________________________________________________________

______________________________________________________________

______________________________________________________________

Page 5 of 14
2010 HWA CHONG INSTITUTION (COLLEGE SECTION)
C2
H2 PHYSICS
6 (a) Define magnetic flux density. [2]

___________________________________________________________________

___________________________________________________________________

___________________________________________________________________

___________________________________________________________________

(b) (i) The magnetic flux density B a distance d from a long, straight wire carrying a [2]
current I is given by
o I
B
2d

Using this relationship with a clearly labeled diagram, show that the force per
unit length between two long, straight, parallel wires carrying currents I1 and I2
respectively in the same direction, separated by a distance d is given by

 o I1 I 2
F
2d

(ii) A light metal spring with its axis vertical is clamped at the top and is hanging
freely unextended. The turns of the spring have a radius r and the longitudinal
distance between adjacent turns is d when the spring is unstretched, as shown
in figure 6.1. The spring obeys Hooke’s Law and has a spring constant k.

r
c
d

Figure 6.1 Figure 6.2


10.0 g

Page 6 of 14
2010 HWA CHONG INSTITUTION (COLLEGE SECTION)
C2
H2 PHYSICS
An e.m.f. source is then connected across points A and B, and a current I
passed through exactly two turns of the spring.

1. On Figure 6.2, complete the circuit connections made at A and B so that the [2]
current passing through the spring can be measured.

2. A student noted that the current causes a change in the length of the spring. [3]
State and explain whether the change in the length of the spring is an
extension or compression.
(Assume the connections themselves are light, and do not disturb the
equilibrium of the spring and the force between any two circular turns of the
spring is the same as that between two straight wires.)

______________________________________________________________

______________________________________________________________

______________________________________________________________

______________________________________________________________

______________________________________________________________

______________________________________________________________

______________________________________________________________

______________________________________________________________

______________________________________________________________

______________________________________________________________

3. Using your answer in (b)(i), derive an expression for the change in the length [2]
of the spring, x, caused by a current I passing through points A and B on the
spring, in terms of I, d, k and r.

x = _____________________

4. The connections made across A and B are now reversed. State whether the [1]
change in the length of the spring is an extension or a compression.

______________________________________________________________

Page 7 of 14
2010 HWA CHONG INSTITUTION (COLLEGE SECTION)
C2
H2 PHYSICS
(c) A physicist designs an electromagnetic braking system for a truck in his model
railway. The top view of the set up is shown in the figure below.

Fig 6.3 Top View

The truck has mass M = 0.15 kg. X and Y are wheel axles with an axle spacing of
l = 0.10 m. The truck is projected at velocity v to the right, along a level metal track
with rail spacing w = 0.050 m. Mechanical friction is negligible.
The truck runs into a limited region of vertical magnetic field B = 0.70 T over the
length L = 0.15 m between points P and Q. The field is negligible outside this region.
The truck has metal wheels and axles. Axles X and Y, together with the section of rail
length l between the axles, form a closed loop. The closed loop has a circuit
resistance R of 0.020 .
When the initial velocity v = 30 mm s-1, the braking system works well with the truck
coming to rest before axle X enters the magnetic field at point P.

(i) Explain how the braking occurs. [3]

__________________________________________________________

__________________________________________________________

__________________________________________________________

__________________________________________________________

__________________________________________________________

Page 8 of 14
2010 HWA CHONG INSTITUTION (COLLEGE SECTION)
C2
H2 PHYSICS

(ii) Hence, by considering the net horizontal force, show that the acceleration, a, [3]
experienced by the truck when it enters the magnetic field is proportional to its
velocity v.

(iii) Suggest and explain briefly one improvement to this braking system to [2]
effectively stop trucks projected at high velocities.

______________________________________________________________

______________________________________________________________

______________________________________________________________

______________________________________________________________

______________________________________________________________

______________________________________________________________

_____________________________________________________________

Page 9 of 14
2010 HWA CHONG INSTITUTION (COLLEGE SECTION)
C2
H2 PHYSICS

7 (a) List two similarities and two differences between electric field and magnetic field. [2]

Similarities:

________________________________________________________________

________________________________________________________________

________________________________________________________________

________________________________________________________________

________________________________________________________________

Differences: [2]

________________________________________________________________

________________________________________________________________

________________________________________________________________

________________________________________________________________

________________________________________________________________

Page 10 of 14
2010 HWA CHONG INSTITUTION (COLLEGE SECTION)
C2
H2 PHYSICS
(b) Fig. 7.1 shows a cyclotron, which is used for accelerating charged particles to very
high kinetic energies. It consists of two hollow semi-circular metal chambers, called
‘dees’ of radius 1.30 m, within which there is a uniform magnetic field, B. A high
frequency square wave alternating voltage V and frequency 3.05 MHz is connected
across the narrow gap between the chambers.

Fig 7.1

Fig. 7.2 shows an enlarged view of the narrow gap at the centre of the cyclotron. Positive
charges called deuterons enter the narrow gap near dee Y with negligible kinetic energy
(point 1). They accelerate towards dee X, which has lower potential (point 2).

There is no electric field within the dees and the deuterons move in semi-circular path
inside the dee X. As soon as they complete a semi-circular path (point 3), dee X reverses
its polarity such that dee Y now has a lower potential. The deuterons then accelerate
towards dee Y (point 4).

This process is repeated until the deuterons have gained sufficient energy, eventually
emerging at a very high speed.

High frequency
alternating voltage
3 Particles released here +V
with negligible K.E.
2 X
X Narrow gap of
negligible width -V

1 Y 0V
Y
4

Fig. 7.2

Page 11 of 14
2010 HWA CHONG INSTITUTION (COLLEGE SECTION)
C2
H2 PHYSICS
(i) Explain why the deuterons move in a semicircular path when they are inside [2]
the dees.

______________________________________________________________

______________________________________________________________

______________________________________________________________

(ii) Show that the radius of the circular path of the deuteron motion inside a dee [2]
is given by
mv
r
Bq
where m is the mass of the deuteron, v is its speed and q is its charge and B is
the magnetic flux density inside the chambers.

(iii) Show that the period of the motion is independent of v and r. [2]

(iv) Sketch the graph showing the variation of the kinetic energy of the deuteron [2]
with distance travelled as it moves from Point 2 to Point 4.

K.E.

Distance
Point Point Point
2 3 4

Page 12 of 14
2010 HWA CHONG INSTITUTION (COLLEGE SECTION)
C2
H2 PHYSICS
(v) There is a uniform magnetic field of 0.400 T directed normally out of the plane
of the dees and the high frequency alternating voltage V of amplitude 100 V
and frequency 3.05 MHz is connected between the dees.

The mass of deuteron is 3.34 X 10-27 kg and its charge is +1.60 X 10-19 C.

1. Calculate the gain in kinetic energy of the deuterons in each complete


revolution.

gain in kinetic energy = ____________ J [2]

2. Show why a frequency of 3.05 MHz for the square wave alternating voltage [3]
is appropriate (Fig 7.1).

3. Hence calculate the time taken for a deuteron to attain a kinetic energy of
1.00 MeV.

time = ___________ s [3]

Page 13 of 14
2010 HWA CHONG INSTITUTION (COLLEGE SECTION)
C2
H2 PHYSICS

- End of Paper -

Page 14 of 14
2010 Hwa Chong Institution C2 Preliminary
H2 Physics Worked Solutions

Solution for 2010 H2 Physics Paper 1:

1 D 11 D 21 D 31 A 2. C. Method 1: If the time t = 0 corresponds


2 C 12 A 22 A 32 A to the moment Ball 1 is launched, the
distance travelled by Ball 1,
3 C 13 B 23 B 33 C
4 C 14 A 24 B 34 D ,
where u is the initial speed it was
5 D 15 B 25 C 35 D
launched, and the distance travelled by
6 A 16 B 26 D 36 B
Ball 2 is .
7 A 17 C 27 D 37 B
Therefore, the sum of the distances
8 A 18 D 28 D 38 A travelled, .
9 C 19 C 29 C 39 B The two balls will meet when the sum
10 C 20 B 30 D 40 B equals the distance s that Ball 1
would travel from A to B.

Worked Solutions
1. D. Option A is correct because the ratio of
diameter of atom to nucleus
= 10-10: 10-15

Option B is correct because ratio of


mass of electron to proton
= 9.11x10-31:1.67x10-27

Option C is correct because the period


of orbit depends on the radius of
circular orbit r according to the
4 2 3
equation, T 2  r where M = mass
GM
of planet. It is independent of the mass ,
of the satellite. When r is doubled, T is
3 and .
increased by a factor of 2 2 = 2.8
The ratio is 1.5:0.5 = 3:1.
Option D is wrong because ratio of
band gap for semiconductor to insulator Method 2: Graphical
is 1eV: 10 eV. The band gap for
insulator is at most tens of eV, but not 3. C
1000 eV.

since this is an
60o
equilateral triangle. So the
change in momentum is mv.

1
2010 Hwa Chong Institution C2 Preliminary
H2 Physics Worked Solutions

4. C . Regardless of the nature of the wall, it


will exert a normal contact force 8. A. The freebody diagram for each mass is
(horizontally) on to the ladder. To as shown:
balance it, a horizontal force acting to
the left must be provided by the
Mass P Mass Q
ground. That is only possible if the T1 T2
ground is rough so as to provide a
frictional force on the ladder. UP UQ

5. D. The neutron has the same mass of the


proton to 3 significant figures (1.67 x
WP WQ
10-27 kg). The result of this elastic
collision (same mass of particles, with
one particle initially stationary) is that T = tension in the string suspending the masses
the proton will leave with the speed of U = upthrust acting on each mass
W= weight
the incident neutron, while the neutron
will come to rest.
For rod to be horizontal:
6. A. If the frictional force is , then
T1  T2  T
when the block just starts to
slip: WP  U P  WQ  U Q
WP   X gV  WQ  Y gV
sin
Since  X  Y , then the weight of fluid
sin
displaced by P is less than the weight of
fluid displaced by Y if both masses had
sin = 0.5
the same volume.

 X gV  Y gV
7. A. The magnitude of acceleration of X
along its path decreases as it is Hence,
released. It decreases from g = 9.81 m
s-2 to 0 at the bottom of the slope. On W P  WQ
the other hand the vertical acceleration m P  mQ
of Y is constantly at g = 9.81 m s-2. The
average speed of descent of Y will be 9. C. Along the horizontal direction:
higher than the average speed of
descent of X. Thus X will take a longer cos 30o - 200
time to fall down the same height as Y. 242.49 N

0.800 + (1.00)(2.00)

2.80 m s-2
cos
(210)(2.8)
= 588 W

2
2010 Hwa Chong Institution C2 Preliminary
H2 Physics Worked Solutions

10. C
At pole: Normal contact force,
13. B. Using and

,
= (50)(9.81)

= 490.50 N then .
At equator:
Hz.

14. A. The upthrust U is equal to the weight


of fluid displaced. When d = D, U =
weight of the cylinder. Since the
cylinder has a uniform cross-sectional
= 488.81 N area, U must vary linearly with d. Given
that the motion is simple harmonic, the
Difference is = 1.69 N
net force .

15. B.
11. D

Gain in
= lost in GPE
KE

Gradient of the graph = . Given


that n is doubled and T is ¼ the original
F temperature, then the gradient will
increase by a factor 2.

TB  mg
16. B. The mean kinetic energy of a molecule
is proportional to the temperature of the gas.

12. A. Change in gravitational potential


energy is the same since the change in
distance to the centre of the earth is the
same. Lost in GPE is the same and
hence the gain in KE will be the same
too.

3
2010 Hwa Chong Institution C2 Preliminary
H2 Physics Worked Solutions

17. C. When electromagnetic waves enter a 19. C. The adjacent segments of a stationary
region of different refractive index, the speed wave on a string are in antiphase. Point
and wavelength change. The direction also B has a larger amplitude of vibration
changes (refraction) if the incident angle is than point A. Thus the maximum kinetic
oblique. The frequency remains unchanged, energy that point B can have during the
however. vibration is greater than A.

20. B.

180° phase change

21. D. Wavelength  = v / f
= (3.4 x 102) /(2.4 x 103)
= 0.142 m = 14.2 cm
reflection
Distance between consecutive
resonances = /2
18. D. Intensity of the source is the power
Therefore the next position of
emitted per unit area over which the
resonance = /2 + 3.4
wave energy is distributed. Assuming a
= 7.1 + 3.4
point source emitting in all directions, = 10.5 cm
the area is the spherical surface
22. A. Vnet = V1 + V2
enclosing the source.
(Po = the power of the source). The
intensity is halved at the same position
only if the power of the source is = -14.7 V
halved. Next, the power of the wave is
proportional to the the square of the 23. B. Electric field is numerically equal to the
amplitude of the wave. In order to potential gradient. The closer the
register the same amplitude, the spacing of the equipotential lines, the
intensity measured by the detector stronger the E field.
must still be the original intensity I.
24. B. Electric between parallel plates is
uniform. Hence force F on the electron
is constant.

4
2010 Hwa Chong Institution C2 Preliminary
H2 Physics Worked Solutions

25. C. Total resistance in series with E1 29. C. Using Fleming’s Left hand rule to
= 8.30 + 1.42 = 9.72 Ω. obtain an upward magnetic force so
Pd across XY that the tensions of the strings
= 8.30/9.72 x 9.00 = 7.69 V. become zero, the current has to flow
leftwards i.e. Q to P.
Pd across l
= 0.745/1.000 x 7.69 Since F = BIL,
= 5.73 V. I = F/BL
This is equal to the emf E2.
= mg/BL
26. D. By potential divider principle, voltmeter
reading increase when effective = ALg/BL = Ag/B.
resistance across thermistor is
increased or resistance R3 is reduced. 30. D. Maximum induced e.m.f. occurs at the
greatest rate of change of magnetic
27. D. Energy supplied by emf flux linkage. Since the coil area
= Sum of energy dissipated by R and r remains the same, then this occurs
when the magnetic field B is changing
Emf = W/Q = (88.3+45.2)/30.0 = 4.45
at greatest rate i.e. at the steepest
V
gradient of the graph.
28. D. Current in PS and QR are
31. A. The same p.d. will be induced across
perpendicular to the magnetic field,
the voltmeter, resulting in no net
thus a magnetic force acts on them.
induced emf.
Current in PQ and SR do experience
a component of the magnetic field that 32. A. RMS current in secondary coil
is perpendicular the current, thus will = V/R = 30/140 = 0.214 A
still experience a magnetic force. IS/IP= NP/NS = VP/VS = 100/20 = 5
Component of B IP = 0.214/5 = 0.0429 A
parallel to PQ 33. C. Sodium having the smallest work
B (given) function will release photoelectrons
with greatest kinetic energy. Sodium
Component of B
will require the largest stopping
perpendicular to PQ
voltage. The stopping voltage is the
value of V such that the photocurrent I
The direction of the magnetic force on = 0 (x-intercept).
PQ is directed into the plane of the
paper.

The magnitude of the magnetic force on


PQ will be less than that on QR since
the magnetic field perpendicular to PQ
is a small component of the magnetic
field perpendicular to QR.

5
2010 Hwa Chong Institution C2 Preliminary
H2 Physics Worked Solutions

38. A. The mass m of the electron is equal to


34. D. Option A is not correct because since the mass of the positron. By the law of
the X-ray spectra will always contain conservation of energy, the total energy
bremsstrahlung radiation whether the of the 2 photons is
electrons bombarding the metal target
has a single energy or have a range of 2hc
 2mc 2
energies. Option B is not correct 
because the the position (wavelength) h
corresponding to the characteristic  
mc
peaks is dependent only on the kind of
target and not on the energy of the ln 2
incoming electrons. Option C is not 39. B.    0.3466 day-1
2
correct because minimum wavelength
(cutoff wavelength) depends on the A  A0 e  t
maximum energy of the incident
electrons.   N 0 e  t
 (0.3466)(1018 )e ( 0.3466)( 5)
35. D. In addition to the law of conservation
energy, the law of conservation of
A  6.1263  1016 day-1
momentum must also be fulfilled. The
atom Y cannot have exactly the = 7.09 x 1011 disintegrations s-1
energy required to excite atom X
because this imply that both atoms Y 40. B Initial activity of alpha emitter:
and X will come to rest after the 160-96= 64 Bq
collision and the total momentum
would be zero, inconsistent with the After 12 days (3 half lives) :
non-zero total momentum before Activity = 64 / 8 = 8 Bq
collision. The total momentum would Initial activity of beta emitter : 96 Bq
not be conserved. Therefore atom Y
has to have a kinetic energy greater After 12 days (4 half lives) :
than the excitation energy of X. After Activity = 96/ 16 = 6 Bq
the collision, atom X must move in the
direction that Y was incident on X in Hence, total activity = 8 + 6 = 14 Bq.
order to conserve momentum.

36. B. P has overlapping energy bands which


would make it a conductor. The atoms
in an ideal gas are isolated from one
another and would have discrete
energy levels of R.

37. B. Choice B is false as the p-type


semiconductor becomes negatively
charged due to the diffusion of mobile
electrons from n-type to p-type at the
pn junction.

6
2010 HWA CHONG INSTITUTION (COLLEGE SECTION) C2
H2 PHYSICS

2010 HCI H2 Physics Preliminary Examinations


Suggested solution for Paper 2

Q1
(a) vy2 = uy2 + 2aysy
+
0 = (usin80.0) 2 + 2 (- 9.81) (1.75) [M1]
u = 5.95 m s-1 [C1]

(b) vy = uu + ayt
0 = 5.95 sin80.0o – 9.81(T/2),
where T is time of flight and is twice the time to reach maximum height,
T = 1.1946 s [C1]
Range Sx= 5.95 cos80.0 (1.1946) = 1.23 m. [A1]

(c)
1. Large angle gives a longer time of flight for a given projection speed and hence
juggler has more time to manipulate balls. [B1]
2. Large angle keeps (horizontal) range small and of the order of the natural distance
between the juggler’s hands. [B1]

(d) The left hand has 1.19/2 = 0.597 seconds before ball 1 arrives. [A1]

(e) The minimum distance between the two hands must be at least greater than the diameter
of the balls. [B1]
Otherwise, an upward moving ball will collide with a downward moving ball. [B1]

Q2
(a) Decrease in GPE = Increase in EPE + Work done against drag force [M1]
m g H= ½ k e2 + F H
80.0(10)H = ½ (100)(H – 20.0)2 + 300 (H) [C1]
800 H = 50( H2 – 40H +400) + 300 H
16 H = H2 – 40H + 400 + 6 H
0 = H2 – 50H +400
0 = (H – 40) (H – 10)
H = 40 m , 10 m (rejected) [A1]

(b) T = ke, where e is the extension of the rope


= 100 (40.0 – 20.0)
= 2000 N [A1]

(c) T – mg = ma [M1]
2260 – 80.0(10) = 80.0a
a = 15 m s-2 [A1]
Direction: Vertically upwards [B1]

1
2010 HWA CHONG INSTITUTION (COLLEGE SECTION) C2
H2 PHYSICS

(d) Tension in
rope

0 [B1]
20.0 H Height Daniel falls through
3(a)
Normal contact
force from ground
The direction of the
Horizontal Normal normal contact force from
Contact force from the wall should be drawn
wall into the paper properly into the page
using the right symbol. In
X the event where it is really
hard to draw, a clearly
written statement to
explain the direction is
required.

Weight of cube

Figure 3b

[B1] mark each for each clearly labeled force X 3

(b) Normal horizontal contact force of wall on cube provides the centripetal force for cube
to spiral. [B1]
Component of the weight down the slope causes the acceleration of cube down slope.
[B1]

(ci) Since the horizontal normal contact force is always perpendicular to the direction of
motion, work done is zero. [B1]

(ci) Rate of increase of kinetic energy = rate of decrease of gravitational


potential energy. [M1]
= m g v sin. [A1]

4(a) Gravitational potential at a point in a gravitational field is the work done by external
force to bring a unit mass from infinite to that point [B1]
without a change in kinetic energy. [B1]

Gm
4(bi) V   [A1]
r

4(bii) Distance of any point on the ring to P = R2  h2 [M1]


4Gm
Potential at P, Vp =  [A1]
R2  h2

2
2010 HWA CHONG INSTITUTION (COLLEGE SECTION) C2
H2 PHYSICS

4(biii) Work done W by external force at constant kinetic energy = the change in
gravitational potential energy of the system. [A1]

W required from h to h1
   
= 4G mM    
1 1
= (Vp)M
  R2  h 2   R2  h2 
 1   

As h1 gets smaller, W gets more negative (or W is negative but increase in


magnitude). [A1]

5 (a) (i) V = 1.398 – 0.281 = 1.117 V [M1]


I = 0.31x10-3 A
R = V/I = 3600 Ω [A1]

(ii) R = ρl/A, thus [M1]


ρ = RA/l = 3600 x 0.800 x 0.210 / 0.900
= 670 Ω (2 s.f.) [A1]
(iii) Any one of these: [B1]
 Among all the readings given, the least significant or most
imprecise is the current reading (only 2 s.f.). The current reading
will be subject to significant random errors.

EITHER: Increase the area of the copper plates in the soil. This
will decrease the resistance of the sample of soil to be measured
and increase the current readings for the same voltage applied.

OR use a higher voltage supply so that current will be greater.

 Use a variable voltage supply in place of the battery. Plot the


graph of the voltmeter reading V against the current I. The gradient
equals the resistance R of the soil and the y-intercept the emf due
to the copper plate interacting with the soil. Calculate the resistivity
using the value of resistance R obtained from the gradient of the
graph.

 Vary the area of the copper plates in the soil (use different depths),
find the corresponding R of the soil between the plates using V
and I, and plot the graph of the R against A-1. The gradient is ρl
where l = x. Calculate resistivity ρ as gradient divided by x.

(b) (i) The labels 60 W and 120 V indicate that the normal operating conditions
for the bulb. If a DC voltage of 120 V is applied across the bulb, then the
power dissipated is 60 W. If an alternating voltage is applied, then the
RMS value of the voltage should be 120 V so that the mean power
dissipated is 60 W.

Vrms = 170/√2 = 120 V [M1]


Mean P = IrmsVrms
Irms = Mean P/V rms = 60/ 120 = 0.500 A [A1]

3
2010 HWA CHONG INSTITUTION (COLLEGE SECTION) C2
H2 PHYSICS

(ii)
Compare the given equation with V=V0 sin (2f t).
Therefore 100 = 2f, and f = 50 Hz.
P/W P=V2/R = (V02/R)sin2(2πf t)
[1] – Correct sine-squared [B2]
curve (note P=0 when t=0)
Peak power 120 [1] – All three values
labelled (120W, 60W, T =
0.020 s)
Mean power 60

0
t/s
T = 1/f
= 1/50
= 0.020 s

6 (a) The gap between the STM probe tip and the sample surface acts like [B1]
a potential barrier to the electrons.

When a pd is applied between the probe and the sample, there is a finite [B1]
probability that electrons can tunnel through this potential barrier
due to the wave nature of the electrons even though the electron does
not have sufficient kinetic energy.
The probability of tunneling is T  e 2 kd where d is the width of the [B1]
potential barrier. Thus tunneling current varies exponentially with the
distance of gap d between the probe and the sample surface. As
probe scans across the surface, the distance d changes and the variation
in tunneling current can be detected and used to plot the topography of
the sample surface.

1st mark: identifying potential barrier


2nd mark: identifying electrons as the particles tunneling the barrier
3rd mark: the tunneling current varies exponentially with the width of the
potential barrier which is identified as the distance of the gap.

(b) (i) Work function energy refers to how much additional energy needs to be
provided to the electron for it to cross the potential barrier classically.
This equals to U-E in the equation for k.

Substituting the values,

k
8 2m(U  E )

 
8 2 9.11 1031 (4.0  1.6  1019 )
[M1]
h2
 
2
6.63  1034
[A1]
 1.02  1010

4
2010 HWA CHONG INSTITUTION (COLLEGE SECTION) C2
H2 PHYSICS

(ii) T  e 2kd
T e 2kd
 2kd [C1]
T0 e 0
T
 e 2k ( d do )
T0

0.0002 21.021010 ( d 11010 )


e
0.0001

d = 6.60 x 10-11 m [A1]

(c) DNA complexes are poor electrical conductors. As a result, electrons [A1]
that have tunneled through to the molecule accumulates. This results in a
strong electrostatic field that will disrupt the tunneling process. The
conducting film helps to channel the tunneling current away.

7 (a) Fl F
Units of E = Units of = Units of [M1]
Ae A
-2 -2
= kg m s m
[A1]
= kg m-1 s-2
(b)(i) The extension of the material is proportional to the applied load if the limit [B1]
of proportionality is not exceeded.
(ii) Mark X on (2.0, 12.4) [B1]
(iii) Material B. [A1]
Glass is brittle and is unlikely to undergo extended stretching/plastic
deformation after it reaches its limit of proportionality before its breaking [B1]
point.
(c)(i) Fl (6.0)(1.60) [C1]
E 
Ae (1.01  10 7 )(1.6  10 3 )
[A1]
= 5.89 x 1010 kg m-1 s-2

(ii) Work = Area under the force-extension graph


1 1
= (12)(0.002)  (12  14.4)(0.0036)  14.8(0.0148) [C1]
2 2
[A1]
= 0.278 J

Working must be clearly shown to be given full credit.


Accepted range (0.264, 0.291)
(d) Rod Z will break first. [A1]
Each rod X and Y only experiences half of the force on Z. The maximum
force that rods X and Y can take is more than half of Z, hence Z will reach [B1]
breaking point first.

5
2010 HWA CHONG INSTITUTION (COLLEGE SECTION) C2
H2 PHYSICS

Question 8

Mark Aim
B1 The aim of this experiment is to investigate how the attractive force varies
with the distance x.
(Some students used another quantity instead of force, eg. Extension of
spring. In such cases, students need to elaborate on how this quantity is
related to the attractive force and justify in order to score this mark.)
Preliminary work
The attractive force can be measured by the formula
D1 F = ke where e is the extension of the spring, which is given by e = L – L0,
where L0 is the initial length of the spring when it is not stretched and L is the
length of the spring when it is stretched. The spring constant k can be
measured by
D2 i) suspending the spring vertically from a retort stand. Its length is measured
using a meter rule Lo.
ii) attach a mass of 50g to the bottom of the spring and measure its new
length L.
iii) calculate the spring constant k from k = F/e = (0.050 )(9.81)/(L-L0)
(Calibration curves are acceptable but elaborations on how to get the data
for this curve and how to use it are necessary. Spring balance is also
acceptable as a means to measure the force directly, however, as with scale
instruments, need elaboration on zero error).
Control Variable
B2 The current in the coil is to be kept constant. This can be done by connecting
an ammeter to the coil as shown in the following diagram.
(Good answers include how a rheostat is necessary to allow for adjustments
to the resistance in the circuit as a means of ensuring constant current.
Some students forgot to include a cell in the circuit diagram)
A4
A

A5 The rheostat is first used to vary the resistance in the circuit and hence the
current. The current should be fixed at a value such that when x = 6.0 cm,
the extension in the spring is more than 1.0 cm. This is to ensure that values
of e recorded in the experiment are large enough such that it can be
measured with acceptable percentage error with a meter rule.
(students should elaborate that an acceptable range is one whereby
variations in data are not just realistic, but also measurable, according to the
resolution of the measuring instruments)
B3 More batteries can be connected in series to increase the current until the
resulting change in e is large enough.
A4
A

6
2010 HWA CHONG INSTITUTION (COLLEGE SECTION) C2
H2 PHYSICS

Hypothesis
D3 Suppose the relationship between the attractive force F and the distance x is
expressed in power law,
F = cxn where c and n are constants to be determined.
lg F = n lg x + lg c
If we plot lgF against lgx, then a straight line will indicate that the relationship
is valid and the gradient is n and the y-intercept is lg c.
(some students neglect the interpretation of the gradient and vertical
intercept of the graph)
Procedure
A1 a) Setup the apparatus as shown in Fig 1.1.
b) Use a meter rule to measure the length of the spring when it is
unstretched. Record this as L0.
D4 c) Switch on the circuit to allow current to flow through the coil. Monitor
the ammeter to ensure that current is constant. Ensure that an
attractive force is exerted on the bar magnet by the coil (this occurs
when the spring is stretched). If there is a repulsive force instead,
switch the polarity of the battery in the circuit to reverse the direction
of current flow.
(Its important to include this step in verifying that an attractive force is
present and if not,elaborate on how one can rectify the problem)
A2 d) Measure the distance between the magnet and the coil x. Also
measure the length of the stretched spring and record it as L.
A3 e) Shift the magnet, together with the spring, corkboard and brick, for at
least 5 different values of x. For each x, record the new stretched
length of the spring L. Tabulate the data in the table below,

x /m L /m F =k(L-Lo)/N lg (x/m) lg (F/N)

f) Plot a graph of lgF against lgx

A1-3 Basic Procedure


A4 Labeled Diagrams
A5 Actions taken to set range readings
B1 Correct Dependent and Independent Variables
B2 Valid choice of control variable
B3 Action to improve accuracy of readings
D1 Theory on how to calculate the attractive force
D2 Pre experiment work necessary to measure spring constant
D3 Linearization
D4 Action to ensure correct current flow
(12 marks in total)

7
2010 HCI H2 Physics Preliminary Examinations
Suggested solution for Paper 3

SECTION A
Q1
(a)
AB: Car travels at constant speed. Tom's reaction time.
BC: Car decelerates uniformly. Tom applies car's brakes.
CD: Car's velocity changes from one direction to the opposite direction. Actual collision of the two
vehicles.
DE: Both vehicles stuck together after the collision and slow down to a rest. Friction of road on
the vehicles causes them to stop eventually. [B4]

(b)
dp d (mv)
F 
dt dt
[M1]
 [30  (20)]  1000 
F  1200 
 0.1  3600 
F  1.67 105 N [A1]

(c) N3L: FVC  FCV


mC ( vC 2  vC 1 ) mV ( vV 2  vV 1 )
 [M1]
t t
mV vC 2  vC 1 50
 
mC vV 2  vV 1 30
5
mV  ( 1200 )  2000kg [A1]
3

(d) Inelastic. [B1]


(EITHER)
The 2 vehicles stuck together and move as one after the collision, as observed from the
velocity-time graph.
(OR)
The total KE of the 2 vehicles after the collision is less than the total KE of the 2 vehicles
before the collision. [B1]

Q2

(a)(i) The increase in the internal energy of the system, U, is the sum of the heat supplied to the
system, Q, and the work done on the system, W. [B2]

(a)(ii) Internal energy U of a system is the sum of all microscopic kinetic energies of the
particles and their potential energies. [B2]

(b)(i) Work done by gas


= (0.070 – 0.020) x (11 x 105) [M1]
= 55 kJ [A1]

This booklet consists of 7 printed pages, inclusive of this page.


2010 HWA CHONG INSTITUTION (COLLEGE SECTION) C2
H2 PHYSICS
(b)(ii)
Process W / kJ Q / kJ ΔU / kJ
AQ + CQ correct [1]
A 201 0 201 DW correct [1]
B -55 74 19 BW correct [1] (for –ve)
C -185 0 -185 ABC ∆U correct [1]
D 0 -35 -35

Q3
(a) x = 2 a sin           [B1]

(b)(i) x = n           [M1]


2 a sin  = n 
 a = (1)(0.165 x 10-9)/ 2 sin(23.5o) [M1]
a = 2.07 x 10-10 m [A1]

(b)(ii)
nd
2 order
st
1 order

[M1]

(c)(i) 2 a sin n 


< 2a sinn
 is maximum when  = 90o and n = 1 [M1]
< 4.14 x 10-10 m [A1]

(c)(ii) < 4.14 x 10-10


h/p < 4.14 x 10-10 [M1]
h / (me vmin ) < 4.14 x 10-10
vmin > h / (me)(4.14 x 10-10) [M1]
vmin > 1.76 x 106 m s-1 [A1]

4(a)(i) A nuclide with 3 protons and 4 neutrons. [B1]

(a) (ii) Applying conservation of energy:


Gain in electric potential energy = loss in kinetic energy
Qq
 qV [M1]
4 0 x
3  1.6  10 19
 400000
4 0 x
[A1]
Solving, x = 1.08 x 10-14 m

Page 2 of 7
2010 HWA CHONG INSTITUTION (COLLEGE SECTION) C2
H2 PHYSICS

(a)(ii) With quantum tunneling, the proton can still penetrate the barrier despite [B1]
having an energy that is less than the potential barrier.

(b)(i) Energy released = (7.0138 + 1.0073 – 4.0015 x 2)(1.66 x 10-27)(3 x 108)2 [M1]
= 2.70414 x 10-12 J [A1]

Energy of each alpha = ½ (2.70414 x 10-12) = 1.3521 x 10-12 J [A1]

(b)(ii)1. 1.35211012
Length =  52 mm [A1]
(5.0 103 )(5.2 1018 )

(b)(ii)2. [A1]



(c) Neutrons have no charge and are therefore able to penetrate more deeply B1
into the positively charged nucleus, resulting in higher probability of nuclear
reactions.

SECTION B

Q5
(a)(i) Advantage: precise (can accurately target 1 specific vehicle) or longer range because laser
is unidirectional/small divergence [B1]

Disadvantage: need to aim properly, need to ensure beam is reflected back to the gun [B1]

(a)(ii) 1. Acceleration always directed towards a fixed point [B1]


and proportional to displacement from that point [B1]

(a)(ii) 2. SHM defining equation x  Cx


Let x  A cos(Bt ) , x   AB sin(Bt ) [M1]
x   B A cos( Bt )   B x , where C  B
2 2 2
[A1]

(a)(ii) 3. From table, maximum distance is 5.100 m, hence amplitude = 0.100 m. [B1]
2
Let t = 0 s when time = 1004 ms (laser speed gun time), hence x  0.100 cos( t)
T
[M1
]
When time = 1005 ms, t = 0.001 s, x = 0.095 m, [M1]
2
 0.095  0.100 cos( 0.001)  T  0.0198  0.020 s [A1]
T
2 2
(a)(ii) 4. Using x  0.100 cos( t ) and t = 0.005 s, x  0.100 cos( 0.005)  0.0016 m [M1]
T 0.0198
Hence Z = x + 5.000 = 4.998 m. [A1]

Page 3 of 7
2010 HWA CHONG INSTITUTION (COLLEGE SECTION) C2
H2 PHYSICS
(a)(ii) 5. t  0.001 s  f  1/ t  1000 samples per second [B1]
Laser gun computes the average speed = Δx / Δt [M1]
Δt should as small as possible so that it approximates instantaneous speed. [A1]

(b)(i) Excitation of electrons/atoms from lower to higher energy levels [B1]


with light of correct frequency f, such that the energy difference between the 2 levels is equal
to hf. [B1]

(b)(ii) When using optical pumping for a 2 levels laser, any incoming photon can cause simulated
adsorption as well as stimulated emission [C1]
Initially most of the atoms are in the ground state, the incoming photons will cause more
simulated adsorption than stimulated emission. [M1]
As more atoms become excited, the rate of simulated emission will eventually increase until
it at most equals that of simulated adsorption, when the number of excited atoms is equal to
the number of ground state atoms. Thus it is very difficult to achieve population inversion.
[A1]

6a) The magnetic flux density is defined as the force per unit length per unit current [B1]
acting on an infinitely long current carrying conductor placed perpendicularly to the magnetic
field. [B1]

 o I1 o I 2
6b) (i) B1  B2  [C1]
2d 2d
F21  B1 I 2 L F12  B2 I1 L
F  II F
F  21  B1 I 2  o 1 2  B2 I 1  12 [A1]
L 2d L
d
6b)(ii) 1.

b)(ii) 2. The current in each turn of the coil of the spring produces a magnetic field that is
perpendicular to the current in the adjacent coil. [B1]
Since the current in the adjacent spring coils is flowing in the same direction, by
Fleming’s left hand rule, an attractive force will be exerted on the coils towards each
other. [B1]
The spring coils will move towards each other and the length (vertical) will shorten.
The change in the length of the spring is therefore a compression. [B1]

F o I 2  I2
b)(ii) 3. Force per unit length    F  o (2r ) where r is the radius of the
L 2d 2d
spring coil. [B1]

Since the spring obeys Hooke’s law, F = kx where k is spring constant.

Page 4 of 7
2010 HWA CHONG INSTITUTION (COLLEGE SECTION) C2
H2 PHYSICS
o I 2 o I 2 r
kx  (2r )  x  [B1]
2d dk

b)(ii) 4. Compression. [A1}

6c)(i) According to the Faraday’s law of Electromagnetic Induction, when the metal axle Y of the
truck first enters the magnetic field entry point P with an initial velocity, there will be an
induced electromotive force (emf) across the two metal wheels of Y whose magnitude is
directly proportional to the rate of change of magnetic flux linkage. The wheel on the left (at
the top according to figure) will be at a higher potential. [B1]
Since the axle Y, the two wheels and the railing form a complete loop, induced current will
hence flow. [B1]
Using Fleming’s left hand rule, a magnetic force opposite to the direction of motion would be
produced due to this current flow. This opposing force on the wheels will cause braking to
occur. [B1]

c)(ii) At any instant when axle Y enters magnetic field,


Induced emf E = IR = Bwv
Bwv
Induced current I  [B1]
R
Since FB = - BIw (using the direction of motion as +ve direction)
Bwv
M a  B ( )w [B1]
R
B 2 w2
a  ( )v   v a  v ----------------(1) [B1]
RM

c)(iii) (If the truck’s initial speed is high, the magnetic braking force experienced upon entering and
leaving the magnetic field may not be able to bring the truck to rest completely.)

To improve, we can either have several regions of PQ placed close to each other for
consecutive braking effects [B2]
OR
Have a stronger vertical magnetic field B. [B1]
Any reasonable explanation that leads to a larger acceleration hence braking force eg. with
reference to (1), would be awarded the second mark. [B1]

7(a)
Similarities:
1. Both fields exert forces on moving charged particles. [B1]
2. They are non-contact forces/action-at-a-distance force [B1]

Alternative answers
3. The forces they exert long range forces.
4. They are conservative fields

Differences
1. Electric field will change the magnitude of the speed of charged particles whereas magnetic
field can be used to change the direction of motion of charged particles. [B1]
2. If the paths of charged particles are not straight line path, it will be parabolic and circular
paths in electric and magnetic field respectively. [B1]

Page 5 of 7
2010 HWA CHONG INSTITUTION (COLLEGE SECTION) C2
H2 PHYSICS

Alternative answers
3. The electric force is either parallel or anti-parallel to the Electric field whereas the magnetic
force is always mutually perpendicular to the magnetic field and the motion of charged
particle.
4. There will not be any magnetic force on stationary charged particle whilst there will be
electric force on charged particle regardless of its state of motion.

b(i) Using Fleming’s Left Hand Rule, the magnetic force will be mutually perpendicular to the
velocity and the magnetic field. [B1]
It provides the necessary centripetal force for uniform circular motion. [B1]

(ii)
m v2
FB  C1 [1]
r
m v2
 Bqv C1 [1]
r
mv
 r
Bq

2
(iii) T

mv v Bq
Since r     C1 [1]
Bq r m
2 m
 T C1 [1]
Bq

In the above derivation for the period, period is found to be dependent on mass and
charge of deuteron and B only

(iv)
K.E B1 B1
[2]
.
distance
Point 2 Point 3 Point 4

(v)

1. Gain in KE crossing each gap = e V [C1]


Hence gain in KE per revolution = 2 e V = 2 (100) ( 1.6 x 10-19) = 3.2 x 10-17 J [C1]

2 m Bq
2.  T  f  C1 [1]
Bq 2 m

0.400 (1.6 1019 )


 C1 [1]
2 (3.34 10 27 )

Page 6 of 7
2010 HWA CHONG INSTITUTION (COLLEGE SECTION) C2
H2 PHYSICS

= 3.05  106 Hz [A1]

1.00  106  1.6  1019


3. Number of revolutions to gain 1.00 MeV =
3.2  1017
= 5000 [C1]
2π (3.34  10 27 )
Time taken = 5000 x [C1]
(0.400)(1.60  10 19 )
= 1.63 x 10-3 s [A1]

(Relativistic effects negligible at these speeds)

Page 7 of 7
INNOVA JUNIOR COLLEGE
JC 2 PRELIMINARY EXAMINATION 2
in preparation for General Certificate of Education Advanced Level
Higher 2

CANDIDATE
NAME

CLASS INDEX NUMBER

PHYSICS 9646/01, 9745/01


Paper 1 Multiple Choice 17 September 2010
1 hour 15 minutes
Additional Materials: Multiple Choice Answer Sheet

READ THESE INSTRUCTIONS FIRST

Write in soft pencil.


Do not use staples, paper clips, highlighters, glue or correction fluid.
Write your name, class and index number on the Answer Sheet in the spaces provided
unless this has been done for you.

There are forty questions on this paper. Answer all questions. For each question, there are
four possible answers A, B, C and D.

Choose the one you consider correct and record your choice in soft pencil on the separate
Answer Sheet.

Read the instructions on the Answer Sheet very carefully.

Each correct answer will score one mark. A mark will not be deducted for a wrong answer.
Any rough working should be done in this booklet.

This document consists of 18 printed pages.

Innova Junior College [Turn over


2

Data
speed of light in free space, c = 3.00 x 108 m s-1
permeability of free space, μo = 4π x 10-7 H m-1
permittivity of free space, εo = 8.85 x 10 F m
-12 -1

≈ (1/(36π)) x 10-9 F m-1


elementary charge, e = 1.60 x 10-19 C
the Planck constant, h = 6.63 x 10-34 J s
unified atomic mass constant, u = 1.66 x 10-27 kg
rest mass of electron, me = 9.11 x 10-31 kg
rest mass of proton, mp = 1.67 x 10-27 kg
molar gas constant, R = 8.31 J K-1 mol-1
the Avogadro constant, NA = 6.02 x 1023 mol-1
the Boltzmann constant, k = 1.38 x 10-23 J K-1
gravitational constant, G = 6.67 x 10-11 N m2 kg-2
acceleration of free fall, g = 9.81 m s-2

Formulae
uniformly accelerated motion, s = ut + ½at2
v2 = u2+ 2as
work done on/by a gas, W = p ΔV
3
average kinetic energy of a molecule of an ideal gas U = kT
2
hydrostatic pressure, p = ρgh
GM
gravitational potential, Φ = −
r
displacement of particle in s.h.m. x = xosin ωt
velocity of particle in s.h.m. v = vocos ωt

= ±ω (x o
2
− x2 )
resistors in series, R = R1 + R2 + …
resistors in parallel, 1/R = 1/R1 + 1/R2 + …
electric potential V = Q/4πεor
alternating current/voltage, x = xo sinωt
transmission coefficient T = exp (-2kd)
8π 2m (U − E )
where k =
h2
radioactive decay, x = xo exp(-λt)
0.693
decay constant, λ =

© IJC 2010 9646,9745/Prelim2 [Turn over


3

1 Which pair contains one vector and one scalar quantity?

A displacement and acceleration


B force and kinetic energy
C momentum and velocity
D power and speed

2 A student makes measurements from which she calculates the speed of sound as
327.66 m s-1. She estimates that her result is accurate to ± 3 %.

Which of the following gives her result expressed to the appropriate number of
significant figures?

A 327.7 m s-1 B 328 m s-1 C 330 m s-1 D 300 m s-1

3 An object has an initial velocity u. It is subjected to a constant force F for t seconds,


causing a constant acceleration a. The force is not in the same direction as the initial
velocity.

A vector diagram is drawn to find the final velocity v.

What does vector X in the diagram represent?

A F B Ft C at D u + at

4 A boy throws a ball vertically upwards. It rises to a maximum height, where it is


momentarily at rest, and falls back to his hands.

Which of the following gives the acceleration of the ball at various stages in its motion?
Take vertically upwards as positive. Neglect air resistance.

rising at maximum height falling


-2
A + 9.81 m s 0 - 9.81 m s-2
B - 9.81 m s-2 0 + 9.81 m s-2
C - 9.81 m s-2 0 - 9.81 m s-2
D - 9.81 m s-2 - 9.81 m s-2 - 9.81 m s-2

© IJC 2010 9646,9745/Prelim2 [Turn over


4

5 Which statement about Newton’s laws of motion is correct?

A The first law follows from the second law.


B The third law follows from the second law.
C Conservation of energy is a consequence of the third law.
D Conservation of energy is a consequence of the first law.

6 A tennis ball of mass 100 g is struck by a tennis racket. The velocity of the ball is
changed as shown.

What is the magnitude of the change in momentum of the ball?

A 1.0 kg m s-1 B 5.0 kg m s-1 C 1000 kg m s-1 D 5000 kg m s-1

7 A stationary body explodes into two components of masses m and 2m.

The components gain kinetic energy X and Y respectively.

What is the value of the ratio X ?


Y

A 0.25
B 0.50
C 2.0
D 4.0

© IJC 2010 9646,9745/Prelim2 [Turn over


5

8 A car with front-wheel drive accelerates in the direction shown.

Which diagram best shows the direction of the total force exerted by the road on the
front wheels?

9 A hinged door is held closed in the horizontal position by a cable.

Three forces act on the door. The forces are the weight W of the door, the tension T in
the cable and the force H at the hinge. The diagram is not drawn to scale.

Which list gives the three forces in increasing order of magnitude?

A W, H, T B W, T, H C H, T, W D T, H, W

10 Two springs P and Q both obey Hooke’s Law. They have spring constants 2k and k
respectively.

The springs are stretched, separately by the same force. The elastic potential energies
stored in spring P and Q are WP and WQ respectively.

How is WP related to WQ?

A WP = ¼ WQ B WP = ½ WQ C WP = 2 WQ D WP = 4 WQ

© IJC 2010 9646,9745/Prelim2 [Turn over


6

11 A ball of mass 0.10 kg is attached to a string and swung in a vertical circle of radius
0.50 m. Its speed at the top of the circle is 6.0 m s-1.

6.0 m s-1

0.50 m

What is the tension in the string at this moment?

A 0.22 N B 6.2 N C 7.2 N D 8.2 N

12 Which diagram shows the variation of gravitational force F on a point mass, and the
gravitational potential energy U of the mass, with its distance r from another point mass?

A B

C D

13 The escape speed of an oxygen molecule at the Earth’s surface is 1.1 × 104 m s-1. What
is the escape speed at a height 0.2 RE above the Earth’s surface, where RE is the radius
of the Earth?

A 0.5 × 104 m s-1


B 1.0 × 104 m s-1
C 1.2 × 104 m s-1
D 2.5 × 104 m s-1

© IJC 2010 9646,9745/Prelim2 [Turn over


7

14 A trolley of mass 2.0 kg with free-running wheels is attached to two fixed points P and Q
by two springs under tension as shown.

The trolley is displaced a small distance of 5.0 cm towards Q by a resultant force of


10 N and is then released. The equation of the subsequent motion is a = -ω 2 x, where x
is the displacement from the equilibrium position.

What is the constant ω2?

A - 10 rad2 s-2 B - 100 rad2 s-2 C 10 rad2 s-2 D 100 rad2 s-2

15 A particle of mass 4.0 kg moves with a simple harmonic motion and its potential energy
U varies with position x as shown.

What is the period of oscillation of the mass?

π 2 8π 4π 2π 2
A s B s C s D s
5 25 5 5

16 The temperature of a body at 100 °C is increased by Δθ as measured on the Celsius


scale.

How is this temperature change expressed on the Kelvin scale?

A Δθ
B Δθ + 273
C Δθ + 273.15
D Δθ + 273.16

© IJC 2010 9646,9745/Prelim2 [Turn over


8

17 A gas undergoes the cycle of pressure and volume changes W → X → Y → Z → W as


shown in the diagram.

What is the net work done on the gas?

A - 600 J B - 200 J C 0J D 200 J

18 A displacement-time graph for a transverse wave is shown in the diagram.

The phase difference between X and Y can be expressed as nπ.

What is the value of n?

A 1.0 B 1.5 C 2.0 D 3.0

© IJC 2010 9646,9745/Prelim2 [Turn over


9

19 A light meter measures the intensity I of the light falling on it. Theory suggests that this
varies with the inverse of the square of the distance d.

Which graph of the results support this theory?

20 A stationary wave has a series of nodes. The distance between the first and the sixth
node is 30.0 cm.

What is the wavelength of the sound wave?

A 5.0 cm B 6.0 cm C 10.0 cm D 12.0 cm

© IJC 2010 9646,9745/Prelim2 [Turn over


10

21 In an interference experiment, two slits are illuminated with white light.

What is seen on the screen?

A The central fringe is black with black and white fringes on each side.
B The central fringe is black with coloured continuous spectrum on each side.
C The central fringe is white with black and white fringes on each side.
D The central fringe is white with coloured continuous spectrum on each side.

22 A narrow beam of monochromatic light is incident normally on a diffraction grating.


Third-order diffracted beams are formed at angles of 40° to the original direction.

What is the highest order of diffracted beam produced by this grating?

A 3rd B 4th C 5th D 6th

23 Which diagram represents the electric field of a negative point charge?

© IJC 2010 9646,9745/Prelim2 [Turn over


11

24 An electric field exists in the space between two parallel charged metal plates.

Which of the following graphs shows the variation of electric field strength E with
distance d from X along the line XY?

25 The current in the circuit is 4.8 A.

What is the rate of flow and the direction of flow of electrons through the resistor R?

rate of flow direction of flow


A 3.0 × 1019 s-1 X to Y
B 6.0 × 10 s
18 -1 X to Y
C 3.0 × 1019 s-1 Y to X
D 6.0 × 1018 s-1 Y to X

26 The resistivity of aluminum is 2.0 times that of silver. An aluminium wire of length L and
diameter d has a resistance R.

What is the diameter of the silver wire, also of length L and resistance R?

A 0.05 d B 0.71 d C 1.4 d D 2.0 d

© IJC 2010 9646,9745/Prelim2 [Turn over


12

27 When four identical lamps P, Q, R and S are connected as shown in diagram 1, they
have normal brightness.

The four lamps and the battery are then connected as shown in diagram 2.

Which statement is correct?

A The lamps do not light up.


B The lamps are less bright than normal.
C The lamps have normal brightness.
D The lamps are brighter than normal.

28 In the circuit, the battery has an e.m.f. of 12 V and an internal resistance of 3.0 Ω. The
ammeter has negligible resistance.

The switch is closed.

What is the reading on the ammeter?

A 0.50 A B 1.0 A C 1.3 A D 2.0 A

© IJC 2010 9646,9745/Prelim2 [Turn over


13

29 A long straight wire XY lies in the same plane as a square loop of wire PQRS which is
free to move. The sides PS and QR are initially parallel to XY. The wire and loop carry
steady currents as shown in the diagram.

What will be the effect of the loop?

A It will move towards the long wire.


B It will move away from the long wire.
C It will rotate about an axis parallel to XY.
D It will be unaffected.

30 A coil has area A and n turns.

A uniform magnetic field of flux density B acts at an angle θ to the plane of the coil, as
shown.

What is the decrease in magnetic flux linkage when the coil rotates so that angle θ is
reduced to zero?

A BAn cosθ B BAn sinθ C 2BAn cosθ D 2BAn sinθ

© IJC 2010 9646,9745/Prelim2 [Turn over


14

31 An alumimium rod moves at right angles to a uniform magnetic field as shown in


diagram 1. Diagram 2 shows the variation with time t of the distance s from point O.

Which graph best shows the variation with t of the e.m.f E induced in the rod?

A B

C D

32 A sinusoidal potential difference V1 shown in graph 1 produces heat at a mean rate W


when applied across a resistor R

graph 1 graph 2

What is the mean rate of production of heat when the square-wave potential difference
V2 shown in graph 2 is applied across the resistor?

W W
A B C 2W D 2W
2 2

© IJC 2010 9646,9745/Prelim2 [Turn over


15

33 A transformer has NP turns in its primary coil and Ns turns in the secondary coil. The
alternating voltage and current in the primary coil are VP and IP respectively. The
alternating voltage and current in the secondary coil are VS and IS respectively.

Which one of the following relations must be correct?

NS VS I S
A = =
NP VP I P
NS VS I P
B = =
NP VP I S
NS VS
C =
NP VP
NS I P
D =
NP I S

34 In a photoelectric emission experiment on a certain metal surface, two quantities, when


plotted as a graph of y against x, give a straight line passing through the origin.

Which of the following correctly identifies x and y with the photoelectric quantities?

x y
A photocurrent threshold frequency
B frequency of incident light maximum kinetic energy of photoelectrons
C light intensity photocurrent
D light intensity maximum kinetic energy of photoelectrons

© IJC 2010 9646,9745/Prelim2 [Turn over


16

35 The diagram shows the energy levels for an atom, drawn to scale. The electron
transitions give rise to the emission of a spectrum of lines of λ1, λ2, λ3, λ4 and λ5.

What can be deduced from this diagram?

A λ1 > λ2
B λ3 = λ4 + λ5
C λ4 is the shortest of the five wavelengths.
D The transition corresponding to wavelength λ3 represents the ionisation of the atom.

© IJC 2010 9646,9745/Prelim2 [Turn over


17

36 The sketch graph shows how the wave function ψ of an electron varies with position.

Which graph, drawn on the same horizontal scale, gives the probability of finding an
electron at each position?

© IJC 2010 9646,9745/Prelim2 [Turn over


18

37 A proton has a kinetic energy of 1.00 MeV. If its momentum is measured with an
uncertainty of 1.00 %, what is the minimum uncertainty in its position?

A 9.08 × 10-13 m
B 2.28 × 10-13 m
C 9.08 × 10-14 m
D 5.64 × 10-14 m

38 The diagram shows the energy band structure of a typical semiconductor.

Conduction band
Energy gap

Filled valence band


Energy E

What is a typical value for the energy gap?

A 0.10 eV B 1.0 eV C 10 eV D 100 eV

10
39 When an isotope of boron, 5 B captures a slow neutron, it splits into a lithium 73 Li and an
alpha particle. An emission of γ-ray occurs during this reaction. The nuclear binding
energies are
10
5B
: 64.94 MeV
7
3 Li : 39.25 MeV
4
2 He : 28.48 MeV

4
If the total kinetic energy of the products produced, 73 Li and 2 He is 2.31 MeV, the
energy of the γ-ray emitted is

A 0.48 MeV B 2.79 MeV C 10.77 MeV D 25.69 MeV

40 The half-life of a certain radioactive material is 3.0 s. How long does it take for its activity
to reduce by 90%?

A 0.46 s B 5.4 s C 10 s D 11 s

END OF PAPER

© IJC 2010 9646,9745/Prelim2 [Turn over


INNOVA JUNIOR COLLEGE
JC 2 PRELIMINARY EXAMINATION 2
in preparation for General Certificate of Education Advanced Level
Higher 2

CANDIDATE
NAME

CLASS INDEX NUMBER

PHYSICS 9646/02
Paper 2 Structured Questions 17 September 2010
1 hour 45 minutes
Candidates answer on the Question Paper
No Additional Materials are required.

READ THESE INSTRUCTIONS FIRST

Write your name, class and index number on all the work you For Examiner’s Use
hand in.
Write in dark blue or black pen on both sides of the paper. 1
6
You may use a soft pencil for any diagrams, graphs or rough
working. 2
Do not use staples, paper clips, highlighters, glue or correction 7
fluid.
3
8
For Section A
Answer all questions. 4
6
It is recommended that you spend about 1 hour and 15
minutes on this section. 5
7
For Section B 6
Answer Question 8. 8
It is recommended that you spend about 30 minutes on this
section. 7
18

At the end of the examination, fasten all your work securely 8


12
together.
The number of marks is given in the brackets [ ] at the end of
each question or part question. Total
72

This document consists of 21 printed pages.

© IJC 2010 9646/Prelim2 [Turn over


Innova Junior College [Turn over
2
For
Data Examiner’s
Use
speed of light in free space, c = 3.00 x 108 m s-1
permeability of free space, μo = 4π x 10-7 H m-1
permittivity of free space, εo = 8.85 x 10 F m
-12 -1

≈ (1/(36π)) x 10-9 F m-1


elementary charge, e = 1.60 x 10-19 C
the Planck constant, h = 6.63 x 10-34 J s
unified atomic mass constant, u = 1.66 x 10-27 kg
rest mass of electron, me = 9.11 x 10-31 kg
rest mass of proton, mp = 1.67 x 10-27 kg
molar gas constant, R = 8.31 J K-1 mol-1
the Avogadro constant, NA = 6.02 x 1023 mol-1
the Boltzmann constant, k = 1.38 x 10-23 J K-1
gravitational constant, G = 6.67 x 10-11 N m2 kg-2
acceleration of free fall, g = 9.81 m s-2

Formulae
uniformly accelerated motion, s = ut + ½at2
v2 = u2+ 2as
work done on/by a gas, W = p ΔV
3
average kinetic energy of a molecule of an ideal gas U = kT
2
hydrostatic pressure, p = ρgh
GM
gravitational potential, Φ = −
r
displacement of particle in s.h.m. x = xosin ωt
velocity of particle in s.h.m. v = vocos ωt

= ±ω (x o
2
− x2 )
resistors in series, R = R1 + R2 + …
resistors in parallel, 1/R = 1/R1 + 1/R2 + …
electric potential V = Q/4πεor
alternating current/voltage, x = xo sinωt
transmission coefficient T = exp (-2kd)
8π 2m (U − E )
where k =
h2
radioactive decay, x = xo exp(-λt)
0.693
decay constant, λ =

© IJC 2010 9646/Prelim2 [Turn over


3
Section A For
Examiner’s
Use
Answer all the questions in this section.

1 (a) Complete Fig. 1.1 to show each quantity and its base units. [2]

quantity base units


speed m s-1
density kg m-3
…………………… kg m s-1
electric field strength ……………………

Fig. 1.1

(b) In the classroom, a student wishes to determine the mass of a plastic semi-circular
protractor.

Fig. 1.2

(i) Give a reasoned estimate of the mass of the semi-circular protractor and express
your answer in an SI unit.

mass = ………………………… unit ……………. [2]

(ii) State an instrument which is most appropriate for the measurement of the
thickness of the protractor x, as indicated in Fig. 1.2.

……………………………………………………………………………..…..…….….. [1]

(iii) For the measurement of x, suggest a way to reduce random errors.

…..…………………………………………………………………………..……………….

……………………………………………………………………………..…..…….….. [1]

© IJC 2010 9646/Prelim2 [Turn over


4
2 ‘Clay pigeon shooting’ is a sport whereby the shooter aims and hits the clay disc For
Examiner’s
projected by a launcher. A certain clay disc is launched from the horizontal ground with a Use
o
velocity of 20 m s-1 at an angle of 30 to the horizontal.

(a) Assuming that air resistance can be neglected, calculate

(i) the maximum height of the disc,

maximum height = ………………………… m [2]

(ii) the horizontal distance between the point from which the disc is launched and
where it lands on the ground.

horizontal distance = ………………………… m [2]

(b) The path N for the above disc is as given in Fig. 2.1 where air resistance is neglected.

20 m s-1
o
30

Fig. 2.1

(i) Draw the path of the clay disc on Fig. 2.1, assuming that air resistance cannot be
neglected. Label this path A. [1]

(ii) Suggest an explanation for any differences between the two paths N and A.

.………………………………………………………………………………………………

.………………………………………………………………………………………………

.………………………………………………………………………………………………

.…………………………………..……………………………………………………… [2]

© IJC 2010 9646/Prelim2 [Turn over


5
3 Fig. 3.1 shows a binary star system where two identical stars each of mass 4.0×1030 kg For
Examiner’s
are moving with a constant speed v in a circular orbit of radius 1.0×1011 m about their Use
common centre of mass.

star

star 1.0×1011 m

Fig. 3.1

You may assume that each star to be a uniform sphere with its mass concentrated at
the centre of the sphere.

(a) (i) On Fig. 3.1, label with the letter ‘X’ a point where the gravitational field strength is
zero. [1]

(ii) Explain why you have chosen the point in (i).

………………………………………………………………………………………………

..………………………………..………………………………………………………. [1]

(iii) Calculate the gravitational potential at point X.

gravitational potential = ………………………… J kg-1 [2]

(b) For each star in the system, calculate

(i) the net force,

net force = ………………………… N [2]

© IJC 2010 9646/Prelim2 [Turn over


6
(ii) the linear speed v. For
Examiner’s
Use

linear speed = ………………………… m s-1 [2]

© IJC 2010 9646/Prelim2 [Turn over


7
4 (a) Define For
Examiner’s
Use
(i) magnetic flux density,

………………………………………………………………………………………………

………………………………………………………………………………………………

...………………………………..………………………………………………………. [1]

(ii) the tesla

………………………………………………………………………………………………

………………………………………………………………………………………………

...………………………………..………………………………………………………. [1]

(b) Fig. 4.1 shows an electric motor which is made up of a rectangular coil of wire of 150
turns. The coil is 0.20 m long and 0.12 m wide. The coil has a current of 0.32 A
flowing through it and its plane is parallel to a field of magnetic flux density 0.36 T.

magnetic field
0.12 m
Y

0.20 m

0.32A

Fig. 4.1

(i) Draw arrows on Fig. 4.1 to represent the directions of the magnetic forces acting
on the coil. Label them F. [1]

(ii) Calculate the magnitude of the magnetic force acting on one side of the coil.

force = ………………………… N [2]

© IJC 2010 9646/Prelim2 [Turn over


8
(iii) Calculate the torque which is exerted on the coil. For
Examiner’s
Use

torque = ………………………… N m [1]

© IJC 2010 9646/Prelim2 [Turn over


9
5 A student sets up the apparatus shown in Fig. 5.1 to demonstrate a two slit interference For
Examiner’s
pattern on the screen. The set-up is modelled after Young’s double slit experiment. The Use
slits S1 and S2 are of the same width.

single double screen


slit slit

S1
laser So d
beam S2

Fig. 5.1

(a) Explain why the single slit So is not necessary in this particular set-up.

.…….……………………………………………………………………………………………

..……………………………….……………………………………………………………. [1]

(b) The laser beam has a wavelength of 630 nm. The separation d is 1.0 mm and the
distance D is 2.5 m. Determine the separation of the fringes on the screen.

separation = ………………………… m [2]

(c) Describe and explain what would be observed on the screen if the slit S1 is made
narrower by half compared to slit S2 while maintaining the same separation d.

.…….……………………………………………………………………………………………

.…….……………………………………………………………………………………………

.…….……………………………………………………………………………………………

..……………………………….……………………………………………………………. [2]

© IJC 2010 9646/Prelim2 [Turn over


10
(d) Explain how the result of this experiment provided evidence that light must have For
Examiner’s
wave properties. Use

.…….……………………………………………………………………………………………

.…….……………………………………………………………………………………………

.…….……………………………………………………………………………………………

………………………………….……………………………………………………………. [2]

© IJC 2010 9646/Prelim2 [Turn over


11
6 (a) (i) Electromagnetic radiation has a wave nature as well as a particulate nature. This For
Examiner’s
is known as wave-particle duality. Describe a situation in which particles can be Use
shown to have a wave nature.

………………………………………………………………………………………………

………………………………………………………………………………………………

………………………………………………………………………………………………

………………………………………………………………………………………………

………………………………………………………………………………………………

…………………………………..………………………………………………………. [3]

(ii) Calculate the wavelength of a particle of mass 1.82 x 10-28 kg when travelling with
a speed which equals to 10% of the speed of light.

wavelength = ………………………… m [2]

(iii) Electromagnetic radiation can also be considered as a transverse wave and


hence it also exhibits wave-like behaviour such as polarisation. Explain why
polarisation is a phenomenon of transverse waves but not longitudinal waves.

………………………………………………………………………………………………

..………………………………..………………………………………………………. [1]

© IJC 2010 9646/Prelim2 [Turn over


12
(b) Fig 6.1 illustrates a phenomenon known as the Compton effect, whereby an incident For
Examiner’s
X-ray photon is scattered by an electron at rest. The wavelength of the scattered Use
photon λ’ is found to be longer than the wavelength λ of the incident photon.

electron after
collision
incident
photon (λ)
θ

electron at φ
rest
Scattered
photon (λ’)

Fig. 6.1

Using de-Broglie’s relation, suggest how this phenomenon demonstrates the


particulate nature of electromagnetic radiation.

.…….……………………………………………………………………………………………

.…….……………………………………………………………………………………………

.…….……………………………………………………………………………………………

………………………………….……………………………………………………………. [2]

© IJC 2010 9646/Prelim2 [Turn over


13
7 Multi-bladed low-speed wind turbines (windmills) similar to the one shown in Fig. 7.1 For
Examiner’s
have been used since 1870, particularly for pumping water on farms. Use

Fig. 7.1

The turbine blades cover almost the whole surface of the wheel and a tail vane behind
the windmill keeps the wheel facing the wind. The diameters of the wheel of windmills of
this type vary from 2 m to a practical maximum of about 12 m. Because of this size
limitation, they are not suited to large power outputs. They will start freely with wind
speeds as low as 2 m s-1 and, at these low speeds, can produce large torques.

© IJC 2010 9646/Prelim2 [Turn over


14
Fig. 7.2 shows how P, the output power of these windmills, varies with the overall For
Examiner’s
diameter of the wheel for different wind speeds, v. Use

v = 10 m s-1 v = 9.0 m s-1

v = 8.0 m s-1
Power P / W

v = 7.0 m s-1 v = 6.0 m s-1

v = 5.0 m s-1

v = 4.0 m s-1

v = 3.0 m s-1

v = 2.0 m s-1

2.0 4.0 6.0 8.0 10.0


diameter / m
Fig. 7.2

© IJC 2010 9646/Prelim2 [Turn over


15
(a) It is thought that, for a given diameter, the output power is related to the wind speed For
Examiner’s
by the equation Use
P = k v n,
where n and k are constants.

Explain how the relation may be tested by plotting a suitable graph.

………………………………….…………………………….…………………………………

…....……………………………….……………………………………………………………

………………………………….…………………………….…………………………………

.…….……………………………………………………………………………………………

………………………………….……………………………………………………………. [3]

(b) (i) Use Fig. 7.2 to determine lg (P / W) for a particular multi-bladed low-speed
windmill with a wheel of diameter 6.0 m and wind speed 3.0 m s-1.

lg (P / W) = ………………………… [1]

(ii) On Fig. 7.3,


1. plot the point corresponding to a wheel diameter of 6.0 m and a wind speed of
3.0 m s-1, [1]

2. draw the line of best fit for the points. [1]


lg (P / W)

x
3.5
x

x
3.0

x
2.5

2.0

1.5

1.0
0.20 0.30 0.40 0.50 0.60 0.70 0.80 0.90 1.00
-1
Fig. 7.3 lg (v / m s )

© IJC 2010 9646/Prelim2 [Turn over


16
(iii) Use the line drawn in (b)(ii) to determine the magnitudes of the constants n and k For
Examiner’s
in the expression in (a). Use

n = …………………………

k = ………………………… [3]

(c) (i) When the wind speed is 8.0 m s-1, calculate the volume of air that reaches the
6.0 m diameter wheel of the windmill in one second.

volume = ………………………… m3 [1]

(ii) The density of air is 1.3 kg m-3. Calculate the kinetic energy of the volume of
moving air in (c)(i).

kinetic energy = ………………………… J [2]

(d) Use your answer in (c)(ii), together with data from Fig. 7.2 to find the fraction of this
power converted into useful output power.

fraction of useful output power = ………………………… [1]

© IJC 2010 9646/Prelim2 [Turn over


17
(e) State three factors, other than wind speed and diameter of wheel, that are likely to For
Examiner’s
influence the output power. In each case, indicate how the power output is likely to Use
be affected.

1. ……………………………….………………………………………………………….……

….……………………………….……………………………………………………………

2. ………………….……………….……………………………………………………………

…….…………………………….……………………………………………………………

3. …….…….……………………………………………………………………………………

.…………………………….……………………………………………………………. [3]

(f) In practice, it has been found difficult to scale up a windmill such as this to a wheel of
30 m diameter, to achieve power outputs of the order of megawatts. Suggest two
reasons for this.

1. ………………….……………….……………………………………………………………

…….…………………………….……………………………………………………………

2. …….…….……………………………………………………………………………………

.…………………………….……………………………………………………………. [2]

© IJC 2010 9646/Prelim2 [Turn over


18
Section B For
Examiner’s
Use
It is recommended that you spend about 30 minutes on this section.

8 Cantilever is widely found in construction, notably in cantilever bridges and balconies.


The cantilever is a beam that is supported on only one end and it can be represented by
a loaded wooden rule, as shown in Fig 8.1. As a result of support only at one end, a
cantilever will oscillate when it is subjected to external forces such as wind.

Fig 8.1

Design a laboratory experiment to investigate how the period of the oscillation of the
loaded wooden rule depends on the overhanging length l of the rule.

You may assume that the following apparatus is available, together with any other
standard equipment that may be found in a school or college science laboratory.

wooden rule
slotted mass
stopwatch
G-clamp

You should draw diagram to show the arrangement of your apparatus. In your account
you should pay particular attention to

(a) the equipment you would use for the investigation,

(b) the procedure to be followed,

(c) the control of variables,

(d) any safety precautions,

(e) any precautions that you would take to improve the accuracy of the experiment. [12]

© IJC 2010 9646/Prelim2 [Turn over


19
Diagram For
Examiner’s
Use

…....……………………………………………………………………………………………………....

…....……………………………………………………………………………………………………....

…....……………………………………………………………………………………………………....

…....……………………………………………………………………………………………………....

…....……………………………………………………………………………………………………....

…....……………………………………………………………………………………………………....

…....……………………………………………………………………………………………………....

…....……………………………………………………………………………………………………....

…....……………………………………………………………………………………………………....

…....……………………………………………………………………………………………………....

…....……………………………………………………………………………………………………....

…....……………………………………………………………………………………………………....

© IJC 2010 9646/Prelim2 [Turn over


20
For
Examiner’s
…....…………………………………………………………………………………………………….... Use

…....……………………………………………………………………………………………………....

…....……………………………………………………………………………………………………....

…....……………………………………………………………………………………………………....

…....……………………………………………………………………………………………………....

…....……………………………………………………………………………………………………....

…....……………………………………………………………………………………………………....

…....……………………………………………………………………………………………………....

…....……………………………………………………………………………………………………....

…....……………………………………………………………………………………………………....

…....……………………………………………………………………………………………………....

…....……………………………………………………………………………………………………....

…....……………………………………………………………………………………………………....

…....……………………………………………………………………………………………………....

…....……………………………………………………………………………………………………....

…....……………………………………………………………………………………………………....

…....……………………………………………………………………………………………………....

…....……………………………………………………………………………………………………....

…....……………………………………………………………………………………………………....

…....……………………………………………………………………………………………………....

…....……………………………………………………………………………………………………....

…....……………………………………………………………………………………………………....

…....……………………………………………………………………………………………………....

…....……………………………………………………………………………………………………....

…....……………………………………………………………………………………………………....

…....……………………………………………………………………………………………………....

…....……………………………………………………………………………………………………....

…....……………………………………………………………………………………………………....

© IJC 2010 9646/Prelim2 [Turn over


21
For
Examiner’s
…....…………………………………………………………………………………………………….... Use

…....……………………………………………………………………………………………………....

…....……………………………………………………………………………………………………....

…....……………………………………………………………………………………………………....

…....……………………………………………………………………………………………………....

…....……………………………………………………………………………………………………....

…....……………………………………………………………………………………………………....

…....……………………………………………………………………………………………………....

…....……………………………………………………………………………………………………....

…....……………………………………………………………………………………………………....

…....……………………………………………………………………………………………………....

…....……………………………………………………………………………………………………....

…....……………………………………………………………………………………………………....

…....……………………………………………………………………………………………………....

…....……………………………………………………………………………………………………....

…....……………………………………………………………………………………………………....

…....……………………………………………………………………………………………………....

…....……………………………………………………………………………………………………....

…....……………………………………………………………………………………………………....

…....……………………………………………………………………………………………………....

…....……………………………………………………………………………………………………....

…....……………………………………………………………………………………………………....

…....……………………………………………………………………………………………………....

…....……………………………………………………………………………………………………....

…....……………………………………………………………………………………………………....

…....……………………………………………………………………………………………………....

…....……………………………………………………………………………………………………....

END OF PAPER

© IJC 2010 9646/Prelim2 [Turn over


INNOVA JUNIOR COLLEGE
JC 2 PRELIMINARY EXAMINATION 2
in preparation for General Certificate of Education Advanced Level
Higher 2

CANDIDATE
NAME

CLASS INDEX NUMBER

PHYSICS 9646/03, 9745/03


Paper 3 Longer Structured Questions 21 September 2010
2 hours
Candidates answer on the Question Paper
No Additional Materials are required.

READ THESE INSTRUCTIONS FIRST


For Examiner’s Use
Write your name, class and index number on all the work you
hand in. Section A
Write in dark blue or black pen on both sides of the paper.
You may use a soft pencil for any diagrams, graphs or rough
1
working. 10
Do not use staples, paper clips, highlighters, glue or correction
fluid. 2
10
Section A 3
Answer all questions. 10

4
Section B 10
Answer any two questions.
Section B
You are advised to spend about one hour on each section.
5
The number of marks is given in the brackets [ ] at the end of 20
each question or part question. 6
20

7
20

Total
80

Penalty

This document consists of 20 printed pages.

© IJC 2010Innova Junior College 9646,9745/Prelim2 [Turnover


[Turn over
2
Data
speed of light in free space, c = 3.00 x 108 m s-1
permeability of free space, μo = 4π x 10-7 H m-1
permittivity of free space, εo = 8.85 x 10 F m
-12 -1

≈ (1/(36π)) x 10-9 F m-1


elementary charge, e = 1.60 x 10-19 C
the Planck constant, h = 6.63 x 10-34 J s
unified atomic mass constant, u = 1.66 x 10-27 kg
rest mass of electron, me = 9.11 x 10-31 kg
rest mass of proton, mp = 1.67 x 10-27 kg
molar gas constant, R = 8.31 J K-1 mol-1
the Avogadro constant, NA = 6.02 x 1023 mol-1
the Boltzmann constant, k = 1.38 x 10-23 J K-1
gravitational constant, G = 6.67 x 10-11 N m2 kg-2
acceleration of free fall, g = 9.81 m s-2

Formulae
uniformly accelerated motion, s = ut + ½at2
v2 = u2+ 2as
work done on/by a gas, W = p ΔV
3
Average kinetic energy of a molecule of an ideal gas U = kT
2
hydrostatic pressure, p = ρgh
GM
gravitational potential, Φ = −
r
displacement of particle in s.h.m. x = xo sin ωt
velocity of particle in s.h.m. v = vo cos ωt

= ±ω (x o
2
− x2 )
resistors in series, R = R1 + R2 + …
resistors in parallel, 1/R = 1/R1 + 1/R2 + …
electric potential V = Q/4πεor
alternating current/voltage, x = xo sinωt
transmission coefficient T = exp (-2kd)
8π 2m (U − E )
where k =
h2
radioactive decay, x = xo exp(-λt)
0.693
decay constant, λ =

© IJC 2010 9646,9745/Prelim2 [Turn over


3
Section A For
Examiner’s
Use
Answer all the questions in this section.

1 (a) (i) Define simple harmonic motion.

………………………………………………………………………………………………

………………………………………………………………………………………………

………………………………………………………………………………………………

…………………………………..……………………………………………………… [2]

(ii) On the axes of Fig 1.1, sketch the variation with displacement x of the
acceleration a of a particle undergoing simple harmonic motion.
[1]

Fig 1.1

(b) A strip of metal is clamped to the edge of a bench and a mass is hung from its free
end as shown in Fig. 1.2.

Fig 1.2

© IJC 2010 9646,9745/Prelim2 [Turn over


4
The end of the strip is pulled downwards and then released. Fig. 1.3 shows the For
Examiner’s
variation with time t of the displacement y of the end of the strip. Use

Fig 1.3

(i) State two times, apart from t = 0, at which the end of the strip is stationary.

.…………………………………..……………………………………………………… [1]

(ii) State two times at which the end of the strip is moving vertically upwards with
maximum speed.

.…………………………………..……………………………………………………… [1]

(iii) State two times at which the end of the strip is moving with maximum downward
acceleration.

.…………………………………..……………………………………………………… [1]

(c) On Fig. 1.4, sketch the corresponding variation with time t of the potential energy Ep
of the vibrating system. [2]

Fig 1.4

(d) The string supporting the mass breaks when the end of the strip is at its lowest point
in an oscillation. Suggest what change, if any, will occur in the period and amplitude
of the subsequent motion of the end of the strip.

Period : ..……………………………………………………………………………..……

Amplitude : ..……………………………………………………………………………….. [2]

© IJC 2010 9646,9745/Prelim2 [Turn over


5
2 (a) (i) What is meant by the term internal energy of a system? For
Examiner’s
Use
………………………………………………………………………………………………

………………………………………………………………………………………………

………………………………………………………………………………………………

.…………………………………..……………………………………………………… [2]

(ii) Write down an equation representing first law of thermodynamics. Define the
symbols that you use.

………………………………………………………………………………………………

………………………………………………………………………………………………

………………………………………………………………………………………………

.…………………………………..……………………………………………………… [2]

(b) Some water in a saucepan is boiling.

(i) Explain why there is a change in internal energy as water changes to steam.

………………………………………………………………………………………………

………………………………………………………………………………………………

………………………………………………………………………………………………

.…………………………………..……………………………………………………… [2]

(ii) Explain why external work is done by the boiling water.

………………………………………………………………………………………………

………………………………………………………………………………………………

………………………………………………………………………………………………

.…………………………………..……………………………………………………… [2]

(iii) With reference to your answers in (b)(i) and (b)(ii), show that thermal energy
must be supplied to the water during the boiling process.

………………………………………………………………………………………………

………………………………………………………………………………………………

………………………………………………………………………………………………

.…………………………………..……………………………………………………… [2]

© IJC 2010 9646,9745/Prelim2 [Turn over


6
3 (a) A charged particle may experience a force in an electric field and in a magnetic field. For
Examiner’s
Use
State two differences between the forces experienced in the two types of field.

1. ……………………………………………………………………………………………...

……………………………………………………………………………………………...

2. ……………………………………………………………………………………………..

...………………………………………………………………………………………… [2]

(b) A proton, travelling in a vacuum at a speed of 4.5 × 106 m s–1, enters a region of
uniform magnetic field of flux density 0.12 T. The path of the proton in the field is a
circular arc, as illustrated in Fig. 3.1.

Fig 3.1

(i) State the direction of the magnetic field.

.…………………………………..……………………………………………………… [1]

(ii) Calculate the radius of the path of the proton, in terms of cm, in the magnetic field.

radius = ………………………… cm [3]

© IJC 2010 9646,9745/Prelim2 [Turn over


7
(c) A uniform electric field is now created in the same region as the magnetic field in For
Examiner’s
Fig. 3.1, so that the proton passes undeviated through the region of the two fields. Use

(i) On Fig. 3.1, mark with an arrow labelled E, the direction of the electric field. [1]

(ii) Calculate the magnitude of the electric field strength.

electric field strength = ………………………… N C-1 [2]

(d) Suggest why gravitational force on the proton has not been considered in the
calculations in (b) and (c).

.…………………………………..……………………………………………..…………… [1]

© IJC 2010 9646,9745/Prelim2 [Turn over


8
4 (a) (i) Explain what is meant by metastable state and population inversion. For
Examiner’s
Use
………………………………………………………………………………………………

………………………………………………………………………………………………

………………………………………………………………………………………………

.…………………………………..……………………………………………………… [2]

(ii) Explain why it is important in the production of laser to excite atoms to the
metastable state.

………………………………………………………………………………………………

………………………………………………………………………………………………

………………………………………………………………………………………………

.…………………………………..……………………………………………………… [2]

(iii) Explain how stimulated emission enables a laser to deliver a monochromatic light
beam.

………………………………………………………………………………………………

………………………………………………………………………………………………

………………………………………………………………………………………………

.…………………………………..……………………………………………………… [2]

© IJC 2010 9646,9745/Prelim2 [Turn over


9
(b) The most common function of a diode is to allow an electric current to pass in one For
Examiner’s
direction while blocking current in the opposite direction. This rectifying behaviour Use
can be achieved by using p-n junctions made of semiconductors. A p-n junction is
formed when a p-type and n-type extrinsic semiconductors are joined as shown in
Fig 4.1.

n-type p-type
material material

Fig 4.1

Explain, with a diagram, how the p-n junction acts as an open switch during the
rectification of an alternating current.

…...……………………………………………………………………………………………...

…...……………………………………………………………………………………………...

…...……………………………………………………………………………………………...

…...……………………………………………………………………………………………...

…...……………………………………………………………………………………………...

…...……………………………………………………………………………………………...

…...……………………………………………………………………………………………...

………………………………………..……………………………………………………… [4]

© IJC 2010 9646,9745/Prelim2 [Turn over


10
Section B For
Examiner’s
Use
Answer two of the questions in this section.

5 (a) (i) State Newton’s first law of motion and show it leads to the concept of force.

………………………………………………………………………………………………

………………………………………………………………………………………………

………………………………………………………………………………………………

………………………………………………………………………………………………

.………………………………..……..…………………………………………………. [2]

(ii) Using diagrams, with labelled arrows showing the velocity v and acceleration a,
describe situations in which an object

1. has an acceleration in the opposite direction to its velocity,


2. has an acceleration at right angles to its velocity.

In each case, include in your diagram, a labelled arrow to illustrate the direction of
the resultant force F acting on the object.

1.

…....……………………………………………………………………………………….

……....………………………………………………………………………………… [2]

2.

…....……………………………………………………………………………………….

……....………………………………………………………………………………… [2]

© IJC 2010 9646,9745/Prelim2 [Turn over


11
(b) A ship of mass 1.2 × 107 kg is moving backwards with a velocity of 0.50 m s-1 towards For
Examiner’s
a dockside. In order to stop the ship, the engines are ordered full ahead. Use

(i) Calculate the initial kinetic energy of the ship.

kinetic energy = ………………………… J [1]

(ii) Assuming that viscous effects are negligible, calculate the magnitude of the
constant retarding force which must be exerted on the ship if it is to stop in a
distance of 15 m.

retarding force = ………………………… N [3]

(iii) Calculate the time taken by the ship to stop under these conditions.

time = ………………………… s [2]

(iv) Explain qualitatively how your answer in (iii) would be affected by viscous forces.

…......……..…………………………………………………………………………………

…......……..…………………………………………………………………………………

…......……..…………………………………………………………………………………

……..……..…………………………………………………………………………………

…......……..…………………………………………………………………………………

...……..………………………………………………………………………………..... [3]

© IJC 2010 9646,9745/Prelim2 [Turn over


12
(v) Calculate the change in momentum of the ship as it comes to a complete halt at For
Examiner’s
the dock. Use

change in momentum = ………………………… N s [2]

(vi) Using your answer in (v) and with the aid of a diagram, explain how the law of
conservation of momentum is applied in this example.

……..……..…………………………………………………………………………………

……..……..…………………………………………………………………………………

……..……..…………………………………………………………………………………

……..……..…………………………………………………………………………………

……..……..…………………………………………………………………………………

...……..……………..…………………………………………………………………... [3]

© IJC 2010 9646,9745/Prelim2 [Turn over


13
6 (a) (i) Draw a labelled diagram of the type of apparatus used by Rutherford, Geiger and For
Examiner’s
Marsden to investigate the nuclear model of the atom. Use
[2]

(ii) With the aid of diagrams, discuss qualitatively two important outcomes of the
experiment that provided the evidence for the nuclear model of the atom.

………………………………………………………………………………………………

………………………………………………………………………………………………

………………………………………………………………………………………………

………………………………………………………………………………………………

………………………………………………………………………………………………

………………………………………………………………………………………………

………………………………………………………………………………………………

…………………………………..……………………………………………………… [6]

© IJC 2010 9646,9745/Prelim2 [Turn over


14
For
(b) A stationary radium ( 226
88 Ra ) nucleus decays into a radon (Rn) nucleus and an Examiner’s
α-particle. Use

(i) Write an equation for the radioactive decay.

….…………………………………..…………………………………………………… [1]
v
(ii) Show that the ratio of speeds α after the decay is approximately 56.
v Rn
[2]

© IJC 2010 9646,9745/Prelim2 [Turn over


15
(iii) After the decay which takes place at a large distance from the gold nucleus, the For
Examiner’s
α-particle moves head-on towards a gold ( 197 79 Au ) nucleus with a speed of Use

1.66 ×107 m s-1.

Sketch 2 curves on Fig. 6.1 to illustrate how the electrical potential energy
between the two particles, and the kinetic energy of the α-particle varies with
position between the α-particle and the gold ( 197
79 Au ) nucleus.

You may assume that the gold ( 197


79 Au ) nucleus remains stationary during the

interaction.
[2]
Point of closest
approach of α-particle B
A
α
vA
E/J

0
Position of α-particle relative to
the point of closest approach
Fig. 6.1

(iv) Calculate the distance of closest approach of the α-particle relative to the gold
( 197
79 Au ) nucleus.

distance = ………………………… m [3]

© IJC 2010 9646,9745/Prelim2 [Turn over


16
(v) With reference to your answer to (b) (iv), comment on the radius of the gold For
Examiner’s
nucleus. Use

..………………………………………………………………………………………..……

……………………………..……..………………………………………………..……. [1]

(vi) Estimate the ratio of

1. the diameter of gold atom to the diameter of its nucleus and

ratio = ………………………… [1]

2. the mass of a gold nucleus to the mass of a gold atom ( 197


79 Au ).

ratio = ………………………… [2]

© IJC 2010 9646,9745/Prelim2 [Turn over


17
7 (a) X-rays are emitted when a metal target is bombarded by high-energy electrons. For
Examiner’s
Fig. 7.1 shows a X-ray spectrum which consists of a broad continuous spectrum and Use
a series of sharp lines known as characteristic X-ray spectrum.

intensity

characteristic spectrum

continuous spectrum

wavelength
λo

Fig. 7.1

(i) Explain how the continuous spectrum in Fig. 7.1 is formed.

…..……………………………………………………………………………………..……

…..……………………………………………………………………………………..……

…..……………………………………………………………………………………..……

…..……………………………………………………………………………………..……

…..……………………………………………………………………………………..……

..…………………………..……..………………………………………………..……. [3]

(ii) Explain why there is a minimum wavelength λo for the continuous spectrum in
Fig. 7.1.

…..……………………………………………………………………………………..……

..…………………………..……..………………………………………………..……. [1]

(iii) Suggest a reason why the lines are called the characteristic X-ray spectrum.

…..……………………………………………………………………………………..……

..…………………………..……..………………………………………………..……. [1]

© IJC 2010 9646,9745/Prelim2 [Turn over


18
(b) Fig. 7.2 shows an experimental setup to investigate the photoelectric effect. For
Examiner’s
Ultraviolet (UV) light of wavelength 237 nm is incident on an emitter of area 2.0 cm2 Use
and a current reading of 2.00 nA is registered.

I
A

collector +
UV light Variable
V d.c supply
_
emitter

Fig. 7.2

(i) Given that every 1 in 5 photons causes a photoelectron to be emitted from the
emitter, show that the rate of photons incident on the emitter is 6.25 × 1010 s-1.
[2]

(ii) Calculate the energy of each photon incident on the emitter.

photon energy = ………………………… J [1]

(iii) Calculate the intensity of the incident radiation.

intensity = ………………………… W m-2 [3]

© IJC 2010 9646,9745/Prelim2 [Turn over


19
(iv) The battery connections in Fig. 7.2 are reversed so that the emitter is made For
Examiner’s
positive with respect to the collector. Use

1. Given that the work function of emitter is 4.7 eV, calculate the stopping
potential.

stopping potential = ………………………… V [2]

2. Explain why the stopping potential in (b)(iv)1. remains the same when the
intensity of the UV light is increased.

……………………………………………………………..……………………………

……………………………………………………………..……………………………

..….……………………………………………..………………………………………

……………………………………………………………..……………………………

..….……………………………………………..………………………………………

.………..……….…………………………..……………………….………………. [3]

© IJC 2010 9646,9745/Prelim2 [Turn over


20
(c) Fig. 7.3 represents the energy levels for an atom. The atom at ground state is For
Examiner’s
bombarded with an electron of energy 17 eV. Use

n=4 -2eV
n=3 -5 eV

n=2 -7 eV

n=1 -20 eV

Fig. 7.3

(i) State all possible photon energies when the atom returns to its ground state.

…..………………..……….……………………………………………………………. [2]

(ii) On Fig.7.4, sketch the appearance of the spectrum which corresponds to the
frequencies of the emitted photons.
[1]

Increasing frequency

Fig. 7.4

(iii) Explain the difference between an emission line spectrum and an absorption line
spectrum.

…..…………………………………………………………………………………………..

……………………………..……..………………………………………………..……. [1]

END OF PAPER

© IJC 2010 9646,9745/Prelim2 [Turn over


Innova Junior College
2010 Prelim 2 H2 Physics
Paper I Solutions

Qn Ans Qn Ans Qn Ans Qn Ans


1 B 11 B 21 D 31 D
2 C 12 C 22 B 32 D
3 C 13 B 23 D 33 C
4 D 14 D 24 C 34 C
5 A 15 D 25 C 35 A
6 B 16 A 26 B 36 B
7 C 17 B 27 C 37 B
8 B 18 D 28 B 38 B
9 A 19 D 29 A 39 A
10 B 20 D 30 B 40 C

1. A displacement (vector) and acceleration (vector)


B force (vector) and kinetic energy (scalar)
C momentum (vector) and velocity (vector)
D power (scalar) and speed (scalar)

2.
ΔV = 0.03×327.66
ΔV = 9.8298
ΔV = 10m s-1 (to 1 s.f.)

∴V = 330 ± 10m s-1

3.

-u
The vector on side X is the sum of vector : v + (-u)
From v = u + at,
v – u = at

4. Since air resistance is neglected, the ball undergoes free fall after it leaves the hand and
before it falls back on the hand. Under free-falling, the acceleration is due to gravity and
acting downwards (negative by convention) all the time, even when the ball is
momentarily at rest at the maximum height.

5. From Newton’s second law, net force is proportional to the rate of change of momentum.
For a case where there is no net force acting on an object, the object will maintain its
momentum (i.e. either stay at rest or continue its motion with a constant speed along a
straight line) , which is Newton’s first law.

1
6. Δ p = pf – pi

Sign convention: Take rightwards as positive.


Δ p = m vf – m vi
Δ p = [0.100 (–30)] – [0.100 (20)]
Δ p = – 5.0 kg m s-1 (to the left)

⏐Δ p⏐ = 5.0 kg m s-1

7.
From Conservation of Momentum,
Initial momentum of system = Final momentum of system
0 = px + p y
py = – p x

p2
Use KE = to establish the ratio,
2m
2
px 2 px
X 2(m ) 2(m )
= 2
= 2
= 2.0
Y py px
2(2m ) 2(2m )

8. Total force is the sum of the normal force and friction.


Normal force by road

Friction by road

2
9.

Ty

Tx

Hx
Hy

Considering the forces in the x-direction,


Tx = Hx

Considering the forces in the y-direction,


Ty = Hy + Wy

Since Tx = Hx and Ty > Hy,


hence T > H (Only Option A and C)

Since Ty > W, and W has no x-component,


Hence T > W
(Leaving option A as the answer)

10. By Hooke’s Law, F = k x

For spring P,
F = (2k) xP
F
xP =
2k

For spring Q,
F = (k) xQ
F
XQ =
k
Work done on spring:
W=½Fx
Since F is constant,
W∝ x

WP xP
=
WQ xQ
F
WP 2k 1
= =
WQ F 2
k
WP = ½ WQ

3
11.

T
At the top position, tension and weight are both acting vertically downwards on the ball.

T and W provides the centripetal force,

mv 2
= T +W
r
mv 2
= T + mg
r
(0.10)(6.0)2
T= - (0.10)(9.81)
(0.50)
T = 6.2 N

-GMm
12. From U = , U has negative values.
r
- dU
From F = , F is the negative gradient of U–r graph.
dr

13. For molecule to escape, it must be given enough KE to overcome the GPE.
-GMm
½ m v2 = 0 – ( )
r
2GM
v=
r
1
v∝
r
v1.2RE RE
=
v RE 1.2 RE

v1.2RE = 1.0 × 104 m s-1

14. At the amplitude position,


ao = - ω 2 x o
Fmax
= ω 2 xo
m
10
= ω 2 (0.05)
2.0
ω 2 =100

4
15.
From graph, PEmax at amplitude position = 1.0 J
By conservation of energy, KEmax at equilibrium position is also 1.0 J.

KEmax = ½ m vo2 = ½ (4.0) vo2 = 1.0


1
vo = m s-1
2

For SHM, vo = xo ω
1 2π
=(0.2)
2 T
1 2π
=
2 5T

2π 2
T= s
5

16. In degree Celsius,


θf - θI = Δθ
Converting to Kelvins,
(θf + 273.15) – (θI + 273.15) = Δθ

17.
For W → X,
work done on gas = area under the p-V curve = 400 J

For Y → Z,
work done on gas = area under the p-V curve = - 600 J

For X → Y and Z → W,
work done on gas = 0 J

For cycle W → X → Y → Z → W
Net work done on gas = 400 + 0 + (-600) + 0 = -200 J

18.
Δφ Δt
=
2π T
Δφ 1.5T
=
2π T
Δφ = 3 π
∴n = 3

1
19. Since I ∝ ,
d2
1
I=k ,
d2
1
For a graph of I against , a straight line passing through the origin is obtained.
d2

5
20. 2.5 λ = 30.0 cm
λ = 12.0 cm

21. The zeroth order fringes of the various wavelengths overlap to give a central bright fringe.
Other than this, the various wavelengths of light are diffracted by different diffraction
angles (red, having the largest wavelength, will be diffracted the most), to form coloured
fringes on each side.

22.
d sin θ = n λ
d sin 40° = 3 λ --- Eqn (1)

d sin 90° = nmax λ


nmax = 4.67
nmax = 4 (round down)

23. The electric field of a negatively point charge is directed radially inwards.

24. The electric field between 2 parallel charged plates is uniform. Hence E is constant when
plotted against d.

25. Q = I t
Ne e = I t
Ne I
=
t e

Ne
= 3.0 × 1019 s-1
t
The electrons flow from the negative to the positive terminal of the battery. Hence Y to X.

26.
ρL
R=
R
ρL
R= 2
⎛d ⎞
π⎜ ⎟
⎝2⎠

Since the wires have the same resistance and length,


ρ alumimium ρ silver
2
= 2
⎛ d alumimium ⎞ ⎛ d silver ⎞
π⎜ ⎟ π⎜ ⎟
⎝ 2 ⎠ ⎝ 2 ⎠

ρ silver d silver 2
=
ρ alumimium d alu min ium 2
1 d silver 2
=
2 d2
d silver = 0.71 d

6
27. The potential difference across each lamp in diagram 2 is the same as that in diagram 1.
Hence power dissipated in each lamp is the same. Each lamp will light up with normal
brightness.

28. The circuit can be redrawn in the following way.

Let the total external resistance be R.


1 1 1
= +
R 6 6
R = 3.0 Ω

E = I (R+r)
12 = I (3.0 + 3.0)
I = 2.0 A

Since I1 + I2 = 2.0 A
And I1 = I2

I1 = 1.0 A

29.
XY carrying a current upwards, produces a magnetic field (into the page) in the region
where PQRS is. Segment PS, carrying a current upwards experiences a magnetic force
directed towards XY while segment QR, carrying a current downwards will experience a
magnetic force away from XY. However, the strength of the magnetic field decreases
with distance from XY, hence PS being nearer to XY will experience a larger force
compared to QR. Hence, the coil experiences a net force towards XY and it will move in
that direction.

The magnetic force acting on PQ upwards and the magnetic force acting on SR is
downwards. These 2 forces are equal in magnitude and opposite in direction, hence they
cancel.

30. Initially,
Φ = nBA cos α , where α is the angle between the magnetic field and normal to the
plane.

Φ = nBA cos (90° - θ)


Φ = nBA sin θ

As θ becomes zero, Φ = 0. Hence the decrease is nBA sin θ.

7
31.

Zero gradient implies


Constant gradient velocity is zero
implies velocity is E = Blv is zero.
constant.
E = Blv is a constant.

32. For a sinusoidal p.d,


Vo 2
P R =W
<P> = o =
2 2
For the square-wave,
Vo 2
<P> = =2W
R

NS VS I P
33. If the transformer is 100% efficient, = = will be true.
NP VP I S
N V
If the transformer is NOT 100% efficient, only S = S can be applied.
NP VP
N V
Hence, S = S must be true.
NP VP

34. As light intensity increases, the number of photons arriving at the metal surface per unit
time increases. An increase in the number of photons per unit time by a certain
proportion will lead to an increase in the number of photoelectrons per unit time (hence
photocurrent) in the same proportion.

35. From E = hc , energy of photon 1 is smaller than energy of photon 2, hence λ1 > λ2.
λ
36. The square of the wavefunction ψ gives the probability of finding the electron at each
2

position.

8
37.
p2
KE =
2m
p2
1.0 × 106 × 1.6 × 10-19 =
2 (1.67 × 10-27 )
p = 2.312 × 10-20 kg m s-1

Δp = 0.01 (2.312 × 10-20) = 2.312 × 10-22 kg m s-1

Using the Heisenberg’s Uncertainty Pricnciple,


h
Δx Δp ≥

h
Δx (2.312 × 10-22 ) ≥

Δx ≥ 2.28 × 10-13
Hence, minimum Δx is 2.28 × 10-13 m.

38. B

39. 10
5 B + 01n → 73 Li + 42 He + net energy release

net energy release = energy release when 73 Li is formed from its constituent nucleons
+ energy release when 42 He is formed from its constituent nucleons
- energy required for 10
5 B to separate into its constituent nucleons

net energy release = 39.25 + 28.48 - 64.94


net energy release = 2.79 MeV
KE of 73 Li and 42 He + energy of γ photon = 2.79 MeV
Thus, energy of γ photon = 0.48 MeV

40. A = Ao e - λt
A
= e − λt
Ao
A
When Ao reduces by 90%, it means = 10%
Ao
⎛ ln2 ⎞
−⎜ ⎟t
0.10 = e ⎝ 3.0 ⎠

t = 9.97 s
t =10 s

9
Innova Junior College
2010 Prelim 2 H2 Physics
Paper 2 Solutions

Section A

1 (a)
kg m s-1 = [mass] × [velocity] = [momentum] or [impulse] [B1]

electric force
electric field strength =
charge
electric force
electric field strength =
charge
electric field strength = [E] = [mass] × [acceleration]
[current] × [time]
kg m s-2
[E] =
As
[E] = kg m A -1 s-3 [B1]

(b) (i) Estimate the density of plastic to be 2 times that of water = 2 g cm-3

Mass of protractor = (Volume) (Density)


Mass of protractor = ½ π r 2 x ρ
Mass of protractor = ½ π (5.0)2 (0.1) (2)
Mass of protractor = 7.9 g

[M1] for logical reasons


[A1] for acceptable range of 5 to 50 g

(ii) Micrometer Screw Gauge [B1]

(iii) Repeat the measurement of x at different parts of the protractor and taking
average. [B1]

2 (a) (i) Taking into consideration, the vertical component of the motion

Vertical component of initial velocity


u y = u sin θ = (20) sin 30 o = 10.0 m s-1

v 2y = u 2y + 2 gh
0 = (10.0) 2 + 2(−9.81)h [M1]
h = 5.10 m [A1]

(ii) The time taken to reach maximum height


v y = u y + gt
0 = 10.0 + (−9.81)t
t = 1.02 s [C1]

1
vx = u x = u cos θ = (20) cos 30o = 17.3 m s-1

The horizontal distance (or the range)


= (u x )(time of flight )
= (17.3)(1.02×2)
= 35.3 m [A1]

(b)

20 m s-1 A
o
30

Fig. 2.1

(i) The maximum height and the horizontal distance will both be reduced. [B1]

Note: The path should be shorter in height and in range slightly. No mark for
excessive reduction in height and range as the effect of air resistance is
slight. The path should be asymmetrical as the net force on the clay disc is
varied.

(ii) Suggest an explanation for any differences between the two paths N and A.

The maximum vertical height is smaller because the net force acting against the
object moving up is larger (weight plus air resistance). The deceleration on the
object is greater, thus the maximum vertical displacement is reduced. [B1]

The shorter time of flight due to the reduced maximum height causes the horizontal
distance covered to be reduced. Or the additional horizontal force due to air
resistance causes the object to be displaced less horizontally. [B1]

3
v

star

star 1.0×1011 m
Fig. 3.1
v

(a)(i) Correct labelling of point X at the centre of rotation. [B1]

2
The stars are directly opposite each other. When an object of any mass is placed at
the centre, it will have two forces of equal magnitude acting in opposite directions
thereby causing the net gravitational force on the object to be zero. Hence, the net
gravitational field strength here will be zero.
[B1]

(ii) Gravitational potential at point X


⎛ GM ⎞
= ⎜− ⎟×2
⎝ r ⎠
⎛ (6.67 × 10 −11 )(4.0 × 10 30 ) ⎞
= ⎜⎜ − ⎟×2
⎟ [M1]
⎝ 1.0 × 1011 ⎠
= -5.34 × 109 J kg-1 [A1]

(Deduct 1 mark if negative sign is not included)

It is also the work done to transfer an object of unit mass from infinity to this point.

GM 2
(b)(i) Net force = Gravitational force =
(2r ) 2
(6.67 × 10 −11 )(4.0 × 10 30 ) 2
= [M1]
(2 × 1.0 × 1011 ) 2
= 2.67×1028 N [A1]

(ii) Gravitational force provides the centripetal force.


Mv 2
= Net force
r
(4.0 × 10 30 )v 2
= 2.67×1028 [M1]
1.0 × 10 11

v = 2.58×104 m s-1 [A1]

4
(a) (i) Magnetic flux density is the magnetic force acting on a straight wire per unit length
per unit current flowing through it, when the wire is placed perpendicular to the
magnetic field. [B1]

(ii) It is the amount of magnetic flux density of a uniform magnetic field when a magnetic
force per unit length per unit current of 1 newton per metre per ampere acts on a
straight wire placed perpendicular to the magnetic field. [B1]

(b) (i)

3
Magnetic field
F 0.12 m
Y

0.20 m

0.32A F

Correct direction for forces acting on each side of the coil showing a couple [B1]

(ii) Magnitude of magnetic force on one side of the coil


F = NBIL sinθ
= (150)(0.36)(0.32)(0.20) sin 90o [M1]
= 3.46 N [A1]

(iii) Torque of a couple


= (Magnitude of one force)×(perpendicular distance between forces)
= (3.46)(0.12)
= 0.415 N m [A1]

5 (a) The single slit So is not necessary because the light source used is a laser and it is
already coherent by nature. [B1]

λD (630 ×10−9 )(2.5)


(b) Separation of fringes x = = [M1]
d (1.0 ×10−3 )
= 1.58×10-3 m
= 1.58 mm [A1]

(c) Since the separation of the slits is maintained, the separation of the fringes is
unchanged. Because one slit is made narrower, the fringes are not so distinct or
clearly observable. [B1]
The amplitude of the light emerging from it will be less than that from the other slit.
The amplitudes are not the same and hence, when the waves combine at the
locations of destructive interference, there are no total cancellations of waves.
[B1]

(d)The observation that there are bright and dark fringes on the screen is an indication
that interference has taken place. [B1]

Bright fringes are where constructive interference takes place while dark fringes are
where destructive interference takes place. The summation of the two combined light
rays is a direct consequence of the principle of superposition which applies to waves.
Thus, light has wave properties. [B1]

4
6
(a) (i) A beam of electrons passes through the graphite ‘diffraction grating’ [B1].
An interference pattern of circular concentric rings is seen on the screen [B1].
The de-Broglie wavelength of the electron is of the same order as the lattice spacing
of the graphite atoms [B1].

(ii)
h
λ=
mv
6.63 × 10−34
= [M1]
(1.82 × 10−28 ) × (0.10 × 3.0 × 108 )
=1.21× 10-13 m [A1]

(iii) For a transverse wave, there are many possible vibration of particles as long as it is
perpendicular to the direction of the wave, but the axis of vibration of particles in a
longitudinal wave is always parallel to the direction of the wave [B1].

(b) As a result of collision, the momentum of the electron increases and the momentum of
h
the photon decreases as the momentum of the scattered photon is less than the
λ'
h
momentum of the incident photon (since λ’ is longer than λ) [B1].
λ
From the principle of conservation of linear momentum [B1], the decrease in the
momentum of the incident photon displays the particulate nature of electromagnetic
radiation.

5
7.
(a) Linearise the equation P = k vn by taking lg on both sides,
lg P = lg (k v n)
lg P = lg k + n lg v [B1]

Plotting lg P against lg v. [B1]

If the points lie close to a straight-line trend, the relationship P = k v n is true. [B1]

(b)
lg (P / W)

3.5
0.91, 3.45

3.0

2.5

2.0 0.38, 1.90

1.5

1.0
0.20 0.30 0.40 0.50 0.60 0.70 0.80 0.90 1.00
-1
lg (v / m s )
(a) (i) For diameter = 6.0 m and v = 3.0 m s-1,
P = 150 W
lg P = 2.18 [A1]

(ii) 1. [B1] for correct point plotted.

(iii) 2. [B1] for suitable best-fit-line drawn.

(iii)
3.45 -1.90
Gradient = [B1]
0.91- 0.38
n = 2.93 (to 3 s.f) [A1] for n = 2.95 ± 0.05

6
Sub (0.38, 1.90) and gradient = 2.93,
1.90 = (2.93) (0.38) + y-intercept
y-intercept = 0.7866
lg k = 0.7866
k = 6.12 [A1]

(c) (i) Consider the air is of a cylindrical volume,


volume πr 2 x
=
time t
volume
= πr 2v
time
2
volume ⎛ 6.0 ⎞
=π ⎜ ⎟ 8.0
time ⎝ 2 ⎠
volume
= 226.2 = 230 m3 s-1
time
In 1 sec,
Volume of air = 230 m3 [A1]

1
(c) (ii) kinetic energy = mv 2
2
1
kinetic energy = ( ρV)v 2
2
1
kinetic energy = (1.3)(226.2)(8)2 [M1]
2

kinetic energy = 9409.92 = 9400 J [A1]

useful power output 2750


(d) = = 0.292 (to 3 s.f) [A1]
total power 9409.92

(e) 3 × [B1] marks for any of the points:


- Height of the windmill. The wind is stronger at higher heights, hence the
output power will be higher.
- Location of the windmill. For example, the wind is stronger near the coast and
the power output will be higher.
- Type of material used for the blades. A lighter material has smaller inertia in
rotating and hence the power output will be higher.
- Shape of the blades. An aerodynamic shape will cause the power output to
be higher.
- Surface area of the blade. A larger surface area implies that more wind is in
contact with the blade and the power output will be higher.
- Friction between the wheel and the axle. A higher friction would mean that
more energy is lost through heat, causing the power output to be less.

(f) 2 × [B1] marks for any of the points:


- The wheel of diameter 30 m may be too heavy to mount during construction.
- The wheel produces a very large torque, hence may be unstable.
- To provide a strong frame to support it, it may be costly.
- There may be space constraints to construct long blades as they extend 30m
high up.

7
Section B
Suggested Solution

Diagram
B
fixed ends load
G-clamp

Diagram marks (D1)


1. Diagram is to show that the cantilever is fixed using g-clamp & bench or retort stand and clamp
or any other appropriate means.

Procedure
1. Set up the apparatus as shown above.

Control Mark (C1)


Variable(s) to be controlled

2. Ensure that the mass of the load at the end of the strip remains unchanged by using the same
slotted mass.
3. Ensure that the position of the mass remains unchanged by taping or gluing it at the end of
the rule.
4. Ensure that the Young modulus of the rule remains unchanged by using the same wooden
rule. (not type or material as not quantifiable)
5. Ensure that the width and/or thickness of the rule remain unchanged by using the same
wooden rule.
(Maximum 1 out of these 4 controls)

Procedure and Measurement Mark (P1 and M1)


setup
Initial

6. Measure the overhanging length l of the rule using the scale on the rule [M1].

7. Slightly displace the end of the rule vertically downwards so that the end of the rule starts
Measurement
of dependent

oscillating in a vertical plane. [P1]


variable

8. Measure the time taken t for many oscillations such that the time is more than 20 seconds
using a stopwatch. [M1] An oscillation is considered when the end of the rule moves from A to
B, then back to B to A.
Repeat

9. Repeat steps 6 to 8 to obtain another 7 set of values of t with different overhanging length l of
the rule by shifting the rule [P1].

Analysis Mark (A1)


t
Plotting
Graph

10. The period of an oscillation T is calculated using T = .


n
11. Plot a graph of lg T against lg l to determine the relationship [A1].

8
Additional Detail Mark (AD1)
Reliability measures
1. The same mass should be used (to keep the mass of the loaded wooden rule constant)
2. The load should be secured to the rule using tape and/or glue (to keep the position of the
mass constant).
3. The same rule should be used (to keep Young Modulus and the dimensions of the rule
constant).
4. Start timing after the oscillations become stable (as the first few oscillations are usually jerky
Reliability

and will constitute to the random errors in measuring the time taken).
5. Take repeated readings of time taken t to reduce the random errors incurred due to the
inconsistent starting and stopping of stopwatch
6. The mass of the load should be large enough to make period T large.
7. A fiducial marker should be used to help in the measurement of the time taken t.
8. Discussion of how motion sensor or light gates can be used to ensure accurate measurement
of time taken t.
9. The amplitude/angle of oscillations should be small (to ensure that the oscillations follow
S.H.M.).

Safety Precautions
1. There should have sand placed below the load just in case the load falls off or the load
Safety

should be secured to prevent the load falling off.


2. No person or fragile object should be placed near the possible landing area of the load if it
falls off.
(Maximum 5 out of all the reliability and safety measures)

END OF PAPER

9
Innova Junior College
2010 Prelim 2 H2 Physics
Paper 3 Solutions

Section A
1
(a) (i) Simple harmonic motion is defined as the periodic motion in which acceleration is
directly proportional to displacement [B1] but opposite in direction to the
displacement [B1].
(ii)

The graph should be a downwards sloping straight line through the origin [B1]

(b)
(i) 0.05 s, 0.10 s, 0.15 s, 0.20 s [B1] for any 2 correct answers
(ii) 0.025 s, 0.125 s [B1] for both correct answers
(iii) 0.05 s, 0.15 s [B1] for both correct answers

(c)

Correct shape (sinusoidal curve with all above t-axis and maximum Ep at t = 0) [B1]
Correct period (2 maximum Ep for every 0.10 s) [B1]

(d)
When the mass breaks at the lowest point, the resultant force increases with the
reduction in the weight (net force is mainly due to the tension in the stretched metal
strip).
With an increased in restoring force and a decrease in mass, acceleration will
increase and hence period will decrease.
The equilibrium position shift higher, hence the amplitude of subsequent motion of
the end of the strip increases.
Note: the total available energy for energy transformation increases with the shift in
equilibrium upwards, hence the amplitude of subsequent motion increases according
to ET = ½ kx02.

1
2
(a) (i) Internal energy of a system is the sum of all the microscopic kinetic and potential
energies of the atoms/molecules within the system. [B1] The kinetic energy is
associated with the random motion of the atoms/molecules while the potential
energy is associated with the intermolecular forces between molecules. [B1]

(ii) ΔU = Q + W [B1]
ΔU : increase in internal energy of the system
Q : thermal energy (heat) supplied to the system
W : work done on system [B1]

(b)
(i) Since there is no change in temperature during boiling, the kinetic energy of the
water molecules remains constant [B1]. However, when the volume of the water
expands during boiling, the separation between water molecules increases, the
potential energy of the atoms increases [B1]. Since internal energy of water is the
sum of potential and kinetic energy of the water molecules, there is a change
(increase) in internal energy.

(ii) The volume occupied by the water molecules increases [B1] on vaporisation.
Hence the water molecules has to do work, pushing back the atmosphere [B1].

(iii) During boiling, the internal energy of the water molecules increases [M1] (ΔU:
+ve). According to first law of thermodynamics, the thermal energy supplied must
be positive since work done on the boiling water is negative [A1],

2
3 (a)
1. The magnitude of the electric force is independent of the speed of the charged
particle while the magnitude of the magnetic force is directly proportional to the
speed of the charged particle. [B1]
2. The direction of electric force is along the direction of the electric field while the
direction of the magnetic force is perpendicular to the direction of the magnetic
field. [B1]

(b)
(i) Since the proton deflects downwards, the magnetic force acting on the proton is
downwards. With velocity towards right, the direction of the magnetic field is out
of the plane of paper [B1] (not upwards) according to Fleming’s Left Hand rule.

(ii) Fc = mac
mv 2
Bqv =
r
1.67 × 10 −27 × 4.5 × 10 6
0.12 × 1.6 × 10 −19 = [M1]
r
r = 0.391 m [C1]
r = 39.1 cm [A1]

(c)

(i)

With magnetic force acting downwards, the electric force acting on the proton
must be upwards. Thus the electric field should be acting upwards.
Arrow pointing vertically upwards [B1].

(ii) When the proton passes undeviated, Fn = 0


FE = FB
Eq = Bqv
E = 0.12 × 4.5 × 106 [M1]
= 5.40 × 105 V m-1 [A1]

(d) The magnitude of gravitational force is negligible as compared to the magnitude of


the electric or magnetic forces [B1].
(‘mass of proton is small’ is an insufficient answer)

3
4 (a)
(i) A metastable state is an excited state of an atom in which the atom can remain
for a relatively longer time interval (~10-3 s) than most other normal excited
states (~10-8 s). [B1]

Population inversion is a situation in which there are more atoms or molecules in


excited states than in lower energy states (or ground state). [B1]

(ii) With atoms in the metastable state, the atoms can stay there longer, so that
population inversion can be achieved [B1].
As such, a photon with appropriate energy equivalent to the energy difference
between the two energy levels can cause stimulated emission to take place [B1],
than absorption.

(iii) When an atom in its excited state encounter an incoming photon with energy
equivalent to the energy difference between the excited state and its lower
energy level [B1], stimulated emission will occur. The incoming photon can
stimulate the de-excitation of the atom such that the subsequent emitted photon
has the same energy (same wavelength) as the incoming photon [B1], hence the
light beam has same wavelength (monochromatic).

(b)

[B1] for correct diagram

When the potential at the p-type material is negative with respect to the n-type
material (reverse bias) [B1], holes and electrons are pulled away from the junction
[B1], creating a larger negative potential difference. The depletion region widens [B1]
i.e. no current.

4
Section B
5.
(a) (i) Newton’s First Law of Motion states that every body continues in its state of rest or
uniform motion in a straight line unless a net external force acts on it to change that
state. [B1]

Hence, if we observe an object initially in a state at rest but which suddenly moves,
or an object moving at uniform speed in a straight line, but which suddenly reduces
or increases in speed or changes in direction, we say that there must be a net
external force acting on it. [B1]

(ii)
1. v

a
F

[B1] for diagram showing 3 arrows.

Description: An object that is projected vertically upwards in the earth’s gravitational


field. [B1]
v
2.
a
F

B1] for diagram showing 3 arrows.

Description: An object that is undergoing circular motion. [B1]

(b)(i)
1 2 1
Initial KE = mu = (1.2 × 107 )(0.50)2
2 2
= 1.5 × 106 J [A1]

(ii)
Using v 2 = u 2 + 2as
0=(0.5)2 + 2a(15)
-(0.5)2
a= [M1]
2(15)
-(0.5)2
net force = ma = (1.2 × 107 )[ ] [M1]
2(15)
= -1.0 × 105 N

magnitude of retarding force is 1.0 × 105 N [A1]

5
(iii)
Using v = u + at ,
−(0.50)2
0 = 0.50 + [ ]t [M1]
2 × 15
t = 60 s [A1]
(iv)
Viscous force FD ∝ v where v is the instantaneous velocity [B1]
Viscous force will be greatest initially as the initial speed is greatest, and it will
reduce to zero when the final speed is zero.
The viscous force will provide a greater retardation force [B1], and the time
taken to stop the ship will be shorter [B1].

(v)
Δp =m(v - u ) = 1.2 × 107 (0 − 0.50) [M1]
Δp = - 6.0 × 10 N s
6
[A1]

Fws
Fsw
Fws = force exerted by water on the ship
Fsw = force exerted by ship on the water
(Diagram - B1)

By considering the ship and the water as an isolated system and with no external
forces acting on the ship and the water [B1], the momentum lost by the ship is equal
to the momentum gained by the water [B1].

6
Beam of alpha particles Gold foil target
(a)(i)

Radium source

Details: Radium source emitting beam of alpha particles, gold foil target &
detector
(Deduct up to 2 × B1 for each missing detail)

(a)(ii)

Nucleus
of gold
atom

Diagram with details-B1

Most of the α-particles passed straight through the foil with little or no deflection [B1].
This concluded that the atom consist of mostly empty space.[B1]

Some α-particles were deflected from their original direction of travel [B1]. This
suggested that there was an electrostatic force of repulsion between the nucleus and
the positively charged alpha particles. This concluded that the nucleus of the atom is
positively charged[B1].

A very small number of alpha particles were deflected with an angle greater than 90
degree from their original path [B1]. This suggested that the positive charge is
concentrated in the nucleus of the atom. This concluded that the nucleus is massive
[B1].

[Any 2 of the above 3 observations with deductions made to be provided by


candidates for marks to be awarded]

7
(b)(i) 226
88 Ra → 222
86 Rn+ 24α [B1]

(ii) From the Principle of conservation of momentum,

0 = mα vα + mRn ( −v Rn ) [C1]
vα m 222u
= Rn = [M1]
v Rn mα 4u
≈ 56 (shown) [A0]

Point of closest
(iii) B
A approach of α-particle

α
vA
E/J

KE

EPE

One curve showing KE –B1


One curve showing EPE –B1
Deduct B1 if the total energy (KE & EPE) is not constant.

8
(iv) From the Principle of Conservation of energy,

1 q q
mα vα 2 = α Au [C1]
2 4πε o r
(2e )(79e )
r=
2πε o (mα )(vα )2
(2)(79)(1.6 × 10−19 )2
= [M1]
2πε o (4 × 1.66 × 10−27 )(1.66 × 107 )2
= 3.97 × 10−14 m [A1]

(v) The radius of the gold nucleus is in the order of 10-14 m [B1].

(vi) 1.
datom = 10–10 m & dnucleus = 10–14 m
datom / dnucleus = 104 [A1]

2.
Mnucleus 79mprotons + 118mneutrons
=
Matom 79mprotons + 118mneutrons + 79melectrons
197(1.67 × 10−27 )
= [M1]
197(1.67 × 10 −27 ) + 79(9.11× 10 −31 )
=1.00 [A1]

7
(a)(i) When an electron collides with one of the target atoms, it loses an amount of kinetic
energy that corresponds to the energy of an X-ray photon[B1].

The electron may continue to lose kinetic energy in a series of collisions with other
atoms, thereby giving off X-ray photons of different energies [B1].

Since different X-ray photons correspond to different wavelengths, the continuous


spectrum is thus formed [B1].

(ii) An electron may lose all its kinetic energy in a single collision with a target atom [B1],
thereby emitting a photon of maximum energy E max . that corresponds to a minimum
wavelength λo .

(iii) The wavelengths of the characterlistic lines are unique for each element [B1].

2.00 × 10−9
(b) (i) Rate of electrons produced = −19
= 1.25 × 1010 [M1]
1.6 × 10
Rate of incident photons = 1.25 × 1010 × 5 [M1]
=6.25 × 1010 s−1 [A0]

(ii)
hc 6.63 × 10−34 × 3.0 × 108
E= =
λ 237 × 10−9
= 8.39 × 10−19 J [A1]

9
(iii) power = 8.39 × 10−19 × 6.25 × 1010 = 5.24 × 10−8 W [M1]
power 5.24 × 10−8
= [M1]
Intensity = area 2.0 × 10−4
= 2.62 × 10−4 W m−2 [A1]

(iv)1.
hf = φ + eVs

Vs =
−19
(
hf − φ 8.39 × 10 − 4.7 1.6 × 10
=
−19
) [M1]
e 1.6 × 10 −19
= 0.544 V [A1]

(iv)2.
Increasing intensity does not affect the photon energy and the photon energy remains
the same as the frequency of UV light is constant. [B1].
The radiation is incident on the same emitter surface (work function is also constant)
[B1].
The maximum kinetic energy of the photoelectron does not change [B1].
Hence the stopping potential does not change.

(c) (i) 2 eV, 13 eV & 15 eV [Deduct B1 for each incorrect answer]

(ii)

Increasing frequency

3 distinct lines drawn with 2 lines closely spaced on the right of the spectrum [B1]

(iii) An emission line spectrum consists of bright lines against a dark background but an
absorption line spectrum consists of dark lines against a continuous spectrum. [B1]

10
Class Adm No

Candidate Name:

Preliminary Examination II 2010


Pre-university 3

H2 Physics 9646
Paper 1: Multiple-Choice Questions
9646/01
Friday 24 September 1h 15m

Additional materials:
OMR

READ THESE INSTRUCTIONS FIRST

Write your name, class and admission number in the spaces at the top of this page and on all the
work you hand in.
Write in soft pencil.
Do not use staples, paper clips, highlighters, and glue or correction fluid.

There are forty questions on this paper. Answer all questions. For each question there are four
possible answers A, B, C, and D.
Choose the one you consider correct and record your answer in soft pencil on the separate
OMR.

Read the instructions on the OMR very carefully.

Each correct answer will score one mark. A mark will not be deducted for a wrong answer.
Any rough working should be done in this booklet

This question paper consists of 21 printed pages.


2

Data

speed of light in free space, c = 3.00  108 m s–1

permeability of free space, 0 = 4  10–7 H m–1

permittivity of free space, 0 = 8.85  10–12 F m–1

= (1/(36))  10–9 F m–1

elementary charge, e = 1.60  10–19 C

the Planck constant, h = 6.63  10–34 J s

unified atomic mass constant, u = 1.66  10–27 kg

rest mass of electron, me = 9.11  10–31 kg

rest mass of proton, mp = 1.67  10–27 kg

molar gas constant, R = 8.31 J K–1 mol–1

the Avogadro constant, NA = 6.02  1023 mol–1

the Boltzmann constant, k = 1.38  10–23 J K–1

gravitational constant, G = 6.67  10–11 N m2 kg–2

acceleration of free fall, g = 9.81 m s–2


3

Formulae

1 2
uniformly accelerated motion, s = ut + at
2

v2 = u2 + 2as

work done on/by a gas, W = pV

hydrostatic pressure, p = gh

gravitational potential, Gm
 = 
r

displacement of particle in s.h.m. x = xo sin t

velocity of particle in s.h.m., v = vo cost

v =   xo2  x 2

mean kinetic energy of a molecule of


an ideal gas E =

resistors in series, R = R1 + R2 + …

resistors in parallel, 1 1 1
=   ...
R R1 R 2

electric potential, Q
V =
4 0 r

alternating current/voltage, x = xo sin t

transmission coefficient T = exp(2kd)

8 2 m(U  E )
where k =
h2

radioactive decay, x = x0 exp(–t)

0.693
decay constant,  = t1
2

[Turn Over
4

1. In the equation, P = Q – RS, P, Q, R and S are physical quantities. For the


equation to be homogeneous, which of the following must be true.

A. P, Q, R and S all have the same units.


B. P, Q, R and S are all scalar quantities.
C. The product RS have the same units as P and Q.
D. The product RS is numerically equal to (Q – P)

2. A student wishes to determine the density  of a cylinder of mass M, height


d 2
h and diameter of cross section d, using the formula M  h . If the
4
percentage uncertainties of his measurements of M, d and h are 3%, 1%
and 2% respectively, what will be the percentage uncertainty of  ?

A. 3% B. 5%

C. 6% D. 7%

3. A train travelling at 2.0 m s-1 passes through a station. The graph below
shows the variation with time, t of the speed, v of the train after leaving the
station.

What is the speed of the train when it is 150 m from the station?

A. 6.0 m s-1 B. 8.0 m s-1

C. 10 m s-1 D. 12 m s-1
5

4. Which of the following is a false statement regarding how two forces give
rise to a couple?

A. They act in opposite directions.


B. They both act at the same point.
C. They both act on the same body.
D. They both have the same magnitude.

5. A string of negligible mass with a weight of 50 N at one end is attached at


the other end to a fixed point P. The sphere is acted upon by a horizontal
force of 80 N such that the string makes an angle to the vertical. What is
the value of ?

A. B.

C. D.

6. Five blocks of equal masses V, W, X, Y and Z are connected by four


identical strings as shown in the diagram below. They are pulled by a
steadily increasing force F. Which of the strings A, B, C or D is most likely
to break first?

[Turn Over
6

7. A 60 kg archer stands at rest on frictionless ice and fires a 0.50 kg arrow


horizontally at 50 m s-1. What is the final velocity of the archer as he moves
across the ice after firing the arrow?

A. 0.42 m s-1 B. 0.48 m s-1

C. 0.60 m s-1 D. 0.72 m s-1

8. A stone of weight 80 N is thrown vertically upwards from the ground with an


initial kinetic energy of 150 J. Ignoring air resistance, the kinetic energy, the
gain in potential energy, and the work done against gravity when the stone
is at 1.00 m above the ground will be

Kinetic Energy Gain in Potential Work Done


Energy
A. 70 J 70 J 70 J
B. 70 J 80 J 70 J
C. 70 J 80 J 80 J
D. 80 J 150 J 80 J

9. A block of iron is suspended in water


7

The block experiences an upthrust from the water.

Which is the explanation for this upthrust?

A. Iron is more dense than water.


B. The area of face R is greater than the area of face S.
C. The density of water increases with depth.
D. The pressure of water increases with depth.
10. A helicopter which has blades of diameter 5.0 m is hovering above the
ground at a particular instance. Its blades are rotating in such a way that
they are pushing air downwards at a speed of 18 m s-1.

The density of the surrounding air can be taken as 1.02 kg m-3.

The upward force acting on the blades is

A. 360 N B. 1400 N

C. 6500 N D. 26000 N

11. To travel at constant speed, a car engine provided 25 kW of useful power.


The driving force on the car is 600 N. At what speed is the car travelling at?

A. 2.5 m s-1 B. 4.0 m s-1

C. 25 m s-1 D. 42 m s-1

[Turn Over
8

12. A satellite is in circular orbit 144 km above the Earth. Assuming that the
radius of the Earth is 5760 km, the gravitational force on the satellite
compared with that when it is at the Earth’s surface is approximately

A. greater by 10%
B. greater by 5%
C. the same
D. less by 5%

13. Why does the Moon stay in its orbit at a constant distance from the Earth?

A. The gravitational pull of the Earth on the Moon is just sufficient to cause
the centripetal acceleration of the Moon.
B. The gravitational pull of the Earth on the Moon balances the
gravitational pull of the Moon on the Earth.
C. The gravitational pull of the Moon on the Earth is negligible at this
distance.
D. The centripetal force the Earth exerts on the Moon balances the
centripetal force the Moon exerts on the Earth.

14. P is a planet with centre O, as shown in the figure below. X and M are two
points of equal gravitational potential, A . Y and N are two other points of
equal gravitational potential, B .
9

Which of the following statements is correct?

A. The work done by an external agent to move a mass from Y to X is


negative.
B. The work done by the gravitational field to move a mass from X to N is
different from that needed to move the same mass from M to Y.
C. The work done by the gravitational field to move a mass from N to M is
negative.
D.  A OX

B OY

15. For one complete oscillation in simple harmonic motion with amplitude, a,
how does velocity, v vary with displacement, x?

A. B.

C. D.

16. An object undergoes simple harmonic motion with an amplitude A, and its

[Turn Over
10

total energy is E. What is the displacement of the object from the equilibrium
3E
position when its kinetic energy is ?
4

A. 3 B. A
A
2 4

C. 3A D. A
4 2

17. Which of the following statement is true?

A. The internal energy of a system can be increased by transfer of energy


by heating.
B. The internal energy of a system is dependent only on its temperature.
C. When the internal energy of a system is increased, its temperature
always rises.
D. When two systems have the same internal energy, they must be at the
same temperature.
18. A heater of power of 300 W is immersed in a filter funnel of crushed ice.
Before the heater is switched on, 10 g of water is collected from the melting
ice in 1 minute. When the heater is switched on, the mass of water collected
from the melting ice in 1 minute is 85 g. The specific latent heat of fusion of
ice, in J g-1 is

A. 5 B. 240

C. 300 D. 1200

19. A star emits electromagnetic waves of wavelengths 50 m, 5 m, 0.5 m and
0.05 m. In which regions of the electromagnetic spectrum are they?

Wavelength

50 m 5 m 0.5 m 0.05 m
A. infra-red infra-red visible ultraviolet
B. infra-red visible ultraviolet x-ray
11

C. microwave visible visible ultraviolet


D. radio infra-red visible ultraviolet

20. A stationary sound wave has a series of nodes. The distance between the
first and the sixth antinode is 30.0 cm.

What is the wavelength of the sound wave?

A. 5.0 cm B. 6.0 cm

C. 10.0 cm D. 12.0 cm

21. Monochromatic light with a wavelength of  is incident normally on a


diffraction grating. The angle between the directions of the two second-order
diffracted beams is  .

What is the spacing of the lines on the grating?

A. 2 B. 
sin  sin 

C. 2 D. 
 2
sin  sin  2

[Turn Over
12

22. A sphere is released from rest, in vacuum, between two parallel, vertical
metal plates with a separation of 0.10 m. The sphere has a weight of
1.6  103 N and an electric charge of +2.0 C. The potential difference
between the two plates is 80 V.

Which of the following depicts the path of the sphere after release?

23. A positive charge is placed at P and a negative charge is placed at Q. The


electric potential at different points between these two charges is shown in
the figure below.

Which of the following graphs correctly shows the variation with distance, x
along line PQ of the electric field strength, E?
13

A. B.

C. D.

24. Two charges, P and Q are 100 mm apart.

X is a point on the line between P and Q. If the potential at X is 0 V, what is


the distance from P to X?

A. 40 mm B. 45 mm

C. 50 mm D. 60 mm

25. What is the definition of resistance?

A. Resistance is the potential difference per unit current.


B. Resistance is the gradient of the graph of potential difference against
current.
C. Resistance is the voltage required for a current of 1 A.
D. l
Resistance is defined by the equation R  , where  is the resistivity
A
of the material, l is the length of the wire, and A is its cross-sectional
area.

[Turn Over
14

26. A main circuit has six identical bulb connected in series. One of the bulbs
has a broken filament. Voltmeters X and Y of infinite resistance are placed in
the circuit as shown.

What are the voltmeter readings?

X reading Y reading
A. 0V 0V
B. 0V 240 V
C. 40 V 40 V
D. 240 V 0V

27. A diode is connected to a battery as shown below.

Which of the following statements about the diode is not true?

A. No current flows through the diode.


B. The resistance is very large.
C. The voltage across the diode is zero.
15

D. The voltage across the diode is the same as the e.m.f. of the source.

28. In the circuit shown below, the light bulb will become brighter when

A. light is incident on the LDR and P is moved to X.


B. light is incident on the LDR and P is moved to Y.
C. the LDR is covered and P is moved to X.
D. the LDR is covered and P is moved to Y.

29. A straight wire PQ carrying a constant current I is placed perpendicularly to a


uniform magnetic field, as represented by the dotted line in the diagram
below.

[Turn Over
16

The current-carrying wire is then rotated through an angle,  about an axis


perpendicular to the plane of the diagram.

Which of the following graph shows how the magnitude of the magnetic force
F on the wire varies with  in the range 0° to 90°?

A. B.

C. D.

30. Two long straight wires, X and Y are placed perpendicular to each other at a
17

small distance, d apart, with wire X vertically above the point R. The current
in wire X is out of the page and the current in wire Y is to the right as shown
in the figure below.

What is the direction of the force acting on wire Y at point P due to the
magnetic field produced by wire X?

A. Out of the page


B. Into the page
C. Upwards
D. Downwards

31. An e.m.f. is induced in a wire that is subjected to a varying magnetic field.


The following graphs show how magnetic flux density, B of this field varies
with time, t.

At what time is the magnitude of the e.m.f induced in the wire at its
maximum?

A. 1 ms B. 2 ms

C. 3 ms D. 4 ms

[Turn Over
18

32. Two flat horizontal coils are mounted as shown

Which of the following actions will cause the galvanometer to register a


current?

1. With switch S closed, Coil 2 is stationary while Coil 1 is moved


upwards.
2. Both coils are stationary and S is switched on and off.
3. With switch S closed, the variable resistor R is increased and
decreased rapidly.

A. 2 only
B. 1 and 3 only
C. 2 and 3 only
D. 1, 2 and 3

33. A sinusoidal alternating current flows through a resistor.

What is the mean power dissipated in the resistor?

A. 1
maximum current 
2

2
B. 1
maximum power 
2
C. 1
 
2
maximum current
2
D. 1
2

maximum power 
19

34. The half-wave rectification of an alternating current sinusoidal voltage of


amplitude 100 V gives the waveform as shown in the figure below. The r.m.s.
value of the rectified voltage is

A. 25 V B. 50 V

C. 71 V D. 100 V

35. A laser beam of power, P has wavelength,  . What is the rate of photons
produced?

A. hc B. Ph
P c

C. Phc D. P
hc

36. Which of the following observations regarding the photoelectric effect is not predicted
by the classical theory of electromagnetic radiation?

A. The rate of photoelectric emission is proportional to the intensity of the incident


radiation.
B. Irrespective of the intensity of radiation, photoelectric emission is instantaneous.
C. Photoelectric emission is observable using any part of the electromagnetic
spectrum.
D. The maximum kinetic energy of the photoelectrons is dependent on the intensity
of the radiation.

[Turn Over
20

37. At room temperature, the charge carriers inside a p-type semi-conductor are

A. holes only
B. electrons only
C. positive ions
D. both holes and electrons

38. The following diagram illustrates the upper energy bands in two different classes of
solids at absolute zero. The shaded areas represent occupied electron energy levels.

What are bands P and Q, and classes X and Y of solids?

Band P Band Q Class X Class Y


A. conduction valence intrinsic semi-conductor metal
B. conduction valence metal intrinsic semi-conductor
C. valence conduction intrinsic semi-conductor metal
D. valence conduction metal intrinsic semi-conductor

39. Which of the following equations correctly shows an -particle causing a nuclear
reaction?

A. 14
7
N  42 He  178 O  11n

B. 17
8
O  42 He  20
9
F  11p

C. 17
8
O  10e  135 B  42 He

D. 14
7
N  11p  116 C  42 He
21

40. Initially, a source comprises No nuclei of a radioactive nuclide. What is the number of nuclei
decayed after a time interval of three half-lives?

A. No B. No
16 8

C. 7No D. 15No
8 16

END OF PAPER

[Turn Over
Class Adm No

Candidate Name:

Preliminary Examination II 2010


Pre-university 3

H2 Physics 9646
Paper 2 Structured Questions
9646/02
Friday 17 September 1h 45m

Candidates answer on the Question Paper.


No Additional Materials are required.

READ THESE INSTRUCTIONS FIRST

Write your name, class and admission number in the spaces at the
top of this page.
Write in dark blue or black pen.
You may use a soft pencil for any diagrams, graphs or rough
working. For Examiner’s Use
Do not use staples, paper clips, highlighters, and glue or correction
fluid. 1
2
Section A
Answer all questions. 3
It is recommended that you spend about 1 hour 15 minutes on this 4
section.
5
Section B 6
Answer Question 8
It is recommended that you spend about 30 minutes on this section 7
8
At the end of the examination, fasten all your work securely together.
The number of marks is given in brackets [ ] at the end of each Total
question or part question.

This question paper consists of 23 printed pages and a blank page.

[Turn over
2

Data

speed of light in free space, c = 3.00  108 m s–1

permeability of free space, 0 = 4  10–7 H m–1

permittivity of free space, 0 = 8.85  10–12 F m–1

= (1/(36))  10–9 F m–1

elementary charge, e = 1.60  10–19 C

the Planck constant, h = 6.63  10–34 J s

unified atomic mass constant, u = 1.66  10–27 kg

rest mass of electron, me = 9.11  10–31 kg

rest mass of proton, mp = 1.67  10–27 kg

molar gas constant, R = 8.31 J K–1 mol–1

the Avogadro constant, NA = 6.02  1023 mol–1

the Boltzmann constant, k = 1.38  10–23 J K–1

gravitational constant, G = 6.67  10–11 N m2 kg–2

acceleration of free fall, g = 9.81 m s–2


3

Formulae

1 2
uniformly accelerated motion, s = ut + at
2

v2 = u2 + 2as

work done on/by a gas, W = pV

hydrostatic pressure, p = gh

gravitational potential, Gm
 = 
r

displacement of particle in s.h.m. x = xo sin t

velocity of particle in s.h.m., v = vo cost

v =   ( x o2  x 2 )

mean kinetic energy of a molecule of 3


an ideal gas E = kT
2

resistors in series, R = R1 + R2 + …

resistors in parallel, 1 1 1
=   ...
R R1 R 2

electric potential, Q
V =
4 0 r

alternating current/voltage, x = xo sin t

transmission coefficient T = exp(2kd)

8 2 m(U  E )
where k =
h2

radioactive decay, x = x0 exp(–t)

0.693
decay constant,  = t1
2

[Turn Over
4 For
Examiner’s
Use

Section A

Answer all questions


It is recommended that you spend about 1 hour 15 minutes on this section.

1. (a) Distinguish between a scalar quantity and a vector quantity, providing an example for
each.

…………………………………………………………………………………………………

…………………………………………………………………………………………………

……………………………………………………………………………………………....[2]

(b) A ship is initially cruising in the direction bearing of 045 at a speed of 10 m s-1
changes direction to head in a new direction bearing of 145 at a speed of 5 m s-1, as
seen in Fig. 1.1. The manoeuvre was completed in 30 s.

Fig. 1.1

Using a vector diagram or otherwise, calculate the acceleration of the ship during the
change in direction.

acceleration of the ship =…………………..m s-2 at bearing of ………… [3]


For
5 Examiner’s
Use

2. (a) A body accelerates at 10 m s-2 over a period of time.

Explain what is meant by the phrase in italics.

…………………………………………………………………………………………………

……………………………………………………………………………………………....[1]

(b) A Singapore Army soldier is undergoing marksmanship training by firing at the target,
as seen in Fig. 2.1 below. He uses the SAR 21 Rifle, which has a muzzle velocity
(velocity at which the bullet exits the rifle) of 900 m s-1.

Fig. 2.1

He ensures his rifle is perfectly horizontal while aiming for the head of the target. He
then fires a bullet at the target, which is 300 m away from his current position.

(i) Calculate the time taken for the bullet to reach the target after it is fired.

time taken =……….…..s [1]

(ii) What is the assumption made in this calculation?

……………………………………………………………………………………………

………………………………………………………………………………………...[1]

[Turn Over
6 For
Examiner’s
Use

(iii) Assuming that it is a perfect shot (i.e. the rifle was perfectly horizontal and was
aimed properly at the head of the target), calculate the vertical distance
between his aiming point and where the bullet actually hits the target.

vertical distance =………………m [2]

(iv) After passing through the target, the bullet is entrenched 4.0 m deep into a
sandbag that is directly behind the target board.

Calculate the average deceleration of the bullet as it moves within the sandbag

average deceleration =…………….m s-2 [2]

(v) If the assumption was not made in (b) (ii), state whether the bullet would hit the
target at a lower or higher point than what was calculated in (b) (iii).

……………………………………………………………………………………………
………………………………………………………………………………………...[1]

3. (a) Using the simple Kinetic Model of Matter, explain the following:

(i) the melting of ice takes place without a change in temperature.

……………………………………………………………………………………………
……………………………………………………………………………………………
………………………………………………………………………………………...[2]

(ii) the specific latent heat of vaporisation of water is higher than its specific latent
heat of fusion.

……………………………………………………………………………………………
……………………………………………………………………………………………
………………………………………………………………………………………...[2]
For
7 Examiner’s
Use

(b) An ideal gas at constant pressure has its volume directly proportional to its absolute
temperature.

Calculate the absolute temperature T when an ideal gas has volume 0.00825 m3,
assuming that the same mass of the ideal gas at the same pressure has volume
0.00424 m3 at a temperature of 273 K.

absolute temperature T =…………….K [2]

(c) (i) State the conversion formula from the Celsius scale (°C) to the thermodynamic
absolute scale (K)
[1]

(ii) Hence or otherwise, comment on whether the following statement is true:

“Today the temperature is 30 °C and yesterday it was 15 °C. Hence it is twice


as hot today as it was yesterday.”

……………………………………………………………………………………………

……………………………………………………………………………………………

………………………………………………………………………………………...[1]

[Turn Over
8 For
Examiner’s
Use

4. (a) A 1 kg solid block of iron, when placed on water, will sink. The same block of iron is
shaped into a model of a boat, and this model would float on water.

Explain why the boat model would float, and not sink.

…………………………………………………………………………………………………

…………………………………………………………………………………………………

…………………………………………………………………………………………………

………………………………………………………………………………….………......[2]

(b) A string supports a solid iron object of mass 200 g. The solid iron object is hence
suspended in mid-air

(i) Calculate the tension in the string when the solid iron object is suspended by
the string in mid-air

tension in the string =…………. N [1]

(ii) Given that the density of iron is 8000 kg m-3, calculate the volume of the solid
iron object

Volume of solid iron object =………….. m3 [1]

(iii) Calculate the new tension in the string when the solid iron object is completely
immersed in a liquid of density 800 kg m-3.

New tension in string =…………… N [2]


For
9 Examiner’s
Use

(c) A solid cube with sides of length, x is placed on the surface of a fluid. The cube is
then slowly pushed downwards into the fluid, as seen in Fig.. The distance from the
base of the cube to the surface of liquid is given by h. The cube is pushed
downwards until h >> x.

Fig. 4.1

Fig. 4.2

Sketch in, Fig. 4.2 above, the graph showing the variation of the upthrust acting on
cube due to the fluid against the distance from the base of the cube to the fluid
surface, h. Your graph should show the variation of upthrust for values of h that are
greater than x.
[2]

[Turn Over
10 For
Examiner’s
Use

5. (a) Define magnetic flux density.

…………………………………………………………………………………………………

…………………………………………………………………………………………………

……………………………………………………………………………………………....[2]

(b) Fig. 5.2 below shows a loosely-coiled metal spring hanging from a fixed point.

Fig. 5.1

The ends of the spring are connected to a circuit as shown in Fig. 5.1. When a
current is switched on, the spring becomes compressed (i.e. the coils move closer to
one another and the vertical length of the spring becomes slightly shorter than
before).

(i) Explain why the spring shortens in length when the current is switched on.

……………………………………………………………………………………………

……………………………………………………………………………………………

………………………………………………………………………………………...[2]
For
11 Examiner’s
Use

(ii) State and explain if the spring is lengthened or compressed if the direction of
current is reverse?

……………………………………………………………………………………………

……………………………………………………………………………………………

………………………………………………………………………………………...[1]

(c) Fig. 5.2 below shows a beam of -particles of speed v entering a region in space
where there is a uniform magnetic field of flux density B.

Fig. 5.2

In the magnetic field, they are deflected from their original pathway and move along
the arc of a circle of radius r.

(i) Indicate on the figure the direction of the magnetic field.


[1]

(ii) Sketch the path of -particles that has a speed greater than v.
[1]

(d) A beam of electrons that is travelling with velocity 3.4  107 m s-1 enters
perpendicularly into a region with magnetic flux density of 8.4 mT.

(i) Calculate the magnitude of the magnetic force on an electron.

magnetic force = …………. N [2]

[Turn Over
12 For
Examiner’s
Use

(ii) State and explain the changes (if any) in the kinetic energy of electron due to
the magnetic force acting on it.

……………………………………………………………………………………………

……………………………………………………………………………………………

………………………………………………………………………………………...[1]

6. (a) State and explain briefly the conditions necessary for the production of a laser beam.

…………………….…………………….…………………….…………………….…………

…………………….…………………….…………………….…………………….…………

…………………….…………………….…………………….…………………….…………

…………………….…………………….…………………….…………………….…………

…………………….…………………….…………………….…………………….…………

…………………………………………………………………………………………………

…………………………………………………………………………………………………

……...…………………….…………………….…………………….……………………..[3]
For
13 Examiner’s
Use

(b) Complete the table below to show the three differences between light from a filament
lamp and laser light.

Light from a filament lamp Laser light

[3]

[Turn Over
14 For
Examiner’s
Use

7. X-rays are emitted when a metal target is bombarded by high-energy electrons. The X-ray
spectrum consists of a broad continuous spectrum and a series of sharp lines known as
the line or characteristic X-ray spectrum.

The K-series line spectrum arises when an incoming electron removed an atomic electron
from the innermost shell, K-shell (n = 1). An electron from the L-shell (n = 2) may fill the
vacancy, and in the process emits a photon. The photon emitted has an energy that
corresponds to the wavelength of the K  -line shown in Fig. 7.1.

Fig. 7.1

(a) (i) Explain how the continuous spectrum is formed.

……………………………………………………………………………………………

……………………………………………………………………………………………

……………………………………………………………………………………………

………………………………………………………………………………………...[2]

(ii) Explain why there is a minimum wavelength 0 for the continuous spectrum.

……………………………………………………………………………………………

……………………………………………………………………………………………

………………………………………………………………………………………...[1]
For
15 Examiner’s
Use

(iii) Explain how the K  -line arises.

……………………………………………………………………………………………

……………………………………………………………………………………………

………………………………………………………………………………………...[1]

(b) In 1913, a British physicist H. G. J. Moseley measured the wavelength of the


characteristics X-rays from a number of elements. He noted that when the square-
root of the frequency f is plotted against the position Z (atomic number) of the
element in the periodic table, a straight line results.

The energy for a multi-electron atom is given by the expression

k Z  1
2
En  
n2

where k is a constant

(i) Show that the frequency f of the K  -line is given by the expression

f  C Z  1

where C is a constant.
[2]

[Turn Over
16 For
Examiner’s
Use

(ii) The experimental data for the variation with Z of f are shown in Fig. 7.2.

On Fig. 7.2, draw the best-fit line for the points.


[1]

Fig. 7.2
For
17 Examiner’s
Use

(iii) Determine the constant C, with its unit.

C =…………………

unit =………………[2]

(iv) Hence, or otherwise, determine the wavelength of the K  -line for copper
whose atomic number Z is 29.

wavelength =……………….. m [2]

(c) The X-ray spectrum can be studied using the X-ray diffraction. A particular set of
crystal planes that have a known spacing d is chosen, as seen in Fig. 7.3 These
planes effectively reflect different wavelengths at different angles.

Fig. 7.3

[Turn Over
18 For
Examiner’s
Use

(i) Fig. 7.4 shows a graph of intensity against angular position  (measured in
degrees) for the diffraction of an X-ray beam by a crystal. The beam consists of
two wavelengths, and the spacing d between the reflecting planes is 0.94 nm.

Fig. 7.4

The condition for constructive interference is given by the expression


2d sin  m , which is known as Bragg’s law.

Using Bragg’s law, calculate the values of these two wavelengths of the X-ray
beam.

1 [1]
1 =………………… pm [1]

2 =………………… pm [1]
For
19 Examiner’s
Use

(ii) A standard optical diffraction grating where the grating spacing is 3000 nm
cannot be used to discriminate between different wavelengths in the X-ray
region of approximately 0.1 nm.

Using the formula for the calculation of the diffraction angle, given the
wavelength and grating spacing, explain quantitatively why this is so.

……………………………………………………………………………………………

……………………………………………………………………………………………

……………………………………………………………………………………………

……………………………………………………………………………………………

………………………………………………………………………………………...[2]

[Turn Over
20 For
Examiner’s
Use

Section B

Answer Question 8
It is recommended that you spend about 30 minutes on this section.

8. A manufacturer of filament lamps for use as a night-light in children's bedrooms is concerned


about the surface temperature of the glass envelope after the lamp has been left on for some
time, The surface temperature must not be too high, otherwise there is a danger of burns or fire
resulting from accidental contact with the surface of the lamp. However, it is important that the
power output from the lamp should not be too small, otherwise the lamp will be too dim to be
useful.

Power supply Voltmeter


Connecting wires Ammeter
Light bulbs of different diameters Oscilloscope
3 V battery Bucket of water
Crocodile clips Thermometer
Signal generator Rheostat
Microphone Thermistor
Bunsen burner Thermocouple
Loudspeaker

Your answer should contain a diagram showing how the chosen equipment would be
arranged, together with details of

(a) the procedure to be followed,

(b) your choice of the type of thermometer used to measure the surface temperature of the
lamp,

(c) how the electrical power would be measured,

(d) any relevant safety precautions you may take when carrying out the experiment,

(e) any particular features of your design that may improve the accuracy of your experiment.

[12]
For
21 Examiner’s
Use

DIAGRAM

………………………………………………………………………………………………………………

………………………………………………………………………………………………………………

………………………………………………………………………………………………………………

………………………………………………………………………………………………………………

………………………………………………………………………………………………………………

………………………………………………………………………………………………………………

………………………………………………………………………………………………………………

………………………………………………………………………………………………………………

………………………………………………………………………………………………………………

………………………………………………………………………………………………………………

………………………………………………………………………………………………………………

………………………………………………………………………………………………………………

………………………………………………………………………………………………………………

[Turn Over
22 For
Examiner’s
Use

………………………………………………………………………………………………………………

………………………………………………………………………………………………………………

………………………………………………………………………………………………………………

………………………………………………………………………………………………………………

………………………………………………………………………………………………………………

………………………………………………………………………………………………………………

………………………………………………………………………………………………………………

………………………………………………………………………………………………………………

………………………………………………………………………………………………………………

………………………………………………………………………………………………………………

………………………………………………………………………………………………………………

………………………………………………………………………………………………………………

………………………………………………………………………………………………………………

………………………………………………………………………………………………………………

………………………………………………………………………………………………………………

………………………………………………………………………………………………………………

………………………………………………………………………………………………………………

………………………………………………………………………………………………………………

………………………………………………………………………………………………………………

………………………………………………………………………………………………………………

………………………………………………………………………………………………………………

………………………………………………………………………………………………………………

………………………………………………………………………………………………………………

………………………………………………………………………………………………………………

………………………………………………………………………………………………………………

………………………………………………………………………………………………………………
For
23 Examiner’s
Use

………………………………………………………………………………………………………………

………………………………………………………………………………………………………………

………………………………………………………………………………………………………………

………………………………………………………………………………………………………………

………………………………………………………………………………………………………………

………………………………………………………………………………………………………………

………………………………………………………………………………………………………………

………………………………………………………………………………………………………………

………………………………………………………………………………………………………………

………………………………………………………………………………………………………………

………………………………………………………………………………………………………………

………………………………………………………………………………………………………………

………………………………………………………………………………………………………………

………………………………………………………………………………………………………………

………………………………………………………………………………………………………………

………………………………………………………………………………………………………………

………………………………………………………………………………………………………………

………………………………………………………………………………………………………………

………………………………………………………………………………………………………………

………………………………………………………………………………………………………………

………………………………………………………………………………………………………………

………………………………………………………………………………………………………………

………………………………………………………………………………………………………………

END OF PAPER
Class Adm No

Candidate Name:

Preliminary Examination II 2010


Pre-university 3

H2 Physics 9646
Paper 3 Longer Structured Questions
9646/03
Monday 20 September 2h

Candidates answer on the Question Paper.


No Additional Materials are required.

READ THESE INSTRUCTIONS FIRST

Write your name, class and admission number in the spaces at the
top of this page.
Write in dark blue or black pen.
You may use a soft pencil for any diagrams, graphs or rough
working. For Examiner’s Use
Do not use staples, paper clips, highlighters, and glue or correction
fluid. 1

Section A 2
Answer all questions. 3
Section B 4
Answer any two questions.
5
You are advised to spend about one hour on each section. 6
At the end of the examination, fasten all your work securely together.
The number of marks is given in brackets [ ] at the end of each 7
question or part question.
Total

This question paper consists of 33 printed pages and a blank page

[Turn over
2

Data

speed of light in free space, c = 3.00  108 m s–1

permeability of free space, 0 = 4  10–7 H m–1

permittivity of free space, 0 = 8.85  10–12 F m–1

= (1/(36))  10–9 F m–1

elementary charge, e = 1.60  10–19 C

the Planck constant, h = 6.63  10–34 J s

unified atomic mass constant, u = 1.66  10–27 kg

rest mass of electron, me = 9.11  10–31 kg

rest mass of proton, mp = 1.67  10–27 kg

molar gas constant, R = 8.31 J K–1 mol–1

the Avogadro constant, NA = 6.02  1023 mol–1

the Boltzmann constant, k = 1.38  10–23 J K–1

gravitational constant, G = 6.67  10–11 N m2 kg–2

acceleration of free fall, g = 9.81 m s–2


For
3 Examiner’s
Use
Formulae

1 2
uniformly accelerated motion, s = ut + at
2

v2 = u2 + 2as

work done on/by a gas, W = pV

hydrostatic pressure, p = gh

gravitational potential, Gm
 = 
r

displacement of particle in s.h.m. x = xo sin t

velocity of particle in s.h.m., v = vo cost

v =   ( x o2  x 2 )

mean kinetic energy of a molecule of 3


an ideal gas E = kT
2

resistors in series, R = R1 + R2 + …

resistors in parallel, 1 1 1
=   ...
R R1 R 2

electric potential, Q
V =
4 0 r

alternating current/voltage, x = xo sin t

transmission coefficient T = exp(2kd)

8 2 m(U  E )
where k =
h2

radioactive decay, x = x0 exp(–t)

0.693
decay constant,  = t1
2

[Turn Over
For
Examiner’s
4 Use

Section A

Answer all the questions in this section.

1. An intelligent elevator as seen in Fig. 1.1, can take passengers to the top of a skyscraper.
The elevator has a built-in weight sensor below its floor that can be used to measure the
combined weight of the elevator’s occupants.

Fig. 1.1

An emergency braking mechanism can be activated within a few seconds if the steel cable
holding onto the elevator snaps.

A man decides to take the elevator to reach the upper floors. He has a mass of 80 kg, and
the elevator has a mass of 1.0  103 kg.
For
5 Examiner’s
Use

(a) Calculate the initial reading of the weight sensor when the elevator is stationary.

initial reading of weight sensor =……………..N [1]

(b) The lift accelerates upwards with an acceleration of 2.5 m s-2.

(i) Draw a free-body diagram of the man, indicating and labelling the forces acting
on him.

There is no need to show the magnitude of the forces involved.


[1]

[Turn over
For
Examiner’s
6 Use

(ii) Hence or otherwise, calculate the reading of the weight sensor when the
elevator is accelerating upwards at 2.5 m s-2.

Reading of weight sensor =…………….N [2]

(iii) If the maximum G-force that can be experienced safely by a human is 1.5G
(i.e. 1.5 times of the human’s weight), calculate the maximum upward
acceleration of the elevator that is still considered safe.

Maximum safe upward acceleration =……………. m s-2 [1]

(c) A computer simulation is designed to test the response of the emergency braking
mechanism and the weight sensor should the elevator’s cable snap when there are
passengers onboard. The simulation takes place in the following sequence:

Time / s Incident
0 Elevator is stationary at a height of 250 m.
t1 Steel cable snaps and elevator starts to
freefall.
t2 Emergency braking system kicks in and
elevator undergoes deceleration.
t3 Elevator comes to a complete stop.
For
7 Examiner’s
Use

Sketch a graph using the axes provided in Fig. 1.2 below to show the variation of the
weight sensor’s reading with time during the simulation. The original reading, W, is
indicated.
[2]

Fig. 1.2

(d) The owner of the skyscraper decided to replace the motor powering the elevator as it
was not efficient enough. He decided to use a motor that is 75% efficient, replacing
the original model which is 60% efficient.

[Turn over
For
Examiner’s
8 Use

(i) If the elevator is designed to hold up to 8000 N of passenger weight, and is


required to rise up to a height of 250 m in 60 s, calculate the power required to
achieve the task.

power required =…………… W [2]

(ii) Hence, calculate the input power that is required to operate the new motor.

input power =…………….W [1]


For
9 Examiner’s
Use

2. (a) State the formula to calculate the centripetal force required to keep a body of mass
m, moving in a circle of radius r with speed v.

…………………………………………………………………………………………………

……………………………………………………………………………………………....[1]

(b) A smooth toy car track is set up in the following manner as seen in Fig. 2.1 below.

Fig. 2.1

Cars 1 and 2 are identical in construction and have a mass of 500 g each. Car 1 is
released from rest at point A, which is at a height of 0.60 m. Car 1 is designed to
move down the slope and complete the circular loop (during which it would be upside
down at the top of the loop), before colliding with Car 2.

(i) Calculate the speed of Car 1 at point B after it has moved down the slope.

speed of Car 1 at point B =………………….. m s-1 [2]

[Turn over
For
Examiner’s
10 Use

(ii) If Car 1 was to just reach point C, i.e., it comes to a complete stop at point C,
state the height of the circular loop.

height of circular loop =……………… m [1]

(iii) Explain why if the circular loop has the height that is calculated in (b) (ii), Car 1
would be unable to complete the entire loop safely.

………………………………………………..…………………………………………

………………………………………………..…………………………………………

…..…………………………………….…………………………………………….. [2]

(iv) Hence or otherwise, calculate the maximum radius of the circular loop.

maximum radius of circular loop =………………m [2]


For
11 Examiner’s
Use

(c) After completing the loop, Car 1 collides elastically with Car 2. Car 1 comes to a
complete stop, and Car 2 moves forward and compresses the spring until it comes to
a complete stop

Given that the spring constant of the spring is 150 N m-1, calculate the compression
of the spring when Car 2 comes to a complete stop.

compression of spring =……………… m [2]

[Turn over
For
Examiner’s
12 Use

3. (a) In Fig. 3.1 below, the battery has an internal resistance r and the ammeter has
negligible resistance.

Fig. 3.1

The graph in Fig. 3.2 below shows how current I in the circuit varies as the potential
difference V across the variable resistor R changes.

Fig. 3.2
For
13 Examiner’s
Use

(i) Define the term potential difference.

……………………………………………………………………………………………

……………………………………………………………………………………………

………………………………………………………………………………………...[2]

(ii) 1. Show that the e.m.f E of the battery is 6 V.


[2]

[Turn over
For
Examiner’s
14 Use

2. What is the power dissipated in the variable resistor R when the current
in the circuit is 1.2 A?

power dissipated =……………..W [2]

3. What is the internal resistance r of the cell?

internal resistance r =……………… [2]


For
15 Examiner’s
Use

(b) Four resistors are connected as shown in Fig. 3.3 below. Point a is at a higher
potential than point b.

Fig. 3.3

If a wire is connected from c to d, state and explain the direction of the current that
will flow through the wire.

…………………………………………………………………………………………………

…………………………………………………………………………………………………

…………………………………………………………………………………………………

………………………………………………………………………………………………[2]

[Turn over
For
Examiner’s
16 Use

4. Fig. 4.1 below shows an arrangement used to determine the wavelength  of


monochromatic light emitted by a laser.

Fig. 4.1

S1 and S2 are slits that are at right angles to the plane of this page. When illuminated by
light from the laser, they form coherent sources of light. An interference pattern is formed
on the screen, from which measurements can be taken to determine .

(a) Explain what is meant by interference of two coherent sources.

…………………………………………………………………………………………………

…………………………………………………………………………………………………

…………………………………………………………………………………………………

………………………………………………………………………………………………[2]
For
17 Examiner’s
Use

(b) (i) Describe briefly the interference pattern produced using the arrangement
shown in Fig. 4.1 above.

……………………………………………………………………………………………

……………………………………………………………………………………………

……………………………………………………………………………………………

………………………………………………………………………………………...[2]

(ii) Describe the roles played by diffraction and interference in the production of
the interference pattern.

……………………………………………………………………………………………

……………………………………………………………………………………………

……………………………………………………………………………………………

………………………………………………………………………………………...[2]

(iii) Calculate the wavelength  of light emitted by the laser, given the following
values:

D = 2.0 m
a = 0.5 mm
Fringe spacing = 2.5 mm.

wavelength  =……………… nm [2]

[Turn over
For
Examiner’s
18 Use

(c) The interference pattern is easier to observe when the amplitudes of the waves from
the two coherent sources are similar.

Explain why this is so.

………………………………………………………………………………………………….

………………………………………………………………………………………………….

………………………….………………………………………………………………………

…………………………………………………………………………………………….....[2]

Section B

Answer two questions in this section

5. (a) Explain what is meant by simple harmonic motion, and give an example of it that
occurs in nature.

…………………………………………………………………………………………………
…………………………………………………………………………………………………
…………………………………………………………………………………………………
……………………………………………………………………………………………….[2]
For
19 Examiner’s
Use

(b) A vertical peg is fixed to the rim of a horizontal turntable of radius r = 15.0 cm,
rotating with a constant angular speed  = 4.0 rad s-1, as shown in Fig. 5.1 below.

Fig. 5.1

Parallel light is incident on the turntable so that the shadow of the peg is observed on
a screen, which is normal to the incident light. At time t = 0,  = 0 and the shadow of
the peg is seen at S.

At some later time t, the shadow is seen at T.

(i) Write down an expression for the angular displacement  in terms of  and t.
[1]

[Turn over
For
Examiner’s
20 Use

(ii) Derive and expression for the distance ST in terms of r,  and t.


[2]

(iii) By reference to your answer in (b) (ii), explain how the motion executed by the
shadow is simple harmonic in nature.

……………………………………………………………………………………………

……………………………………………………………………………………………

……………………………………………………………………………………………

……………..………………………………………………………………………….[2]

(iv) Calculate the period of the motion of the shadow on the screen.

period of shadow’s motion =…………… s [1]


For
21 Examiner’s
Use

(v) Hence or otherwise, calculate


1. the speed of the shadow as it passed through S,

speed of shadow =……………… m s-1 [2]

2. the magnitude of the acceleration of the shadow when it is


instantaneously at rest

magnitude of acceleration =………………. m s-2 [2]

(c) A light spring hangs vertically from a fixed support and a metal sphere of mass
m is attached to its free end. The mass is displaced vertically and then
released. The variation of the kinetic energy of the mass Ek, with time t of the
mass is shown in Fig. 5.2 below.

Fig. 5.2

[Turn over
For
Examiner’s
22 Use

(i) Determine the period T of the oscillation.

period =…………….. s [1]

(ii) Using the data from the kinetic energy-time graph in Fig. 5.2, sketch the
displacement-time graph of the oscillating mass for two complete oscillations
using the axes in Fig. 5.3 below.

On the axes, indicate and label clearly the amplitude and the period of the
oscillation.
[2]

Fig. 5.3
For
23 Examiner’s
Use

(d) To investigate the effects of damping on the oscillations of the metal sphere in (c), an
electromagnet is placed near it, and light damping is observed.

(i) Explain what is meant by damping.

……………………………………………………………………………………………

……………………………………………………………………………………………

……..………………………………………………………………………………….[1]

(ii) Sketch on Fig. 5.2, the kinetic energy-time graph of the lightly damped
oscillating system.
[2]

(iii) Suggest how critical damping of the metal sphere’s oscillations may be
achieved using the electromagnet.

……………………………………………………………………………………………

……………………………………………………………………………………………

……..………………………………………………………………………………….[2]

[Turn over
For
Examiner’s
24 Use

6. (a) Fig. 6.1 below shows 2 coils X and Y wound on a soft iron core.

Fig. 6.1

(i) Magnetic flux links coils X and Y when there is a current in coil X.

1. What happens in coil Y when the magnetic flux in the soft iron core
changes?

……………………………………………………………………………………………

……………………………………………………………………………………………

………………………………………………………………………………………...[1]

2. State a law that justifies your answer in (i) (1).

……………………………………………………………………………………………

………………………………………………………………………………………...[1]

(ii) The output of coil Y is connected to a 12 V, 3 A light bulb. The input of coil X is
now connected to a 240 Vr.m.s 100 Hz mains supply.
(Assume that the efficiency of energy transfer between the two coils is 100%
and the bulb lights with full intensity.)

Calculate the current in coil X.

current in coil X =…………….A [2]


For
25 Examiner’s
Use

(iii) Sketch a graph to show how the power dissipated in the light bulb connected to
coil Y varies with time over one cycle of the alternating output voltage.
[2]

(b) Fig. 6.2 below shows the path of an -particle as it passes near the nucleus of a gold
atom.

Fig. 6.2

[Turn over
For
Examiner’s
26 Use

(i) Explain why the -particle was deflected as shown in Fig. 6.2.

……………………………………………………………………………………………

……………………………………………………………………………………………

………………………………………………………………………………………...[2]

(ii) Indicate on the diagram the direction of the electric force acting on the
-particle.
[1]

(c) Xenon-139 has a half-life of 41 s and is generated at a constant rate during the
fission of a specific sample of Uranium-235. The number of Xenon-139 nuclei in the
sample increases initially and finally becomes constant.

(i) Explain the meaning of the following terms:

1. half-life,

……………………………………………………………………………………………
……………………………………………………………………………………………
………………………………………………………………………………………...[1]

2. fission.

……………………………………………………………………………………………
……………………………………………………………………………………………
………………………………………………………………………………………...[1]

(ii) Suggest a reason why the number of Xenon-139 nuclei in the sample becomes
constant.

……………………………………………………………………………………………
……………………………………………………………………………………………
………………………………………………………………………………………...[1]
For
27 Examiner’s
Use

(iii) The activity of Xenon-139 is 3.4  108 Bq when the number of Xenon-139
nuclei has reached a constant.

Calculate
1. the number of Xenon-139 nuclei present in the sample.

number of Xenon-139 nuclei present =……………..[2]

2. the mass of Xenon-139 in the sample.

mass of Xenon-139 =………………kg [2]

[Turn over
For
Examiner’s
28 Use

(d) The energy released in the fission reaction of Uranium-235 occurs partly as kinetic
energy of the fission products (167 MeV) and of the neutrons (5 MeV).

In a nuclear power station, 25% of the energy of the fission products is converted
into electrical energy. The number of uranium nuclei in 1.0 kg of Uranium-235 is
2.56  1024 .

(i) Calculate the electrical energy generated from the fission of 1.0 kg of
Uranium-235.

electrical energy generated =…………….. J [2]

(ii) Calculate the average power output of the power station if the duration of the
fission reaction of Uranium-235 is 24 hours.

average power output =………………. MW [2]


For
29 Examiner’s
Use

[Turn over
For
Examiner’s
30 Use

7. (a) Explain clearly each of the following observations:

(i) Light waves seem to travel only in straight lines while sound waves and water
waves can go around corners.

……………………………………………………………………………………………

……………………………………………………………………………………………

………………………………………………………………………………………...[2]

(ii) Sound waves cannot be polarized but radio waves can.

……………………………………………………………………………………………

……………………………………………………………………………………………

………………………………………………………………………………………...[1]

(b) Fig. 7.1 below shows some of the possible energy levels of an electron orbiting
inside a mercury atom.

The lowest possible energy level is Level 1. The diagram below is not drawn to
scale.

Fig. 7.1
For
31 Examiner’s
Use

(i) Explain how Fig. 7.1 can be used to account for the emission line spectrum.

……………………………………………………………………………………………

……………………………………………………………………………………………

………………………………………………………………………………………......

……………………………………………………………………………………………

……………………………………………………………………………………………

………………………………………………………………………………………...[3]

(ii) Cool mercury vapour is bombarded with a stream of electrons that have been
accelerated from rest through a potential difference of 7.3 V.

1. Determine the kinetic energy of an electron inside the stream.

kinetic energy of electron =………………J [2]

2. Calculate the frequency of an emitted photon when an electron falls


from Level 2 to Level 1.

frequency of emitted photon =………………. Hz [2]


[Turn over
For
Examiner’s
32 Use

3. State and explain whether photons with frequency in (b) (ii) (2) would
be emitted if electrons with energy of 4.15 eV collide with the mercury
atoms.

………………………………………………………………………………………......

……………………………………………………………………………………………

……………………………………………………………………………………………

………………………………………………………………………………………...[2]

4. Draw on Fig. 7.1 the number of possible transitions when the mercury
atoms are bombarded by electrons with energy of 7.6 eV.
[2]

(c) A particle of mass m and kinetic energy E has a de Broglie wavelength .

Show that the expression for de Broglie wavelength  in terms of m and E is given by
h

2mE
[2]

(d) An electron travels at 0.5 times the speed of light.


For
33 Examiner’s
Use

(i) Calculate its de Broglie wavelength.

de Broglie wavelength =…………………. m [2]

(ii) Comment and explain what is observed if such an electron beam is passed
through a thin film of crystalline material.

………………………………………………………………………………………......

……………………………………………………………………………………………

……………………………………………………………………………………………

………………………………………………………………………………………...[2]

END OF PAPER

[Turn over
Millennia Institute
PU3 H2 Physics
Preliminary Examination II
Mark Scheme

Paper 1

1 C 11 D 21 C 31 D
2 D 12 D 22 C 32 B
3 B 13 A 23 A 33 D
4 B 14 A 24 A 34 B
5 A 15 C 25 A 35 D
6 A 16 D 26 D 36 B
7 A 17 A 27 C 37 D
8 C 18 B 28 A 38 A
9 D 19 A 29 D 39 B
10 C 20 D 30 A 40 C

QN ANS SOLUTION
1 C -
2 D d 2
M h
4
4M

 d 2h
 m d h


m
2
d

h
 3  2 1  2  7%
3 B v u 62
a   0.2
t 20
  
v  u 2  2as  22  2 0.2 150  8 m s-1

4 B In order to cause turning effect, forces cannot act on same


point
5 A Tx  T sin  80
80
T
sin
Ty  T cos   50
50
T
cos 
80 50

sin cos 
8
  tan1
5
6 A Common a

TA  mW  mx  mY  mz a 

1
7 A m1v1  m2v 2  0
60v1  0.50(50)  0
v1  0.42 m s-1
8 C 
W  GPE  80 1.00  80 
KEf  KEi  GPE  150  80  70 J

9 D B not correct as even if faces R and S have same area, face R


will still experience a larger force due to the higher pressure at a
greater depth.

10 C 2
m V  5.0 
t

t
 1.02 
 2 
18  360.5  
p m
F
t

t
 
v  360.5 18  6500 N

11 D P  Fv
25000
v  42 m s-1
600

12 D Let F be gravitational force on surface and F’ is force 144 km


above the surface.

GMm GMm
F 
r2 5760 2
GMm GMm
F'  
r2 5904 2
F
F'  2
 0.95F
 5904 
 5760 

13 A Centripetal force of Moon is provided by gravitational force on


Moon due to Earth.

14 A Move from Y to X, object loses GPE, and hence positive work is


done by the G field  negative work done by external agent.

15 C
v  w a2  x2
Hence circular.

2
16 D 1
KEmax  mv002  E
2
2
3E 1 1 3v
 mv 2  m 0
4 2 2 4
2
3v
v2  0
4
x2
v   x02  x 2   x0 1
x02
3v 02  x2 
 v 02  1 2 
4  x0 
3 x2
 1 2
4 x0
x0 A
x 
2 2

17 A Internal energy = kinetic energy + potential energy


Temp increase = KE increase

18 B m  85  10  75
W  ml f
 
300 60  75l f
l f  240 J g-1

19 A Visible: 0.4 - 0.7 m  0.5 m is visible


Hence: 0.05 m is UV
5 m and m is IR.

20 D 1st to 6th node is 2.5 wavelengths


Hence wavelength is 12.0 cm

21 C d sin  n
n 2
d 
sin 
sin
2

22 C Electrical force same magnitude as gravitational force.


Hence C and not B. B assumes that there is horizontal velocity,
hence incorrect.

23 A Electric field strength is proportional to the rate of change of


electric potential with respect to distance. Close to P and Q,
lines become closer, hence increased E.

3
24 A VP  VQ  0
k 4   k 6  0
x 100  x
4 6
 
x 100  x
400  4x  6x
x  40 mm

25 A V
R
I

26 D Once filament is broken, circuit is open, and all the potential is


dropped across X. Hence Y is zero.

27 C Diode is in reverse bias, hence infinite resistance. Hence it


behaves like an open circuit, and therefore potential across it is
equal to the cell’s emf.

28 A Potential divider
Light incident on LDR, its resistance would drop.
Hence more potential dropped across XY

Shifting P towards X increases pd across bulb. Hence brighter.

29 D F  BILcos

30 A Flemming’s LHR. B-field at R is upwards. Hence force is out of


page

31 D Max induced e.m.f. when rate of change of B and hence flux


linkage wrt time is highest. Hence 4 ms.

32 B 1. Flux density increases, hence increase in flux linkage


2. No change in flux density, hence no change in flux
linkage
3. Current in coil 2 increases, increasing flux density and
hence changing flux linkage.

33 D P0
P by definition.
2

34 B T T

Vrms 
0
V 2 dt


0
Vo2 sin2  t dt

Vo
T T 2
After rectification,
1 T 2 2
V sin  t dt 
Vrms  2  0 o 

1 Vo Vo 100
   50 V
T 2 2 2 2

4
35 D hc
E  hf 

hc
E n  n hc
P  
t t t 
n p

t hc

36 B Classical theory suggests that there is a time lag. If intensity is


lower, time lag would be longer.

37 D In p-type, holes are majority carriers but electrons are present


as well.

38 A Conduction band above valence band (last fully occupied band).


Y is metal as it has an occupied conduction band.

39 B A is wrong as neutron is represented by


1
n , and not 11n .
0

40 C 3
N  N 7N0
N  1  0   1 0 
 2 8 8

5
Paper 2

1a Scalar: Magnitude and no direction (mass) B1

Vector: Magnitude and direction (weight) B1

 
v v v v
1b V  B  A  B   A
x : Ax  Bx  10 sin 45  5 sin35  4.2
y : Ay  By  10 cos 45  5 cos35  11.2 M1

V  4.22  11.22  11.9


11.9
a  0.40
30
4.2
  tan1  20.6 M1
11.2

Hence 0.40 m s-2 at bearing of 201 A1

2a Velocity changes by 10 m s-1 per second. B1


[Do not accept definition of acceleration.]

2bi sx 300 C1
t   0.333 s
ux 900
2bii No air resistance. B1

2biii 1 2
s y  uy t  gt
2
sy  0 
1
2
 
9.81 0.333 2 M1

sy  0.544 m A1

2biv v x2  ux2  2as


M1
0  9002  2a 4 
a  101000 m s-2 A1

2b Lower B1

3ai During melting, molecules move further apart from lattice B1


structure to clusters of molecules

Energy input is used to gain PE and not change KE, hence T B1


constant

3aii During boiling, molecules escape from liquid and escape into B1
surrounding atmosphere.

Energy input required to gain PE and also to overcome B1


atmospheric pressure

6
3b V T
V1 T1
 M1
V2 T2
0.00825 T
 1 A1
0.00424 273
T1  531 K
3ci T    273.15 (Do not accept other values) B1

3cii False. Need to measure on absolute scale, ie. thermodynamic B1


scale. In that case, difference is not 2 times.

4a Boat has greater volume compared to solid block (more empty M1


space within)

Hence displaces more water and therefore achieves sufficient A1


upthrust to support its weight.

4bi  
T  W  mg  0.200 9.81  1.96 N C1

4bii m 0.200
V   2.5  105 m3 C1
 8000
4biii T U W
T W U M1

   
T  1.96  2.5  105 800 9.81  1.76 N A1

4c B1:
Constant
gradient
up till h =
x

B1: no
increase
thereafter

5a The magnetic flux density at a point is the force per unit length B1
of conductor per unit current carried

placed at that point at right angles to the field. B1

5bi Current is moving in each coil of the spring creates a magnetic B1


field.

The magnetic field of one coil interacts with the magnetic field B1
of adjacent coil resulting in magnetic forces between coils.

7
5bii Current in each coil flows in the same direction. The spring is B1
compressed due to attractive magnetic forces.

5ci Into the paper B1

5cii Larger radius of curvature – above the given path. B1

5di FB =Bev M1

FB = (8.4 x 10-3 )(1.60 x 10-19 ) (3.4 x 107) = 4.57 X 10-14 N A1

5dii Work done on electron = 0 since magnetic is perpendicular to B1


displacement.
No change in KE of electron

6a 1. Atoms must be in the metastable state to bring about B1


stimulated emission of
coherent monochromatic laser.

2. Atoms must attain population inversion in order to cause B1


multiplication of
stimulated emission.

3. The emitted photons must be confined in the system long B1


enough to allow them
to stimulate further emission from other excited atoms.

6b

B1 each
(max 3)
Total 3
marks

7ai When an electron collides with one of the target atoms, it may B1
lose an amount of energy that corresponds to the energy of an
X-ray photon.

The electron may continue to lose energy in a series of B1


collisions with other atoms, thereby giving off X-ray photons of
different energies. Since different X-ray photons correspond to
different wavelengths, the continuous spectrum is thus formed.

7aii Min wavelength corresponds to max frequency, hence max loss B1


of energy due to most energetic electron losing all its KE.

7aiii Electron from ground state is excited and moves up to next B1


immediate state before dropping back down and releasing a
photon of wavelength corresponding to K

7bi For the energy of the photon for the K  -line,

8
E  E 2  E1
M1
k Z  1  k Z  12 
2
E     
22  12 
3k Z  1
2
hf 
4
3k Z  1
2
f 
4h
f  C Z  1 A1

3k
where C  and is a constant.
4h

7bii Best fit line B1

7biii From the graph,


gradient  5.0  10 7 (  0.2  10 7 ) M1

1
Therefore, the constant C is 5.0  10 Hz . 7 2 A1

7biv Since f  C Z  1 ,
f  5.0  10 7 29  1
f  1.4  10 9
C1
c
Since   ,
f
3.0  10 8


1.4  10 9
2

10
  1.5  10 m C1

7ci The first two peaks are the 1st order maxima and the next two
peaks are the 2nd order maxima for the two wavelengths of the
X-ray beam.

Using 2d sin  m ,

2  0.94  10 9  sin 0.8  1 1 C1


1  26 pm

2  0.94  10 9  sin 2.4  2  2 C1


2  39 pm

7cii Using the diffraction grating expression d sin  n , and


considering the 1st order maxima,

3000  10 9  sin  1 0.1 10 9 M1

9
  0.0019

The 1st order maxima is too close to the central bright fringe and A1
so is unable to discriminate between different wavelengths in
the X-ray region.

Purpose
Experiment is to investigate how the surface temperature of the glass envelope of a
filament lamp varies with the electrical power delivered to the lamp.

1. Basic Procedure
BP1
Switch on a.c. supply, measure current and voltage, measure temperature of lamp
surface.

2. Diagram

Lamp, a.c. power supply, voltmeter, ammeter, variable resistor, thermocouple on D1


surface of
lamp. D2
Correct setup

3. Procedure

1. Switch on the power supply


2. Measure the current and voltage across the lamp with the ammeter and voltmeter. M1
3. Power supplied to lamp P = V/I M2
4. Measure the surface temperature with a thermocouple. M3
5. Repeat steps 1-4 for different values of V by varying the variable resistor M4
6. Thermocouple is placed at the same part of the lamp. (any one 5 / 6) CV1
OR The room temperature is kept constant.

Safety
1. Do not touch the surface of the bulb FS1
2. Switch off bulb before adjusting the thermocouple FS2
3. Use glove to protect burns from lamp surface.

Improvement
1. Temperature reading take at various points on the surface. FI1

10
2. Tape junction of thermocouple on to the surface to ensure proper thermal contact. FI2

11
Paper 3
1a W = 80 (9.81) = 780 N C1

1bi Weight acting downwards from centre of object Both


presen
Normal force acting upwards from base of object t: B1

1bii F N W
a  M1
m m
N  780
2.5 
80
N  980 N A1

1biii F N W
a 
m m
1.5W  W
a C1
m
a  0.5g  4.9 m s-2

1c Correc
tF
values:
B1

Correc
t
timing:
B1

1di W Fs
P  M1
t t

P
 
8000  1 103 250
60
P  37.5 kW A1

1dii Eo
 0.75
Ei
37.5
 0.75
Ei
C1
E i  50.0 kW

2a mv 2 B1
F
r

12
2bi GPE  KE M1
1
mgh  mv 2
2
 
v  2gh  2 9.81 0.60 
A1
v  3.4 m s-1

2bii 0.60 m B1

2biii mv 2
At top, v  0  F  0
r
M1
F  N W  0  N  0

If contact force, N is less than zero, car breaks contact from track A1
and drops

2biv Min N = 0
mv 2
N  0& F  N W 
r
mv 2
W 
r
Wr 1
 mv 2 M1
2 2

GPE  GPE top  KE top

 
mg 0.6  mg 2r   Wr
2
5r
 0.6
2
r  0.24 m A1

2c GPE  EPE M1
1
mgh  kx 2
2

 
0.5 9.81 0.60   21 150x 2

x  0.20 m A1

3ai The potential difference between two points is the amount of B1


electrical energy converted into other forms of energy

per unit charge moved across the points. B1

3aii E = V + Ir  V = - Ir + E  y = mx + c M1
1
The y-intercept of the graph gives the e.m.f. E of the battery = 6 V A1

3aii From graph when I = 1.2 A, V = 4.2 V M1

13
2
P = I V  P 1.2 x 4.2 = 5.04 W A1

3aii E = V + Ir M1
3
 6 = 4.2 + 1.2 r

 r = (6 - 4.2)/1.2 = 1.5 Ω A1

3b Current in both branches the same. Pd across c and b is higher (I x M1


2Ω) compared to across d and b (I x 1Ω).

Potential at c higher than potential at d. The current will flow from c A1


to d.

4a The superposition of two waves meeting at a point in space to give a B1


resultant wave where

amplitude is given by the Principle of Superposition. B1

4bi Alternating bright and dark fringes B1

Bright fringes decrease in intensity as order increases B1

4bii Diffraction occurs at the slits where the beams spread out and B1
overlap.

Interference occurs when the 2 waves meet at different positions on B1


the screen. Constructive interference occurs if both waves are in
phase, and destructive if out of phase.

4biii

ax

 
0.5  103 2.5  103  M1

D 2.0
  625 nm A1

4c Constructive interference results in twice of original amplitude and B1


destructive results in zero amplitude.

Hence better contrast for observation B1

5a Oscillatory motion in which the acceleration is proportional to the B1


displacement from the equilibrium position and opposite direction to
the displacement.

Sea waves/ Tides B1

5bi   t B1

5bii ST  r sin M1

ST  r sin  t A1
5biii The shadow moves up and down about S. B1

14
ST  y
y  r sin  t
v  r  cos  t
a  r  2 sin  t   2 y M1
Hence SHM

5biv 2

T
2 2
T   1.57 s C1
 4

5bv v  v 0  wr M1

 
1
v  4 15.0  102  0.60 m s-1 A1

5bv a   2 x0 M1
2

a  4.0 0.15 
a  2.4 m s-2 A1

5ci T = 0.200 s B1

5cii Cosine curve starts from positive amplitude B1

T = 0.200 s B1

5di Damping is where the amplitude of oscillations gradually decreases B1


due to resistive forces and energy is lost.

5dii Same period as before. B1

Decreasing amplitude in each loop B1

5diii Increase the magnitude of the current B1

Until the mass does not oscillate but quickly returns to its equilibrium B1
position when displaced.

6ai1 An e.m.f. is induced in coil Y. B1

6ai2 Faraday's law states that the induced e.m.f. is equal to the rate of B1
change of flux linking the coil.

6aii Fission is a process in which a heavy nucleus split into 2 lighter


nuclei roughly equal mass
M1

A1

15
6aiii
2 power cycles for a cycle of voltage B1

Positive sinusoidal variation with zero to max power B1

6bi Both the -particle and nucleus contains protons which are positively B1
charged.

The deflection is due to repulsion between like charges B1

6bii Force vector from centre of gold nucleus to path. B1

6ci1 The half-life of a radioactive isotope is the time taken for half of the B1
unstable nuclei in the sample to decay.

6ci2 Fission is a process in which a heavy nucleus split into 2 lighter B1


nuclei roughly equal mass

6cii When the rate of production Xenon - 39 nuclei in the sample from B1
the Uranium-235 fission process is equal to its rate of decay of
Uranium-235 nuclei in the sample.

6ciii A 3.4 x 108


1 A  N  N =  M1
 ln 2
t1/2
3.4 x 108 x 41
  2.01x1010 A1
ln 2
6ciii N N M1
2 m x molar mass  m = x 0.139 kg mol1
NA NA
2.01 x 1010
 x 0.139  4.6 x 1015 kg
6.02 x 1023 A1

16
6di

M1

A1

6dii M1

A1

7ai Sound waves and water waves have wavelengths comparable to the B1
size of the obstacles in their path. Sound and water waves and
undergo diffraction around corners easily.

Light waves have wavelengths that are too small compared to the
dimensions of the obstacles, and will not undergo diffraction. Thus B1
light waves will appear to travel only in straight lines.

7aii Sound waves are longitudinal whereas radio waves are transverse. B1
Only transverse waves can be polarized.

7bi When the electron in a particular excited state falls to a lower energy
state, it loses energy by emitting a photon of energy. B1

Only photons of specific energies that are equal to the difference


between two energy levels of the will be emitted. B1

Differences in energy levels E  hf , the frequencies corresponding


to these photon energies give rise to an emission line spectrum.
B1

7bii W  qV  E=1.6 x 1019 x 7.6 =1.22 x 1018 J M1


1 A1

7bii E  E2  E1  5.72  (10.38)  4.66 eV M1


2
E 4.66 x 1.6 x 1019
E=hf  f=  34
 1.12 x 1015 Hz
h 6.63 x 10 A1

7bii Energy of electrons 4.15 eV is not sufficient to cause electron in the M1


3 lowest level to move to next higher Energy Level 2 (E = 4.66 eV)

Not possible for photons of this frequency to be emitted.


A1
7bii Working Not required 3

17
4 Energy of electrons = 7.3 eV correct
Possible transitions are: : B2
E3 – E1 > - 3.71 eV to - 10.38 eV E = 6.67 eV
E2 – E1 > - 5.72 eV to -10.38 eV E = 4.66 eV 1 or 2
E3 – E2 > - 3.71 eV to - 5.72 eV E = 2.01 eV correct
: B1
Therefore, the number of possible transitions (or frequencies) is 3.

7c 1 2
Energy of particle E  mv
2
p
momentum of particle p  mv (Sub. v  )
m
1 p p2
E  m( ) 2  E  p= 2mE M1
2 m 2m
h h
de Broglie wavelength  =  = A1
p 2mE

7di 1 M1
KE of electron E= x (9.11 x 1031 ) x (0.5 x 3 x 108 ) 2  1.02 x 1014 J
2
h 6.63 x 1034
   4.86 x A1
1012 m
31 14
2mE 2 x (9.11 x 10 ) x 1.02 x 10

7dii The de Broglie wavelength of the electron is similar in size as the B1


atomic spacing between atoms inside the crystal.

The spacing between atoms acts as a diffraction grating and B1


diffraction effect of electrons will be observed.

18
Preliminary Examination Meridian Junior College 24 September
2010
JC2 H2 Physics 2010

MERIDIAN JUNIOR COLLEGE


Preliminary Examination
Higher 2

___________________________________________________________________

H2 Physics 9646/1
Paper 1 24 September 2010

1 hour 15 mins
___________________________________________________________________
READ THESE INSTRUCTIONS FIRST

Class Reg Number

Candidate Name _____________________________

Do not open this booklet until you are told to do so.

There are forty questions in this section. Answer all questions. For each question, there
are four possible answers A, B, C and D. Choose the one you consider correct and
record your choice in soft pencil on the Optical Mark Sheet (OMS).

Read very carefully the instructions on the OMS.

Write your name and class in the spaces provided on the OMS.

Shade your Index Number column using the following format:

1) first 2 digits is your index number in class (e.g. 5th student is shaded as “05”);
2) ignore the last row of alphabets.

This document consists of 17 printed pages


Preliminary Examination Meridian Junior College 24 September 2010
JC2 H2 Physics 2010

DATA AND FORMULAE

Data
speed of light in free space c = 3.00 x 108 m s-1
permeability of free space μo = 4π x 10-7 H m-1
permittivity of free space ε0 = 8.85 x 10-12 F m-1
= (1/(36π)) x 10-9 F m-1
elementary charge e = 1.60 x 10-19 C
the Planck constant h = 6.63 x 10-34 J s
unified atomic mass constant u = 1.66 x 10-27 kg
rest mass of electron me = 9.11 x 10-31 kg
rest mass of proton mp = 1.67 x 10-27 kg
molar gas constant R = 8.31 J K-1 mol-1
the Avogadro constant NA = 6.02 x 1023 mol-1
the Boltzmann constant k = 1.38 x 10-23 J K-1
gravitational constant G = 6.67 x 10-11 N m2 kg-2
acceleration of free fall g = 9.81 m s-2

Formulae
uniformly accelerated motion 1
s = ut + at2
2
v2 = u2 + 2as
work done on/by a gas W = p ΔV
hydrostatic pressure p = ρgh
gravitational potential φ = -Gm/r
displacement of particle in s.h.m. x = xo sin ωt
velocity of particle in s.h.m. v = vo cos ωt

± ω xo - x
2 2
=
resistors in series R = R1 + R2 + …
resistors in parallel 1/R = 1/R1 + 1/R2 + …
electric potential V = Q/4πεor
alternating current/voltage x = xo sin ωt
transmission coefficient T = exp(-2kd)

8π m(U − E )
2

where k = 2
h
radioactive decay x = xo exp(-λt )
decay constant 0.693
λ =
t1
2

2
Preliminary Examination Meridian Junior College 24 September 2010
JC2 H2 Physics 2010

Answer all 40 questions in this paper and shade your answers on the answer sheet provided.

1 The length of a rectangle is given as L ± l and its width as W ± w. What is the uncertainty
in its area?

A l+w
B Lw + W l
C Ll + W w
l w
D +
L W

2 For which quantity is the magnitude a reasonable estimate?

A mass of an atom 500 pg


B wavelength of green light 500 nm
C frequency of a radio wave 500 μHz
D the acceleration due to free fall 981 mm s-2

3 A housewife released a bag of rubbish weighing 54 N from rest into a refuse chute from
her unit in a HDB flat. The rubbish passes through two speed detectors at different
locations and the readings shown in the detectors are 14.7 m s-1 and 49.1 m s-1
respectively.

Neglecting the effect of air resistance, the distance between the two speed detectors is

A 20.3 m
B 31.9 m
C 71.7 m
D 112 m

4 When a ball is thrown upwards at an angle to the horizontal with an initial speed.
Assuming that air resistance is not negligible, which of the following statement is
incorrect?

A The path of the ball is asymmetrical about the highest point.


B Horizontal range of the ball is shorter than the case with negligible air resistance.
C The maximum height reached by the ball is smaller than the case with negligible air
resistance.
D The time taken for the flight up to the highest point is longer than the time taken for the
flight down.

3
Preliminary Examination Meridian Junior College 24 September 2010
JC2 H2 Physics 2010

5 A car of weight Wc is driven across a uniform bridge of length l and weight Wb. The bridge is
supported by two ropes having tensions T1 and T2 when the car is a distance x from the rope
on the left as shown in the figure below.

T1 T2

x
Wc Wb

Which of the following expressions for the tensions T1 and T2 is correct?

Tension T1 Tension T2
Wb Wc x Wb x
A + + Wc (1 − )
2 l 2 l
Wb Wc l Wb l
B + + Wc (1 − )
2 x 2 x
Wb Wc Wb Wc
C + +
2 2 2 2
Wb x Wb Wc x
D + Wc (1 − ) +
2 l 2 l

6 Three identical stationary discs P, Q and R are placed in a line on a horizontal, flat,
frictionless surface. Disc P is projected straight towards disc Q.

P Q R

If all consequent collisions are perfectly elastic, predict the final motion of the three discs.

P Q R
A moving left moving left moving right
B moving left stationary moving right
C stationary stationary moving right
D moving right moving right moving right

4
Preliminary Examination Meridian Junior College 24 September 2010
JC2 H2 Physics 2010

7 In the Pixar movie, Up, an old man lifted his house using about 20000 helium balloons.
Assuming that the average volume of each balloon used is 0.17 m3, determine the weight
of the old man’s house.
(density of air = 1.2 kg m-3, density of helium = 0.18 kg m-3)

A 6000 N B 34000 N C 40000 N D 46000 N

8 A sphere of mass 3.00 kg rests on a frictionless slope inclined at 300 above the horizontal
as shown below.

The spring constant is 500 N m-1. Determine the compression of the spring.

Wall
300

A 7.67 mm B 29.4 mm C 34.3 mm D 51.0 mm

9 A small metal sphere of mass m is moving through a viscous liquid of height h. When it
reaches a constant downward velocity v, which of the following describes the changes
with time in the kinetic energy and gravitational potential energy of the sphere?

Kinetic Energy Gravitational Potential Energy


A constant and equal to ½mv2 decreases at a rate mgv
B constant and equal to ½mv2 decreases at a rate mgh
C increases at a rate ½mv2 decreases at a rate (½mv2 – mgv)
D increases at a rate mgv decreases at a rate (mgv – ½mv2)

10 Singapore is on the Equator. Seoul is at a latitude of 37.50 N as depicted in the diagram


below.
Seoul

37.50
Singapore

The centripetal acceleration at Singapore due to the Earth’s rotation about its axis is
asingapore . The centripetal acceleration at Seoul due to the Earth’s rotation about its axis is
asingapore
aseoul . What is the ratio ?
aseoul

A 0.79 B 1.00 C 1.26 D 9.81

5
Preliminary Examination Meridian Junior College 24 September 2010
JC2 H2 Physics 2010

11 Tarzan, whose mass is 80.0 kg, needs to swing across a river filled with crocodiles in
order to save Jane of mass 45.0 kg, at the middle of the river as shown in the diagram
below. He has to swing from a branch, which is assumed to be fixed in position, on a vine
of length 30.0 m, and initially making angle of 50o with the vertical.

50o 30.0 m

Tarzan swings towards Jane and grabs hold of her. Assuming that this is a completely
inelastic collision, determine their common speed just after the collision.

A 9.28 m s-1 B 12.4 m s-1 C 15.6 m s-1 D 19.5 m s-1

12 A particle of mass m performs vertical circular motion as show in the diagram below.
C

The following two graphs show the vertical and horizontal components of the velocity of
the particle along path ABC.

Calculate the centripetal acceleration at point C.

A zero B 4.91 m s-2 C 9.81 m s-2 D 22.1 m s-2

6
Preliminary Examination Meridian Junior College 24 September 2010
JC2 H2 Physics 2010

13 Mercury is 5.79 × 1010 m away from the Sun and it takes 0.241 earth years for Mercury to
make one revolution around the Sun. Neptune is 450 × 1010 m away from the Sun,
calculate the period of its orbit around the Sun.

A 2.12 years
B 18.7 years
C 165 years
D 330 years

14 The figure below shows the gravitational potential near the surface of the asteroid 951
Gaspra. Determine the mass of the asteroid.

A 1.00 × 1016 kg
B 2.00 × 1016 kg
C 3.00 × 1016 kg
D 4.00 × 1016 kg

15 A 0.10 kg mass hanging from a light helical spring produces an equilibrium extension of
0.10 m. The mass is pulled vertically downwards by a distance of 0.020 m and then
released.

Taking g as 10 m s-2, the equation relating the displacement x of the mass from its
equilibrium position and the time t after release is

A x = 0.12 sin (10t )


B x = 0.10 cos(0.20π t )
C x = 0.020 cos ( 0.10t )
D x = 0.020 cos (10t )

7
Preliminary Examination Meridian Junior College 24 September 2010
JC2 H2 Physics 2010

16 Which graph correctly shows the variation with time t of kinetic energy Ek of an object
undergoing simple harmonic motion of period T?

17 A microwave source S is placed in front of a detector D, and a metal reflecting screen R is


placed beyond D such that its plane is perpendicular to the line joining S to D. As the
detector is moved slowly away from the source, it registers a series of maxima and
minima.

S D R

It is observed that the detector moved through a distance of 5.6 cm between the first and
fifth minimum. What is the frequency of the microwaves in GHz?

A 5.4
B 10.7
C 13.4
D 27.5

8
Preliminary Examination Meridian Junior College 24 September 2010
JC2 H2 Physics 2010

18 In a two-slit interference experiment, one slit transmits twice the amplitude of the other slit.
If the maximum intensity of the interference pattern is Io, the minimum intensity in the
pattern would be

A zero
B Io/9
C Io/4
D Io/2

19 Two sources of waves are said to be coherent if

A they are in phase.


B they have a constant phase difference.
C they have the same frequency and amplitude.
D they can interfere constructively.

20 A single traveling light wave in vacuum is able to

A carry momentum.
B create a standing wave.
C propagate at any velocity.
D create interference patterns.

21 Which statement about internal energy is correct?

A The internal energy of a system can be increased without transfer of energy by


heating.
B The internal energy of a system is the sum of the kinetic energies of the molecules.
C When the internal energy of a system is increased, its temperature always rises.
D When two systems have the same internal energy, they must be at the same
temperature.

22 An ideal gas is contained in a cylinder with a movable piston. At pressure p, volume V and
temperature T, it has Nv molecules per unit volume. If the pressure of the gas is changed
to 0.50p, and the temperature to 2.0T, the number of molecules per unit volume becomes

A 0.25 Nv
B 0.50 Nv
C 1.0 Nv
D 4.0 Nv

9
Preliminary Examination Meridian Junior College 24 September 2010
JC2 H2 Physics 2010

23 A negatively-charged oil drop is held stationary between two horizontal, charged metal
plates, the upper plate being positive.

+
Oil drop

The oil-drop then acquires an additional negative charge. In order to keep the oil-drop
stationary, what change should be made?

A Move the plate closer together.


B Reverse the charges on the plates.
C Increase the electric field strength between the plates.
D Decrease the potential difference between the plates.

24 Two point charges of -5 μC and +5 μC, are situated at points P and Q respectively as
shown below. X lies midway between P and Q while Y is at the same vertical position as X
but displaced to the right.

X Y

At point X At point Y
Electric Electric
Electric field Electric field
potential potential
A towards Q zero downwards zero
B towards Q negative downwards negative
C towards P zero upwards zero
D towards P negative upwards negative

10
Preliminary Examination Meridian Junior College 24 September 2010
JC2 H2 Physics 2010

25 In the circuit diagram below, D is an ideal diode. The voltage supply has negligible
internal resistance and the voltmeter reads 12 V.
2.0 Ω

V
Voltage
supply 2.0 Ω
2.0 Ω

If the connections to the terminals of the voltage supply are reversed, the voltmeter
reading would be

A 6.0 V B 8.0 V C 9.0 V D 18 V

26 The circuit shown in Fig. 1 may be used to determine the internal resistance of a battery.
An oscilloscope is connected across the battery as shown. Fig. 2 represents the screen of
the oscilloscope.


Fig. 1 Fig. 2

The time base of the oscilloscope is switched off throughout the experiment.
Initially the switches S1 and S2 are both open. Under these conditions, the spot on the
oscilloscope screen is at A.

Switch S1 is now closed, with S2 remaining open. The spot moves to B.


Switch S1 is kept closed and S2 is also closed. The spot moves to C.
The vertical sensitivity of the oscilloscope is 0.50 V per division.

Calculate the internal resistance of the battery.

A 0.24 Ω B 2.3 Ω C 14.0 Ω D 16.4 Ω

11
Preliminary Examination Meridian Junior College 24 September 2010
JC2 H2 Physics 2010

27 A row of 30 decorative lights, connected in series, is connected to a mains transformer.


When the supply is switched on, the lights do not work. The owner uses a voltmeter to test
the circuit. When the voltmeter is connected across the fifth bulb in the row, a reading of
zero is obtained.

Which of the following scenarios described is not possible?

A Only the filament of the fifth bulb has broken.


B The fuse in the mains transformer has blown.
C The filament of at least one of the other bulbs has broken.
D There is a break in the wire from the supply to the transformer.

28 The diagram shows a network of three resistors. Two of these marked R, are identical.
The other one has a resistance of 5.0 Ω.
●Y


R
5.0
X● ●

R ●

●Z
The resistance between Y and Z is found to be 2.5 Ω.
Determine the resistance between X and Y.

A 1.00 Ω B 1.9 Ω C 2.5 Ω D 4.2 Ω

12
Preliminary Examination Meridian Junior College 24 September 2010
JC2 H2 Physics 2010

29 In the diagram below, P is a horizontal circular coil of wire carrying a steady current I1 . A
conducting rod, which is free to move, is supported by 2 fixed horizontal parallel rails TQ
and SR which are perpendicular to the length of the conducting rod and carry a constant
current I 2 as shown in the diagram below.

P
Q R
I2
I2

I2

T S

I1
The conducting rod will

A move towards TS with increasing speed.


B move towards QR with increasing speed.
C move towards QR with decreasing speed.
D be lifted off the horizontal parallel rails momentarily.

30 The wire AC is free to move vertically while the wire BD is fixed. Suppose that both wires
carry equal currents of 100 A in opposite directions and that the mass per unit length for
each wire is 20.0 g m-1. Calculate the height which the wire AC is above wire BD. (You
may assume that the magnetic field, B at a distance r from a wire carrying current I to be
μo I
B= )
2π r

A C

I = 100 A

B D

I = 100 A

A zero, wires AC and BD are attracted to each other.


B 0.102 cm
C 1.02 cm
D 10.2 cm

13
Preliminary Examination Meridian Junior College 24 September 2010
JC2 H2 Physics 2010

31 A short bar magnet passes at a steady speed through a long solenoid. A galvanometer is
connected across the solenoid.

S N

Which graph best represents the variation of the current I of the galvanometer with time t?

I A II C
B
A

00 tt 00 tt

II C II
B D

00 tt 00 tt

32 A copper ring is suspended by a long, light rod pivoted at X so that it may swing as a
pendulum, as shown in the diagram below. An electromagnet is mounted so that the ring
passes over it as it swings.

K
The ring is set into oscillation with switch K open. What happens to the motion after switch
K has been closed?

A The periodic time will decrease.


B The oscillations will be heavily damped.
C The amplitude will increase because the ring is accelerated towards the magnet.
D The oscillations will continue at constant amplitude while the battery can supply
energy.

14
Preliminary Examination Meridian Junior College 24 September 2010
JC2 H2 Physics 2010

33 A sinusoidal potential difference V1 as shown in Fig. A, is applied across a resistor R and


produces heat at a mean rate W. What is the mean rate of heat produced when another
potential difference V2 as shown in Fig. B is applied across the same resistor?

V1 V2

Vo 2Vo

0
T time

-Vo 0
T time

Fig. A Fig. B

A ½W B 2W C 4W D 8W

34 An electric kettle has the following label:

Power : 2000 to 2400 W


Voltage : 220 to 240V
Frequency : 50 to 60 Hz

Which of the following is a probable expression of the current that passes through the
kettle when used in Singapore?

A I = 8.33 sin (315t)


B I = 10.9 sin (315t)
C I = 14.1 sin (375t)
D I = 16.0 sin (375t)

35 A potential barrier has a width W and potential height 6.0 MeV. The probability of a
3.0 MeV electron tunneling through it is 0.025.

Suppose that the potential height of the barrier is now doubled, what must the width of the
barrier be in order for a 3.0 MeV electron to have the same probability of transmission?

A 0.577 W B 0.707 W C 1.41 W D 1.73 W

36 The decay energy of a short-nuclear excited state is measured to be 150 keV ± 1%. What
is the shortest lifetime it can have?

A 3.5 × 10-40 s B 3.5 × 10-37 s C 5.3 × 10-35 s D 2.2 × 10-19 s

15
Preliminary Examination Meridian Junior College 24 September 2010
JC2 H2 Physics 2010

37 Which of the following statements about electron transitions between energy levels is
true?

A Stimulated emission describes a scenario when an atom get ‘excited’ from a lower
energy state E1 to a higher energy state E2 when one of its electron absorbs a photon
of energy E2 – E1.
B Spontaneous emission occurs more rapidly when the lifetime of the excited state is
long.
C Stimulated emission occurs more rapidly when the stimulating photon beam is of high
intensity.
D Spontaneous emission competes more strongly with stimulated emission in transitions
that emit red or infrared radiation.

38 A semiconductor device is connected in a series circuit with battery and resistance. A


current is found to pass through the circuit. When polarities of the battery are reversed the
current drops to zero. The device may be:

A pn junction
B intrinsic semiconductor
C p-type semiconductor
D n-type semiconductor

39 The radioactive isotope of iodine, 123I, is often used to test for overall thyroid function in
patients. The thyroid of an individual with hypothyroid condition will accumulate less
iodine than that of a normal individual.

An initial dose of 123I with an activity of 30 μCi was administered intravenously to a


patient. Twenty-four hours after injecting the radiopharmaceutical, the activity emanating
from the thyroid region is monitored and found to be 4 μCi.
123
What percentage of the injected I was concentrated in the thyroid
(Half-life of 123I = 13 hrs).

A 13% B 24% C 26% D 48%

16
Preliminary Examination Meridian Junior College 24 September 2010
JC2 H2 Physics 2010

40 The figure below shows the apparatus used to repeat the alpha-particle scattering
experiment. The detector measures the intensity of the alpha-radiation I at various
angular positions θ.
+90o

alpha gold
particles +170o foil
0o
θ
–170o

detector
–90o

Which of the graphs best represents the variation of I with θ?

Ι Ι

A B

θ θ
ο ο ο ο ο ο
−170 −90 0 +90 +170 −170 −90 0 +90 +170
ο ο ο ο

Ι Ι

C D

θ θ
ο ο ο ο ο ο
−170 −90 0 +90 +170 −170 −90 0 +90 +170
ο ο ο ο

End of Paper

17
MERIDIAN JUNIOR COLLEGE
Preliminary Examination
Higher 2

___________________________________________________________________

H2 Physics 9646/2
Paper 2 21 September 2010

1 hour 45 mins
___________________________________________________________________
READ THESE INSTRUCTIONS FIRST

Class Reg Number

Candidate Name _____________________________

This booklet contains 7 questions.

Do not open this booklet until you are told to do so.

Answer all questions.

Write your answers on this question booklet in the blanks provided.

Examiner’s Use

Section A
INFORMATION FOR CANDIDATES
Q1 /15
The number of marks is given in brackets [ ] at the end
of each question or part question. Marks will be Q2 /8
deducted if units are not stated where necessary or if
answers are not quoted to the appropriate number of Q3 /8
significant figures.
Q4 /6
All working for numerical answers must be shown. You
are reminded of the need for good English and clear Q5 /8
presentation of your answers.
Q6 /15

Section B

Q7 /12

Deductions

Total /72

This document consists of 21 printed pages


2
[Turn Over
Preliminary Examination Meridian Junior College 21 September 2010
JC2 H2 Physics 2010

DATA AND FORMULAE

Data
speed of light in free space c = 3.00 x 108 m s-1

permeability of free space μo = 4π x 10-7 H m-1


permittivity of free space ε0 = 8.85 x 10-12 F m-1
= (1/(36π)) x 10-9 F m-1
elementary charge e = 1.60 x 10-19 C
the Planck constant h = 6.63 x 10-34 J s
unified atomic mass constant u = 1.66 x 10-27 kg
rest mass of electron me = 9.11 x 10-31 kg
rest mass of proton mp = 1.67 x 10-27 kg
molar gas constant R = 8.31 J K-1 mol-1
the Avogadro constant NA = 6.02 x 1023 mol-1
the Boltzmann constant k = 1.38 x 10-23 J K-1
gravitational constant G = 6.67 x 10-11 N m2 kg-2
acceleration of free fall g = 9.81 m s-2

Formulae
uniformly accelerated motion 1
s = ut + at2
2
v2 = u2 + 2as
work done on/by a gas W = p ΔV
hydrostatic pressure p = ρgh
gravitational potential φ = -Gm/r
displacement of particle in s.h.m. x = xo sin ωt
velocity of particle in s.h.m. v = vo cos ωt

± ω xo - x
2 2
=
resistors in series R = R1 + R2 + …
resistors in parallel 1/R = 1/R1 + 1/R2 + …
electric potential V = Q/4πεor
alternating current/voltage x = xo sin ωt
transmission coefficient T = exp(-2kd)

8π m(U − E )
2

where k = 2
h
radioactive decay x = xo exp(-λt )
decay constant 0.693
λ = t1
2

3
Preliminary Examination Meridian Junior College 21 September 2010
JC2 H2 Physics 2010

Answer all the questions in the spaces provided.

1 (a) State the Principle of Superposition.

.............................................................................................................................................

.............................................................................................................................................

.............................................................................................................................................

.................................................................................................................................... [2]

(b) Two sinusoidal transverse waves W1 and W2, of the same type, are incident
simultaneously on a point P. The amplitude of W2 is the same as the amplitude of W1.
The frequency of W2 is half the frequency of W1.

At a certain instant (time t = 0) at P, both waves have zero displacement and then both
displacements increase in the same direction.

Fig. 1.1 is a graph of displacement at P against time for wave W1.

displacement
(wave W1)

time Fig. 1.1

displacement
(wave W2)

time Fig. 1.2

displacement
(resultant
wave)

time Fig. 1.3

(i) On Fig. 1.2, sketch a graph of displacement against time for wave W2. [3]

(ii) On Fig. 1.3, sketch a graph to show the resultant wave produced by the
superposition at P of waves W1 and W2. [3]

4
Preliminary Examination Meridian Junior College 21 September 2010
JC2 H2 Physics 2010

(iii) The frequency of wave W1 is 4.2 x 1015 Hz. Determine the frequency of the
resultant wave produced by the superposition of waves W1 and W2.

frequency = ………………. Hz [2]

(iv) Explain why it is incorrect to say that waves W1 and W2 are coherent.

...................................................................................................................................

.......................................................................................................................... [1]

(c) Blue light of wavelength 485.6 nm from a star is incident normally on a diffraction
grating. The light is diffracted into a number of beams as shown in Fig. 1.4.

second order

first order

zero order

first order

second order
grating
Fig. 1.4

The angular separation of the two second order beams is 45.7o. Calculate the
number of lines per millimeter on the grating.

Lines per millimetre = ………………. [4]

5
Preliminary Examination Meridian Junior College 21 September 2010
JC2 H2 Physics 2010

2 (a) Air near the Earth conducts electricity to a small extent. Because there is an electric field
directed towards the Earth’s surface, a small electric current exists in the atmosphere.
Measurements of the electric field near the Earth’s surface give an average of about
1.0 x 102 V m-1.

(i) Estimate the potential difference between the head and feet of an average adult.

potential difference = ………………. V [1]

(ii) The electric field near the Earth’s surface is as large as 1.0 x 102 V m-1, explain
why you do not experience an electric shock when you stand upright.

……………………………………………………………………………………................

……………………………………………………………………………………................

……………………………………………………………………………………................

……………………………………………………………………………………................

……………………………………………………………………………..............… [2]

(b) A photomultiplier tube is a device which has a common electrode (the photocathode) and
a number of other electrodes (the dynodes), which must be maintained at definite
potentials relative to the common electrode. A potential divider circuit may be used to
supply these potentials.

Fig. 2.1 illustrates a photomultiplier tube with a photocathode and six dynodes,
numbered 1 to 6. A potential divider of six resistors, each of resistance R, using a supply
voltage of 1050 V, is connected to the electrodes in the tube.

photomultiplier tube

1 2 3 4 5 6
photocathode dynodes

R R R R R R

potential divider

1050 V

Fig. 2.1

6
Preliminary Examination Meridian Junior College 21 September 2010
JC2 H2 Physics 2010

(i) For the situation where there is no electron current inside the photomultiplier tube,
determine the potential difference between dynode 4 and the photocathode.

p.d. between dynode 4 and the photocathode = …………………V [2]

(ii) A fault develops inside the photomultiplier tube causing a short circuit between
dynodes 3 and 5. Determine the new potential difference between dynode 4 and
the photocathode. Explain your reasoning.

new p.d. between dynode 4 and the photocathode = …………….. V

Explanation: …………..……………………………………………………………………

……………………………………………………………………………………................

……………………………………………………………………………………................

……………………………………………………………………………………....... [3]

7
Preliminary Examination Meridian Junior College 21 September 2010
JC2 H2 Physics 2010

3 From the power station, the voltage is stepped up to about 230 kV along the high voltage
cables before a series of step down transformers near homes are used to reduce the voltage
to 240 V at our homes. Fig. 3.1 shows the last iron-cored step down transformer before our
homes with the actual turns ratio indicated. The home 240 V output has a frequency of 50 Hz
is connected to a 58 Ω resistor of a home appliance.

iron-core

58 Ω
home

primary coil secondary coil

Fig. 3.1

(a) Determine the number of similar turns-ratio transformers required to step down from the
high voltage (230 kV) cables to the 240 V output used at homes. State one assumption
made.

Number of transformers = ..................

Assumption: ........................................................................................................................

...................................................................................................................................... [4]

(b) Explain the purpose of the iron core in the transformer.

.............................................................................................................................................

.............................................................................................................................................

.................................................................................................................................... [1]

8
Preliminary Examination Meridian Junior College 21 September 2010
JC2 H2 Physics 2010

(c) Determine the average power dissipated in the 58 Ω resistor of a home appliance.

Average power = …………….. W [2]

(d) Suggest a typical household appliance with such power rating.

Appliance is ……………..…………….. [1]

4 A p-n junction is formed between slices of p-type and n-type semiconductor material as shown
in Fig. 4.1

p-type n-type

Fig. 4.1

(a) On Fig. 4.1, draw an arrow indicating the direction of movement of holes when the two
slices are brought into contact. [1]

(b) Boron is used as a dopant in one of the semiconductor slices. State and explain whether
the addition of Boron creates a p-type or a n-type semiconductor.

…………………………………………………………………………………………...................

…………………………………………………………………………………………...................

…………………………………………………………………………………………...................

…………………………………………………………………………………………...................

…………………………………………………………………………………………...................

…………………………………………………………………………………………...................

……………………………………………………………………………………................. [3]

9
Preliminary Examination Meridian Junior College 21 September 2010
JC2 H2 Physics 2010

(c) State and explain what will happen to the depletion region when a battery is connected in
as shown in Fig. 4.2.

Fig. 4.2

…………………………………………………………………………………………...................

…………………………………………………………………………………………...................

…………………………………………………………………………………………...................

…………………………………………………………………………………………...................

……………………………………………………………………………………................. [2]

5 (a) Explain what is meant by binding energy.

…………………………………………………………………………………………...................

…………………………………………………………………………………………...................

……………………………………………………………………………………................. [1]

(b) Calculate the binding energy of a thorium nucleus 226


90 Th.
Given that
rest mass of 226
90 Th = 226.0249 u
rest mass of proton = 1.0073 u
rest mass of neutron = 1.0087 u

Binding energy = ……………… MeV [2]

10
Preliminary Examination Meridian Junior College 21 September 2010
JC2 H2 Physics 2010

(c) A thorium nucleus 226


90 Th originally at rest decays and forms a radium nucleus 222
88 Ra* and
an alpha particle as shown below. The radium nucleus 222
88 Ra* is in an excited state.

226
90 Th Æ 222
88 Ra* + 42 He
Given that
rest mass of 226
90 Th = 226.0249 u
rest mass of 222
88 Ra = 222.0154 u
rest mass of 4
2 He = 4.0026 u

(i) Calculate the kinetic energy of the radium nucleus if the alpha particle is emitted
with a kinetic energy of 2.38 MeV.

Kinetic energy of Ra = …………… MeV [3]

(ii) The excited radium nucleus, 222


Ra* further undergoes a gamma decay as shown below:
88

88 Ra + γ
Ra* Æ 222
222
88

Calculate the energy of the gamma ray.

Energy of the gamma ray = ……………… MeV [2]

11
Preliminary Examination Meridian Junior College 21 September 2010
JC2 H2 Physics 2010

6 Most Singapore buildings are built using a framework of concrete beams, slabs and columns.
The concrete columns need to carry both the ultimate vertical load, N and the ultimate bending
moment, M induced from the attached beam/s as shown in the 3-D pictorial diagram of
Fig. 6.1. In practice, the concrete columns are reinforced with steel bars.
N

Beam

Column

Fig. 6.1

To design for the steel bars in such columns, design charts are available from the British
Standard Structural Use of Concrete, BS8110.

12
Preliminary Examination Meridian Junior College 21 September 2010
JC2 H2 Physics 2010

In order to choose the correct design chart, the values of the following must match the design
details:

fcu = Ultimate crushing pressure limit of concrete


fy = Ultimate characteristic strength of steel bar
d
where d is the distance from the edge of the column to the centre of the steel bar that is
h
furthest from that edge and h is the larger dimension of the column

The cross section of a column is shown in Fig. 6.2: b = 200mm

h = 450 mm

d
h = 450 mm

Height of
column
32 mm cover= 40 mm
diameter steel
bar

b = 200 mm

Fig. 6.2 Fig. 6.3

Dimensions of the column are:


h = 450 mm (larger dimension of the column)
b = 200 mm
d = distance from the edge of the column to the centre of the steel bar that is furthest from that
edge.

(a) The chosen design details for the column above are:

fcu = 50 N mm-2
fy = 460 N mm-2
cover = 40 mm (from edge of column to edge of steel bar)

Assuming that 4 numbers of 32 mm diameter steel bars are to be used. Determine the
d
value of and hence explain why Chart No. 49 is appropriate to be used.
h

d
= ………………
h [1]
Explanation:

..............................................................................................................................................

..................................................................................................................................... [1]
13
Preliminary Examination Meridian Junior College 21 September 2010
JC2 H2 Physics 2010

N M
(b) The coordinates of the chart derived from the values of and can be used to
bh bh2
100 ASC
establish the corresponding value of such that ASC (the total required
bh
cross-sectional area of steel bars) can be calculated. It is necessary to provide enough
steel bars, with a total area more than ASC.

100 ASC
An example of reading off the value is:
bh

N
Value of = 32.5 N mm-2
bh
M
Value of 2
= 3.20 N mm-2
bh

From Chart No. 49, the corresponding coordinate is marked with a cross and labelled
100 ASC
‘A’. This coordinate corresponds to a value of 4 < < 5 . More specifically, the
bh
100 ASC
value of is 4.5. Using this value, ASC can then be calculated.
bh

Based on the same design details in (a), the loads carried by the column in Fig 6.2 are:
M = Ultimate bending moment = 91.2 kN m
N = Ultimate vertical load = 2460 kN

N M
(i) Determine the values of and for the column in Fig 6.2.
bh bh2

N
= ……………… N mm-2
bh
M
2
= ……………… N mm-2 [2]
bh

14
Preliminary Examination Meridian Junior College 21 September 2010
JC2 H2 Physics 2010

(ii) Using Chart No. 49, determine whether 4 steel bars of diameter 32 mm with a
total area of 3220 mm2 is sufficient. [4]

(c) A young engineer designed the same column except that he used a different concrete
with an ultimate crushing pressure limit, fcu of 25 N mm-2. Compare your calculated value
N
of in (b)(i) with this value of fcu = 25 N mm-2. Comment what would happen to the
bh
concrete.

.............................................................................................................................................

.............................................................................................................................................

.................................................................................................................................... [2]

(d) The height of the column designed is 3.8 m. Suggest one possible problem with another
12.0 m height column of similar size subjected to a similar vertical load and bending
moments.

.............................................................................................................................................

.................................................................................................................................... [1]

15
Preliminary Examination Meridian Junior College 21 September 2010
JC2 H2 Physics 2010

100 ASC M
(e) The design of another column has the values of = 2.0 and = 2.0 .
bh bh 2
Using Charts No. 39 and 49, determine the percentage decrease in ultimate vertical
load, N if fcu = 50 N mm-2 changes to fcu = 40 N mm-2.

Percentage decrease of N = .....................................% [4]

16
Preliminary Examination Meridian Junior College 21 September 2010
JC2 H2 Physics 2010

17
Preliminary Examination Meridian Junior College 21 September 2010
JC2 H2 Physics 2010

18
Preliminary Examination Meridian Junior College 21 September 2010
JC2 H2 Physics 2010
Section B

It is recommended that you spend about 30 minutes on this question

7 The piezoelectric effect describes an effect that converts a mechanical effect to an electrical
signal or vice versa. For example, piezoelectric sensors such as quartz, silicon, manmade
crystals or even flexible polymer sheets can produce electricity when squeezed, moved or
bent. On the other hand, piezoelectric transducers, such as those used in speakers, rapidly
change shape when subjected to an electrical current.

Many modern devices, like the Nintendo Wii remote as well as smartphones such as the Apple
iPhone have tiny inbuilt piezoelectric accelerometers to help sense motion and other actions.

Compressive force

Electrical signal developed across


Piezoelectric here when piezoelectric material is
sample compressed

Although called an accelerometer, it is actually the inertial force on a known mass that is
measured by the piezoelectric material when the device is moved, which is then converted into
an electrical signal and interpreted accordingly.

Design an experiment using a sample of piezoelectric material to find how the strength of the
electrical signal depends on how much force it is subjected to.

The equipment available includes the following, besides common apparatus found in the
laboratory:

• An A4 size flexible sheet of piezoelectric material (you may assume that the surfaces are
conducting)
• Digital Multimeters
• 10 slotted masses

You should draw diagrams to show the arrangement of your apparatus. In your account you
should pay particular attention to
a) The equipment you would use for the investigation,
b) The procedure to be followed,
c) The control of variables,
d) Any safety precautions,
e) Any precautions that you would take to improve the accuracy of the experiment

19
Preliminary Examination Meridian Junior College 21 September 2010
JC2 H2 Physics 2010
Diagram:

Please make use of this page and next two pages to write your answers to Q7:

................................................................................................................................................

................................................................................................................................................

................................................................................................................................................

................................................................................................................................................

................................................................................................................................................

................................................................................................................................................

................................................................................................................................................

................................................................................................................................................

................................................................................................................................................

................................................................................................................................................

................................................................................................................................................

................................................................................................................................................

20
Preliminary Examination Meridian Junior College 21 September 2010
JC2 H2 Physics 2010

................................................................................................................................................

................................................................................................................................................

................................................................................................................................................

................................................................................................................................................

................................................................................................................................................

................................................................................................................................................

................................................................................................................................................

................................................................................................................................................

................................................................................................................................................

................................................................................................................................................

................................................................................................................................................

................................................................................................................................................

................................................................................................................................................

................................................................................................................................................

................................................................................................................................................

................................................................................................................................................

................................................................................................................................................

................................................................................................................................................

................................................................................................................................................

................................................................................................................................................

..............................................................................................................................................................

21
Preliminary Examination Meridian Junior College 21 September 2010
JC2 H2 Physics 2010

................................................................................................................................................

................................................................................................................................................

................................................................................................................................................

................................................................................................................................................

................................................................................................................................................

................................................................................................................................................

................................................................................................................................................

................................................................................................................................................

................................................................................................................................................

................................................................................................................................................

................................................................................................................................................

................................................................................................................................................

................................................................................................................................................

................................................................................................................................................

................................................................................................................................................

................................................................................................................................................

................................................................................................................................................

................................................................................................................................................

................................................................................................................................................

................................................................................................................................................

End of Paper

22
MERIDIAN JUNIOR COLLEGE
Preliminary Examinations
Higher 2

___________________________________________________________________

H2 Physics 9646/3
Paper 3 15 September 2010

2 hours
___________________________________________________________________
READ THESE INSTRUCTIONS FIRST

Class Reg Number

Candidate Name _____________________________

This booklet contains Sections A and B of the Preliminary exam paper 3.

Do not open this booklet until you are told to do so.

Section A Examiner’s Use


Answer all questions. Section A

Section B Q1 /8
Answer any two questions.
Q2 /10
In the event that all 3 questions are attempted, only the
first 2 questions will be marked. Q3 /6

You are advised to spend about one hour on each section. Q4 /8


Write your answers on this question booklet in the blanks
provided. Q5 /8

Section B
INFORMATION FOR CANDIDATES
Circle the questions you
have attempted
The number of marks is given in brackets [ ] at the end of
each question or part question. Marks will be deducted if Q6 /20
units are not stated where necessary or if answers are not
quoted to the appropriate number of significant figures. Q7 /20

All working for numerical answers must be shown. You are Q8 /20
reminded of the need for good English and clear presentation
of your answers. Deductions

Total /80

This document consists of 21 printed pages


Preliminary Examination Meridian Junior College 15 September 2010
JC2 H2 Physics 2010

DATA AND FORMULAE

Data
speed of light in free space c = 3.00 x 108 m s-1

permeability of free space μo = 4π x 10-7 H m-1


permittivity of free space ε0 = 8.85 x 10-12 F m-1
= (1/(36π)) x 10-9 F m-1
elementary charge e = 1.60 x 10-19 C
the Planck constant h = 6.63 x 10-34 J s
unified atomic mass constant u = 1.66 x 10-27 kg
rest mass of electron me = 9.11 x 10-31 kg
rest mass of proton mp = 1.67 x 10-27 kg
molar gas constant R = 8.31 J K-1 mol-1
the Avogadro constant NA = 6.02 x 1023 mol-1
the Boltzmann constant k = 1.38 x 10-23 J K-1
gravitational constant G = 6.67 x 10-11 N m2 kg-2
acceleration of free fall g = 9.81 m s-2

Formulae
uniformly accelerated motion 1
s = ut + at2
2
v2 = u2 + 2as
work done on/by a gas W = p ΔV
hydrostatic pressure p = ρgh
gravitational potential φ = -Gm/r
displacement of particle in s.h.m. x = xo sin ωt
velocity of particle in s.h.m. v = vo cos ωt

= ± ω xo 2 - x 2
resistors in series R = R1 + R2 + …
resistors in parallel 1/R = 1/R1 + 1/R2 + …
electric potential V = Q/4πεor
alternating current/voltage x = xo sin ωt
transmission coefficient T = exp(-2kd)

8π m(U − E )
2

where k = 2
h
radioactive decay x = xo exp(-λt )
decay constant 0.693
λ = t1
2

2
Preliminary Examination Meridian Junior College 15 September 2010
JC2 H2 Physics 2010

Section A
Answer all the questions in this section.

1 (a) A charged body falls vertically in a vacuum near the Earth’s surface. The variation
with time t of its vertical speed v is shown in Fig. 1.1 below.

v/ m s-1

0 t/ s

Fig. 1.1

An electric field induces a horizontal force on the body that causes the body to
accelerate horizontally at 2.25 m s-2. Calculate the displacement of this body after
0.50 s falling from rest.

displacement = ................................m

angle = ........................... [4]

3
Preliminary Examination Meridian Junior College 15 September 2010
JC2 H2 Physics 2010

(b) Another object moving in a straight line has a graph of the variation with time of its
velocity shown in Fig. 1.2.

v/ ms-1

0 t1 t2
t/ s

Fig. 1.2

(i) Sketch on Fig. 1.2, a graph of the variation of the acceleration with [2]
time for the same object within the same time frame.

(ii) Explain your sketch in (i) between time t1 and t2.

…………………………………………………………………………………………........

………………………………………………………………………………………….......

……………………………………………………………………………………..... [2]

4
Preliminary Examination Meridian Junior College 15 September 2010
JC2 H2 Physics 2010

2 (a) It is often assumed that air resistance acting on a moving object will result in the
object slowing down. Air resistance can however indirectly make an object speed
up. Consider a 1000 kg satellite orbiting at 280 km above the Earth’s surface. A
small force of air resistance makes the satellite descend into a circular orbit with an
altitude of 100 km.
[Radius of Earth = 6.37 x 106 m, mass of Earth = 5.98 x 1024 kg]

(i) By calculating the speed of the satellite at both orbits, show that the satellite
is indeed travelling faster at the lower orbit. [3]

(ii) Show that the total mechanical energy of the satellite, E can be expressed
as:

GME ms
E=−
2Ro
where ME is the mass of earth, ms is the mass of satellite and, Ro is the
radius of orbit. [2]

5
Preliminary Examination Meridian Junior College 15 September 2010
JC2 H2 Physics 2010

(iii) Explain the significance of the negative sign in the expression for the total
mechanical energy of the satellite.

…………………………………………………………………………………........

…………………………………………………………………………………........

……………………………………………………………………………..... [1]

(iv) Hence, calculate the change in mechanical energy due to air resistance.

change in mechanical energy = ……………….. J [2]

(c) Black holes are formed when massive stars collapse towards a singularity i.e. a
point mass. There exist a boundary, known as the event horizon, surrounding a
black hole where a even body travelling at the speed of light (if it is possible)
can barely escape.

Consider the gravitational potential energy of the body at the event horizon,
deduce an expression for the radius of the event horizon, Revent horizon in terms of
the mass of the black hole, M and speed of light c.

Revent horizon = ……………….. [2]

6
Preliminary Examination Meridian Junior College 15 September 2010
JC2 H2 Physics 2010

3 (a) A thin layer of copper is deposited uniformly on the surface of an iron wire of radius
0.60 mm and length 3.0 m shown in Fig. 3.1.
iron wire

thin layer of copper

Fig. 3.1 (Not to scale)

Determine the effective resistance between the ends of the copper-plated wire,
given that the thickness of the copper is 1.78 x 10-5 m.

[Resistivity of iron = 8.90 x 10-8 Ω m; resistivity of copper = 1.60 x 10-8 Ω m]

effective resistance = .......................... Ω [3]

7
Preliminary Examination Meridian Junior College 15 September 2010
JC2 H2 Physics 2010

(b) Fig. 3.2 shows a system in which an unmodulated audio frequency signal is
transmitted from the transmitter to the receiver through a cable. The cable consists
of two strands of insulated copper wire.

copper wire insulation


I
transmitter receiver
12.5 mW

Fig. 3.2

The power output of the transmitter is 12.5 mW and the corresponding current in
each wire is 2.5 mA. Power is lost to the surroundings due to the rise in
temperature produced by this current. For transmitted signal to be detected the
power input to the receiver must be at least 1.5 mW.

The resistance of each 1.0 m of the copper wire used in the cable is 0.27 Ω.

Calculate the maximum distance between the transmitter and receiver at which the
transmission can be detected successfully.

maximum distance = ……………………m [3]

4 (a) State Faraday’s law of electromagnetic induction.

………………………………………………………………………………………….......

………………………………………………………………………………………….......

……………………………………………………………………………………...... [1]

8
Preliminary Examination Meridian Junior College 15 September 2010
JC2 H2 Physics 2010

(b) A long straight wire carries a direct current. A rigid loop of conducting wire is
placed near the wire such that the wire is in the plane of the loop. The loop is
moved at constant speed away from the wire as shown in Fig. 4.1.

wire

current

conducting
loop

direction of motion of loop

Fig. 4.1

(i) Explain why an e.m.f. is induced in the loop.

…………………………………………………………………………………………........

…………………………………………………………………………………………........

…………………………………………………………………………………………........

……………………………………………………………………………………...... [2]

(ii) On Fig. 4.1, draw an arrow to indicate the direction of the current induced in
the loop and explain your answer below.

…………………………………………………………………………………………........

…………………………………………………………………………………………........

…………………………………………………………………………………………........

……………………………………………………………………………………...... [2]

(iii) It was found that energy is dissipated in the wire loop. Explain how the
movement of the loop gives rise to energy dissipation.

………………………………………………………………………………………….........

…………………………………………………………………………………………........

………………………………………………………………………………………….......

………………………………………………………………………………………….......

……………………………………………………………………………………...... [3]
9
Preliminary Examination Meridian Junior College 15 September 2010
JC2 H2 Physics 2010

5 (a) Explain what is meant by internal energy of a system.

…………………………………………………………………………………………........

…………………………………………………………………………………………........

……………………………………………………………………………………...... [2]

(b) The temperature, T of an ideal gas at pressure p is defined by the equation

p = nkT

(i) Identify the quantity n.

……………………………………………………………………………………...... [1]

(ii) State an equation relating k and R, molar gas constant.

……………………………………………………………………………………...... [1]

(c) State the process and give one practical example of each of the following :

(i) a process in which heat is supplied to a system without causing an increase


in temperature.

…………………………………………………………………………………………........

…………………………………………………………………………………………........

……………………………………………………………………………………...... [2]

(ii) a process in which no heat enters or leaves a system but the temperature
changes.

…………………………………………………………………………………………........

…………………………………………………………………………………………........

……………………………………………………………………………………...... [2]

10
Preliminary Examination Meridian Junior College 15 September 2010
JC2 H2 Physics 2010

Section B
Answer two questions in this section.

6 (a) (i) State Newton’s 1st law of motion and show it leads to the concept of force.

…………………………………………………………………………………………........

…………………………………………………………………………………………........

…………………………………………………………………………………………........

……………………………………………………………………………………....... [2]

(a) (ii) Express the unit of force in terms of S.I. base units.

S.I. base units of force is ….....…………… [1]

(b) Michael drove a car of mass 1200 kg which had a maximum speed of 150 km h-1.
During a driving test, it was found that the average retarding force from the air and
ground added up to 1200 N when the car was accelerating uniformly.

(i) Calculate the forward driving force when the car accelerated uniformly from
rest to the maximum speed in 11.0 s under driving test conditions.

forward driving force= ……………… N [3]

11
Preliminary Examination Meridian Junior College 15 September 2010
JC2 H2 Physics 2010

(ii) Hence, find the maximum power delivered by the engine.

maximum power = ……………… W [2]

(c) Michael wishes to find out how his car will fare during a car crash. He visited a
laboratory where several cars like his own were used in controlled car crash
testing.

The magnitude F of the force required to crush the barrels was shown below in
Fig. 6.1 as a function of the distance x the automobile had moved into the cushion.
F / kN
y

170
130
x
90

x/ m
0.0 1.5 4.0

Fig. 6.1

In a particular crash test, the car was travelling at 100 km h-1 before it struck a
crash cushion in which the car was brought to rest by successively crushing steel
barrels.

(i) Neglecting friction, predict by using the Work Energy Theorem the distance
the car would move into the cushion of steel barrels before coming to rest.

distance = ……………… m [4]


12
Preliminary Examination Meridian Junior College 15 September 2010
JC2 H2 Physics 2010

(ii) State and explain in terms of energy considerations whether the actual
distance will be longer or shorter than the value in (c)(i).

…………………………………………………………………………………………........

…………………………………………………………………………………………........

…………………………………………………………………………………………........

……………………………………………………………………………………....... [2]

(d) Michael parked his 1200 kg car at an underground carpark in Orchard Road. It
started to rain very heavily and rainwater quickly entered and filled the
underground carpark such that Michael’s car is floating in the water as shown in
Fig. 6.2.

Fig. 6.2

The total volume of the car is 6.43 m3 and the volume of air space in the car is
5.50 m3.

[Density of rainwater = 1000 kg m-3]

(i) State Archimedes’ principle.

…………………………………………………………………………………………........

……………………………………………………………………………………...... [1]

(ii) Initially, no water enters the passenger compartment. Determine the volume
of car below the water surface when the car is floating as shown in Fig. 6.2.

volume of car = …………………….. m3 [2]

13
Preliminary Examination Meridian Junior College 15 September 2010
JC2 H2 Physics 2010

(iii) Water slowly enters the car. Determine the volume of water in the car at the
point when it first disappears completely below the water surface.
(Assuming that the car remains horizontal throughout the sinking process)

volume of water = …………………… m3 [3]

14
Preliminary Examination Meridian Junior College 15 September 2010
JC2 H2 Physics 2010

7 (a) Define the Tesla.

………………………………………………………………………………………….......

………………………………………………………………………………………….......

……………………………………………………………………………………...... [2]

(b) Fig. 7.1 shows an arrangement used to accelerate an initially stationary alpha
particle and make it travel in a uniform magnetic field.

Path of alpha particle

A B
Region W

alpha
emitter

Uniform magnetic field of


4.0 kV 2.00 T directed into the
plane of the paper

Vacuum

Fig. 7.1

(i) On Fig. 7.1, draw a possible trajectory of the alpha particle in the uniform
magnetic field. [1]

(ii) Explain whether the force experienced by the alpha particle due to the
magnetic field changes its kinetic energy.

………………………………………………………………………………….........

…………………………………………………………………………………........

………………………………………………………………………………….........

…………………………………………………………………………………........

……………………………………………………………………………........ [3]

15
Preliminary Examination Meridian Junior College 15 September 2010
JC2 H2 Physics 2010

(iii) Show that the alpha particle will attain a speed of 6.21 x 105 m s-1 when it
reaches the slit opening of plate B.
[ Mass of alpha particle = 6.644 × 10−27 kg ] [1]

(iv) Calculate the electric field that needs to be applied in Region W for the
alpha particle to pass through the uniform magnetic field undeflected.

Magnitude of electric field = ……………….. [2]

Direction of Electric Field = ……………….. [1]

(c) Suppose that the arrangement in Fig. 7.1 is now modified so that the alpha particle
enters the uniform magnetic field at an angle of 30o to the horizontal as shown in
Fig. 7.2 below.

A B

300

alpha
emitter
4.0 kV Uniform magnetic field of
2.00 T directed along the
plane of the paper

Vacuum
Fig 7.2

16
Preliminary Examination Meridian Junior College 15 September 2010
JC2 H2 Physics 2010

The motion of the alpha particle can be described as a helix as shown on Fig. 7.3
below.

B field

300

+qα

Pitch, p

Helical path of alpha particle in uniform B Field

B field

r
Radius, r

Cross – section of helical path

Fig. 7.3

(i) Calculate the radius r of the helical path.

radius r = ………………..m [2]


17
Preliminary Examination Meridian Junior College 15 September 2010
JC2 H2 Physics 2010

(ii) Show that the period, T of the helical path can be expressed as:

2π mα
T=
qα B

where mα and qα is the mass and charge of the alpha particle respectively.
[1]

(iii) Calculate the pitch, p.

pitch, p = ………………..m [3]

(iv) 0 +
Describe and explain how the helical path will change if a positron, +1e (i.e.
a particle with the mass of an electron and charge +1.6 × 10−19 C ) with the
same initial velocity was to be used in the experiment instead. You may
quote relevant equations to substantiate your answer.

.......……………………….……………………………………………………….….

.......……………………….……………………………………………………….….

.......……………………….……………………………………………………….….

….………………………………………………………………………………........

........………………………………………………………………………………......

..........…………………………………………………………………………………

……………………………………………………………………………........ [4]
18
Preliminary Examination Meridian Junior College 15 September 2010
JC2 H2 Physics 2010

8 (a) The X-ray spectrum is first produced by an X-ray tube with tungsten (atomic
number, Z = 74) anode. Another X-ray spectrum is then produced using barium
(atomic number, Z =56).

Fig. 8.1

(i) Describe how the characteristic X-ray spectrum is formed.

………………………………………………………………………………..…........

………………………………………………………………………………..…........

………………………………………………………………………………..….......

………………………………………………………………………………..…........

………………………………………………………………………………..…........

……………………………………………………………………………........ [4]

(ii) The accelerating potential used to produce the X-ray spectra using tungsten
and barium are the same. Explain how this can be deduced from Fig. 8.1.

…………………………………………………………………………………..........

……………………………………………………………………………........ [1]

19
Preliminary Examination Meridian Junior College 15 September 2010
JC2 H2 Physics 2010

(iii) Hence calculate the accelerating voltage for the barium spectrum.

accelerating voltage = …………....…. V [2]

(iv) Explain the difference between the intensity of the Kα- line of the tungsten
and barium spectrum.

…………………………………………………………………………………..........

………………………………………………………………………………….........

……………………………………………………………………………........ [2]

(v) Calculate the minimum accelerating potential to produce Kα – line for


tungsten. Hence sketch on Fig. 8.1 without any additional numerical
labels, the spectrum for tungsten if the accelerating potential is
reduced to 50 kV.

minimum accelerating potential = …………....…. V [3]

(b) An X-ray machine is accelerating electrons through a p.d. of 200 kV. The current
is 25 mA. The target is a heavy metal mass 1.0 kg, and specific heat capacity
300 J kg K-1 and has a melting point of 3000 K. The machine is at 300 K when it is
first started.  While the machine is operating, the cooling fails and its temperature
increases by 16.5 K within a second.

(i) Calculate efficiency of X-ray production.

efficiency of X-ray production = .....................% [2]

20
Preliminary Examination Meridian Junior College 15 September 2010
JC2 H2 Physics 2010

(c) Fig. 8.2 below shows portions of the energy-level diagrams of the helium (He) and
neon (Ne) atoms. The He atom is excited from its ground state to state of
20.61 eV. The excited level of helium at 20.61 eV is very close to a level in neon at
20.66 eV. Upon collision with a neon atom, the energy can be transferred from the
helium to the neon atom. This excites the Ne atoms to the E3 state at 20.66 eV.
Lasing action takes place for electron transitions from E3 to E2 in the Ne atoms.

Helium Atom Neon Atom


Collision
20.61 eV E3
Lasing
18.70 eV
E2
Energy

Ground state
E1
Fig. 8.2

(i) State any two unique characteristics of laser light.

………………..………………………………………………………………….......

……………………………………………………………………………....... [1]

(ii) Explain what is meant by population inversion and how energy state E3 in
Ne enables lasing to occur.

………………..………………………………………………………………….......

………………..………………………………………………………………….......

………………..………………………………………………………………….......

………………..………………………………………………………………….......

……………………………………………………………………………....... [3]

(iii) Explain why direct optical pumping (the supply of photons) excitation
method using photons of energy Δ E = E3 - E1 on Neon atom, in the
absence of He atom, is generally not used.

………………..…………………………………………………………………......

………………..…………………………………………………………………......

………………..………………………………………………………………….......

……………………………………………………………………………....... [2]
End of Paper
21
Preliminary Examinations 2010

JC2 Physics H2 Solutions

Paper 1 Solutions

1 B 11 A 21 A 31 A
2 B 12 D 22 A 32 B
3 D 13 C 23 D 33 C
4 D 14 C 24 C 34 C
5 D 15 D 25 C 35 A
6 C 16 D 26 B 36 D
7 B 17 B 27 A 37 C
8 B 18 B 28 B 38 A
9 A 19 B 29 A 39 D
10 C 20 A 30 C 40 D

1 Solution: B

A = LW
ΔA l w
= +
A L W
⎛l w⎞
ΔA = ⎜ + ⎟A
⎝L W ⎠
ΔA = L w + W l

2 Solution: B

Frequency of radio wave ranges from 3 Hz to 300 GHz


Question: 500 μHz = 500 x 10-6 Hz---- too small

Mass of lightest atom; Hydrogen-1 is 1.007825u; ~ 1.66 x 10-27 kg


Mass of heaviest stable atom; Lead-208 is 207.9766521u ~ 3.45 x 10-25 kg
Question: 500 pg = 500 x 10-12 g = 5 x 10-10 kg (too heavy)

Acceleration due to free fall 981 mm s-2 (too small--- should be 9810 mm s-2)

3 Solution: D

v2=u2+2as
49.12 = 14.72 + 2(9.81)s
s= 111.86 m; 112m

1
4 Solution: D

The time taken for the flight up to the highest point should be shorter than the time
taken for the flight down.

5 Solution: D

To find T1, take pivot at rightmost end of bridge


⎛l⎞
Wb ⎜ ⎟ + Wc (l − x ) = T1 (l )
⎝2⎠
⎛l⎞
Wb ⎜ ⎟ + Wc (l − x )
T1 = ⎝2⎠
l
Wb x
= + Wc (1 − )
2 l

To find T2, take pivot at leftmost end of bridge


⎛l⎞
Wb ⎜ ⎟ + Wc ( x) = T2 (l )
⎝2⎠
⎛l⎞
Wb ⎜ ⎟ + Wc ( x)
T2 = ⎝2⎠
l
Wb x
= + Wc ( )
2 l

6 Solution: C

As linear momentum is conserved, there is total transfer of momentum during


collision as all 3 discs are identical.

7 Solution: B

Upthrust = Weight of balloons + Weight of house


ρairgVhelium = ρheliumgVhelium + Weight of house
Weight of house = 1.2 x 9.81 x 0.17 x 20000 – 0.18 x 9.81 x 0.17 x 20000
= 34000 N

8 Solution: B

By Hooke’s Law, the component of the sphere’s weight down the incline causes the
spring to compress by a value e.

mg sin θ = ke
(3.00)(9.81) sin 30 0
or e = ≈ 0.0294 m
500
= 29 .4 mm

2
9 Solution: A

The kinetic energy is ½mv2 and remains constant as it reaches constant velocity,
Rate of change of GPE = mgh / t = mg(h/t) = mgv

10 Solution: C

aspore = rsporeω 2
aseoul = rseoulω 2
( )
= rspore cos 37.5o ω 2
aspore 1
= = 1.26
aseoul cos 37.5o

11 Solution: A

By the principle of conservation of energy,


PEi + KEi = PEf + KEf
1
mgh + 0 = 0 + mv 2
2
1
80.0(9.81)(30.0 − 30.0 cos 50o ) = (80.0)v 2
2
v = 14.5 m s-1

By the principle of conservation of momentum,


mT uT + m J u J = ( mT + m J )v
80.0(14.5) + 0 = (80.0 + 45.0)v
v = 9.28 m s -1

12 Solution: D

At point C, particle's velocity is entirely horizontal.


i.e. v = −3.0 m s-1

By COE,
KE A = GPEC + KEC
1
m (v A2 − v C2 ) = mg (2R )
2
52 − 32
R= = 0.408 m
4g

v2 32
ac = = = 22.1 m s−2
R 0.408

3
13 Solution: C

T 2 ∝ R3
3
2
Tmercury ⎛ Rmercury ⎞ ⎛ 5.79 ⎞
3
=⎜ ⎟⎟ = ⎜ ⎟
2
Tneptune ⎜ Rneptune ⎝ 450 ⎠
⎝ ⎠
3
⎛ 450 ⎞
2
Tneptune (
= 0.2412 ⎜ ) ⎟
⎝ 5.79 ⎠
Tneptune = 165 years

14 Solution: C

GM
φ =−
R
GM
−89.73 = − (1)
R + 2300
GM
−97.14 = − (2)
R + 600
(1) 89.73 R + 600
= =
(2) 97.14 R + 2300
7.41R = 148095
R = 19985.83 m
M = 3.00 × 1016 kg

15 Solution: D

At equilibrium position,
mg = kxo
mg 0.10 × 10
k= = = 10 N m−1
xo 0.10

When extended distance x from equilibrium position,


mg − k ( xo + x ) = ma
mg − kxo − kx = ma
k
a=− x
m
k 10 N m−1
⇒ω = = = 10 rad s−1
m 0.10 kg

∴ x = xo cos(ωt )
= 0.020 cos(10t )

4
16 Solution: D

Ek = ½ mw2xo2sin2wt

17 Solution: B

2λ = 0.056
v = fλ
(3.0 x 108) = f(0.056/2)
f = 10.7 GHz

18 Solution: B

I α A2
For maximum intensity, Io α (3A)2
For minimum intensity, I α (A)2
Io/I = (3A/A)2
I = Io/9

19 Solution: B

20 Solution: A

For option B and D, we require two progressive waves to create interference pattern
and standing wave.

Option C is not true because light wave only travels at the speed of light, not at any
velocity.

A wave can also behave as a particle. As such, it can carry a momentum.

21 Solution: A

For option B and C, it is only true if the system is an ideal gas.

For option D, different states of the system will have different internal energy.
Therefore, it is only true if the two system are at the same state.

According to 1st law of thermodynamics, the internal energy of a system depends on


the heat absorbed and the work done on the system. Thus, the internal energy of
the system can also be changed without heating the system.

22 Solution: A

pV = NkT
p = (N/V)kT
p = NvkT 0.50p = N’k(2.0T)
N’ = 0.25 Nv

5
23 Solution: D

To remain stationary, mg = eE = e(V/d). The equation can also be arranged as

mg = 2e(0.5V/d)

when the oil drop acquire an additional negative charge.

24 Solution: C

Electric potential is a scalar. It add up to zero at point X and Y. The vector sum of
the electric field at point X due to + 5 μC and - 5 μC point upwards.

The electric field at point Y due to + 5 μC points in Northeast direction while that
due to
-5 μC points Northwest direction. As a result, the resultant field at point Y points
upwards.

25 Solution: C

2.0 Ω
X
D

V
Y 2.0 Ω
Z 2.0 Ω

Before voltage supply is reversed, diode is forward-bias.


Total current in circuit = V / RX = 12 / 2 = 6.0 A
Since diode is ideal, hence current across resistor Y or Z = 6.0 / 2 = 3.0 A
Potential difference across resistor Y or Z = I (R) = 3.0 x 2 = 6.0 V
Hence, potential difference across voltage supply = 12 + 6.0 = 18 V

After voltage supply is reversed, diode is reverse-bias, no current will flow through
the diode.
Effective resistance = 2.0 + 2.0 = 4.0 Ω
Total current in circuit = V / R = 18 / 4.0 = 4.5 A
Hence, voltmeter reading = Iacross X (R) = 4.5 x 2 = 9.0 V

6
26 Solution: B

p.d across the 14 Ω resistor = 3 x 0.5 = 1.5 V

I = V / R = 1.5 / 1.4 = 0.11 A (or 0.107 A)

ε = 3.5 x 0.5 = 1.75 V

Using ε = V +Ir
1.75 = 1.5 + 0.107r
r = 2.3 Ω

27 Solution: A

Only A is the conclusion we can draw from the information given.


If the filament of the fifth bulb has broken, the voltmeter will register the voltage
across the transformer and not a zero reading.

We cannot conclude that C is correct. B and D are possible but further testing need
to be done.

28 Solution: B

The equivalent networks are:

5.0 Ω R

Y Z X Y
and
R R R 5.0 Ω

−1 −1
⎛1 1 ⎞ ⎛1 1 ⎞
RYZ = ⎜ + ⎟ R XY = ⎜ + ⎟
⎝ 5 2R ⎠ ⎝ R R + 5.0 ⎠
−1
1 2R + 5 ⎛ 1 1 ⎞
= =⎜ + ⎟
2.5 5(2R ) ⎝ 2.5 7.5 ⎠
R = 2.5 Ω = 1.88 Ω

29 Solution: A

B field within coil P will be directed upwards.

Using LHR, force on wire will be directed towards TS.

Hence rod will move towards TS with acceleration i.e. increasing speed.

7
30 Solution: C

μoI
FB = ×I ×L
2π d
μ I 2L
= o
2π d
FB μoI 2
=
L 2π d

FB mg
Since the wire is suspended, =
L L
μoI 2 ⎛ m ⎞
= g
2π d ⎜⎝ L ⎟⎠
4π × 10 −7 × 100.02
= (0.020)(9.81)
2π d
2 × 10 −7 × 100.02
d=
0.020 × 9.81
= 0.0102 m
= 1.02 cm

31 Solution: A

Magnet (N-pole) approaches coil, there would be changing magnetic flux linking
coil. According to Faraday’s law an induced emf is generated. Since, circuit is
closed, an induced current is formed. The direction of current would produce a ‘N’
pole.
When magnet is inside the coil, there is no change in flux linkage and no emf and
current is induced.
When magnet (S-pole) leaves the coil, there would be changing flux linkage and an
induced emf and current is produced. According to Lenz’s law, the current flow
would produce a ‘N’ pole on the right end of the coil. This is opposite to when it
approaches. Hence, diagram A.

32 Solution: B

Reasoning:
As the ring moves closer to the electromagnet, it has more flux linkage. Faraday’s
law states that an emf will be induced and Lenz’ law tells us that there will be an
opposing force acting on the ring to slow it down. This external force will damp the
system. The best answer is B.

8
33 Solution: C

2
⎛ V0 ⎞
Vrms 2 ⎜ ⎟ V2
=⎝
2⎠
Fig A, Mean Power, W = = 0
R R 2R

area under the V 2 − t curve over aperiod


Fig B, Vrms =
T

1
( 2V0 )
2
T +0
= 2 = 2V0
T

( )
2
Vrms 2 2V0 2V0 2
Mean power for Figure B, PB = = = = 4W
R R R

34 Solution: C

Irms = P/V = 8.333 A to 10.9 A

Corresponding I0= 11.79 A to 15.4 A

Expression for I= I0 sinwt

35 Solution: A

T ∝ , where k =

Thus if T’ = T ⇒ k’d’ = kd

W’ = W

W’ = W

W’ = W
W’ = 0.577 W

9
36 Solution: D

ΔE = 1% × 150 keV = 1.5 keV = 2.4 × 10-16 J


ΔE Δ t ≥ h/4π
Δt ≥ h/ΔE4π = 2.2 × 10-19 s

37 Solution: C

Option A describes Stimulated Absorption – An atom can be ‘excited’ from a


lower energy state (or ground state) E1 to a higher energy state E2 when one of its
electron absorbs a photon of energy corresponding to hf = __ E2 – E1 .

Option B should be: Spontaneous emission occurs more rapidly when the lifetime of
the excited state is short.

Option D should be: spontaneous emission competes more strongly with stimulated
emission in transitions that emit blue or ultraviolet light than for transitions that emit
red or infrared radiation (revisit the visible spectrum and compare the frequencies of
the two radiations).

38 Solution: A

When the battery is reversed, its current drops to zero: This indicates that the
semiconductor device is acting like a PN junction in reverse bias.

39 Solution: D

A = A0 e −λt
⎛ ln 2 ⎞
−⎜ ⎟ 24
A = 30e ⎝ 13 ⎠

A = 8 . 34 × 10 −6
Ci (this is the activity of radio isotope in the body after 24 hours)

4
Percentage found in the thyroid = × 100% = 48%
8.34

40 Solution: D

A significant majority of the alpha particles will pass through the gold foil
undeflected (0o). Deflection by large angles (up to 180o) is possible but likelihood is
low and decreases with magnitude of angle.

10
11
Paper 2 Solutions

1 (a) The Principle of Superposition states that when two waves of the same kind
meet at a point in space, the resultant displacement at that point is the vector
sum of the displacements that the two waves would separately produce at that
point. [B2]

(b) (i) Must be a sine wave i.e. correct phase at start [B1]
Amplitude = 1 div [B1]
Period = 8 div [B1]

(ii)

Period = 8 div [B1]


Two big peaks, two small peaks
Approximate correct heights/times [B2]

(iii) The frequency of wave W1 is 4.2 x 1015 Hz.

The frequency of wave W2 is 2.1 x 1015 Hz. [M1]

Frequency of the resultant wave is the same as the wave with a lower
frequency i.e. the frequency of resultant wave is 2.1 x 1015 Hz. [A1]

(iv) Must have two waves of the same wavelength for coherence. [B1]

OR

Do not have a constant phase difference. [B1]

(c) θ = 45.70/2 = 22.85o [C1]

d sin θ = nλ

d sin 22.85o = 2(485.6 x 10-9) [M1]


d = 2.5 x 10-6 m [M1]

number of lines per millimeter = 1/d = 400 [A1]

2 marks out of 4 for using θ = 45.70

12
2 (a) (i) The average height of an adult is about 1.75 m.
The potential difference between the head and feet,
V= Ed = 1.0 x 102 x 1.75 = 1.75 x 102 V [A1]

acceptable range of average height of an adult: 1.50 m to 2.00m 

(ii) 175 V is a relatively high potential difference. It would have given a


nasty electric shock if it were applied by the power mains. This is
because the power mains has an infinite supply of electric charge
in the forms of electrons. A large current would thus have
conducted through the body.

However, the amount of free electrons in the atmosphere is very


[B1]
small. Thus, although there is a large potential difference between
the head and feet, this potential difference is not able to deliver
[B1]
sufficient current through the body. Therefore, an electric shock is
not experience.

(b) (i) Although there is no current inside the tube, there is current flowing
through the external resistors.

The current through the 6 resistors, I = V / R =1050 / 6R. [M1]

The potential difference between the dynode 4 and the


photocathode, V = IR = (1050 / 6R) x 4R = 700 V [A1]

OR

Using potential divider rule,


4R
p.d. across photocathode and dynode 4 = × 1050 [M1]
6R
= 700 V [A1]

(ii) With dynode 3 and 5 short-circuited, the resistance between them


is zero. Current will flow directly from dynode 5 to dynode 3,
bypassing the two resistances between dynode 3 and 5
completely.

Since there is no current flowing through the two resistors between


dynode 3 and dynode 5, there is no potential difference across
either of them.
Hence, the potential at dynode 3 is equal to potential at dynode 4,
which is also equal to the potential at dynode 5. [M1]

The total resistance of the entire series dropped from 6R to 4R.


The current I’ = V / R = 1050 / 4R.

The potential difference between dynode 3 and the photocathode


is V = IR = (1050 / 4R) x 3R
= 788 V [M1]
[A1]

13
3 (a) N P Vp 8
= = [M1]
N S VS 3

n
Therefore, V p = V s ⎛⎜ 8 ⎞⎟
⎝3⎠

n
⎛8⎞ [M1]
230000 = 240 ⎜ ⎟
⎝3⎠

n = 6.999 ~7 [A1]

Assumption (one mark for any one of the following):


1) The transformers are ideal with 100% efficient.
2) The resistance of the primary and secondary coils rp and rs

are zero, so that no energy is lost in the coils.


3) There are no magnetic flux losses and hence both coils
have the same flux through them.

(b) The purpose of the iron core is to confine the magnetic field lines to
ensure maximum magnetic flux linkage between the primary and
secondary coils. [B1]

(c) Vrms 2 240 2


Average Power = = = 993 W [M1, A1]
R 58

(d) Hair dryer


Vacuum cleaner
Microwave oven
Or any other appliances that has about 1000 W power rating. [B1]

4 A p-n junction is formed between slices of p-type and n-type semiconductor material
as shown in Fig. 4.1

p-type n-type

[B1]

Fig. 4.1

(a) See diagram above.

(b) Boron is a Group III element. [C1]

When a boron atom replaces a silicon atom in the lattice, its three valence

14
electrons form covalent bonds with neighbouring silicon atoms, leaving an
electron deficiency, a hole. [B1]

This hole can carry current in the presence of an electric field by accepting
electrons. Hence Boron creates an acceptor semiconductor/ p-type
semiconductor. [B1]

(c) When the battery is connected in this manner, the applied voltage will cause the
holes in the p-type region and the electrons in the n-type region to move into the
depletion region. [M1]

This reduces the width of the depletion region. [A1]

5 (a) Binding energy of a nucleus is the work done on the nucleus to separate
it into its constituent neutrons and protons. [A1]

Other acceptable answers:


Binding energy is the energy released when a nucleus is formed from its
separate constituent neucleons (neutrons and protons).

(b) Δm = (Zmp + Nmn) – mA

Binding energy ΔE = Δmc2


= [(90 x 1.0073) + (136 x 1.0087) -226.0249] u c2 [M1]

= 1.8153 (934)

= 1695 MeV [A1]

15
(c)(i)
NOTE: (c) (i) and (ii) are marked together as the same part.

226
90 Th Æ 222
88 Ra* + 42 He
Given that
rest mass of 226
90 Th = 226.0249 u
rest mass of 222
88 Ra = 222.0154 u
rest mass of 4
2 He = 4.0026 u

By COE:

mTh c2 = mRac2 + KERa + mα c2 + KEα

(226.0249 u) c2 = (222.0154 u) c2 + KEradium + (4.0026u) c2 + 2.38 x 1.6x10-19

KE radium Ra* = 4.06 MeV

(ii) To find KE of γ :

In order to find the KE of γ need to find KE of radium Ra as well and take the two
equations as one and take Ra* would have decayed to Ra, and make assumption that
momentum of γ ray is negligible.

226
90 Th Æ 222
88 Ra + 4
2 He + γ

0 = mRavra + mα vα
(mRava)2 = ( mα vα )2
(½ mRavra2) mRa = ( ½ mα vα2 ) mα
(½ mRa v2Ra ) m
= α
½ mα vα2 m Ra

KE of Ra = 0.0429 MeV

88 Ra + γ
Ra* Æ 222
222
88

By COE :
mRa* c2 + KERa* = mRac2 + KERa + KE γ
Since there is no difference in the masses of Ra* and Ra.

KE of γ = 4.06 – 0.0429 = 4.02 MeV

…………………………………………………………………………………………….

Alternatively: A number for students use COM for the first part would have found the KE of
Ra instead of Ra*.

16
Total momentum before decay = Total momentum after decay

0 = mRavra + mα vα

(mRava)2 = ( mα vα )2
(½ mRavra2) mRa = ( ½ mα vα2 ) mα
(½ mRa v2Ra ) m
= α [M1]
½ mα vα2 m Ra

(½ m R a v 2R a ) = 4.0026 [M1]
2.38 222.0154

KE of Ra = 0.0429 MeV [A1]

This will give the answer of the KE of Ra. Acceptable as question is not clear.
Then will need to find the KE of Ra* (see earlier method to find using COE).

17
6 (a) d/h = 394/450 = 0.876 ~ 0.90 [B1]

Since Chart No.49 has the correct fcu, fy and d/h, Chart No.49 is appropriate to
be used for this design. [B1]

(b) (i) M/bh2 = 2.25 [M1]

N/bh = 27.3 [M1]

(ii) From Chart No.49,

100Asc/bh = 2.6 [M2 - allow errors up to 0.05 i.e. 2.55 < x < 2.65]

(If 100Asc/bh = 2.7 or 2.5, allocate only one mark)

Therefore, Asc = 2340 mm2 [A1]

Provided four 32 mm reinforcement bars ~ 3220 mm2 > Asc, therefore the
provided reinforcement steel is sufficient. [B1]

(c) N/bh > fcu [B1]

Column would be crushed/break/crack. [B1]

(d) At 12 metres, the column is too slender with respect to its dimension. Might
twist and buckle under load. [B1, any of the underlined words]

(e) From Chart No.39, corresponding N/bh = 21.0; allow 21.0 ± 0.5 [B1]

From Char No.49, corresponding N/bh = 25.5; allow 25.5 ± 0.5 [B1]

Decrease in N = 3.48bh kN

Percentage decrease of N = 3.48/25.42 x100 = 13.7% [M1 and A1; ecf


allowed]

18
Q7 Solution- Suggested marking schemes

Basic Procedure   [2]  This mark is scored as long as described procedures involve: 


  • Applying a force, Measuring an electrical signal 
• Varying the force and repeating. 
 
Ignore  specifics  –  e.g.  method  of  applying  force,  varying  force, 
what electrical signal, what it is measured with. 
 
E.g.   ‐  Place  slotted  masses  on  the  piezoelectric  material, 
  measure p.d. across the face. 
‐ Repeat for different total weight on it 
Diagram shows    [1]  Diagram is married to the procedure and meant to be instructive. 
E.g.  
• How to force is to be applied/masses are to be placed. 
• Appropriate electrical connections to the piezoelectric sample 
(across opposing surfaces. If the ONLY issue is connections not 
clearly shown, then award ½ ) 
• Appropriate circuit setup  
‐  Reject  circuits  that  have  1  lead  to  the  DMM,  or  DMM 
connected to CRO connected to computer.  
‐    Reject  circuits  that  have  a  EMF  source,  as  the  I,  V  now  are 
not  due  to  electrical  signals  generated  by  the  piezoelectric 
sample 
 
Workability  of  procedure  is  also  assessed/  gauged  here.  If 
suggested setup/connection is not feasible, withhold credit. 
 
Mark can be allocated for different procedure/diagram, provided it 
is clear how the setup is to  
• Diagram  shows  piezoelectric  sample  with  slotted  masses 
stacked on top of it, with  
• DMM connected across the sample face  
Method of measuring  • Measurement of the masses being placed on the sample 
/varying/ determining the  (I.V. to be measured with appropriate apparatus and not 
I.V. & D.V.      [2]  assumed to be known values) 
 
• Measurement  of  the  p.d.  developed  across  the  sample 
face (accept measurement of current as well, even though 
unlikely material can sustain a current flow. 
Accept power, P = IV as well) 
 
Reject  suggestions  where  force  cannot  be  determined  accurately 
(e.g. dropping/throwing the weight onto the sample, poking it with 
a  finger,  clamping  it  TAUT  then  resting  the  weight  on  it),  as  well 
suggestions that  
 
Control of Variables [max 2]  • ACTIVELY  Maintaining/monitoring  the  temperature,  as 
expansion  or  contraction  due  to  temp  change  affecting 
result. 
(e.g.  monitor  with  a  surroundings  with  a  thermometer, 
perform  in  an  climate  controlled  environment,  e.g.  aircon 

19
room) 
• ACTIVELY  Monitor  surrounding  atmospheric  pressure 
during course of experiment, as changes in pressure might 
affect the forces it is subjected to. 
(e.g. with a barometer) 
• If material is cut to size, then maintaining the size is valid. 
(e.g. cut out a circular portion ~3‐4 cm in diameter, about 
the same size as the slotted masses) 
• The  arrangement/placement  of  masses  on  the 
piezoelectric sheet.  
 
Control of Other Variables refers specifically to less trivial items. If 
student says use same material, A4 size, same wires, same DMM, 
do not award credit. 
Other  details  [max 4]  • Ensure  good  electrical  contact  between  the  DMM  probes 
(enhanced  and  the  surfaces  of  the  sample  (ensure  clean  from 
reliability/accuracy)  oxide/oil, or solder) 
  • Wires/foil attached to the surface such that DMM  probes 
can be in electrical contact with the surface while allowing 
even force applied on the surface. 
• Measuring p.d. for no weights (checking for possible “zero 
error” p.d. due to sample’s own weight) 
• Ensure expt performed on insulating surface 
• Ensure absence of any other electrical devices or signals in 
the vicinity 
• Practicing  loading/unloading  with  reasoning  (e.g.  test  of 
path dependence) 
• Repeat  entire  experiment  with  different  sample  size/area 
of force distribution/pressure (e.g. testing if indeed [force 
∝ p.d.] or (pressure ∝ p.d.) 
 
• Repeating and taking average + Plotting of an appropriate 
graph to illustrate the relationship 
 
 
Safety [max 1]  • Safety precautions pertaining to handling heavy slotted 
masses with due care 
 
• Comment on relative safety of experiment [e.g. no high 
voltages, toxic substances, high temperatures] 
 
(suggestions on electric shocks or electrocutions or high 
temperatures of wires are not very realistic.  
But if clear effort put in e.g. elaboration, > 1 non‐ridiculous 
suggestions made can award ½ mark to encourage active 
consideration of safety) 
 

20
Prelim H2 P2 Q7 Planning Training Task Suggested Solutions
(Disclaimer: These solutions do not represent the only acceptable answer. Other methods, apparatus, procedures,
precautions may also be acceptable. It is best you check with your tutor regarding the acceptability of your solutions if
they differ from what is presented below.)

Task – The Piezoelectric Effect

A) Problem Interpretation:
• Independent Variable:
The force applied to the piezoelectric sample. This will be practically done by placing variable
number of the slotted masses on it.

• Dependent Variable:
The potential difference developed across the piezoelectric sample, measured by a DMM.

• Aim of the Experiment:


To investigate how the potential difference developed across the piezoelectric sample varies with
the force it is subjected to.

• Control Variables:
− Temperature of the surroundings/piezoelectric sample.
− Size of the piezoelectric sample

B) Experimental Setup and Procedure

21
• Description of Procedure
1. Cut a small disc sized sample from the A4 flexible sheet of piezoelectric material, about 5
cm in diameter, using a ruler to measure out the diameter.

2. The apparatus is to be set up as shown in Fig 1 above.

3. Connect the probes of the DMM set to measure potential difference (prob ~mV or μV)
across the faces of the piezoelectric sample.
To ensure good electrical contact between the entire face of the piezoelectric sample and the
DMM probes, place a sheet of metallic foil over the faces of it. The foil can extend beyond the
face of the sample so that the probe can still be in contact with the sample bottom while still
allowing even distribution of force on it. (see Fig 2)
(if either the piezoelectric disc sample or the slotted mass is resting on the DMM probe [see Fig
3], the force will not be evenly distributed & the readings may not be accurate)

4. Measure the mass of a slotted masses with an electronic balance. Record the value, M. The
force on the piezoelectric disc sample can be calculated by W = Mg.

5. Place the slotted mass on the piezoelectric disc sample, and record the corresponding
potential difference given by the DMM. Record this as V.

6. Increase the number of slotted masses by one, measuring the total mass with the electronic
balance, M. Repeat step 4 until all the slotted masses have been used for a total of 10 sets of
data.
Stack the slotted masses are properly on top of each other to ensure uniform distribution of
force.

7. Repeat step 4 and 5 again, this time decreasing the number of slotted masses and thus the
force. The readings should not differ appreciably for the same M or W value. Record the
corresponding DMM readings as V’.

8. Process and tabulate all data.


Then plot a graph of force, W against potential difference, V to determine the relationship
between them.
e.g.

W=Mg W↑ W↓
# M W/N V/V V’/V
1
2

22
Reliability considerations in the procedure are highlighted with an

• Additional reliability considerations


1. Monitor the temperature of the surroundings during the experiment, as it is identified
as a possible factor. Perform the experiment in a climate controlled environment if possible,
such as an air-conditioned room. If not, monitor the temperature and only accept all the data
provided the temperature does not change significantly over the course of the experiment.

2. Clean the surface of the piezoelectric disc sample of dirt, oils or other contaminants
before applying the foil. (e.g. use alcohol wipes).

3. Stack the slotted masses neatly and vertically over the sample to ensure even
distribution of weight. Do not let it tilt or skew to one side.

4. The experiment is to be conducted on an insulated surface to avoid electrical


interference to avoid inaccurate results.

5. Measure the p.d. across the sample even with no weights on it. This is to account for
the sample’s own weight which might cause possible “zero error”.

• Safety considerations

1. The experiment is relatively safe as it involves no heavy weights, high voltages, high
temperatures or harmful substances.

End of paper 2 solutions

23
Proposed Solutions to 2010 JC2 H2 Paper 3

1 (a) Sx = ut + ½ at2
Sx = ½ (2.25) 0.502 [M1]
Sx = 0.281 m

Sy= ut + ½ at2
Sy= 0.5 x 9.81 x 0.502 [M1]
Sy= 1.23 m

Net displacement = S x 2 + S y 2 = 1.26 m [A1]


Angle with respect to horizontal = 77.1 degree below horizontal [ A1]

1 (b) (i)
Graph must be negative before any mark is awarded.

Graph: 1 mark for identifying that acceleration is zero at start and end of
graph

Graph: 1 mark for identifying that acceleration is negative and constant at


middle part of graph

1
b ii)
The acceleration between time t1 and t2 is negative value because the B1
negative slope of v-t graph.

The acceleration between time t1 and t2 is a constant maximum negative B1


value since it corresponds to the steepest gradient of the v-t graph.

2 (a) (i) GME ms ms v 2


=
r2 r
GME
v= [M1]
r

v 200 =
( 6.67 × 10 )(5.98 × 10 )
−11 24

( 6.37 × 10 ) + ( 2.8 × 10 )
6 5

= 7745 m s-1 [A1]

v100 =
( 6.67 × 10 )( 5.98 × 10 )
−11 24

( 6.37 × 10 ) + (1× 10 )
6 5

= 7852 m s-1 [A1]


v100 > v 200

(ii) 1
KineticEnergy = mv 2
2
1 GME ms
ms v 2 = [M1]
2 2r
GME ms ⎛ −GME ms ⎞
E= +⎜ ⎟ [M1]
2r ⎝ r ⎠
GME ms
=− [A0]
2r

(iii) The total mechanical energy of the satellite is negative because it is


bounded i.e. energy must be invested to remove the satellite from the
gravitational field and place it at infinity. [B1]

(iv) Mechanical E. lost = Ef − E i


−GM E ms ⎛ 1 1⎞
= ⎜ − ⎟
2 ⎝ rf ri ⎠

=−
( )
6.67 × 10 −11 5.98 × 10 24 1000
2
⎛ 1 1 ⎞
×⎜ − ⎟ [M1]
(
⎜ 6.37 × 10 6 + 1× 105
⎝ ) (
6.37 × 10 + 2.8 × 105
6
) ⎟

= −8.34 × 108 J [A1]

2
(c) At the event horizon, the gravitational potential energy a body of mass m is

GMm
U=−
Revent horizon

For a body to escape to infinity, its kinetic energy must be:

1 ⎛ GMm ⎞ GMm
mv 2 = 0 − ⎜ − ⎟= [M1]
2 ⎝ Revent horizon ⎠ Revent horizon

Since even a body travelling at the speed of light cannot escape,


GMm 1
= mc 2
Revent horizon 2
2GM
Revent horizon = [A1]
c2

3 (a) (i) ρ iron l


Riron =
Airon
8.90 x 10-8 (3.0)
=
π (0.60 x 10-3 )2
= 0.236 Ω [M1]

ρ copper l
Rcopper =
Acopper
1.60 x 10 -8 (3.0)
=
2π (0.60 x 10 -3 )(1.78 x 10 -5 )
= 0.715 Ω
or [M1]
ρ copper l
Rcopper =
Acopper
1.60 x 10 -8 (3.0)
=
π (0.60 x 10 +1.78 x 10 -5 )2 − π (0.60 x 10-3 )2
-3

= 0.705 Ω

When copper is deposited on the surface of the wire, it acts as a


parallel shunt to the iron wire.
1 1 1
= +
Reff Riron Rcopper
−1
⎡ 1 1 ⎤
Reff =⎢ +
⎣ 0.236 0.715 ⎥⎦
= 0.178 Ω

[A1]

3
(b) Method 1
Power loss per metre of cable (consist of 2 wires)
= I2R x 2
= (2.5 x 10-3)2 x 0.27 x 2 [M1]
= 3.38 x 10-6 W

Power loss in cable = 12.5 – 1.5 = 11.0 mW [M1]

Maximum distance = 11.0 x 10-3 / 3.38 x 10-6


= 3.25 x 103 m [A1]

Method 2
Power loss in cable = 12.5 – 1.5 = 11.0 mW
[M1]
Resistance of one wire = P / I2
= 5.5 x 10-3 / (2.5 x 10-3)2
=880 Ω
[M1]
Maximum distance = Rwire / Rper m = 880 / 0.27 = 3.26 x 103 m
[A1]

Method 3
Power loss in cable = 12.5 – 1.5 = 11.0 mW

Power loss in cable = 11 x 10-3 = I2Reff’


Reff’ = (11 x 10-3) / (5 x 10-3)2
= 440 Ω

Reff per m of both wires = (1/0.27 + 1/0.27)-1 = 0.35 Ω

Maximum distance = Reff’ / Reff per m = 440 / 0.35 = 3.26 x 103 m

4 (a) The induced e.m.f. ε is directly proportional to the rate of change of magnetic
flux linkage Φ (or rate of cutting of magnetic flux).

(b) (i) • Changing flux linkage approach:


The current in the wire generates a magnetic field that decreases with
distance from it (B ∝ 1/r). Thus the magnetic flux linkage through the loop
will be less further away from the loop. [M1]

According to faraday’s law, when the magnetic flux linkage through the loop
changes with time, emf is induced in the loop. [A1]

OR

• Cutting of flux approach:


The current in the wire generates a magnetic field that decreases with
distance from it (B ∝ 1/r).
As the loop moves away, the sides AB and DC cut the magnet flux and thus

4
generate emf (A → B, D → C)

Since AB is nearer to the wire, it cuts flux at a greater rate than DC and
generates more emf. (εAB > εDC), hence there is a net emf that is able to
drive current around the loop.
A B

D C

(ii) Changing flux linkage approach:


Using Right Hand Grip Rule, the B field is pointing into the paper through
the loop. [C0]

When the loop is moving downwards, the magnetic flux linkage directed into
the paper through the loop is decreasing.
According to Lenz’s law, the induced current must produce a flux into the
paper in order to oppose the decrease.
Hence, the induced current flows clockwise. [M1]

Clockwise direction [A1]

OR

Cutting of flux approach:


Both AB and DC produce emf pointing to the right (A to B, D to C) when
moving downwards, as determined by Fleming’s Right Hand Rule
(Motion + B-field → induced emf). [M1]

ε ε
As AB > DC (as determined in (i)), there is net emf that drives a current
around the loop in a clockwise direction. [A1]

(iii) Movement of the loop away from the wire caused in a change in flux linkage
that resulted in an induced e.m.f. and current around the closed loop. [B1]

This results in power dissipated P = I2R as heat (Joule heating) as the


conducting loop has some resistance. [B1]

The power is supplied by the external force that keeps the loop moving at
constant speed, otherwise it would slow down. [B1]

(Another accepted explanation revolves around the work done to overcome


the attractive forces that are set up between the loop and the current in the
wire to keep it moving at constant speed.)

5
5 (a) The internal energy of a system is the sum of the potential energies and kinetic
energies of the molecules in the system B2

(b) (i) n is the number density or the number of molecules per unit volume B1

(ii) R
k= B1
NA

(c) (i) Isothermal process/ Phase change. B1


Melting or boiling B1

(ii) Adiabatic process B1


Releasing gas from a balloon rapidly or pumping air into the bicycle’s
tyre B1

6
Section B
Answer two questions in this section.

6 (a) (i)
First Law - An object will remain at rest or move along a straight line with constant
speed. [1] unless it is acted upon by a resultant force.
[1]
This implies that an object resist change of its state of rest or motion. A force is
needed to change the state of the body.

(a) (ii) units of F = units of m x units of a


= kg m s-2 [B1]

(b) (i) 150 km h-1 = 150 x 1000 / 3600 = 41.67 m s-1

v = u + at
41.666 = 0 + a (11)
a = 3.7879 m s-2 [M1]

Fnet = ma
= 1200 x 3.7879
= 4545.45 N [M1]

Fd - f = ma
Fd = 4545.45 + 1200
= 5745.45 N [A1]

(b) (ii) Pmax = Fd vmax


= 5745.45 x 41.67 [M1]
= 239.393 kW
= 239 kW (to 3 sf) [A1]

7
(c) (i) Loss in KE:
1 1
m ( u 2 − v 2 ) = (1200 ) ( 27.82 − 0 )
ΔK = 2 2 [B1]
= 463704 J

For x ≤ 1.5 m
W = Fx = (90000)(1.5) = 135000 J
For x ≤ 4.0 m
Wtot = (90000)(1.5)
+ (130000)(4.0 − 1.5) = 460000 J [C1]

As this is lesser than the loss in KE,


x0 > 4.0 m, [C0]
where x0 is the greatest distance the car moved into the cushion of
barrels before coming to rest.

By the Work Energy Theorem,


Loss in KE = work done against the force provided by the cushion of
steel barrels
463704 = 460000 + 170000(x0 − 4.0) [M1]
Hence x0 = 4.02m [A1]

(ii) The actual distance would be shorter than 4.02 m [A1]

because frictional forces would slow the car further and the car’s kinetic
energy would be converted into heat, sound and work involved in deforming
the car body. [M1]

(d) (i) Archimedes’ Principle states that an object completely or partially


submerged in a fluid experiences an upward buoyant force equal to the
weight of the fluid it displaces. [B1]

(ii) When no water enters, the car, weight of car = 1200 × 9.81 N
By the principle of flotation,
Weight of car = upthrust = weight of fluid displaced
1200 × 9.81 = Vfluid × 1000 × 9.81 [M1]

Vcar = Vfluid = 1.20 m3 [A1]

8
(iii) At the point at which the car first disappears completely below the water,

Weight of car and water = weight of fluid displaced by car


(1200 + mwater)g = (6.43) × 1000 × g [M1]

mwater = 6430 - 1200


= 5230 kg [M1]
volume of water in the car = 5230 ÷ 1000 = 5.23 m3 [A1]

7 (a) 1 Tesla is defined as the magnetic flux density of a uniform magnetic field when a
wire of length 1m, carrying a current of 1A, placed perpendicular to the field,
experiences a force of 1N in a direction at right angles to both the field and the
current. [B2]

(b) (i)

A B

α emitter

Uniform magnetic field of


4.0 kV
2.00 T directed into the
plane of the paper

Vacuum

Award [B1] if student’s answer shows evidence of a circular path and


particle is deflected upwards.

(ii) The magnetic force acting on the α-particle is directed at right angles to the
velocity of the particle. [B1]

The resulting acceleration is directed at right angles to the velocity of the


particle and thus will not alter the speed but merely changes its direction.
[B1] OR The magnetic force hence does not do work on the particle. [B1]

The kinetic energy of the particle thus does not change. [B1]

9
(iii) 1
mv 2 = qV
2

v=
(
2 2 × 1.6 × 10 −19 ) ( 4000 ) [M1]
6.644 × 10 −27
= 6.207 × 105 m s-1 [A0]

(iv) FB = FE
qvB = qE
E = vB
= (6.207 × 105 ) ( 2.00 ) [M1]
= 1.24 × 106 V m −1 [A1]
E Field is directed downwards [B1]

(c) (i) FB = Fc
mv 2
qvB =
r
mv
r=
qB

=
(
6.644 × 10−27 6.207 × 105 sin30o ) [M1]
( 2 × 1.6 × 10 ) ( 2.00)
−19

= 3.22 × 10−3 m [A1]

(ii) qα v y B = mα v y ω
⎛ 2π ⎞
qα B = mα ⎜ ⎟ [M1]
⎝T ⎠
2π mα
T = (shown)
qα B

(iii) 2π mα
T =
qα B

=
(
2π 6.644 × 10 −27 )
( 2 × 1.6 × 10 ) (2.00)
−19

= 6.522 × 10 −8 s [M1]

Pitch, p = v xT
( )
= 6.207 × 105 cos 30o × 6.522 × 10 −8 [M1]
= 0.0351 m [A1]

10
(iv) 2π m
For a positron, the period, T = will decrease [A1]
qB
1
since while qpositron = qα , mpositron << mα [M1]
2
for e.g.

mpositron 9.11× 10−31


= −19
= 5.69 × 10−12
qpositron 1.6 × 10
malpha 6.644 × 10−27
= −19
= 2.07 × 10−8
qalpha 2 × 1.6 × 10

mv
For the same reason, the radius r = will also decrease. [A1]
qB

Since period decreases [M0], the pitch p = v xT will decrease. [A1]

8 (a) (i) An accelerated electron from the cathode collides into an electron of a target
atom that is orbiting in the K-shell. [B1] If sufficient energy is transferred by
the accelerated electron to the orbiting electron, the orbital electron can be
ejected from the target atom. [B1]

When the vacancy in the K-shell (n = 1) is filled by an electron from the L-


shell (n = 2), an X-ray photon of the Kα characteristic X-ray is emitted. [B1]

If the vacancy in the K-shell is filled by an electron dropping from the M-shell
(n = 3), an X-ray photon of the Kβ characteristic X-ray is emitted. [B1]

Note:
As the energy differences between electrons in the various energy levels
are characteristics of the target atom, the wavelengths of the Kα and Kβ
characteristic X-rays are unique for each element.

(ii)
From the graph, λmin is the same for both spectra. [B1]
hc
eV =
λmin

11
(iii)
From the graph, λmin = 16 × 10 −12 m
hc
eV =
λmin

V=
( 6.63 × 10 )( 3.0 × 10 )
−34 8

(16 × 10 )(1.6 × 10 ) [M1]


−12 −19

= 7.77 x 104 V [A1]

(iv) Spectrum comprises contribution from continuous and characteristic


radiation i.e.

12
Difference in intensities between the two Kα lines is primarily due to a
difference in the continuous spectrum of tungsten and barium. [B1]

AND

The continuous radiation contribution for tungsten is more because


tungsten has a larger and more positive nuclei as compared to
Barium. Hence more electrons passing in the vicinity of the tungsten
nuclei are likely to deflect and give out an x ray photon. [B1]

Thus the continuous spectrum of tungsten has a higher intensity as


compared to Barium.

Alternatively, tungsten has a larger and more positive nuclei. Thus for
an electron at a fixed distance from the nuclei, the electron is likely to
deflect more and give out a higher energy x ray photon. The intensity
of the high energy x ray photons for tungsten is thus larger. [B1] Thus
the continuous radiation contribution for tungsten is more.

13
(v) For tungsten
hc
eVmin =
λα

Vmin =
( 6.63 × 10 )( 3.0 × 10 ) = 59 kV
−34 8

( 2.1× 10 )(1.6 × 10 )
−11 −19
[A1]

When the accelerating potential is reduced to 50 kV < 59 kV, i.e., less than
the minimum potential required (from (b) above), the characteristic X-rays
will not be generated
Sketch:

1. Characteristic wavelength will not be produced. [B1]


2. The minimum wavelength increases and intensity decrease. [B1]

b (i)
Heat dissipated = 1 x 300 x 16.5 = 4950 J

Power = 250 x 25 = 5000 W


5000 − 4950
Efficiency = × 100 [M1]
5000

= 1% [A1]

(c) (i) Any of these:


monochromatic, high intensity, coherent, minimal divergence. [B1]

(ii) Population inversion is a condition where there are more atoms in the excited
state as compared to ground state. [B1]

For lasing to occur, the number of photons that cause stimulated absorption is
clearly much less than the number of photons that produce stimulated emission.
(rate of stimulated emission is larger than rate of spontaneous absorption) [B1]

E3 is a metastable state with a comparatively higher life-time as compared to E2


allowing the accumulation of excited electrons to achieve population inversion.
[B1]

(iii) Optical pumping directly to Neon atom (without Helium collision) will result in
equal probability of upward and downward transition and stimulated absorption
and stimulated emission will then occur at the same rate [B1] because the
numbers of electrons in E3 and E1 will be the same at steady state. Population
inversion will not occur. [B1]

End of Paper
14
1

NATIONAL JUNIOR COLLEGE


PRELIMINARY EXAMINATIONS
Higher 2

CANDIDATE
NAME

SUBJECT REGISTRATION
CLASS NUMBER

PHYSICS 9646/01
Paper 1 Multiple Choice 20 September 2010
Additional Materials: Multiple Choice Answer Sheet 1 hour 15 minutes

READ THE INSTRUCTION FIRST

Write in soft pencil.


Do not use staples, paper clips, highlighters, glue or correction fluid.
Write your name, Centre number and index number on the Answer Sheet in the
spaces provided unless this has been done for you.

There are forty questions on this paper. Answer all questions. For each question there
are four possible answers A, B, C and D.
Choose the one you consider correct and record your choice in soft pencil on the
separate Answer Sheet.

Read the instructions on the Answer Sheet very carefully.

Each correct answer will score one mark. A mark will not be deducted for a wrong
answer.
Any rough working should be done in this booklet.

This document consists of 17 printed pages.


FOR NJC INTERNAL USE ONLY 9646/01/2010 [Turn Over
2

Data
speed of light in free space, c = 3.00 x 108 ms-1

permeability of free space, μ0 = 4π x 10-7 Hm-1

permittivity of free space, ε0 = 8.85 x 10-12 Fm-1

elementary charge, e = 1.60 x 10-19 C

the Planck constant, h = 6.63 x 10-34 Js

unified atomic mass constant, u = 1.66 x 10-27 kg

rest mass of electron, me = 9.11 x 10-31 kg

rest mass of proton, mp = 1.67 x 10-27 kg

molar gas constant, R = 8.31 JK-1mol-1

the Avogadro constant, NA = 6.02 x 1023 mol-1

the Boltzmann constant, k = 1.38 x 10-23 JK-1

gravitational constant, G = 6.67 x 10-11 Nm2kg-2

acceleration of free fall, g = 9.81 ms-2

Formulae
1 2
uniformly accelerated motion, s = ut + at
2
work done on/by a gas, W = p ΔV

hydrostatic pressure p = ρgh


Gm
gravitational potential, φ=−
r
displacement of particle in s.h.m., x = x0 sin ωt

velocity of particle in s.h.m., v = v 0 cos ωt and v = ±ω x0 − x 2


2

resistors in series, R = R1 + R2 + …
1 1 1
resistors in parallel, = + + ...
R R1 R2
Q
electric potential, V =
4πε 0 r
alternating current/voltage, x = x0 sin ωt

T = ex p(-2kd) Where k = 8π m(U − E )


2
Transmission coefficient 2
h
radioactive decay, x = x0 exp (-λt)
0 . 693
λ =
decay constant, t1
2

FOR NJC INTERNAL USE ONLY 9646/01/2010 [Turn Over


3

1 A stationary charged object is accelerated from rest through an electric field. Given the data
below, what is the final velocity of the object (with its uncertainty)?

Charge on the object = 0.200 ± 0.001 C


Mass of the object = 200 ± 1 g
Potential Difference across the electric field = 1000 ± 20 V

A 44.721 ± 0.015 ms-1


B 44.72 ± 0.03 ms-1
C 44.7 ± 0.7 ms-1
D 45 ± 1 ms-1

2 Which of the following estimations is correct?

A The weight of a pair of school shoes is 2.0 N.


B The surface area of a standard soccer ball is 0.15 m2.
C The current drawn by a laptop is 15 A.
D The volume of a pen is 4.0 x10-3 m3.

3 A car is travelling along a straight road from rest. The graph shows the variation with time t of
its acceleration a during part of the journey.
a
Y

0 V Z
X t

At which points on the graph does the car have its greatest velocity and greatest
displacement?
Greatest Greatest
velocity displacement
A V W
B X Y
C Y Z
D Z Z

4 A body is thrown vertically upwards in a medium in which the viscous drag cannot be
neglected. If the times of flight for the upward motion tU and the downward motion td (to return
to the same level) are compared, then

A td > tU , because the body moves faster on its downward flight and therefore the viscous
force is greater.
B td = tU, because the effect of the viscous force is the same whether the body is moving
upwards or downwards
C td < tU, because at a given speed the net force when the body is moving downwards is
greater than the net force when it is moving upwards.
D td > tU, because at a given speed the net force when the body is moving downwards is
smaller than the net force when it is moving upwards.
FOR NJC INTERNAL USE ONLY 9646/01/2010 [Turn Over
4

5 In the figure below, a helicopter is flying with a constant horizontal speed 35.0 ms-1 and 100
m above the ground and the tank is traveling with a constant horizontal speed 15.0 ms-1 and
is 550 m away from the helicopter.

helicopter

100 m
tank

550 m

Assuming at t= 0, the helicopter and the tank is at the above position, how many seconds
later should the helicopter drop a bomb so that it will hit the tank? Ignore air resistance.

A 4.51 s B 10.2 s C 23.0 s D 27.5 s

6 Water is ejected at a speed of 50 cms-1 on a wall from the nozzle of a hose with a
diameter of 1.0 cm. Find the force exerted on the wall, assuming that the water does not
rebound. The density of water is 1.0 gcm-3.

A 5.0 x 10-3 N
B 2.0 x 10-2 N
C 2.5 x 10-2 N
D 7.9 x 10-2 N

7 A bob hangs from the ceiling of a train when the train is moving at a constant velocity. After
some time later, the train accelerates forward at 4.91 ms-2. Determine the angle of inclination
of the bob to the vertical. The mass of the bob is 200 g.

A 26.6o, with the bob moved forward


B 26.6o, with the bob moved backward
C 63.4o, with the bob moved forward
D 63.4o, with the bob moved backward

FOR NJC INTERNAL USE ONLY 9646/01/2010 [Turn Over


5

8 Smith and Jones are skating on ice (assumed frictionless) so that they are moving with equal
speeds in the same straight line. Smith is skating backwards facing Jones. Smith throws a
ball to Jones at time t1 and receives it back at time t2. Assuming that the time of flight of the
ball is negligible, which one of the sketches below gives the correct speed-time relationship
for the two skaters?

A B
Speed Speed

Smith Smith

Jones
Jones

t1 t2 time t1 t2
time

C Speed D Speed

Smith
Smith

Jones
Jones

t1 t2
time t1 t2 time

9 A uniform rod of weight 10 N is freely hinged to a wall as shown below. The rod is supported
at the other end by a cable perpendicular to the rod. If a load of 18 N is suspended at the
other end of the rod, what is the tension in the cable?

cable

60o
rod

18 N

A 12 N B 16 N C 20 N D 28 N

FOR NJC INTERNAL USE ONLY 9646/01/2010 [Turn Over


6

10 One reason why the Titanic sank was because the captain ordered that the ship continued
its course despite knowing that there were icebergs ahead. Unfortunately icebergs are much
bigger than they appear, in fact only about 10% of the entire iceberg is actually above the
sea-level. What can be said about the density of sea water as compared with ice?

A Ice has a density which is about 0.9 times that of sea water.
B Ice has a density which is about 0.1 times that of sea water.
C Sea water has a density 0.9 times that of ice.
D Ice has a lower density than water but nothing more can be said about
their relative densities.

11 The graphs below were obtained from four different experiments. Which of the shaded areas
does not have units of energy?

Experiment Graph obtained


A Power output of a
lamp Power

0
time

B Compressing a gas
cylinder
Pressure

0 volume
C A ball thrown
horizontally Force of air
resistance

0
distance
D Motion of a space
craft away from the
Earth Gravitational field

0
distance

FOR NJC INTERNAL USE ONLY 9646/01/2010 [Turn Over


7

12 With the effect of air resistance, a cyclist and his bicycle of mass 80.0 kg can coast down a
5.0o (with respect to the horizontal) hill at a steady speed of 1.38 ms-1. If the air resistance,
Fair is proportional to the speed v so that Fair = cv (where c is a constant), then the additional
force that the cyclist must apply in order to descend the hill at a steady speed of 5.55 ms-1 is

A 107 N B 207 N C 307 N D 407 N

13 A mass m, attached to the end of an unstretched spring, is initially supported by a platform


as shown in Figure 13.1. This platform is then removed and the mass falls, eventually
coming to rest at the position shown in Figure 13. 2.

Figure 13.1 Figure 13.2

Which of the following correctly relates the changes in potential energy and heat dissipation
which may occur during the process?

A Decrease of gravitational potential = increase of elastic potential energy


energy + energy dissipated as heat
B Decrease of gravitational potential = decrease of elastic potential energy + energy
energy dissipated as heat
C Decrease of gravitational potential = increase of elastic potential energy
energy
D Decrease of gravitational potential = increase of elastic potential energy
energy + energy dissipated as heat

14 The earth has a radius of 6.38 x106 m, and rotates on its axis once every 24 hours. At what
latitude (i.e., the angle θ in the drawing) is the tangential speed one third that of a person
living at the equator?

A 20.5o
B 30.5o
C 60.5o
θ
D 70.5o
Equator

FOR NJC INTERNAL USE ONLY 9646/01/2010 [Turn Over


8

15 Which one of the following diagrams shows the variation of gravitational force F on a point
mass and gravitational potential energy U of the mass at a distance r from another point
mass?
F,U F,U F,U F,U

A B C D

16 An experimental satellite on Earth is found to have a weight W when assembled before


launching from a rocket site. It is placed in a circular orbit at a height h = 5R above the
surface of the Earth (of radius R).
What is the gravitational force acting on the satellite whilst in orbit?

A W B W C W D W
5 6 25 36

17 The acceleration due to free fall at the surface of Planet X was found to be equal to that at
the surface of Earth. If the diameter of Planet X were twice the diameter of Earth, what is the
ratio of the mean density of the planet to that of Earth?

A 0.25 B 0.5 C 2 D 4

18 The diagram below shows a displacement-time graph of a body performing simple harmonic
motion.

Displacement

Y Z

Time
U
W

At which one of the points, U, V, W, X, Y or Z, is the body travelling and accelerating in


the opposite direction?

A U, Y B V, X C W, Z D X, Z
FOR NJC INTERNAL USE ONLY 9646/01/2010 [Turn Over
9

19 Which of the statements is not true for a body vibrating in oscillations when damping is
present?

A The damping force is in the opposite direction to the velocity.


B The damping force is in the opposite direction to the acceleration.
C The presence of damping gradually reduces the maximum potential energy of the
system.
D The presence of damping gradually reduces the maximum kinetic energy of the
system.

20 A water tank of heat capacity 5000 J K-1 contains 10 kg of water at 25 oC. What is the time
taken to raise the temperature of the water to 45 °C using a heater coil of power of 3.0 kW,
given that the specific heat capacity of 4200 J kg-1 K-1 ?

A 61 s B 280 s C 310 s D 610 s

21 A fixed mass of ideal gas undergoes a cycle of changes as shown in the figure. At which
point on the graph is the gas hottest?

22 The figure below shows a setup where a laser beam is directed towards two polaroids. The
polaroids are adjusted such that zero intensity is detected by the light sensor. Without
changing the orientation of either Polaroid A or Polaroid B, how may we adjust the setup
such that the sensor detects a non-zero intensity?

Polaroid A Polaroid B

Laser Light sensor

A Place another Polaroid between the laser and Polaroid A.


B Place another Polaroid between Polaroid A and Polaroid B.
C Place another Polaroid between Polaroid B and the light sensor.
D There is no possible method.

FOR NJC INTERNAL USE ONLY 9646/01/2010 [Turn Over


10

23 A vibrating string fixed at two ends has a fundamental frequency of 250 Hz. When the same
string is vibrating in 750 Hz,

A it has 4 nodes.
B its wavelength is 3 times the original wavelength.
C it is vibrating in the second mode.
D its wave speed has increased by 3 times.

24 Which of the following methods would result in no change in fringe separation in a


double slit experiment, as shown below?

Light source

Single slit Double slit


Screen

A Conduct the double slit experiment in water.


B Use light of the same intensity but of different frequency.
C Keep the distance between the single slit and the screen constant, but move the double
slit closer to the single slit.
D Increasing the slit width of the double slits.

25 Which graph correctly relates the electric field strength or electric potential in the field of a
positive point charge, with a distance r from the charge?

A B

Field Field
strength strength

0
1/r
0 r2

C D

Potential Potential

0 0
1/r r
FOR NJC INTERNAL USE ONLY 9646/01/2010 [Turn Over
11

26 Two large parallel metal plates X and Y are situated in a vacuum as shown. The potential
difference between plate X and Y is V. A negatively charged particle with charge –q is
suspended stationary midway between the 2 plates. Which of the following statements is
false?

Plate X
+

Negatively
charged V
particle

-
Plate Y

A When V is halved, the particle will accelerate downwards with an acceleration of 0.5 g.
B When V is halved, the particle will reach plate X with kinetic energy equals 0.5 qV.
C When V is doubled, the particle will accelerate upwards with an acceleration of g.
D When V is doubled, the particle will reach plate X with kinetic energy equals qV.

27 The figure below shows a network of three resistance wires marked R1, R2 and R3. All three
wires have the same cross-sectional area A. Wire R1 has resistivity ρ and length l. Wire R2
has resistivity 2ρ and length 2 l, while wire R3 has resistivity 0.5ρ and length 2l What is the
resistance between X and Y?

ρl ρl ρl ρl
A 0.75 B 0.83 C 1.3 D 4.0
A A A A

FOR NJC INTERNAL USE ONLY 9646/01/2010 [Turn Over


12

28 Five light bulbs with difference resistances as indicated in terms of R in the diagram below
are connected to a constant voltage d.c. supply, E, as shown below. If bulb 1 blows as
shown below, what happens to the brightness of the remaining bulbs?
E

R 2R
2R
Bulb 1 Bulb 2
R 2R
Bulb 5

Bulb 3 Bulb 4

Bulb 2 Bulb 3 Bulb 4 Bulb 5


A Decreases Increases Decreases Decreases
B Increases Decreases Increases Decreases
C Decreases Decreases Decreases Decreases
D Decreases Increases Decreases Increases

29 Which of the following statements is false about the circuit shown?

A When switch K is closed, as the resistance R2 increases, the balanced length l


decreases.
B When switch K is open, as the resistance R2 increases, the balanced length l, remains
unchanged.
C When switch K is open, as resistance R1 increases, the balanced length l increases.
D Whether switch K is open or closed, changes in the internal resistance of E2 will
produce no change in the balanced length l.

FOR NJC INTERNAL USE ONLY 9646/01/2010 [Turn Over


13

30 An electron enters a region where there are uniform electric field and magnetic field of 0.50 T
acting at right angles to its horizontal velocity v, as shown in the figure below. The electron
remained undeflected when it enters the region. When the magnetic field is switched off, the
electron immediately moved a vertical displacement of 0.15 mm in 1.0 ns. Calculate the
speed of the electron v.
+

_
A 0.85 x 103 ms-1 B 3.4 x 103 ms-1 C 150 x 103 ms-1 D None of the above
answer

31 Five straight and parallel wires are arranged as shown in the diagram below. Wire A, C and
D carry a steady current I. Wire at B and E (at centre of wire) carry a steady current 2I.
Which of the following statements about the wire at E is correct?

A d B

E
d

D C
A It does not experience any force.
B It experiences a force in the direction of A
C It experiences a force in the direction of B.
D It experiences a force in the direction of D.

FOR NJC INTERNAL USE ONLY 9646/01/2010 [Turn Over


14

32 A wire of length 0.50 m, forming part of a complete circuit, is positioned at right angles to a
uniform magnetic field. The graph shows how the force acting on the wire due to the
magnetic field varies as the current in the wire is increased.

Force/mN

Current/A
The circuit is switched off. The same piece of wire is then strummed such that it vibrates with
an amplitude of 1.5 mm and at 100 Hz.
Calculate the maximum emf induced across the wire.

A 3.00 x 10-4 V B 1.88 x 10-3 V C 1.18 V D 1.88 V

33

Three vertical tubes, made from copper, lead and rubber respectively have identical
dimensions. Identical, strong cylindrical magnets P, Q and R are released simultaneously
from the same distance above each tube. The magnets are observed to emerge from the
bottom of the tubes at different times.

Which line, A to D, in the table shows the correct order in which they will emerge?
Resistivity of copper = 1.7 x 10-8 Ωm
Resistivity of lead = 22 x 10-8 Ωm
Resistivity of rubber = 50 x 1013 Ωm

Emerges first Emerges second Emerges last


A P Q R
B R P Q
C R Q P
D P R Q

FOR NJC INTERNAL USE ONLY 9646/01/2010 [Turn Over


15

34 The figure below shows the variation with time of a periodic current.
I/A
4

0
1 2 3 4 5 6 7 8 time / ms
−2

What is the root mean square value of the current?


A 2.50 A B 2.65 A C 2.82 A D 3.00 A

35 The diagram below shows a typical X-ray spectrum produced by a X-ray tube.

Which of the following statements is true about the x-ray spectrum?

A The continous background radiation is formed as incoming electrons lose different


amount of kinetic energy thus the wavelength of X-ray photons emitted span across a
continous spectrum.
B A high speed electron collides and knocks off an electron in an outer shell of the atom.
The vacancy in the outer shell is filled by an electron in the inner shell. A photon of
wavelength λ2 is emitted as a result of this de-excitation.
C Presence of λ1 is due to electrons losing some of its kinetic energy to be radiated out
as a single X-ray photon when it collide with the metal surface.
D The peak at λ 2 in the figure above is known as K β .

36 Which of the following statements is true about excitation of electron in an atom?

A An electron will be excited as long as the incoming free electron kinetic energy is
greater than the energy difference between the initial and any other energy states of
the atom.
B Electron will be excited as long as the photon’s energy is of the same value as any of
the energy state of the atom.
C When visible white light is shone onto an insulator, electrons are excited.
D Excitation of electrons will only take place when two hydrogen atoms collide elastically.

FOR NJC INTERNAL USE ONLY 9646/01/2010 [Turn Over


16

37 An electron whose total energy E is 6.0 eV is approaching a barrier whose height is 7.0 eV
and whose thickness is 0.60 nm. What is the transmission coefficient T for this electron?

A 3.13 x 10-4 B 5.63 x 10-4 C 2.15 x 10-3 D 4.63 x 10-3

38 A photodiode is a doped semiconductor to detect light. Which of the following statements is


true of a photodiode detector?

A Electron-hole pairs are created by each light photon absorbed in the depletion region
and the electrons will drift to the n-side and the holes will drift to the p-side.

B Electron-hole pairs are created by each light photon absorbed in the depletion region
and the electrons will drift to the p-side and the holes will drift to the n-side.

C Electron-hole pairs are created by each light photon absorbed in the entire p-n
semiconductor and the electrons will diffuse along the p-side and the holes will diffuse
along to the n-side.

D Electron-hole pairs are created by each light photon absorbed in the entire p-n
semiconductor and the electrons will diffuse along the n-side and the holes will diffuse
along to the p-side.

39 When a detector is pointed towards a radioactive sample that is 80.0 cm away and has a
half-life of 20 minutes, it gives an average count-rate of 78 s−1. In the absence of the source,
the average count-rate is 10 s−1. What average count-rate is expected 40 minutes later, with
the detector still pointed towards the sample but now located 40.0 cm away? Regard the
sample to be a point source of radiation.

A 17 s−1 B 27 s−1 C 68 s−1 D 78 s−1

FOR NJC INTERNAL USE ONLY 9646/01/2010 [Turn Over


17

40 The nucleus of an unstable element decays by α-particle emission into a stable daughter.
The process is observed in an experiment.
y y

x x
Graph 1 Graph 2
y y

x x
Graph 3 Graph 4

Number of daughter nuclei vs time Rate of alpha emissions vs number of


parent nuclei present
A Graph 2 Graph 3
B Graph 2 Graph 1
C Graph 4 Graph 3
D Graph 4 Graph 1

End of Paper

FOR NJC INTERNAL USE ONLY 9646/01/2010 [Turn Over


1

NATIONAL JUNIOR COLLEGE


PRELIMINARY EXAMINATIONS
Higher 2

CANDIDATE
NAME

SUBJECT REGISTRATION
CLASS NUMBER

PHYSICS 9646/02
Paper 2 Structured Questions 20 September 2010
Candidate answers on the Question 1 hour 45 minutes
Paper.

No Additional Materials are required.

For Examiner’s Use


READ THE INSTRUCTION FIRST
Write your subject class, registration number and name on all 1
the work you hand in.
Write in dark blue or black pen on both sides of the paper. 2
You may use a soft pencil for any diagrams, graphs or rough
working. 3
Do not use staples, paper clips, highlighters, glue or correction
fluid. 4
Answers all questions.
5
The number of marks is given in brackets [ ] at the end of each
question or part question. 6

Total

FOR NJC INTERNAL USE ONLY 9646/02/2010 [Turn over


2

Data
speed of light in free space, c = 3.00 x 108 ms-1
permeability of free space, μ0 = 4π x 10-7 Hm-1
permittivity of free space, ε0 = 8.85 x 10-12 Fm-1
elementary charge, e = 1.60 x 10-19 C
the Planck constant, h = 6.63 x 10-34 Js
unified atomic mass constant, u = 1.66 x 10-27 kg
rest mass of electron, me = 9.11 x 10-31 kg
rest mass of proton, mp = 1.67 x 10-27 kg
molar gas constant, R = 8.31 JK-1mol-1
the Avogadro constant, NA = 6.02 x 1023 mol-1
the Boltzmann constant, k = 1.38 x 10-23 JK-1
gravitational constant, G = 6.67 x 10-11 Nm2kg-2
acceleration of free fall, g = 9.81 ms-2
Formulae
1 2
uniformly accelerated motion, s = ut + at
2
work done on/by a gas, W = pΔV

hydrostatic pressure p = ρgh


Gm
gravitational potential, φ=−
r
displacement of particle in s.h.m., x = x0 sin ωt
v = v 0 cos ωt and v = ±ω x0 − x 2
2
velocity of particle in s.h.m.,

resistors in series, R = R1 + R2 + …
1 1 1
resistors in parallel, = + + ...
R R1 R2
Q
electric potential, V =
4πε 0 r
alternating current/voltage, x = x0 sin ωt
T = ex p(-2kd) Where k = 8π m(U2 − E )
2
Transmission coefficient
h
radioactive decay, x = x0 exp (-λt)
0 . 693
λ =
decay constant, t1
2

FOR NJC INTERNAL USE ONLY 9646/02/2010 [Turn over


3

Answer all questions in the spaces provided

1 (a) Distinguish between systematic and random errors.

[2]

(b) There are two possible methods of measuring the volume of a cylindrical
container. The first method is by measuring the inner diameter and the
height of the container and calculating the volume. The readings are as
follows:

Inner diameter = 2.57 ± 0.01 cm


Height = 7.8 cm ± 0.1 cm

The second method is by filling the container with water, and then pouring
the water out into a measuring cylinder. The measuring cylinder can read
to the nearest cm3. With the aid of suitable calculations, explain which
method would give more precise value of the volume.

[3]

FOR NJC INTERNAL USE ONLY 9646/02/2010 [Turn over


4

2 Fig. 2.1 shows an improved water collection system.


dirty water from roof
pulley

clean water tipping gutter

bucket Water tank

Fig. 2.1
The uniform tipping gutter of mass 50.0 g is supported by the edge of the water
tank and by a rope that passes over a pulley. The other end of the rope is
attached to a bucket. By trial and error, the set-up is adjusted so that, when the
bucket is empty, the tipping gutter slopes down towards the bucket and all
parts remain at rest. The empty bucket is 30 cm tall and is about 10 cm
underneath the left end of the gutter.

A simplified diagram when the bucket is empty is as shown below.

5 cm
10 cm
30 cm

(a) (i) Indicate the all the forces acting on the gutter in a free body diagram
when the bucket of base area = 0.10 m2 is empty. Identify and label
the forces. [1]

FOR NJC INTERNAL USE ONLY 9646/02/2010 [Turn over


5

(ii) Assuming the gutter measures 1.5 m long, determine the mass of
the empty bucket. [2]

Mass =…………………kg
(b) (i) When the rain starts to fall, dirty water from the roof flows into the
suspended bucket. Explain how this system enables clean water to
be collected in the water tank.
Assume that the roof and gutter are cleaned up when the gutter is at [1]
a horizontal position.

(ii) In practice, the pulley is not frictionless which offers an advantage.


Suggest what this can be. [1]

(c) In a downpour, the rainwater is flowing from the roof into the gutter at a
rate of 50 cm3 s-1 and has a speed of 0.50 ms-1 when it leaves the gutter.
Density of rainwater = 1000 kgm-3.

(i) Determine the force exerted by the rainwater on the bucket when it
first falls into the bucket. [3]

Force = …………………….N

FOR NJC INTERNAL USE ONLY 9646/02/2010 [Turn over


6

(ii) In 2c (i), you may have made assumption about the final speed of
the water upon contact with the bucket. State and explain what
would happen to your answer in 2c (i) if the assumption fails.
[2]

3 -2q

-q +4q

-q

Fig. 3.1
Fig. 3.1 shows three negative point charges (-q, -q, -2q) and one positive
point charge (+4q), along with some electric field lines drawn between the
charges. The 3 negative charges are at the same distance from the positive
charge.

(i) Identify and explain any 2 of the mistakes in this figure.

[2]

(ii) In the space below, draw the correct distribution of electric field lines
[2]

FOR NJC INTERNAL USE ONLY 9646/02/2010 [Turn over


7

(iii) If the distance from the +4q charge in fig.3.1 to all the other 3 charges is
x, determine in terms of q and x the work done by the external agent in
order to bring the +4q charge to infinity. [2]

Work done = ………………………………..

4 (a) State the laws of electromagnetic induction.

[2]

(b) An accelerometer in a vehicle consists of a steel bar mounted on two flexible


blades. The bar passes through two coils, as shown in Fig. 4.1. An alternating
voltage is applied to the primary coil.

Fig. 4.1

(i) Explain why a voltage is induced in the secondary coil.

[2]

FOR NJC INTERNAL USE ONLY 9646/02/2010 [Turn over


8

(ii) Explain why the bar is displaced from its equilibrium position when the
vehicle accelerates in a direction parallel to the axis of the bar, as shown
in Fig. 4.2.

Fig. 4.2

[2]

(iii) Explain why the amplitude of the secondary voltage changes when the
bar is displaced from its equilibrium position in a direction along the axis
of the bar.

[2]

5 One of the reasons why earthquakes can cause such devastation is that they can
cause buildings to resonate. For a multi-storey building, it is found that the natural
period of oscillation is approximately 0.10 s multiplied by the number of storeys.

(a) A typical earthquake oscillation has a frequency of 2.5 Hz.

(i) Estimate the number of storeys that a building would have in order
to resonate at this frequency. [1]

Number = ………………

FOR NJC INTERNAL USE ONLY 9646/02/2010 [Turn over


9

(ii) Sketch a well-labeled graph showing how the horizontal


displacement of the building might be expected to change with
time just after the earthquake had finished. Assume that the
building was not damaged. Show three complete cycles of
oscillation in your graph.
[2]

(iii) Explain with appropriate calculations why a building of 10 storeys


more than that in part (a)(i) would be less likely to be damaged by
the earthquake. [1]

(iv) The diagram below shows two buildings of different heights


situated close together. Explain why the two buildings are more
likely to suffer damage than if they were built further away from
each other. [1]

FOR NJC INTERNAL USE ONLY 9646/02/2010 [Turn over


10

(b) Buildings in earthquake-prone region are not closely built to minimize


damage due to collision of buildings. When a tall building vibrates at
1.5 Hz, the maximum acceleration of the top of the building is 107 ms-2.
Assuming that the vibration is simple harmonic, calculate the minimum
separation between the buildings in regions where the typical
earthquake frequency is 1.5 Hz. [2]

Minimum separation = ………………………m

6 With the help of the energy level diagrams given below, describe how the He-Ne
laser produces a continuous red laser output.

E3*
E2
632.8 nm laser

Excitation by Excitation E2*


electric current through
collision with
He atoms

E1 E1*

Helium Energy Levels Neon Energy Levels

FOR NJC INTERNAL USE ONLY 9646/02/2010 [Turn over


11

[5]

7 This question asks you to consider the force a space cabin exerts on an astronaut from take
off until the cabin is docked alongside the International Space Station (ISS). The ISS is in a
permanent orbit of radius 6.71 × 106 m around the Earth. The radius of the Earth is
6.37 × 106 m. The graph in Fig. 7.1 shows how the magnitude of the force the cabin exerts
on the astronaut varies over a period of time from just before blast off until docking takes
place. Fig. 7.2 shows how the gravitational field strength, g, of the Earth varies with distance
from the Earth’s surface.

Fig.7.1 Fig.7.2

(a) Use the graphs to determine

(i) the mass of the astronaut [1]

mass =................... kg

(ii) the maximum force applied to the astronaut [1]

force =.................... N

(iii) the pull of the Earth on the astronaut when 200 km from the Earth’s surface. [1]

FOR NJC INTERNAL USE ONLY 9646/02/2010 [Turn over


12

pull of Earth at 200 km =................... N


(b) Explain why
(i) the rocket does not produce a constant acceleration even when the thrust is constant.

[2]

(ii) the area under the force–time graph, of Fig. 7.1, will not equal the increase in the
momentum of the astronaut.

[1]

(c) Show that the value of the centripetal acceleration of the ISS is 8.83 m s–2.
[2]

(d) Calculate

FOR NJC INTERNAL USE ONLY 9646/02/2010 [Turn over


13

(i) the speed of the ISS [2]

speed = ................ ms–1

(ii) the kinetic energy of the astronaut as she circles the Earth in the ISS. [1]

kinetic energy = ................ J

FOR NJC INTERNAL USE ONLY 9646/02/2010 [Turn over


14

(e) In the weightless conditions of space astronauts on long missions in the ISS may lose
mass due to muscle deterioration. Scientists investigated a model of a spring system (see
Fig.7.3) which will operate in such conditions and could be used to monitor changes in
mass.

~0.4 m

Fig. 7.3

It has been suggested that the relationship between the period of oscillation T and the
mass M is given by:

1 p
2
= +q
T M
p is a constant for a fixed number of springs and q is a constant.

The experimental results obtained are shown below in Fig.7.4. The time taken for the
oscillation was measured using a stopwatch.

Time taken for 20


oscillations
M/kg t1/s t2/s T/s 1/T2/ S-2 1/M/kg-1

0.100 10.34 10.38 0.5180 3.727 10.0

0.150 12.07 11.93 0.6000 2.778 6.67

0.200 13.39 13.43 0.6705 2.242 5.00


0.250 14.34 14.32 0.7165 1.948 4.00
Fig. 7.4
0.300 15.23 15.23 0.7615 1.724 3.33

0.350 15.70 15.64 0.7835 1.629 2.86

The corresponding values of 1/T2 and 1/M for the data in Fig. 7.4 are plotted on the graph
of Fig. 7.5.
FOR NJC INTERNAL USE ONLY 9646/02/2010 [Turn over
15

Fig. 7.5

(i) On Fig. 7.5, draw the best-fit line for all the plotted points. [1]

(ii) Determine the values of p and q. [3]

p = …………………………

q = …………………………

FOR NJC INTERNAL USE ONLY 9646/02/2010 [Turn over


16

(iii) Hence comment on the validity of the relation given in 7(e). Explain your answer.

[1]

(iv) According to medical opinion a mass loss of 10% is considered serious. It has been
2 pT 3
suggested that ΔT = x (percentage mass loss). Determine the variation of
M
period ΔT corresponding to such a percentage loss in a mass of 0.5 kg. Discuss
whether this experiment is accurate enough to detect this variation.

[3]

FOR NJC INTERNAL USE ONLY 9646/02/2010 [Turn over


17

8 Under normal conditions, air is a poor conductor of electricity. However, when


the electric field in the air reaches a breakdown value, the electrons are stripped
or ‘ionised’ from the air molecules. This ionised air allows the sudden and
massive flow of electric current. The enormous amount of heat associated with
this current forms a spark.

Design an experiment to investigate how the minimum voltage needed to


generate a spark across a fixed gap of air varies with temperature of the air in a
chamber. You should appreciate that the breakdown voltage of air is affected by
the thickness, pressure and temperature of air.

You may assume that the following equipment is available, together with any
other apparatus which may be found in a college science laboratory.

Air chamber which is not resistant to flame.


High voltage power supply
Electrodes
Vacuum or Air Pump

You should draw diagrams to show the arrangement of the apparatus. In your
account, you should pay particular attention to
(a) how the temperature of the air can be varied and the pressure of the air in
the chamber could be monitored.
(b) the procedure to be followed
(c) the control of variables
(d) any safety precautions you would take. [12]

FOR NJC INTERNAL USE ONLY 9646/02/2010 [Turn over


18

Diagram

FOR NJC INTERNAL USE ONLY 9646/02/2010 [Turn over


19

FOR NJC INTERNAL USE ONLY 9646/02/2010 [Turn over


20

-END OF PAPER-
FOR NJC INTERNAL USE ONLY 9646/02/2010 [Turn over
1

NATIONAL JUNIOR COLLEGE


PRELIMINARY EXAMINATIONS
Higher 2

CANDIDATE
NAME

SUBJECT REGISTRATION
CLASS NUMBER

PHYSICS 9646/03
Paper 3 Longer Structured Questions 2 Sep 2010
Candidate answers on the Question Paper.
No Additional Materials are required. 2 hours

Section A
READ THE INSTRUCTION FIRST For Examiner’s
Write your subject class, registration number and name on all the Use
work you hand in.
1
Write in dark blue or black pen on both sides of the paper.
You may use a soft pencil for any diagrams, graphs or rough
working. 2
Do not use paper clips, highlighters, glue or correction fluid.
Answers all questions.
3
You are advised to spend one hour on each section.

The number of marks is given in brackets [ ] at the end of each 4


question or part question.
Total

NJC (FOR INTERNAL USE ONLY)  9646/03/2010  [Turn over 


2

Data
speed of light in free space, c = 3.00 x 108 ms-1

permeability of free space, μ0 = 4π x 10-7 Hm-1

permittivity of free space, ε0 = 8.85 x 10-12 Fm-1

elementary charge, e = 1.60 x 10-19 C

the Planck constant, h = 6.63 x 10-34 Js

unified atomic mass constant, u = 1.66 x 10-27 kg

rest mass of electron, me = 9.11 x 10-31 kg

rest mass of proton, mp = 1.67 x 10-27 kg

molar gas constant, R = 8.31 JK-1mol-1

the Avogadro constant, NA = 6.02 x 1023 mol-1

the Boltzmann constant, k = 1.38 x 10-23 JK-1

gravitational constant, G = 6.67 x 10-11 Nm2kg-2

acceleration of free fall, g = 9.81 ms-2

Formulae
1 2
uniformly accelerated motion, s = ut + at
2
work done on/by a gas, W = pΔV

hydrostatic pressure p = ρgh


Gm
gravitational potential, φ=−
r
displacement of particle in s.h.m., x = x0 sin ωt
v = v 0 cos ωt and v = ±ω x0 − x 2
2
velocity of particle in s.h.m.,

resistors in series, R = R1 + R2 + …
1 1 1
resistors in parallel, = + + ...
R R1 R2
Q
electric potential, V =
4πε 0 r
alternating current/voltage, x = x0 sin ωt
T = ex p(-2kd) Where k = 8π m(U2 − E )
2
Transmission coefficient
h
radioactive decay, x = x0 exp (-λt)
0 . 693
λ =
decay constant, t1
2

NJC (FOR INTERNAL USE ONLY)  9646/03/2010  [Turn over 


3

Section A
Answer all the questions in this section.

1 (a) Define the term angular velocity. [1]

(b)(i) A 10 kg baggage is left on a rotating baggage carousel at an airport. The baggage stays
at a fixed position on the slope of the carousel and rotates about in a circle
(r = 11.0 m) at a constant speed. The frictional force acting on the suitcase is 59.4 N.

Direction of rotation
r

θ = 36.0º

Side View

Use Newton’s Laws to explain why the baggage will experience a net force towards the
centre of the circle. [2]

(ii) Show on a fully labelled diagram the forces acting on the baggage. [2]

NJC (FOR INTERNAL USE ONLY)  9646/03/2010  [Turn over 


4

(iii) Considering the forces acting on the baggage in the vertical direction, show that the
normal contact on the baggage is about 78.1 N. [2]

(iv) How much time is required for the suitcase to complete one full rotation? [3]

Time taken = ………………..

2 A monoatomic ideal gas is subject to a cycle of changes ABCA. Figure 2 shows a graph of
pressure p against volume V for one cycle of changes for the gas.

p /105 Pa

V /10 - 4 m3
Figure 2
(a)(i) Using data from the graph, verify that process BC is isothermal. Show your workings
clearly. State an assumption of the gas you must make to support your verification.
[2]

NJC (FOR INTERNAL USE ONLY)  9646/03/2010  [Turn over 


5

(ii) Explain the term internal energy in relation to an ideal gas. [1]

(b) Temperature of the gas at point C is 385 K. Calculate the temperature of the gas
in oC at point A. [1]

Temperature of the gas = ………..………….oC

(c)(i) Calculate the change in the internal energy of the gas during the process AB. [2]

Change in the internal energy = ……………….. J

(ii) Work is done by the gas in the change AB. State what must be done to the
system for this change to occur. Explain using the first law of thermodynamics. [2]

NJC (FOR INTERNAL USE ONLY)  9646/03/2010  [Turn over 


6

(d) Use the Kinetic Theory of gases to explain why the pressure of an ideal gas
increases in the change BC when it contracts at constant temperature. [2]

3 (a) Describe how an emission line spectrum can be produced in the laboratory. Describe
the appearance of the emission line spectrum when viewed through a grating
spectrometer. [3]

(b)(i) The experiment below confirms that electrons occupied only discrete, quantized energy
states.

Q R

Figure 3.1 Figure 3.2


Schematic diagram apparatus Graph of anode current Ia against V1

Electrons emitted at the cathode C are accelerated by a potential difference of V1


toward a positively charged grid G, in a glass tube filled with mercury vapor. Beyond
the grid is an anode A, held at a voltage of V2 of 1 V negative with respect to the grid.

A graph of anode current Ia against V1 is shown in Figure 3.2.

NJC (FOR INTERNAL USE ONLY)  9646/03/2010  [Turn over 


7

The values of accelerating voltage where the current dropped gave a measure of the
energy necessary to force an electron to an excited state.

Account for the shape of the graph when


1. V1 is less than P. [1]

2. V1 is between P and Q. [1]

(ii) As shown in Figure 3.2, when the accelerating voltage reaches 4.9 V, the current
sharply drops, indicating the sharp onset of a new phenomenon. Suggest with
explanation what the new phenomenon is. [3]

(iii) 1. Using the values from the graph in Figure 3.2, find the wavelength of the radiation
emitted by the mercury atoms as they return to their ground state. [1]

Wavelength = ……………….. m

2. State the region of the EM radiation which the wavelength calculated in b(iii)(1) can
be found. [1]

NJC (FOR INTERNAL USE ONLY)  9646/03/2010  [Turn over 


8

4 (a) Define the ohm. [2]

(b)(i) The figure below shows a potentiometer setup where the potentiometer wire, ab, is
uncalibrated. Es is a known standard cell. Describe how it is used to measure the emf of
the unknown source Ε x . [2]

a c b

switch
Ex
G

Es

(ii) Discuss one advantage of using the potentiometer setup to measure the emf Ex.
[1]

NJC (FOR INTERNAL USE ONLY)  9646/03/2010  [Turn over 


9

(c) The potentiometer wire ab of length 1 metre has a resistance of 600 Ω . The rheostat,
R, has a resistance 400 Ω for the entire length of 50 cm. The previous circuit has been
altered as follows:
E = 5.0 V Rheostat, R = 400 Ω

0 cm 50 cm
midpoint

a c b

r = 10 Ω
G
Ex = 2.0 V

(i) Determine the balance length, ac. [2]

ac = ……………….. m

(ii) State the direction of the current flowing through the dry cell, Ex, when the rheostat R is
adjusted from the midpoint to the right at the 40 cm mark. [1]

(iii) Find the new balance length, ac’. [2]

ac’ = ……………….. m

NJC (FOR INTERNAL USE ONLY)  9646/03/2010  [Turn over 


10

BLANK PAGE

NJC (FOR INTERNAL USE ONLY)  9646/03/2010  [Turn over 


11

NATIONAL JUNIOR COLLEGE


PRELIMINARY EXAMINATIONS
Higher 2

CANDIDATE
NAME

SUBJECT REGISTRATION
CLASS NUMBER

PHYSICS 9646/03
Paper 3 Longer Structured Questions 2 Sep 2010

Candidate answers on the Question Paper. 2 hours


No Additional Materials are required.

Section B For Examiner’s Use


Answer any two questions.
5
You are advised to spend one hour on each section.

The number of marks is given in brackets [ ] at the end of


each question or part question.
6
Circle the questions you attempted.

Submit Section A and B separately.

Total

NJC (FOR INTERNAL USE ONLY)  9646/03/2010  [Turn over 


12

Section B
Answer two questions in this section.

5 (a) 226 222


88 Ra
is a stationary radioactive isotope which decays to 86 Rn with the release of
an alpha particle.

(Mass of 24 He = 4.00260 u, mass of proton = 1.00783 u,


mass of neutron = 1.00867 u, mass of 222
86 Rn = 222.018 u)
(i) Define binding energy of a nuclide and explain how this quantity could be a measure of
the stability of a nuclide. [2]

(ii) Given the binding energy per nucleon of 226


Ra is 7.66831 MeV, show that its mass is
88
226.025 u. [2]

(iii) Starting from first principles, show that



Q = Kα ( 1 + )
M Rn
where Q is the energy released in the decay reaction, Kα is the kinetic energy of the
alpha particle, Mα is the mass of the alpha particle and MRn is the mass of 222
86 Rn . [3]

NJC (FOR INTERNAL USE ONLY)  9646/03/2010  [Turn over 


13

(iv) Hence, calculate Kα. [3]

Kα = ……………….. J

(v) Radon (Rn) decays by alpha emission to polonium and a tube containing an isotope of
radon is to be implanted in a patient. Suggest and explain two reasons why an alpha
emitter is preferred to the beta or gamma emitter for such purpose. [2]

(b) The graphs below show the activity of two samples of sodium nuclides, X and Y.

NJC (FOR INTERNAL USE ONLY)  9646/03/2010  [Turn over 


14

(i) Define activity and half-life of a radioactive nuclide. [2]

(ii) number of undecayed X nuclei


Determine the ratio when the activities of the two
number of undecayed Y nuclei
samples are the same. [2]

Ratio =………………..

(iii) How would you tell from the graphs, as drawn, that the background radiation is
negligible? [2]

(iv) Explain clearly how you would show that the activity of the nuclides decay
exponentially. [2]

NJC (FOR INTERNAL USE ONLY)  9646/03/2010  [Turn over 


15

6 (a) State what is meant by coherent waves. [1]

(b)(i) Two coherent sources of sound waves are located at position X and Y as shown in
Figure 6.1 below. The sources have zero phase difference. An observer stands at
position O. If the frequency of the sound wave is 660 Hz, with suitable calculations,
determine whether or not the observer experiences constructive or destructive
interference. (Take the speed of sound to be 330 ms-1) [3]

Sound source at X Sound source at Y

9m

12 m

Observer at O
Figure 6.1

(b)(ii) The source at position Y is slowly moved to the right until it eventually reaches
position Y’, as shown in Figure 6.2. Describe what is experienced by the observer at
O while the source is being moved. [2]

Sound source at X Sound source at Y’

9m 9m

12 m

Observer at O

Figure 6.2

NJC (FOR INTERNAL USE ONLY)  9646/03/2010  [Turn over 


16

(b) Would you expect the observer to hear complete silence when there is destructive
(iii) interference? Explain. [1]

(c) The setup in (b)(i) has been changed by replacing one of the sound sources with a
reflecting plane (as shown in Fig 6.3). With suitable calculations, describe what is
experienced by observer at O. (Note that reflected wave from a hard surface
undergoes a phase change of π radian with respect to the incident wave.
[3]

Sound source at Y

8m

12 m
Reflecting
plane

Observer at O
Fig 6.3

(d) Explain what is meant by diffraction of a wave. [2]

NJC (FOR INTERNAL USE ONLY)  9646/03/2010  [Turn over 


17

(e) A simplified model of the way the human eye responds to light of different
wavelengths incident normally on a diffraction grating of spacing d = 2.5 µm is as
follows:

Light: Perceived as:


Single wavelength between 0.40 µm and 0.50 µm Blue
Single wavelength between 0.50 µm and 0.60 µm Green
Single wavelength between 0.60 µm and 0.70 µm Red

Determine whether there is any overlapping between the first order and second order
spectra. [3]

(f) The spectrometer setup below shows how light from a collimator is made to fall
normally on a diffraction grating.

θ = θ 2 − θ1

The telescope can be used to locate the second order bright fringes of any particular
wavelength, λ at angular positions θ1 and θ 2 . Sodium vapour lamp of wavelength
589.3 nm is first used and the angle between θ 2 and θ1 , θ is shown in the table
below. The sodium vapour light is then replaced by a discharge tube containing a
mixture of gases and the θ values are recorded in table below for two pairs of
second order bright fringes.
Gas θ = θ 2 − θ1
Sodium 90.033°
Unknown 1 71.367°
Unknown 2 93.667°

NJC (FOR INTERNAL USE ONLY)  9646/03/2010  [Turn over 


18

(i) State the purpose of placing a single slit before the light source. [1]

(ii) Identify the gases (unknown 1 and unknown 2) in the tube by using the data
in the table below which shows the wavelength of the spectral lines emitted
by various gases.
[4]
Gas Wavelength/ nm
Helium 668
Carbon dioxide 608
Hydrogen 486
Oxygen 441

NJC (FOR INTERNAL USE ONLY)  9646/03/2010  [Turn over 


19

7 (a)(i) Define power. [1]

(ii) Derive an equation for power in terms of force and velocity. [2]

(b) Solar Impluse (see figure below), a plane powered by sunlight, ended its flight in
Switzerland on 8 Jul 2010 after remaining aloft for 26 hours. It was able to fly in the
darkness powered entirely by the energy its batteries had stored during the daytime
flight.
Solar cells
Electric Engine

Solar Impulse

(i) Describe qualitatively the energy transformation that takes place during the flight. [3]

NJC (FOR INTERNAL USE ONLY)  9646/03/2010  [Turn over 


20

(ii) During the day, the average intensity of the sunlight received by the plane was 250
Wm-2. The solar panel covers an area of 200 m2. The average power achieved by the
plane’s four engines is 6.0 kW.

1. Show that the efficiency of the solar cells and its battery system is about 12%. [2]

2. The average flying speed of the aircraft is 70 kmh-1. Determine the magnitude of
the air resistance acting on the aircraft. [2]

Air resistance = ……………..….. N

3. For a daylight period of 14 hours, calculate the solar energy needed to be stored in
the battery so as to complete the entire flight. [2]

Energy required = ……………..… J

(c) The aircraft is powered by 12,000 solar cells. Solar cells use p-n junctions to convert
sunlight directly into electricity. With the aid of a diagram, discuss qualitatively the
origin of the depletion region in a p-n junction. [4]

NJC (FOR INTERNAL USE ONLY)  9646/03/2010  [Turn over 


21

(d)(i) To provide a useful supply for the plane, a bank of solar cells consists of many cells
connected in a series and parallel array. The figure below shows the arrangement,
using a smaller number of cells than is used in practice.

Selection B

Selection A

Give one advantage for connecting the cells

1. in series as shown in selection A. [1]

2. in parallel as shown in selection B. [1]

(ii) Even if the arrangement of supplies as shown above are installed and in working order,
there may be still no power available. Explain why this could happen and what might
be done to provide suitable back-up power. [2]

End of Paper

NJC (FOR INTERNAL USE ONLY)  9646/03/2010  [Turn over 


NATIONAL JUNIOR COLLEGE PHYSICS DEPARTMENT

2010 Prelim H2 Physics Paper 1 Answer Key


1 C
2 B
3 D
4 D
5 C
6 B
7 B
8 C
9 C
10 A
11 D
12 B
13 A
14 D
15 C
16 D
17 B
18 A
19 B
20 C
21 D
22 B
23 A
24 D
25 C
26 B
27 C
28 A
29 A
30 B
31 D
32 B
33 C
34 B
35 A
36 A
37 C
38 A
39 D
40 D
NATIONAL JUNIOR COLLEGE PHYSICS DEPARTMENT

2010 H2 Prelim Physics Paper 2 Suggested Solutions


1(a)
A systematic error is one that occurs consistently more or consistently less than the
actual reading. [1]
A random error is one that gives a scatter of readings about a mean value. [1]

(b)
Method 1:
Volume = π(d/2)2h = 40.46 cm3
Δvol/vol = Δh/h + 2(Δd/d) = 0.0206 [1]
Δvol = (0.0206)(40.46) = 0.8 cm3 [1]

Method 2
Δvol = 1 cm3 (given by question)

Conclusion:
Method 1 has a Δvol that has 1 more d.p. than Method 2 [1], hence Method 1 is more
precise.

2
(a) (i) [1]

1 mistake : minus 0.5 mark


2 mistakes : zero mark
(ii) The gutter is in rotational equilibrium. [2]
Taking moment about the right side of the gutter,

Tcosθ (1.5) = Mgcosθ(1.5/2) where θ is the angle of inclination.


T = M/2(9.81)

Since T = weight of empty bucket, hence mass of empty bucket =


M/2 = 25.0 g

(b) (i) Dirty water flows into the bucket. The heavier bucket now provides a [1]
clockwise moment about the pivot that is greater than that by the
weight of the gutter.
The gutter is lifted and subsequent clean water flows along the gutter
into the tank.

(ii) The bucket will only gradually start to move downwards as it fills with [1]
water and hence the motion is less jerky.

FOR NJC INTERNAL USE ONLY 1


NATIONAL JUNIOR COLLEGE PHYSICS DEPARTMENT

(c) (i) Considering the vertical motion: [3]


vf2 = 2gh + vi2
velocity of water just before hitting bottom of bucket vf = 2.80 ms-1,
where h = 30+10 cm and vi = 0.50sinθ, sin-1θ = 5/150

Force on bucket by water, assuming all water comes to rest upon


impact
dm dV
= (2.80 – 0) = (2.80 – 0) ρ
dt dt
50x10-6(1000)(2.80-0) = 0.140 N

(ii) The assumption made in (c)i is that the final speed of the water is [2]
zero. Otherwise, the water will rebound. The change in momentum of
the water and hence the force on the bucket will be larger.

3 (i)
Field lines should not cross one another.
Fields line should originate or enters the charge perpendicularly.
The number of field lines interacting with the -2q charge by the +4q charge should be
more than the lines interacting with the –q charge.
Straight parallel field lines are used for uniform electric field. The field lines
interacting by the +4q should not be parallel as it is a non-uniform electric field.
The shape of the electric field lines interacting 2 charges of different magnitude are
asymmetrical.

(ii)

-2q

-q +4q

-q

−q −q − 2q
(iii) Total electric potential at the +4q charge, V = + +
4πε o x 4πε o x 4πε o x

FOR NJC INTERNAL USE ONLY 2


NATIONAL JUNIOR COLLEGE PHYSICS DEPARTMENT

− 4q
=
4πε o x

Work done by external agent, W = Q ΔV


− 4q
= (+4q) ( 0- ( )
4πε o x
16q 2
=
4πε o x

4(a) Faraday's Law states that the magnitude of the induced emf E in a circuit is
directly proportional to the rate of change of flux-linkage or to the rate of cutting
of magnetic flux. Lenz's Law states that the direction of the induced current
tends to oppose the change in magnetic flux that causes it.

(b)(i) The alternating current in the primary coil produces an alternating magnetic
flux in the bar which links to the secondary coil. Since there is a rate of change
in the magnetic flux linkage, there will be induced alternating voltage in the
secondary coil.

(ii) The bar is displaced from its equilibrium position because there is force
acting on it to accelerate it together with the vehicle. This accelerating force is
provided by the bending of the blades.

(iii) Since the magnetic flux in the bar is non-uniform, the magnetic flux in the
secondary coil changes when bar is displaced. By Faraday’s law, since the
secondary voltage is proportional to rate of change of magnetic flux, the
amplitude of secondary voltage changes.

Note: For this qn, the magnetic flux at that point in the bar decreases with
increasing distance away from primary coil. The area of bar at which the primary
coil enclosed has constant magnetic flux.

FOR NJC INTERNAL USE ONLY 3


NATIONAL JUNIOR COLLEGE PHYSICS DEPARTMENT

5
(a) (i) Let n be the number of storeys. [1]

N (0.1) = 2/5
N=4
(ii) Exponential decay in amplitude; constant period or slightly [1]
reduced; period labeled on the graph.
(iii) A building of 14 storeys will have natural frequency (0.714 Hz) [1]
which is smaller than the driving frequency of 2.5 Hz. [B1]
Hence resonance does not take place.

(iv) If one of the buildings have a natural frequency that matches the [1]
driving frequency of an earthquake.[B0.5] That building will topple
and crash into the other building thus causing more damages and
casualties.[B0.5]
(b) At maximum displacement, acceleration, a = ω2xo = (2πx1.5)2(xo) = 107 [2]
ms-2
Hence amplitude of vibration of one building is 1.2 m. [1]
Hence minimum building separation = 1.2 + 1.2 = 2.4 m. [1]
(c) When the natural frequency matches the driving frequency, the building which [1]
undergoes forced oscillation will resonate, meaning it vibrates with large
amplitude due maximum energy transfer from the earthquake to it.

Q6. The He-Ne laser consists of a mixture of gaseous He and Ne atoms.

An input power/ optical pumping is passed through the sample and this excites the
atoms in the sample. [1 m]

The important feature in the He-Ne laser is that the E2 state of He is a meta-stable state
and the E3 state of Ne is also a meta-stable state and since the both energy difference
is very close to each other, the excited He atoms can transfer its energy rather efficiently
and continuously to the ground state Ne atoms during collision. [2 m]

The lasing transition for Ne is from the E3 to E2 state which is the population inversion
region, giving rise to red photons/ laser. [1 m]

In practice, mirrors are positioned at the two ends of the sealed glass tube to reflect the
emitted radiation back and forth along the tube so as to stimulate more emission from
the excited Ne atoms. [1 m]

FOR NJC INTERNAL USE ONLY 4


NATIONAL JUNIOR COLLEGE PHYSICS DEPARTMENT

7 (a)(i) reading from graph, weight before take-off = 600 N


Mass = 600/9.81 = 61.2 kg

(ii) reading from graph = 3280 ± 10 (N)

(iii) reading from graph g = 9.2 N kg-1


pull on astronaut = 9.2 x 61.2 = 563 N

(b)(i) Any 2 of the following:

As the rocket fuel is burnt up so during the ascent, the mass of rocket gets less.
Thus for the same force on less mass gives greater acceleration.

The resultant force in the following cases will be greater and thus its acceleration
will be greater also. The cases are:

Less friction as atmosphere is less dense.


The gravitational field strength is lesser as distance from Earth increases.

(ii) The area under the force time graph in Fig 7.1 will not give the momentum of
the astronaut as the given force in the figure is the force on the astronaut due to the
cabin. To give the momentum of the astronaut, the force should be the net force or
resultant force acting on the astronaut.

(c) centripetal acceleration = value of g


distance from Earth = (6.71 – 6.37) x 106 = 340 km
which gives g = 8.83 ms-2 from the Figure 7.2.

(d) (i) centripetal acceleration = 8.83 ms-2= v2/r


v2 = 8.83 x 6.71 x 106
v = 7700 ms-1

(ii) k.e. = ½mv2 = ½ x 61.2 x (7700)2


= 1.81 x 109 J

(e)

FOR NJC INTERNAL USE ONLY 5


NATIONAL JUNIOR COLLEGE PHYSICS DEPARTMENT

(9.2, 3.5)

(2.4, 1.5)

(i) Draw the best fit graph.

(ii)
Gradient = (3.5 – 1.5)/(9.2 – 2.4) =2.0/6.8 = 0.294
Using point (9.2, 3.5) and gradient = 0.294, y = mx+c
3.5 = 0.294 (9.2) + c
C = y-intecept = 0.795
1 p
2
= +q
T M
Hence p = 0.294
q = 0.795
(iii) From the experimental data, a straight line is obtained with gradient =0.294
and a y-intercept of 0.795 as seen in the best fit line drawn. Hence the
experimental data follows the given relationship which confirms its validity.
(iv)
2 pT 3
ΔT = x (percentage mass loss).
M
1 0.294
When M = 0.5 kg, 2
= + 0.795
T 0.5
T = 0.850 s

FOR NJC INTERNAL USE ONLY 6


NATIONAL JUNIOR COLLEGE PHYSICS DEPARTMENT

2(0.294)(0.850) 3
ΔT = x 0.1 = 0.0723 s = 0.08 s
0.5

The period T obtained from the experiment is accurate to the fourth decimal place.
Since the uncertainty found is up to 2 decimal places, therefore the experiment
conducted is able to detect the changes it is able to detect up to the fourth decimal
place.

Description Points

1. Diagram
A
- Correct connection of voltmeter/CRO (or any appropriate instrument)
to measure the p.d. across the high voltage supply in order to
measure the breakdown voltage.
- The use of thermocouple/ temperature sensor attached to data 2m (All 3 must be
logger (or any appropriate instrument) to measure the temperature of present in order to get
the air in the chamber. 2m). Award 1m for any
- Use of pressure gauge/ pressure sensor (or any appropriate 2.
instrument) attached to data logger to measure pressure
B
- Correct use of the 2 electrodes 1m
1m
- Appropriate heating element (preferably a circuit). Do not accept
Bunsen burner.

2. Variables
Independent Variable: Temperature of air between the electrodes. 1m(must be all correct)
Dependent Variable: voltage when spark is first seen, measured by the
voltmeter.
Control Variables:
- Always ensure that distance between the electrode is kept constant. 1m (At least these 2
-The use of vacuum/air pump to regulate the constant pressure in the must be present)
chamber.

3. Measurement
a) Keep the pressure of air constant by the use of pressure gauge and
air/vacuum pump.
b) Use the heater to increase the temperature of air in steps of 5oC. This 1m
can be done by passing a constant current through the circuit. Temperature
(T) will be measured by the instrument listed in Diagram. (Note that
students must mention the use of calibration curve if thermocouple is used.) 1m
c) At a certain temperature, use the high voltage supply to slowly increase

FOR NJC INTERNAL USE ONLY 7


NATIONAL JUNIOR COLLEGE PHYSICS DEPARTMENT

the voltage across the electrodes until a spark is first seen. Record this 1m
voltage (V)
d) Then reset the voltage and repeat a-c to get 10 sets of readings of V and
T.
4.Analysis
A table which contains values of V and T is recorded. A graph of lg(V) vs 1m
lg(T) is then plotted and the gradient and y-intercepts of the graph are
determined. Then the relationship between the v and p could be determined
in the form V = kTn, where the constants n is the gradient and k = 10y-intercept

5. Precaution
The risks involved in this experiment can be substantial.
- Keep hands dry and wear gloves to prevent electrocuted when handling
high voltage supply.
- Ensure that the pressure do not exceed too much by constantly monitoring
the pressure sensor and regulating the air/ vacuum pump. 1m (only if they give 2
- Any reasonable precaution. precautions)

6. Additional Details
- Thickness between the electrodes must be kept constant at all times 1m (Any 1 reasonable
- Air chamber must be properly sealed to have reliable pressure readings details and they must
- Time must be allowed for air in chamber to reach an equilibrium give at least 3 details)
temperature.
- Any reasonable detail.

FOR NJC INTERNAL USE ONLY 8


NATIONAL JUNIOR COLLEGE PHYSICS DEPARTMENT

2010 H2 Prelim Physic Paper 3 Suggested Solution

Qn. Suggested Solutions

1 1a. Angular velocity is defined as the rate of change of angular displacement.

1bi.

An object moving with constant speed will continue in its uniform motion in a straight
line unless acted on by a net/resultant force according to Newton’s First Law. To
change its direction, there must be a net force acting on it. Since the baggage is
rotating at constant speed, this net force is acting perpendicular to its motion changing
it’s direction but not it’s speed. As the change in velocity is directed to the centre of
the rotation, by Newtion’s 2nd law the net force applied must also acts in the direction
of the change in velocity.

1bii. N, normal f, friction


contact force

w, weight of baggage

1biii. N, normal
contact force Taking upwards as positive:

θ Ncosθ +fsinθ - w = 0

θ N = (w-fsinθ)/cosθ
θ
= (10x9.81 – 59.4sin 36o)/( cos 36o)

θ = 78.1 N
w, weight of baggage

1biv. Consider forces horizontally,

f cos 36o - Nsin 36o = mrω2

59.4 x cos 36o – 78.1 sin 36o = 10 x 11.0 x (2π/T)2

FOR NJC INTERNAL USE ONLY 1


NATIONAL JUNIOR COLLEGE PHYSICS DEPARTMENT

2π/T = 0.140

T = 44.9 s or 45 s

2 ai.

Show calculation to determine value of PV at C, B and one other point on the curve
BC

Conclude that since PV is constant for fixed mass of gas, the process BC is
isothermal.

Mass of ideal gas is fixed during the process.

ii.

Internal energy of an ideal gas is only made up of the sum of microscopic kinetic
energy due to random motion of its molecules

b.

Using Pc/Tc = PA/TA , TA = 192.5 K = 192.5 – 273.15 = - 80.7 oC

c.

Method 1

3 3
ΔU AB = P(VB − V A ) = 1.7 x10 5 (9.8 − 5) x10 −4 = 122.4 = 122
2 2

Method 2

3 3
ΔU AB = nRΔTAB = n(8.31)(385 − 192.5)
2 2

pcVc
But n = = 0.053136
8.31(Tc )

Hence ΔU AB = 127.5 = 128

FOR NJC INTERNAL USE ONLY 2


NATIONAL JUNIOR COLLEGE PHYSICS DEPARTMENT

W is negative when the gas expands during the change AB.

At the same time, its internal energy has increased since its PV value increased
during the expansion.

Using the First Law of Thermodynamics, ΔU = q + W, this implies that more thermal
energy must be supplied to the gas than its work done (i.e. q is positive and > W).

(d)

Pressure of gas is due to collision of molecules with the container walls.

During contraction at constant temperature, the molecules move at and collide

with same speed. But due to contraction in volume, the frequency of

collision increases. Or more collisions per unit time increases.

This implies rate of change of momentum due to the collision of molecules

and hence pressure increases.

Note: A larger change in momentum from each collision (ie molecules move faster) or
higher frequency of collision (i.e. higher temperature or smaller volume) will

lead to larger pressure.

FOR NJC INTERNAL USE ONLY 3


NATIONAL JUNIOR COLLEGE PHYSICS DEPARTMENT

3 (a) A high voltage is applied across the discharge tube which contained the hot gases
(for example, sodium vapour lamp or mercury vapour lamp). When the light emitted
by hot gases in discharge tubes is passed through a diffraction grating, distinctive
bright coloured lines against a dark background is observed. This is known as an
emission line spectrum.

discharge lamp
grating
High
voltage

spectrometer

(b)(i)(1) When V1 is less than P, the electrons reaching G do not have enough kinetic
energy to reach A. Since no electrons reach A, Ia is zero.

Note: When electron moves from G to A, the electric force (repulsive force) will
decelerate the speed of the electrons.

(i)(2) Electrons reaching G have enough kinetic energy to reach A. With increasing
V1, more electrons are able to reach A per unit time as they have higher kinetic
energy, thus Ia increases.

(ii) Kinetic energy from the accelerated electrons are passed to the mercury
atoms (though inelastic collision) where electrons in the ground state are
excited to a higher state. The sudden onset suggest that the mercury
electrons cannot except energy until it reaches the threshold for elevating
them to an excited state.

(iii)(1) 4.9 x 1.6x10-19 = hc/λ

λ = 2.54 x 10-7 m or 254 nm

(iii)(2) It is found in the ultra violet light

4 a. One ohm is defined as the resistance of a conductor when a current of one amp
ere passes through it when the potential difference across it is one volt.

bi.

The wire of uniform cross-section and carries a constant current supplied by battery
E. To measure the unknown Ex, the contact c is moved until the galvanometer G
reads zero. The potential difference between ac is proportional to the length ac and is
equal to Ex. To calibrate a potentiometer, one switches from Ex to a standard cell Es

FOR NJC INTERNAL USE ONLY 4


NATIONAL JUNIOR COLLEGE PHYSICS DEPARTMENT

and moves the slide to a point d on wire ab to obtain another zero galvanometer
reading. Then Ex can be easily computed from the known Es of the standard cell and
the measured lengths ac and ad from Ex / Es = ac / ad.

ii. The potentiometer can measure the terminal potential difference with high accuracy
without drawing any current from the unknown source.

c.i.

600
Vab = × 5V = 3.75V
600 + 200

Vac
Therefore lac = ×1.0m = 0.533m
Vab

ii. Left

iii.

600
Vab ' = × 5V = 3.26V
40
600 + × 400
50

Vac
Therefore lac ' = ×1.0m = 0.613m
Vab '

5 (a) (i) Refer to notes. The higher the B.E. per nucleon of a nuclide, the more stable
the nuclide is.

(ii) 7.66831x1.6x10-13 = (88x1.00783 u + 138 x 1.00867 u – M)c2

M = 226.025 u (shown)

(iii) By Conservation of linear momentum, 0 = pα + pRn


By Conseravtion of energy, Q = KEα + KERn (KE = p2/2m)


Hence, Q = Kα ( 1 + )
M Rn

(iv) Q = 6.336x10-13

Kα =6.22x10-13

(v) Alpha has low penetrating power and short range and thus able to be used on
localized area. It also has the highest ionization ability and thus effective in killing
harmful cells.

(b)(i) Refer to notes

FOR NJC INTERNAL USE ONLY 5


NATIONAL JUNIOR COLLEGE PHYSICS DEPARTMENT

(ii) A = λN,

Ratio = 0.33(±5%)

(iii) Activity becomes half for each half life.

(iv) Either show 3 half-lives are constant or explain that when take the
natural log ln, the graph is a straight line.

6 6(a) Two waves are said to be coherent when there is a constant phase difference
between them.

6(b)(i) Wavelength = 330/660 = 0.5 m

Path of X to O = 15 m

Path Difference between the two sources = 15-12 = 3 m

Path Difference = 6λ which shows that the 2 waves are in phase at O.

Since the sources have zero phase difference and path difference shows that the
waves are in phase at O, hence constructive interference.

6(b)(ii) At Y’, the new path difference = 0m, thus there is constructive interference.

As source moved from initial position to Y’, path difference goes from 6λ to 0. Thus
observer at O experiences 6 soft sound (or 6 destructive interference) when path
difference are 5.5λ, 4.5λ, 3.5λ, 2.5λ, 1.5λ, 0.5λ and 7 loud sound (or 7 constructive
interference) when path difference 6λ, 5λ, 4λ, 3λ, 2λ, 1λ, 0)

6(b)(iii) No. Amplitude of the waves depends on the intensity of the wave which
depends on the distance travel from the source to the observer. Since the 2 waves
travel different distance from the source to the observer, the amplitudes of the two
waves will not be exactly the same, hence there will not be complete cancellation

Note: Do not accept background noise or the 2 sources did not have the same
amplitude.

6(c)Path of reflected wave = 10 m

Path difference = 20-12 = 8 m = 16λ which shows that the 2 waves are in phase at O.

Though the 2 waves are in phase at O due to π radians phase shift at reflection, there
will be destructive interference hence a soft sound.

(d) Diffraction is the spreading of waves when the waves pass through a gap or an
obstacle.

(e) In order to determine if there is overlap of the first and second spectra, we have to

FOR NJC INTERNAL USE ONLY 6


NATIONAL JUNIOR COLLEGE PHYSICS DEPARTMENT

check the maximum angular deviation of the first spectra and the minimum angular
deviation of the second spectra.

In the first order spectra, red light with the longest wavelength will constitute the
largest angular deviation,

Using dsinθn = n λ , λ R = 0.7 μm , d= 2.5 μm

For red light, nR =1, θR,1 = 16.3˚

Therefore, the maximum angular deviation for the first spectra is 16.3˚.

In the second order spectra, blue light with the shortest wavelength will constitute
the smallest angular deviation,

Using dsinθn = n, λ B = 0.4 μm , d= 2.5 μm

For blue light, nB =2, θB,2 = 18.7˚

From the above calculation, angular deviation for 2rd order blue light > angular
spacing for 1st order red light. There is no overlapping of 2nd order blue light and 1st
order red light.

Therefore, there is no overlapping between the first order and second order spectra.

(f)(i) It is to produce a coherent light when it pass through the collimator.

(ii) dsinθ = n λ

λ' sin ϑ ' ϑ − ϑ1


= where ϑ = 2
λ sin ϑ 2

90.033°
For Na: ϑ = = 45.017°
2

⎛ 71.367° ⎞
'

sin ⎜ ⎟
λ1 ⎝ 2 ⎠
For pair 1: = => λ1 = 486nm => H2
589.3 sin 45.017

⎛ 93.667° ⎞
'

sin ⎜ ⎟
λ2 ⎝ 2 ⎠
For pair 2 = => λ 2 = 608nm => CO2
589.3 sin 45.017

FOR NJC INTERNAL USE ONLY 7


NATIONAL JUNIOR COLLEGE PHYSICS DEPARTMENT

7 7ai.

Power is defined as the the rate of work done.

ii. Power , P = Work done (W)/ time taken (t)

Work is done when the point of application of force (F) on the object moves in the
direction of the displacement (S) of the object. Hence W = F.s

Power, P = F.s/ t where s/t is the rate of change of displacement which is velocity (v)
of the object.

Therefore P = F.v (shown)

7bi.

From solar energy, the energy is converted to electrical energy by the solar panel
which is then stored in the battery as chemical energy. To drive the plane’s engines,
the chemical energy is converted to electrical energy which is then coverted to
mechanical energy or kinetic energy of the propellers. Part of the kinetic enerygy is
used to do work against air resistance while the rest is used to change its velocity.

7bii.1.

Efficiency, η= (Power Output/ Power input) x 100

= (Mechanical Power/solar power (Ps) ) x100

Intensity, I = Power received by the solar panel (Ps)/Area of the solar panel

250 = Ps/ 200

Ps = 50,000 = 50 kW

Hence, η = ( 6.0x103/50 x103) x100 = 12.0 %

7bii.3.

Power output by the engines, Po = F.v where F is the force provided by the engines
and v is the velocity of the plane. F, forward force
provided by the engine
6.0 x103 = F. (70x103/3600) Air resistance,f

F = 308.6 N = 309 N

To maintain constant flight speed, the force (F) provided by the engines must be
equal to force due to the air resistance (f).

Hence the magnitude of the air resistance = 309 N

FOR NJC INTERNAL USE ONLY 8


NATIONAL JUNIOR COLLEGE PHYSICS DEPARTMENT

7bii.3.

No. of hours of darkness = 24 -14 = 10 hours.

Mechanical energy required for 10 hours, E = Power by engines x time

= 6x103 x 10 x 3600

= 216 MJ

With efficiency of 12%, hence the solar energy required to be stored = 216x106/0.12

= 1.8 GJ

OR Solar energy required = Power received by the solar panel x time (10hrs)

= 50,000 x (3600 x 10 ) = 1.8 GJ

7c.

Depletion Region of a p-n Junction

(1) A p-n junction is obtained by joining a p-type semiconductor and an n-type


semiconductor as shown below:

(2) Origin of the depletion region.

In the narrow region near a p-n junction, electrons tend to diffuse from the n-type side
through the junction into the p-type side while holes tend to diffuse in the opposite
direction.

The mobile n-side donor electrons nearest the junction diffuse to the p-side and fill

FOR NJC INTERNAL USE ONLY 9


NATIONAL JUNIOR COLLEGE PHYSICS DEPARTMENT

holes located there, leaving behind immobile positive ions (Gp V atoms) that are fixed
in the lattice. While this happens, we can model the holes that are being filled as
diffusing to the n-side, leaving behind a region of fixed negative ions (Gp III atoms).

The electrons and holes in the region tend to recombine and the region becomes
virtually depleted of mobile charge carriers. This region is commonly called the
depletion region. Other names include metallurgical region, transition region or
space charge region.

These immobile charge ions thereby create an electric field across the depletion
region. At equilibrium, the electric field set up in the depletion region will be strong
enough to prevent more electrons from diffusing across from the n-type to the p-type
semiconductor. The potential difference due to this electric field within the depletion
region can be thought of as a “potential energy barrier” that prevents the further
migration of electrons across the junction.

di.

1. Series connection enables the individual small emf of the solar cells to be added
up to a larger voltage that is useful for charging the plane’s battery.

2. Parallel connection enables the entire circuit to work even if though if 1 unit fails.

dii.

Cloud cover or flying in the night where there will be no adequate solar energy to be
captured and stored.

The plane must carry extra fully charged batteries or alternative fuel for the plane’s
propulsion.

FOR NJC INTERNAL USE ONLY 10


NANYANG JUNIOR COLLEGE
JC 2 PRELIMINARY EXAMINATION
Higher 2

PHYSICS 9646/01
Paper 1 Multiple Choice 28 September 2010
1 hour 15 minutes
Additional Materials: Multiple Choice Answer Sheet

READ THESE INSTRUCTIONS FIRST

Write in soft pencil.


Do not use staples, paper clips, highlighters, glue or correction fluid.
Write your name, class and tutor’s name on the Answer Sheet in the spaces provided unless this has been
done for you.

There are forty questions on this paper. Answer all questions. For each question there are four possible
answers A, B, C and D.
Choose the one you consider correct and record your choice in soft pencil on the separate Answer Sheet.

Read the instructions on the Answer Sheet very carefully.

Each correct answer will score one mark. A mark will not be deducted for a wrong answer.
Any rough working should be done in this booklet.

This document consists of 14 printed pages.

[Turn over
2

DATA AND FORMULAE

Data
8 –1
speed of light in free space c = 3.00 x 10 m s
–7 –1
permeability of free space μo = 4π x 10 H m
–12 –1
permittivity of free space εo = 8.85 x 10 Fm
–9 –1
(1/(36π)) x 10 F m
–19
elementary charge e = 1.60 x 10 C
–34
the Planck constant h = 6.63 x 10 Js
–27
unified atomic mass constant u = 1.66 x 10 kg
–31
rest mass of electron me = 9.11 x 10 kg
–27
rest mass of proton mp = 1.67 x 10 kg
–1 –1
molar gas constant R = 8.31 J K mol
23 –1
the Avogadro constant NA = 6.02 x 10 mol
–23 –1
the Boltzmann constant k = 1.38 x 10 JK
–11 2 –2
gravitational constant G = 6.67 x 10 N m kg
–2
acceleration of free fall g = 9.81 m s

Formulae

uniformly accelerated motion s = ut + ½ at2


v2 = u2 + 2as
work done on/by a gas W = pΔV
hydrostatic pressure p = ρgh
gravitational potential φ = –Gm/r
displacement of particle in s.h.m. x = xosinωt
velocity of particle in s.h.m. v = vocosωt
2
= ± ω ( xo − x 2 )
resistors in series R = R1 + R2 + ....
resistors in parallel 1/R = 1/R1 + 1/R2 + ....
electric potential V = Q/4πεor
alternating current/voltage x = xo sinωt
transmission coefficient T = exp(–2kd)

8π 2 m(U − E )
where k =
h2
radioactive decay x = xo exp(–λt)
0.693
decay constant λ =
t 1/ 2

NYJC 2010 9646/01/PRELIM/10


3

1 Express volt in SI base units.

A kg m2 s-3 A-1 B V C kg m s-2 A-1 D kg m2 s-2 C-1

2 Two students A and B carry out a series of experiments to determine the values of the
density of water (true value = 1000 kg m-3)

A B
ρ / kg m-3 ρ / kg m-3
1002 998
998 998
997 998
1001 997
999 997

What of the following statements below correctly compares the two sets of experimental
results?

A Results of Student A is more accurate and more precise than those of Student B.
B Results of Student A is less accurate but more precise than those of Student B.
C Results of Student A is more accurate but less precise than those of Student B.
D Results of Student A is less accurate and less precise than those of Student B.

3 The graph is a displacement-time (s-t) graph for a tennis ball during part of a game. Which
part of the graph shows the highest speed?
s

A
C
D

t
4 A stone of mass m is projected with velocity v from a point P as shown below.

45°

P Q
Neglecting the effects of air resistance, the magnitude of the change in momentum between
leaving P and arriving at Q is

A zero B mv C mv 2 D 2mv

[Turn over
NYJC 2010 9646/01/PRELIM/10
4
5 A man of mass m is standing inside a lift on top of a weighing machine. The variation with
time t of the reading on the weighing machine is shown below.
Reading

c d
mg
a b

Which of the following describes correctly the motion of the lift in a building based on the
reading shown?

a-b b-c c-d


Moving up with uniform Moving up with Moving up with uniform
A
increase in acceleration. constant acceleration decrease in acceleration
Moving up with uniform Moving uo with uniform
B Stationary
decrease speed. increase in speed.
Moving up with constant Moving up with Moving up with uniform
C
speed increasing speed decrease in speed.
Moving down with uniform Moving down with Moving down with uniform
D
decrease in speed. constant speed. increase in speed.

6 Blocks A and B each of mass 2.0 kg and 1.0 kg respectively are connected by a spring and
arranged as shown in Fig 5(a). After that, they are rearranged as shown in Fig 5(b). The
extension of the spring in (a) and (b) is in the ratio

A B B A
F F
2.0 kg 1.0 kg 1.0 kg 2.0 kg

Fig 5(a) Fig 5(b)

A 1:1 B 2 :1 C 1 :2 D 1 :3

7 The diagram shows two blocks of mass m and 2m connected by a light cord passing over a
light, free-running pulley. At what angle θ must the smooth slope be inclined such that the
two blocks remain stationary?

2m
m

A 19° B 30° C 45° D 60°

NYJC 2010 9646/01/PRELIM/10


5
8 The graph shows the variation with extension x of the load F on a certain spring.

25

20

15
F /N

10

0
0.00 0.02 0.04 0.06 0.08 0.10
x /m

A load of 10 N is placed on the spring. How much additional elastic potential energy will be
stored in the spring if it is then extended a further 0.040 m?

A 0.200 J B 0.450 J C 0.600 J D 0.800 J

9 A mass hanging on a spring oscillates vertically between X and Z. Its equilibrium position is
at Y. The spring has negligible mass. The spring is just taut when mass is at X.

X
Y
Z

Which of the following gives the correct description of energy and force?

Elastic energy of spring is Largest net downward


Kinetic energy of mass is
maximum when mass is force acts on mass when
maximum at
at it is at
A X Z Y
B Y X Z
C Z X Y
D Y Z X

NYJC 2010 9646/01/PRELIM/10 [Turn over


6
10 Free electrons are drifting along a conductor when an electrical potential difference is
applied across it. Which of the following statement is incorrect?

A The electrical potential energy of the electrons is decreasing.


B The mean kinetic energy of the electros is increasing.
Kinetic energy of the drifting electrons is converted to internal energy of the
C
conductor continuously.
D The path of the electrons is not in a straight line.

11 A train travels with uniform speed on a curved horizontal track. The centre of curvature of
the curve is at O. Which of the arrows represents a possible direction for the resultant force
exerted by the track on the train?

C B

O
D A

12 A straight length of tape winds on to a roll rotating about a fixed axis with constant angular
velocity, the radius of the roll increasing at a steady rate.

radius

Which one of the graphs below correctly shows how the speed v at which the tape moves
towards the roll varies with time?

A v B v

0 0
0 t 0 t

C v D v

0 0
0 t 0 t

NYJC 2010 9646/01/PRELIM/10


7
13 At a point on the surface of a uniform sphere of diameter d, the gravitational field due to the
sphere is Y. What would be the corresponding value on the surface of a uniform sphere of
the same density but of diameter 2d?
A 2Y B 4Y C 8Y D 16Y

14 Two satellites, A and B, orbiting around Earth have the same kinetic energy. Satellite A has
a larger mass than satellite B. Which of the following statements is false?

A Satellite A has a larger period.


B Satellite A has a larger orbital radius.
C Satellite A has a smaller total energy.
D Satellite A has a smaller angular velocity.

15 Which of the following graphs represents the variation with displacement x of the potential
energy Ep of a body undergoing simple harmonic motion?

A Ep B Ep

x x

C Ep D Ep

x x

16 The rise and fall of water in a harbour is simple harmonic. The depth varies between 1.0 m
at low tide and 3.0 m at high tide. The time between successive low tides is 6 hours.

3.0 m high tide

depth

1.0 m low tide

time
A boat, which requires a minimum depth of water of 1.5 m, approaches the harbour at low
tide. How long will the boat have to wait before entering?

A 0.5 hours B 1.0 hours C 2.0 hours D 2.5 hours

[Turn over
NYJC 2010 9646/01/PRELIM/10
8
17 For an ideal gas at 300 0C, the mean kinetic energy of the gas molecules is 1.6 x 10-23 J.
What are the values for the temperature of the gas and the mean kinetic energy of the gas
molecules when its molecules are, on average, travelling at twice the speed?

Temperature / 0C Kinetic energy / J


A 600 3.2 x 10-23
B 1100 3.2 x 10-23
C 2000 6.4 x 10-23
D 2300 6.4 x 10-23

18 An ideal monatomic gas has 1000 J of heat added to it and it does 500 J of work; its
temperature changes by T1. When twice the amount of heat is added to it and it does the
same amount of work, its temperature changes by T2. The ratio of T1 / T2 is

A 1/5
B 1/3
C 3/5
D 1

19 A sound wave of period T is traveling in a gas at a speed of 330 m s-1.



When the phase difference between two points is radians, the distance between the
4
two points is 1.05 mm. Calculate T.

A 1.96 x 105 s
B 3.14 x 105 s
C 5.03 x 105 s
D 5.09 x 10-6 s

20 A student blows at one end of a drinking straw of length 10.0 cm while the other end is
dipped inside his drink for 6.0 cm. What is the lowest frequency of sound produced?
[Speed of sound in air = 320 m s-1]

A 800 Hz B 1600 Hz C 2000 Hz D 4000 Hz

21 When coherent monochromatic light falls on a double slits, interference pattern is observed
on a screen some distance from the slits. The fringe separation can be increased by

A Decreasing the distance between the screen and the slits.


B Increasing the distance between the slits.
C Using monochromatic light of lower frequency.
D Immense the whole set up in water.

22 A diffraction grating has 500 lines per mm and is illuminated normally by monochromatic
light of wavelength 600 nm. The total number of bright lines seen on the screen is

A 5 B 7 C 8 D 9

NYJC 2010 9646/01/PRELIM/10


9
23 The diagram below shows two small charged spheres P and Q of small mass which are
hung by identical fine nylon threads from a fixed point X. It is found that, in equilibrium, the
angle a is greater than the angle b.
X

a
b

P Q

Which of the following statements must be correct?

A The mass of P is less than that of Q.


B The mass of P is greater than that of Q.
C The charge of P is numerically smaller than that of Q.
D The charge of P is numerically greater than that of Q.

24 A metal sphere of radius 0.1 m was insulated from its surroundings and given a large
positive charge. A small charge was brought from a distant point to a point 0.5 m from the
sphere’s centre. The work done against the electric field was W and the force on the small
charge in its final position was F. If the small charge had been moved to only 1 m from the
centre of the sphere, what would have been the values for the work done and the force?

work done force

A W/4 F/2

B W/2 F/4

C W/2 F/2

D W/2 F/ 2

25 A strain gauge consists of a length of wire with uniform cross-sectional area. Its resistance
is 2.000 kΩ. It is attached to a gas container. When the container expands, the strain gauge
changes its dimensions. Its length increases by 0.40% and diameter reduces by 1.0 %.
What is the new resistance of the strain gauge?

A 1.006 kΩ
B 1.968 kΩ
C 2.028 kΩ
D 2.567 kΩ

26 There are 1.3 x 1019 electrons passing through a point in a series circuit in 1.0 hour. If the
potential difference across a resistor connected in series to the source is 10 kV, what is the
power dissipated in the resistor?
A 3.6 W B 5.1 W C 5.8 W D 21 kW

[Turn over
NYJC 2010 9646/01/PRELIM/10
10
27 A cell of e.m.f. 5.0 V and negligible internal resistance is connected to four similar resistors
and a variable resistor T, as shown.

V
T

5.0 V

Resistance of each resistor is 1.0 kΩ and resistance of T is 5.0 kΩ.

What is the reading of the ideal voltmeter?

A 0V B 2.0 V C 3.0 V D 5.0 V

28 In the circuit as shown, cell A has an e.m.f. of 2.0 V and negligible internal resistance. Wire
XY is 1.0 m long with a resistance of 4.0 Ω.

Cell B has an e.m.f. of 1.5 V and internal resistance 1.0 Ω. Calculate the length XP required
to produce null deflection in the galvanometer.

A 0.66 m
B 0.75 m
C 0.90 m
D 0.94 m

29 A bubble chamber photograph shows a proton moved in a circular arc with radius 10 cm
and perpendicular to a magnetic field of flux density 0.30 T. The kinetic energy of the proton
is

A 4 keV
B 40 keV
C 400 keV
D 4 MeV

NYJC 2010 9646/01/PRELIM/10


11
30 The diagram shows a compass near a rectangular coil of wire.

A direct current flows in the coil creating a magnetic field. The field has caused the
compass to be deflected away from the north and towards the east. The angle of deflection
is about 45°.

The current in the coil is doubled. Which one A to D is the approximate value of the new
angle of deflection?

A 20°
B 60°
C 90°
D 135°

31 The iron cores of transformers are usually ‘laminated’, that is, the iron core is constructed of
thin strips.

Which one of A to D below is the principal purpose of this lamination?

A To reduce eddy currents in the core.


B To increase the magnetic field strength in the core.
C To reduce the electrical resistance of the core.
D To improve the flux linkage of the two coils through the core.

[Turn over
NYJC 2010 9646/01/PRELIM/10
12
32 A coil of wire of one turn only with a negligible resistance is connected to a resistor. A
magnetic field B that changes with time t as shown in the graph below is applied
perpendicularly to the coil.
B

0 t
t1 t2 t3

Which of the following graphs best shows the variation of the heating power P in the
resistor with time t?

A B P
P

0
t1 t2 t3 t

0 t
t1 t2 t3

C P D P

0 0
t1 t2 t3 t t1 t2 t3 t

33 A rectifier is connected in series with load P and an alternating voltage supply as shown in
the figure below.

Vin / V Vin

Vo
t 2t 3t 4t t/s

What is the value of the r.m.s. voltage across load P?


A 0.18 Vo B 0.35 Vo C 0.50 Vo D 0.71 Vo

NYJC 2010 9646/01/PRELIM/10


13
34 An alternating current I / A varies with time t / s according to the equation

I = 5.0 sin (100π t).

Calculate the mean power, in watts, developed by the current in a resistive load of
resistance 10 Ω.

A 50 B 62.5 C 125 D 250

35 Which of the following statements about the photon and wave models of light is correct?

A The probability of arrival of a photon at a place is proportional to the intensity of


the waves at that place.
B The probability of arrival of a photon at a place is inversely proportional to the
wavelength at that place.
C The probability of transmission of a photon through a transparent material is
proportional to the speed of light waves in that material.
D The probability of emission of photons from a source is proportional to the
energy of the photons.

36 The diagram shows part of the energy level picture of a particular element. The energy
change for Eo to E1 is the same as that for E1 to E2.

E2

E1
increasing energy

Eo

If the transition E2 to E1 corresponds to a red line in the element’s spectrum, then the
transition Eo to E1 corresponds to

A absorption of red light.


B emission of red light.
C absorption of infra-red radiation.
D emission of infra-red radiation.

37 Which of the following statements is not related to laser production?

A Light amplification
B Stimulated emission
C Population inversion
D Electron deceleration

[Turn over
NYJC 2010 9646/01/PRELIM/10
14
38 A p-n junction is formed when a p-type semiconductor is in contact with a n-type
semiconductor. The most important property of a p-n junction is that it rectifies an
alternating current. Which of the following statements is incorrect?

A When a p-n junction is reverse biased, the p-type semi-conductor becomes less
negative.
B When a p-n junction is reverse biased, the width of the depletion region
becomes larger as the externally applied p.d. adds to the junction potential.
C When a p-n junction is forward biased, if the applied p.d. is greater than the
junction potential, electrons will cross steadily from the n-type side to the p-type
side while the holes will cross steadily in the opposite direction.
D When the p-n junction is not biased, there is a small potential difference across
the junction.

39 The fission of a heavy nucleus gives, in general, two lighter product nuclei. One such
reaction takes place when Uranium-235 nucleus undergoes fission by slow-moving
neutron:

U + 01n → 139
235
92 54 Xe + Sr + 2 01n
95
38

Which of the following statements is true about the product nuclei produced?

A They have a total rest mass that is greater than that of the original nucleus.
B The sum of their mass defect is greater than that of the original nucleus.
C The sum of their kinetic energies is smaller than that of the original nucleus.
D The sum of their binding energies is less than that of the original nucleus.

40 Radioactive 14C dating was used to find the age of a wooden archeological specimen.
Measurements were taken in three situations for which the following count rates were
obtained:

specimen count rate


1 g sample of living wood 80 counts per minute
1 g sample of archeological specimen 35 counts per minute
no sample 25 counts per minute

If the half life of 14C is known to be 5700 years, what was the approximate age of the
archeological specimen?

A 3700 years B 6800 years C 14000 years D 17000 years

~ End of paper ~

NYJC 2010 9646/01/PRELIM/10


15
Answers
1 A 11 B 21 C 31 A
2 C 12 D 22 B 32 D
3 A 13 A 23 A 33 B
4 C 14 C 24 B 34 C
5 D 15 C 25 C 35 A
6 C 16 C 26 C 36 A
7 B 17 C 27 C 37 D
8 C 18 B 28 D 38 A
9 D 19 D 29 B 39 B
10 B 20 C 30 B 40 C

NYJC 2010 9646/01/PRELIM/10


NANYANG JUNIOR COLLEGE
JC 2 PRELIMINARY EXAMINATION
Higher 2

CANDIDATE
NAME

TUTOR’S
CLASS
NAME

PHYSICS 9646/02
Paper 2 Structured Questions 15 September 2010
1 hour 45 minutes
Candidates answer on the Question Paper.
No Additional Materials are required

READ THESE INSTRUCTIONS FIRST


Write your name and class on all the work you hand in.
Write in dark blue or black pen on both sides of the paper.
You may use a soft pencil for any diagrams, graphs or rough working.
Do not use staples, paper clips, highlighters, glue or correction fluid.

Section A
Answer all questions.
It is recommended that you spend about 1 hour 15 minutes on this section.

Section B
For Examiner’s Use
Answer Question 8.
It is recommended that you spend about 30 minutes on this section.
1
At the end of the examination, fasten all your work securely together. 2
The number of marks is given in brackets [ ] at the end of each question or part
question. 3

Total

This document consists of 17 printed pages.

[Turn over
2
Data

speed of light in free space, c = 3.00 x 108 m s-1


permeability of free space, μo = 4π x 10-7 H m-1
permittivity of free space, εo = 8.85 x 10-12 Fm-1
(1 / (36 π)) x 10-9 Fm-1
elementary charge, e = 1.60 x 10-19 C
the Planck constant, h = 6.63 x 10-34 J s
unified atomic mass constant, u = 1.66 x 10-27 kg
rest mass of electron, me = 9.11 x 10-31 kg
rest mass of proton, mp = 1.67 x 10-27 kg
molar gas constant, R = 8.31 J K-1 mol-1
the Avogadro constant, NA = 6.02 x 1023 mol-1
the Boltzmann constant, k = 1.38 x 10-23 J K-1
gravitational constant, G = 6.67 x 10-11 N m2 kg-2
acceleration of free fall, g = 9.81 m s-2

Formulae

uniformly accelerated motion, s = ut + ½at2


v2 = u2 + 2as
work done on/by a gas, W = pΔV
hydrostatic pressure, p = ρgh
gravitational potential, φ = −Gm / r
displacement of particle in s.h.m. x = xo sin ωt
velocity of particle in s.h.m. v = vo cos ωt
= ±ω (x o
2
− x2 )
resistors in series, R = R1 + R2 + …
resistors in parallel, 1/R = 1/R1 + 1/R2 + …
electric potential, V = Q / 4πεor
alternating current/voltage, x = xo sin ωt
transmission coefficient, T = exp(-2kd)

where k = 8π 2 m (U − E )
h2
radioactive decay, x = xo exp (-λt)
decay constant 0.693
λ =
t1
2

NYJC 2010 9646/02/PRELIM/10


3 For
Examiner’s
Use
Section A

Answer all questions.


It is recommended that you spend about 1 hour 15 minutes on this section.

1 (a) State, in words, the 2 conditions that need to be satisfied in order to achieve static
equilibrium.

Condition 1:

[1]

Condition 2:

[1]

(b) A uniform trapdoor of mass 12 kg and length 1.00 m is smoothly hinged to the wall as
shown in Fig. 1.1 (not drawn to scale). It is supported in equilibrium by a stay wire
connecting the wall to a point on the trapdoor at a distance of 0.25 m from its free end.
The stay wire makes an angle of 60° with the wall and the trapdoor makes an angle of
30° with the horizontal.

60°

0.25 m

30°

Fig. 1.1

Show that the tension in the stay wire is 78 N.

[2]

NYJC 2010 9646/02/PRELIM/10 [Turn over


For
4 Examiner’s
Use

(c) A 20.0 kg sphere of uniform density rests between two smooth planes as shown in
Fig. 1.2.

Plane A
20 kg
Plane B
70° 30°

Fig. 1.2
Determine the magnitude of the force acting on the sphere exerted by each plane.

force due to plane A = N

force due to plane B = N [3]

2 The question is about reverse bungee jumping.

Fig. 2.1 shows the set up of a reverse bungee jumping. A capsule is connected by two
identical elastic cords each attached to a tower 30.0 m tall. The mass of the capsule when
fully loaded with three passengers has a total mass of about 300 kg. When released, the
capsule will shoot up at high speed.

10.0 m

Elastic
cords

30.0 m

Ground level
capsule
Fig. 2.1
NYJC 2010 9646/02/PRELIM/10
For
Examiner’s
5 Use

(a) The original length of each of the elastic cords is 25.0 m with an elastic constant of
19 000 N m-1 and the capsule has an effective diameter of 2.0 m. Prove that the total
elastic potential energy at the ground level = 510 kJ when the cord length is 30.2 m.

[1]

(b) Fill in the blanks in the table below to determine the various amounts of energy when
the capsule starts from the ground level and shoots up to its highest point.

Gravitational
Total elastic Kinetic energy of
potential energy of
potential energy /kJ capsule /kJ
capsule /kJ

Ground level 510 0 0

30 m above the
ground

Highest point 174


[2]

(c) Use the value in (a) to determine the speed reached by the capsule when the cords
first become loose.

speed = m s-1 [2]

(d) State and explain the position where the apparent weight of the passenger will be the
greatest.

[2]

NYJC 2010 9646/02/PRELIM/10 [Turn over


For
6 Examiner’s
Use

3 A flat horizontal plate is made to oscillate with simple harmonic motion in a vertical direction
as shown in Fig. 3.1. The plate starts its oscillation at the equilibrium position and moves
downwards initially.

plate

oscillator

Fig. 3.1

A graph of velocity against displacement for this oscillation is shown in Fig. 3.2. Point S
marks the start of the oscillation.

velocity / m s-1
0.8

0.6

0.4

0.2
displacement
/m
0
- 0.04 - 0.03 - 0.02 - 0.01 0.01 0.02 0.03 0.04

- 0.2

- 0.4

- 0.6
S
- 0.8
Fig. 3.2
(a) Deduce, from Fig. 3.2,

(i) the amplitude of the oscillation,

amplitude = m [1]

(ii) the angular frequency ω of the oscillation.

NYJC 2010 9646/02/PRELIM/10


For
Examiner’s
7 Use

ω= rad s-1 [2]


(b) A mass of 0.100 kg is placed on the plate before oscillation is started.

(i) Determine the displacement of the plate when the mass just loses contact with
the plate.

displacement = m [3]

(ii) Mark on Fig. 3.2 the point C when the mass just loses contact. [1]

4 (a) State two conditions that must be satisfied in order to obtain observable interference
patterns.

[2]

The apparatus shown in Fig. 4.1 below (not to scale) is used to demonstrate two-source
interference.

Double slit Screen

Light,
D
wavelength λ

Fig. 4.1

NYJC 2010 9646/02/PRELIM/10 [Turn over


For
8 Examiner’s
Use

(b) The separation of the two slits in the double slit arrangement is a and the interference
fringes are viewed on a screen at a distance D from the double slits. When light of
wavelength λ is incident on the double slit, the separation of the bright fringes on the
screen is x.

(i) Write the equation that links the quantities described in the above paragraph, and
state the assumption made in the use of that equation.

[2]

(ii) The slits are separated by a distance of a, with the screen at a distance of 1.00 m
from the plane of the slits. The slits are illuminated by monochromatic light of
wavelength 589.3 nm traveling perpendicular to the plane of the slits. It was
observed that the distance between the two 4th order bright fringes are 20 mm.
Calculate the separation of the slits, a.

a= m [2]

(c) (i) Explain why the central fringe is always a bright one.

[1]

(ii) Explain why an experiment using two separate sources of light will not show
interference.

[1]

NYJC 2010 9646/02/PRELIM/10


For
Examiner’s
9 Use

5 (a) Define electric field strength.

[1]

(b) Fig. 5.1 shows a stream of electrons entering a region between two parallel plates
which have a potential difference.
- 200 V

Stream of electrons

+200 V

Fig 5.1

(i) Draw on Fig 5.1, the electric field lines between the plates and the expected path
of the stream of electrons between and after the plates. [2]

(ii) Calculate the distance between the plates given that the electric field strength
between the plates is 2.0 x 104 N C−1.

distance = m [1]

(iii) Calculate the acceleration on the electrons between the plates

acceleration = m s-2 [1]

(iv) Hence, given that the length of each plate is 0.040 m and initial horizontal speed
of the electrons is 1.0 x 108 m s-1, calculate the vertical deflection of the electron
at the end of the plates.

deflection = m [2]

NYJC 2010 9646/02/PRELIM/10 [Turn over


For
10 Examiner’s
Use

6 A stationary Polonium-212 nuclide may undergo alpha decay spontaneously to produce the
stable lead-208 daughter nuclide as shown in the equation below:

212
84 Po ⎯⎯
→ 208
82 Pb + 24He

The rest masses of these nuclei are


212 208 4
84 Po : 211.9888 u; 82 Pb : 207.9766 u and 2 He : 4.0026 u.

(a) Calculate the total kinetic energy of the decay products.

total kinetic energy = J [2]

kinetic energy of He-4


(b) Determine the ratio of . Give your answer in 3 s.f.
kinetic energy of Pb-208

ratio = [2]

(c) Hence, determine the kinetic energy of the alpha particle in MeV.

kinetic energy = MeV [2]

(d) To escape from the nucleus, the alpha particle must overcome the Coulomb barrier of
26 MeV. Using your answer in (c), comment on how the alpha particle can penetrate
the barrier.

[2]

NYJC 2010 9646/02/PRELIM/10


For
Examiner’s
11 Use

7 In the first half of the last century, numerous experiments were conducted to investigate the
absorption and the scattering of X-ray by matter.

It was discovered that when a monochromatic beam of X-rays is incident on a light element
such as carbon, the scattered X-rays have wavelengths dependent on the angle of scattering.

Compton (1923) assumed that the scattering process could be treated as an elastic collision
between an X-ray photon and a ‘free’ electron, and that energy and momentum would be
conserved.

(a) Explain what is meant by a photon.

[1]

(b) The elastic collision between a photon and a stationary electron may be represented as
in Fig 7.1.

scattered photon
momentum ps
energy Es

θ
φ
incident photon
momentum pi
energy Ei
electron
mass m, speed v
Fig. 7.1

The incident photon has momentum pi and energy Ei. The photon is scattered through
an angle θ and, after scattering, has momentum ps and energy Es. The electron of mass
m, which was originally stationary, moves off with speed v at an angle φ to the original
direction of the incident photon.

(i) Write down equations, in terms of pi, ps, Ei, Es, m, v, θ and φ, that represent,
for this interactions,

1. Conservation of energy,

[1]

2. Conservation of momentum along the direction of the incident photon,

[1]

NYJC 2010 9646/02/PRELIM/10 [Turn over


For
12 Examiner’s
Use

(ii) Suggest, with a reason, whether the scattered photon will have a wavelength that
is greater or less than that of the incident photon.

[2]

(c) In an experiment to provide evidence to justify Compton’s theory, measurements were


made on the wavelength λi of the incident photon, the wavelength λs of the scattered
photon and the angle θ of scattering. Some data from this experiment are given in Fig.
7.2.

λi / 10-12 m λs / 10-12 m θ/o


191.92 193.27 57
153.30 154.65 57
965.04 966.84 75

Fig. 7.2

Use the data in Fig 7.2 to show that, when a photon is scattered, the change in
wavelength produced is independent of the wavelength of the incident photon.

[2]

(d) In this experiment, the uncertainty in the measurement of θ is ±5o.

Determine the value of cos θ, with its uncertainty, for the angle θ = 75o ± 5o.

cos θ = ± [3]

(e) Compton’s theory suggests that the change in wavelength Δλ is related to the angle θ
of scattering by the expression

Δλ= k (1 – cos θ)

where k is a constant.

Experimental data for the variation with cos θ of Δλ are shown in Fig 7.3.

NYJC 2010 9646/02/PRELIM/10


For
Examiner’s
13 Use

Fig. 7.3

(i) On Fig. 7.3, draw the best-fit line for the points. [1]

(ii) Use two different ways to determine the constant k from the graph of Fig. 7.3.
Find the average value of k.

average value of k = [4]

(f) For a carbon atom, the binding energy of an electron is of the order of a few
electronvolts. Compton's theory assumes that the electrons are not bound in the atoms
but are free. Suggest whether, for 30 keV photons, this assumption is justified.

[1]

NYJC 2010 9646/02/PRELIM/10 [Turn over


For
14 Examiner’s
Use

Section B

It is recommended that you spend about 30 minutes on this section.

8 Many musical instruments, such as organ pipes, flutes and clarinets, employ resonating air
columns to produce note of particular frequencies. The length of the resonating column may
be changed to produce a note of a different frequency. It is suggested that different volumes
of air in a container may resonate at frequencies which depend on the volume of the air.

Design an experiment to investigate how the resonant frequency of the fundamental mode of
vibration of air in a container depends on the volume of the air. You may assume that the
following apparatus is available with any other standard equipment which may be found in a
school or college:

Flute Microphone Ammeter


Measuring cylinder Bunsen burner Oscilloscope
Containers having different volumes Loudspeaker Bucket of water
Signal generator Voltmeter Thermometer

Your answer should contain a diagram showing how the chosen equipment would be
arranged, together with details of

(a) the procedure to be followed,

(b) the method by which the volume of the air and the resonant frequency may be
measured,

(c) the control of variables,

(d) any precautions you would take which may improve the accuracy of your experiment.

[12]

NYJC 2010 9646/02/PRELIM/10


For
Examiner’s
15 Use

Diagram

NYJC 2010 9646/02/PRELIM/10 [Turn over


For
16 Examiner’s
Use

NYJC 2010 9646/02/PRELIM/10


For
Examiner’s
17 Use

NYJC 2010 9646/02/PRELIM/10 [Turn over


NANYANG JUNIOR COLLEGE
JC 2 PRELIMINARY EXAMINATION
Higher 2

CANDIDATE
NAME

TUTOR’S
CLASS
NAME

PHYSICS 9646/03
Paper 3 Longer Structured questions 23 September 2010
2 hours
Candidates answer on the Question Paper.
No Additional Materials are required

READ THESE INSTRUCTIONS FIRST


Write your name and class on all the work you hand in.
Write in dark blue or black pen on both sides of the paper.
You may use a soft pencil for any diagrams, graphs or rough working.
Do not use staples, paper clips, highlighters, glue or correction fluid.

Section A
Answer all questions.
Section B For Examiner’s Use
Answer any two questions.
Section A
At the end of the examination, fasten all your work securely together.
The number of marks is given in brackets [ ] at the end of each question or part 1
question.
2
3
4
5
Section B
6
7
8
Total

This document consists of 17 printed pages.

[Turn over
2
Data

speed of light in free space, c = 3.00 x 108 m s-1


permeability of free space, μo = 4π x 10-7 H m-1
permittivity of free space, εo = 8.85 x 10-12 Fm-1
(1 / (36 π)) x 10-9 Fm-1
elementary charge, e = 1.60 x 10-19 C
the Planck constant, h = 6.63 x 10-34 J s
unified atomic mass constant, u = 1.66 x 10-27 kg
rest mass of electron, me = 9.11 x 10-31 kg
rest mass of proton, mp = 1.67 x 10-27 kg
molar gas constant, R = 8.31 J K-1 mol-1
the Avogadro constant, NA = 6.02 x 1023 mol-1
the Boltzmann constant, k = 1.38 x 10-23 J K-1
gravitational constant, G = 6.67 x 10-11 N m2 kg-2
acceleration of free fall, g = 9.81 m s-2

Formulae

uniformly accelerated motion, s = ut + ½at2


v2 = u2 + 2as
work done on/by a gas, W = pΔV
hydrostatic pressure, p = ρgh
gravitational potential, φ = −Gm / r
displacement of particle in s.h.m. x = xo sin ωt
velocity of particle in s.h.m. v = vo cos ωt
= ±ω (x o
2
− x2 )
resistors in series, R = R1 + R2 + …
resistors in parallel, 1/R = 1/R1 + 1/R2 + …
electric potential, V = Q / 4πεor
alternating current/voltage, x = xo sin ωt
transmission coefficient, T = exp(-2kd)

where k = 8π 2 m (U − E )
h2
radioactive decay, x = xo exp (-λt)
decay constant 0.693
λ =
t1
2

NYJC 2010 9646/03/PRELIM/10


3 For
Examiner’s
Use
Section A

Answer all the questions in this section.

1 (a) Define acceleration.

[1]

(b) A boy throws his slipper from a height of 1.0 m at 75o above the horizontal in an
attempt to hit a durian hanging on a tree which is 4.75 m above the ground. The slipper
reached its maximum height of 0.25 m directly below the durian. Neglect air resistance.

(i) Calculate the velocity of the slipper when the slipper leaves the boy’s hand.

velocity = m s-1 [2]

(ii) Calculate the horizontal distance travelled when his slipper hits the ground.

distance = m [2]

(iii) Sketch a labelled velocity - time graph for the vertical component of the slipper
from the time it leaves the boy to the time when it hits the floor.

[2]

(iv) If the velocity calculated in part (b)(i) is the maximum velocity he can provide
when throwing his slipper, suggest with a reason what he should do in order to
ensure his slipper hits the durian.

[1]

NYJC 2010 9646/03/PRELIM/10 [Turn over


For
4 Examiner’s
Use

2 (a) Define gravitational field strength and gravitational potential.

[2]

(b) A point S is located between the Earth and the Moon at a distance r from the Earth
along on the line joining the centres of the Earth and the Moon.
The distance from the centre of the Earth to the centre of the Moon is 3.8 x 108 m, the
mass of the Earth is 6.0 x 1024 kg and the mass of the Moon is 7.0 x 1022 kg.

Find the value of distance r, if S is the point where the resultant gravitational field
strength is zero.

r= m [2]

(c) Sketch a graph (without any values) showing the variation of gravitational field strength
with distance from the Earth, along the line joining the Earth and the Moon.

[2]

(d) Explain why a space craft would require more energy to move from the Earth to the
Moon compared to the return journey from the Moon back to the Earth.

[2]

NYJC 2010 9646/03/PRELIM/10


For
5 Examiner’s
Use

3 Fig 3.1 shows a circuit for measuring a small e.m.f. produced by a solar cell.
10 V
0.5 Ω

5.0 Ω

Solar cell
V

Fig 3.1

(a) The galvanometer shows null deflection when the variable resistor is set to 300 Ω.
Determine the value of the e.m.f., V of the solar cell.

V= V [2]

(b) Fig 3.2 shows the 5.0 Ω resistor being replaced with a 1.2 m uniform resistance wire
PQ of total resistance of 7.0 Ω. The variable resistor remains at 300 Ω.

10 V
I 0.5 Ω

P Q

Solar cell moveable


contact R
V

Fig 3.2
(i) Calculate the current I, when the galvanometer shows null deflection

I= A [2]

NYJC 2010 9646/03/PRELIM/10 [Turn over


For
6 Examiner’s
Use

(ii) Calculate the distance from P that contact R must be connected to wire PQ such
that the galvanometer shows null deflection.

Distance from P = m [3]

(iii) Explain why, this circuit is not suitable for measuring the e.m.f. of the solar cell
when the value of the e.m.f. of the solar cell is of the order of millivolts.

[1]

4 Fig. 4.1 shows the cross section of the mechanism of a coin-operated vending machine.

magnets
coin insert speed
sensors

¢
¢
B
coin A

holder inlet
C ¢
reject
path

Fig. 4.1

On entering the vending machine at A, the holder will stop the coin momentarily before
releasing it into the inlet.

At the inlet, the coin will pass through 2 pairs of magnets and 2 pairs of speed sensors where
eddy current will be generated in the coin. The coin travels down the inlet more slowly due to
the presence of the magnet as compared to an inlet without the pair of magnets. If the speed
sensors indicate that the coin has the correct speed, the gate at B will open and accept the
coin. Otherwise, the gate at C will open and the coin will be rejected.

NYJC 2010 9646/03/PRELIM/10


For
7 Examiner’s
Use

(a) State Faraday’s law of electromagnetic induction.

[1]

(b) Use Faraday’s Law of electromagnetic induction to explain the generation of eddy
current as the coin travels down the inlet.

[3]

(c) Using energy conservation, explain why the magnets cause the coin to travel down
more slowly as compared to an inlet without the pair of magnets.

[2]

(d) Describe how this mechanism can help to detect the correct denomination of coins.

[2]

NYJC 2010 9646/03/PRELIM/10 [Turn over


For
8 Examiner’s
Use

5 (a) Explain why a laser can cut metal plates but ‘normal’ light of the same power cannot do
so.

[2]

(b) Fig. 5.1 below shows the energy levels of a neon atom. In a helium-neon gas laser,
helium atoms in an excited state will excite the neon atoms from the state with energy
Eo to the state with energy E2 during collision. If the process of collision is to continue
throughout the container containing the helium and the neon gas, population inversion
will take place. Laser photons will be produced, whenever electrons make a transition
from energy level E2 to energy level E1.

E2 20.6 eV

E1 18.7 eV

E0 0 eV

Fig 5.1

(i) Describe what is meant by population inversion, with reference to the energy
level diagram in Fig 5.1.

[2]

(ii) Explain how stimulated emission results in the laser properties.

[2]

(iii) By referring to Fig 5.1, calculate the wavelength of the light produced by the laser.

wavelength = m [2]

NYJC 2010 9646/03/PRELIM/10


For
9 Examiner’s
Use

Section B

Answer two questions in this section.

6 (a) Define linear momentum.

[1]

Fig. 6.1 shows an airboat of mass 420 kg which is propelled forward by a propeller
generating a column of air backward.
propeller

airboat

Fig. 6.1

(b) (i) By using Newton’s laws, show that the forward thrust acting on the airboat is
given by F = πr2ρv2 where ρ is the density of air, r is the effective radius of
propeller and v is the speed of the air moving backward.

[3]

(ii) Calculate the initial acceleration of the airboat when r = 0.70 m, v = 20 m s-1 and
ρ = 1.2 kg m-3.

acceleration = m s-2 [2]

NYJC 2010 9646/03/PRELIM/10 [Turn over


For
10 Examiner’s
Use

(c) (i) Calculate the rate of transfer of kinetic energy to the air by the propeller.

Rate of transfer = W [2]

(ii) Given that the propeller is powered by a motor with a useful power 16 kW and
moving along a river with a constant speed of u. Use the answer for (c)(i) to
determine u.

u= m s-1 [3]

(d) (i) Fig. 6.2 show a side view of the boat partially submerged in water. Draw the other
forces acting on the airboat paying particular attention to the point of application
of these forces.

water level
Drag force
Total weight of
airboat [2]

Fig. 6.2

NYJC 2010 9646/03/PRELIM/10


For
11 Examiner’s
Use

(ii) Fig. 6.3 shows the outline of an airboat and a speed boat. Based on the diagram
below, the front part of the airboat is wider or less streamline than a speedboat.
By considering the stability of the airboat, explain why this is so.

airboat speed boat

Fig. 6.3

[2]

(iii) By considering the airboat and the air generated as a system, explain why the
total momentum of this system is not conserved when the airboat is moving at a
constant speed.

[2]

(e) (i) Suggest why airboats are more suitable for use in shallow rivers compared to
other types of boats.

[1]

(ii) Discuss two problems caused by using a much bigger propeller in order to
increase the forward thrust.

[2]

NYJC 2010 9646/03/PRELIM/10 [Turn over


For
12 Examiner’s
Use

7 (a) Explain what is meant by the internal energy of a system.

[2]

(b) State what is meant by saying a temperature is on an absolute scale.

[1]

(c) A cake of mass 0.90 kg is cooked in an oven at a temperature of 180°C. It is taken out
of the baking tin onto a rack to cool in a kitchen of 20°C.

(i) State the final temperature of the cake.

final temperature = K [1]

(ii) Calculate the energy released from the cake in cooling. Take the specific heat
capacity of the cake to be 990 J kg-1 K-1.

energy released = J [2]

(d) The oven of volume 0.10 m3 cools down from 180°C to 25°C.

Calculate the change in the mass Δm of air in the oven between the two temperatures.
The pressure in the oven remains at an atmospheric pressure of 1.0 x 105 Pa. Assume
that air behaves ideally. [Relative molecular mass of air = 0.030 kg mol-1]

Δm = kg [4]

NYJC 2010 9646/03/PRELIM/10


For
13 Examiner’s
Use

(e) Air is mainly made up of nitrogen and oxygen. The mass of 1 nitrogen molecule is 28 u
while the mass of 1 oxygen molecule is 32 u.

Find the ratio


average speed of oxygen molecule at 180°C
average speed of nitrogen molecule at 25°C

ratio = [3]

(f) The gas in the cylinder of a diesel engine can be considered to undergo a cycle of
changes of pressure, volume and temperature. One such cycle, for an ideal gas, is
shown in Fig. 7.1.

Fig. 7.1
The table below shows the increase in internal energy which takes place during each of
the changes A to B, B to C, C to D and D to A.

Section of cycle Heat supplied Work done Increase in internal


to gas / J on gas / J energy of gas / J
A to B 0 - 300 - 300

B to C - 450 0 - 450

C to D 0 650 650

D to A

Using Fig. 7.1, fill in the missing values in the table above. [3]

NYJC 2010 9646/03/PRELIM/10 [Turn over


For
14 Examiner’s
Use

(g) In a continuous flow method for determining the specific heat capacity a liquid, the
liquid flows through the tube at 0.15 kg min-1, while the heater provides power at 25 W.
The temperatures of the liquid at the inlet and outlet are 15 oC and 19 oC, respectively.

With the inlet and outlet temperatures unchanged, the flow rate is increased to
0.23 kg min-1 and the power of the heater is increased to 37 W.

(i) Explain why it is necessary for the inlet and outlet temperatures to remain
unchanged.

[1]

(ii) Determine the rate of heat loss of the liquid.

rate of heat loss = W [3]

NYJC 2010 9646/03/PRELIM/10


For
15 Examiner’s
Use

8 (a) ‘X-rays are used to investigate the atomic structure of solids.’ Deduce from this
statement the wavelength of the X-rays used.

[1]

(b) ‘Sometimes, for example, in the case of rubber, electrons with a de Broglie wavelength
of about 0.11 nm are used instead of X-rays.’
Showing the steps in your calculations, determine the momentum of such electrons,

momentum = N s [2]

(c) An X-ray tube operates with a potential difference of 100 kV between the anode and
cathode. The tube current is 20 mA. Only 1% of the energy of the cathode rays is
converted into X-rays.

Determine

(i) the rate of heat to be removed from the target in order to keep it at a steady
temperature,

rate = W [1]

(ii) the number of electrons which reach the target each second,

number = [1]

(iii) the maximum energy of an X-ray photon produced, explain your working.

maximum energy = J [2]

NYJC 2010 9646/03/PRELIM/10 [Turn over


For
16 Examiner’s
Use

(d) Fig 8.1 is a sketch of the X-ray spectrum produced by this tube for a particular metal
target. Fig 8.2 shows a sketch of the energy level of target material and how the Kα line
is formed. The tube voltage is 100 kV and the current is 20 mA.

Intensity N-shell

M-shell

L-shell

0
0 Photon
energy K-shell
Fig 8.1
Fig 8.2

(i) Label the maximum energy of the X-ray photon you have calculated in (c)(iii) with
M on Fig 8.1. [1]

(ii) Label the Kα, and Lβ spectrum lines on Fig 8.1. [2]

(iii) Sketch on Fig 8.1 a spectrum for X-ray from the tube if the tube voltage is
reduced to 50 kV, the current remaining at 20 mA. Label this spectrum A. [1]

(iv) Also on the same axes, sketch a spectrum for for X-ray from the tube if the tube
current is increased to 30 mA the tube voltage remaining at 100 kV. Label this
spectrum B. [1]

(d) Explain how the characteristic and continuous parts of the spectrum are formed.

(i) Formation of characteristic parts of X-ray spectrum:

[2]

NYJC 2010 9646/03/PRELIM/10


For
17 Examiner’s
Use

(ii) Formation of continuous parts of X-ray spectrum:

[2]

(f) The energy required to remove an electron from the various shells of the nickel atom is:

K shell 1.36 x 10-15 J


L shell 0.16 x 10-15 J
M shell 0.08 x 10-15 J

An X-ray tube with a nickel target emits the X-ray K radiation of nickel.

Determine

(i) the minimum potential difference across the tube,

potential difference = V [2]

(ii) the energy of the X-ray quantum of longest wavelength in the K-spectrum of
nickel.

energy = J [2]

NYJC 2010 9646/03/PRELIM/10 [Turn over


2010 NYJC Prelim H2 Paper 1 Solutions

1 A 11 B 21 C 31 A
2 C 12 D 22 B 32 D
3 A 13 A 23 A 33 B
4 C 14 C 24 B 34 C
5 D 15 C 25 C 35 A
6 C 16 C 26 C 36 A
7 B 17 C 27 C 37 D
8 C 18 B 28 D 38 A
9 D 19 D 29 B 39 B
10 B 20 C 30 B 40 C
1
Section A

Answer all questions.


It is recommended that you spend about 1 hour 15 minutes on this section.

1 (a) State, in words, the 2 conditions that need to be satisfied in order to achieve static
equilibrium.

Resultant
Condition 1: external force acting on the body is zero.
Resultant torque about any point is zero.
[1]

Condition 2:

[1]

(b) A uniform trapdoor of mass 12 kg and length 1.00 m is smoothly hinged to the wall as
shown in Fig. 1.1 (not drawn to scale). It is supported in equilibrium by a stay wire
connecting the wall to a point on the trapdoor at a distance of 0.25 m from its free end.
The stay wire makes an angle of 60° with the wall and the trapdoor makes an angle of
30° with the horizontal.

60°

0.25 m

30°

Fig. 1.1
Show that the tension in the stay wire is 78 N.

Taking moments about the hinge,


∑τ = 0
mg cos300 × 0.50 = T sin600 × 0.75
0.50 × 12 × 9.81cos300
T=
0.75 sin600
= 78 N
[2]
(c) A 20.0 kg sphere of uniform density rests between two smooth planes as shown in
Fig. 1.2.

Plane A
20 kg
Plane B
70° 30°

Fig. 1.2
Determine the magnitude of the force acting on the sphere exerted by each plane.

Let the force due to plane A be FA and the force due to plane B be FB. These 2
forces must be perpendicular to the sides of the respective planes.

Resolving vertically,
FB cos 300 + FA cos 700 = mg

Resolving horizontally,
FB sin 300 = FA sin 700

Solving simultaneously,
FA = 99.6 N
FB = 187 N
force due to plane A = N

force due to plane B = N [3]

2 The question is about reverse bungee jumping.

Fig. 2.1 shows the set up of a reverse bungee jumping. A capsule is connected by two
identical elastic cords each attached to a tower 30.0 m tall. The mass of the capsule when
fully loaded with three passengers has a total mass of about 300 kg. When released, the
capsule will shoot up at high speed.
10.0 m

Elastic
cords

30.0 m

Ground level
capsule
Fig. 2.1
(a) The original length of each of the elastic cords is 25.0 m with an elastic constant of
19 000 N m-1 and the capsule has an effective diameter of 2.0 m. Prove that the total
elastic potential energy at the ground level = 510 kJ when the cord length is 30.2 m.

Extension ≈ 30.2- 25 = 5.2 m

Total EPE = 2 × ½ kx2 = (19000)(5.2)2 = 510 kJ (proven)


[1]

(b) Fill in the blanks in the table below to determine the various amounts of energy when
the capsule starts from the ground level and shoots up to its highest point.

Gravitational
Total elastic Kinetic energy of
potential energy of
potential energy /kJ capsule /kJ
capsule /kJ

Ground level 510 0 0

30 m above the
0 88 422
ground

Highest point 336 174 0


[2]

(c) Use the value in (a) to determine the speed reached by the capsule when the cords
first become loose.

When the cord is unstretched,


Ht above the ground = 30 – √(252 – 42) = 10.8 m above the ground.
By conservation of energy,

gain in KE + gain in GPE = loss in EPE


½(300)v2 + (300)(9.81)(10.8) = 510 000

v = 56.5 m s-1
speed = m s-1 [2]

(d) State and explain the position where the apparent weight of the passenger will be the
greatest.

The apparent weight of the passenger will be the greatest at the lowest
point immediately after the capsule is released. The EPE is the largest, this
implies that extension is the largest which will results in the largest force.
[2]

3 A flat horizontal plate is made to oscillate with simple harmonic motion in a vertical direction
as shown in Fig. 3.1. The plate starts its oscillation at the equilibrium position and moves
downwards initially.

plate

oscillator

Fig. 3.1

A graph of velocity against displacement for this oscillation is shown in Fig. 3.2. Point S
marks the start of the oscillation.

velocity / m s-1
0.8

0.6
C
0.4

0.2
displacement
/m
0
- 0.04 - 0.03 - 0.02 - 0.01 0.01 0.02 0.03 0.04
- 0.2

- 0.4

- 0.6
S
- 0.8
Fig. 3.2
(a) Deduce, from Fig. 3.2,
(i) the amplitude of the oscillation,

amplitude = 0.035 m [1]

(ii) the angular frequency ω of the oscillation.

v 0 = ωx 0
0.66 = ω(0.035) (Accept v0 to be 0.65 – 0.67)
−1
ω = 19 rad s

ω= 19 rad s-1 [2]


(b) A mass of 0.100 kg is placed on the plate before oscillation is started.

(i) Determine the displacement of the plate when the mass just loses contact with
the plate.

ΣF = ma a = −ω2 x
mg − N = ma When a = g ,
N = m (g − a ) −9.81 = − (18.86)2 x
N = 0 when a = g x = 0.0276 m

displacement = m [3]

(ii) Mark on Fig. 3.2 the point C when the mass just loses contact. [1]

4 (a) State two conditions that must be satisfied in order to obtain observable interference
patterns.

The sources must be coherent.


The amplitude of the waves at the point of interference must be about
the same.
The distance between the two sources is much larger than the [2]
wavelength of the waves emitted.
The apparatus shown in Fig. 4.1 below (not to scale) is used to demonstrate two-source
interference.

Double slit Screen

Light,
D
wavelength λ

Fig. 4.1

(b) The separation of the two slits in the double slit arrangement is a and the interference
fringes are viewed on a screen at a distance D from the double slits. When light of
wavelength λ is incident on the double slit, the separation of the bright fringes on the
screen is x.

(i) Write the equation that links the quantities described in the above paragraph, and
state the assumption made in the use of that equation.

λD
x= condition: D>> a
a

[2]

(ii) The slits are separated by a distance of a, with the screen at a distance of 1.00 m
from the plane of the slits. The slits are illuminated by monochromatic light of
wavelength 589.3 nm traveling perpendicular to the plane of the slits. It was
observed that the distance between the two 4th order bright fringes are 20 mm.
Calculate the separation of the slits, a.

x = 20 / 8 = 2.5 mm

λD λD (589.3 × 10 - 9)(1.00)
x= a= = = 0.24 mm
a x 2.5 × 10 - 3

a= m [2]
(c) (i) Explain why the central fringe is always a bright one.

Since the central fringe is equal distance from the double slits, path
difference = 0 λ. Hence the waves from each slit must arrive in phase
resulting in constructive interference.

[1]

(ii) Explain why an experiment using two separate sources of light will not show
interference.

Two separate sources of light will not be coherent and hence


interference cannot take place.

[1]

5 (a) Define electric field strength.

The electric field strength at a point in an electric field is defined as the


electrostatic force acting per unit positive charge on a test charge placed at
that point. [1]

(b) Fig. 5.1 shows a stream of electrons entering a region between two parallel plates which
have a potential difference.

- 200 V

Parabolic path
between plates
Stream of electrons
Straight path
after plates

+200 V
Both blue and red paths are
Fig 5.1 accepted

(i) Draw on Fig 5.1, the electric field lines between the plates and the expected path
of the stream of electrons between and after the plates. [2]

(ii) Calculate the distance between the plates given that the electric field strength
between the plates is 2.0 x 104 N C−1.

V V 400
E= => d = = = 0.020 m
d E 2.0 × 104

distance = m [1]
(iii) Calculate the acceleration on the electrons between the plates

F = qE = 1.6 × 10 −19 × 2.0 × 10 4 = 3.2 × 10 −15 N


F 3.2 × 10 −15
a= = = 3.51 × 1015 m s −2 (downwards )
me 9.11 × 10 −31

acceleration = m s-2 [1]

(iv) Hence, given that the length of each plate is 0.040 m and initial horizontal speed
of the electrons is 1.0 x 108 m s-1, calculate the vertical deflection of the electron
at the end of the plates.

sx 0.040
t= = = 4.0 × 10−10 s
v x 1.0 × 108
1
sy = u y t + ay t 2
2
1
sy = (3.51× 1015 )(4.0 × 10−10 )2 = 0.00028m (downwards )
2

deflection = m [2]

6 A stationary Polonium-212 nuclide may undergo alpha decay spontaneously to produce the
stable lead-208 daughter nuclide as shown in the equation below:

212
84 Po ⎯⎯
→ 208
82 Pb + 24He

The rest masses of these nuclei are


212 208 4
84 Po : 211.9888 u; 82 Pb : 207.9766 u and 2 He : 4.0026 u.

(a) Calculate the total kinetic energy of the decay products.

Mass difference = 211.9888 u – (207.9766 u + 4.0026 u) = 0.0096 u

Total kinetic energy = (0.0096 u) c2 = (0.0096)(1.66 × 10-27)(3.00 × 108)2 = 1.43 ×


1012 J

total kinetic energy = J [2]

kinetic energy of He-4


(b) Determine the ratio of . Give your answer in 3 s.f.
kinetic energy of Pb-208
0 = MPbVPb − mHev He
MPbVPb = mHe v He
1 1
MPbVPb2
MPb = mHe v He
2
mHe
2 2
K .E .He MPb 207.9766
= = = 52.0
K .E .Po mHe 4.0026

ratio = [2]

(c) Hence, determine the kinetic energy of the alpha particle in MeV.

K .E .He
= 51.9604
K .E .Po
K .E .He
= 51.9604
K .E .Total − K .E .He
K .E .He = 51.9604(K .E .Total − K .E .He )
52.9604 K .E .He = 51.9604(1.43424 × 10 −12 )
K .E .He = 1.40716 × 10 −12 J
= 8.79 MeV

kinetic energy = MeV [2]

(d) To escape from the nucleus, the alpha particle must overcome the Coulomb barrier of
26 MeV. Using your answer in (c), comment on how the alpha particle can penetrate
the barrier.
The alpha particle’s 8.79 MeV of K.E. is unable to overcome the 26 MeV
Coulomb barrier by classical physics. However, the alpha particle can behave
as a wave and be associated with a wave function. The square of the amplitude
of the wave function represents the probability of locating the particle at that
point. Thus it is able to tunnel through the potential barrier and appear outside
the barrier with non-zero amplitude.
[2]
7 In the first half of the last century, numerous experiments were conducted to investigate the
absorption and the scattering of X-ray by matter.

It was discovered that when a monochromatic beam of X-rays is incident on a light element
such as carbon, the scattered X-rays have wavelengths dependent on the angle of scattering.

Compton (1923) assumed that the scattering process could be treated as an elastic collision
between an X-ray photon and a ‘free’ electron, and that energy and momentum would be
conserved.

(a) Explain what is meant by a photon.

A photon is a quantum of electromagnetic energy. The energy of a photon is


proportional to the frequency of the electromagnetic waves.

[1]

(b) The elastic collision between a photon and a stationary electron may be represented as
in Fig 7.1.

scattered photon
momentum ps
energy Es

θ
φ
incident photon
momentum pi
energy Ei
electron
mass m, speed v
Fig. 7.1

The incident photon has momentum pi and energy Ei. The photon is scattered through
an angle θ and, after scattering, has momentum ps and energy Es. The electron of mass
m, which was originally stationary, moves off with speed v at an angle φ to the original
direction of the incident photon.

(i) Write down equations, in terms of pi, ps, Ei, Es, m, v, θ and φ, that represent,
for this interactions,

1. Conservation of energy,

Ei = Es + ½ m v2
[1]
2. Conservation of momentum along the direction of the incident photon,

Pi = Ps cos θ + m v cos φ
[1]

(ii) Suggest, with a reason, whether the scattered photon will have a wavelength that
is greater or less than that of the incident photon.

Since some energy of the photon is converted to the k.e. of electron,


Ei > Es, hfi > hfs, λI < λs , the scattered photon has longer wavelength.

[2]

(c) In an experiment to provide evidence to justify Compton’s theory, measurements were


made on the wavelength λi of the incident photon, the wavelength λs of the scattered
photon and the angle θ of scattering. Some data from this experiment are given in Fig.
7.2.

λi / 10-12 m λs / 10-12 m θ/o Δλ=λs - λI /m

191.92 193.27 57 1.35 x 10-12

153.30 154.65 57 1.35 x 10-12

965.04 966.84 75 1.80 x 10-12

Fig. 7.2

Use the data in Fig 7.2 to show that, when a photon is scattered, the change in
wavelength produced is independent of the wavelength of the incident photon.

[2]

When θ is unchanged, Δλ is a constant. When θ is changed, Δλ changes.


Hence Δλ is independent of λI but dependent on θ.
(d) In this experiment, the uncertainty in the measurement of θ is ±5o.

Determine the value of cos θ, with its uncertainty, for the angle θ = 75o ± 5o.

cos 80o = 0.1736 cos 75o = 0.2588 cos 70o = 0.3420


Δ cos θ = (0.3420 - 0.1736)/2 = 0.084 = 0.08 or 0.09 (1 s.f.)
cos 75o = 0.2588 = 0.26 (2 d.p.)

or 0.2588 - 0.1736 = 0.0852 0.3420 - 0.2588 = 0.0832


Take max uncertainty,
Δ cos θ = 0.085 = 0.09 (1 sig)

cos θ = 0.26 ± 0.09 [3]

(e) Compton’s theory suggests that the change in wavelength Δλ is related to the angle θ
of scattering by the expression

Δλ= k (1 – cos θ)

where k is a constant.

Experimental data for the variation with cos θ of Δλ are shown in Fig 7.3.

(-0.50, 3.70)

2.50

(1.00, 0.12)

(i) On Fig. 7.3, draw the best-fit line for the points. [1]
(ii) Use two different ways to determine the constant k from the graph of Fig. 7.3.
Find the average value of k.
-12
Δλ= k –average
k cos θ value
(-kof
iskthe
= gradient
2.45 × 10and [4]
mk is the intercept)

k is the y-axis intercept.


k = 2.50 x 10-12

(3.70 − 0.12) X 10 −12


k = - gradient= − = 2.39 X 10−12
(−0.50) − 1.00

<k> = 2.45 x 10-12 m

(f) For a carbon atom, the binding energy of an electron is of the order of a few
electronvolts. Compton's theory assumes that the electrons are not bound in the atoms
but are free. Suggest whether, for 30 keV photons, this assumption is justified.

The binding energy of an electron of a carbon atom is very much smaller


than the energy of the incident photons (30 keV), the binding energy of the
electrons can be ignored. Thus it is justified to assume that the electrons
are free.
[1]
Section B

It is recommended that you spend about 30 minutes on this section.

8 Many musical instruments, such as organ pipes, flutes and clarinets, employ resonating air
columns to produce note of particular frequencies. The length of the resonating column may
be changed to produce a note of a different frequency. It is suggested that different volumes
of air in a container may resonate at frequencies which depend on the volume of the air.

Design an experiment to investigate how the resonant frequency of the fundamental mode of
vibration of air in a container depends on the volume of the air. You may assume that the
following apparatus is available with any other standard equipment which may be found in a
school or college:

Flute Microphone Ammeter


Measuring cylinder Bunsen burner Oscilloscope
Containers having different volumes Loudspeaker Bucket of water
Signal generator Voltmeter Thermometer

Your answer should contain a diagram showing how the chosen equipment would be
arranged, together with details of

(a) the procedure to be followed,

(b) the method by which the volume of the air and the resonant frequency may be
measured,

(c) the control of variables,

(d) any precautions you would take which may improve the accuracy of your experiment.

[12]
Diagram

Oscilloscope

Loudspeaker

Resonance
bottle
Signal
generator

1. Perform this experiment in a sound-proof room.


2. Fill up the resonance bottle completely with water. Pour the water from the resonance bottle
into a measuring cylinder, pouring out in stages if necessary, until the total volume VT of
water has been measured. Read all volumes at the lower meniscus.
3. Ensure room temperature is constant by monitoring using a mercury in glass thermometer.
4. Using a measuring cylinder, measure and pour a volume VW of 50 cm3 of water into the
empty resonance bottle. The volume of the air in the resonance bottle is now V = VT – Vw
5. Set up the resonance bottle with the apparatus as shown in the diagram above. Throughout
the experiment, clamp the resonance bottle and the measuring cylinder to avoid spillage or
breakage.
6. Switch on the signal generator. Starting with the lowest possible value of frequency,
gradually increase the frequency output of the signal generator. The loudness of the sound
from the resonance bottle will increase gradually even though the amplitude of the signal is
not adjusted. Stop increasing the frequency when the sound is perceived to be the loudest.
Make small changes to this frequency to locate the frequency at which the loudest value is
heard.
7. The amplitude of the sound wave to the speaker is kept constant.
8. The speaker is placed at the same position for every part of the experiment.
9. By multiplying the number of divisions on the x-scale occupied by a complete waveform in
the oscilloscope screen with the time scale of the oscilloscope, the periodic time of the
1
sound wave can be found. Calculate the resonant frequency by f = .
T
10. Add another 20 cm3 of water to the resonance bottle. Repeat the experiment to find 8 sets
of values of f vs V
11. Plot a graph of f vs V
OTHER ACCEPTABLE VARIATIONS
1. Use of constant frequency and gradually changing volume of water.
ƒ Start with full container of water and gradually remove water, to ensure first resonance
detected is fundamental mode. Use syringe or dropper for fine changes to the volume of
the container
ƒ Use of a container with some means of gradually draining water out.
2. Use of tuning forks instead of signal generator with loudspeaker
3. Use of ruler/vernier calipers to measure dimensions of resonance bottle and using
calculations to find volume of air in container.
ƒ V = π r2 d
4. Use of microphone and oscilloscope to detect loudest sound. Microphone must be place
OUTSIDE the resonance bottle.
1
Section A

Answer all the questions in this section.

1 (a) Define acceleration.

The acceleration of a body at any instant is defined as the time rate of


change of velocity of the body at that instant.
[1]

(b) A boy throws his slipper from a height of 1.0 m at 75o above the horizontal in an
attempt to hit a durian hanging on a tree which is 4.75 m above the ground. The slipper
reached its maximum height of 0.25 m directly below the durian. Neglect air resistance.

(i) Calculate the velocity of the slipper when the slipper leaves the boy’s hand.

v y 2 = u y 2 + 2a y s
+ve
0 = u y 2 + 2(−9.81)(4.75 − 1.0 − 0.25)
u y = 8.29 m s −1
8.29
sin75o = => u = 8.58 m s −1
u

velocity = m s-1 [2]

(ii) Calculate the horizontal distance travelled when his slipper hits the ground.

1
sy = uy t + ay t 2
2 +ve
1
−1.0 = (8.29)t + (−9.81)t 2
2
t = 1.80 s or − 0.113 s (rej )

sx = ux t
= (8.58cos75o ) × 1.80 = 4.0 m

distance = m [2]

(iii) Sketch a labelled velocity - time graph for the vertical component of the slipper
from the time it leaves the boy to the time when it hits the floor.

v/m s-1

8.29

t/ s
0.845 1.80

- 9.37
[2]
(iv) If the velocity calculated in part (b)(i) is the maximum velocity he can provide
when throwing his slipper, suggest with a reason what he should do in order to
ensure his slipper hits the durian.

He should go closer to the durian. Directly below the durian so that he can
exert his force in the vertical direction and initial vertical velocity is at its
greatest. [1]

2 (a) Define gravitational field strength and gravitational potential.

Gravitational field strength at a point is the force that would be experienced


per unit mass placed at that point.

Gravitational potential at a point is the work done per unit mass by an


external agent in bringing the mass from infinity to that point.

[2]

(b) A point S is located between the Earth and the Moon at a distance r from the Earth
along on the line joining the centres of the Earth and the Moon.
The distance from the centre of the Earth to the centre of the Moon is 3.8 x 108 m, the
mass of the Earth is 6.0 x 1024 kg and the mass of the Moon is 7.0 x 1022 kg.

Find the value of distance r, if S is the point where the resultant gravitational field
strength is zero.

GM ME MM
g= gE S gM
r2 x

At S,
gE – g M = 0
GME GME
2
− =0
r (r0 - r )2
6.0 x 1024 7.0 x 1022
=
r2 (3.8 x 108 - r )2
r = 3.4 x 108 m

r= m [2]

(c) Sketch a graph (without any values) showing the variation of gravitational field strength
with distance from the Earth, along the line joining the Earth and the Moon.

S r
E 0 M

[2]
(d) Explain why a space craft would require more energy to move from the Earth to the
Moon compared to the return journey from the Moon back to the Earth.

To move from Earth to Moon, the craft needs to do work against the larger
average net force acting over a longer distance up to the point S, beyond
that the net force does the work.
On the return journey the net average force from the moon to S is smaller
and over a shorter distance so the work done by the craft is less.
[2]
OR
For each journey, change in potential would be the area under the g-r
graph.
From the graph in (c) it can be seen that for a journey from E to M, there is
a greater increase in potential from E to S than a decrease from S to M.
For the return journey, the increase in potential from M to S is less than
the decrease in potential from S to E.
Thus, more work needs to be done in the forward journey from E to M.

3 Fig 3.1 shows a circuit for measuring a small e.m.f. produced by a solar cell.
10 V
0.5 Ω

5.0 Ω

Solar cell
V

Fig 3.1

(a) The galvanometer shows null deflection when the variable resistor is set to 300 Ω.
Determine the value of the e.m.f., V of the solar cell.

Total resistance = 0.5 + 5.0 + 300 = 305.5 Ω


By potential divider,
5.0
× 10 = 0.164 V ≈ 0.16 V
305.5

V= V [2]
(b) Fig 3.2 shows the 5.0 Ω resistor being replaced with a 1.2 m uniform resistance wire PQ of
total resistance of 7.0 Ω. The variable resistor remains at 300 Ω.

10 V
I 0.5 Ω

P Q

Solar cell moveable


contact R
V

Fig 3.2
(i) Calculate the current I, when the galvanometer shows null deflection

Total resistance = 0.5 + 7.0 + 300 = 307.5 Ω


V 10
I= = = 0.33 A
R 307.5

I= A [2]

(ii) Calculate the distance from P that contact R must be connected to wire PQ such
that the galvanometer shows null deflection.

Total resistance = 307.5 Ω


R
× 10 = 0.164
The resistance from P to the contact point R => 307.5
R => 5.04 Ω
5.04
Distance from P = × 1.2 = 0.864 m
7.0

Distance from P = m [3]

(iii) Explain why, this circuit is not suitable for measuring the e.m.f. of the solar cell
when the value of the e.m.f. of the solar cell is of the order of millivolts.

When the value of e.m.f. is of the order of millivolts, the fractional uncertainty
of the answer will be large as the answer is small, therefore it is not suitable.
[1]
4 Fig. 4.1 shows the cross section of the mechanism of a coin-operated vending machine.

magnets
coin insert speed
sensors

¢
¢
B
coin A

holder inlet
C ¢
reject
path

On entering the vending machine at AFig.


, the4.1holder will stop the coin momentarily before
releasing it into the inlet.

At the inlet, the coin will pass through 2 pairs of magnets and 2 pairs of speed sensors where
eddy current will be generated in the coin. The coin travels down the inlet more slowly due to
the presence of the magnet as compared to an inlet without the pair of magnets. If the speed
sensors indicate that the coin has the correct speed, the gate at B will open and accept the
coin. Otherwise, the gate at C will open and the coin will be rejected.

(a) State Faraday’s law of electromagnetic induction.

Faraday’s Law states that when there is a change in the magnetic flux linkage
of a conductor, an e.m.f. is induced in it and the magnitude of the e.m.f.
induced is proportional to the rate of change of magnetic flux linkage.
[1]

(b) Use Faraday’s Law of electromagnetic induction to explain the generation of eddy
current as the coin travels down the inlet.

As the coin moves down the inlet, it will approach and move away from the
magnets, experiencing increase and decrease of magnetic flux. By Faraday’s
Law, an e.m.f. will be induced in the coin. As the coin is a conductor, the e.m.f.
induced will induce an eddy current in the coin, which will produce an induced
flux to oppose the change causing it.

[3]
(c) Using energy conservation, explain why the magnets cause the coin to travel down
more slowly as compared to an inlet without the pair of magnets.

The generation of the eddy current generates electrical energy which will
dissipate as heat energy in the coin. By conservation of energy, the electrical
energy generated has been converted from the kinetic energy of the coin,
thus, the coin will travel down more slowly as compared to an inlet without the
pair of magnets.
[2]

(d) Describe how this mechanism can help to detect the correct denomination of coins.

Different denomination of coins has different dimensions and may be made of


different material. As such, the resistances of the coins are different which will
affect the magnitude of the eddy current induced in the coins. This will affect
the speed of the coins as they approach the speed sensors. By presetting
quantities to correspond to various dimensions, we can differentiate the
various denominations. [2]

5 (a) Explain why a laser can cut metal plates but ‘normal’ light of the same power cannot do
so.

Laser light is unidirectional, causing it to be more intense, where the


energy of the laser is concentrated in a smaller area thus more penetrative.
[2]

(b) Fig. 5.1 below shows the energy levels of a neon atom. In a helium-neon gas laser,
helium atoms in an excited state will excite the neon atoms from the state with energy
Eo to the state with energy E2 during collision. If the process of collision is to continue
throughout the container containing the helium and the neon gas, population inversion
will take place. Laser photons will be produced, whenever electrons make a transition
from energy level E2 to energy level E1.

E2 20.6 eV

E1 18.7 eV

E0 0 eV

Fig 5.1

(i) Describe what is meant by population inversion, with reference to the energy
level diagram in Fig 5.1.

Population inversion will occur if the ratio of the number of atoms in the
upper level to the number in the lower level is higher than normal. In this
particular example it would mean there are many more atoms in level E2
than in E1.
[2]
(ii) Explain how stimulated emission results in the laser properties.

Stimulated emission causes emission of electrons that are identical in


terms of frequency, phase and direction. The resultant beam is therefore
collimated, monochromatic and intense.

[2]

(iii) By referring to Fig 5.1, calculate the wavelength of the light produced by the laser.

hc
(20.6 -18.7) x (1.6 x 10-19) =
λ
λ = 6.5 x 10-7 m

wavelength = m [2]

Section B

Answer two questions in this section.

6 (a) Define linear momentum.

Linear momentum is defined as the product of a body’s mass and its


linear velocity.
[1]

Fig. 6.1 shows an airboat of mass 420 kg which is propelled forward by a propeller
generating a column of air backward.
propeller

airboat

Fig. 6.1

(b) (i) By using Newton’s laws, show that the forward thrust acting on the airboat is
given by F = πr2ρv2 where ρ is the density of air, r is the effective radius of
propeller and v is the speed of the air moving backward.

Rate of change of momentum of air = (mv-mu)/t


=m/t(v) [1] = pπr2v(v) = pπr2v2
By N2L, force acting on air by propeller = pπr2v2 backward
N3L, force acting on propeller by air = pπr2v2 forward. (shown)

[3]
(ii) Calculate the initial acceleration of the airboat when r = 0.70 m, v = 20 m s-1 and
ρ = 1.2 kg m-3.

∑F= ma
pπr2v2 = ma [1]
a = (1.2)(π)(0.72)(202)/420 = 1.76 m s-2

acceleration = m s-2 [2]


(c) (i) Calculate the rate of transfer of kinetic energy to the air by the propeller.

Rate of rate of transfer of KE = 1/2mv2 /t


= ½(ρ)(πr2)(v) (v2)
= (0.5)(1.2)(π)(0.702)(203) = 7400 W

Rate of transfer = W [2]

(ii) Given that the propeller is powered by a motor with a useful power 16 kW and
moving along a river with a constant speed of u. Use the answer for (c)(i) to
determine u.

Since speed is constant, a = 0


∑F= 0, F – f = 0
Hence f = 740 N

By conservation of energy,
Power = rate of transfer of KE to air + rate of work done against f
16 000 = 7400 + (740)(u)
u = 12 m s-1

u= m s-1 [3]
(d) (i) Fig. 6.2 show a side view of the boat partially submerged in water. Draw the other
forces acting on the airboat paying particular attention to the point of application
of these forces.

Force on
U, upthrust propeller

water level
Drag force
Total weight of
airboat [2]

Fig. 6.2
(ii) Fig. 6.3 shows the outline of an airboat and a speed boat. Based on the diagram
below, the front part of the airboat is wider or less streamline than a speedboat.
By considering the stability of the airboat, explain why this is so.

airboat speed boat

Fig. 6.3

When the front part is wider, it can displace more water in front,
causing the point of application of U to be closer to the front.

This will help to create a restoring torque to stabilize the boat.


[2]

(iii) By considering the airboat and the air generated as a system, explain why the
total momentum of this system is not conserved when the airboat is moving at a
constant speed.

The external forces acting on the system will be weight, upthrust and
drag force. Since mg is counter by U, hence the drag force will
results in a net external force acting on the system. Therefore, total
momentum is not conserved. [2]
(e) (i) Suggest why airboats are more suitable for use in shallow rivers compared to
other types of boats.

Since the propeller is above water, there is no concern about


whether the propeller will get struck in the river bed.

[1]

(ii) Discuss two problems caused by using a much bigger propeller in order to
increase the forward thrust.

Problem 1: A bigger propeller will results in an greater uneven


distribution of the weight affecting the stability of the boat.

Problem2: When the diameter of the air column increase, air


turbulence may come which will greatly reduce the efficiency of the
propeller.
[2]
Problem 3: The perpendicular distance between the thrust and drag
force will increase. The increase in torque will affect the stability of
boat.

7 (a) Explain what is meant by the internal energy of a system.

It is the sum of a random distribution [1] of kinetic and potential energies [1]
associated with the molecules of a system.

[2]

(b) State what is meant by saying a temperature is on an absolute scale.

An absolute scale means that the scale is theoretical and does not depend on
the property of any particular substance.
[1]

(c) A cake of mass 0.90 kg is cooked in an oven at a temperature of 180°C. It is taken out
of the baking tin onto a rack to cool in a kitchen of 20°C.

(i) State the final temperature of the cake.

T = 273.15 + 20 = 293.15 K
≈ 293 K
final temperature = K [1]

(ii) Calculate the energy released from the cake in cooling. Take the specific heat
capacity of the cake to be 990 J kg-1 K-1.

Q = mc(Δθ )
= 0.90 × 990 × 160 [1]
= 1.4 × 105 J [1]
energy released = J [2]

(d) The oven of volume 0.10 m3 cools down from 180°C to 25°C.

Calculate the change in the mass Δm of air in the oven between the two temperatures.
The pressure in the oven remains at an atmospheric pressure of 1.0 x 105 Pa. Assume
that air behaves ideally. [Relative molecular mass of air = 0.030 kg mol-1]
Let mi be the initial mass of the air.
pV = nRT
m
1.0 × 105 × 0.10 = i R (180 + 273.15) [1]
Mr
1.0 × 105 × 0.10 × 0.030
mi = [1]
R (180 + 273.15)
Let mf be the final mass of the air.
pV = nRT
m
1.0 × 105 × 0.10 = f R (25 + 273.15)
Mr
1.0 × 105 × 0.10 × 0.030
mf = [1]
R (25 + 273.15)
Δm = mf – mi = 0.041 kg [1]

Δm = kg [4]
(e) Air is mainly made up of nitrogen and oxygen. The mass of 1 nitrogen molecule is 28 u
while the mass of 1 oxygen molecule is 32 u.

Find the ratio


average speed of oxygen molecule at 180°C
average speed of nitrogen molecule at 25°C

1 3
m v 2 = kT
2 O2 O2 2 180
3kT180
vO = [1]
2 mO
2

3kT25
vN =
2 mN
2

vO mN T180
2
= 2
[1]
vN mO T25
2 2

28 × (180 + 273.15)
=
32 × (25 + 273.15)
= 1.15 [1]
ratio = [3]
(f) The gas in the cylinder of a diesel engine can be considered to undergo a cycle of
changes of pressure, volume and temperature. One such cycle, for an ideal gas, is
shown in Fig. 7.1.

Fig. 7.1
The table below shows the increase in internal energy which takes place during each of
the changes A to B, B to C, C to D and D to A.

Section of cycle Heat supplied Work done Increase in internal


to gas / J on gas / J energy of gas / J
A to B 0 - 300 - 300

B to C - 450 0 - 450

C to D 0 650 650

D to A 850 -750 100


Using Fig. 7.1, fill in the missing values in the table above. [3]

(g) In a continuous flow method for determining the specific heat capacity a liquid, the
liquid flows through the tube at 0.15 kg min-1, while the heater provides power at 25 W.
The temperatures of the liquid at the inlet and outlet are 15 oC and 19 oC, respectively.

With the inlet and outlet temperatures unchanged, the flow rate is increased to
0.23 kg min-1 and the power of the heater is increased to 37 W.

(i) Explain why it is necessary for the inlet and outlet temperatures to remain
unchanged.

This is to ensure that the rate of heat lost from the apparatus to the
environment remains unchanged for both of the experiments.

[1]
(ii) Determine the rate of heat loss of the liquid.

Q/t = (m/t) c (ΔT) + heat loss

0.15
25 = ( )c (19 − 15) + Ploss (1) [1]
60
0.23
37 = ( )c (19 − 15) + Ploss (2) [1]
60

Using equations (1) & (2),


Ploss = 2.5 W [1]

rate of heat loss = W [3]

8 (a) ‘X-rays are used to investigate the atomic structure of solids.’ Deduce from this
statement the wavelength of the X-rays used.

The wavelength of the X-rays is about the separation of the atoms in the solid (or
about the diameter of the atom, or about 10-10 m)

(b) ‘Sometimes, for example, in the case of rubber, electrons with a de Broglie wavelength
of about 0.11 nm are used instead of X-rays.’
Showing the steps in your calculations, determine the momentum of such electrons,

p = h/λ = 6.63 x 10-34/ 0.11 x 10-9 = 6.0 x 10-24 kg m s-1

momentum = N s [2]

(c) An X-ray tube operates with a potential difference of 100 kV between the anode and
cathode. The tube current is 20 mA. Only 1% of the energy of the cathode rays is
converted into X-rays.

Determine

(i) the rate of heat to be removed from the target in order to keep it at a steady
temperature,
Power developed = VI = 100x103 x 20 x 10-3 = 2.0 x 103 W
Power converted to heat = 2.0 x 103 x 99% = 1.98 x 103 W (2.0 kW)

rate = W [1]

(ii) the number of electrons which reach the target each second,

I = (N/t) e , (N/t) = 20 x 10-3/ 1.60 x 10-19 = 1.25 x 1017 s-1

number = [1]
(iii) the maximum energy of an X-ray photon produced, explain your working.

All the kinetic energy of an accelerated electron is converted to


one single photon of X-ray.
eV = hf
hf = 1.60 x 10-19 x 100x103 = 1.60 x 10-14 J

maximum energy = J [2]

(d) Fig 8.1 is a sketch of the X-ray spectrum produced by this tube for a particular metal
target. Fig 8.2 shows a sketch of the energy level of target material and how the Kα line
is formed. The tube voltage is 100 kV and the current is 20 mA.

N-shell
B
Intensity
Kγ M-shell

L-shell
Lα Kα Fig 8.1


A

M
0
0 Photon K-shell
energy
Fig 8.2

(i) Label the maximum energy of the X-ray photon you have calculated in (c)(iii) with
M on Fig 8.1. [1]

(ii) Label the Kα, and Lβ spectrum lines on Fig 8.1. [2]

(iii) Sketch on Fig 8.1 a spectrum for X-ray from the tube if the tube voltage is
reduced to 50 kV, the current remaining at 20 mA. Label this spectrum A. [1]

(iv) Also on the same axes, sketch a spectrum for for X-ray from the tube if the tube
current is increased to 30 mA the tube voltage remaining at 100 kV. Label this
spectrum B. [1]
(d) Explain how the characteristic and continuous parts of the spectrum are formed.

(i) Formation of characteristic parts of X-ray spectrum:

Line spectrum: When a high speed electron knocks out an orbiting electron in
the inner shells of a target atom, a ‘hole’ is formed. When an electron from an
outer shell of this atom fall into this ‘hole’, an X-ray photon of a particular
frequency will be emitted.

[2]

(ii) Formation of continuous parts of X-ray spectrum:

When high speed electrons are decelerating when they hit the target, according
to classical physics, electromagnetic waves in the X-ray region are given out.
Because the braking is a continuous process, X-rays of continuous spectrum is
obtained.

[2]

(f) The energy required to remove an electron from the various shells of the nickel atom is:

K shell 1.36 x 10-15 J


L shell 0.16 x 10-15 J
M shell 0.08 x 10-15 J

An X-ray tube with a nickel target emits the X-ray K radiation of nickel.

Determine

(i) the minimum potential difference across the tube,

To emit K radiation, must be able to knock out an electron from K-shell.

K.E. of colliding electron = loss in P.E.


1.36 x 10-15 = eV
V = 1.36 x 10-15 / 1.60 x 10-19 = 8.49 kV

potential difference = V [2]

(ii) the energy of the X-ray quantum of longest wavelength in the K-spectrum of
nickel.

X-ray photon of least energy in K-spectrum is obtained from transition of


electron from L to K shell.

(1.36 – 0.16) 10-15 = hf


hf = 1.20 x 10-15 J

energy = J [2]
PIONEER JUNIOR COLLEGE
Preliminary Examination

PHYSICS 9646/01
Higher 2

Paper 1 Multiple Choice


23 September 2010

1 hour 15 minutes
Additional Material: Multiple Choice Answer Sheet

READ THESE INSTRUCTIONS FIRST

Write in soft pencil.


Do not use staples, paper clips, highlighters, glue or correction fluid.
Write your name, class and index number on the Answer Sheet in the spaces provided.

There are forty questions on this paper. Answer all questions. For each question there
are four possible answers A, B, C and D.
Choose the one you consider correct and record your choice in soft pencil on the
separate Answer Sheet.

Read the instructions on the Answer Sheet very carefully.

Each correct answer will score one mark. A mark will not be deducted for a wrong
answer.
Any rough working should be done in this booklet.

This document consists of 16 printed pages.

[Turn over
2

Data

speed of light in free space, c = 3.00 × 10 8 m s–1

permeability of free space, μ 0 = 4π × 10 −7 H m–1

permittivity of free space, ε 0 = 8.85 × 10 −12 F m–1

= (1 (36π )) × 10 −9 F m–1

elementary charge, e = 1.60 × 10 −19 C

the Planck constant, h = 6.63 × 10 −34 J s

unified atomic mass constant, u = 1.66 × 10 −27 kg

rest mass of electron, me = 9.11× 10 −31 kg

rest mass of proton, m p = 1.67 × 10 −27 kg

molar gas constant, R = 8.31 J K–1 mol–1

the Avogadro constant, N A = 6.02 × 10 23 mol–1

the Boltzmann constant, k = 1.38 × 10 −23 J K–1

gravitational constant, G = 6.67 × 10 −11 N m2 kg–2

acceleration of free fall, g = 9.81 m s–2


3

Formulae

1 2
uniformly accelerated motion, s = ut + at
2
v 2 = u 2 + 2as

work done on/by a gas, W = pΔV

hydrostatic pressure, p = ρgh

Gm
gravitational potential, φ=−
r

displacement of particle in s.h.m., x = x 0 sin ωt

velocity of particle in s.h.m., v = v 0 cos ωt


2
= ±ω x 0 − x 2

resistors in series, R = R1 + R 2 + ...

resistors in parallel, 1/ R = 1/ R1 + 1/ R 2 + ...

Q
electric potential, V =
4πε 0 r

alternating current/voltage, x = x 0 sin ωt

8π 2 m(U − E )
transmission coefficient, T = exp(− 2kd ) where k =
h2

radioactive decay, x = x 0 exp( −λt )

0.693
decay constant, λ=
t1
2

[Turn over
4

1 In a simple electrical circuit, the potential difference across a resistor is measured as


(3.20 ± 0.01) V. The resistor is marked as having a value of 6.3 Ω ± 5 %.
If these values were used to calculate the power dissipated in the resistor, what would be
the percentage uncertainty in the value obtained?

A 5.3 % B 5.6 % C 6.0 % D 6.3 %

2 A helicopter ascends from rest from ground with an acceleration of 2.0 m s−2 upwards.
When it reaches an altitude of 100 m above the ground, it releases a pack of mass 10 kg.

Taking g as 10 m s−2 and ignoring air resistance, what is the speed of the pack just
before it reaches the ground?

A 20 m s−1 B 25 m s−1 C 45 m s−1 D 49 m s−1

3 A stone is thrown from a point X and follows a parabolic path. The highest point reached
is Y.

Assuming that air resistance is negligible, the vertical acceleration of the stone is

A zero at Y.

B greatest at Y.

C greatest at X.

D the same at X as at Y.

4 A tractor of mass 1500 kg pulls a trailer of mass 1500 kg. The total resistance to motion
has a constant value of 5000 N. One quarter of this resistance acts on the trailer.

When they are moving with an acceleration of 1.0 m s−2, the force exerted on the tractor
by the trailer is

A 1500 N. B 2750 N. C 5250 N. D 8000 N.

5 A body, initially at rest, explodes into two fragments of masses M and 3M having total
kinetic energy E.

The kinetic energy of the fragment of mass M after the explosion is

E E 2E 3E
A . B . C . D .
4 3 3 4
5

6 An object is acted on by two forces P and Q. A frictional force F holds the object in
equilibrium.

Which vector triangle could represent the relationship between these forces?

A B

Q F Q F

P P

C D

F Q F Q

P P

7 A bob of mass 0.50 kg is suspended by a string from the ceiling inside a train moving on
a straight level rail.

If the train has an acceleration of 0.20 g, what is the tension in the string in terms of g?

A 0.20 g B 0.51 g C 0.54 g D 0.60 g

8 What is the power required to give a body of mass m a forward acceleration a when it is
moving with velocity v up a frictionless slope inclined at an angle of θ to the horizontal?

A mav sin θ + mgv

B mav

C (mav + mgv ) sinθ


D mav + mgv sin θ

[Turn over
6

9 An object of mass m is released from rest from point A which is at a height 2h above X
and slides down a frictionless surface. The object passes point X with a velocity u as
shown in the diagram.

B 2h
u
h

A second object of mass 2m is released from rest from point B which is at a height h
above X.

The velocity with which the second object passes point X in terms of u is

1 1
A 2u . B u. C u. D u.
2 2

10 The maximum safe speed of a car rounding an unbanked corner of radius 50 m is


20 m s−1.

If the road is banked at an angle θ to the horizontal, no frictional force is required for the
car to turn round the banked road with the same speed and turning radius.

What is the fractional change in the normal reaction force on the car?

A 0.29 B 1.29 C 11.8 D 12.8

11 Two moons were discovered to be orbiting around Jupiter with the same period.

Which one of the following quantity for the moons must be the same?

A mass

B momentum

C kinetic energy

D radii of orbit

12 The Earth has density ρ, radius R and gravitational field strength g on the surface, and
mass M.

An expression for g is

GM GM 4πGρ 4πGρ
A . B 2
. C . D .
R R 3R 3R 2
7

13 A 20 kg satellite requires a speed of 2 km s−1 to stay in a circular orbit at a certain height


above the Earth.

What would be the speed, in km s−1, of a 100 kg satellite in the same orbit?

A 1 B 2 C 2 D 4

14 A particle oscillates with simple harmonic motion along a line with a maximum speed v 0 .

When the displacement of the particle is half of its amplitude, its speed is

1 1 3 3
A v0 . B v0 . C v0 . D v0 .
4 2 4 2

15 A point mass moves with simple harmonic motion.

Which of the following statements is false?

A The maximum kinetic energy of the mass is dependent on the frequency of the
oscillation.

B The time taken for the system to change from maximum kinetic energy to maximum
potential energy is a quarter of the period of the oscillation.

C An oscillation system with larger amplitude will have a greater maximum velocity.

D An oscillation system with larger amplitude will have a longer period.

16 Which statement about internal energy is correct?

A The internal energy of a system can be increased without transfer of energy by


heating.

B The internal energy of a system depends only on its temperature.

C When the internal energy of a system is increased, its temperature always rises.

D When two systems have the same internal energy, they must be at the same
temperature.

[Turn over
8

17 A container of ice is heated by an electric heater. The graph below shows the variation of
the temperature of the ice with time.

temperature

T
S
U
R

V Q
P
time

Which part of the graph shows that the specific latent heat of vaporisation of water is
greater than its specific latent heat of fusion?

A The gradient of the graph at T is greater than the gradient at R.

B The length of line S is greater than the length of line Q.

C The gradient of the graph at P is greater than the gradient at R.

D The value of U is greater than the value of V.

18 A mass of an ideal gas of volume V and pressure p undergoes the cycle of changes
shown in the graph below.

p / 10 5 Pa

7 K

L
4

1 J M
V / 10 −10 m3
0 1 3 8

Which of the following shows the states of the gas arranged in the order of decreasing
temperature?

A JKLM

B JKML

C KLMJ

D LMKJ
9

19 A plane wave of amplitude A is incident on a surface area S placed so that it is


perpendicular to the direction of travel of the wave. The energy per unit time intercepted
by the surface is E.

A
The amplitude of the wave is reduced to and the area of the surface is increased to
2
2S.

How much energy per unit time is intercepted by this surface?

E
A 4E B E C 2E D
2

20 Which of the following statements is true between a stationary and a progressive wave?

A A progressive wave would undergo plane polarisation while a stationary wave will
not.

B The particles in a stationary wave are stationary while that of a progressive wave are
vibrating.

C The particles in a progressive wave are oscillating at simple harmonic motion while
those in a stationary wave are vibrating about their fixed positions.

D The particles in a stationary wave have varying amplitude while a progressive wave
has particles with fixed amplitude.

21 The length l of an air column is slowly increased from zero while a note of constant
frequency is produced by a tuning fork placed in front of it.

air column
tuning fork

When l reaches 20 cm the sound increases greatly in volume.

What is the wavelength of the sound wave produced by the tuning fork?

A 20 cm B 40 cm C 80 cm D 100 cm

[Turn over
10

22 The figure below represents a longitudinal wave travelling from the left to the right at a
frequency of 100 Hz. Two particles in the medium are labelled X and Y.

X Y

If the distance between X and Y is 100 m, what is the speed of the wave in the medium?

A 2000 m s−1 B 2200 m s−1 C 3300 m s−1 D 4000 m s−1

23 Electric field strength is defined as force per unit positive charge on a small test charge.

Why is it necessary for the test charge to be small?

A so that the test charge does not distort the electric field

B so that the force on the test charge is small

C so that the test charge does not create any forces on nearby charges

D so that the Coulomb’s law for point charges is obeyed

24 The electric potentials V are measured at distances x from P along a line PQ.

The results are:

V/V 13 15 18 21 23
x/m 0.020 0.030 0.040 0.050 0.060

The component along PQ of the electric field for x = 0.040 m is approximately

A 300 V m−1 towards Q.

B 300 V m−1 towards P.

C 450 V m−1 towards Q.

D 450 V m−1 towards P.

25 Two wires P and Q, each of the same length and material, are connected in parallel to a
battery. The diameter of P is half that of Q.

What fraction of the total current passes through P?

A 0.20 B 0.25 C 0.33 D 0.50


11

26 The figure below shows a circuit with two batteries in opposition to each other. One has
an e.m.f. E1 of 6.0 V and internal resistance r1 of 2.0 Ω and the other an e.m.f. E 2 of
4.0 V and internal resistance r2 of 8.0 Ω.

E1 = 6.0 V, r1 = 2.0 Ω

X Y

E2 = 4.0 V, r2 = 8.0 Ω

What is the potential difference across X and Y?

A 1.0 V B 2.0 V C 5.0 V D 5.6 V

27 Three similar light bulbs are connected to a constant voltage d.c. supply as shown in the
diagram. Each bulb operates at normal brightness and the ammeter (of negligible
resistance) registers a steady current.

The filament of one of the bulb breaks. What happens to the ammeter reading and to the
brightness of the remaining bulbs?

ammeter reading bulb brightness

A increases decreases

B increases increases

C unchanged unchanged

D decreases unchanged

[Turn over
12

28 The diagram shows a network of three resistors. Two of which, marked R, are identical.
The other one has a resistance of 5.0 Ω. The resistance between Y and Z is found to be
2.5 Ω.
Y

R
X

5.0 Ω
R

What is the resistance between X and Y?

A 0.21 Ω B 0.53 Ω C 1.9 Ω D 4.8 Ω

29 An electron enters a region at point O at an angle of 45° to the x-axis as shown. The
region contains a magnetic field which is directed into the plane of the page.

magnetic
y field
X
45°
x
O

An electric field can be applied such that the electron is able to pass through the region
undeflected. Which of the following correctly shows the direction of the electric field
from O?
y

A
D B

x
O

C
13

30 An electron is moving along the axis of a solenoid carrying a current.

Which of the following is a correct statement about the electromagnetic force acting on
the electron?

A The force acts radially inwards.

B The force acts radially outwards.

C The force acts in the direction of motion.

D No force acts.

31 A magnetic field is applied perpendicularly to the plane of a flat coil of copper wire. The
⎛ 2π ⎞
time variation of the magnetic flux density is given by B0 sin⎜ t ⎟ , as shown graphically
⎝T ⎠
below.

B0

T 3T
2 4
t
0 T T 3T
8 4 8

− B0

At which of the following values of t is the magnitude of the e.m.f. induced in the coil a
maximum?

T 3T T 3T
A B C D
8 8 2 4

[Turn over
14

32 At the beginning of a horse-race, a horizontal straight wire of length 20 m is raised


vertically through a height of 3.0 m in 0.20 s.

20 m

3.0 m

The horizontal component of the Earth’s magnetic field strength perpendicular to the wire
is 2.0 × 10 −5 T.

What is the average e.m.f. induced across the ends of the wire?

A zero B 0.24 mV C 1.2 mV D 6.0 mV

33 A signal generator produces either (a) a sinusoidal or (b) a square wave with the same
peak value V0 of e.m.f. as shown below.

e.m.f. e.m.f.

V0 V0

t t
0 T 2T 0 T 2T

− V0 − V0

(a) (b)

mean power of case (a)


When the signals are applied to a resistor, what is the ratio of ?
mean power of case (b)

1 1 1
A B C D 1
4 2 2
15

34 An alternating current of peak value 2 A and a steady direct current I flowing through
identical resistors dissipate heat at equal rates.

What is the value of current I ?

1
A A B 2 A C 2A D 2 2 A
2 2

35 A clean plate, made of metal with work function energy of 2.36 eV, is illuminated
with ultra violet light of wavelength 370 nm.

What is the maximum energy of the emitted photoelectrons?

A 1.00 eV B 3.36 eV C 5.38 eV D 5.72 eV

36 The diagram below shows a typical X-ray spectrum produced by an X-ray tube.

intensity

wavelength
0 λ1 λ2

The accelerating voltage across the X-ray tube is increased.

Which of the following corresponds to the changes, if any, in λ1 and λ2 ?

λ1 λ2

A no change no change

B no change decrease

C decrease no change

D decrease decrease

[Turn over
16

37 Which of the following best describes a hole in an intrinsic semiconductor?

A A missing valence electron in a Group III atom.

B A missing valence electron in a Group IV atom.

C A missing valence electron in a Group V atom.

D An ionised acceptor atom.

38 Why is laser light monochromatic?

A The excited electrons are in a metastable state.

B The system is in a state of population inversion.

C The emitted photon and the incident photon are of the same phase.

D Photons of the same energy as that of the incident photons are emitted when the
electrons transit down from a higher energy level.

39 The half-life of a certain radioactive isotope is 32 hours.

What fraction of a sample would remain after 16 hours?

A 0.25 B 0.29 C 0.71 D 0.75

40 A high energy α -particle collides with a 14


7N nucleus to produce a 17
8O nucleus.

What could be the other products of this collision?

A a γ -photon alone

B a γ -photon and a β -particle

C a γ -photon and a neutron

D a γ -photon and a proton

End of paper
5 Name Class Index Number

PIONEER JUNIOR COLLEGE


Preliminary Examination

PHYSICS 9646/02
Higher 2

Paper 2 Structured Questions


16 September 2010

1 hour 45 minutes
Candidates answer on the Question Paper.
No Additional Materials are required.

READ THESE INSTRUCTIONS FIRST

Write your name, class and index number on all the work you hand in.
Write in dark blue or black pen.
You may use a soft pencil for any diagrams, graphs or rough working.
Do not use staples, paper clips, highlighters, glue or correction fluid.

Section A
Answer all questions.
It is recommended that you spend about 1 hour 15 minutes on this section.

Section B
Answer Question 7.
It is recommended that you spend about 30 minutes on this
section.
For Examiner’s Use
At the end of the examination, fasten all your work securely 1 / 6
together.
The number of marks is given in brackets [ ] at the end of 2 / 6
each question or part question. 3 / 12
4 / 10
5 / 10
6 / 16
7 / 12
Data
Total / 72

This document consists of 22 printed pages.

[Turn over
2

Data

speed of light in free space, c = 3.00 × 10 8 m s–1

permeability of free space, μ 0 = 4π × 10 −7 H m–1

permittivity of free space, ε 0 = 8.85 × 10 −12 F m–1

= (1 (36π )) × 10 −9 F m–1

elementary charge, e = 1.60 × 10 −19 C

the Planck constant, h = 6.63 × 10 −34 J s

unified atomic mass constant, u = 1.66 × 10 −27 kg

rest mass of electron, me = 9.11× 10 −31 kg

rest mass of proton, m p = 1.67 × 10 −27 kg

molar gas constant, R = 8.31 J K–1 mol–1

the Avogadro constant, N A = 6.02 × 10 23 mol–1

the Boltzmann constant, k = 1.38 × 10 −23 J K–1

gravitational constant, G = 6.67 × 10 −11 N m2 kg–2

acceleration of free fall, g = 9.81 m s–2


3

Formulae

1 2
uniformly accelerated motion, s = ut + at
2
v 2 = u 2 + 2as

work done on/by a gas, W = pΔV

hydrostatic pressure, p = ρgh

Gm
gravitational potential, φ=−
r

displacement of particle in s.h.m., x = x 0 sin ωt

velocity of particle in s.h.m., v = v 0 cos ωt


2
= ±ω x 0 − x 2

resistors in series, R = R1 + R 2 + ...

resistors in parallel, 1/ R = 1/ R1 + 1/ R 2 + ...

Q
electric potential, V =
4πε 0 r

alternating current/voltage, x = x 0 sin ωt

8π 2 m(U − E )
transmission coefficient, T = exp(− 2kd ) where k =
h2

radioactive decay, x = x 0 exp( −λt )

0.693
decay constant, λ=
t1
2

[Turn over
4

Section A

Answer all questions.


It is recommended that you spend about 1 hour 15 minutes on this section.

1 (a) Define velocity and acceleration.

.......................................................................................................................................

.......................................................................................................................................

................................................................................................................................. [2]

(b) Fig. 1.1 shows the variation with time t of the velocity v for an object.

v / m s–1

20

10

t/s
0 2.0 4.0 6.0 8.0 10.0 12.0

−10

−20

Fig. 1.1

(i) State the time at which the object is at maximum displacement from the starting
point.

time = ........................................ s [1]

(ii) Calculate the displacement of the object at t = 12.0 s.

displacement = ........................................ m [1]


5

(iii) On Fig. 1.2, sketch a graph to show the variation with time t of the displacement s for
the object. (You are not expected to label values of the displacement.)

s/m

t/s
0 2.0 4.0 6.0 8.0 10.0 12.0

Fig. 1.2

[2]

[Turn over
6

2 A solid iron sphere of density 8000 kg m–3 and volume 4.50 × 10 −4 m3 is completely
submerged in a liquid of density 800 kg m–3. The iron sphere is resting on a spring, as
shown in Fig. 2.1. The spring is compressed by 10.2 cm.

iron sphere
liquid

compressed spring

Fig. 2.1

(a) Show that the upthrust on the iron sphere is 3.53 N.

[1]

(b) Hence, calculate the spring constant of the spring.

spring constant = ........................................ N m–1 [2]


7

(c) A string of breaking strength 32.0 N is used to lift the iron sphere vertically upwards,
as shown in Fig. 2.2. The iron sphere is then lifted partially out of the liquid as shown
in Fig. 2.3.

string

Fig. 2.2 Fig. 2.3

(i) Explain why the string breaks.

..................................................................................................................................

..................................................................................................................................

........................................................................................................................... [1]

(ii) Calculate the volume of the fluid displaced at the instant when the string breaks.

volume = ........................................ m3 [2]

[Turn over
8

3 (a) A wine glass can be shattered through resonance by maintaining a certain frequency
of high intensity sound wave. Fig. 3.1 shows the side view of a wine glass vibrating in
response to such a sound wave.

On Fig. 3.2, sketch a possible standing wave pattern on the rim of the glass as seen
from the top.

Fig. 3.1 Fig. 3.2

[2]

(b) The speed v of a progressive wave is given by the expression

v = fλ .

A stationary wave does not have a speed. By reference to the formation of a


stationary wave, explain the significance of the product fλ for a stationary wave.

.......................................................................................................................................

.......................................................................................................................................

.......................................................................................................................................

.......................................................................................................................................

.......................................................................................................................................

................................................................................................................................. [3]

(c) Explain what is meant by diffraction of a wave.

.......................................................................................................................................

.......................................................................................................................................

................................................................................................................................. [2]
9

(d) A narrow beam of coherent light of wavelength 589 nm is incident normally on a


diffraction grating having 4.00 × 10 2 lines for every 1 mm.

(i) Determine the number of orders of diffracted light that are visible on each
side of the zero order.

number of orders = ........................................ [2]

(ii) A student suspects that there are in fact two wavelengths of light in the incident
beam, one at 589.0 nm and the other at 589.6 nm.

1. State the order of diffracted light at which the two wavelengths are most likely
to be distinguished.

order = ........................................ [1]

2. The minimum angular separation of the diffracted light for which two
wavelengths may be distinguished is 0.10°. By means of suitable calculations,
explain whether the student can observe the two wavelengths as separate
images.

..................................................................................................................................

..................................................................................................................................

........................................................................................................................... [2]

[Turn over
10

4 (a) Explain what is meant by a field of force.

.......................................................................................................................................

................................................................................................................................. [1]

(b) Describe, by means of a well-labelled diagram, the motion of electrons moving at


right angles in a uniform

(i) electric field,

..................................................................................................................................

........................................................................................................................... [2]

(ii) magnetic field.

..................................................................................................................................

........................................................................................................................... [2]
11

(c) A rectangular strip of copper of dimensions 10 mm × 5 mm × 1 mm carries a


conventional current I. A magnetic field B of flux density 1.0 T is applied in a direction
perpendicular to the strip as shown in Fig. 4.1.

10 mm

P Q 5 mm
I

S R
1 mm
B = 1.0 T

Fig. 4.1

(i) Explain how a voltage is set up across side PQ with respect to SR.

..................................................................................................................................

..................................................................................................................................

..................................................................................................................................

..................................................................................................................................

..................................................................................................................................

........................................................................................................................... [3]

(ii) Calculate the voltage, given that the velocity of the electrons is 2.52 × 10 −5 m s–1.

voltage = ........................................ V [2]

[Turn over
12

5 (a) Explain what is meant by

(i) binding energy of a nucleus,

..................................................................................................................................

..................................................................................................................................

........................................................................................................................... [2]

(ii) nuclear fission.

..................................................................................................................................

..................................................................................................................................

........................................................................................................................... [2]

(b) The binding energy per nucleon varies with nucleon number in the way shown in
Fig. 5.1.

binding energy / (MeV per nucleon)

9.0
8.9
8.7

nucleon number
90 143 236

Fig. 5.1

During one particular fission process, a Uranium-236 nucleus gives, among its fission
products, a Strontium-90 nucleus and a Xenon-143 nucleus. The equation is given by

236
92 U + 01n → 90
38 Sr + 143
54 Xe + 401 n + energy .
13

(i) Use the values on Fig. 5.1 to calculate the energy released during this fission
process.

energy released = ........................................ MeV [2]

(ii) What is the role of the neutrons in the fission reaction? Why are they not taken
into account in the calculation in (b)(i)?

..................................................................................................................................

..................................................................................................................................

..................................................................................................................................

........................................................................................................................... [2]

(iii) Why does a release of energy occur when there is an increase in the binding
energy?

..................................................................................................................................

..................................................................................................................................

..................................................................................................................................

........................................................................................................................... [2]

[Turn over
14

6 X-rays are emitted when a metal target is bombarded by high-energy electrons. The
X-ray spectrum consists of a broad continuous spectrum and a series of sharp lines
known as the line or characteristic X-ray spectrum.

(a) Explain how the K α -line arises.

.......................................................................................................................................

.......................................................................................................................................

.......................................................................................................................................

.......................................................................................................................................

................................................................................................................................. [2]

(b) In 1913, a British physicist H. G. J. Moseley measured the wavelength of the


characteristics X-rays from a number of elements. He noted that when the square-
root of the frequency f is plotted against the position Z (atomic number) of the
element in the periodic table, a straight line results.

The energy for a multielectron atom is given by the expression

k (Z − 1)
2
En = −
n2

where k is a constant, and n is an integer. ( n = 1 for K-shell, n = 2 for L-shell, etc.)

(i) Show that the frequency f of the K α -line is given by the expression

f = C (Z − 1)

where C is a constant.

[2]
15

(ii) The experimental data for the variation with Z of f are shown in Fig. 6.1.

On Fig. 6.1, draw the best-fit line for the points. [1]

1
9
f / 10 Hz 2

1.6

1.5

1.4

1.3

1.2

1.1

1.0

0.9 Z
21 22 23 24 25 26 27 28 29 30 31 32 33

Fig. 6.1

[Turn over
16

(iii) State two different ways by which the constant C may be determined from the
graph of Fig. 6.1.

..................................................................................................................................

........................................................................................................................... [1]

(iv)Determine the constant C, with its unit, using one of the ways described in (b)(iii).

C = ........................................ [2]

(v) Hence, or otherwise, determine the wavelength of the K α -line for copper whose
atomic number Z is 29.

wavelength = ........................................ m [2]


17

(c) The X-ray spectrum can be studied using the X-ray diffraction. A particular set of
crystal planes having a known spacing d is chosen. These planes effectively reflect
different wavelengths at different angles.

incident beam reflected beam


ray 1

ray 2

θ θ
upper plane
d
lower plane

Fig. 6.2

For constructive interference to take place, the path difference between the two rays
must be an integer multiple of the wavelength λ , where

path difference = mλ

and m = 1, 2, 3, … …

(i) Using Fig. 6.2, show that the path difference between the two rays is 2d sin θ ,
where θ is the angle between the rays and the crystal planes.

[2]

[Turn over
18

(ii) Fig. 6.3 shows a graph of intensity against angular position θ (measured in
degrees) for the diffraction of an X-ray beam by a crystal. The beam consists of
two wavelengths, and the spacing d between the reflecting planes is 0.94 nm.

intensity

θ
0 0.4 0.8 1.2 1.6 2.0 2.4 2.8

Fig. 6.3

The condition for constructive interference is given by the expression


2d sin θ = mλ found in (c)(i), which is known as the Bragg’s law.

Using Bragg’s law, calculate the values of these two wavelengths of the X-ray
beam.

λ1 = ........................................ pm [1]

λ2 = ........................................ pm [1]

(iii) A standard optical diffraction grating where the grating spacing is 3000 nm
cannot be used to discriminate between different wavelengths in the X-ray region
of approximately 0.1 nm. Explain quantitatively why this is so.

..................................................................................................................................

..................................................................................................................................

........................................................................................................................... [2]
19

Section B

It is recommended that you spend about 30 minutes on this section.

7 The attenuation of a γ-ray beam is the reduction in its intensity due to its passage
through a material. One way of investigating the attenuation of a γ-ray beam is to
measure the half-value thickness. The half-value thickness is the thickness of material
that reduces the intensity of the γ-ray beam to half its original value.

Design an experiment to measure the half-value thickness of a material. You should


draw a diagram showing the arrangement of your equipment. In your account you should
pay particular attention to

(a) the procedure to be followed,

(b) how the γ-ray would be detected and the measurements that would be taken,

(c) the control of variables,

(d) any safety precautions,

(e) any precautions that you would take to improve the accuracy of the experiment.

[12]

[Turn over
20

Diagram

....................................................................................................................................................

....................................................................................................................................................

....................................................................................................................................................

....................................................................................................................................................

....................................................................................................................................................

....................................................................................................................................................

....................................................................................................................................................

....................................................................................................................................................

....................................................................................................................................................

....................................................................................................................................................

....................................................................................................................................................

....................................................................................................................................................

....................................................................................................................................................

....................................................................................................................................................
21

....................................................................................................................................................

....................................................................................................................................................

....................................................................................................................................................

....................................................................................................................................................

....................................................................................................................................................

....................................................................................................................................................

....................................................................................................................................................

....................................................................................................................................................

....................................................................................................................................................

....................................................................................................................................................

....................................................................................................................................................

....................................................................................................................................................

....................................................................................................................................................

....................................................................................................................................................

....................................................................................................................................................

....................................................................................................................................................

....................................................................................................................................................

....................................................................................................................................................

....................................................................................................................................................

....................................................................................................................................................

....................................................................................................................................................

....................................................................................................................................................

....................................................................................................................................................

....................................................................................................................................................

....................................................................................................................................................

....................................................................................................................................................

....................................................................................................................................................

[Turn over
22

....................................................................................................................................................

....................................................................................................................................................

....................................................................................................................................................

....................................................................................................................................................

....................................................................................................................................................

....................................................................................................................................................

....................................................................................................................................................

....................................................................................................................................................

....................................................................................................................................................

....................................................................................................................................................

....................................................................................................................................................

....................................................................................................................................................

....................................................................................................................................................

....................................................................................................................................................

....................................................................................................................................................

....................................................................................................................................................

....................................................................................................................................................

....................................................................................................................................................

....................................................................................................................................................

....................................................................................................................................................

....................................................................................................................................................

....................................................................................................................................................

....................................................................................................................................................

....................................................................................................................................................

....................................................................................................................................................

....................................................................................................................................................

....................................................................................................................................................

End of paper
Name Class Index Number

PIONEER JUNIOR COLLEGE


Preliminary Examination

PHYSICS 9646/03
Higher 2

Paper 3 Longer Structured Questions


20 September 2010

2 hours
Candidates answer on the Question Paper.
No Additional Materials are required.

READ THESE INSTRUCTIONS FIRST

Write your name, class and index number on all the work you hand in.
Write in dark blue or black pen.
You may use a soft pencil for any diagrams, graphs or rough working.
Do not use staples, paper clips, highlighters, glue or correction fluid.

Section A
Answer all questions.

Section B
Answer any two questions.

You are advised to spend about one hour on each section.


For Examiner’s Use
At the end of the examination, fasten all your work securely
together. 1 / 12
The number of marks is given in brackets [ ] at the end of
each question or part question. 2 / 9
3 / 10
4 / 9
5 / 20
6 / 20
7 / 20
Total / 80

This document consists of 26 printed pages.

[Turn over
2

Data

speed of light in free space, c = 3.00 × 10 8 m s–1

permeability of free space, μ 0 = 4π × 10 −7 H m–1

permittivity of free space, ε 0 = 8.85 × 10 −12 F m–1

= (1 (36π )) × 10 −9 F m–1

elementary charge, e = 1.60 × 10 −19 C

the Planck constant, h = 6.63 × 10 −34 J s

unified atomic mass constant, u = 1.66 × 10 −27 kg

rest mass of electron, me = 9.11× 10 −31 kg

rest mass of proton, m p = 1.67 × 10 −27 kg

molar gas constant, R = 8.31 J K–1 mol–1

the Avogadro constant, N A = 6.02 × 10 23 mol–1

the Boltzmann constant, k = 1.38 × 10 −23 J K–1

gravitational constant, G = 6.67 × 10 −11 N m2 kg–2

acceleration of free fall, g = 9.81 m s–2


3

Formulae

1 2
uniformly accelerated motion, s = ut + at
2
v 2 = u 2 + 2as

work done on/by a gas, W = pΔV

hydrostatic pressure, p = ρgh

Gm
gravitational potential, φ=−
r

displacement of particle in s.h.m., x = x 0 sin ωt

velocity of particle in s.h.m., v = v 0 cos ωt


2
= ±ω x 0 − x 2

resistors in series, R = R1 + R 2 + ...

resistors in parallel, 1/ R = 1/ R1 + 1/ R 2 + ...

Q
electric potential, V =
4πε 0 r

alternating current/voltage, x = x 0 sin ωt

8π 2 m(U − E )
transmission coefficient, T = exp(− 2kd ) where k =
h2

radioactive decay, x = x 0 exp( −λt )

0.693
decay constant, λ=
t1
2

[Turn over
4

Section A

Answer all the questions in this section.

1 (a) State Newton’s second law of motion.

.......................................................................................................................................

................................................................................................................................. [1]

(b) Fig. 1.1 shows block A of mass 4.0 kg and block B of mass 1.0 kg connected by a
light cord that passes over a frictionless pulley. Block A lies on a rough plane inclined
at 45° to the horizontal. The frictional force between block A and the plane is 15 N.

4.0 kg
A

rough plane
6.0 kg
1.0 kg B
C
45° smooth ground

Fig. 1.1

(i) Determine the magnitude of the acceleration of the two blocks and the tension in
the cord.

acceleration = ........................................ m s–2

tension = ........................................ N [4]


5

(ii) When block A is 1.0 m vertically above the ground, the cord breaks. The velocity
of block A at that instant is 0.5 m s–1. Calculate the speed of block A just before it
reaches the ground.

speed = ........................................ m s–1 [2]

(c) After reaching the smooth ground, block A travels some further distance before
colliding with a stationary block C of mass 6.0 kg. The velocity of block A before
collision is 1.6 m s–1, as shown in Fig. 1.2.

1.6 m s–1 6.0 kg

C
A

Fig. 1.2

(i) State the principle of conservation of momentum.

..................................................................................................................................

........................................................................................................................... [1]

(ii) Upon collision, block C moves to the right with a speed of 0.70 m s–1. Calculate
the speed of block A immediately after the collision.

speed = ........................................ m s–1 [2]

[Turn over
6

(iii) Hence, discuss quantitatively, whether the collision between blocks A and C is
elastic.

..................................................................................................................................

........................................................................................................................... [2]
7

2 A body M of mass 200 g moves with simple harmonic motion of amplitude 15 cm and
period 2.0 s.

(a) Calculate

(i) its maximum speed,

maximum speed = ........................................ m s–1 [1]

(ii) its maximum acceleration,

maximum acceleration = ........................................ m s–2 [1]

(iii) its displacement 0.10 s after passing through its maximum displacement.

displacement = ........................................ m [2]

[Turn over
8

(b) (i) Show that the total energy of a body of mass m undergoing simple harmonic
1 2
motion is mω 2 x 0 .
2

[2]

(ii) On Fig. 2.1, sketch the graphs of total energy ET , kinetic energy E K and
potential energy E P of the body against its displacement x from the equilibrium
position. Label the graphs clearly, with appropriate quantities m , ω and x 0
marked on the axes.

energy / J

x/m
0

Fig. 2.1

[3]
9

3 Fig. 3.1 shows a potential divider arrangement using a fixed resistor of resistance 4.0 kΩ
and a variable resistor of maximum resistance 20 kΩ with a slide contact connected to
terminal S.

20 kΩ S

12 V Y

4.0 kΩ

Fig. 3.1

The e.m.f. of the battery is 12 V and it has negligible internal resistance. It is possible to
obtain different continuously-variable ranges by selecting, as the output, particular pairs
of terminals from S, X, Y and Z.

(a) (i) Calculate the voltage range obtainable between the terminals S and X.

voltage range = .................V to................... V [2]

(ii) Hence, or otherwise, calculate the voltage range between the terminals S and Z.

voltage range = .................V to................... V [1]

[Turn over
10

(b) The slide contact S is set at the mid-point of the 20 kΩ resistance track. A voltmeter
of resistance of 10 kΩ is then connected between S and Y. Calculate the reading on
the voltmeter.

voltmeter reading = ........................................ V [3]

(c) The variable resistor in Fig. 3.1 is replaced by a thermistor T, as shown in Fig. 3.2.
At room temperature, the resistance of the thermistor is 12 kΩ. When it is placed in
hot liquid, its resistance falls to 2.0 kΩ.

12 V Y

4.0 kΩ

Fig. 3.2
11

(i) On Fig. 3.3, sketch the temperature characteristic of the thermistor.

resistance / Ω

0 temperature / °C

Fig. 3.3

[1]

(ii) Using the band theory, explain the variation of the thermistor’s resistance with
temperature.

..................................................................................................................................

..................................................................................................................................

..................................................................................................................................

..................................................................................................................................

..................................................................................................................................

..................................................................................................................................

..................................................................................................................................

..................................................................................................................................

..................................................................................................................................

........................................................................................................................... [3]

[Turn over
12

4 Fig. 4.1 shows a simple iron-cored transformer. The primary coil is connected to a
240 V r.m.s. 50 Hz power supply and the output voltage across the 6.0 Ω resistor in the
secondary coil is 12 V r.m.s.

primary secondary
240 V coil coil 6.0 Ω

Fig. 4.1

(a) (i) State Faraday’s law of electromagnetic induction.

..................................................................................................................................

..................................................................................................................................

........................................................................................................................... [1]

(ii) Use Faraday’s law to explain why a transformer will operate for an alternating
input voltage but not for a direct voltage.

..................................................................................................................................

..................................................................................................................................

..................................................................................................................................

..................................................................................................................................

........................................................................................................................... [2]

(b) (i) Explain what is meant by root-mean-square value of an alternating voltage.

..................................................................................................................................

..................................................................................................................................

........................................................................................................................... [1]
13

(ii) Calculate the peak output voltage.

peak output voltage = ........................................ V [1]

(iii) Calculate the mean power dissipated in the 6.0 Ω resistor.

mean power = ........................................ W [2]

(iv) On Fig. 4.2, sketch a graph to show the variation with time t of the power P
dissipated in the 6.0 Ω resistor over one period.

P/W

0 t/s

Fig. 4.2

[2]

[Turn over
14

Section B

Answer two questions from this section.

5 (a) The value of the gravitational potential φ at a point in the Earth’s field is given by the
equation

GM
φ=−
r

where M is the mass of the Earth and r is the distance of the point from the centre of
the Earth such that r is greater than the radius of the Earth R E .

(i) Define gravitational potential at a point.

..................................................................................................................................

........................................................................................................................... [1]

(ii) Explain why the potential has a negative value.

..................................................................................................................................

..................................................................................................................................

........................................................................................................................... [1]

(b) Fig. 5.1 shows the equipotential lines for Earth, where point A is at a potential of
− 4.0 × 10 7 J kg−1 and points B and C are at a potential of − 5.0 × 10 7 J kg−1.

− 4.0 × 10 7 J kg−1

Earth − 5.0 × 10 7 J kg−1

B
C

Fig. 5.1

(i) On Fig. 5.1, draw the equipotential line for the gravitational potential of
− 4.5 × 10 7 J kg−1. [1]
15

(ii) Calculate the work done by the gravitational field in bringing a body of mass
3000 kg from A to B.

work done = ........................................ J [2]

(iii) The work done by the gravitational force in bringing the mass from B to C along
the equipotential line is zero. Explain why this is so.

..................................................................................................................................

..................................................................................................................................

........................................................................................................................... [1]

(c) (i) Show that a body projected from the Earth’s surface (assumed stationary) with a
speed equal to or greater than the escape speed 2gR E will never return. State
any assumption(s) made in your workings for this result to be valid.

[3]

[Turn over
16

(ii) Information related to the Earth and the Sun is given below.

mass of Sun
= 3.3 × 10 5
mass of Earth

radius of Sun
= 110
radius of Earth

Given that the escape speed from the Earth is 1.1× 10 4 m s−1, calculate the
escape speed from the Sun.

escape speed = ........................................ m s–1 [2]

(iii) The surface temperature of the Sun is about 6000 K and hydrogen is the most
abundant element in the Sun’s atmosphere. Explain why this is so by means of
suitable calculations, assuming that hydrogen is an ideal gas.

..................................................................................................................................

..................................................................................................................................

........................................................................................................................... [2]
17

(d) Fig. 5.2 shows the way in which the gravitational potential energy of a body of mass
m depends on r.

energy
Y

RE R D
r

C
potential energy
tangent B

Fig. 5.2

(i) What does the gradient of the tangent to the curve at r = R E represent?

..................................................................................................................................

........................................................................................................................... [1]

(ii) The body is projected vertically upwards from the Earth’s surface. At a certain
distance R from the centre of the Earth, the total energy of the body may be
represented by a point on the line XY. Five points, A, B, C, D, E have been
marked on this line.

Explain clearly which point(s) could represent the total energy of the body

1. if it were momentarily at rest at the top of its trajectory,

..................................................................................................................................

..................................................................................................................................

........................................................................................................................... [2]

2. if it were falling towards the Earth,

..................................................................................................................................

..................................................................................................................................

........................................................................................................................... [2]

[Turn over
18

3. if it were moving away form the Earth, with sufficient energy to reach an
infinite distance?

..................................................................................................................................

..................................................................................................................................

........................................................................................................................... [2]
19

6 (a) State what is meant by saying that a temperature is on an absolute scale.

.......................................................................................................................................

................................................................................................................................. [1]

(b) Explain what is meant by

(i) an ideal gas,

..................................................................................................................................

........................................................................................................................... [1]

(ii) absolute zero on the Kelvin scale,

..................................................................................................................................

........................................................................................................................... [1]

(iii) the internal energy of a gas.

..................................................................................................................................

..................................................................................................................................

........................................................................................................................... [2]

(c) A car tyre has a fixed internal volume of 0.0160 m3. On a day when the temperature
is 27°C, the pressure in the tyre has to be increased from 2.76 × 10 5 Pa to
3.91× 10 5 Pa.

(i) Assuming that the air is an ideal gas, calculate the amount of air which has to be
supplied at constant temperature.

amount of air = ........................................ mol [3]

[Turn over
20

(ii) A portable supply of air used to inflate tyres has a volume of 0.0117 m3 and is
filled with air at a pressure of 1.165 × 10 6 Pa. Show that, at 27°C, there is more
than enough air in it to supply four tyres, as in (c)(i), without the pressure falling
below 4.00 × 10 5 Pa.

[3]

(iii) Show that the internal energy of a molecule of air at a temperature of 27°C is
6.21× 10 −21 J. Assume that the air behaves as a monatomic ideal gas.

[2]

(iv) Hence, calculate the internal energy of one mole of the air at a temperature of
27°C.

internal energy = ........................................ J [1]


21

(d) In order to study the sudden compression of a gas, some dry air is enclosed in a
cylinder fitted with a piston, as shown in Fig. 6.1.

piston
cylinder

Fig. 6.1

The mass of air in the cylinder is constant. The material of the cylinder and the piston
is an insulator so that no thermal energy enters or leaves the air.

The volume and pressure of air are measured. The piston is then moved suddenly to
compress the air and the new volume and pressure are measured.

The variation with volume V of the pressure p of the air is shown in Fig. 6.2.

p / 105 Pa

4.0 B

3.0

2.0

1.0

A
0 1.0 2.0 3.0 4.0 5.0 V / 10−3 m3

Fig. 6.2

It may be assumed that the dry air behaves as an ideal gas.

(i) By considering the pressure and volume of the dry air at points A and B, and
using the equation of state for an ideal gas, show that the temperature of the air
increases when the air is compressed.

[3]

[Turn over
22

(ii) The dry air then goes through two more processes.

Process 1: The gas is cooled while keeping the piston at the same position.

Process 2: The gas then expands, while kept at constant temperature, to return
to its original state.

On Fig. 6.2, draw and label the p-V graphs of the two processes described
above. [3]
23

7 (a) Explain how the photoelectric effect provides evidence for a particulate nature of
electromagnetic radiation.

.......................................................................................................................................

.......................................................................................................................................

.......................................................................................................................................

.......................................................................................................................................

.......................................................................................................................................

.......................................................................................................................................

.......................................................................................................................................

.......................................................................................................................................

.......................................................................................................................................

................................................................................................................................. [5]

(b) Two metal plates X and Y are contained in an evacuated container and are
connected as shown in Fig 7.1. Metal plate X is then illuminated with monochromatic
light.
incident monochromatic light

X Y
A

O F

E G

Fig. 7.1

[Turn over
24

Graph A shown in Fig 7.2 depicts the relationship between the voltmeter reading and
the ammeter reading.

I/A

P Q
A

B
C

− V1 p.d. / V

Fig. 7.2

(i) In order to obtain the part PQ on graph A, the sliding contact O would have to be
shifted. Discuss and explain the position of O to obtain part PQ of graph A.

..................................................................................................................................

..................................................................................................................................

..................................................................................................................................

..................................................................................................................................

..................................................................................................................................

........................................................................................................................... [3]

(ii) Given that the work function of X is 1.3 eV and the wavelength of the light is
550 nm, calculate the value of the stopping potential V1 .

V1 = ........................................ V [2]
25

(iii) Discuss the changes that would have been made in the experiment to produce
graphs B and C if the metal plate X used is the same.

..................................................................................................................................

..................................................................................................................................

..................................................................................................................................

..................................................................................................................................

..................................................................................................................................

..................................................................................................................................

........................................................................................................................... [3]

(c) (i) Explain the term stimulated emission in the production of laser.

..................................................................................................................................

..................................................................................................................................

........................................................................................................................... [2]

(ii) A 4.0 W laser emits a beam of wavelength 633 nm.

Calculate

1. the energy of each photon,

energy = ........................................ J [1]

2. the number of photons emitted in 0.05 s.

number of photons = ........................................ [1]

[Turn over
26

(d) The simplest intrinsic semiconductor that can be used to fabricate a diode is Silicon
(Si). Each silicon atom has four electrons in its outermost shell.

Gp III Gp IV Gp V Gp VI Gp VII
boron carbon nitrogen oxygen flourine

11
5
B 12
6
C 14
7
N 16
8
O 19
9
F
aluminium silicon phosphorus sulfur chlorine

27
13
Al 28
14
Si 31
15
P 32
16
S 35
17
Cl
zinc gallium germanium arsenic selenium bromine

65
30
Zn 70
31
Ga 72
32
Ge 75
33
As 79
34
Se 80
35
Br
cadium indium tin antimony tellurium iodine

112
48
Cd 115
49
In 119
50
Sn 122
51
Sb 128
52
Te 127
53
I

Fig. 7.3

(i) From the extract of a periodic table provided in Fig. 7.3, suggest an element that
can be used to dope silicon to obtain the n-type extrinsic semiconductor.

element = ........................................ [1]

(ii) Using the band theory, explain how doping changes the conductivity of silicon for
n-type semiconductor.

..................................................................................................................................

..................................................................................................................................

..................................................................................................................................

..................................................................................................................................

..................................................................................................................................

........................................................................................................................... [2]

End of paper
1

Answers to JC2 Preliminary Examination Paper 1 (H2 Physics)

1 B 9 C 17 B 25 A 33 B
2 D 10 A 18 D 26 D 34 B
3 D 11 D 19 D 27 D 35 A
4 B 12 B 20 D 28 C 36 C
5 D 13 C 21 C 29 C 37 B
6 C 14 D 22 D 30 D 38 D
7 B 15 D 23 A 31 C 39 C
8 D 16 A 24 B 32 D 40 D

Suggested Solutions:

V2
1 P=
R
ΔP ⎛ ΔV ΔR ⎞
× 100% = ⎜ 2 + ⎟ × 100%
P ⎝ V R ⎠
⎛ 0.01 ⎞
= ⎜2 + 0.05 ⎟ × 100%
⎝ 3.20 ⎠
≈ 5.6%

Answer: B

2 Considering helicopter accelerating upwards,


v 2 = 2as
v = 2(2.0 )(100 )
v = 20 m s−1

Considering pack accelerating downwards,


v ' 2 = u ' 2 +2a' s '
v ' 2 = (− 20 ) + 2(10 )(100 )
2

v ' ≈ 49 m s−1

Answer: D

3 The vertical acceleration of the stone is the same at both X and Y as the stone’s
resultant force (weight of stone) at both X and Y is the same.

. Answer: D
a
4 T − f = ma
T − 0.25(5000 ) = 1500(1.0 )
T
T = 2750 N
f = 0.25(5000) N
Trailer
Answer: B
2

v2
5 v1

M 3M
4M
v1
By conservation of momentum, 0 = −Mv 1 + 3Mv 2 ⇒ v 2 =
3
1 1
Mv 1 + (3M )v 2 = E
2 2

2 2
1 1 ⎛v 2 ⎞
Mv 1 + (3M )⎜ 1 ⎟ = E
2

2 2 ⎜ 9 ⎟
⎝ ⎠
2 2
Mv 1 = E
3
1 2 3
Mv 1 = E
2 4

Answer: D

6 In equilibrium, the vector arrows form a closed triangle, either clockwise or anticlockwise.

Answer: C

7 ma = m(0.20g)

mg
T

T = (mg )2 + (ma )2 = 0.50 g 2 + (0.20g ) ≈ 0.51g


2

Answer: B

N F
8 F − mg sin θ = ma
F = mg sin θ + ma

Since P = Fv ,
P = mgv sin θ + mav
θ W

Answer: D
3

9 For mass A:
1
mg (2h ) = mu 2
2
2
u = 4gh
For mass B:
(2m )gh = 1 (2m )v 2
2
2
v = 2gh
1
v 2 = u2
2
1
v= u
2

Answer: C

10 On level road, N = mg
On banked road,
v2
N ' sin θ = m
r
N ' cos θ = mg
v2
Therefore, tan θ =
rg
θ ≈ 39.2°
N '−N
fractional change =
N
1
= −1
cos 39.2°
≈ 0.29

Answer: A

2
GMm ⎛ 2π ⎞
11 2
= mR ⎜ ⎟
R ⎝T ⎠
T 2 ∝ R3
Since T is constant, R is constant.

Answer: D

GM
12 g =
R2
4πGR 3 ρ
g=
3R 2
4πGRρ
g=
3

Answer: B
4

GMm mv 2
13 =
r2 r
GM
v=
r
Hence, for the 100 kg satellite to stay in the same orbit as that of the 20 kg satellite, the
influencing factors on its required speed v are G, the mass M of the Earth, and its
distance r from the centre of the Earth, which is independent of the satellite’s mass.

Answer: C

2
2 x
14 v = ω x 0 − 0
4
3 3
v= ωx 0 = v0
2 2

Answer: D

1 1
mω 2 x 0 = m (2πf ) x 0 and thus KEmax. depends on the frequency of the
2 2 2
15 A: KE max . =
2 2
oscillation.

B: True.

C: v max = ωx 0 and thus maximum velocity is directly proportional to amplitude.

D: The period of oscillation is independent of the amplitude of the oscillation.

Answer: D

16 Internal energy of a system can be increased (without heating) by doing work on the
system.
Internal energy of an ideal gas depends only on its temperature.
The internal energy of a system may increase because of an increase in potential
energy, meaning that temperature will remain constant.
Two systems that have the same internal energy may have a different mass, hence be
at different temperatures.

Answer: A

17 Latent heat change occurs when temperature is constant (changing of state). Assuming
that the electric heater power is constant, the longer the time taken to change the state
(region S than region Q) means more energy is required. Hence specific latent heat of
vaporisation is larger.

Answer: B
5

18 Using pV = nRT , product of pV is proportional to T for fixed mass of gas.


pV value at J = 1 x 10–5 J
pV value at K = 7 x 10–5 J
pV value at L = 12 x 10–5 J
pV value at M = 8 x 10–5 J
.
Value is largest at point L, followed by M, K then J (in descending order).

Answer: D

Power
19 Intensity =
Surface Area
Intensity ∝ (Amplitude )
2

Power
Therefore, ∝ (Amplitude ) 2
Surface Area
Amplitude 2 E/S
2
=
⎛ Amplitude ⎞ E ' / 2S
⎜ 2 ⎟⎠

2E E
E' = =
4 2

Answer: D

20 Particles in a stationary wave have varying amplitude due to their fixed positions
between two nodes.

Answer: D

λ
21 = 20 cm (open and close end air column)
4
λ = 80 cm

Answer: C

22 Between X and Y, there are 3 compressions and 3 rarefactions.


2.5λ = 100
λ = 40 m

Since v = fλ ,
v = 100 × 40 = 4000 m s−1

Answer: D

23 Answer: A
6

ΔV
24 E =
Δx
At regions around 0.040 m,
21 − 15
E = = 300 V m−1
0.050 − 0.030

Electric field lines are directed from a higher potential region to a lower potential region.
Hence, the electric field is 300 V m−1 towards P.

Answer: B

ρl ρl 1
25 R = = 2
∴R ∝
A ⎛d ⎞ d2
π⎜ ⎟
⎝2⎠
2
RP ⎛ 2 ⎞ 4
=⎜ ⎟ =
RQ ⎝ 1 ⎠ 1
IP 1
∴ =
IQ 4
1
Hence, the fraction of total current through is or 0.20.
5

Answer: A

26
E1 = 6.0 V, r1 = 2.0 Ω

I
X Y

E2 = 4.0 V, r1 = 8.0 Ω

Net e.m.f. in circuit = 6.0 – 4.0 = 2.0 V


2. 0
Current, I = = 0.20 A
2 .0 + 8 .0

Therefore, terminal p.d. VXY = E1 – Ir1


= 6.0 – (0.20)(2.0)
= 5.6 V
Alternatively,
Terminal p.d. VXY = E2 + Ir2
= 4.0 + (0.20)(8.0) = 5.6 V

Answer: D
7

27 Let R1 be the resistance of each bulb.


R1
When three bulbs are working, their effective resistance in circuit, R =
3
R1
When filament of one bulb breaks, R =
2
Since effective resistance of circuit increases, current (ammeter reading) decreases.
With parallel circuit, the p.d. across each bulb is the same and hence, their brightness
remain unchanged.

Answer: D

28 Considering the circuit between Y and Z,

1 1 1
+ =
2R 5.0 2.5
R = 2.5 Ω

Considering the circuit between X and Y,


1 1 1
= +
R XY R R+5
1 1 1
= +
R XY 2.5 2.5 + 5
R XY = 1.9 Ω

Answer: C

29
conventional FE
current

x
O

FB

By Fleming’s left hand rule, the direction of magnetic force FB is in the direction shown
in the diagram above. In order for the electron to pass through undeflected, the direction
of the electric force, FE is in the opposite direction of FB. Hence, the field must be acting
as shown in option C.

Answer: C

30 The direction of B-field of a solenoid is along the axis of the solenoid. Since the electron
is moving along the axis, the direction of the current is along the axis. Hence, there is no
electromagnetic force on the electron.

Answer: D
8

dΦ dB
31 E = = NA cos 0°
dt dt
dB T
Maximum E is when is maximum, which is when t = .
dt 2

Answer: C

⎛ 3.0 ⎞
32 E = Blv = 2.0 × 10 −5 (20 )⎜ ⎟ = 6.0 mV
⎝ 0.20 ⎠

Answer: D

V0
33 For (a), Vrms =
2
For (b), Vrms ' = V0
2
V0
Pa 1
= 2R2 =
Pb V0 2
R

Answer: B

I0 2
34 I rms = = = 2 A
2 2
I dc = 2 A

Answer: B

hc
35 KE = −φ
λ
= 0.99979 eV
≈ 1.00 eV

Answer: A

36 As accelerating voltage increases, the minimum wavelength in the continuous spectrum


will be reduced. Therefore, λ1 (min. wavelength) will be reduced. λ2 which represents
the k-line characteristic spectra will remain the same.

Answer: C

37 Silicon which is mainly used to manufacture semiconductors is a group four atom. With
one valence electron missing from its outermost shell, a hole arises.

Answer: B

38 Condition for stimulated emission to occur is for photons to have the same energy as the
difference in energy levels during the transition.

Answer: D
9

ln 2
39 λ =
32
ln 2
N − ×16
= e 32 ≈ 0.71
N0

Answer: C

40 By conservation of charge and mass numbers,


14 4 17 1
7 N + 2 He → 8 O + 1 H

Therefore, one of the products must be a proton.

Answer: D
1

Answers to JC2 Preliminary Examination Paper 2 (H2 Physics)

Suggested Solutions:

No. Solution Remarks


1(a) Velocity is the rate of change of displacement. [1]
Acceleration is the rate of change of velocity. [1]

1(b)(i) 6.0 s [1]

1
(6.0)(20 ) + 1 (6.0 + 2.0)(10 )
1(b)(ii)
Displacement = −
2 2
= −20 m [1]

1(b)(iii) [2]
s/m

t/s
2.0 4.0 6.0 8.0 10.0 12.0

2(a) ( )
Upthrust = 800 4.50 × 10 −4 (9.81) = 3.5316 ≈ 3.53 N [1] for correct
substitution

2(b) At equilibrium, [1] for correct


kx + U = mg substitution
mg − U
k=
x

k=
( )
8000 4.50 × 10 −4 (9.81) − 3.5316
≈ 312 N m−1 [1] for correct
0.102 answer

2(c)(i) When the sphere is lifted out of the liquid, the volume of [1]
liquid displaced is reduced. This causes the upthrust acting
on sphere to decrease. To maintain equilibrium, the tension
will increase and the string breaks when the tension
exceeds the maximum allowable value.
2

2(c)(ii) At breaking point, [1] for correct


T + U = mg substitution
(
32.0 + 800(V )(9.81) = 8000 4.50 × 10 −4 (9.81)) [1] for correct
−4 3
V ≈ 4.23 × 10 m answer

3(a)
[2] for stationary
wave drawn
around rim

3(b) A stationary wave is formed by the superposition of two [2] for explaining
progressive waves of the same type with equal speed, formation of
frequency, wavelength and amplitude travelling in opposite stationary waves
directions. The waveform does not advance and there is no
net energy transfer.
The product fλ in a stationary wave refers to the speed of [1] for explaining
the two progressive waves that superpose to form the significance of fλ
stationary wave.

3(c) Diffraction is the spreading of waves through an aperture or [2]


around an obstacle.
It is observable when the wavelength of the wave is of the
same order of magnitude as the dimensions of the aperture
or obstacle.

3(d)(i) Slit separation [1] for slit


1× 10 −3 separation
d= 2
= 2.5 × 10 −6 m
4.00 × 10


Since sin θ = , and the maximum possible angle is 90°,
d

<1
d
n < 4.24
Therefore, n = 4 (always round down) [1] for max. order

3(d)(ii) 4 [1]
1. The higher the order, the greater the separation between the
fringes.

3(d)(ii) For wavelength 589.0 nm,


2. (2.5 × 10 −6 ) sin θ = 4(589.0 × 10 −9 ) [1] for calculation
of angle for both
θ ≈ 70.459°
wavelengths
3

For wavelength 589.6 nm, [1] for calculating


(2.5 × 10 −6 ) sin θ ' = 4(589.6 ×10 −9 ) the difference
between the 2
θ ' ≈ 70.624°
angles and correct
∴ angular separation = 70.624° − 70.459° = 0.165° conclusion

This is greater than minimum required, therefore students


will be able to observe the wavelengths as separate image.

4(a) A field of force is a region of space within which a force is [1]


experienced by a body placed in the field.

4(b)(i)
electric field
direction

velocity [1] labelled


diagram

electron
path

The electron moves in a projectile motion towards the higher [1]


potential region.

4(b)(ii)
X X X X
− velocity magnetic
X X X X field into
the paper
X X X X [1] labelled
diagram
X X X X
electron
path

The electron moves in a circular motion. [1]

4(c)(i) As current flows through the conductor, the electrons


experience a downward magnetic force, according to [1]
Fleming’s left hand rule. Negative charge accumulates on [1]
SR, leaving a net negative charge on SR and a net positive
charge on PQ. The voltage is set up across the sides. This [1]
voltage eventually stops further vertical migration of
electrons.
4

4(c)(ii) Since current is not deviated in the magnetic and electric


fields,
FE = FB
qE = Bqv [1]
V
q = Bqv
x
V = Bvx
[1]
V = (1.0)(2.52 × 10 −5 )(5 × 10 −3 ) = 1.26 × 10 −7 V

5(a)(i) Binding energy of a nucleus is the minimum amount of [2]


energy required to break the nucleus into its constituent
particles.
OR
Binding energy of a nucleus is the minimum amount of
energy released when a nucleus is formed from its
constituent particles.

5(a)(ii) Nuclear fission is the disintegration of a heavy nucleus into [2]


lighter, more stable nuclei, with the release of energy.

5(b)(i) Energy released = 90(8.9) + 143(9.0) − 236(8.7) [1] for correct


= 34.8 MeV substitution
[1] for correct
answer

5(b)(ii) The role of neutrons is to initiate more nuclear fission [1]


reactions by bombarding other 236
92 U nuclei such that a chain
reaction results.

Neutrons have no binding energy. [1]

5(b)(iii) An increase in binding energy means that the product nuclei [1]
have higher binding energy per nucleon compared to the
reactant nuclei. The products are more stable than the
reactant.

The energy released when forming the products is more [1]


than the energy required when breaking the reactants into
constituent particles.

6(a) An incoming electron removed an atomic electron from the [1] for
innermost shell, K-shell. understanding that
An electron from the L-shell may fill the vacancy, and in the innermost electron
process emits a photon. The photon emitted has an energy is removed
that corresponds to the wavelength of the K α -line.
[1] for clear
explanation
5

6(b)(i) For the energy of the photon for the K α -line,


E α = E 2 − E1 [1] for calculating
the energy
k (Z − 1) ⎡ k (Z − 1)2 ⎤
2
difference between
Eα = − − ⎢− ⎥
22 ⎣⎢ 12 ⎦⎥ E 2 and E1 that
gives rise to the
3k (Z − 1)
2
hf = energy for the K α -
4
line
3k (Z − 1)
2
f =
4h
f = C (Z − 1)
[1] for correct
3k manipulation of
where C = and is a constant.
4h equation

6(b)(ii) 1
f / 10 9 Hz 2
1.7 [1] for best-fit line
drawn

1.6

1.5

1.4

1.3

1.2

1.1

1.0

0.9 Z
21 22 23 24 25 26 27 28 29 30 31 32 33

6(b)(iii) Since f = C (Z − 1) ,
f = CZ − C
[1] for stating the
Since the equation is of the form Y = mX + c , the constant correct methods
C can be obtained either from the gradient of the graph, or using the equation
from the vertical intercept of the graph.
6

6(b)(iv) From the graph, [1] for correct


gradient ≈ 5.0 × 10 7 ( ± 0.2 × 10 )7 calculation for
gradient or C
1
Therefore, the constant C is 5.0 × 10 7 Hz 2 . [1] for correct units
for C

6(b)(v) Since f = C (Z − 1) ,
f = 5.0 × 10 7 (29 − 1) [1] for correct
f = 1.4 × 10 9 substitution
c
Since λ = ,
f
3.0 × 10 8
λ=
(
1.4 × 10 9
2
) [1] for correct
−10
λ ≈ 1.5 × 10 m calculated value

6(c)(i)
ray 1

ray 2

θ θ

d θ
x

Using the Pythagoras’ theorem, [1] for identifying


x = d sinθ path difference

Since the difference in the path between ray 1 and ray 2 is [1] for correct
2x, the path difference is 2d sin θ . calculation of path
difference

6(c)(ii) The first two peaks are the 1st order maxima and the next
two peaks are the 2nd order maxima for the two wavelengths
of the X-ray beam. [1] for correct
calculation of the
Using 2d sin θ = mλ , shorter wavelength

2 × 0.94 × 10 −9 × sin 0.8° = 1× λ1 [1] for correct


calculation of the
λ1 ≈ 26 pm
longer wavelength

2 × 0.94 × 10 −9 × sin 2.4° = 2 × λ 2 Subtract 1 mark for


λ2 ≈ 39 pm wrong conversion
of m to pm.
7

6(c)(iii) Using the diffraction grating expression d sin θ = nλ , and


considering the 1st order maxima,

3000 × 10 −9 × sinθ = 1× 0.1× 10 −9 [1] for correct


θ ≈ 0.0019° calculation of angle
of diffraction using
The 1 order maxima is too close to the central bright fringe appropriate order
st

and so is unable to discriminate between different


wavelengths in the X-ray region. [1] for clear
explanation with
A grating with spacing approximately equal to the reference to the
wavelength will be desirable, but such gratings cannot be calculation made
constructed mechanically.

7 Generic Mark Scheme


A1 – Basic procedure [1]
A2 – Labelled diagram of a workable procedure [2]
B1 – Method of measurement of independent variable [1]
B2 – Method of measurement of dependent variable [2]
C1 – Keeping other variables constant [2]
C2 – Safety precautions or steps to ensure reliability. [2]
D1 – Detail marks [2]

Define the problem


To investigate how the intensity of the γ-ray beam varies as
the thickness of the material changes.

Variables
The dependent variable is the count rate recorded by
detector.
The independent variable is the thickness of the lead.

The intensity of the γ-ray beam can be measured by count


rate recorded by the detector.

Diagram

γ-ray source A2 labelled diagram


of workable
GM counter procedure

lead block

Procedure
(a) Set up the apparatus as shown. The source and GM C1
counter should be placed along the same line on opposite
sides of the lead block.

(b) Remove the source and measure the count rate of the D1
background radiation with the GM counter.
8

(c) Remove the block to measure the count rate Co when


there is no attenuation through the block.

(d) Measure the thickness of the lead block, d, using a pair B1


of vernier calipers.

(e) Replace the source and then record the count rate, C, of D1
the γ-ray after it passes through the lead block.

(f) Variation of d: use lead blocks with increasing thickness. A1


Measure the count rate C again for the different thickness of
lead.

(g) Repeat steps (d) and (e) and take the average of the C2
count rates for different thickness. This is to account for the
random nature of the γ-radiation.

(h) Plot a graph of C against d. Using the graph, identify the D1


thickness, d, where the value of C is reduced to half.

Control of other variables


The distance from the source to the GM counter is kept C1
constant throughout the experiment.

Use a source that has constant activity (~long half life) Use
the same source specimen throughout the experiment to
ensure that the emission of γ-ray is the constant.

Steps to ensure reliability or safety/further details


Handle the source with tongs C2
Do not point the source at anyone
Replace the source in a lead box immediately after use
Range of values of thickness of lead ~1 to 10 cm

Alternatives (Not required in answer)


Use concrete as the absorber instead of lead
Use other detectors of radiation: scintillation counter, spark
chamber
1

Answers to JC2 Mid-Year Examination Paper 3 (H2 Physics)

Suggested Solutions:

No. Solution Remarks


1(a) The rate of change of momentum of a body is directly [1]
proportional to the resultant force that acts on it and has the
same direction as the resultant force.

(b)(i) [1] correct free


T N body diagrams
T

15

1.0g
4.0gsin45°
4.0gcos45°

4.0g
T – 1.0g = 1.0a ------- (1) [1] correct
4.0g(sin45°) – T – 15 = 4.0a ------- (2) equations

Solving (1) and (2), [1] each for correct


T ≈ 10.4 N value of T and a
a ≈ 0.587 m s−2

(ii) Loss in GPE = Gain in KE + WD against friction


1 1 ⎛ 1.0 ⎞ [1] for correct
4.0g (1.0 ) = (4.0 )v 2 − (4.0 )(0.5 ) + ⎜ ⎟(15 )
2
substitution
2 2 ⎝ sin 45° ⎠
v ≈ 3.04358 m s−1 [1] for correct value
v ≈ 3.04 m s−1 of v

(iii) The total momentum of a system is constant, provided no [1]


external resultant force acts on it.

(iv)
1.6 m s−1 v1 0.7 m s−1

A C A C
[1] for correct
By conservation of momentum, substitution
4.0(1.6) = 4.0v1 + 6.0(0.7) [1] for correct value
v1 = 0.55 m s−1 of v
2

(v) Velocity of approach = 1.6 – 0 = 1.6 m s−1 [1] correct


Velocity of separation = 0.7 – 0.55 = 0.15 m s−1 calculation of
speed of approach
Since velocity of approach is not equal to velocity of and speed of
separation, collision is not elastic. separation
[1] for correct
conclusion

2(a)(i) v max = ωx 0 [1] for correct


substitution and
⎛ 2π ⎞
=⎜ ⎟x0 answer
⎝T ⎠
⎛ 2π ⎞
=⎜ ⎟0.15
⎝ 2 .0 ⎠
≈ 0.47 m s−1

(ii) a max = ω 2 x 0 [1] for correct


2 substitution and
⎛ 2π ⎞ answer
=⎜ ⎟ x0
⎝T ⎠
2
⎛ 2π ⎞
=⎜ ⎟ 0.15
⎝ 2.0 ⎠
≈ 1.5 m s−2

(iii) Using x = x 0 cos ωt [1] for correct use


of equation
⎛ ⎛ 2π ⎞ ⎞
x = 0.15 cos⎜⎜ ⎜ ⎟(0.10 )⎟⎟
⎝ ⎝ 2 .0 ⎠ ⎠ [1] for correct
x ≈ 0.14 m answer

(b)(i) 1 [1] for using


KE = mv 2 appropriate
2
2 equation
1
KE = m⎛⎜ ω x 0 − x 2 ⎞⎟
2

2 ⎝ ⎠ [1] for correct


1 2
(
KE = mω 2 x 0 − x 2
2
) derivation

At x = 0, we have max. KE which is equivalent to the total


1 2
energy since PE = 0. Hence, total energy = mω 2 x 0 .
2

(ii) [1] for correct each


1 2
energy mω 2 x 0 graph
2
ET
Ep

Ek
displacement, x
- xo xo
3

3(a) (i) When the slide contact S is moved all the way up to the top [1]
of the 20 kΩ resistance, Vxs = 0 V.

When the slide contact S is moved all the way down, Vxs can
be found using the potential divider principle:

⎛ 20 ⎞ [1]
Vxs = ⎜ ⎟12 = 10 V
⎝ 20 + 4 ⎠

Hence, the voltage range of VXS is 0 V to 10 V.

3(a)(ii) When the slide contact S is moved all the way up to the top
of the 20 kΩ resistance,

Vsz = 12 – 0 = 12 V

When the slide contact S is moved all the way down,

Vsz = 12 – 10 = 2V [1]

Hence the voltage range of VSZ is 2 V to 12 V.

3(b) When the slide contact is placed at midpoint of the 20 kΩ


track, R = 10 kΩ.

1 1 1
= +
R SY 10 10
[1]
R SY = 5 kΩ

Using potential divider principle,

5
VSY = (12) ≈ 3.2 V [1]
10 + 5 + 4

The voltmeter reading = VSY


[1]
≈ 3.2 V

3(c)(i) [1]
R/Ω

0 t/°C
4

3(c)(ii) The thermistor is a semiconductor and the gap between the [1]
conduction band and the valence band is small (~1.0 eV).
When the thermistor is placed in a hot liquid, the thermal
energy gained from the hot liquid is sufficient to cause the
electrons to move across the small band gap to become free [1]
electrons in the conduction band. At the same time, holes
are being formed in the valence bands. Both the free
electrons in the conduction band and holes in the valence
band can now conduct electricity. Thus, there is a large [1]
increase in the number of mobile charge carriers which
decreases the resistance of the thermistor.

3(c)(iii) The thermistor can be used as a temperature sensor in air- [1]


condition, oven, toaster, fridge etc. (regulate temperature)

4(a)(i) Faraday’s law of electromagnetic induction states that the [1]


induced e.m.f. is directly proportional to the rate of change
of magnetic flux linkage.

4(a)(ii) An alternating input voltage will result in a constantly [1]


changing magnetic flux linkage in the secondary coil. An
e.m.f. will be induced in the secondary coil.

A direct input voltage will result in a constant magnetic flux [1]


linkage in the secondary coil. No induced e.m.f. in the
secondary coil.

4(b)(i) Root-mean-square value of an alternating voltage is the [1]


steady direct voltage that, when applied across a given
resistance, will convert electrical energy to other forms of
energy at the same rate as the alternating voltage will.

4(b)(ii) Peak output voltage = 12 2 ≈ 17 V [1] for correct


answer

4(b)(iii) 2
Vr .m.s. 12 2 [1] for correct
Mean power = = = 24 W substitution
R 6 .0
[1] for correct
answer

4(b)(iv) [1] for correct


P/W shape of graph

48 [1] for correct max.


P and correct T

0 t/s
0.02
5

5(a)(i) Gravitational potential at a point is the work done in bringing [1]


unit mass from infinity to that point.

5(a)(ii) Gravitational force is attractive in nature and the external [1] for any
agent is acting against the gravitational force. Hence, the acceptable
work done by the external agent is negative as the explanation
displacement and the external force is opposite in direction.

5(b)(i) [1] for line drawn


nearer to the
A − 5.0 × 10 7 J kg−1
equipotential line

− 4.0 × 10 7 J kg−1

Earth
− 5.0 × 10 7 J kg−1

B
C

5(b)(ii) Work done by gravitational field [1] for correct


= m(φ A − φ B ) substitution
= 3000 × (− 4.0 + 5.0 ) × 10 7
[1] for correct
= 3.0 × 1010 J answer

5(b)(iii) Since the force and displacement vectors are always [1] for correct
perperdicular to each other, the work done is therefore zero. explanation
[zero mark if
mention B and C
are at same
potential]

5(c)(i) For an object of mass m to escape to infinity from the


Earth’s surface, by conservation of energy, assuming that [1] for correct
no energy is lost due to friction of air, assumption
loss in kinetic energy = gain in gravitational potential energy
1 2 1 2 ⎛ GMm ⎞
mv escape − mv ∞ = 0 − ⎜⎜ − ⎟⎟ [1] for correct use
2 2 ⎝ RE ⎠ of conservation of
1 2 GMm 1 2 energy and
mv escape = + mv ∞
2 RE 2 substitution

Assuming that the kinetic energy at infinity is zero or more,


1 2 GMm
mv escape ≥
2 RE
6

2 2GM
v escape ≥
RE
[1] for applying
gravitational field
Since gravitational field strength at the Earth’s surface is
strength at Earth’s
GM 2
surface
g= 2
, we have GM = gR E .
RE

Hence,
2
2 2gR E
v escape ≥
RE
v escape ≥ 2gR E [subtract one mark
for unclear
workings]
Therefore, a body projected from the Earth’s surface with a
speed equal to or greater than the escape speed 2gR E
will never return.

5(c)(ii) 2GM E
Since v escape from Earth ≥ , we have
RE

v escape from Earth M E RS


=
v escape from Sun M S RE

1.1× 10 4 110 [1] for correct


= manipulation and
v escape from Sun 3.3 × 10 5
substitution
[1] for correct
v escape from Sun ≈ 6.1× 10 5 m s–1 answer

5(c)(iii) Kinetic energy of an atom of mass m is given as


1 2 3
mv r .m.s. = kT
2 2
2 [1] for correct
1× 1.66 × 10 × v r .m.s. = 3 × 1.38 × 10 −23 × 6000
−27
calculation of r.m.s.
v r .m.s. ≈ 1.2 × 10 4 m s–1 speed

Since the r.m.s. speed of the hydrogen atom is less than the [1] for correct
escape speed, the concentration of hydrogen atoms is deduction
abundant on the Sun’s atmosphere.

5(d)(i) The gradient of the tangent at r = R E represents the [1] for correct
negative of the gravitational force acting on mass m. explanation

5(d)(ii) If the body is momentarily at rest, then its kinetic energy will [2] for correct
1. be zero. Hence, its total energy is entirely gravitational explanation
potential energy only. Therefore, the total energy is [no marks awarded
represented by point B. for stating correct
point without any
explanation]
7

5(d)(ii) If the body is falling towards Earth, its kinetic energy is non- [2] for correct
2. zero and so the total energy will be greater than the explanation
gravitational potential energy. However, as the body does [no marks awarded
not escape from Earth, its total energy is negative as it is still for stating correct
a bound system. Hence, the total energy is represented by point without any
point C. explanation]

5(d)(ii) If the body has sufficient energy to escape to infinity, the [2] for correct
3. kinetic energy must be equal or greater than the explanation
gravitational potential. The total energy can thus be equal to [no marks awarded
or more than zero, which is represented by point D or E. for stating correct
point without any
explanation]

6(a) It means that the absolute scale does not depend on the [1]
property of a particular substance.

(b)(i) An ideal gas is one that obeys the equation of state for an [1]
ideal gas, pV = nRT for all values of pressure, volume and
temperature.

(b)(ii) Absolute zero on Kelvin scale is the temperature at which all [1] minimal internal
substances have a minimal internal energy. energy
Do not accept ‘zero
internal energy’

(b)(iii) Internal energy of a gas is the sum of all the microscopic [1]
kinetic and potential energies of the molecules or atoms in [1]
the gas.

(c)(i) Δ pV = ΔnRT
Δ pV
Δn =
RT

=
( 3.91 − 2.76 ) × 105 × 0.0160 [1] finding Δp
8.31× (27 + 273.15) [1] substitution
= 0.737698 mol
[1] final ans
≈ 0.738 mol

(c)(ii) pV
no. of moles of air in supply of air =
RT
11.65 × 105 × 0.0117
= [1] finding amount
8.31× 300.15 of air in supply
= 5.464777 mol

No. of moles of gas needed for 4 tyres


= 4 × 0.737698 = 2.950792 mol [1] find amount of
Î there is sufficient air in the supply to fill up 4 tyres. gas for 4 tyres
8

Pressure in supply after filling 4 tyres [1] for showing


(5.464777 − 2.950792) × 8.31× 300.15 pressure does not
= fall below
0.0117
4.00×105 Pa
= 5.359 × 10 Pa > 4.00 × 105 Pa
5

(c)(iii) 3
U= KT
2
3
= × 1.38 × 10 − 23 × 300.15 [1] substitution
2
= 6.21× 10 −21 J [1] final ans

(c)(iv) 3
U= NKT
2
[1] for correct
= 6.21× 10−21 × 6.02 × 1023
substitution and
= 3740 J answer

(d)(i) Using pV = nRT , product of p and V at a point is


proportional to T. [1]

At pt. A, p AV A = 5 × 0.5 × 10 2 = 250 kg m2 s−2


[1]
At pt. B, pBVB = 1.2 × 4 × 10 2 = 480 kg m2 s−2

Hence temperature at B is higher; temperature of air [1]


increases when the air is compressed

(d)(ii) [1] correct shape of


lines
5
p / 10 Pa
[1] direction of
B
4.0 processes shown

[1] correct value at


3.0
p ~ 2.1 Pa for point

2.0

1.0

A
–3 3
0 1.0 2.0 3.0 4.0 5.0 V / 10 m

Fig. 6.2
9

7(a) 1. Only radiation of frequency above a threshold can cause [3] for the 3
photoemission. observations in
photoelectric effect
2. The electrons’ max. KE is dependent on the frequency of experiment
radiation but independent of intensity.
[2] for the
Particulate characteristics: particulate
In assuming that photons are particles, the frequency characteristic
constitutes the energy of each photon. Intensity will only
determine the number of photons per unit time, thus not
affecting its energy.

3. No time lag between arrival of photon and emission of


photoelectrons.
Particulate characteristics:
In Newtonian mechanics where we study about the
interaction of particles in a collision, the transfer of
momentum is instantaneous. This would lead to emission of
electrons being instantaneous after interaction with photons.

All the above shows that the interaction between a photon


and that of an electron is particulate as the collision of
particles in Newtonian terms is similar.

7(b)(i) The sliding contact O has to be in the region FG or the right


of F. (OR idea has to be communicated through other
means, drawing) [1] position of O

This is because PQ represents the saturation current which [1] current potential
depends on intensity. At the current position, the potential of of X and Y
X is higher than Y which will create a stopping potential that
would stop the electrons flow. [1] to explain Y to
be at a higher
In order for electrons to flow to constitute saturation current potential
PQ, O has to be shifted to the left of F to allow Y to be
higher potential than X.

7(b)(ii) hf = KE MAX + φ
hc
= eVS + φ
λ
(6.63 × 10 −34 )(3.0 × 10 8 ) [1] equation
−9
= (1.6 X 10 −19 )VS + (1.3 × 1.60 × 10 −19 )
550 × 10
[1] answer
VS ≈ 0.96 V

7(b)(iii) Graph B has a smaller maximum current and larger stopping [2] Graph B
potential than A. The larger stopping potential indicates that
an incident radiation of higher frequency OR incident
radiation of lower wavelength is used.

The lower maximum photocurrent is due to smaller intensity.

Graph C has a smaller maximum photocurrent but the same [1] Graph C
stopping potential as A. Since stopping potential is the
10

same, the frequency of incident radiation must be the same.


The smaller maximum current is due to a lower intensity of
light.

7(c)(i) An atom (or electron) in an excited state can interact with an [2]
incident resonant photon to emit another photon of the same
frequency, in phase and in the same direction of travel as
the incident photon.

7(c)(ii) Let E be energy of each photon.


1.
hc
E = hf =
λ
6.63 × 10−34 (3 × 108 )
E= = 3.142 × 10 −19 J [1] energy
633 × 10 −9
2.
Let thenumber of photons emittedbe N.
[1] number of
Pt 4.0 × 0.05
N= = −19
= 6.37 × 1017 photons
E 3.142 × 10

7(d)(i) Any group V element [1]

7(d)(ii) In a n – type semi-conductor, a small amount of an element [1] doping process


with five outer electrons, such as Arsenic, is added to semi-
conductor silicon. In silicon atoms doped with a few arsenic
atoms, the extra electron doesn’t fit into the crystal lattice
and so is free to move about. This type of semi-conductor is
called n – type because it is the electrons that seem to carry
the electric current.

In n – type semi-conductors, the impurity energy level is just [1] donor level
below the conduction band Electrons in this energy level
need only about 0.05 eV of energy to reach the conduction
band; hence this transition readily occur at room
temperature.
RAFFLES INSTITUTION
2010 Preliminary Examination

PHYSICS 9646 / 01
Higher 2
9745 / 01
Paper 1 Multiple Choice

24 September 2010
1 hour 15 minutes
Additional Materials: OMR form
Soft clean eraser
Soft pencil (type B or HB is recommended)

READ THESE INSTRUCTIONS FIRST

Do not open this booklet until you are told to do so.

Fill in your particulars on the OMR form.

There are forty questions in this paper. Answer all questions. For each question there
are four possible answers A, B, C and D. Choose the one you consider correct and
record your choice in soft pencil on the OMR form.

Read the instructions on the OMR form very carefully.

Each correct answer will score one mark. A mark will not be deducted for a wrong
answer.

Any rough working should be done in this booklet.

This booklet consists of 22 printed pages including the cover page.


2

Data

speed of light in free space, c = 3.00 x 108 m s−1

permeability of free space, μ0 = 4π x 10−7 H m−1

permittivity of free space, ε0 = 8.85 x 10−12 F m−1


(1 / (36 π)) x 10−9 F m−1

elementary charge, e = 1.60 x 10−19 C

the Planck constant, h = 6.63 x 10−34 J s

unified atomic mass constant, u = 1.66 x 10−27 kg

rest mass of electron, me = 9.11 x 10−31 kg

rest mass of proton, mp = 1.67 x 10−27 kg

molar gas constant, R = 8.31 J K−1 mol−1

the Avogadro constant, NA = 6.02 x 1023 mol−1

the Boltzmann constant, k = 1.38 x 10−23 J K−1

gravitational constant, G = 6.67 x 10−11 N m2 kg−2

acceleration of free fall, g = 9.81 m s−2


3

Formulae

uniformly accelerated motion, s = ut + 21 at 2

v2 = u 2 + 2as

work done on/by a gas, W = pΔV

hydrostatic pressure, p = ρgh

Gm
gravitational potential, φ = −
r

displacement of particle in s.h.m., x = x0 sin ω t

velocity of particle in s.h.m., v = v0 cos ω t

= ±ω (x 2
0 − x2 )
mean kinetic energy of a molecule 3
of an ideal gas E = kT
2

resistors in series, R = R1 + R2 + …

resistors in parallel, 1/R = 1/R1 + 1/R2 + …

Q
electric potential, V =
4πε 0 r

alternating current/voltage, x = x0 sin ω t

transmission coefficient, T = exp(−2kd)

8π 2 m ( U − E )
where k =
h2

radioactive decay, x = x0 exp (−λt)

0.693
decay constant, λ =
t1
2
4

1 The coefficient of viscosity, η , for a fluid is given by:

FL
η=
Av
where F is the external force on the fluid, v is the relative motion of the fluid layers, L and A
are the width and area of the fluid layer respectively. The base units for η are

A N s-1 m-2 C kg m s-1


B N s m-2 D kg m-1 s-1

2 Estimate the number of atoms in 1 cm3 of a solid.

A 1010 C 1030
B 1024 D 1040

3 At time t = 0 s, a ball was released from rest above a floor. In the velocity-time graph shown
below, at which time does the ball reach its maximum height after bouncing from the floor?

velocity v

B
0 time t
A C D
5

4 Consider a falling raindrop undergoing constant acceleration. Which pair of quantities would
yield a straight line graph when plotted to represent the motion of the raindrop?

A Velocity of the raindrop and its displacement.


B Displacement of the raindrop and its time in motion.
C Kinetic energy of the raindrop and its displacement.
D Kinetic energy of the raindrop and its time in motion.

5 Two particles of identical masses are initially projected towards each other on a smooth
surface with speeds u1 and u2 respectively. They collide elastically with each other, and their
directions and speeds after the collision are shown in the figure below.

u1 u2

Before collision

v1 v2

After collision

Which one of the following equations cannot be applied to the collision of this system?

A u1 - u2 = v 2 + v 1 C u12 – u22 = v12 + v22


B u1 + u 2 = v 2 – v 1 D u12 + u22 = v12 + v22

6 A movable notice-board of mass 2.0 kg is placed on a smooth floor. What is the initial
acceleration of the notice-board when a horizontal stream of water, travelling at speed
8.0 m s-1, strikes it at a rate of 1.0 kg s-1 for a duration of 50 s.

A 0.16 m s-2 C 4.2 m s-2


B 4.0 m s-2 D 8.0 m s-2
6

7 A pendulum bob is suspended in a bus of mass 3000 kg undergoing constant deceleration.


The pendulum makes an angle of 18° with the vertical. What is the deceleration of the bus?

direction of motion
18°

A 0.32 m s-2 C 3.2 m s-2


B 3.0 m s-2 D 9.3 m s-2

8 A clown on a unicycle accelerates to the left.

direction of motion

road

What is the direction of the resultant force due to the road acting on the wheel of the
unicycle?

A C

B D

9 A 1.6 kg block slides down a plane that is inclined at 25° with the horizontal, at a constant
speed of 2.0 m s-1. At what rate is the frictional force doing work on the block?

A -28 W C 13 W
7

B -13 W D 28 W
10 A 100 kg crate is pulled from rest across a floor with a constant force of 320 N. For the first
20.0 m, the floor is frictionless and for the next 10.0 m, a constant frictional force of 30.0 N
acts on the crate. What is the final speed of the crate?

A 8.00 m s-1 C 13.6 m s-1


B 8.37 m s-1 D 13.9 m s-1

11 A car travels on a curved track of radius 150 m. The track is banked at an angle of 15o. At
what speed must the car travel such that friction is not required for it to travel safely in the
circular path?

A 13 m s-1 C 28 m s-1
B 20 m s-1 D 38 m s-1

12 A roller coaster starts from rest on a hill-top. It accelerates along a frictionless track and
enters a loop-the-loop of radius 60 m as shown below.

roller coaster

loop-the-loop

H
H

m
60

In order for the roller coaster to just remain in contact with the track when it is at the top of the
loop-the-loop, the vertical height H between its starting point and the entrance of the loop-the-
loop must be

A 90 m C 150 m
8

B 120 m D 180 m
13 A rock is thrown vertically upward near the surface of Planet X with a velocity of 45 m s-1 and
it comes to an instantaneous rest 5.2 s later. If the same rock is now thrown up near the
surface of Planet Y with the same initial velocity as that on Planet X, at 5.2 s later it is still
moving upwards at a speed of 25 m s-1. If both planets do not have atmosphere, the ratio of
the gravitational field strength near the surface of Planet Y to that of Planet X is

A 0.25 C 0.44
B 0.38 D 0.62

14 Suppose a planet has radius R and mass M. An object of mass m is moved from the surface
of the planet to a height h above the surface, where the planet’s gravitational field is
negligible. What is the change in gravitational potential energy of the object?

A GMm C GMm

R+h R+h
B GMm D GMm

R R

15 A body in simple harmonic motion makes n complete oscillations in 1.0 min. What is the
angular frequency ω of this motion?
n nπ
A rad s−1 C rad s−1
60 30

B n rad s−1 D 2π n rad s−1


9

16 A mass of 2.0 kg is executing simple harmonic motion. The net force F acting on the mass
varies with displacement x as shown. What is the maximum speed of the mass?

A 1.0 m s−1 C 1.3 m s−1


B 1.4 m s−1 D 2.0 m s−1

17 Four different solids A, B, C and D of equal masses at 20°C are separately heated at the
same rate. Their melting points and specific heat capacities are as shown in the table below.
Which of these solids will start to melt first?
Liquid Melting point/ °C Specific heat capacity/ J kg−1 K−1
A 80 1200
B 100 800
C 150 600
D 300 250

18 The piston of a gas-tight syringe containing an ideal gas is pulled outwards quickly. Which of
the following changes is incorrect?

A The density of the ideal gas decreases.


B The pressure of the ideal gas decreases.
C The temperature of the ideal gas decreases.
D The root-mean-square speed of the atoms increases.
10

19 Which of the following statements about electromagnetic waves is not true?

A They can be polarised.


B They are transverse waves.
C They always travel at 3.0 × 108 ms-1 .
D They are diffracted when they pass through a small aperture.

20 Some fine sand particles are present in a long transparent tube. A speaker is placed at the
end of the tube, and the frequency of the sound emitted is varied until the fine sand settles
into a series of small heaps. The diagram below shows a section of the tube and some of the
heaps that were formed.

Which of the following statements is true?

A The air molecules are vibrating vertically.


B The wavelength of the sound is given by L.
C The air pressure where the heaps are is the lowest.
D The positions of the heaps show the positions of the displacement nodes.
11

21 A horizontal steel wire is fixed at one end and is kept under tension by means of weights
suspended over a pulley. The length of wire between the fixed end and the pulley is 1.0 m.
Magnets are placed near the centre of the wire, and an alternating voltage supply is
connected to the wire between the fixed end and the pulley. Standing waves are formed when
the voltage supply is turned on. Five antinodes are observed on the wire.

1.0 m

S
pulley

N
Fixed
end

weights

Given that the speed of the wave on the wire is 24 m s-1, what is the frequency of the voltage
supply?

A 48 Hz C 96 Hz
B 60 Hz D 120 Hz

22 In a diffraction grating experiment, the first order image of a 438 nm blue light occurred at an
angle of 16.2°. A second order coloured light was observed at 47.4°. What is the wavelength
of this coloured light?

A 578 nm C 637 nm
B 631 nm D 696 nm
12

23 Two charges + 2q and – q are placed at a distance 2d apart. The electric potential at X, a
distance d away from – q is

2d d
+2q –q X

−q −q
A C
4πε 0d 36πε 0d 2

−q −7q
B D
12πε 0 d 36πε 0d 2

24 The diagram shows two plates J and K, a distance 0.080 m apart in a vacuum. An electron,
originally at rest, is accelerated by a uniform electric field of 3.0 × 105 N C −1 from K to J. What
is the gain in the electron’s kinetic energy?

Plate J

0.080 m

Plate K
Electron
moving
towards J

A 4.3 × 10 −26 J C 4.8 × 10 −14 J


B 3.8 × 10 −15 J D 6.0 × 10 −13 J
13

25 Eight small conductors of charge Q are placed on the edge of an insulating disc of diameter
D. The angular frequency of rotation of the disc is ω .

Q
Q Q

D
Q Q

Q Q
Q

What is the equivalent electric current at the edge of the disc?

4Qω
A C 8Qω
π
8Qω 16Qπ
B D
πD ω

26 A car battery of e.m.f. 12 V and internal resistance 0.020 Ω is connected to a load of 4.0 Ω. If
the potential difference across the load is 10 V, what is the power lost in the connecting
wires?

A 0.13 W C 4.9 W
B 1.0 W D 5.0 W
14

27 Five resistors of equal resistance are connected as shown.

S Q

Which two points would give the maximum combination resistance?

A PQ C PS
B PR D QS

28 The diagram below shows a simple potentiometer circuit used to determine the internal
resistance of a cell of e.m.f. E. The driver cell has an e.m.f. of 2.0 V with negligible internal
resistance and the metre wire PQ is 1.0 m long. The cell is connected in parallel with a
resistor of 2.0 Ω. When the switch is open, the balance length is 0.70 m and when the switch
is closed, the balance length is 0.50 m.

2.0 V

P Q

E r
Switch S

2.0 Ω

What is the internal resistance of the cell?

A 0.15 Ω C 0.50 Ω
15

B 0.40 Ω D 0.80 Ω
29 Wire 1 carries a current of 4.2 A to the right as shown in the figure below. What is the
magnitude and direction of the current that is carried in Wire 2 so that the net magnetic flux
density at point A is 4.0 x 10-6 T.

Wire 1 I = 4.2 A

Wire 2

The magnetic flux density at a perpendicular distance r from a straight current-carrying


μo I
conductor is given by B = , where I is the current in the conductor.
2π r

A 0.40 A to the left C 1.4 A to the left


B 0.40 A to the right D 1.4 A to the right
16

30 One end of a flat rectangular coil of negligible mass is placed at the centre of a 1000-turn
circular coil of diameter 25 cm as shown in the figure below. A current of 5.0 A is passed
through the rectangular coil and when a 5.0 g paper rider is placed at 2.0 cm to the right of
the pivot, the rectangular coil is balanced horizontally. What is the magnitude of the current
that the 1000-turn circular coil must carry in order for the rectangular coil to remain
horizontal?

15 cm 2.0 cm
paper rider
4 .0

5.0 A
cm

1000-turn coil of
diameter 25 cm

The magnetic flux density at the centre of a flat circular coil of N turns and radius r is given by
μo N I
B= where I is the current carried in the coil.
2r

A 3.3 A C 6.5 A
B 5.0 A D 9.0 A
17

31 Large alternating currents in a straight conductor can be measured by the e.m.f. induced in a
small coil. Which of the following arrangements of the coil induces the largest e.m.f.?

A B C D

32 A right-hand drive car heads East at a speed of 20 m s-1. It cuts the vertical component of the
Earth’s magnetic field of flux density 5.0 X 10-5 T acting downwards. Taking the width of the
car’s bonnet to be 1.5 m, what is the e.m.f. generated across the bonnet and which side of the
car will be positive?
E.m.f. generated Side of car which is positive
A 0.67 mV Driver
B 0.67 mV Passenger
C 1.5 mV Driver
D 1.5 mV Passenger

33 An alternating current I in amperes in a load resistor of 8.0 Ω varies with time t in seconds
according to the equation:
I = 5.0 sin(100πt )
Which of the following is the mean power dissipated in the resistor?

A 40 W C 100 W
B 50 W D 200 W
18

34. A transformer steps up 120 V at the primary coil to 240 V at the secondary coil. If the
current in the primary coil is 2.0 A and the power loss in the windings and core of the
transformer is 48 W, what is the current in the secondary coil?

A 0.2 A C 1.0 A
B 0.8 A D 1.2 A

35 In a photoelectric emission experiment, a metal is irradiated with photons of wavelength λ.


The minimum frequency to cause photoelectric emission is f0. If c is the speed of light, what
fraction of the photon energy is converted to kinetic energy in the electron travelling with the
greatest speed?

A λ C f0 c
1−
λ − f0c λ
B c D f0 λ
1−
c − f0 λ c

36 The figure below shows the wave function ψ(x) of an electron.


ψ(x)

x
P 0 Q

Which of the following statements is correct?

A The probability of locating the electron at x = 0 is the highest.

ψ ( x ) is the probability of locating the electron within a given region.


2
B

C There is greater probability of locating the electron on the left of the vertical axis.
Q
∫ ψ ( x ) dx .
2
D The probability of locating the electron between positions P and Q is
P
19

37 Which of the following about doped semiconductors is correct?


Charge of semiconductor Majority charge carriers

p-type positive holes


A
n-type negative electrons
p-type neutral holes
B
n-type neutral electrons
p-type positive protons
C
n-type neutral neutrons
p-type neutral protons
D
n-type neutral electrons

38 The figure below shows how the potential V(x) varies with the distance x across a p-n
junction.
V(x)

Which of the following graphs correctly shows the variation of V(x) when reverse bias is
applied across the p-n junction?
20

A
V(x)
n - type

p - type

B
V(x)
p - type

n - type
21

C V(x)

n - type

p - type

D V(x)

p - type

n - type
22

39 Two samples of radioactive nuclides X and Y are prepared. Y has twice the initial
activity and twice the half-life of X. After 6 half-lives of X, what is the ratio of the activity
of X to Y?
1 1 1 1
A B C D
2 4 8 16

40 The nuclear reaction P + Q → X + Y proceeds with a release of energy. Which of the


following statement must be correct?
A Mass of X and Y is larger than mass of P and Q.
B Momentum of X and Y is larger than momentum of P and Q.
C Total binding energy of X and Y is larger than total binding energy of P and Q.
D Binding energy per nucleon of both X and Y are larger than binding energy per
nucleon of P or Q.

END OF PAPER
Centre Number Index Number Name Class

       

RAFFLES INSTITUTION
2010 Preliminary Examination

PHYSICS
9646 / 02
Higher 2

Paper 2
21 September 2010
1 hour 45 minutes

Candidates answer on the Question Paper.


No Additional Materials are required.

READ THESE INSTRUCTIONS FIRST


Write your Centre number, index number, name and class in the spaces provided at the top of this
page.
Write in dark blue or black pen.
You may use a soft pencil for any diagrams, graphs or rough working.
Do not use staples, paper clips, highlighters, glue or correction fluid.

Answer all questions.


Write your answers in the spaces provided in this booklet.
The number of marks is given in brackets [ ] at the end of each question or part question.

For Examiner’s Use


1 / 5
2 / 9
3 / 7
4 / 7
5 / 7
6 / 7
7 / 18
8 / 12
Total / 72

This booklet consists of 20 printed pages including the cover page.



 
DATA

speed of light in free space, c = 3.00 x 108 m s−1

permeability of free space, μ0 = 4 π x 10−7 H m−1

permittivity of free space, ε0 = 8.85 x 10−12 F m−1

elementary charge, e = 1.60 x 10−19 C

the Planck constant, h = 6.63 x 10−34 J s

unified atomic mass constant , u = 1.66 x 10−27 kg

rest mass of electron, me = 9.11 x 10−31 kg

rest mass of proton, mp = 1.67 x 10−27 kg

molar gas constant, R = 8.31 J K−1 mol−1

the Avogadro constant, NA = 6.02 x 1023 mol−1

the Boltzmann constant, k = 1.38 x 10−23 J K−1

gravitational constant, G = 6.67 x 10−11 N m2 kg−2

acceleration of free fall, g = 9.81 m s−2

 
 

 
FORMULAE 

uniformly accelerated motion, s = ut + 21 at 2

v 2 = u 2 + 2as

work done on/by a gas, W = pΔV

hydrostatic pressure, p = ρgh

Gm
gravitational potential, φ=−
r

displacement of particle in s.h.m., x = x0 sin ωt

velocity of particle in s.h.m., v = v 0 cos ωt

= ±ω x02 − x 2

mean kinetic energy of a molecule


3
of an ideal gas, E= kT
2

resistors in series, R = R1 + R2 + . . . .

resistors in parallel, 1/R = 1/R1 + 1/R2 + . . . .

electric potential, V = Q/4πε0r

alternating current/voltage, x = x0 sinωt

8π 2 m(U − E )
transmission coefficient, T = exp(−2kd) where k =
h2

radioactive decay, x = x0 exp(−λt)

0.693
decay constant, λ=
t1
2

 
 
 

 
Answer all questions in the spaces provided.

1 A cylindrical thermos flask is used to store hot water. The internal diameter and depth of
the thermos flask are measured to be (8.50 ± 0.01) cm and (17.0 ± 0.1) cm respectively.

(a) State the instrument used to measure its diameter and a systematic error that can
occur with the use of this instrument.

[2]

(b) Calculate the capacity of the thermos flask and its associated uncertainty.

Volume = cm3 [3]

 
 
 
 
 

 
2 (a) A mass hanging from a spring balance in air gives a reading of 50 N. When the
mass is completely immersed in water, the reading on the balance is 40 N. It is
now completely immersed in another liquid, giving a reading of 34 N. Calculate the
density of this liquid. Assume that the density of water is 1000 kg m-3.

Density = kg m-3 [2]

(b) In Fig. 2 below, a uniform beam of length 10.0 m and weight 500 N is hinged to a
wall at point O. Its far end is supported by a cable that makes an angle of 53.0°
with the horizontal. A 70.0 kg worker stands on the beam.

cable

53.0°
O
s beam
Fig. 2

(i) Draw a labelled diagram showing the forces acting on the beam.

[2]

 
(ii) The worker walks towards the far end of the beam from O. Calculate the
furthest distance s he can walk if the maximum possible tension in the cable
is 1000 N.

s= m [2]

(iii) Calculate the magnitude of the force exerted by the hinge on the beam
when the tension in the cable is 1000 N.

Reaction force = N [3]

 

 
3 (a) Gravitational field strength g and gravitational potential φ at a point due to a

spherical body are related by the equation g = − where r is the distance
dr
 
from the centre of the body to the point. Explain the significance of the negative
sign.
 

[1]

(b) Given the mass of Earth is 5.98 x 1024 kg and its radius is 6370 km, determine
the minimum kinetic energy required to project a spacecraft of mass 2550 kg
from the surface of Earth so that it completely escapes from the gravitational
field of Earth. Ignore air resistance.

Minimum energy = J [3]

(c) As a spacecraft falls towards Earth, it loses gravitational potential energy. State
the energy conversions for the spacecraft when it is falling through Earth’s
atmosphere at constant speed.

[1]

(d) An astronaut in a spacecraft orbiting around Earth can be said to experience


weightlessness. Explain why this is not true weightlessness.

[2]

 

 
4 (a) Explain what is meant by internal energy of a gas.

[1]

(b) A cylinder fitted with a piston contains 0.20 mole of an ideal gas. Initially the
volume and pressure of the gas are 5.0 × 10 −3 m3 and 1.0 × 105 Pa respectively.
(i) Calculate the initial temperature of the gas.

Initial temperature = K [2]

(ii) The gas is


(1) heated at constant volume to twice its initial temperature
(2) cooled at constant pressure to its initial temperature, and finally
(3) expanded isothermally to its initial volume.
Sketch the above changes on a clearly labelled p-V diagram.

p/ 105 Pa

2.0

1.0

V/ 10 3 m3
0 2.5 5.0
[4]

 
5 Two small identical styrofoam balls of mass 0.50 g and charge +15 nC are placed in a
hemispherical bowl of radius R with frictionless, non-conducting walls. At equilibrium,
they are at a distance of 0.50R apart as shown in Fig. 5.

hemispherical bowl

R R

0.50 R

Fig. 5

(a) (i) Show that θ ≈ 14.5° . [1]

(ii) On Fig. 5, indicate all the forces acting on one of the balls in the bowl. [2]

(iii) Hence determine the radius R of the bowl.

R= cm [3]

(b) State the effect on θ if the Styrofoam balls are replaced with metal ones of the
same size and charge.

[1]
10 
 
6 Fig.6.1 shows the front view of a large flat circular coil connected to a sinusoidal
alternating voltage supply and a small flat circular coil placed at the centre of the large
coil such that the planes of the two coils are coincident. The smaller coil is connected to
a cathode-ray oscilloscope (c.r.o).

large coil small coil

               
To a.c. supply To c.r.o.

Front view

Fig.6.1

(a) If the variation of the sinusoidal alternating current to the large coil is as shown in
Fig.6.2, draw sketch graphs, one in each case, to show the variation with time of
(i) the magnetic flux through the small coil and
(ii) the induced e.m.f. in the small coil as displayed on the c.r.o.. [2]

Current
In large
coil

time

Fig.6.2

Magnetic flux
through small
coil

time

Induced
e.m.f in
small coil
.

time
11 
 
(b) Justify the shape of your sketches in (a).

  [3]

(c)  State and explain how the trace on the screen of the c.r.o. would be affected if the
small coil is rotated such that the angle between the planes of the two coils
increase from zero to 900 whilst maintaining a constant root mean square current
in the large coil.

  [2]

 
12 
 
7 Radiation is a significant component of heat transfer in buildings, especially for sun-
exposed surfaces and regions of large temperature differences. Most countries have
building regulations that contain instructions about limiting heat transfer in order to
reduce the amount of heating or air-conditioning required.

In order to calculate heat transfer, a thermal transmittance coefficient or U-value is


P
measured for each type of building material. Mathematically, U =  where P is the
AΔT
rate of heat transfer in watts, A is the surface area of the structure and ΔT is the air
temperature difference between each side of the structure in Kelvin. The U-values of
three construction components are given below:

Component U-value / W m-2 K-1

Single-glazed window 5.6

Double-glazed window 3.2

Uninsulated roof 1.9

A house has windows of total area 24 m2 and a roof of area 60 m2. On average, the
owner heats the house for 3000 hours per year to a temperature that is 14 K above that
of the air outside.

(a) (i) Calculate the amount of energy lost in a year through single-glazed windows.

Energy loss = kWh [3]

(ii) By installing double-glazed windows, calculate the owner’s annual savings if


electricity costs $0.25 per kWh.

Savings = $ [3]
13 
 
(b) The roof is now insulated with two 50 mm thick layers of thermal insulation on
each side to reduce heat transfer, as shown in Fig. 7 below.
t

Fig. 7

U-value for thermal insulation 50 mm thick = 1.4 W m-2 K-1


To calculate the rate of heat transfer P through such a roof, a composite U-value,
Uc, has to be used. Uc can be expressed in terms of the U-values of the individual
materials by the equation
1 1 1
= + + .....
UC U1 U2
(i) Using the above equation, show that the rate of heat transfer P through the
roof with thickness of thermal insulation on each side t = 50 mm is 430 W.
[1]
14 
 
(ii) Complete the table below for the different values of t. Leave your answers for
P to 2 significant figures.

t / mm P/W
50 430

100

150

200
[2]

(iii) Using the data from the table, plot a graph of P against t. [2]

P/ W

400

300

200

100

50 100 150 200 250

thickness of thermal insulation on each side/ mm

(iv) Explain why the rate of heat transfer for a thickness of 250 mm thermal
insulation on each side cannot be accurately determined from the above
graph.

[1]
15 
 
(c) External work is required to get heat to flow from a cold reservoir to a hot reservoir.
A heat pump is such a device, which applies external work W to extract an amount
of heat QC from a cold reservoir, and delivers heat QH to a hot reservoir, as shown
in the illustration below.

QH
-
QC
Thermal efficiency, e, of a heat pump is defined as the ratio of W to QH during one
cycle of the process. W, which is equivalent( to ) , maintains a particular
temperature difference between the hot and cold reservoirs.

Effectiveness of a heat pump can be described in terms of its coefficient of


performance, COP, given by the relationship:
1
COP(heating mode) =
e
The graph below shows the relationship between thermal efficiency, e, of an ideal
heat pump and the temperature of the hot reservoir, Th, for a temperature
difference of 27 K.

x
x
x
x
16 
 
(i) The house loses energy at a rate of 5.00 kW when the interior temperature is
287 K and the outside temperature is 260 K. Assuming a heat pump
operates with a coefficient of performance that is 60.0% of the ideal value,
calculate the electric power the heat pump needs to maintain the interior
temperature at 287 K.

P W [3]

(ii) Electric resistance heaters convert all of the electrical energy supplied to
internal energy. Explain why heat pumps are preferred over electric
resistance heaters.

[2]

(iii) Give an example of an everyday household appliance that behaves like a


heat pump.

[1]

 
 
 
 
 
 
 
 
 
 
 
 
 
 
 
 
 
 
17 
 
It is recommended that you spend about 30 minutes on this question.

8 Light-dependent resistors or LDRs are used in light sensor circuits. The resistance of a
LDR is very high when the surrounding is dim and very low when it is illuminated with
light.
A student wishes to investigate how the resistance of a LDR varies with the amount of
light falling on it. The LDR has a resistance of 100 Ω when it is in bright light and a
resistance of 500 kΩ when no light falls on it.

Design a laboratory experiment to investigate how the resistance of the LDR depends
on the intensity of the illumination incident on the LDR.

You may assume that the following apparatus is available, together with any other
standard equipment which may be found in a college science laboratory:

electric light bulb


rheostat
light-dependent resistor (LDR)
light meter with light intensity sensor
digital multimeters
dry cells
connecting wires
black cardboard tube

You should draw diagrams to show the arrangement of your apparatus and important
electrical connections. In your account you should pay particular attention to

(a) the procedure to be followed,

(b) the measurements that would be taken,

(c) the control of variables,

(d) how the data would be analysed,

(e) any safety precautions that you would take.

[12]
18 
 
Diagram
19 
 
20 
 

 
 
End of Paper
Centre Number Index Number Name Class

RAFFLES INSTITUTION
2010 Preliminary Examination
PHYSICS 9646/03
Higher 2
9745/03
Paper 3 Longer Structured Questions
15 September 2010
2 hours
Candidates answer on the Question Paper.
No Additional Materials are required.

READ THESE INSTRUCTIONS FIRST

Write your Centre number, index number, name and class in the spaces provided at the top
of this page.
Write in dark blue or black pen.
You may use a soft pencil for any diagrams, graphs or rough working.
Do not use staples, paper clips, highlighters, glue or correction fluid.

Section A
Answer all questions.

Section B
Answer any two questions.

You are advised to spend about one hour on each section.


Write your answers in the spaces provided in this booklet.
At the end of the examination, enter the Section B questions you have answered in the grid
below.
The number of marks is given in brackets [ ] at the end of each question or part question.

For Examiner’s Use


1 / 10
2 / 10
Section A 3 / 10
4 / 10
/20
Section B /20
Total /80

This booklet consists of 23 printed pages.


2

Data

speed of light in free space, c = 3.00 x 108 m s-1

permeability of free space, μo = 4π x 10-7 H m-1

permittivity of free space, εo = 8.85 x 10-12 Fm-1


(1 / (36 π)) x 10-9 Fm-1

elementary charge, e = 1.60 x 10-19 C

the Planck constant, h = 6.63 x 10-34 J s

unified atomic mass constant, u = 1.66 x 10-27 kg

rest mass of electron, me = 9.11 x 10-31 kg

rest mass of proton, mp = 1.67 x 10-27 kg

molar gas constant, R = 8.31 J K-1 mol-1

the Avogadro constant, NA = 6.02 x 1023 mol-1

the Boltzmann constant, k = 1.38 x 10-23 J K-1

gravitational constant, G = 6.67 x 10-11 N m2 kg-2

acceleration of free fall, g = 9.81 m s-2


3

Formulae

uniformly accelerated motion, s = ut + ½at2

v2 = u2 + 2as

work done on/by a gas, W = pΔV

hydrostatic pressure, p = ρgh

gravitational potential, Gm
φ = −
r

displacement of particle in s.h.m. x = xo sin ωt

velocity of particle in s.h.m. v = vo cos ωt

= ±ω (x o
2
− x2 )
mean kinetic energy of a molecule 3
of an ideal gas E = kT
2

resistors in series, R = R1 + R2 + …

resistors in parallel, 1/R = 1/R1 + 1/R2 + …

electric potential, V = Q / 4πεor

alternating current/voltage, x = xo sin ωt

transmission coefficient, T = exp(-2kd)

8π 2 m (U − E )
where k =
h2

radioactive decay, x = xo exp (-λt)

decay constant 0.693


λ =
t1
2
4

SECTION A
Answer all the questions in this section.

1 (a) Define simple harmonic motion.

[2]

(b) The set up in Fig. 1 is used to demonstrate forced oscillation.


A paper card and pin are taped to the top end of a flexible rod B, which is pivoted
at its lower end such that it can only oscillate in a vertical plane. A pair of magnets
can slide along rod B.
A heavy pendulum P is attached to rod A which is pivoted at a fixed axle. The
position of the heavy pendulum can be adjusted along rod A.

Fig. 1

(i) State, with a reason, whether B or P is being forced to oscillate.

[2]
5

(ii) Suggest the effect of the following changes on the oscillations of rod B.
1. Turning the paper card by 90° about a vertical axis.

2. Moving the pair of magnets higher.

3. Increasing the number of rubber bands.

[3]

(iii) During the experiment, the frequency fP of oscillation of the heavy pendulum
P is kept constant while the frequency fB of oscillation of rod B is adjusted in
steps. After each adjustment on rod B, its amplitude A is noted when the
oscillation becomes steady. Sketch two labelled graphs to show the variation
of A with fB for
1. one rubber band, and
2. two rubber bands.
[Assume light damping is present.]

fB
[3]
6

2 (a) Distinguish between resistance and resistivity of a conductor.

[2]

(b) A cell of e.m.f. 2.50 V and internal resistance R is connected to two uniform
resistive wires in series as shown in Fig. 2.1. The wires are made of the same
material but have different lengths and diameters. Wire AB is 50.0 cm long and
has a diameter d, whereas wire BC is 30.0 cm long and has a diameter 0.30 d.
The ammeter and connecting wires are assumed to have no resistance.

2.50 V
A
R

A B C
Fig. 2.1

RAB
Show that = 0.150
RBC [2]
7

(c) A battery of e.m.f. 2.00 V and internal resistance r is connected across wire BC in
parallel with another resistor of resistance r as shown in Fig 2.2. The galvanometer
shows no deflection when the jockey J is at the midpoint of wire BC.

2.50 V
A
R

A B C
J

2.00 V
r

r
Fig 2.2

(i) Show that VBC = 2.00 V [1]

(ii) Determine the internal resistance R of the 2.50 V cell if the ammeter shows a
reading of 0.400 A.

R= Ω [3]

(d) Suggest and explain whether your answer in part (c)(ii) is an over-estimate or
under-estimate if the ammeter is not ideal.

[2]
8

3 (a) Fig. 3.1 shows the essential energy levels of an atom in the production of laser
light.

E3

E2

E1

Fig. 3.1

(i) Identify each of the energy levels, E1, E2 and E3, with ‘metastable state’,
‘ground state’ and ‘excited state’.
E1

E2

E3 [2]

(ii) By drawing arrows on Fig. 3.1 to represent the movement of electrons,


explain the production of laser light.

[3]
9

(iii) In a ruby laser, electrons may reside at the metastable state for up to 3.0 ms.
Calculate the minimum uncertainty in the frequency of the photon emitted
during the production of laser light.

Minimum uncertainty in frequency = Hz [2]

(b) Fig. 3.2 shows a typical X-ray spectrum produced by an X-ray tube where
electrons are accelerated through a constant accelerating potential towards a
metal target.

Intensity

0 Wavelength
λmin
Fig. 3.2

(i) Account for the value λmin.

[1]

(ii) Explain the changes in the X-ray spectrum when the accelerating potential is
decreased. Sketch the new spectrum on Fig. 3.2.

[2]
10

4 A cyclotron is a device used to accelerate ions to very high speeds. Fig. 4 shows a
diagram of a cyclotron viewed from above. It is composed of two hollow, semi-circular
electrodes called “Dees”. The “Dees” are encased inside a vacuum chamber and
exposed to a perpendicular uniform magnetic field. An ion source lies in between the
“Dees” at point A. An alternating voltage supply is connected across the “Dees”.
During operation, the voltage supply produces an alternating electric field in the small
gap between the “Dees”. This is to ensure that the ions are accelerated each time they
cross the gap. On entering the “Dees”, the uniform magnetic field causes the ions to
move in a circular path. As the ions speed up, they travel in ever larger circles within the
“Dees”. Once the ions reach a sufficiently large speed, they exit through an outlet in one
of the “Dees” which is aimed at a target.

Fig. 4

At any time when an ion of mass m and charge q accelerates across the small gap, the
potential difference between the “Dees” is V. The ion then travels in a circular path in
the “Dees” where a uniform magnetic field of flux density B is applied perpendicularly.
11

2π m
(a) Show that the time T for the ion to complete one revolution is .
Bq
Ignore relativistic effects. [2]

(b) A helium nucleus of mass 6.68 x 10-27 kg and charge 2e is accelerated in the
cyclotron by applying an alternating potential difference of 450 V across the
“Dees”. The magnetic flux density through the “Dees” is 0.850 T.

(i) Calculate the time T to complete one revolution for the helium nucleus.

T= s [2]

(ii) Determine the frequency f of the alternating voltage supply so that the
helium nucleus is accelerated everytime it crosses the gap between the
“Dees”.

f= Hz [2]
12

(iii) State an expression for the gain in kinetic energy of the helium nucleus after
one revolution in terms of e and V.

Gain in kinetic energy = [1]

(iv) Hence, determine the speed v of the helium nucleus after five revolutions.

v= m s-1 [3]
13

SECTION B
Answer two questions in this section.

5 A ball of mass m = 3.00 kg is released from rest at a height h = 0.500 m on a frictionless


incline as shown in Fig 5.1. The incline, which makes an angle θ = 30.0o to the
horizontal, is fastened to an immovable table of height H = 2.00 m.
m

h
θ

Fig. 5.1 R

(a) Determine the contact force between the incline and the ball, after the ball is
released.

Contact force = N [2]

(b) Determine the acceleration of the ball as it slides down the incline.

Acceleration = m s-2 [2]


14

(c) Hence, or otherwise, show that the speed of the ball as it leaves the incline is
3.13 m s-1. [2]

(d) Calculate the horizontal range R of the ball.

Horizontal range = m [4]

(e) The estimated normal contact force acting on the ball upon hitting the floor is
shown in Fig. 5.2. Assume that the floor is frictionless.

Normal
contact force / N
360

0 Time / s
0.200
Fig. 5.2
15

(i) Determine the impulse delivered to the ball in the vertical direction.

Vertical component of impulse = Ns [2]

(ii) Hence, determine the vertical speed of the ball at the instant it rebounds from
the floor.

Vertical speed of rebound = m s-1 [4]

(iii) State and explain whether this is an elastic or inelastic collision. Describe the
energy changes during the collision.

[4]
16

6 (a) State two conditions for observable interference of two waves.

[2]

(b) In an aircraft landing system, it is important to guide the aircraft along the centre-
line of the runway prior to landing. In a simple landing system, rows of light guides
are lined along the runway to help guide the pilot.

The minimum power of light that can be detected by the human eye of area
0.50 cm2 is about 2.5 × 10-11 W. If an aircraft is 12 km away from the runway, find
the required power of one light guide such that it is observable by the pilot.
Assume that the light guide is a point source and that there are no energy losses.

Power = W [3]
17

(c) In another type of landing system, aircrafts are guided using interference of radio
waves. Fig. 6.1 shows two radio wave emitters P and Q positioned 50 m apart at
the end of the runway. The two emitters emit radio waves of frequency f1 in phase.
The aircraft can be guided by searching for the strong signal radiated along the
lines of constructive interference, also known as anti-nodal lines. To ensure that
the aircraft is along the centre-line of the runway, the aircraft needs to “lock on” to
the central anti-nodal line.

Top view
(figure not to scale) Anti-nodal lines

Q
runway

Fig. 6.1

(i) Suggest why radio waves are used instead of waves of shorter wavelengths
(e.g. microwaves, etc.).

[2]

(ii) Explain why the entire centre-line will always be an anti-nodal line.

[2]
18

(d) One particular aircraft at a vertical height of 480 m strays off the centre-line as
shown in Fig. 6.2. Fig. 6.3 shows the radio wave signals from P and Q detected by
the aircraft in this position.

480 m

(diagram not to scale)

180 m

4800 m

50 m Q Fig. 6.2

Signal detected by aircraft

Signal A
time

Signal B

Fig. 6.3

(i) The source of signal B is emitter P. Using Fig. 6.3, explain why this is so.

[1]

(ii) State the phase difference between signals A and B.

Phase difference = rad [1]


19

(iii) Hence determine the frequency f1 of the radio wave used.

f1 = Hz [4]

(e) As an additional precaution to prevent the aircraft from “locking on” to the wrong
anti-nodal line, the emitters can simultaneously emit another radio wave of a
different frequency f2. However, for this precaution to work, the ratio of the two
f 1 2 4
frequencies 1 should not be an integer ratio (e.g. , , , etc.).
f2 2 3 3

(i) Explain how this precaution can prevent the aircraft from “locking on” to the
wrong anti-nodal line.

[1]

(ii) Explain why the ratio of the two frequencies should not be an integer ratio.

[2]
20

(f) Suggest one advantage and one disadvantage of the wave-interference system
over the light guide system in guiding aircrafts to land safely.

Advantage:

[1]

Disadvantage:

[1]
21

7 (a) Define binding energy.

[1]

(b) A minimum energy Q is required to remove a neutron from a helium-4 nuclide to


form a helium-3 nuclide. The following data is given:
Binding energy per nucleon of helium-4 nuclide = 6.8465 MeV
Binding energy per nucleon of helium-3 nuclide = 2.2666 MeV
Mass of neutron = 1.0087 u
1 u = 931.494 MeV

(i) Write the nuclear equation for this reaction.


[1]

(ii) Calculate Q.

Q= MeV [3]

(iii) Hence, calculate the difference in mass between the helium-3 and helium-4
nuclides.

Difference in mass = u [3]


22

(iv) With reference to the above process, explain why the mass difference is less
than the mass of a neutron.

[1]

(c) Helium-2 is a hypothetical isotope of helium which consists of two protons and no
neutrons. It has negative binding energy.
(i) Explain the implication of the italicized terms on helium-2.

[1]

(ii) Suggest a reason for this.

[1]

32 35
(d) A radioactive source contains a mixture of nuclides 15 P and 16 S . The half-life of
35 32
16 S is 6.14 times that of 15 P . At time t = 0 s, 90% of the total activity from this
32
source comes from the 15 P nuclides.

(i) Define half-life.

[1]

32
(ii) If the number of 15 P nuclides is N at time t = 0 s, determine the number of
35
16 S nuclides in the source in terms of N.

35
Number of 16 S nuclides = [3]
23

(iii) Calculate the time elapsed for 90% of the total activity to come from the
35 32
16 S nuclides, given that the half-life of 15 P is 14.3 days.

Time elapsed = days [4]

32
(iv) Suggest a possible use for 15 P , which is a beta-emitter with a half-life of
14.3 days.

[1]

END OF PAPER
Question Answer
1 D
2 B
3 D
4 C
5 C
6 B
7 C
8 B
9 B
10 C
11 B
12 C
13 C
14 D
15 C
16 B
17 B
18 D
19 C
20 D
21 B
22 A
23 B
24 B
25 A
26 C
27 B
28 D
29 A
30 C
31 D
32 D
33 C
34 B
35 D
36 A
37 B
38 A
39 D
40 C
2010 JC2 Prelim H2 Physics Paper 2 Suggested Solutions

1 (a) vernier callipers


zero error (do not accept parallax)
(b) ⎛ d2 ⎞
V = π⎜ ⎟h
⎝ 4 ⎠
= 964.665 cm3
ΔV 2Δd Δh
= +
V d h
⎛ 2×0.01 0.1 ⎞
ΔV = ⎜ + ⎟ ×964.665
⎝ 8.50 17.0 ⎠
= 8 cm3
V = (965 ± 8) cm3

2 (a) U1 = W - T1
= 50 - 40
= 10 N
10 = ρVg
1

10
V=
1000×9.81
= 1.02×10-3 m3
U2 = 50 - 34
= 16 N
16
ρ2 =
1.02×10-3 ×9.81
= 1600 kg m-3
(b) (i)
reaction tension
force

weight of
force by worker
beam
on beam

(ii) Taking moments about O,


5.00×500 +(70×9.81× s ) =1000sin53.0°×10.0
s = 7.99 m
(iii) At equilibrium, the net vertical and horizontal forces must be zero.
Rx = Tcos53.0° = 602 N
Ry = 500 +(70×9.81) - Tsin53.0° = 388 N
R = 6022 + 3882
= 716 N
1
3 (a) Gravitational field strength acts in the direction of decreasing potential / As distance
from the centre of a spherical mass increases, gravitational field strength decreases
but the gravitational potential increases.

(b) Total energy on surface = Total energy at infinity where PE=0 & KE=0 (since
spacecraft launched with minimum kinetic energy)
⎛ GME ms ⎞
KEmin + ⎜ − ⎟=0
⎝ RE ⎠

⎛ 6.67×10-11 ×5.98×1024 ×2250 ⎞


∴KEmin = ⎜ ⎟
⎝ 6370000 ⎠
11
=1.41×10 J
(c) Gravitational potential energy Æheat (air resistance) + light (fire over its body) +
sound energy (due to rapid vibrations of the body of the spacecraft)

(d) True weightlessness occurs in a situation where the astronaut is remote from the
gravitational field of celestial bodies i.e. gravitational force is zero / net gravitational
field strength is zero / experiences no net acceleration due to zero gravitational
force on him.

The astronaut and the spacecraft he is in are both accelerating with the same
acceleration, hence the normal contact force on him is zero / the gravitational force
on the astronaut entirely provides for the centripetal force for him to be able to
move in circular orbit, hence the normal contact force on him is zero. Here,
weightlessness is just a sensation as the gravitational force on him still exists.

4 (a) It is the sum of the kinetic energies and potential energies of the particles in the
gas.
(b) (i)
Using pV = nRT ,
pV 1.0 × 105 × 5.0 × 10−3
T = =
nR 0.20 × 8.31
= 300 K

2
(ii)

5 (a) (i) 0.25R


sin θ = ∴θ = 14.48 = 14.5°
R
Sine rule and cosine rule are also accepted.

(ii) hemispherical bowl

Normal reaction
force due to the
2θ bowl, N

R R

0.50 R Electrostatic
Force due to
other ball, FE

Weight of ball, W

(iii) FE = mg tan θ

(15 × 10 )
2
−9
1
( )
= 0.5 × 10−3 ( 9.81) tan (14.5 )
4πε 0 ( 0.5R )
2

R = 0.0799 m
= 8.0 cm
(b) θ will be smaller.

3
6 (a)

Magnetic flux
through small
coil

time

Induced
e.m.f in
small coil
.

time

(b) Magnetic flux through the small coil is proportional to the magnetic flux density
which is proportional to the alternating current in the large coil. Hence the magnetic
flux-time graph through the small coil has the same shape as that of the current-
time graph for the large coil.

Induced e.m.f. in the small coil is proportional to the rate of change of magnetic flux
linkage which is obtained from the negative of the slope of the magnetic flux- time
graph.

(c) When the planes of the two coils are at 900 to each other, the magnetic field due to
the current in the large coil is parallel to the plane of the small coil. Hence there is
no magnetic flux linkage with the small coil and no e.m.f. will be induced in the
small coil.

The trace on the c.r.o. will show the amplitude of the induced e.m.f. reduced to
zero.

7 (a) (i) P = UAΔT = 5.6 × 24 × 14 = 1881.6


E = Pt = 1881.6 × 3000 × 60 × 60 = 2.0321× 10 4 MJ
1 kWh = 1000 × 60 × 60 = 3.60 × 106 J
∴ 2.0321× 104 MJ = 5644.7 kWh

(ii) ΔP = (U1 − U2 )AΔT = (5.6 − 3.2) × 24 × 14 = 806.4


ΔE = ΔPt = 806.4 × 3000 × 60 × 60 = 8.7091× 109 J = 2419.2 kWh
Savings = 2419.2 × 0.25 = $604.80

4
(b) (i) 1 1 1 1 1 1 1
= + + = + +
UC U1 U2 U3 1.4 1.9 1.4
−1
⎛ 1 1 1 ⎞
UC = ⎜ + + ⎟ = 0.51154
⎝ 1.4 1.9 1.4 ⎠
P = UC AΔT = 0.51154 × 60 × 14 = 430 W

(ii) t / mm P/W
50 430

100 250

150 170

200 130

P/W
(iii)

400 x

300

x
200

x
100 x

0
t / mm
50 100 150 200 250
Note: Heat transfer via conduction dominates and the relationship between rate
kAΔT
of heat transfer P and thickness t of thermal insulation is given to be P = ,
t
where k is the thermal conductivity of the medium, A is the surface area normal
to direction of heat transfer and ΔT is the difference tempearture Thus, graph
should be an inverse curve.

5
(iv) The extrapolation of the graph to 250 mm may not be accurate because:
- 250mm is outside of the range of data collected.
- the behaviour of the graph/relationship may change beyond 200mm.
- there are too few data-points to determine the shape of the curve accurately.
- it is difficult to extrapolate a non-linear graph accurately.

(c) (i) From the graph,


e = 0.094
1
(COP )ideal = = 10.638
e
Qh
(COP )heating mod e = 0.60 × 10.638 = 6.3828 =
W
W Qh / t 5000
P= = = = 783 W
t (COP )heating mod e 6.3828
(ii) A heat pump requires less power than actually needed,
thus they are more cost effective (although not efficient).
(iii) Refrigerator, air-conditioner

6
8 Diagram

Arrangement of LDR and electric light bulb:

black cardboard tube

electric light bulb


LDR / light
intensity sensor
to LDR circuit to electric light
bulb circuit

Electric light bulb circuit:


A

V 12.0 V battery

LDR circuit:
mA

1.5 V dry cell V

Problem Definition
To investigate how the resistance of the LDR depends on the intensity of the
illumination incident on the LDR.
Dependent variable (R): resistance of LDR
Independent variable (i): intensity of illumination incident on the LDR
Controlled variables: distance between light source and LDR or light intensity
sensor
e.m.f. of dry cells
alignment of light source with LDR or light intensity sensor

7
Apparatus and Material
electric light bulb
rheostat
light-dependent resistor
light meter with light intensity sensor
digital ammeter
digital milli-ammeter
digital voltmeter
dry cells
connecting wires
black cardboard tube
sticky tape
metre rule

Procedures
1. Set up the apparatus as shown in the diagrams above. Tape the LDR in
position at one end of the black cardboard tube with adhesive tape. Tape
the electric light bulb at the other end of the black cardboard tube with
adhesive tape, using a meter rule to ensure that it is aligned along the
same horizontal axis as the LDR.
2. Adjust the rheostat in the electric light bulb circuit to maximum resistance.
Close the circuit to switch on the light bulb.
3. Measure and record the p.d reading V on the voltmeter and current
reading I on the milli-ammeter in the LDR circuit.
4. Replace the LDR with the light intensity sensor, connected to a light meter,
at the same position. Record the intensity reading i on the light meter.
5. Increase the resistance of the rheostat and repeat steps 3 and 4 to obtain
at least 6 sets of readings.
V
R=
6. The resistance of the LDR can be calculated using the equation I
7. The distance between the electric light bulb and the LDR or light intensity
sensor is kept at a constant distance throughout the experiment by using
adhesive tape to fix them in position and using a metre rule to measure the
distance between them before each reading to ensure that it is constant.
The e.m.f. of the dry cells is checked by connecting a voltmeter across them
before each reading to ensure that they remain at a constant value. The alignment
and orientation of the light bulb and LDR or light intensity sensor are kept constant
throughout the experiment by using adhesive tape to fix them in position.

Analysis
Assume that
R = k in
Where i is the intensity of the illumination incident on the LDR,
R is resistance of the LDR , and
k and n are constants
Taking lg on both sides, lg R = n lg i + lg k .
Plot a graph of lg R against lg k.
If the above relationship is true, a straight line graph will be obtained where the
gradient is equal to n and the y-intercept is equal to lg k.
Hence k = 10 where c is the y-intercept.
c

8
Safety Precautions
1. Do not look directly at the bright light source. Wear polaroid protective
glasses.
2. Do not touch the hot light source with bare hands. Wear gloves when
handling the light bulb after use.
3. Do not handle electrical circuits with wet hands.
Producing Reliable Results / Additional Details
1. As the resistance of the LDR is quite large, the current reading will be
small. Hence a milli-ammeter should be used to measure the current in the
LDR circuit.
2. The voltmeter should be placed to measure the potential difference across
both the ammeter and LDR when the resistance of the LDR is high. This
will ensure that the ammeter will measure the small current through LDR.
3. The electric light bulb and LDR or light intensity sensor are placed in a
black cardboard tube to minimise light from the surroundings from
reaching the LDR or light intensity sensor.
4. Wait for intensity and multimeter meter readings to stabilise before
recording.

Other accepted variations of answer


1. The multimeter can be used as an ohmmeter to measure the resistance of
the LDR (note that in such a case, the ohmmeter is connected directly to
the LDR, no e.m.f. source is required).
2. Intensity of the light incident on LDR can be varied by varying the distance
between light source and LDR. Power of the light source must be kept
constant in such a case.

9
1
Physics Prelim Paper 3 Marking Scheme

SECTION A

1 (a) It is the motion of a body where its acceleration is directly proportional to its
displacement from a fixed point and is always directed towards that fixed point.

(b) (i) B is being forced to oscillate because P is heavier.

(ii) 1. Increases the damping of the rod B.


2. Decreases the natural frequency of rod B.
3. Increases the coupling between pendulum P and rod B.
Increases the amplitude of the driving force on rod B.

(iii)

2 (a) - The electrical resistance R of a conductor is defined as the ratio of the p.d. V
across it to the current I through it.
[No definition of electrical resistance – minus 1M]
- The electrical resistivity ρ of a material is the constant of proportionality relating
the electrical resistance R to the dimensions of the material (length and area).
L A
OR word definition of R = ρ or ρ = R
A L
OR R depends on dimensions while ρ is a material characteristic/property

(b) L L 4ρ L
R=ρ =ρ =
A
(
πd2
4 ) π d2

2 2
RAB LAB dBC LAB dBC
= 2 × OR × 2
RBC d AB LBC LBC d AB

50.0 ( 0.3d )
2

= 2 × = 0.15
d 30.0
2
Physics Prelim Paper 3 Marking Scheme

(c) (i) Terminal p.d. for the 2.00 V battery:


r
Vr = × 2.00 = 1.00 V = VBJ (null deflection)
2r
1
Since balance length LBJ = LBC , VBC = 2VBJ = 2Vr = 2.00 V .
2

(ii) Method 1 Method 2


From part (c)(i), From part (c)(i),
VBC = 2.00 V VBC 2.00
RBC = = = 5.00 Ω
I 0.400
R AB VAB R AB
From part (b), = = 0.15 From part (b), = 0.15
RBC VBC RBC

VAB = 2.00 × 0.15 = 0.30 V R AB = 5.00 × 0.15 = 0.75 Ω


VAC = 0.30 + 2.00 = 2.30 V R AC = 0.75 + 5.00 = 5.75 Ω

OR OR
VAC = 2.00 × 1.15 = 2.30 V RAC = 1.15 × 5.00 = 5.75 Ω

E = VAC + IR 2.50
RT = = RAC + R
2.50 = 2.30 + ( 0.400 ) R 0.400
R = 0.500 Ω
R = 0.500 Ω

(d) Over-estimate. 0.20 V is actually the p.d. across R as well as that of the ammeter.
E − VAC = I ( R + RA )
R + RA = 0.500 Ω ⇒ R < 0.500 Ω
To show that the calculated R is an overestimate, there must be some statement
relating the resistance of the ammeter to either
- pd calculated for R actually includes the pd across the ammeter
- resistance calculated of R actually includes the resistance of the ammeter

3 (a) (i) E1 Ground state


E2 Metastable state
E3 Excited state
B2 for any 2 correct answers
3
Physics Prelim Paper 3 Marking Scheme

(ii) Electrons are pumped up / excited from E1 to E3.


Electrons at E3 will quickly spontaneously decay from E3 to E2. Since E2 is
the metastable state, a population inversion is created between E2 and E1.
When one electron falls spontaneously from E2 to E1, a photon will be
emitted. This photon will go on to stimulate emission of electrons at E2.
These photons form the laser light.

(iii) h
ΔE Δt ≥
2
h
ΔE ≥
4πΔt
h
h Δf ≥
4π (3.0 × 10 −3 )
Δf ≥ 26.53 Hz
⎛ ΔE Δf Δf ⎞
⎜ E = hf ⇒ E = f ⇒ ΔE = f hf = hΔf ⎟
⎝ ⎠
minimum Δf = 26.5 Hz

(b) Intensity

0 Wavelength
λmin
(i) Occurs when the most energetic electrons are stopped completely and all
their kinetic energy is converted to photon energy.

(ii) hc
Since λmin = , minimum wavelength will increase when the accelerating
eΔV
potential is decreased.
The intensity at all wavelengths will decrease because the speeds / kinetic
energy of the electrons are decreased.
4
Physics Prelim Paper 3 Marking Scheme

4 (a) In the magnetic field,


mv 2 mv
Bqv = ⇒r =
r Bq
2πr
Since T =
v ,
2π ⎛ mv ⎞ 2π m
∴T = ⎜ ⎟ ⇒T =
v ⎝ Bq ⎠ Bq

(b) (i) 2π (6.68 x 10-27 )


∴T = -19
= 1.54x10-7 s
0.85 x 2 x 1.6 x 10

(ii) In order for the nucleus to accelerate when it crosses the gap, freq. of the
alternating voltage = orbital freq. of the nucleus

1
∴f = -7
= 6.49 x 10 6 Hz
1.54 x 10

(iii) KE after one revolution = Work done by e-field on helium nucleus = 2qV =
2(2e)V = 4eV

(iv) The gain in KE after each rev. = 4eV


The gain in KE after five rev. = 20eV
1
∴ mv 2 = 20eV
2
40eV
v= = 6.57 × 105 m s-1 [B1 – for sub]
m [A1 for final answer]

Comments:
(b)(ii) Common mistake is the failure to realize that in order for the nucleus to accelerate
when it crosses the gap, freq. of the alternating voltage = orbital freq. of the nucleus
because the ion crosses the gap twice in one revolution. A handful of students halved
the period of the ion and took the reciprocal to calculate the frequency of the voltage
supply which is incorrect.
(b)(iii) Common mistake 1: KE = 2eV.
Common mistake 2: KE = 2e (450 V) = 900 eV (eV is not electron-volt !)
5
Physics Prelim Paper 3 Marking Scheme

SECTION B

5 (a) N = W cos θ
= (3.00 x 9.81) cos 30.0o
= 25.5 N

(b) a// = g sin θ


= 9.81 sin 30.0o
= 4.91 m s-2

(c) h/s// = sin θ


s// = h / sin θ
=0.500 / sin 30.0o
= 1.00 m

v//2 = u//2 + 2 a// s//


= 02 + 2 x 4.91 x 1.00
= 9.82 or 9.81

v// = √9.82 = 3.13 (shown)

(d) sy = uy t + ½ ay t2
2.00 = (3.13 sin 30.0o) t + ½ (9.81) t2
4.905 t2 + 1.565 t – 2.00 = 0
t = 0.499 s

sx = ux t + ½ ax t2
= (3.13 cos 30.0o) (0.499) + 0
= 1.35 m

(e) (i) Impulse = area under F-t graph


= ½ (0.200) (360)
= 36.0 Ns

(ii) vy = u y + a y t
= (3.13 sin 30.0o) + (9.81) (0.499)
= 6.46 m s-1
6
Physics Prelim Paper 3 Marking Scheme

Δpy = m [vy – uy]


36.0 = 3.00 [vy – (–6.46)]
vy = 5.54 m s-1

(iii) Since the KE or speed after collision is smaller, the collision is inelastic.
During collision, the KE of the ball is converted into sound energy, thermal
energy and/or elastic PE as the ball deforms.

6 (a) Any two:


1. The waves must be coherent.
2. The waves must have approximately the same amplitude.
3. The waves must be unpolarised or polarised in the same plane (for
transverse waves).
4. The waves must interfere to give regions of maxima (constructive
interference) and minima (destructive interference).

(b) Intensity required at 12 km away,

I=
Peye
=
( 2.5 × 10 ) = 5.0 × 10
−11
−7
W m-2
Aeye ( 0.50 × 10 )
−4

Consider light guide,


Plight
I=
A
Plight = I .A

( ⎣ )
= 5.0 × 10 −7 ⎡ 4π (12000 ) ⎤
2


= 905 W

(c) (i) Shorter wavelengths means the anti-nodal lines will be closer to one another.

Hence, aircrafts may “lock on” to the wrong line of maxima / difficult to identify
the central line of maxima / difficult to differentiate the lines of maxima.

(ii) Since the two radio waves are in phase, along centre-line, path difference is
always zero / phase difference is always zero / P & Q are equidistant from
any point on the centre-line. Hence constructive interference occurs.

(d) (i) P is nearer to the aircraft.


Hence intensity (or amplitude) of signal should be higher.
7
Physics Prelim Paper 3 Marking Scheme

(ii) π
From Fig. 6.3, phase difference = rad
2

(iii) π
Since phase difference of signals = ,
2
λ
path difference =
4

Distance from P to plane = 48002 + 1802 + 4802 = 4827.30 m

Distance from Q to plane = 48002 + 2302 + 4802 = 4829.42 m


λ
Hence ( 4829.42 − 4827.30 ) =
4
λ = 8.49 m
c
f = = 35.3 × 106 Hz
λ

Note:
λD
Do not accept if student uses formula x = as it is not a 2-D problem.
a
5λ 9λ
Accept if student choose path difference as , , etc
4 4

(e) (i) If aircraft is on the central anti-nodal line,


it should detect maximum signals from both frequencies / the maximum
signal will be stronger
OR
If aircraft is on wrong anti-nodal line,
only one of the frequencies will show a strong signal.

(ii) If the ratio is an integer ratio,


It means that higher orders of maxima will still coincide/overlap
Hence the aircraft could still detect maximum signals from both frequencies
even though it is not on the central anti-nodal line.
8
Physics Prelim Paper 3 Marking Scheme

(f) Advantage:
Can still work under low visibility conditions / Use of detector to align aircraft is
more accurate than using visual inspection.

Disadvantage:
Possible interference of signals from other sources (e.g. radio stations,
telecommunication base stations, etc) / It is more costly to install the emitters and
receivers on every airplane.

7 (a) Binding energy is defined as the amount of energy needed to split a nucleus into
its individual nucleons.

(b) (i) 4
2 He → 32 He + 01n

(ii) Q = Difference in total BE


= 4 ( 6.8465 ) − 3 ( 2.2666 )
= 20.5862 MeV

(iii) Q
m ( n ) + m (He-3 ) − m (He-4 ) =
931.494
m (He-4 ) − m (He-3 )
20.5862
= 1.0087u −
931.494
= 0.9866u

(iv) Energy is supplied in order to conserve mass-energy.

(c) (i) Helium-2 is unstable and cannot exist in a bound state.

(ii) Large coulomb repulsion between the protons.

(d) (i) Half life is the time taken for a sample of radioactive atoms to decay to half
its initial number.
9
Physics Prelim Paper 3 Marking Scheme

(ii) AP = λP N .......... (1)


AS = λS NS ........... ( 2 )
(1) / ( 2) :
AP λN
= P
AS λS NS
ln 2
N
TP
9=
ln 2
NS
TS
N 6.14TP
9= ×
TP NS
NS = 0.682N

(iii) Let the total initial activity be A0


ln2
− t
AS = 0.1A0 e 6.14TP

ln2
− t
AP = 0.9 A0e TP

ln2
− t
6.14TP
AS 0.1A0 e
= ln2
=9
AP −
TP
t
0.9 A0 e
ln2 ln2
t− t
e TP 6.14TP
= 81
⎛ 1 1 ⎞
t ln 2 ⎜ − ⎟ = ln81
⎝ TP 6.14TP ⎠
ln81
t= = 108 days
⎛ 1 1 ⎞
ln 2 ⎜ − ⎟
⎝ TP 6.14TP ⎠

(iv) It can be used to trace a plant’s fertilizer uptake.


RIVER VALLEY HIGH SCHOOL
YEAR 6 PRELIMINARY EXAMINATION

H2 PHYSICS 9646
PAPER 1
20 SEP 2010
1 HOUR 15 MIN

CANDIDATE
NAME

CENTRE INDEX
NUMBER S NUMBER

CLASS 6
INSTRUCTIONS TO CANDIDATES

DO NOT OPEN THIS BOOKLET UNTIL YOU ARE TOLD TO DO SO.

Read these notes carefully.


Write your name, class and index number in the spaces above.

There are forty questions in this paper. Answer all questions. For each question, there are four
possible answers, A, B, C and D.
Choose the one you consider correct and record your choice in soft pencil on the separate
Answer Sheet.

Read the instructions on the Answer Sheet very carefully.

Each correct answer will score one mark. A mark will not be deducted for a wrong answer. Any
rough working should be done on the Question Paper.

The total number of marks for this paper is 40.

____________________________________________________________________________
This Question Paper consists of 20 printed pages.

River Valley High School Pg 1 of 20 Year 6 H2 Physics 9646


Preliminary Examination 2010
Data

speed of light in free space, c = 3.00 × 108 m s–1

permeability of free space, μO = 4 π × 10–7 H m–1

permittivity of free space, εO = 8.85 × 10–12 F m–1

(1/(36 π )) × 10–9 F m–1

elementary charge, e = 1.60 × 10–19 C

the Planck constant, h = 6.63 × 10–34 J s

unified atomic mass constant, u = 1.66 × 10–27 kg

rest mass of electron, me = 9.11 × 10–31 kg

rest mass of proton, mp = 1.67 × 10–27 kg

molar gas constant, R = 8.31 J K–1 mol–1

the Avogadro constant, NA = 6.02 × 1023 mol–1

the Boltzmann constant, k = 1.38 × 10–23 J K–1

gravitational constant, G = 6.67 × 10–11 N m2 kg–2

acceleration of free fall, g = 9.81 m s–2

River Valley High School Pg 2 of 20 Year 6 H2 Physics 9646


Preliminary Examination 2010
Formulae

1 2
uniformly accelerated motion, s = ut + at
2
v 2 = u 2 + 2as

work done on/by a gas, W = pΔV

hydrostatic pressure, p = ρ gh

GM
gravitational potential, φ=−
r

displacement of particle in s.h.m., x = xO sin ωt

velocity of particle in s.h.m., v = v O cos ωt

= ±ω (x 2
O − x2 )
3
mean kinetic energy of a molecule of an ideal gas, E= kT
2

resistors in series, R = R1 + R2 + K

resistors in parallel, 1/ R = 1/ R1 + 1/ R2 + K

Q
electric potential, V=
4πε O r

alternating current/voltage, x = xO sin ωt

transmission coefficient, T ∝ exp ( −2kd )

8π 2 m (U − E )
where k =
h2

radioactive decay, x = xO exp ( −λ t )

0.693
λ=
decay constant, t1
2

River Valley High School Pg 3 of 20 Year 6 H2 Physics 9646


Preliminary Examination 2010
For each question there are four possible answers, A, B, C and D. Choose the one you
consider to be correct.

1 Which of the following could be the correct expression for the speed v of sound in
a gas of density ρ and at a pressure P?
(γ is a dimensionless constant.)

γ γρ γP
A v= B v= C v= D v = γρ P
ρP P ρ

2 When a drop of oil of mass m and density ρ is put on a water surface it spreads
over a circular area of diameter d. Assuming that this area consists of a
monomolecular layer which one of the following gives the approximate diameter
of a molecule?

m ρ 3m 4m
A B C D
πρ d π dm 4π d 3 ρ π d 2ρ

3 The figure below shows a graph of an object’s motion. Which sentence is a


correct interpretation?
displacement

0 time

A The object slides along a flat surface. Then it slides forward down a smooth
incline plane, and then finally stops.
B The object is moving at constant velocity. Then it slows down and stops.
C The object is initially stationary. Then it moves backwards and then finally
stops.
D The object moves along a flat area, moves backwards down a smooth
incline plane, and then it keeps moving.

River Valley High School Pg 4 of 20 Year 6 H2 Physics 9646


Preliminary Examination 2010
4 An object has an initial velocity u. It is subjected to a constant acceleration a.
The force is not in the same direction as the initial velocity.
A vector diagram is drawn to find the final velocity v.

What is the length of side X of the vector diagram?

A v–u B v+u C at D u + at

5 The diagram shows two trolleys X and Y held stationary and connected by an
extended elastic cord. The mass of X is twice that of Y.

The trolleys are released at the same instant. They move towards each other and
stick together on impact. Just before the collision, the speed of X is 20 cm s–1.

What is the speed of Y after the collision?

A zero B 5.0 cm s–1 C 7.0 cm s–1 D 10 cm s–1

6 A 2.0 kg object moving at 10 m s−1 collides normally with a wall and bounces off
with half of its original kinetic energy. What is the magnitude of the impulse
applied by the wall?

A 5.9 N s B 14 N s C 34 N s D 50 N s

River Valley High School Pg 5 of 20 Year 6 H2 Physics 9646


Preliminary Examination 2010
7 A ball falls vertically and bounces on the ground.
Which of the following statements is true when the ball is in contact with the
ground?

A The force that the ball exerts on the ground is always less than the weight of
the ball.
B The force that the ball exerts on the ground is always equal to the weight of
the ball.
C The weight of the ball is always equal in magnitude and opposite in direction
to the force that the ground exerts on the ball.
D The force that the ball exerts on the ground is always equal in magnitude
and opposite in direction to the force the ground exerts on the ball.

8 A solid has density 4.0 g cm−3. What is the density of a liquid in which the solid
would float with one-fifth of its total volume above the liquid surface?

A 4.0 g cm−3 B 5.0 g cm−3 C 7.5 g cm−3 D 8.5 g cm−3

9 Two 20 g flatworms climb over a very thin wall, 10 cm high. One of the worms is
20 cm long, the other is wider and only 10 cm long. Determine the ratio of work
done against gravity by the longer worm to the shorter one when half of their
bodies are over the top of the wall.
thin wall
Me
too!
I am a
flatworm.

2 3
A B 1 C D 2
3 2

10 A cannon of mass 3000 kg fires a cannonball of mass 50 kg with a horizontal


velocity of 85 m s−1. Determine the kinetic energy of the cannon.

A 1.1 × 107 J B 1.8 × 105 J C 3.0 × 103 J D 50 J

River Valley High School Pg 6 of 20 Year 6 H2 Physics 9646


Preliminary Examination 2010
11 A turntable is rotating at a constant number of revolutions per second. A coin on
the turntable is stationary with respect to the turntable. The frictional force
between the coin and the surface of the turntable is F.
Which of the following graphs best represents the relation between F and its
distance r from the centre of the turntable?

A B
F F

r r

C D
F F

r r

River Valley High School Pg 7 of 20 Year 6 H2 Physics 9646


Preliminary Examination 2010
12 An aircraft flies along a horizontal circle of radius 10 km with a constant speed of
155 m s–1. What is the angle θ between its wings and the horizontal?
wings of
aircraft
θ

A 14° B 22° C 68° D 76°

13 The Earth’s gravitational field close to the surface of the Earth can be considered
uniform. Which of the following statements is not correct?

A The field lines are parallel to each other.


B The units of gravitational field strength are equivalent to m s–2.
C The gravitational force on an object is proportional to its height above the
Earth’s surface.
D The direction of the field lines is towards the Earth.

14 Two satellites A and B of the same mass are moving in circular orbits round the
Earth. The radius of A’s orbit is r and that of B’s orbit is 2r. Their kinetic energies
are TA and TB respectively.
Which of the following is true?

A TA = 2TB
B 1
TA = TB
2
C TA = 2 TB
D 1
TA = TB
2

River Valley High School Pg 8 of 20 Year 6 H2 Physics 9646


Preliminary Examination 2010
15 An object performs linear simple harmonic motion between P and Q with a period
T. X and Y are the midpoints between the equilibrium position O with P and Q
respectively as shown.

What is the minimum time required for the object to move from X to Y?

T T T T
A B C D
4 6 8 12

16 A sphere attached to a horizontal spring is oscillating on a smooth horizontal


surface at a frequency of 2.0 Hz. The amplitude of oscillation is 2.0 cm.
What is the speed of the sphere when it is at a distance 1.0 cm from equilibrium
position?
A 6.3 cm s–1 B 22 cm s–1 C 25 cm s–1 D 31 cm s–1

17 Two ideal gases X and Y, are contained in a cylinder at constant temperature.


The mass of the atoms of X is m and of Y is 4m.
Which one of the following is the correct value of the ratio
Average kinetic energy of atoms X
?
Average kinetic energy of atoms Y

A 1 B 2 C 4 D 16

River Valley High School Pg 9 of 20 Year 6 H2 Physics 9646


Preliminary Examination 2010
18 A sample of an ideal gas initially having internal energy U1 is allowed to expand
under constant temperature by absorbing heat Q1 and doing external work W.
Heat Q2 is then supplied to it, keeping the volume constant at its new value, until
the pressure returns to its original value. The final internal energy is U2.

Pressure

U1 U2

Volume

The increase in internal energy, U2 – U1 , is equal to

A 0 B Q 1 + Q2 C W + Q2 D Q2

19 The figure shows the shape at a particular instant of part of a transverse wave
travelling from left to right along a string.

Which statement about the motion of elements of the string at this instant is
correct?

A The speed of Q is higher than S.


B Both Q and R are moving upwards.
C The energy of P and S is entirely kinetic.
D The acceleration of P and R is a maximum.

River Valley High School Pg 10 of 20 Year 6 H2 Physics 9646


Preliminary Examination 2010
20 A source of sound of frequency 2300 Hz is placed several metres from a vertical
reflecting board. A microphone, connected to a cathode-ray oscilloscope, is
moved from A to B which are both positions of maximum intensity through ten
minimum intensity positions as shown.

If L is 0.75 m, what is the speed of sound in air?

A 310 m s–1 B 330 m s–1 C 350 m s–1 D 380 m s–1

21 A narrow beam of monochromatic light falls at normal incidence on a diffraction


grating. Second-order diffracted beams are formed at angles of 20° to the
original direction.
What is the number of diffracted beams produced by this grating?

A 5 B 6 C 10 D 11

22 X and Y are two identical conducting spheres separated by a distance d. X has a


charge +6 μC and Y has a charge –2 μC. The electric force between them is −F
(ie attractive). The spheres are touched together and are then returned to their
original separation d.
The force between them now is nearest to

F F
A +F B –F C + D −
3 3

River Valley High School Pg 11 of 20 Year 6 H2 Physics 9646


Preliminary Examination 2010
23 A positively charged sphere is released in a vacuum between two long parallel
plates carrying opposite charges. Which one of the following diagrams best
shows the path followed by the sphere?

A B

– + – +

– + – +

– + – +

C D

– + – +

– + – +

– + – +

24 Two cubes, X and Y are cut from the same block of metal. The linear dimension
R
of X is 2 times that of Y. What is the ratio X of the resistances between the
RY
opposite faces of X and of Y?

1
A B 1 C 2 D 4
2

River Valley High School Pg 12 of 20 Year 6 H2 Physics 9646


Preliminary Examination 2010
25 The electrical characteristic of a component is shown below.
I

Which graph below shows the way the resistance of the component varies with
applied voltage?

A B

C D

River Valley High School Pg 13 of 20 Year 6 H2 Physics 9646


Preliminary Examination 2010
26 The battery in the circuit below has negligible internal resistance.

2.0 Ω 2.0 Ω 2.0 Ω

I
6.0 V

The current I is

A 1.0 A B 2.0 A C 4.5 A D 9.0 A

27 In the circuit below, the 8.0 V cell has an internal resistance of 3.0 Ω. The
galvanometer reading is zero.
15 Ω
G

8.0 V R 2.0 V
r = 3.0 Ω

What is the resistance of R?

A 3.0 Ω B 5.0 Ω C 6.0 Ω D 9.0 Ω

River Valley High School Pg 14 of 20 Year 6 H2 Physics 9646


Preliminary Examination 2010
28 Two long current carrying conductors are placed perpendicular to each other.
The current flowing through one of the wires is 4.0 A upwards, while the current
through the other wire is 2.0 A towards the left.

X 3.0 m
4.0 A
3.0 m

2.0 A

What is the magnitude and direction of the resultant magnetic field at a point X,
which is 3.0 m perpendicularly away from both wires? Ignore the Earth's
magnetic field.
(Magnetic flux density at a distance d from a long straight conductor carrying
μ I
current I is B = o .)
2π d
A 1.33 x 10−7 T out of the plane of the page
B 4.00 x 10−7 T into the plane of the page
C 4.00 x 10−7 T out of the plane of the page
D 2.67 x 10−7 T into the plane of the page

29 A horizontal wire PQ of length 0.50 m and weight 0.50 N is placed at an angle 30°
to the magnetic field as shown below. The wire is balanced by the magnetic force
of the magnetic field of magnetic flux density of 1.0 T.
Top view
P
30°

What is the magnitude and direction of the current in the wire?

magnitude direction
A 1.2 A P to Q
B 2.0 A P to Q
C 1.2 A Q to P
D 2.0 A Q to P
River Valley High School Pg 15 of 20 Year 6 H2 Physics 9646
Preliminary Examination 2010
30 A loop of wire moves parallel to a long straight current carrying conductor as
shown.
v

I
Which of the following statements is correct?

A The direction of the induced current in the loop is clockwise.


B The magnitude of the induced current in the loop varies with the speed at
which the loop moves.
C The magnitude of the induced current in the loop varies with the magnitude
of the current in the long straight conductor.
D There is no induced current.

31 A metallic rod slides along a frictionless wire frame of width 1.5 m with a speed of
4.0 m s−1 across a magnetic field of field strength 5.0 T perpendicular to the plane
of the frame as shown.

. . . . . . . . .

. . . −1. . . . . .
R 4ms B 1.5 m
. . . . . . . . .

. . . . . . . . .

What is the direction of the induced current and the power dissipated by the
resistor R of resistance 12 Ω?

Direction of induced current Power dissipated


A Clockwise 2.5 W
B Anti clockwise 2.5 W
C Clockwise 75 W
D Anti clockwise 75 W

River Valley High School Pg 16 of 20 Year 6 H2 Physics 9646


Preliminary Examination 2010
32

Which of the following graphs below represents the variation of current I with time
t through XY of the circuit above?

A B

C D

33 When an electric kettle is connected to an a.c. source, the power output is half of
that when it is connected to a 10 V d.c. source. What is the peak voltage of the
a.c. source?

A 5.0 V B 7.0 V C 10 V D 14 V

River Valley High School Pg 17 of 20 Year 6 H2 Physics 9646


Preliminary Examination 2010
34 When electromagnetic radiation falls on a particular metal surface, photoelectrons
may be emitted. The variation of the maximum kinetic energy E of these
electrons with the frequency f of the radiation is shown in the figure below.

When the experiment is repeated using another metal with a smaller work
function, which graph best represent the variation of E with f of this metal (solid
line)?

A B

C D

35 A beam of light of wavelength λ is incident normal to a surface of area S and is


completely absorbed by the surface. The rate of photons arriving at the surface is
n. The pressure exerted on the surface by the light is

nhλ nh 2nhλ 2nh


A B C D
S λS S λS

River Valley High School Pg 18 of 20 Year 6 H2 Physics 9646


Preliminary Examination 2010
36 The x-ray spectrum obtained by bombarding a molybdenum target with electrons
is shown in the figure.

The two peaks Kα and K β are produced when the electrons in the lowest energy
level of the molybdenum atoms are knocked out by the incident electrons and
electrons in the next two higher energy levels of the atom made the transition to
the lowest energy level.
What is the energy difference of the two higher energy levels?

A 21 keV B 18 keV C 13 keV D 3 keV

37 Which of the following statements describes stimulated emission?


A An electron transiting from a higher energy level to a lower energy level
emitting a photon.
B An electron is emitted from an atom after being hit by a high energy photon.
C A photon incident on an electron causing the emission of an electron.
D A photon causing a transition of an electron emitting another photon of
exactly the same frequency.

River Valley High School Pg 19 of 20 Year 6 H2 Physics 9646


Preliminary Examination 2010
38 Which of the following statements is not true about the p-n junction?
A The holes from the p-type material and the electrons from the n-type
material diffuse and recombine, forming a region free of mobile charge
carriers.
B At equilibrium, the electric potential at the n-type material is higher than the
electric potential at the p-type material.
C Under forward-bias, the electrons from the n-type material move to the p-
type material, resulting in a current flow.
D Under reverse-bias, the electric potential at the p-type material is higher than
the electric potential at the n-type material.

39 The sketch graph shows how the binding energy per nucleon varies with the
nucleon number for naturally occurring nuclides.

binding energy per nucleon /


pJ per nucleon

nucleon number

156
What is the total binding energy of the nuclide 64 Gd ?

A 1.3 pJ B 1.4 pJ C 90 pJ D 203 pJ

40 A sample of a radioactive isotope of half-life t1/2 initially contains N atoms. Which


one of the following gives the number of atoms of this isotope that have decayed
after a time 3t1/2?

7 2 1 1
A N B N C N D N
8 3 3 8

END OF PAPER
River Valley High School Pg 20 of 20 Year 6 H2 Physics 9646
Preliminary Examination 2010
RIVER VALLEY HIGH SCHOOL
YEAR 6 PRELIMINARY EXAMINATION

H2 PHYSICS 9646
PAPER 2
13 SEP 2010
1 HOUR 45 MIN
CANDIDATE
NAME

CENTRE INDEX
NUMBER S NUMBER

CLASS 6
INSTRUCTIONS TO CANDIDATES
DO NOT OPEN THIS BOOKLET UNTIL YOU ARE TOLD TO DO SO.
Read these notes carefully.
Write your name, centre and index number and class in the spaces above.

Section A and Section B FOR EXAMINERS’ USE


Candidates answer on the Question Paper. Section A
Write in dark blue or black pen. 1 /7
You may use a soft pencil for any diagrams, graphs or rough
working. 2 /6
Do not use paper clips, highlighters, glue or correction fluid.
3 /8
Section A
Answer all questions. 4 /6
It is recommended that you spend about 1 hour 15 minutes on
this section. 5 /8
Section B 6 /7
Answer Question 8.
It is recommended that you spend about 30 minutes on this 7 /18
section.
8 /12
TOTAL /72
The number of marks is given in brackets [ ] at the end of each question or part question.
____________________________________________________________________________
This Question Paper consists of 22 printed pages.
River Valley High School Pg 1 of 22 Year 6 H2 Physics 9646
Preliminary Examination 2010
Data

speed of light in free space, c = 3.00 × 108 m s–1

permeability of free space, μO = 4 π × 10–7 H m–1

permittivity of free space, εO = 8.85 × 10–12 F m–1

(1/(36 π )) × 10–9 F m–1

elementary charge, e = 1.60 × 10–19 C

the Planck constant, h = 6.63 × 10–34 J s

unified atomic mass constant, u = 1.66 × 10–27 kg

rest mass of electron, me = 9.11 × 10–31 kg

rest mass of proton, mp = 1.67 × 10–27 kg

molar gas constant, R = 8.31 J K–1 mol–1

the Avogadro constant, NA = 6.02 × 1023 mol–1

the Boltzmann constant, k = 1.38 × 10–23 J K–1

gravitational constant, G = 6.67 × 10–11 N m2 kg–2

acceleration of free fall, g = 9.81 m s–2

River Valley High School Pg 2 of 22 Year 6 H2 Physics 9646


Preliminary Examination 2010
Formulae

1 2
uniformly accelerated motion, s = ut + at
2
v 2 = u 2 + 2as

work done on/by a gas, W = pΔV

hydrostatic pressure, p = ρ gh

GM
gravitational potential, φ=−
r

displacement of particle in s.h.m., x = xO sin ωt

velocity of particle in s.h.m., v = v O cos ωt

= ±ω (x 2
O − x2 )
3
mean kinetic energy of a molecule of an ideal gas, E= kT
2

resistors in series, R = R1 + R2 + K

resistors in parallel, 1/ R = 1/ R1 + 1/ R2 + K

Q
electric potential, V=
4πε O r

alternating current/voltage, x = xO sin ωt

transmission coefficient, T ∝ exp ( −2kd )

8π 2 m (U − E )
where k =
h2

radioactive decay, x = xO exp ( −λ t )

0.693
λ=
decay constant, t1
2

River Valley High School Pg 3 of 22 Year 6 H2 Physics 9646


Preliminary Examination 2010
Section A

Answer all questions.


It is recommended that you spend about 1 hour 15 minutes on this section.

1 (a) A lecturer holds a flat S$2 note just above a student’s open fingers as shown in
Fig. 1.1.

specimen
Fig. 1 1

He challenges the students that whoever can catch the S$2 note when he
releases it can keep it. Explain quantitatively whether any student would be
able to catch the note when the lecturer releases it.

[3]

River Valley High School Pg 4 of 22 Year 6 H2 Physics 9646


Preliminary Examination 2010
(b) A lecturer sees a student who owes him homework a distance d away. The
lecturer immediately moves towards the student with a constant velocity vL.
The student sees the lecturer moving towards him to seconds later and starts
moving away in the same direction at a constant velocity vS.

Write down an expression for the time taken t for the lecturer to catch up with
the student from the instant he sees the student. Show your derivation
clearly.

[4]

River Valley High School Pg 5 of 22 Year 6 H2 Physics 9646


Preliminary Examination 2010
2 Jane, whose mass is 50.0 kg, needs to swing across a river (having width D) filled
with man-eating crocodiles to save Tarzan from danger. She must swing on a vine
into a wind exerting a constant horizontal force F. The vine has a length L and
initially makes an angle θ with the vertical (Fig. 2.1). Take D = 50.0 m, F = 110 N,
L = 40.0 m, and θ = 50.0°.

φ
L

Wind Jane
F
Tarzan

Fig 2.1

(a) Show that the angle φ is 28.9°.

[1]

River Valley High School Pg 6 of 22 Year 6 H2 Physics 9646


Preliminary Examination 2010
(b) Calculate the minimum speed Jane needs to begin her swing in order for her to
just reach Tarzan.

minimum speed of Jane = ………………………… m s–1 [3]

(c) Once the rescue is complete, Tarzan and Jane must swing back across the
river. With what minimum speed must they begin their swing if Tarzan has a
mass of 80.0 kg?

minimum speed = ………………………… m s–1 [2]

River Valley High School Pg 7 of 22 Year 6 H2 Physics 9646


Preliminary Examination 2010
3 A small immersion electrical heater, operating at a constant power, was used to heat
64 g of water in a thin plastic cup. The mass of the cup was negligible. The
temperature of the water was recorded at regular intervals for 30 minutes and a
graph of temperature against time is drawn as shown in Fig. 3.1 below.

60

Temperature/ /°C
Temperature °C

50

40

30

20
0 5 10 15 20 25 30
Time / minutes
Time / minutes
Fig. 3.1

(a) (i) Use the graph to determine the initial rate of temperature rise of the
water.

rate of temperature rise = ………………………… °C min–1 [2]

(ii) The specific heat capacity of water is 4200 J kg–1 K–1. Determine the
rate at which energy was supplied to the water by the heater.

rate of energy supply = ………………………… J min–1 [2]


River Valley High School Pg 8 of 22 Year 6 H2 Physics 9646
Preliminary Examination 2010
(b) After 26 minutes the rate of temperature rise became very small. Explain why.

……………………………………………………………………………………………

……………………………………………………………………………………… [1]

(c) The experiment was repeated using the same mass of water in a thick ceramic
mug. The initial temperature of the water was the same and the water was
heated for the same length of time.

(i) On Fig 3.1, sketch a possible graph of temperature against time for the
water in the thick ceramic mug. [1]

(ii) Explain your reasoning for your graph.

……………………………………………………………………………………

……………………………………………………………………………... [2]

River Valley High School Pg 9 of 22 Year 6 H2 Physics 9646


Preliminary Examination 2010
4 (a) A and B are two identical conducting spheres each carrying a charge +Q. They
are placed in a vacuum with their centres distance d apart as shown in Fig. 4.1.

A B

Fig. 4.1

Explain why the force F between them is not given by the expression
Q2
F=
4πε 0d 2

……………………………………………………………………………………………

……………………………………………………………………………………………

……………………………………………………………………………………… [2]

(b) Electric fields and magnetic fields may be represented by lines of force. Fig. 4.2
shows some lines of force.

A B

Fig. 4.2

(i) State whether the field strength at the vicinity of A and at the vicinity of B
is constant, increasing or decreasing when measured in the direction
from A towards B.

at A: ……………………………………………………………………….. [1]

at B: ……………………………………………………………………….. [1]

River Valley High School Pg 10 of 22 Year 6 H2 Physics 9646


Preliminary Examination 2010
(ii) Explain why the field lines can never touch or cross.

……………………………………………………………………………………

……………………………………………………………………………………

……………………………………………………………………………… [2]

River Valley High School Pg 11 of 22 Year 6 H2 Physics 9646


Preliminary Examination 2010
5 (a) Distinguish between electromotive force and potential difference.

……………………………………………………………………………………………

……………………………………………………………………………………………

……………………………………………………………………………………………

……………………………………………………………………………………… [2]

(b) An electric hotplate is designed to operate on a power supply of 240 V has two
coils of wire of resistivity of 9.8 × 10–7 Ω m. Each coil of wire has a length of
16 m of cross-sectional area 0.20 mm2.

(i) For one of the coils, calculate


1. its resistance,

resistance = ………………………… Ω

2. the power dissipation when a 240 V supply is connected across it.

power = ………………………… W [4]

River Valley High School Pg 12 of 22 Year 6 H2 Physics 9646


Preliminary Examination 2010
(ii) Fig. 5.1 shows how the two coils can be connected to operate at
different powers.
240 V

B
C

Fig. 5.1

On Fig. 5.2, fill up the table with “ON” or “OFF” to obtain the lowest and
highest levels of operating power.

switch A switch B switch C


Lowest

Highest

Fig. 5.2
[ 2]

River Valley High School Pg 13 of 22 Year 6 H2 Physics 9646


Preliminary Examination 2010
6 The isotope Iron-59 is a β-emitter with a half-life of 45 days. In order to estimate
engine wear, an engine component is manufactured from non-radioactive iron
throughout which the isotope Iron-59 has been uniformly distributed. The mass of
the component is 2.4 kg and its initial activity is 8.5 × 107 Bq.

The component is installed in the engine 60 days after manufacture of the


component, and then the engine is tested for 30 days. During the testing period, any
metal worn off the component is retained in the surrounding oil. Immediately after
the test, the oil is found to have a total activity of 880 Bq.

Calculate
(a) the decay constant for the isotope Iron-59,

decay constant = ………………………… s–1 [2]

(b) the total activity of the component when it was installed,

activity = ………………………… Bq [2]

(c) the mass of iron worn off the component during the test.

mass of iron = ………………………… g [3]

River Valley High School Pg 14 of 22 Year 6 H2 Physics 9646


Preliminary Examination 2010
7 Read the following passage and answer the questions which follow.
Gamma-ray detectors
In radioactive decay, γ-rays may be emitted. γ-rays emitted from different
substances may have different energies.
When γ-rays photons are incident on a sodium iodide crystal, some of the photons
may be absorbed by the crystal. When a photon is absorbed, it causes the emission
of a small pulse of light known as a scintillation. These scintillations may be
detected and converted into electrical pulses by a photomultiplier tube which, when
coupled to a suitable counter, enables γ-ray photons to be counted. This is
illustrated in Fig 7.1.

Fig 7.1
The crystals used in such detectors may be of various shapes. Fig 7.2 shows one
particular shape of crystal which is a solid cylinder.
The γ-ray source S is placed on the axis of the crystal, a distance x in front of one
face. The source S is assumed to emit photons uniformly in all directions.

Fig 7.2
Not all of the γ-ray photons emitted by the source will be absorbed by the crystal.
The efficiency Q of a detector is defined by the equation
number of photons producing scintillations in the crystal
Q=
total number of photons emitted by the source

(a) By reference to the passage, explain what is meant by a scintillation.

…………………………………………………………………………………….... [1]

(b) The passage refers to γ-ray photons. Explain the underlined terms.

……………………………………………………………………………………………

…………………………………………………………………………………….... [2]

River Valley High School Pg 15 of 22 Year 6 H2 Physics 9646


Preliminary Examination 2010
Fig 7.3 illustrates the variation of the efficiency Q with the γ-ray photon energy E.
Curves are shown for various values of x, the distance of the γ-ray source from the
crystal.

E / 10−15 J
Fig 7.3

(c) With reference to Fig 7.3 and considering γ-ray photons of energy 10 × 10−15 J,
complete Fig. 7.4 with corresponding values of Q and x for γ-ray photons of this
energy.

Q x / cm

Fig. 7.4 [2]

River Valley High School Pg 16 of 22 Year 6 H2 Physics 9646


Preliminary Examination 2010
(d) (i) Use your values in Fig. 7.4, draw a graph of Q against x on Fig. 7.5.

0.5

0.4

0.3
Q

0.2

0.1

0
0 1.0 2.0 3.0 4.0 5.0
x / cm
Fig. 7.5 [3]

(ii) Hence determine the rate of change of Q with x when x = 0.5 cm.

rate of change of Q with x = ………………………… cm–1 [3]

(e) (i) By reference to Fig 7.3 or your graph on Fig. 7.5, suggest a maximum
possible value of the efficiency Q.

Qmax = ………………………… [1]

River Valley High School Pg 17 of 22 Year 6 H2 Physics 9646


Preliminary Examination 2010
(ii) By reference to Fig 7.2 and the definition of efficiency, give a reason for
this maximum value.

……………………………………………………………………………….......

……………………………………………………………………………….......

……………………………………………………………………………….......

……………………………………………………………………………… [2]

(f) Using Fig 7.3, suggest why, for any one value of x,

(i) the efficiency is constant at low photon energies,

…………………………………………………………………………………....

…………………………………………………………………………………....

…………………………………………………………………………………....

……………………………………………………………………………… [2]

(ii) the efficiency decreases with increasing photon energy.

…………………………………………………………………………………....

…………………………………………………………………………………....

…………………………………………………………………………………....

……………………………………………………………………………… [2]

River Valley High School Pg 18 of 22 Year 6 H2 Physics 9646


Preliminary Examination 2010
Section B

It is recommended that you spend about 30 minutes on this section.

8 The Vikings are said to have used the polarisation of the sky with “special sun
stones” to navigate. Honeybees do a “waggle dance”, oriented to the polarised light
of the sky, to communicate the location of a food source to other bees in the hive.
We benefit from many technological applications of polarised light and polarising
filters; one example is the liquid-crystal displays (LCDs) on digital watches and
calculators.

“Special sun stones” Waggle Dance LCD Watch

You are to design an experiment to investigate the relationship between the intensity
of the light passing through a pair of polarising filters and the angle between their
planes of polarisation.

You may assume that the following apparatus is available, together with any other
standard equipment which may be found in a school or college science laboratory.

♦ A pair of polarising filters


♦ A light source
♦ LDR and ohmmeter together with a calibration curve enabling resistance
measurements to be converted to light intensity in a unit called lux
♦ Light shield

You should draw a diagram showing how the chosen apparatus would be arranged.
In your account you should pay particular attention to

(a) the procedure to be followed,

(b) the readings that should be taken,

(c) the method of determining the intensity of light,

(d) the control of variables,

(e) any precautions you would take which may improve the accuracy of your
experiment.

River Valley High School Pg 19 of 22 Year 6 H2 Physics 9646


Preliminary Examination 2010
…………………………………………………………………………………………………..

…………………………………………………………………………………………………..

…………………………………………………………………………………………………..

…………………………………………………………………………………………………..

…………………………………………………………………………………………………..

…………………………………………………………………………………………………..

…………………………………………………………………………………………………..

…………………………………………………………………………………………………..

…………………………………………………………………………………………………..

…………………………………………………………………………………………………..

…………………………………………………………………………………………………..

…………………………………………………………………………………………………..

…………………………………………………………………………………………………..

…………………………………………………………………………………………………..

…………………………………………………………………………………………………..

…………………………………………………………………………………………………..

…………………………………………………………………………………………………..

River Valley High School Pg 20 of 22 Year 6 H2 Physics 9646


Preliminary Examination 2010
…………………………………………………………………………………………………..

…………………………………………………………………………………………………..

…………………………………………………………………………………………………..

…………………………………………………………………………………………………..

…………………………………………………………………………………………………..

…………………………………………………………………………………………………..

…………………………………………………………………………………………………..

…………………………………………………………………………………………………..

…………………………………………………………………………………………………..

…………………………………………………………………………………………………..

…………………………………………………………………………………………………..

…………………………………………………………………………………………………..

…………………………………………………………………………………………………..

…………………………………………………………………………………………………..

…………………………………………………………………………………………………..

…………………………………………………………………………………………………..

…………………………………………………………………………………………………..

…………………………………………………………………………………………………..

…………………………………………………………………………………………………..

…………………………………………………………………………………………………..

…………………………………………………………………………………………………..

…………………………………………………………………………………………………..

…………………………………………………………………………………………………..

…………………………………………………………………………………………………..

…………………………………………………………………………………………………..

…………………………………………………………………………………………………..
River Valley High School Pg 21 of 22 Year 6 H2 Physics 9646
Preliminary Examination 2010
…………………………………………………………………………………………………..

…………………………………………………………………………………………………..

…………………………………………………………………………………………………..

…………………………………………………………………………………………………..

…………………………………………………………………………………………………..

…………………………………………………………………………………………………..

…………………………………………………………………………………………………..

…………………………………………………………………………………………………..

…………………………………………………………………………………………………..

…………………………………………………………………………………………………..

…………………………………………………………………………………………………..

…………………………………………………………………………………………………..

…………………………………………………………………………………………………..

…………………………………………………………………………………………………..

…………………………………………………………………………………………………..

…………………………………………………………………………………………………..

…………………………………………………………………………………………………..

…………………………………………………………………………………………………..

…………………………………………………………………………………………………..

…………………………………………………………………………………………………..

…………………………………………………………………………………………………..

…………………………………………………………………………………………………..

…………………………………………………………………………………………………..

……………………………………………………………………………………............ [12]

END OF PAPER

River Valley High School Pg 22 of 22 Year 6 H2 Physics 9646


Preliminary Examination 2010
RIVER VALLEY HIGH SCHOOL
YEAR 6 PRELIMINARY EXAMINATION

H2 PHYSICS 9646
PAPER 3
17 SEP 2010
2 HOUR
CANDIDATE
NAME

CENTRE INDEX
NUMBER S NUMBER

CLASS 6
INSTRUCTIONS TO CANDIDATES
DO NOT OPEN THIS BOOKLET UNTIL YOU ARE TOLD TO DO SO.
Read these notes carefully.
Write your name, centre and index number and class in the spaces above.

Candidates answer on the Question Paper. FOR EXAMINERS’ USE


Section A
Write in dark blue or black pen.
You may use a soft pencil for any diagrams, graphs or rough 1 /7
working.
Do not use paper clips, highlighters, glue or correction fluid. 2 /11
Section A 3 /12
Answer all questions.
4 /10
Section B
Section B
Answer any two questions.
5 /20
6 /20
7 /20
TOTAL /80
The number of marks is given in brackets [ ] at the end of each question or part question.
____________________________________________________________________________
This Question Paper consists of 20 printed pages.

River Valley High School Pg 1 of 20 Year 6 H2 Physics 9646


Preliminary Examination 2010
Data

speed of light in free space, c = 3.00 × 108 m s–1

permeability of free space, μO = 4 π × 10–7 H m–1

permittivity of free space, εO = 8.85 × 10–12 F m–1

(1/(36 π )) × 10–9 F m–1

elementary charge, e = 1.60 × 10–19 C

the Planck constant, h = 6.63 × 10–34 J s

unified atomic mass constant, u = 1.66 × 10–27 kg

rest mass of electron, me = 9.11 × 10–31 kg

rest mass of proton, mp = 1.67 × 10–27 kg

molar gas constant, R = 8.31 J K–1 mol–1

the Avogadro constant, NA = 6.02 × 1023 mol–1

the Boltzmann constant, k = 1.38 × 10–23 J K–1

gravitational constant, G = 6.67 × 10–11 N m2 kg–2

acceleration of free fall, g = 9.81 m s–2

River Valley High School Pg 2 of 20 Year 6 H2 Physics 9646


Preliminary Examination 2010
Formulae

1 2
uniformly accelerated motion, s = ut + at
2
v 2 = u 2 + 2as

work done on/by a gas, W = pΔV

hydrostatic pressure, p = ρ gh

GM
gravitational potential, φ=−
r

displacement of particle in s.h.m., x = xO sin ωt

velocity of particle in s.h.m., v = v O cos ωt

= ±ω (x 2
O − x2 )
3
mean kinetic energy of a molecule of an ideal gas, E= kT
2

resistors in series, R = R1 + R2 + K

resistors in parallel, 1/ R = 1/ R1 + 1/ R2 + K

Q
electric potential, V=
4πε O r

alternating current/voltage, x = xO sin ωt

transmission coefficient, T ∝ exp ( −2kd )

8π 2 m (U − E )
where k =
h2

radioactive decay, x = xO exp ( −λ t )

0.693
λ=
decay constant, t1
2

River Valley High School Pg 3 of 20 Year 6 H2 Physics 9646


Preliminary Examination 2010
Section A (40 marks)

Answer all the questions in this section.

1 (a) Fig. 1.1 shows two small dots P and P’ on a printed signboard at a distance L
away from the eyes of a reader.
P

θ s

Eye P’
L

Fig.1.1

The distance between the dots is s. Write down an expression for the angular
separation θ between the dots, in terms of L and s.

…………………………………………………………………………………….... [1]

(b) The visual acuity α of the eye is the minimum angular separation of two
equidistant points which can just be distinguished by the eye.

Experiments show that most people just failed to see the division of a millimetre
on a ruler when the ruler is about 2.0 m away from the observer. Estimate the
visual acuity α of the average person.

visual acuity = ………………………… rad [2]

(c) For the average person, the least distance of distinct vision d is about 25 cm.
Any nearer object will appear blur to the person. Estimate the thickness of the
finest line of a printed drawing that can be distinguished clearly by an average
reader.

thickness of line = ………………………… mm [2]

River Valley High School Pg 4 of 20 Year 6 H2 Physics 9646


Preliminary Examination 2010
(d) Some laser printers have a print resolution of 500 dots per inch. If a line
consisting of alternate white and black dots is printed by such a printer, can the
individual dots be distinguished?
[1 inch = 25.4 mm]

[2]

River Valley High School Pg 5 of 20 Year 6 H2 Physics 9646


Preliminary Examination 2010
2 (a) Define moment of a force.

……………………………………………………………………………………………

……………………………………………………………………………………… [1]

(b) A person supports a load of 20 N in his hand as shown in Fig 2.1. The system
of the hand and load is represented by Fig 2.2. The rod represents the forearm
and T represents the tension exerted in the biceps. The forearm weighs 65 N.

(i) Show that the tension T in the biceps is 410 N.

[2]

River Valley High School Pg 6 of 20 Year 6 H2 Physics 9646


Preliminary Examination 2010
(ii) Determine the magnitude and direction of the force acting at the elbow.

force acting at the elbow = ………………………… N

direction the force: ………………………… [4]

(c) A karate expert can split a stack of bricks by bringing her arm and hand swiftly
against the bricks with considerable speed. Using Newton’s laws of motion,
explain why she has to execute the karate strike very quickly.

……………………………………………………………………………………………

……………………………………………………………………………………………

……………………………………………………………………………………………

……………………………………………………………………………………………

……………………………………………………………………………………………

……………………………………………………………………………………………

……………………………………………………………………………………………

……………………………………………………………………………………………

……………………………………………………………………………………………

…………………………………………………………………………….............. [4]

River Valley High School Pg 7 of 20 Year 6 H2 Physics 9646


Preliminary Examination 2010
3 A mass P, 80 g, is attached to the free end of a horizontal spring on a smooth
surface. The spring-mass system is set into simple harmonic motion by pulling P to
the right of the equilibrium position and is released from rest as shown in Fig. 3.1.

Fig. 3.1
If the air resistance on P is negligible, the variation of the velocity v of P with
displacement x is shown in Fig. 3.2. Vectors to the right are taken to be positive.

Fig. 3.2

(a) For the motion of P, determine

(i) the amplitude,

amplitude = ………………………… mm [1]

River Valley High School Pg 8 of 20 Year 6 H2 Physics 9646


Preliminary Examination 2010
(ii) the frequency.

frequency = ………………………… Hz [2]

(b) If the air resistance on P is not negligible, sketch on Fig. 3.2 the variation of the
velocity of P with displacement x. [2]

(c) A periodic force is now exerted on the spring-mass system. When the periodic
force is at a certain frequency, P is in resonance.

(i) Explain what is meant by the term resonance.

……………………………………………………………………………………

…………………………………………………………………………….... [1]

(ii) 1. Using energy consideration, explain why the total energy of the
system increases to another value at steady state.

……………………………………………………………………………….

……………………………………………………………………………….

……………………………………………………………………………….

………………………………………………………………………………... [2]

2. Given that the total energy of the spring-mass system at steady


state is doubled. Determine the new maximum speed of P.

maximum speed of P = ………………………… m s–1 [2]

(iii) On Fig. 3.2, sketch the variation of the velocity of P, at resonance, with
displacement x. [2]

River Valley High School Pg 9 of 20 Year 6 H2 Physics 9646


Preliminary Examination 2010
4 Fig. 4 shows a pair of identical loudspeakers A and B placed 2.00 m apart and
emitting coherent sound waves of frequency 470 Hz. An observer walks from X to Y.
The perpendicular distance between the sources and XY is 12.0 m. As he walks, he
hears sound of maximum intensity at P, followed by minimum intensity at Q and the
next maximum intensity at R. R is 4.50 m away from P.

Fig. 4

(a) Explain why the observer hears sound of maximum and minimum intensity as
he moves from X to Y.

……………………………………………………………………………………………

……………………………………………………………………………………………

……………………………………………………………………………………………

……………………………………………………………………………………… [2]

(b) (i) AR is 12.5 m, show that BR is 13.2 m to 3 significant figures.

[1]

(ii) Determine the wavelength of the sound.

wavelength = ………………………… m [2]


River Valley High School Pg 10 of 20 Year 6 H2 Physics 9646
Preliminary Examination 2010
(iii) Determine the speed of the sound.

speed of sound = ………………………… m s–1 [2]

(c) The power of the loudspeakers A and B are identical. Suggest why the
intensity at Q is not zero.

…………………………………………………………………………………………...

…………………………………………………………………………………………...

…………………………………………………………………………………………...

…………………………………………………………………………………………...

…………………………………………………………………………………………...

……………………………………………………………………………………… [3]

River Valley High School Pg 11 of 20 Year 6 H2 Physics 9646


Preliminary Examination 2010
Section B (40 marks)

Answer two questions in this section.

5 (a) Define gravitational potential at a point in a gravitational field and state its unit.

……………………………………………………………………………………………

……………………………………………………………………………………………

……………………………………………………………………………………… [2]

(b) Fig. 5.1 shows the variation of gravitational potential between the surface of
Moon and the surface of Earth along the line joining the centres.
X Y
Earth

P Moon

– 1.3

– 3.9

– 62.3

potential/106 J kg–1

Fig. 5.1

The following data is required in answering the question.

mass of the Earth 5.98 × 1024 kg


mass of the Moon 7.35 × 1022 kg
distance from the centre of the Moon to the centre 3.84 × 108 m
of the Earth

(i) State how the resultant gravitational field strength can be deduced from
Fig. 5.1.

………………………………………………………………………………........

……………………………………………………………………………… [2]
River Valley High School Pg 12 of 20 Year 6 H2 Physics 9646
Preliminary Examination 2010
(ii) State the gravitational field at point P.

……………………………………………………………………………… [1]

(iii) Hence, or otherwise, determine distance X.

distance X = ………………………… m [3]

(iv) A rocket of mass 2.7 × 106 kg on a mission to the Moon is to be


launched from Earth. In order to reach the surface of the Moon, the
rocket must be launched with a minimum speed.

1. Using Fig. 5.1, determine this minimum speed. Explain your


working clearly.

[4]

2. With this minimum speed, calculate the speed at which the rocket
will land on the Moon’s surface.

landing speed on the Moon = ………………………… m s–1 [2]


River Valley High School Pg 13 of 20 Year 6 H2 Physics 9646
Preliminary Examination 2010
(c) The Moon is a natural satellite of the Earth. It can be assumed that the Moon
travels at a constant speed around the Earth in a circular path, with the Earth at
the centre of the circle.

(i) Using Newton’s laws of motion, explain why an object travelling in a


circle with constant speed has an acceleration. State the direction of
this acceleration.

……………………………………………………………………………………

……………………………………………………………………………………

……………………………………………………………………………………

……………………………………………………………………………………

……………………………………………………………………………………

……………………………………………………………………………… [3]

(ii) Show that orbital period T of a satellite and its distance r from the Earth
is given by
4π 2 3
2
T = r
GM E
where G is the gravitational constant, ME is the mass of the Earth.

[3]

River Valley High School Pg 14 of 20 Year 6 H2 Physics 9646


Preliminary Examination 2010
6 Fig 6.1 shows a system used by an engineer to determine the rate of revolution of a
rotating axle.

Fig 6.1

Four small bar magnets are embedded in the axle as shown. The N pole of each
magnet is towards the outside of the axle. A voltage is produced between the
terminals of a coil placed close to the rotating axle. The voltage produced is
monitored using an oscilloscope. The waveform produced is shown in Fig 6.2.

Fig 6.2
The Y gain setting = 5 mV cm−1
The time-base setting = 10 ms cm−1

River Valley High School Pg 15 of 20 Year 6 H2 Physics 9646


Preliminary Examination 2010
(a) Determine the number of revolutions made by the axle in one minute.

number of revolutions per minute = ………………………… min−1 [3]

(b) (i) State the laws of electromagnetic induction and use them to explain the
shape of the voltage pulses produced.

…………………………………………………………………………………….

…………………………………………………………………………………….

…………………………………………………………………………………….

…………………………………………………………………………………….

…………………………………………………………………………………….

…………………………………………………………………………………….

…………………………………………………………………………………….

…………………………………………………………………………………….

…………………………………………………………………………………….

…………………………………………………………………………………….

………………………………………………………………………………. [6]

River Valley High School Pg 16 of 20 Year 6 H2 Physics 9646


Preliminary Examination 2010
(ii) The coil has 350 turns. Determine the maximum rate of change of flux
through the coil and state its units.

maximum rate of change of flux = …………………………

units = ………………………… [4]

(c) State and explain what would be observed if the direction of revolution is
changed.

……………………………………………………………………………………………

……………………………………………………………………………………………

……………………………………………………………………………………... [2]

(d) On Fig 6.2, draw the waveform that shows the changes you would expect to
see when the rate of revolution of the axle is doubled. Label the waveform E.
[3]

(e) It is often said that the very act of measuring affects the measurement itself.
Comment on this statement with reference to your answer in (a).

……………………………………………………………………………………………

……………………………………………………………………………………………

……………………………………………………………………………………………

……………………………………………………………………………………... [2]

River Valley High School Pg 17 of 20 Year 6 H2 Physics 9646


Preliminary Examination 2010
7 Fig. 7.1 shows a simplified representation of the 5 lowest energy levels of doubly
ionised lithium ( Li 2+ ) that has only one electron.

Fig. 7.1

(i) Explain how emission spectral lines provide the evidence for the
existence of discrete energy levels in an atom.

……………………………………………………………………………………

……………………………………………………………………………………

……………………………………………………………………………………

……………………………………………………………………………………

……………………………………………………………………………………

……………………………………………………………………………… [3]

(ii) Explain why the ionised lithium vapour must be heated in order to
produce an emission spectrum.

……………………………………………………………………………………

……………………………………………………………………………… [1]

River Valley High School Pg 18 of 20 Year 6 H2 Physics 9646


Preliminary Examination 2010
(b) Considering transitions between only these levels,

(i) determine the wavelengths of the spectral transition that produce the
shortest and longest wavelength.

shortest wavelength = ………………………… m

longest wavelength = ………………………… m [3]

(ii) state the number of emission spectral lines that can be produced by
transitions among these levels.

……………………………………………………………………………… [1]

(iii) sketch the emission spectrum of a gas consisting of these ions on


Fig.7.2. Use vertical lines to denote the relative positions of the spectral
lines.

Fig. 7.2 [4]

(c) The electronic configuration of a lithium atom ( 73 Li ) is as shown in Fig. 7.3.

Fig. 7.3
The ionisation energies of a lithium atom are:
• first ionisation energy – 5.42 eV
• second ionisation energy – 76.0 eV
River Valley High School Pg 19 of 20 Year 6 H2 Physics 9646
Preliminary Examination 2010
(i) 1. Explain what is meant by the term ionisation energy.

............................................................................................................

………………………………………………………………………… [1]

2. State the value of the third ionisation energy.

………………………………………………………………………… [1]

(ii) The work function of lithium metal is less than 3 eV. Explain why the
ionisation energies of an atom are always higher than the work function
of the metal of the same element.

……………………………………………………………………………………

……………………………………………………………………………………

……………………………………………………………………………………

……………………………………………………………………………………

……………………………………………………………………………… [2]

(iii) The ionisation energy E of an atom can be used to estimate the radius
of the atom. Using the uncertainty principle, estimate the radius of a
lithium atom.

[4]

END OF PAPER

River Valley High School Pg 20 of 20 Year 6 H2 Physics 9646


Preliminary Examination 2010
CONFIDENTIAL

River Valley High School


2010 Year 6 Preliminary Examination
H2 Physics

Answer Key and Solutions

Paper 1

No. Answer Key No. Answer Key No. Answer Key No. Answer Key
1 C 11 A 21 D 31 D
2 D 12 A 22 C 32 B
3 C 13 C 23 B 33 C
4 C 14 A 24 A 34 C
5 A 15 B 25 B 35 B
6 C 16 B 26 D 36 D
7 D 17 A 27 C 37 D
8 B 18 D 28 A 38 D
9 A 19 D 29 B 39 D
10 C 20 C 30 D 40 A

CONFIDENTIAL 1
CONFIDENTIAL

River Valley High School


2010 Year 6 Preliminary Examination
H2 Physics

Paper 2

Qn No. Marking Scheme


1(a) Length of S$2 = 12.7 cm (10 ~ 15 cm),
Human reaction time = 0.3 s (0.1 ~ 0.5 s)

u = 0, s = 0.127 m, a = 9.81 m s−2


s = ut + ½ at2 Î 0.127 = 0 + 4.905 t2 Î t = 0.17 s

Since human reaction is around 0.3 s, more than 0.17 s, hence student B
would not be able to catch the note.

(b) Distance travelled by lecturer in time t = vL t

Distance travelled by student in time t = vs (t – to)

vL t = vs (t − to) + d

t = (d − vs to) / (vL− vs )

2(a) Lsin θ + Lsin φ = D


(40.0 sin 50.0° + 40.0 sin φ) = 50.0
φ = 28.9° (Shown)

(b) Vertical height Jane is above Tarzan, h =


40.0 cos φ − 40.0 cos 50.0° = 9.307 m

By conservation of energy,
1
MJvJ2 + MJgh = F × D (work done against friction)
2

Therefore, vJ = 6.12 m s−1

(c) By conservation of energy,


1
(MJ + MT)v2 + F × D = (MJ + MT)gh
2

Therefore, v = 9.90 m s−1

3(a) (i) To find gradient at start of graph

Range 0.07 – 0.18 K s–1 or 4.4 – 11.0 K min–1


(ii) Formula ∆Q/∆t = mc∆T/∆t used

Value for rate within acceptable range 18 – 50 W

(b) (rate of) energy lost to the surroundings OR due to evaporation[do not credit
boiling] approaches (rate of ) energy supply OR increases with temperature
difference.

CONFIDENTIAL 2
CONFIDENTIAL

(c) (i) Curve of reducing gradient starting at 20 °C, 0 s initially below given graph.
Reference of need to heat mug
Hence reduced rate of temperature rise
Reference to insulating properties of mug

4(a) Coulomb’s Law is only applicable to point charges.

In order for the formula to be valid, the distance d >> radius of the conducting
spheres.

(b) (i) A: constant

B: decreasing
(ii) The tangent of the field lines represents the direction of the force that a test
positive charge experiences.

Thus, if the field lines were to touch or cross, it implies that the force a test
positive charge experiences at that point is not unique which is not possible.

5(a) Electromotive force of a source is the amount of energy converted from


non-electrical forms to electrical energy when unit charge passes
through
the source.

The potential difference between two points in a circuit is one volt if one
joule of electrical energy is converted to other forms when one coulomb
of charge moves from one point to the other.

(b) (i) 16
(
1. R = 9.8 × 10 −7 )
( )
2
0.20 × 10−3
R = 78.4 Ω
( 240 )
2

2. P=
78.4
P = 735 W
(ii)
switch A switch B switch C
Lowest OFF ON OFF

Highest ON OFF ON

6(a) ln2
Decay constant λ =
t1/ 2

= 0.0154 day−1

(b) A = Ao e−λt = 8.5 × 107 e−0.0154(60)

A = 3.37 × 107 Bq

CONFIDENTIAL 3
CONFIDENTIAL

(c) Activity after 90 days

A = 8.5 × 107 e−0.0154(90) = 2.1256 × 107 Bq

880
Mass of iron worn off = × 2.4 kg
2.1256 × 107

= 9.94 × 10−5 kg

(a) small pulse of light

(b) γ-ray is electromagnetic radiation.


photon is a discrete packet of energy

(c) all values tabulated correctly


Q x / cm
0.05 5.0
0.10 3.5
0.15 2.5
0.21 1.8
0.30 1.0
0.38 0.5
0.50 0.0

(d) (i)
(ii) gradient = Δy/Δx
−0.19 to −0.21 cm−1

(e) (i) Qmax = 0.5


(ii) Since the source is assumed to emit photons uniformly in all
directions,
at most half of the photons emitted could be incident on a surface (and
that could only happen when x = 0 cm and if all the incident photons
have to be absorbed by the crystal).

crystal
source S

(f) (i) At low photon energies, most of the incident photons are absorbed
before they could penetrate the crystal (as they have lower penetrating
power).
Hence, the number of photons producing scintillations will remain
constant at about the number of photons incident on the crystal.

(ii) With increasing photon energy, more of the incident photons could
penetrate the crystal before they could be absorbed (because of
higher penetrating power).
Therefore, the number of photons producing scintillations will decrease

CONFIDENTIAL 4
CONFIDENTIAL

with increasing photon energy and Q decreases accordingly.

8 Aim: To investigate the relationship between the intensity of the light


passing through a pair of polarising filters and the angle between their
planes of polarisation.

Diagram
Small distance
Light Shield apart
Ohmmeter

. LDR
Light
Source

Polarising filters

I = k (cos θ )n
ln I = n ln(cos θ ) + ln k
y = mx + c

Use of cos θ rather then θ

Procedures
9 LDR connected to ohmmeter
9 Two polararising filters between light source and LDR shown
9 Measure θ with protractor [shown or stated]
9 A correct technique showing overlapping filters
9 Determining of intensity of light received by LDR through the calibration
graph.
9 Starting point when maximum (or minimum) LDR resistance θ = 90° (or θ
= 0°)

θ/° I / lux ln(I / lux) ln (cos θ)


0
10
30
50
70

Relevant experimental precautions:

CONFIDENTIAL 5
CONFIDENTIAL

Light shield around LDR

Control of variables
9 Distance of LDR to polarising filters, light source to polarising filters,
distant between the polarising filters kept constant.
9 Distance between polarising filters kept small
9 Repeating of experiment
9 Same source
9 Aligned polarisers, source and LDR

CONFIDENTIAL 6
CONFIDENTIAL

River Valley High School


2010 Year 6 Preliminary Examination
H2 Physics

Paper 3

Section A

1(a) θ = s/L

(b) α = distance between 2 lines/viewing distance

= 1.0 × 10-3 m / 2.0 m = 0.00050 radian

(c) thickness of the line = s, separation between the two edges of the line

visual acuity α ≤ s / d,
least distance of view s ≥ αd
≈ ( 0.00050 )( 25 cm )
= 0.0125 cm ≈ 0. 125 mm

(d) At 500 dots per inch, the size of each dot


= 25.4/500 = 0.0508 mm
The separation between two adjacent dots
= 2 × dot size
= 0.102 mm

At the least distance of distinct vision, the angular separation θ


= 0.102 mm /25 cm
= 0.000408 radians

As θ < α, the individuals are indistinguishable.

2(a) The torque τ (or the moment) of a force F about an axis is the product of
that force and the perpendicular distance from the line of action of the
force to the axis.

2(b) (i) Taking moments about the elbow,


Ty x 3.5 = 65 x 10 + 20 x 35
Ty = 385.7 N
T = Ty / cos 20° = 410 N

(ii) Tx = 385.7 / tan 20° = 140.4 N

∑ Fx = 0
Tx = Rx = 140.4 N

∑ Fy = 0
Ty = Ry + 65 + 20
Ry = 385.7 -65 – 20 = 300.7 N

Resultant force acting at elbow (pivot)

CONFIDENTIAL 7
CONFIDENTIAL

= Rx2 + Ry2 = 332 N

State direction

(c) N2L: rate of change of momentum is proportionally to the force applied

Larger change in momentum (higher speed)

Shorter time interval

N3L: force exerted on hand equal in magnitude and opposite in direction to


the force on wood

3(a) (i) 40 mm
(ii) v max = ω x0 ⇒ 0.60 = ( 2π f )( 0.040 )
f = 2.4 Hz

(b)

Spiral towards origin.


Clockwise spiral starting at (40, 0).

(c) (i) When the frequency of the periodic driving force is the same as the natural
frequency of the system, the amplitude of the system is at its maximum.
(ii) 1. The periodic force is transferring energy to the oscillating system and the
total energy of the system increases.

When the rate of energy transfer equal to the energy lost due to resistance,
the total energy reaches a steady value.

1 E ′ v ′2 2E v ′2
2. ET = mv 0 2 ⇒ ET ∝ v 0 2 ⇒ T = 0 2 ⇒ T = 0 2
2 ET v 0 ET 0.6
v 0′ = 0.85 m s−1
(iii) At resonance, frequency of P is near to its natural frequency of 2.4 Hz.
v 0 = ω x0 ⇒ v 0 ∝ x0 ⇒ x0′ = 2 x0 = 57 mm .

CONFIDENTIAL 8
CONFIDENTIAL

Maximum speed at 0.85 m s-1 and amplitude at


57 mm.
Elliptical shape.

4(a) The distances of a point on XY from the two sources are different.

When the difference in distance is integer multiples of wavelength apart, the


wave meet in phase and constructive interference occurs. This results in
maximum intensity.

When the difference in distance is half-integer multiples of wavelength apart,


the wave meet out-of-phase and destructive interference occurs. This results
in minimum intensity.

(b) (i) BR = 122 + 5.52 = 13.2 m


(ii) Since a maximum is detected at P and next at Q, the path difference from A
and B is one wavelength.
[Explanation must be shown to obtain 1 mark]

λ = BR − AR = 13.2 − 12.5 = 0.700 m


(iii) v = f λ = 470 × 0.700
v = 329 m s−1

(c) Distances of Q from A and B are different.

Since intensity at a position from a point source is inversely proportional to


the square of the distance between them, the intensities of the waves
arriving at Q from A and B are different.

Since intensity is proportional to the square of the amplitude, the amplitude


of the waves arriving from A and B will be different.

Q is a position with destructive interference without complete cancellation of


the waves occurs. Hence the intensity at Q is not zero.

Section B

5(a) Gravitational potential φ at a point is defined as the work done in bringing


unit mass from infinity to the point.

units: m s–2 OR J kg–1

(b) (i) dφ
g=−
dr
(ii) zero
(iii) GME GMM
=
(D − x )
2 2
x
⎛ ME ⎞ ME
⎜⎜ 1 + ⎟⎟ x = D
⎝ MM ⎠ MM

CONFIDENTIAL 9
CONFIDENTIAL

⎛ 5.98 × 1024 ⎞ 8 5.98 × 10


24
⎜ 1 + ⎟ x = 3.84 × 10
⎜ 7.35 × 1022 ⎟ 7.35 × 1022
⎝ ⎠

x = 3.46 × 108 m

(iv) 1. increase in GPE = loss in KE

Δφ = ⎡⎣ −1.3 − ( −62.3 ) ⎤⎦ × 106 = 61.0 × 106 J

ΔGPE = mΔφ

v = 1.10 × 104 m s–1

1 1
2. mv 2 + m ( −62.3 ) × 106 = mv ′2 + m ( −3.9 ) × 106
2 2

v ′ = 2.28 × 103 m s–1

(c) (i) direction of motion changing – N1L, there must be a resultant force

since speed in constant, resultant force must be perpendicular to the


direction of motion

direction of acceleration is towards the centre of the circular path

(ii) ∑ F = mr ω 2

GME m
2
= mr ω 2
r

2
GME m ⎛ 2π ⎞
2
= mr ⎜ ⎟
r ⎝T ⎠

4π 2 3
T2 = r
GM E

6(a) time between 2 magnets passing coil


= 71 – 76 ms
time for 1 revolution = 284 – 304 ms
(4 × their time between pulses)
number of revs per minute = 208 – 211

(b) (i) Faraday’s law:


Induced emf is directly proportional to rate of change of flux linkage
movement of magnet changes flux (linkage) with coil
voltage induced proportional to rate of change of flux (linkage) or use flux
cutting idea

CONFIDENTIAL 10
CONFIDENTIAL

Lenz’s law:
Lenz's law states that the direction of the induced e.m.f is such that it may
produce an effect that opposes the change causing it.

Magnetic N-pole approaches coil, magnetic effect of induced current


repelled (N), current is in one direction

Magnetic N-pole moves away from coil, magnetic effect of induced current
attracted (S), current flows in opposite direction
(ii) peak voltage = 1.5 × 5 mV = 7.5 (mV)

Δ(Nϕ )
induced emf =
Δt
or rate of change of flux = induced emf/N

2.14 × 10–5

Wb s-1 (T m2 s-1)

(c)

(d) higher peaks (similar amplitude + and -)


positive and negative peaks closer together
narrower/sharper peaks
sets of peaks closer together

(e) Yes, the magnetic effect of induced emf will provide a slight resistive force to
slow down the motion of the rotating axle.
The slower motion will induce an emf with peaks further apart from which the
measurement of rate of revolution would be taken to be slower than it was.

7(a) (i) Electromagnetic radiation is emitted as photons when electrons losses


energy in the atom. The energy of the photons is hf, f is the frequency of the
radiation.

Spectral lines are discrete with well-defined frequency. Therefore


electrons loss energy in discrete amount in atoms.

This is only possible if electrons in an atom transit between discrete energy


level.
(ii) Heating causes the sizable number of ions to be excited, i.e., electrons are
at higher energy levels.

When the electrons de-excite to lower energy states, photons with discrete
frequencies are emitted.

(b) (i) hc hc hc
E= ⇒λ = ⇒ λemission =
λ E ΔE

hc
Shortest wavelength = = 10.6 nm (3 s.f.)
(121.9 − 4.9 ) e

CONFIDENTIAL 11
CONFIDENTIAL

hc
Longest wavelength = = 460 nm (3 s.f.)
( 7.6 − 4.9 ) e
(ii) Number of spectral lines occurs between two levels, number of ways to
produce spectral lines is 5 C2

Number of spectral lines is 10.


(iii)
Approx. correct position for spectral lines (5 → 1) and (5 → 4) at 10 nm and
460 nm.

Approx. correct relative position for spectral lines (5 → 2) and (5 → 3).

3 other lines very close and to the right of (5 → 1) spectral line.

2 other lines slightly further and increasing wavelength to the right of (5 → 2)


spectral line.

(c) (i) 1. Ionisation energy is the energy required to remove (to infinity) the
outermost electron in an atom.

2. 122 eV

(ii) Work function is the energy to remove delocalised electrons in the


conduction band.

Ionisation energy is the energy to remove electrons in an energy level of an


atom.

The conduction band occupies energies higher than the discrete energy
levels in an atom.

Therefore less energy is required to remove delocalised electrons.


(iii)
The average distance of the outermost electron from the nucleus determines
size of an atom.
1st ionisation energy is 5.42 eV. Since
p2
E= ⇒ p = 2mE
2m

Lowest energy occurs when the momentum is the lowest but uncertainty
principle imposes a limit to the least momentum. Consider the least
momentum Δp = p = 2mE .

h
Using uncertainty principle ΔpΔx ≥ , ΔpΔx ≥ h or ΔpΔx ≥ h
2
h
Δx ≥
2 2mE

6.63 × 10 −34
Δx ≥ = 42.0 pm (3 s.f.)
4π 2 × 9.11× 10−31 × 5.42e

CONFIDENTIAL 12
CONFIDENTIAL

CONFIDENTIAL 13
1

ST. ANDREW’S JUNIOR COLLEGE


JC2 2010
Preliminary Examinations

PHYSICS, Higher 2 9646/01


Paper 1

22nd September 2010 1 hour 15 minutes


(1400 Hrs – 1515 Hrs)
Additional Materials: Optical Mark Sheet (OMS)

READ THESE INSTRUCTIONS FIRST

Write in soft pencil.


Do not use staples, paper clips, highlighters, glue or correction fluid.
Write your name, Civic Group and index number on the separate Optical Mark Sheet (OMS).

There are forty questions in this paper. Answer all the questions. For each question there
are four possible answers A, B, C, D.
Choose the one you consider correct and record your choice in soft pencil on the separate
Optical Mark Sheet (OMS).

Each correct answer will score one mark. A mark will not be deducted for a wrong answer.
Any rough working should be done in this booklet.

Instructions for using the Optical Mark Sheet (OMS)

Class no.
Write your
numbers here

Register no.

1. Fill in your class number (e.g. 09S03 = "03", 09S22 = “22”) in the first two rows.
2. Fill in your class register number in the next two rows. (e.g. register number 1 is filled
in as "01").
3. Write your class and register numbers into the column on the left (ie. 2201)

For Student’s Use

Paper 1 / 40 Paper 2 / 72 Paper 3 / 80

Total / 192 Percentage / 100 Grade

This Question Paper consists of 18 printed pages

SAJC 2010 Prelims/9646/01 [Turn Over


2

Data
speed of light in free space , c = 3.00 x 108 m s-1

permeability of free space, μo = 4 π x 10-7 H m-1

permittivity of free space, εo = 8.85 x 10-12 F m-1

(1 / (36π)) x 10-9 F m-1

elementary charge, e = 1.60 x 10-19 C

the Planck constant, h = 6.63 x 10-34 J s

unified atomic mass constant, u = 1.66 x 10-27 kg

rest mass of electron, me = 9.11 x 10-31 kg

rest mass of proton, mp = 1.67 x 10-27 kg

molar gas constant, R = 8.31 J K-1 mol-1

the Avogadro constant, NA = 6.02 x 1023 mol-1

the Boltzmann constant, k = 1.38 x 10-23 J K-1

gravitational constant, G = 6.67 x 10-11 N m2 kg-2

acceleration of free fall, g = 9.81 m s-2

SAJC 2010 Prelims/9646/01 [Turn Over


3

Formulae

uniformly accelerated motion, s = ut + ½at2

v2 = u2 + 2as

work done on/by a gas, W = pΔV

hydrostatic pressure, p = ρgh

Gm
gravitational potential, φ =- r

displacement of particle in s.h.m., x = x0 sinωt

velocity of particle in s.h.m., v = v0 cosωt

= ± ω ( x 02 − x 2 )

mean kinetic energy of a molecule of an ideal gas,

3
E = 2 kT

resistors in series, R = R1 + R2 +…

resistors in parallel, 1/R = 1/R1 + 1/R2 +…

Q
electric potential, V =
4πε 0 r

alternating current/voltage, x = x0 sinωt

transmission coefficient, T α exp(-2kd)

8π 2 m(U − E)
where k =
h2

radioactive decay, x = x0 exp(-λt)

0.693
decay constant, λ =
t1
2

SAJC 2010 Prelims/9646/01 [Turn Over


4

1 Which of the following provides the most accurate estimate?

A Mrs Quek’s black Subaru Forester 2.0-litre car has a mass of 3000 kg.
B The floor area of the SAJC Cultural Centre ground floor is 700 m2.
C The temperature of a yellow bunsen flame is 600 K.
D The upthrust acting on a totally submerged adult round-tube float is
50 N.

2 A Fomula One car travels a distance of (100 ± 1) m from rest. If the


acceleration of the car is (23.1 ± 0.5) m s-2, what would be its final velocity at
the end of the distance covered?

A (68 ± 1) m s-1
B (67.9 ± 1.0) m s-1
C (68.0 ± 1.1) m s-1
D (67.97 ± 1.08) m s-1

3 Which of the following is a random error?

A Error as a result of using g = 10 m s–2, instead of g = 9.81 m s-2


B Error due to the timing of the experimenter
C Error due to a stopwatch running too fast
D Zero error of a measuring instrument

4 Frankie throws a small rubber ball vertically downwards at a speed of


3.0 m s-1. It hits the ground and rebounds vertically. The graph below shows
the velocity-time graph for the first 1.9 s of the motion of the rubber ball.
v / ms-1
8

time / s
0 0.6 1.5 1.9

-3

-9

What is the displacement of the ball between the point at which it was first
thrown and the highest point of the motion?

A zero
B 1.8 m
C 3.6 m
D 7.2 m

SAJC 2010 Prelims/9646/01 [Turn Over


5

5 A device launches two identical balls, x and y, simultaneously in a horizontal


direction from the same height, as shown in the diagram below. The results
are as indicated.

Fig. 5.1

Which statement correctly describes what happens?

A x hits the ground before y as it is closer to the launch site.


B y hits the ground before x as it has a higher launch velocity.
C x and y hit the ground simultaneously with the same velocity.
D x and y hit the ground simultaneously with different velocities.

6 A ball of mass m is released from rest from point Y which is at a height of h


above point X and slides down a frictionless slope. The ball passes point X
with a velocity v as shown below.
Z

Y
v
h 2h

A second ball of mass 0.5m is released from rest from point Z which is at a
height of 2h above X. The velocity with which the second object passes point
X in terms of v is

A 2v B 1.41v C v D 0.5v

SAJC 2010 Prelims/9646/01 [Turn Over


6

7 A man drives along a straight road with a constant speed from right to left and
tosses a coin vertically upwards. If effects of air resistance are significant,
which diagram best represents the trajectory of the coin seen by a stationary
observer?

A B

C D

8 Three charges +2q, -q and +q are placed at the corners W, X and Y of a


square WXYZ respectively as shown below.

resultant electrostatic
force on X
W X
+2q -q

Z Y
Q +q

A fourth charge Q is placed at Z, after which the charge X experiences a net


electrostatic force indicated by the arrow in the above diagram. What is the
value of Q?

A -2.8q B -1.4q C 1.4q D 2.8q

SAJC 2010 Prelims/9646/01 [Turn Over


7

9 A roller coaster is traveling in a circular path in a vertical plane. The top and
bottom of the track have the same radius of curvature R. The acceleration due
to gravity is g. If the roller coaster has a speed v, the passenger is most likely
to feel weightless

Top

R
R

Bottom

A at the top of the path when v > gR


B at the top of the path when v < gR
C at the bottom of the path when v > gR
D at the bottom of the path when v < gR

10 In a ride at an entertainment park, a person sits in a cage which moves in a


vertical circle at a constant speed.

At the instant shown, what is the direction of the force exerted by the cage on
the person?

D
B

SAJC 2010 Prelims/9646/01 [Turn Over


8

11 A car of mass 1000 kg travels at a constant speed of 8 m s-1 up a slope


inclined at 20° to the horizontal. Given that the constant frictional force against
the motion is 500 N, the engine power of the car is

A 22.8 kW B 30.8 kW C 69.7 kW D 77.7 kW

12 Which diagram shows the variation of gravitational force F on a point mass,


and of gravitational potential energy U of the mass, with its distance r from
another point mass?

13 Figure shows two points X and Y at distances L and 2L from the centre of the
earth. The gravitational potential at X is - 8 kJ kg-1.

Earth

X Y

L
2L

When a 1 kg mass is taken from X to Y the work done on the mass is

A - 4 kJ B - 2 kJ C 2 kJ D 4 kJ

14 Which quantity is not necessarily the same for satellites that are in
geostationary orbits around the Earth?

A angular velocity
B centripetal acceleration
C kinetic energy
D orbital period

SAJC 2010 Prelims/9646/01 [Turn Over


9

15 A fixed mass of an ideal gas is heated at constant volume. Which one of the
following graphs best shows the variation with Celsius temperature t of
pressure p of the gas?

16 In the figure below, the curve is an isotherm (a curve which joins up all the
points having the same temperature) for a fixed mass of ideal gas.
pressure

volume

Which of the following statements can be deduced for the process from A to B?

A Positive work is done on the gas and heat is supplied to the gas.
B Positive work is done by the gas without any heat supplied to the gas.
C The internal energy of the gas decreases as heat is released by the gas.
D Heat is supplied to the gas and the internal energy of the gas remains
unchanged.

SAJC 2010 Prelims/9646/01 [Turn Over


10

17 An ideal gas undergoes the cycle of pressure and volume changes


W→X→Y→Z as shown in the diagram.

What is the work done by the gas in the process Y→Z and the net work done
by the gas as it undergoes a complete cycle of pressure and volume change?

Work done by the gas in


Net work done by the gas
process Y→Z
A -300 J 600 J
B 300 J -600 J
C -400 J 1200 J
D 400 J -1200 J

18 The graph shows the shape at a particular instant of part of a transverse wave
travelling along a string.

Which statement about the motion of elements of the string is correct?

A The speed of the element at P is a maximum.


B The displacement of the element at Q is always zero.
C The energy of the element at R is entirely kinetic.
D The acceleration of the element at S is a maximum.

SAJC 2010 Prelims/9646/01 [Turn Over


11

19 Which one of the following could be an effect of critical damping?

A A toilet door takes a long time to close after a student enters.


B A rubber ball drops to ground and stops bouncing almost immediately.
C A voltmeter fluctuates several times before registering a steady reading.
D A passenger in a car hardly notices that the car has just crossed a
hump.

20 Light of wavelength 550 nm is incident normally on a diffraction grating having


400 lines per millimetre. What is maximum number of bright fringes that can
be observed?

A 4 B 5 C 9 D 11

21 Two wave generators S1 and S2 produce water waves of wavelength 0.5 m. A


detector is placed at position X, 3 m from S1 and 2 m from S2 as shown in the
diagram below. Each generator produces a wave of amplitude A at X when
operated alone. The generators are operating together and producing waves
which have a constant phase difference of π radians. What is the resultant
amplitude at X?

3m 2m

S2
S1

A 0 B 0.5A C A D 2A

SAJC 2010 Prelims/9646/01 [Turn Over


12

22 A closed insulated wire with a circular kink is placed in a uniform magnetic


field with a flux density of 2.5 x 10-2 T, directed into the page. The diameter of
the kink is 1.5 cm. The wire is quickly pulled taut and the kink is straightened
in a time of 0.050 s. If the wire resistance is 1.6 Ω, what is the power
generated in the wire?

Pull Pull

kink

A 3.3 x 10-10 W
B 4.9 x 10-9 W
C 7.7 x 10-8 W
D 8.8 x 10-7 W

23 An insulated wire is bent into a circular coil and placed above a straight
portion as shown. The terminals of the wire are connected to an alternating
voltage. What is the direction of the force acting at point P, the centre of the
circle, on the wire?

Y
a.c.

A It is oscillating in the X and Y direction


B It is pointing in X-direction
C It is pointing in Y-direction
D It is oscillating in and out of the page

SAJC 2010 Prelims/9646/01 [Turn Over


13

24 An electron moves in a circular orbit in a uniform magnetic field. Which of the


following statements is correct?

A The period of the orbit is independent of the speed of the electron.


B The momentum of the electron is dependent on its charge.
C The radius of the orbit is directly proportional to its charge.
D The magnetic force on the electron is dependent on the mass of the
electron.

25 Which one of the following statements about the electric potential and electric
field at a point is correct?

A The potential at the point is always zero when the electric field at that
point is zero.
B The electric potential is given by the rate of change of electric field
intensity with distance.
C The electric field at a point is zero when the potential around the point
is constant.
D The potential at a point is zero when the electric field around the point
is constant.

26 Two long straight wires, X and Y, are placed perpendicularly to each other at
a small distance apart. The current in wire X is flowing into the page and the
current in wire Y is flowing to the right.

Wire X

Wire Y P

What is the direction of the force acting on wire Y at point P due to the
magnetic field produced by wire X?

A out of the page


B into the page
C upwards
D downward

27 The force of attraction between two unlike charges is 1.5 N. If the distance
between the charges is doubled, the force of attraction between them is

A 0.75 N
B 0.38 N
C 3.0 N
D 6.0 N

SAJC 2010 Prelims/9646/01 [Turn Over


14

28 Charges of 2 μC and -2 μC situated at points P and Q respectively as shown.


X lies midway between P and Q while Y is at a mid-line. Which of the
following correctly describes the directions of the electric fields and electric
potentials at points X and Y?

2 μC - 2 μC
X

P Q

At point X At point Y
Electric Field Potential Field Electric Field Potential Field
A right zero right zero
B upwards positive upwards negative
C right zero left zero
D downwards negative downwards positive

29 The diagram shows a circuit consisting of five 1-Ω resistors. A multimeter is


used to measure the resistance across different terminals. Which of the
following statements is false?

A B

C D

A The resistance measured between B and C is 0.5 Ω.


B The resistance measured between B and C will be smaller if an
additional resistor is connected in parallel across AC.
C The resistance measured between A and D is 1 Ω.
D The resistance measured between A and D will be smaller if a zero
resistance wire is connected across BC.

SAJC 2010 Prelims/9646/01 [Turn Over


15

30 A potential difference of 2.0 V is connected to a uniform resistance wire of


length 3.0 m and cross-sectional area of 8.0 x 10-9 m2. A current of 0.1 A flows
in the wire. What is the current flowing in the wire if the length and diameter of
the wire are doubled with the same potential difference connected to it?

A 0.10 A B 0.20 A C 0.30 A D 0.40 A

31 In an ideal transformer, the most important function of the soft-iron core is

A to reduce eddy-currents.
B to improve the flux-linkage between the primary and secondary coils.
C to dissipate the heat generated by the two coils.
D to produce a uniform magnetic field in the two coils

32 A steady current I dissipates a certain power in a variable resistor. When a


sinusoidal alternating current is used, the variable resistor has to be reduced
to one quarter of its initial value to obtain the same power.
What is the peak value of the alternating current?

A 2I
B 2I
C 2 2I
D 4 2I

33 Which of the following statements, referring to photoelectric emission, is


always true?

A No emission of electrons occurs for very low intensity illumination.


B For a given metal there is a minimum frequency of radiation below
which no emission occurs.
C The velocity of the emitted electrons is proportional to the intensity of
the incident radiation.
D The number of electrons emitted per second is independent of the
intensity of incident radiation.

SAJC 2010 Prelims/9646/01 [Turn Over


16

34 The table gives relative values for three situations for the barrier tunnelling of
an electron through a potential barrier. Rank the situations according to the
probability of the electron tunnelling barrier, greatest first.

Electron Energy Barrier Height Barrier Thickness


X E 5E L
Y E 17E 2L
Z E 2E 3L

A XYZ
B XZY
C ZXY
D YZX

35 The diagram shows the energy levels of a gas atom.

____________________ 0 J

____________________ -2.4 x 10-19 J

____________________ -5.4 x 10-19 J

____________________ -21.8 x 10-19 J

A free electron of kinetic energy of 20.0 x 10-19 J collides with the cool gas
atoms. What is the kinetic energy of the free electron after the collision?

A 1.8 x 10-19 J
B 3.6 x 10-19 J
C 5.4 x 10-19 J
D 16.4 x 10-19 J

SAJC 2010 Prelims/9646/01 [Turn Over


17

36 A ruby laser is a solid-state laser and emits at a wavelength of 694 nm. A


CO2 laser is a gas laser and emits at a wavelength of 10.6 µm. Why are CO2
lasers (instead of ruby lasers) used in the metalworking industry to cut steel?

A CO2 lasers do not possess a metastable state


B The energy of a single photon emitted by the CO2 laser is larger than
that of the energy of a single photon emitted by the ruby laser
C CO2 lasers do not emit coherent photons
D CO2 lasers emits laser light in the infrared region

37 Once the active medium in a LASER is excited, the first photons of light are
produced by which physical process?

A Planck’s oscillation
B blackbody radiation
C spontaneous emission
D synchrotron radiation

38 The most important property of a p-n junction is that it rectifies an alternating


current. Which of the following statements is false?

A During reverse bias condition of a p-n junction, the p-type semi-


conductor becomes less negative.
B During reverse bias condition of a p-n junction, the width of the
depletion region becomes larger as the externally applied p.d. adds to
the junction potential.
C During forward bias condition of a p-n junction, if the applied p.d.
overcomes the junction potential, electrons will cross steadily from the
n-type side to the p-type side while the holes will cross steadily in the
opposite direction.
D Under increasingly high reverse biased p.d., current can increase
sharply through the p-n junction

39 Uranium-235 undergoes fission as shown in the equation below.


235 1 143 90 1
92 U + 0 n Æ 55 Cs + 37 Rb + 3 0 n

195 MeV of energy was released in the reaction. Given that the binding
energy per nucleon for uranium-235 is 7.6 MeV, and those for caesium and
rubidium are approximately X MeV, determine the value of X.

A 8.5 MeV
B 9.7 MeV
C 11.2 MeV
D 13.3 MeV

SAJC 2010 Prelims/9646/01 [Turn Over


18

60
40 The number of undecayed radioactive nuclei, N, of cobalt 27 Co at three
different timings are as follows:

t / years N
0 500000
12 M
16 31250

What is the value of M?

A 25000
B 62500
C 100000
D 125000

SAJC 2010 Prelims/9646/01 End of Paper


[Turn Over
1

Class Index Number Name


09

ST. ANDREW’S JUNIOR COLLEGE


JC 2 2010
Preliminary Examinations

PHYSICS, Higher 2 9646/02


Paper 2 Structured Questions

17th September 2010 1 hour 45 minutes


(0800 Hrs – 0945 Hrs)

Candidates answer all questions on the Question Paper.


No additional materials are required.

READ THESE INSTRUCTIONS FIRST

Write your name, index number and Civics Group on all the work you hand in.
Write in dark blue or black pen on both sides of the paper.
You may use a soft pencil for any diagrams, graphs or rough working.
Do not use staples, paper clips, highlighters, glue or correction fluid.

Section A
Answer all questions.
It is recommended that you spend about 1 hour 15 minutes on this section.

Section B
Answer Question 8.
It is recommended that you spend about 30 minutes on this section.

At the end of the examination, fasten all your work securely together.
The number of marks is given in brackets [ ] at the end of each question or part
question.

For Examiner’s Use

Section A / 60
Section B / 12
Total / 72

This document consists of 23 printed pages.

SAJC 2010 Prelims/9646/02 [Turn Over


2

Data
speed of light in free space , c = 3.00 x 108 m s-1

permeability of free space, μo = 4 π x 10-7 H m-1

permittivity of free space, εo = 8.85 x 10-12 F m-1

(1 / (36π)) x 10-9 F m-1

elementary charge, e = 1.60 x 10-19 C

the Planck constant, h = 6.63 x 10-34 J s

unified atomic mass constant, u = 1.66 x 10-27 kg

rest mass of electron, me = 9.11 x 10-31 kg

rest mass of proton, mp = 1.67 x 10-27 kg

molar gas constant, R = 8.31 J K-1 mol-1

the Avogadro constant, NA = 6.02 x 1023 mol-1

the Boltzmann constant, k = 1.38 x 10-23 J K-1

gravitational constant, G = 6.67 x 10-11 N m2 kg-2

acceleration of free fall, g = 9.81 m s-2

SAJC 2010 Prelims/9646/02 [Turn Over


3

Formulae

uniformly accelerated motion, s = ut + ½at2

v2 = u2 + 2as

work done on/by a gas, W = pΔV

hydrostatic pressure, p = ρgh

Gm
gravitational potential, φ =- r

displacement of particle in s.h.m., x = x0 sinωt

velocity of particle in s.h.m., v = v0 cosωt

= ± ω ( x 02 − x 2 )

mean kinetic energy of a molecule of an ideal gas,

3
E = 2 kT

resistors in series, R = R1 + R2 +…

resistors in parallel, 1/R = 1/R1 + 1/R2 +…

Q
electric potential, V =
4πε 0 r

alternating current/voltage, x = x0 sinωt

transmission coefficient, T α exp(-2kd)

8π 2 m(U − E)
where k =
h2

radioactive decay, x = x0 exp(-λt)

0.693
decay constant, λ =
t1
2

SAJC 2010 Prelims/9646/02 [Turn Over


4

Section A

Answer all questions.


It is recommended that you spend about 1 hour 15 minutes on this section.

1 The three sketch graphs in Fig. 1.1, plotted against time, show changes which
occur in a small fraction of a second and which result in almost vertical lines
on the graphs. These three sketch graphs are possible for ordinary objects.

velocity acceleration

time

time
A B

resultant force

time
C
Fig. 1.1

SAJC 2010 Prelims/9646/02 [Turn Over


5

From the graphs of Fig. 1.1, choose any two graphs and describe and explain
an everyday situation which illustrates how that graph can arise.

1. Graph letter ……………….

…………………………………………………………………………………............

…………………………………………………………………………………............

…………………………………………………………………………………............

…………………………………………………………………………………............

…………………………………………………………………………………............

……..………………………………………………………………………………..[3]

2. Graph letter ……………….

…………………………………………………………………………………............

…………………………………………………………………………………............

…………………………………………………………………………………............

…………………………………………………………………………………............

…………………………………………………………………………………............

……..………………………………………………………………………………..[3]

SAJC 2010 Prelims/9646/02 [Turn Over


6

2 A system of two bodies A and B are connected by an inextensible cord


over a frictionless pulley and are resting on inclined planes as
shown in Fig. 2.1. Body A of mass 2.00 kg and body B of mass
5.00 kg move, in the directions as indicated, a distance of 0.500 m and
each experiences a frictional force of 3.00 N.

B
A

40° 50°

Fig. 2.1

(a) Calculate the change in gravitational potential energy of the system.

change in gravitational potential energy = …………………… J [2]

(b) Determine the final speed of system after travelling 0.500 m.

final speed = …………………………….m s−1 [3]

SAJC 2010 Prelims/9646/02 [Turn Over


7

(c) On the axes of Fig. 2.2, sketch a clearly labelled graph of the variation
with time of the gravitational potential energy Ep, kinetic energy Ek and
work done against frictional forces Wf. [3]

Energy

0 Time

Fig. 2.2

SAJC 2010 Prelims/9646/02 [Turn Over


8

3 (a) The Earth may be considered to be a uniform sphere of radius 6370 km,
spinning on its axis with a period of 24.0 hours.

(i) Calculate the angular velocity of a 2.00 kg mass situated at the


equator.

angular velocity = …………rad s-1 [1]

(ii) At the North Pole, an accurate spring balance supporting the


2.00 kg mass gives a reading of 19.66 N. What is the reading on
the spring balance if the measurement is taken at the equator?

force = ………..N [2]

(iii) Explain the difference (if any) in the readings of the spring
balance in (a) (ii).

………………………………………………………………………......

………………………………………………………………………......

..……………………………………………………………………….[1]

SAJC 2010 Prelims/9646/02 [Turn Over


9

(b) An astronaut in a spacecraft orbiting the Earth may be described as


weightless. Explain why this is so.

…………………………………………………………………………..............

…………………………………………………………………………..............

…………………………………………………………………………..............

…………………………………………………………………………..............

…………………………………………………………………………..........[2]

SAJC 2010 Prelims/9646/02 [Turn Over


10

4 (a) Define magnetic flux density

………………………………………………………………………………….

………………………………………………………………………………..[1]

(b) A magnetohydrodynamic drive (MHD) is a method for propelling


seagoing vessels using only electric and magnetic fields. Sea water is
propelled through the MHD tube using magnetic forces. Fig. 4.1 below
shows the components of the MHD. A pair of powerful magnet is
placed across the MHD tube where a current flows.

Fig. 4.1

Fig. 4.2

(i) Indicate on Fig. 4.2 above the direction of the magnetic field
that allows the MHD to propel water in the direction shown. [1]

(ii) Calculate the magnetic flux density required to produce a force


of 16000 N with a current of 10.0 A. The separation between the
electrode is 30.0 cm.

magnetic flux density= ………………… T [2]

SAJC 2010 Prelims/9646/02 [Turn Over


11

(iii) Suggest one advantage and one disadvantage of MHD.

……………………………………………………………………………….

……………………………………………………………………………….

……………………………………………………………………………….

………………………………………………………………………………..[2]

(c) A bar magnet attached to a spring falls vertically with its south pole
entering a horizontal closed metallic wire loop as shown in Fig. 4.3
below.

S
Fig. 4.3

(i) Sketch a graph of the variation of the induced current in the loop
with time as the magnet passes through the loop. Draw at least
2 cycles of the change. [2]

(ii) Explain the features of the graph in part (i).

……………………………………………………………………………….

……………………………………………………………………………….

……………………………………………………………………………….

………………………………………………………………………………..[2]

SAJC 2010 Prelims/9646/02 [Turn Over


12

5 (a) State an advantage of using alternating current for the transmission of


electrical energy.

…………………………………………………………………………………

………………………………………………………………………………[1]

(b) State why high voltage is used in the transmission of electrical energy.

…………………………………………………………………………………

………………………………………………………………………………[1]

(c) An a.c. generator delivers power to a resistive load in a remote location


over a cable transmission line of resistance 6.0 kΩ. The potential
difference across the load is 80 kV, and the power delivered by the
generator is 250 kW.

(i) Show that the current flowing in the cables is 2.61 A. [2]

(ii) Calculate the total voltage drop across the transmission cables.

voltage drop = …………. V [1]

(iii) Calculate the power dissipated in the transmission cables as


thermal energy.

power dissipated = …………. W [1]

SAJC 2010 Prelims/9646/02 [Turn Over


13

(iv) Calculate the power delivered to the resistive load.

power delivered = …………. W [1]

SAJC 2010 Prelims/9646/02 [Turn Over


14

6 (a) Distinguish between the conduction band and the valence band.

…………………………………………………………………………………

…………………………………………………………………………………

…………………………………………………………………………………

…………………………………………………………………………………

……..…………….…………………………………………………………. [2]

(b) Table 6.1 below shows the colour, wavelength and photon energy of
light emitted by three sources.

colour wavelength / nm photon energy / eV


red 650 1.91
green 550 2.26
blue 450 2.76

Table 6.1

Suggest why

(i) an insulator like glass may be transparent to all three colours.

…………………………………………………………………………

…………………………………………………………………………

…………………………………………………………………………

…………….…………………………………………………………. [2]

(ii) metals are opaque and reflective to all three colours.

…………………………………………………………………………

…………………………………………………………………………

…………………………………………………………………………

…………….…………………………………………………………. [2]

SAJC 2010 Prelims/9646/02 [Turn Over


15

(c) State two differences between p-type semiconductors and n-type


semiconductors.

…………………………………………………………………………………

…………………………………………………………………………………

…………………………………………………………………………………

…………………………………………………………………………………

……..…………….…………………………………………………………. [2]

SAJC 2010 Prelims/9646/02 [Turn Over


16

7 For thousands of years, Man has studied the night sky and some ancient
buildings provide evidence of careful and patient astronomical observations
by people of many different cultures. As instrumentation has improved, so has
the precision with which astronomical observations could be made. Between
1576 and 1597, Brahé made comprehensive observations of planetary
positions and, on his death, these records became available to Kepler.

Kepler was able to interpret the observations and deduced three laws, one of
which had a great impact on later discoveries. He deduced that, for a circular
orbit of a planet around the Sun, if T is the period of rotation and r is the radius
of the orbit, then

T2 ∝ r3 .

As a result of Kepler's work Newton formulated the law of gravitation.

(a) (i) State an equation representing Newton's law of gravitation,


explaining the symbols used.
[2]

(ii) By relating the gravitational force on a planet to the centripetal


acceleration it causes, show that, for a circular orbit,

24π2r3
T = GM .
[2]

SAJC 2010 Prelims/9646/02 [Turn Over


17

(b) The planet Jupiter has a number of moons. Data for some of these are
given in Fig. 7.1.

moon period Mean distance from log10(T/days) log10(r/m)


T/days centre of Jupiter r/109m

Sinope 758 23.7 2.88 10.37


Leda 239 11.1
Callisto 16.7 1.88
Lo 1.77 0.422
Metis 0.295 0.128 -0.530 8.11

Fig. 7.1

(i) Complete Fig. 7.1 by calculating values for log10(T/days) and


log10(r/m). [1]

(ii) On the axes of Fig. 7.2, plot a graph of log10(T/days) against


log10(r/m). [2]

(c) (i) Determine the gradient of the graph in Fig. 7.2.

gradient = ……………….. [1]

(ii) Hence discuss whether the data in Fig. 7.1 support the relation
given in (a)(ii).

…………………………………………………………………………..............

…………………………………………………………………………..............

…………………………………………………………………………..............

…………………………………………………………………………..............

…………………………………………………………………………..............

…………………………………………………………………………..............

…………………………………………………………………………..........[3]

SAJC 2010 Prelims/9646/02 [Turn Over


18

log10 (T / days)

Fig. 7.2

SAJC 2010 Prelims/9646/02 [Turn Over


19

(d) Observation shows that the moon Ganymede orbits Jupiter with a
period of 7.16 days. Use the graph of Fig. 7.2 to estimate the orbital
radius of Ganymede.

radius = ………….. m [2]

(e) Suggest whether the graph of Fig. 7.2 could be used to check data on
the orbital radii and periods of the moons of another planet (e.g.
Saturn).

…………………………………………………………………………………...

…………………………………………………………………………………...

…………………………………………………………………………………...

…………………………………………………………………………………...

………………………………………………………………………………. [2]

SAJC 2010 Prelims/9646/02 [Turn Over


20

Section B

It is recommended that you spend about 30 minutes on this section.

8 A motor engineer wishes to investigate how a spark may be produced under


different conditions. The spark is formed across a fixed gap in a device known
as a spark plug when a high voltage is applied across its electrodes.
Fig. 8.1 shows a typical spark plug.

air gap

Fig. 8.1

You are required to design an experiment in the laboratory to investigate how


the pressure of the air affects the minimum voltage required to generate a
spark across a gap between two electrodes.

In your account you should pay particular attention to the following

(a) the basic procedure to be followed,

(b) the control of variables,

(c) how the minimum voltage required to generate a spark is achieved,

(d) any relevant design features of your experiment.

You may assume that the following apparatus is available, together with any
other standard equipment which may be found in a school or college science
laboratory.

Electrodes with connecting leads Pump

Pressure chamber Variable high voltage supply

Pressure gauge Voltmeter

[ 12 ]

SAJC 2010 Prelims/9646/02 [Turn Over


21

Diagram

…………………………………………………………………………………………………...

…………………………………………………………………………………………………...

…………………………………………………………………………………………………...

…………………………………………………………………………………………………...

…………………………………………………………………………………………………...

…………………………………………………………………………………………………...

…………………………………………………………………………………………………...

…………………………………………………………………………………………………...

…………………………………………………………………………………………………...

…………………………………………………………………………………………………...

…………………………………………………………………………………………………...

…………………………………………………………………………………………………...

SAJC 2010 Prelims/9646/02 [Turn Over


22

…………………………………………………………………………………………………...

…………………………………………………………………………………………………...

…………………………………………………………………………………………………...

…………………………………………………………………………………………………...

…………………………………………………………………………………………………...

…………………………………………………………………………………………………...

…………………………………………………………………………………………………...

…………………………………………………………………………………………………...

…………………………………………………………………………………………………...

…………………………………………………………………………………………………...

…………………………………………………………………………………………………...

…………………………………………………………………………………………………...

…………………………………………………………………………………………………...

…………………………………………………………………………………………………...

…………………………………………………………………………………………………...

…………………………………………………………………………………………………...

…………………………………………………………………………………………………...

…………………………………………………………………………………………………...

…………………………………………………………………………………………………...

…………………………………………………………………………………………………...

…………………………………………………………………………………………………...

…………………………………………………………………………………………………...

…………………………………………………………………………………………………...

…………………………………………………………………………………………………...

…………………………………………………………………………………………………...

SAJC 2010 Prelims/9646/02 [Turn Over


23

…………………………………………………………………………………………………...

…………………………………………………………………………………………………...

…………………………………………………………………………………………………...

…………………………………………………………………………………………………...

…………………………………………………………………………………………………...

…………………………………………………………………………………………………...

…………………………………………………………………………………………………...

…………………………………………………………………………………………………...

…………………………………………………………………………………………………...

…………………………………………………………………………………………………...

…………………………………………………………………………………………………...

…………………………………………………………………………………………………...

…………………………………………………………………………………………………...

…………………………………………………………………………………………………...

…………………………………………………………………………………………………...

…………………………………………………………………………………………………...

…………………………………………………………………………………………………...

…………………………………………………………………………………………………...

…………………………………………………………………………………………………...

…………………………………………………………………………………………………...

…………………………………………………………………………………………………...

…………………………………………………………………………………………………...

…………………………………………………………………………………………………...

…………………………………………………………………………………………………...

…………………………………………………………………………………………………...

SAJC 2010 Prelims/9646/02 End ofOver


[Turn Paper
1

Class Index Number Name


09

ST. ANDREW’S JUNIOR COLLEGE


JC 2 2010
Preliminary Examinations

PHYSICS, Higher 2 9646/03


Paper 3 Longer Structured Questions

20th September 2010 2 hours


(1400 Hrs – 1600 Hrs)

Candidates answer all questions on the question paper.


No additional materials are required.

READ THESE INSTRUCTIONS FIRST

Write your name, index number and Civics Group on all the work you hand in.
Write in dark blue or black pen on both sides of the paper.
You may use a soft pencil for any diagrams, graphs or rough working.
Do not use staples, paper clips, highlighters, glue or correction fluid.

Section A
Answer all questions.

Section B
Answer any two questions.

You are advised to spend about one hour on each section.

At the end of the examination, fasten all your work securely together.
The number of marks is given in brackets [ ] at the end of each question or part
question.

For Examiner’s Use

Section A / 40
Section B / 40
Total / 80

This question paper consists of 24 printed pages.

SAJC 2010 Prelims/9646/03 [Turn Over


2

Data
speed of light in free space , c = 3.00 x 108 m s-1

permeability of free space, μo = 4 π x 10-7 H m-1

permittivity of free space, εo = 8.85 x 10-12 F m-1

(1 / (36π)) x 10-9 F m-1

elementary charge, e = 1.60 x 10-19 C

the Planck constant, h = 6.63 x 10-34 J s

unified atomic mass constant, u = 1.66 x 10-27 kg

rest mass of electron, me = 9.11 x 10-31 kg

rest mass of proton, mp = 1.67 x 10-27 kg

molar gas constant, R = 8.31 J K-1 mol-1

the Avogadro constant, NA = 6.02 x 1023 mol-1

the Boltzmann constant, k = 1.38 x 10-23 J K-1

gravitational constant, G = 6.67 x 10-11 N m2 kg-2

acceleration of free fall, g = 9.81 m s-2

SAJC 2010 Prelims/9646/03 [Turn Over


3

Formulae

uniformly accelerated motion, s = ut + ½at2

v2 = u2 + 2as

work done on/by a gas, W = pΔV

hydrostatic pressure, p = ρgh

Gm
gravitational potential, φ =- r

displacement of particle in s.h.m., x = x0 sinωt

velocity of particle in s.h.m., v = v0 cosωt

= ± ω ( x 02 − x 2 )

mean kinetic energy of a molecule of an ideal gas,

3
E = 2 kT

resistors in series, R = R1 + R2 +…

resistors in parallel, 1/R = 1/R1 + 1/R2 +…

Q
electric potential, V =
4πε 0 r

alternating current/voltage, x = x0 sinωt

transmission coefficient, T α exp(-2kd)

8π 2 m(U − E)
where k =
h2

radioactive decay, x = x0 exp(-λt)

0.693
decay constant, λ =
t1
2

SAJC 2010 Prelims/9646/03 [Turn Over


4

Section A

Answer all the questions in this section.

1 (a) For a spacecraft launched into the outer regions of the solar system, it
is not practical to have its battery recharged by solar panels. Such
spacecrafts use Plutonium-238 (Pu-238), which is an alpha emitter with
a half-life of 88 years, as fuel.

(i) If each alpha particle is emitted with a kinetic energy of 5.0 MeV,
calculate that the minimum activity of the source required to
produce an alpha particle beam of 20 W.

activity = ……………….. Bq [2]

(ii) Show that the decay constant, λ, of Pu-238 is 2.5 x 10-10 s-1. [1]

(iii) Calculate the mass of Pu-238 required to generate the activity


shown in (i).

mass = ………………… kg [3]

SAJC 2010 Prelims/9646/03 [Turn Over


5

(iv) Plutonium is one of the most dangerous radioactive substances


known. It has been estimated that even a small amount of this
substance, suitably distributed, would be enough to kill more
than a billion people. Comment on the risks on the population on
Earth involved in using plutonium as a fuel for spacecraft.

…………………………………………………………………………………...

…………………………………………………………………………………...

…………………………………………………………………………………...

…………………………………………………………………………………...

………………………………………………………………………………[2]

(b) The radioactive isotope, Iodine-131, with a half-life of 30 years, and


Caesium-137, with a half-life of 8 days, are the nuclear wastes
generated from nuclear power plants.

Consider that each of these wastes is of similar mass and sufficient to


pose a hazard initially, explain which of these nuclear wastes should
be stored for a longer period of time.

…………………………………………………………………………………...

…………………………………………………………………………………...

…………………………………………………………………………………...

…………………………………………………………………………………...

………………………………………………………………………………[2]

SAJC 2010 Prelims/9646/03 [Turn Over


6

2 (a) Fig. 2.1 shows the variation of displacement y with time t of a sound
wave incident on a person’s ear drum.

Fig. 2.1

Assume that the eardrum vibrates with simple harmonic motion and
with the same frequency and amplitude as the incident sound wave.

(i) State the amplitude and frequency of the oscillating eardrum.

amplitude = ……………………m [1]

frequency = …………………..Hz [1]

(ii) Calculate the mass of a human eardrum if the maximum kinetic


energy of the oscillating eardrum is 2.4 ×10-19 J.

mass = ……………………kg [2]

SAJC 2010 Prelims/9646/03 [Turn Over


7

(iii) On the axes of Fig. 2.2, sketch a clearly labelled graph to show
the variation of the velocity of the ear drum v with displacement y.

Fig. 2.2 [2]

(b) During the 2010 FIFA World Cup, the Vuvuzela (refer to Fig. 2.3), the
South African musical horn, was badly received by some television
viewers as the playing of the horns by the stadium spectators created a
constant annoying humming sound for the viewers.

Fig. 2.3
Audio engineers have suggested the use of noise-cancelling
technology to generate tones of approximately 235 Hz, 470 Hz and
705 Hz to cancel out the humming sounds.

By considering the Vuvuzela to be a 70 cm long pipe with two open


ends and taking the speed of sound to be 330 m s-1, explain the choice
of the frequencies used and suggest how this method could cancel out
the humming sounds.

…………………………………………………………………………………

…………………………………………………………………………………

…………………………………………………………………………………

…………………………………………………………………………………

…………………………………………………………………………………

…………………………………………………………………………………

…………………………………………………………………………………

………………………………………………………………………………[4]

SAJC 2010 Prelims/9646/03 [Turn Over


8

3 In one photoelectric experiment, a sodium surface was illuminated


with monochromatic light of suitable wavelength. The emitted
photoelectrons were collected at an adjacent electrode. The way in
which the photocurrent I depends on the potential difference V
between the collector and emitter is shown in Fig. 3.1 below.

-Vs 0 Vext
Fig. 3.1

(a) Fig. 3.1 shows that, for slightly positive values of V, the value of
I is constant. Give a physical explanation for this.

…………………………………………………………………………………..

……………………………………………………………………………….[1]

(b) When the potential difference is changed from zero to negative values
of V, the value of I falls from its constant value to zero. Give a physical
explanation for this.

…………………………………………………………………………………...

…………………………………………………………………………………...

…………………………………………………………………………………...

……………………………………………………………………………….[2]

(c) Draw on the graph (Fig. 3.1) above, the results of another experiment
using light of higher frequency but of the same power. [2]

SAJC 2010 Prelims/9646/03 [Turn Over


9

(d) When the emitter is illuminated with electromagnetic radiation of


photon energy 4.7 eV at a power of 3.8 mW, the constant
current is 0.8 x 10-8 A and the current goes to zero at -0.5 V.

(i) Calculate the rate at which the photons are incident on


the emitter.

rate of photons = ………………….. s-1 [2]

(ii) Calculate the maximum rate at which electrons leave the


emitter.

rate of electrons = …………………. s-1 [1]

(iii) Suggest two reasons for the difference between the


two answers above.

…………………………………………………………………………………...

…………………………………………………………………………………...

…………………………………………………………………………………...

…………………………………………………………………………………...

……………………………………………………………………………….[2]

SAJC 2010 Prelims/9646/03 [Turn Over


10

4 (a) Fig. 4.1 shows a cell of e.m.f. 2.0 V and internal resistance 0.20 Ω
connected in parallel to two identical lamps L1 and L2. The ammeters
A1 and A2 in the circuit have negligible resistance and A2 reads 0.50 A.

A1

A2

2.0 V

0.20 Ω L2
L1

Fig. 4.1

(i) Calculate the potential difference across L1.

potential difference = …………V [2]

(ii) If another identical lamp L3 is connected in parallel with L1 and L2,


explain whether the current in ammeter A1 remains the same,
increases or decreases.

…………………………………………………………………………...

…………………………………………………………………………...

…………………………………………………………………………...

…………………………………………………………………………...

……………………………………………………………………….[2]

SAJC 2010 Prelims/9646/03 [Turn Over


11

(b) Fig. 4.2 shows a circuit which is used to measure the emf of Cell Y.

Cell Y

A J B
8.0 Ω

Cell X, 2.00 V

Fig. 4.2

Cell X has an e.m.f. of 2.00 V and negligible internal resistance. It is


connected in series with a 8.0 Ω resistor and resistance wire AB. The
resistance wire AB has a length 100.0 cm and a resistance, 2.0 Ω.

(i) Calculate the potential difference across AB.

potential difference = …………V [1]

(ii) The movable contact J is now moved along AB. When the
galvanometer indicates a zero reading, the length AJ is 30.0 cm.
Calculate the e.m.f., in mV, of Cell Y.

e.m.f.= ……..….. V [2]

SAJC 2010 Prelims/9646/03 [Turn Over


12

(iii) Without changing the length AB, suggest three modifications to


the circuit of Fig. 4.2 that would cause the contact J to be closer
to A when the galvanometer gives a zero reading.

1. ………………………………………………………………………..

………………………………………………………………………..[1]

2. ………………………………………………………………………..

………………………………………………………………………[1]

3. ………………………………………………………………………..

………………………………………………………………………[1]

SAJC 2010 Prelims/9646/03 [Turn Over


13

Section B

Answer two questions from this section.

5 (a) (i) State Newton’s first law of motion and show how it leads to the
concept of force.

…………………………………………………………………………..

…………………………………………………………………………..

…………………………………………………………………………..

…………………………………………………………………………..

……………………………………………………………………….[2]

(ii) Newton’s second law states that “the rate of change of


momentum of a body is proportional to the resultant force acting
on it”.

Show how this law, together with a suitable definition of the unit
of force, leads to the relationship F = ma for a body of constant
mass. [3]

(b) A stone is dropped from a point a few metres above the Earth’s surface.
Considering the system of stone and Earth, discuss briefly how the
principle of conservation of momentum applies before the impact of the
stone with the Earth.

…………………………………………………………………………………...

…………………………………………………………………………………..

…………………………………………………………………………………..

…………………………………………………………………………………..

………..………………………………………………………………………[3]

SAJC 2010 Prelims/9646/03 [Turn Over


14

(c) A stationary radium nucleus ( 224


88 Ra) of mass 224 u spontaneously

emits an α-particle ( 42 He) of mass 4 u. The α-particle is emitted with an


energy of 9.2 x 10-13 J and the reaction gives rise to a nucleus of radon
(Rn).

(i) Write down a nuclear equation to represent the α-decay of a


radium nucleus. [1]

(ii) Show that the speed at which the α-particle is ejected from the
radium nucleus is 1.7 x 107 m s-1.

[2]

(iii) Calculate the speed of the radon nucleus on emission of the


α-particle. Explain how the principle of conservation of linear
momentum is applied in your calculation.

…………………………………………………………………………………

…………………………………………………………………………………

…………………………………………………………………………………

………………………………………………………………………………[3]

SAJC 2010 Prelims/9646/03 [Turn Over


15

(d) (i) Define the centre of gravity of an object.

………………………………………………………………………………….

…………………………………………………………………………….. [1]

(ii) State the principle of moments.

………………………………………………………………………………….

…………………………………………………………………………….. [1]

(iii) Fig. 5.1 shows a massive column held stationary in position by a


group of people pulling at a rope.

Fig. 5.1

The 4.0 m high column has a mass of 180 kg and its centre of
gravity X is at a distance of 2.3 m from the base. The rope
makes an angle of 350 to the column and the column itself
makes an angle of 450 to the horizontal.

1. Show that the moment exerted by the weight of the column


about the base is 2.9 x 103 N m.

[2]
2. Calculate the tension T in the rope.

T = ………… N [2]

SAJC 2010 Prelims/9646/03 [Turn Over


16

6 (a) (i) There is no attraction between the molecules of an ideal gas.


Use this information to explain why the internal energy of an
ideal gas is proportional to its temperature.

…………………………………………………………………………

…………………………………………………………………………

…………………………………………………………………………

…………………………………………………………………………

…………………………………………………………………………

………………………………………………………………………[3]

(ii) Hence, explain how this relationship between the internal energy
of an ideal gas and the absolute temperature, gives rise to the
concept of an absolute zero of temperature.

…………………………………………………………………………

…………………………………………………………………………

…………………………………………………………………………

………………………………………………………………………[2]

SAJC 2010 Prelims/9646/03 [Turn Over


17

(b) Fig. 6.1 represents how the temperature of a small mass of water
changes when it is heated steadily from room temperature to above its
boiling point in a large sealed container.

Fig. 6.1

Describe and explain the features of the graph in terms of the changes
which occur to the separation of the molecules and to their potential
and kinetic energies. Three distinct sections of the graph have been
labelled to aid your description.

(i) A to B

…………………………………………………………………………

…………………………………………………………………………

………………………………………………………………………[2]

(ii) B to C

…………………………………………………………………………

…………………………………………………………………………

………………………………………………………………………[2]

(iii) C to D

…………………………………………………………………………

…………………………………………………………………………

………………………………………………………………………[2]

SAJC 2010 Prelims/9646/03 [Turn Over


18

(c) This question is about the atmosphere treated as an ideal gas.

(i) The equation of state of an ideal gas is pV = nRT. Data about


gases are often given in terms of density ρ rather than volume V.
Show that the equation of state for a gas can be written as

ρRT
p= M

where M is the mass of one mole of gas.

[2]

SAJC 2010 Prelims/9646/03 [Turn Over


19

(ii) One simple model of the atmosphere assumes that air behaves
as an ideal gas at a constant temperature. Using this model the
pressure p of the atmosphere at a temperature of 20 °C varies
with height h above the Earth’s surface as shown in Fig. 6.2.

Fig. 6.2

Use data from the graph to show that the variation of pressure
with height follows an exponential relationship.

[3]

(iii) The ideal gas equation in (c)(i) shows that, at constant


temperature, pressure p is proportional to density ρ. Use data
from Fig. 6.2 to find the density of the atmosphere at a height of
8.0 km. (Density ρ of air at h = 0 m is 1.3 kg m–3)

ρ = .............................................. kg m–3 [2]

SAJC 2010 Prelims/9646/03 [Turn Over


20

(iv) In the real atmosphere the density, pressure and temperature all
decrease with height. At the summit of Mt. Everest, 8.0 km
above sea level, the pressure is only 0.30 of that at sea level.
Take the temperature at the summit to be –23 °C and at sea
level to be 20 °C. Calculate, using the ideal gas equation, the
density of the air at the summit.
(Density ρ of air at sea level = 1.3 kg m–3)

ρ = ............................................. kg m–3 [2]

SAJC 2010 Prelims/9646/03 [Turn Over


21

7 (a) Electrons from a filament source enter a region between the parallel
plates after being accelerated by an electric field. Fig. 7.1 below shows
the electrons travelling horizontally at a speed of 2.50 x 107 m s-1
entering the pair of parallel plates.

+V

40 mm
7 -1
2.5 x 10 m s

0V
Fig. 7.1

(i) The electrons deviate by 30° on leaving the parallel


plate of 80.0 mm long. The separation between the plates is
40.0 mm.

Calculate the time taken for the electrons to traveling through


the plates.

time = ………………….. s [1]

(ii) Calculate the vertical component of the velocity when the


electrons exit the parallel plates.

vertical component of velocity = ……………… m s-1 [2]

SAJC 2010 Prelims/9646/03 [Turn Over


22

(iii) Hence calculate the acceleration of the electrons.

acceleration = ………………..m s-2 [1]

(iv) Calculate the p.d. V between the two plates.

p.d. = ………………… V [2]

(b) A magnetic field is applied to the region of the electric field in Fig. 7.1
so that the electrons pass straight through undeflected.

(i) Indicate in words or sketch the direction of the magnetic flux


density for Fig. 7.1 in the space below. [1]

(ii) Calculate the magnetic flux density required to produce this


effect.

magnetic flux density = ………………… T [2]

SAJC 2010 Prelims/9646/03 [Turn Over


23

(iii) The electrons that passes straight through are allowed to hit a
tungsten target. Calculate the wavelength of the most energetic
x-ray photons produced.

wavelength = ……………………… m [2]

(c) The parallel plates in Fig. 7.1 are removed but the magnetic field is
kept at the same value and direction.

(i) Draw the path taken by the electrons in the magnetic field in the
space below. [1]

(ii) Calculate the radius of curvature of these electrons.

radius = ……………….. m [2]

(iii) Determine the speed of the electrons upon leaving the field.
Explain your reasoning.

…………………………………………………………………………………

…………………………………………………………………………………

…………………………………………………………………………………

………………………………………………………………………………[2]

SAJC 2010 Prelims/9646/03 [Turn Over


24

(d) The magnetic field in part (c) above is removed. The speed of the
electron is reduced and the rate of electrons reduced to one per
second. The electrons are allowed to hit a microgrid with a vacuum-
deposited gold film. It was found that high quality interference patterns
are observed on photographic films placed beyond the microgrid.

(i) Explain how the electrons can produce the interference pattern
even when there is only one electron per second approaching
the grid.

…………………………………………………………………………………

…………………………………………………………………………………

…………………………………………………………………………………

…………………………………………………………………………………

…………………………………………………………………………………

………………………………………………………………………………[3]

(ii) State how the interference pattern can be made wider by


changing the properties of the electrons only.

…………………………………………………………………………………

…………………………………………………………………………………

………………………………………………………………………………[1]

SAJC 2010 Prelims/9646/03 End ofOver


[Turn Paper
2010 SAJC H2 Prelim Solutions
Paper 1

Q1 Q2 Q3 Q4 Q5 Q6 Q7 Q8 Q9 Q10
B A B A D B D A A A

Q11 Q12 Q13 Q14 Q15 Q16 Q17 Q18 Q19 Q20
B C D C A D A D D C

Q21 Q22 Q23 Q24 Q25 Q26 Q27 Q28 Q29 Q30
A B C A C B B A D B

Q31 Q32 Q33 Q34 Q35 Q36 Q37 Q38 Q39 Q40
B C B B B D C A A B

Paper 2

Section A

1 [Choose two of the following three sets of answers]

Graph A : [ 1 mark per point, max. 3 marks]


- The direction in which the object is travelling is suddenly reversed.
- This occurs when something hits a vertical wall
- very large acceleration is required
- example : ball bouncing against wall / ball hit by racquet

Graph B : [ 1 mark per point]


- acceleration going from zero to a high value.
- example : letting go of a ball, dropping an egg
- acceleration changes suddenly as the force holding the object in
place is removed

Graph C : [ 1 mark per point]


- constant force being applied drops to zero suddenly.
- example taking foot off car accelerator/ceasing to pedal bicycle
- so zero subsequent acceleration

Generally,

• for explaining what is happening [1]


• for sensible example [1]
• for relating to sketch graph [1]
2 (a) ΔGPE

= mA g ( ΔhA ) + mB g ( ΔhB )

= 2.00 × 9.81× 0.500 sin 40° + 5.00 × 9.81× ( −0.500 sin50°) [1]

= −12.481 J

= −12.5 J (3 sig fig) [1]

(b) Loss in GPE = Gain in KE + Work done against friction [1]

1
mv 2 = 12.481 − 0.500 × 3.00 × 2 [ 1 ] for correct W.D. against friction
2

1
(2.00 + 5.00)v 2 = 9.481
2

v = 1.65 m s-1 (3 sig fig) [1]

{ alternatively :

ΣF = ma => 5gsin50° - 2gsin40° - 3 x 2 = (5 + 2)a


a = 2.709 m s-2
v2 = u2 + 2as => v2 = 02 + 2(2.709)s
v = 1.65 m s-1 }

(c) Energy / J
12.5

9.5 Ek [ 1 ]

3.0 Wf [ 1 ]

Ep [ 1 ]
0
Time
dθ 2π
3 (a) (i) ω = dt = 24 x 60 x 60 = 7.27 x 10-5 rad s-1 [1]

(ii) (Gravi force) – (Tension) = m r ω2 [1]

Tension = 19.66 – (2.00)(6.37x106)(7.27x10-5)2

= 19.59 N [1]

(iii) Part of the gravitational force on the mass supplies the centripetal force
for the mass to move in circular motion. [1]

(b) • Astronaut and spacecraft are both accelerating at the same value towards the
Earth. [1]

• No (normal) reaction force experienced by the astronaut. [1]

4 (a) The magnetic flux density is defined as the force experienced by a


conductor of unit length and carrying unit current, placed at right angles
to the magnetic field. [1]

(b) (i) pointing upwards [1]


(ii) F = BIL [1]

16000
B = 10 x 0.30 = 5330 T [1]

(iii) Advantage: no wear and tear due to moving parts, low noise [1]
Disadvantage: high magnetic field is required [1]

(c) (i)  
y f(x)=5*sin(2*x)*exp(-x/5)

Changing directions [1]

Decreasing magnitude [1]


(ii) Changing direction: As the south pole approaches, the induced
current produced in the coil flows in such a way that a repulsive
force is produce. When the South pole withdraws from the coil,
the induced current flows in the opposite direction so as to
produce an attractive force. [1]

The induced current flowing in the coil will generate heat in the
coil. The energy lost is not recovered by the system. [1]

5 (a) A.C. can be easily stepped up or down using a transformer [1]

(b) To reduce power loss as the transmission current is reduced. [1]

(c) (i) Given: Rc = 6 kΩ, Vload = 80 kW, Power delivered = 250 kW

To show: Ic = 2.61 A

By principle of conservation of energy/power,

Pgen = Power Output + Power Loss in cables, Pc

ie, 250 kW = IcVload + Ic2Rc [1]


= Ic (80 x 103) + Ic2(6 x 103)
Solving, Ic = 2.61A [1]

(ii) Total voltage drop across cables = Ic Rc = 2.61 x 6 x 103

= 15.7 kV [1]

(iii) Power dissipated in cables = Ic2 Rc


= 2.612 x 6 x 103
= 41 x 103 W [1]
{or, = Ic x Ans in (ii)}

(iv) Power delivered to load = Ic x Vload = 2.61 x 80 x 103


= 2.1 x 105 W [1]

{ or, = Pgen - Ploss = 250 kW – Ans in (iii) }

6 (a) The conduction band is the lowest energy band that is empty or
partially filled when the atoms are unexcited. [1]

The valence band is the highest energy band that is completely


populated (by electrons). [1]
(b) (i) The energy gap between the valence and conduction band in an
insulator may be greater than the photon energy of all the given
wavelengths; [1]

These photons will pass through the insulator without being


absorbed. [1]

(ii) Energy levels of electrons in the conduction band are finely


spaced; [1]

Photons of all visible light wavelengths have sufficient energy to


excite an electron in the conduction band to a higher energy
level; [1]

Photons are re-emitted from the surface as reflected light. [1]

(c) The impurities in a p-type semiconductors are trivalent atoms while


the impurities in a n-type semiconductors are pentavalent atoms. [1]

The majority charge carriers in a p-type semiconductors are holes


while the majority charge carriers in a n-type semiconductors are
electrons. [1]

GMm GMm
7 (a) (i) F= r2 Or F=- r2 [1]

where F is the gravitational force


M and m are two point masse
r is the separation between the point masses [1]
GMm 2
(ii) r2 = m r ω [1]

GM = r3 ( T )2
4π2r3
T2 = GM [1]

(b) (i)

moon period Mean distance from centre log10(T/days) log10(r/m)


T/days of Jupiter r/109m

Sinope 758 23.7 2.88 10.37


Leda 239 11.1 2.38 10.05
Callisto 16.7 1.88 1.22 9.27
Lo 1.77 0.422 0.248 8.63
Metis 0.295 0.128 -0.530 8.11
(allow 2 slips) [1]
(ii)

(10.45, 3.00)

(7.80, -1.00)

• for plotting 5 plots [1]


• for a straight line which passes almost perfectly through the 5 plots
[1]
3.00 - (-1.00)
(c) (i) gradient = 10.45 - 7.80
= 1.51 [1]
(accept 1.43 to 1.59)
4π2r3
(ii) T2 = GM
4π2
2 lg T = lg ( GM ) + 3 lg r

4π2 3
Show: lg T = ½ lg( GM ) + 2 lg r [1]

Conclusion: Data support the relation because

• straight line graph is obtained [1]

• gradient of about 1.5 is obtained [1]

(d) When T = 7.16 days, lg T = 0.85 [1]

From the graph, lg r = 9.02

r = 1.05 x 109 m [1]


(accept 0.95 – 1.16 x 109 m)

(e) • No, because a different planet has a different mass M. [1]

• Although the gradient remains the same, the y-intercept changes. [1]

Diagram and Procedure Marks

Labelled diagram of workable (with high voltage source) experimental 1


arrangement {Note : Battery as voltage source not acceptable}

Measure pressure p using pressure gauge 1


(instrument shown on diagram is acceptable)

Measure high voltage V using a voltmeter/CRO 1


(instrument shown on diagram is acceptable)

Change pressure using {air/vacuum} pump 1

Keep air gap constant, or 1

Keep type of gas constant (specifically mentioned)

Keep pressure constant (or set p to a certain value), vary voltage V slowly until 1
sparks occur.

Repeat experiment several times using different pressure 1


Analysis

Plot a graph of lg V against lg p. 1

Good design/experimental details

1) perform experiment in the dark {hence enhance visibility of spark}

2) increase V slowly until sparks are seen 1 mark


each,
3) describe a suitable pressure chamber (eg air-tight container with {toughened} max. 4
glass sides)

4) use goggles to view spark {possible danger of uv radiation}

5) use safety screen around pressure chamber {in case of explosion/implosion}

6) Repeat measurement of voltage V, and taking the average

Total 12
Paper 3

Section A

1 (a) (i) Energy carried by alpha particle = 5 x 106 x 1.6 x 10-19


= 8.0 x 10-13 J [1]
20
Activity required = 8.0 x 10-13 = 2.5 x 1013 Bq [1]

0.693
(ii) λ= 88 x 365 x 24 x 3600 = 2.5 x 10-10 s-1 [1]

A
(iii) Number of nuclei required, N = λ [1]

2.5 x 1013
= 2.5 x 10-10

= 1.0 x 1023 [1]

N Mr 1.0 x 1023 x 0.238


Mass required = NA = 6.02 x 1023
= 0.040 kg [1]

(iv) Rocket may burn up in atmosphere during launch or


re-entry and failure [1]

But plutonium would still be radioactive and being vaporized, it


could be ingested. [1]

(b) Iodine-131 should be stored for a longer period of time because its activity
decreases more slowly and hence will remain hazardous for a longer period of
time. [2]

2 (a) (i) Amplitude = 2×10-11 m [1]

Frequency = 500 Hz [1]

1
(ii) Max KE = mv 0 2 = 2.4 × 10−19
2

1
m(2π fx0 )2 = 2.4 × 10 −19 [1]
2

1
m(2π × 500 × 2 × 10−11 )2 = 2.4 × 10 −19 [ 1 ] for correct v0
2

m = 1.22 × 10 −4 kg (3 sig fig) [1]


(iii) [ 1 ] for correct shape

[ 1 ] for correct labels

62.8

−0.02 0.02

−62.8

(b) Consider the Vuvuzela as a tube with two open ends.

At the fundamental frequency,

λ v 330
= 0.70 , f = = = 236 Hz [1]
2 λ 1.4
The next few harmonics will be multiples of the fundamental frequency, hence
236×2 = 472 Hz and 236×3 = 708 Hz. [1]

By generating sound at the same resonant frequencies as well as almost


similar amplitudes [1]

as the vuvuzela but 180º out of phase, [1]


destructive interference occurs and both sound waves cancel out.

3  (a) All electrons emitted from the surface will be accelerated towards the collector

by a small positive potential [1]

(b) Electrons with kinetic energies less than the maximum kinetic
energy will be stopped by the negative potential, which explains
the decreasing current. [1]

The current drops to zero when the most energetic electrons are
stopped by the potential. [1]
I / mA 

(c)

‐Vs  0  V ext / mV 

Intensity x area = (no of photons/time ) x (hf)

Lower current (because of higher f) [1]

More negative stopping potential [1]

(d) (i) Energy of each photon = 4.7 x 1.6 x 10-19


= 7.52 x 10-19 J [1]
3.8 x 10-3
Rate = energy of photon = 5.1 x 1015 s-1 [1]

I 0.8 x 10-8
(ii) Rate of electrons = e = 1.6 x 10-19
= 5.0 x 1010 s-1 [1]

(iii) Most of the photons are reflected, scattered or absorbed


by the metal. [2]

{one mark per underlined, maximum two marks}

4 (a) (i) Current through A1, I1 = 0.50 + 0.50 = 1.0 A [1]

e.m.f. = V + I1 r

2.0 = V + 1.0 (0.20)

V = 1.80 V [1]

(ii) • Connecting L3 parallel to L1 and L2 will lower the effective resistance of


the circuit. [1]

• hence, current in A1 increases [1]

2.0
(b) (i) VAB = ( 8.0 + 2.0 ) 2.00

= 0.40 V [1]
(ii) At balance point,

e.m.f. of Cell Y = VJB

0.40
= 100.0 x 70.0

= 0.28 V [2]

(iii) • Replace the resistance wire with one of lower resistivity [1]

• Replace Cell X with one of lower e.m.f. [1]

• Replace the 8.0 Ω resistor with one of higher resistance. [1] 

5 (a) (i) Newton’s 1st law: Every object continues in its state of rest or
uniform motion in a straight line unless a net (external) force acts
on it to change that state. [1]

The law implies that a “force” is that quantity which is required to

change velocity. [1]

d ( mv) d ( mv)
(ii) Mathematically, resultant force, F ∝ =k [1]
dt dt
(where the value of the proportionality const k would depend on
the definition of the unit of force.)

Since the definition of the newton was chosen as “the force


which causes a mass of 1 kg to have an acceleration of 1 m s-2,

thus, 1N = k x 1 kg x 1 m s-2
k =1 [1]

d (mv) dv
Thus, F = = m dt (since m = const) [ 1 ]
dt
= ma

(b) As the stone accelerates towards Earth, its downward momentum is increasing
from an initial value of zero momentum. [1]

Since there is no net force acting on the system of stone & Earth, the total
momentum of the stone-Earth system must remain unchanged, ie = 0. [1]

Thus, the Earth must accelerate upwards towards the falling stone. [1]
{Assume air resistance is negligible}

(c) (i)

[1]
224
88 Ra → 220
86 Rn + 24He
(ii) Given KEα = 9.2 x 10-13 J

Thus ½ mαvα2 = 9.2 x 10-13 J, where mα = 4u = 4x1.66x10-27 kg [1]

2 x9.2 x10 −13


→ Vα =
4 x1.66 x10 −27
= 1.67 x 107 = 1.7 x 107 m s-1 (proved) [1]

(iii) Since no net force is involved in radioactive decay, & since total
momentum before decay = zero (as Ra is initially stationary),
[1]

by principle of conservation of momentum,


mαvα = mRnvRn [1]

mαvα
Thus, vRn = m
Rn

(4u)(1.7 x 107)
= 220u
= 3.1 x 105 m s-1 [1]

(d) (i) Centre of gravity of an object is defined as that point through which

the entire weight of the object may be considered to act. [1]

(ii) Principle of Moments:


For a body to be in equilibrium, the sum of all the anticlockwise
moments about any point must be equal to the sum of all the
clockwise moments about that same point. [1]

(iii)

Moment exerted by the weight about the base


= mg x (2.3cos450) [1]

= 180 x 9.81 x 2.3cos450 [1]

= 2.9 x 103 N m (shown)


(iv) Applying the principle of moments about the base,

T x (4.0 sin350) = 2.9 x 103 [1]

→ T = 1.26 x 103 = 1.3 x 103 N [1]

6 (a) (i) internal energy = sum of p.e. + k.e. of molecules [1]


no p.e. as no force of attraction [1]
mean k.e. of molecules α absolute temperature [1]

(ii) minimum k.e. of molecules is 0 [1]


minimum absolute temperature is 0 (or cannot be negative) [ 1 ]

(b) (i) AB: k.e. increases with temperature [1]


separation, and thus p.e. remains constant or increase very
slightly [1]

(ii) BC: k.e. remains constant as temperature constant [1]


separation, and thus p.e increases greatly; as change of
phase/state occurs [1]

(iii) CD: k.e. increases with temperature [1]


p.e. separation remains constant [1]

m
( V )RT
m m nRT
(c) (i) n=M;ρ= V; p= V = M [1]
ρRT
= M [1]

(ii) suitable test, i.e. ratio test, half-height, etc [2]

test carried out successfully [1]

(iii) p at 8 km = 3.5 ± 0.3 x 104 Pa [1]

p ρ
p o = ρo
ρ = 0.35 x 1.3 = 0.455 (kg m-3) [1]

p
(iv) ρT = constant
105 3 x 104
1.3 x 293 = ρ x 250 [1]
ρ = 0.457 kg m-3 [1]
80 x 10-3
7 (a) (i) Time = 2.50 x 107 = 3.2 x 10-9 s [1]

Vy
(ii) tan 30º = 2.50 x 107 [1]

Vy = tan 30º x 2.5 x 107

= 1.44 x 107 ms-1 [1]

Vy
(iii) Acceleration = t = 4.51 x 1015 ms-2 [1]

eE e(V/d)
(iv) a = m = m [1]

adm
V = e = 1.02 x 103 V [1]

(b) (i) Pointing into the page [1]

(ii) electric force = magnetic force

eE = Bev [1]

E V
B = v = dv

1.02 x 103
= 40 x 10-3x 2.5 x 107
= 1.03 x 10-3 T [1]

(iii) All the energy of the electron is converted to photon energy

hc
½ mv2 = [1]
λ

λ = 6.99 x 10-10 m [1]

(c) (i) [1]


mv2
(ii) Bev = r [1]

r = 0.138 m [1]

(iii) Same speed [1]

The magnetic force does not do any work on the electron. [1]

(d) (i) The electron behave like waves. [1]


The electron wave diffract around the micro-grid [1]
The diffracted wave interferes with each other to produce
the pattern. [1]

(ii) Using electrons with longer wavelength [1]


(or using lower potential)
1

SERANGOON JUNIOR COLLEGE


General Certificate of Education Advanced Level
Higher 2

NAME

CG INDEX NO.

PHYSICS 9646
Preliminary Examination 26 August 2010
Paper 1 Multiple Choice 1 hour 15 minutes

Additional Materials: Optical Mark Sheet (OMS)

READ THIS INSTRUCTIONS FIRST


Write your name, civics group and index number in the spaces at the top of this page.

There are forty questions on this paper. Answer all questions. For each question, there
are four possible answers labeled A, B, C and D.

Choose the one you consider correct and record your choice in soft pencil on the OMS.

Read the instructions on the OMS very carefully.

Each correct answer will score one mark. A mark will not be deducted for a wrong answer.
Any rough working should be done in this question paper.

For Examiner’s Use

Section A

Total / 40

This document consist of 21 printed pages and 1 blank page

SRJC 2010 9646/Prelim/2010


2

DATA AND FORMULAE


Data
speed of light in free space, c = 3.00 x 108 m s1
permeability of free space, 0 = 4 x 107 H m1
permittivity of free space, 0 = 8.85 x 1012 F m1
(1 / (36π)) x 109 F m1
elementary charge, e = 1.60 x 1019 C
the Planck constant, h = 6.63 x 1034 J s
unified atomic mass constant, u = 1.66 x 1027 kg
rest mass of electron, me = 9.11 x 1031 kg
rest mass of proton, mp = 1.67 x 1027 kg
molar gas constant, R = 8.31 J K1 mol1
the Avogadro constant, NA = 6.02 x 1023 mol1
the Boltzmann constant, k = 1.38 x 1023 J K1
gravitational constant, G = 6.67 x 1011 N m2 kg2
acceleration of free fall, g = 9.81 m s2

Formulae
uniformly accelerated motion, s = ut + ½ at2
v2 = u2 + 2as
work done on/by a gas, W = pV
hydrostatic pressure, p = gh
Gm
gravitational potential,  = –
r
displacement of particle in s.h.m., x = x0 sin t
velocity of particle in s.h.m., v = vo cost
v = ω x 2
0  x2 
resistors in series, R = R1 + R2 + …
resistors in parallel, 1/R = 1/R1 + 1/R2 + …
electric potential, V = Q / 4or
alternating current/voltage, x = x0sin t
transmission coefficient, T α exp(2kd)
8 π 2 m(U  E)
where k =
h2
radioactive decay, x = x0 exp(t)
0.693
decay constant,  =
t1
2

SRJC 2010 9646/Prelim/2010


3

1 In a speed test, an engineer has determined the top speed of an automobile to be


240.57 km h-1. The accuracy of his equipment is ±5%.

Which of the following is the correct representation of his results?

A (241 ± 2) km h-1 B (240 ± 10) km h-1


C (241 ± 12) km h-1 D (240.57 ± 12.03) km h-1

Ans: B

V/V = 0.05
V = 0.05 x 240.57
= 10 km h-1
V = (240 ± 10) km h-1

2 A certain particle is travelling at a uniform velocity of 20 m s-1. From t = 2 s onwards, it


decelerates uniformly to 5 m s-1, in 40.0 m.

From t = 0 s, what is the distance needed for the particle to come to rest if it continues at the
same uniform deceleration?

A 2.7 m B 22.7 m C 42.7 m D 82.7 m

Ans: D

v2 = u2 + 2as
52 = 202 + 2a(40)
a = -4.688 m s-2

v2 = u2 + 2as
02 = 52 + 2(-4.688)s
s = 2.666 m

Total distance needed = (20)(2) + 40 + (2.666) = 82.7 m

SRJC 2010 9646/Prelim/2010


4

3 The figure below shows the acceleration-time graph of an object moving in the positive
direction.

-2
a/ms

0 3 10 t/s

Which of the following statements best describes what is happening to the object from
t = 3 s to t = 10 s?

A The object is slowing down at a slower rate as time passes


B The object is slowing down at a faster rate as time passes
C The object is speeding up at a slower rate as time passes.
D The object’s speed remains constant as time passes.

Ans: C

4 A heavy metal ball suspended from an elastic string is fully immersed in oil in a beaker,
resting on a scale balance.

The force that the ball exerts downwards on the oil in the beaker forms an action-reaction
pair with

A the weight of the metal ball


B the average tension in the string
C the Archimedean upthrust on the object
D the force which the oil-filled beaker exerts on the scale balance

Ans: C

When the ball exerts a force downwards on the oil, the oil reacts by exerted an equal and
opposite force upwards on the ball. This upward force is the upthrust.

SRJC 2010 9646/Prelim/2010 [Turn Over


5

5 Alice, whose actual weight is 540 N, registers to her delight, a reading of 500 N on a
weighing machine, when she is in the lift of the five-star hotel she is staying in.

Which of the following situations is happening?

A The lift is decelerating and moving upwards


B The lift is accelerating and moving upwards
C The lift is decelerating and moving downwards
D The lift is moving with a constant velocity.

Ans: A N, normal contact force between Alice and


floor of lift
N – mg = ma a
N = m(g + a)
N = mg + ma
Alice’s actual weight = mg Alice
Thus, ma has to be negative, a is negative.
i.e. a is opposite to assumed upwards direction.

mg

6 What volume of a 300 cm3 ice is above water that has a density of 1.0 g cm-3? The ice has a
density of 0.9 g cm-3.

A 15 cm3 B 30 cm3 C 270 cm3 D 300 cm3

Ans: B

Weight of ice = upthrust of water on ice


ρiVig = ρwVxg, where Vx = volume of ice below water
Let volume of ice above water = Vy, then Vi = Vx + Vy

ρiVig = ρw(Vi - Vy)g


   i 
Vi =  w Vi
 w 
 1.0  0.9 
=   (300)
 1.0 
= 30 cm3

SRJC 2010 9646/Prelim/2010


6

7 Which of the following correctly describes the situation where an object is floating in a body
of liquid correctly?

The weight of liquid which would occupy a volume equal to the volume of the object

A must be equal to the weight of the object


B below the fluid surface must be equal to the weight of the object below the fluid surface
C below the fluid surface must be equal to the weight of the object
D above the fluid surface must be equal to the weight of the object below the fluid surface

Ans: C

8 A car is moving upwards along a slope with a constant speed.

Which of the following statements best describe the conversion of energy taking place?

The energy output from the engine is transformed into

A an increase in gravitational potential energy of the car.

B an increase in gravitational potential energy and kinetic energy of the car.

C an increase in gravitational potential energy as well as work done against resistive


forces.

D an increase in gravitational potential energy and kinetic energy of the car, as well as
work done against resistive forces.

Ans C
Energy from the engine is not used to transform into kinetic energy as there is no gain in the
kinetic energy. The energy is transformed into an increase in gravitational potential energy as
well used to do work against the frictional forces.

SRJC 2010 9646/Prelim/2010 [Turn Over


7

9 An object is pushed from rest in a straight line by a variable force F along a rough ground.
The ground exerts a constant frictional force of 5 N throughout the motion of the object. The
graph of how the variable force F varies with displacement x is shown below.

F/N

K
5
L M

10 x/m

The magnitude of the kinetic energy of the object when it has travelled 10 m is

A the area K.
B the area K + L.
C the area K + L + M.
D impossible to obtain as mass of object is not given.

Ans A
Gain in KE = Net Work Done
= Area under resultant force – time graph
= Area K

SRJC 2010 9646/Prelim/2010


8

10 A metal sphere is hanging on a string. The metal sphere is then swung around in a horizontal
circle as shown.

Which of the following shows the correct free-body diagram of the sphere at this instance?

Tension Tension

A B Centripetal
Centripetal force
force

Weight Weight

Centripetal
force Tension
C D

Weight
Weight

Ans: D

11 Which of the following statement is true for an object in horizontal circular motion with
constant angular velocity?

A The kinetic energy remains constant.


B The kinetic energy and linear momentum remain constant.
C The linear velocity does not vary.
D Acceleration remains constant.

Ans: A. The kinetic energy remains constant.

12 The distance between a point, X, in space and the centre of the Earth is 25 times the radius
of Earth. The gravitational field strength due to Earth at that point is

A 1.56 x 10-2 N kg-1. B 1.57 x 10-2 N kg-1.


C 3.90 x 10-1 N kg-1. D 3.92 x 10-1 N kg-1.

SRJC 2010 9646/Prelim/2010 [Turn Over


9

Ans: B
GM E
gX = ……….. (1)
 rX  2

GM E
Since gE =  gE (rE)2 = GME
 rE  2

gE  rE 
2
9.81
Hence from (1) : gX = = = 1.57 x 10-2 N kg-1
 25rE  2
625

13 A satellite in orbit encounters significant friction and spirals gradually to Earth.

Which of the following statements is correct?

A The satellite gains gravitational potential energy.


B The satellite loses kinetic energy.
C The total energy (gravitational potential energy and kinetic energy) of the satellite
remains constant.
D The total energy (gravitational potential energy and kinetic energy) of the satellite
decreases.

Ans: D

A satellite spirals towards Earth when it encounters significant resistance in the atmosphere.
The satellite’s total energy decreases because some of the energy is converted to heat. As
the satellite falls, its gravitational potential energy reduces while its kinetic energy increases.

14 The total energy of a horizontal oscillating mass spring system is doubled. This will cause its
amplitude to

A increase by 2. B increase by 2.
C increase by 2 2 . D increase by 4.

Ans: A
1 2
TE = kA
2
1
kA12
TE1
Therefore,  2
TE2 1
kA22
2
TE1 A12

2TE1 A22
2TE1 2
A22  A1
TE1
A2  2A1
SRJC 2010 9646/Prelim/2010
10

15 A toy car moving along a horizontal plane in simple harmonic motion starts from the
amplitude at t = 0 s. If the amplitude of its motion is 5.00 cm and frequency is 2.00 Hz, the
magnitude of the acceleration of the toy car at 1.7 s is

A 0.25 m s-2 B 0.51 m s-2


C 1.60 m s-2 D 6.40 m s-2

Ans: D
x = x0 cos (ωt)
= 5.00 cos (4.00π (1.7))
= - 4.05 cm
2
a = ω x = (4π)2(4.05 x 10-2) = 6.40 m s-2

16 An ideal gas underwent an isothermal expansion from state A to state B, followed by an


isobaric expansion from state B to state C and an adiabatic contraction from state C back to
state A.

Which of the following is a correct description of the net work done by the ideal gas after
completing one full cycle of the above process?

A Net work done by gas is zero because the gas returns to its initial state.
B Net work done by gas is positive because there is a net heat loss during isobaric
expansion and no heat lost during isothermal and adiabatic process.
C Net work done by gas is negative because the gas expands through two processes
and contracts in one process only.
D Net work done on gas is positive because the adiabatic process occurs at a higher
average pressure than the other two processes.

Ans: D

17 3.0 mol of ideal gas at an initial pressure of 6.0 x 10-5 Pa expanded to 3 times its original
volume while the temperature reduced by half, from 20 ۫C to 10 ۫C.

What is the final pressure of the gas?

A 1.00 x 10-5 Pa B 1.74 x 10-5 Pa


C 1.93 x 10-5 Pa D 2.07 x 10-5 Pa

Ans: C
pV  nRT
pV pV
( )i  ( )f
T T
V Tf .
V 28315
Pf  i ( )Pi  ( )Pi  1.93  105 Pa
V f Ti .
3V 29315

SRJC 2010 9646/Prelim/2010 [Turn Over


11

18 The diagram shows two waves X and Y.

displacement
Wave X
Wave Y

0 time

Wave Y has amplitude 8 cm and frequency 150 Hz.

What are the possible amplitude and frequency of wave X?

amplitude / cm frequency / Hz
A 4 50
B 4 450
C 16 50
D 16 450

Ans: C

Since there are three complete cycle of wave Y in wave X for the same time duration, hence
frequency of wave X is one third that of wave Y, i.e. 150/3 = 50 Hz.

And its amplitude is twice that of wave Y so wave X amplitude


= 8 x 2 = 16 cm.

19 Light can exhibit all of the phenomena listed.

Which phenomenon can sound not exhibit?

A interference B polarisation
C refraction D total internal reflection

Ans: B
Sound waves are longitudinal waves. Hence polarisation is not evident in sound waves.

SRJC 2010 9646/Prelim/2010


12

20 The diagram shows a standing wave on a string. The standing wave has three nodes N1, N2
and N3.

N1 N2 N3

Which statement is correct?

A All points on the string vibrate in phase.


B All points on the string vibrate with the same amplitude.
C Points equidistant from N2 vibrate with the same frequency and in phase.
D Points equidistant from N2 vibrate with the same frequency and the same amplitude.

Ans: D
A standing wave will have particles in phase with each other in a loop and in anti phase with
particles in the adjacent loop. Hence the particles equidistant from N2 would only have equal
amplitude and not similar phase.

21 A beam of monochromatic light of wavelength 550 nm is incident normally on a diffraction


grating that has 300 lines per mm.

What is the total number of images produced by light transmitted through this grating?

A 6 B 7 C 12 D 13

1x103
Ans: d = = 3.33 x 10-6 m
300
d sin θ = nλ
For maximum images sin θ = 1
d 3.33 x 10-6
n= = = 6.06
 550 x 10-9
Hence max no of images = 2 x 6 + 1 = 13

SRJC 2010 9646/Prelim/2010 [Turn Over


13

22 3 charges X, Y and Z of charge 3.2 x 10-16 C, -1.4 x 10-16 C and -0.8 x 10-16 C respectively,
are placed at the corners of an equilateral triangle of sides 1.7 cm long, as shown below.

1.7 cm

Y
Z

What is the electric potential at position of charge Y due to X and Z?

A 7.47 x 10-3 V B 1.27 x 10-4 V


C 2.12 x 10-4 V D 1.78 x 10-20 V

Ans: B
1
V ( 3.2  1016  0.8  1016 )  1.27  104 V
4 0 r

SRJC 2010 9646/Prelim/2010


14

23 Two large parallel plates are situated 50 mm apart as shown. The potential difference
between the plates is +V.

0V +V
X

10 mm
Y

50 mm

A unit positive charge moves from X to Y.

Which graph shows the variation of electric potential energy U of the unit positive charge with
respect to the distance s measured perpendicularly from the plate of zero potential as the
charge moved from X to Y?

U
U
A B

s
s

C D

U U

s s

Ans: A
dV
As field strength is constant, is a constant and hence, the gradient is constant and the
ds
graph is a straight line graph which does not pass through the origin.

SRJC 2010 9646/Prelim/2010 [Turn Over


15

24 Four identical lamps are powered by an e.m.f source.

Which of the following will cause the batteries to use up all its energy in the shortest time?

The bulbs are

A all arranged in parallel.


B all arranged in series.
C arranged in combination of series and parallel.
D arranged such that there are an equal number of lamps arranged in series and parallel.

Ans: A. The lamps will be brightest when they are all arranged in parallel.

25 Which of the following statements is true about the e.m.f. of a cell?

A It is the electrical force required to move a unit charge within a circuit.


. B It is the electrical power changed into other forms per unit charge within the cell.
C It is the electrical energy supplied per unit current by the cell.
D It is the electrical energy supplied per unit charge by the cell.

Ans: D. It is the electrical energy supplied per unit charge by the cell.

26 Jackie arranged a circuit setup as shown below. The ideal voltmeter reads 7 V when J1 is
closed and J2 is opened. The voltmeter reads 9 V when J2 is closed and J1 is opened.

J1 J2

2Ω 3Ω

What will the voltmeter reading be when both J1 and J2 are closed?

A 4.55 V B 4.70 V C 4.85 V D 5.05 V

Ans: C
Let e.m.f. of source be E and internal resistance of source be r
Scenario 1: J1 closed and J2 open:
2
7= x E ------------------------(1)
2r

Scenario 2: J1 open and J2 closed:


3
9= x E ------------------------(2)
3 r
9 3 2r
 
7 3 r 2
18 (3+r) = 21 (2+r)
SRJC 2010 9646/Prelim/2010
16

54 + 18r = 42 + 21r
3r = 12
r = 4 Ω subst in (1)
2
7= xE
24
E = 21 V

Scenario 3: J1 closed and J2 closed:


23 6
Total resistance (not including the internal resistance) =  
23 5
6
Voltmeter reading = 5  21
6
4
5
= 4.85 V

27 An 18 V battery of negligible internal resistance connected to a potentiometer X of total


resistance 150 Ω as shown below. Filament lamps L1 and L2 are connected as shown with
the sliding contact positioned such that there is equal division of the resistance of X. Both
lamps are marked ‘9 V, 27 W’.

18 V

X = 150 Ω

L1 L2

Which of the following will be observed when switch S is closed?

A L1 will not be as bright as L2


B L1 will remain as bright as before.
C L1 will be dimmer than before S was closed.
D L1 will be brighter than before S was closed.

Ans: D
V 2 92
R  =3Ω
P 27
Scenario 1: Switch S open
Potential difference across L1 much less than 9 V because effective resistance is much less
than 75 Ω. L1 is dim.

SRJC 2010 9646/Prelim/2010 [Turn Over


17

18 V

75 Ω 75 Ω

L1

Scenario 2: Switch S closed


Potential difference across L1 will be 9 V, and across L2 will also be 9 V. Both lamps will have
the normal brightness.
18 V

75 Ω 75 Ω

3Ω 3Ω

L1 L2

28 An electric charge present in a magnetic field of experiences a non-zero magnitude constant


force when

A the charge moves at an oblique angle to the magnetic field lines with constant speed.
B the charge moves perpendicular to the magnetic field lines with increasing speed.
C the charge moves in the same direction as the magnetic field lines with constant
speed.
D the charge moves in the same direction as the magnetic field lines with increasing
speed.

Ans: A.

29 The forces between two parallel wires with current flowing

A in the same direction causes the wires to attract.


B in the same direction causes the wires to repel.
C is independent of the current directions.
D is in a direction that depends on the magnitude of the currents.

Ans: A. The force between two parallel wires with current flowing in the same direction
causes the wire to attract.

SRJC 2010 9646/Prelim/2010


18

30 A student uses a long copper wire to form a square coil with side 7 cm of 50 turns. He places
it such that it is perpendicular to a magnetic field. The flux density of the field is changed from
10 mT to 60 mT in 2.0 s.

What is the average e.m.f. induced in the coil?

A 6.1 mV B 7.4 mV
C 12 mV D 74 mV

Ans: A.
A = 0.07 x 0.07 = 0.0049
e.m.f. = N/t
= 50 x 0.0049 x (60 - 10) x 10-3 / 2.0
= 6.1 mV

31 Which of the following lists are all forms of conservation of energy?

A First Law of thermodynamics, Lenz’s Law and Conservation of linear momentum.


B First Law of thermodynamics, Lenz’s Law and Einstein’s photoelectric equation
C Lenz’s Law, Einstein’s photoelectric equation and Conservation of linear momentum.
D First Law of thermodynamics, Einstein’s photoelectric equation and Conservation of
linear momentum.

Ans: B

32 A varying d.c. current is passed through a certain resistor as shown below.

I/A

6.0
4.0
2.0
0 t/s
2.0 4.0 6.0 8.0 10.0 12.0 14.0 16.0 18.0 20.0

The r.m.s value of the given current is

A 4.10 A B 5.80 A C 8.49 A D 16.8 A

Ans: A
I2/A2
36.0

16.0
4.0
0 t/s
2.0 4.0 6.0 8.0 10.0 12.0 14.0 16.0 18.0 20.0

SRJC 2010 9646/Prelim/2010 [Turn Over


19

Area = [(16.0 x 8.0) + (20.0 x 2.0)] = 168

area under graph for one period


root-mean-square e.m.f 
one period
168

10.0
 4.10 V

33 The output voltage of a certain generator is V = V0 sin 2πft. The frequency f and the
amplitude V0 are directly proportional to one another. The generator is connected to a
resistor.

When the frequency is varied, the instantaneous peak power developed in the resistor is

A proportional to f2
B proportional to f
C inversely proportional to f2
D inversely proportional to f

Ans: A
V0 2
Peak power, P0 
R
Since V0  f ,
f2
P0 
R

34 In a typical photoelectric effect experiment, monochromatic light is irradiated onto a metal


surface. The graph below shows the results of how y varies with x.

x
0

What is y and x?
y x
A p.d across emitter and collector frequency of radiation
B Intensity of source frequency of radiation
C photocurrent intensity of source
D photocurrent p.d across emitter and collector

Ans D
SRJC 2010 9646/Prelim/2010
20

35 The intensity of a beam of monochromatic light is halved. Which one of the following
represents the corresponding change if any in the momentum of each photon of the radiation?

A halved B one quarter of original


C the same D doubled

Ans C
By de Broglie’s equation
h
p= .

However, changing the intensity of the light source does not change any of the variables in
the equation, therefore the momentum of the photon is unchanged.

36 The maximum energy of photons emitted from an X-ray tube is certain to be decreased by

A using a target metal with a higher atomic number.


B decreasing the voltage across the tube.
C cooling the metal target.
D using a target metal with a larger interatomic spacing.

Ans B
The maximum energy of photons emitted is emitted when all the KE of an incident electron is
transformed into a E-M radiation.
The KE of the electrons is reduced when the voltage across the tube is decreased.

37 An electron has kinetic energy of 0.5 MeV. If its momentum is measured with an uncertainty
of 1.00 %, what is the minimum uncertainty in its position?

A 1.38  1011 m B 1.38  1012 m


C 3.30  1020 m D 3.30  1021 m

Ans A
0.5 x 106 x 1.6 x 10-19 = ½ mv2
½ mv2 = (0.5)( 9.11 x 1031) v2
v = 419.08 x 106 m s-1

p = mv = ( 9.11 x 1031)( 419.08 x 106) = 3.818 x 10-22


1.00
Δp = 3.818 x 10-22 x = 3.818 x 10-24
100

Using Heisenberg’s uncertainty principle, x p 


2
 6.63×10 -34

 

x  =   = 1.38×10-11 m
2p 2×3.818×10 -24

SRJC 2010 9646/Prelim/2010 [Turn Over


21

38 Which of the following statements about a semiconductor diode in reversed-bias is correct?

A The p-type material of the diode is connected to the positive terminal of the source of
e.m.f.
B Electrons in the n-type side of the diode will cross steadily to the p- type side.
C The applied potential difference from the source of e.m.f. reinforces the junction
potential.
D The depletion layer is reduced.

Ans C

219
39 The half-life of 86 Rn is 4.0 s.

What is the activity after 3.4 s when the initial sample size at time t = 0 s is 1.5 mol of
219
86 Rn ?

A 0.144 Bq B 8.68 x 1022 Bq


C 1.33 x 1023 Bq D 7.68 x 1023 Bq

Ans: B
A  A0 e  t
A0   N 0  ( 6.02  1023  1.5 )
ln 2
 ( 3.4 )
A  ( 6.02  1023  1.5 )e 4
 8.68  1022 Bq

40 238
92 U decays through a series of transformation. It underwent one α-emission, two β-
emissions and one γ-emission.

Which of the following is the final product of this series of transformation?

236 234 238 234


A 91 Pa B U
92 C U
92 D 93 Np

Ans: B
92U 
238 234
92 U  24  2 10  00

SRJC 2010 9646/Prelim/2010


SERANGOON JUNIOR COLLEGE
General Certificate of Education Advanced Level
Higher 2

NAME

CG INDEX NO.

PHYSICS 9646
Preliminary Examination 20 August 2010
Paper 2 Structured Questions 1 hour 45 minutes

Candidates answer on the Question Paper.


No additional Materials are required.

READ THIS INSTRUCTIONS FIRST


Write your name, civics group and index number in the spaces at the top of this page.
Write in dark blue or black pen on both sides of the paper.
You may use a soft pencil for any diagrams, graphs or rough working.
Do not use staples, paper clips, highlighters, glue or correction fluid.

Answer all questions.

At the end of the examination, fasten all your work securely For Examiner’s Use
together.
The number of marks is given in bracket [ ] at the end of each 1
question or part question.
2

Total

This document consist of 20 printed pages and no blank page

SRJC 2010 9646/Prelim/2010


For
1 Examiner’s
Use

C 4D F 12
1) (a) In a certain equation, A =
E 2 (B1  3B2 ) H 4

where C has unit Newtons, D has the unit metres, E has the unit Pascal, B1 and B2
have the unit metres, F has the unit metres, and H has the unit Joules.

(i) Determine the unit of A. [2]

1 Pa = 1 kg m s-2 m-2
1 N = 1 kg m s-2
1 J = 1 kg m2 s-2

[A] =
kg m s  2 4
m m12
kg m s m 2  m kg m s 2 
2 2 2 4

= m8

unit of A = ……………… [1]

(ii) The value of C is (2.05  0.02) x 10-9 m, what percentage uncertainty does this
introduce into the value of A?

percentage uncertainty = ………………% [1]

0.02
percentage uncertainty introduces into the value of Q = 4 x x 100% = 3.90 %
2.05

(b) Peter decides to go skydiving. From an altitude of 3500 m, he jumps off the aircraft. He
delays opening his parachute until he reaches 800 m. He attains terminal velocity
before he reaches 800 m. Assume that he falls vertically.

(i) Describe qualitatively, and with an explanation, Peter’s motion after he jumped
off till he reaches terminal velocity.

………………………………………………………………………………….…………

……………………………………………………………………………...…………..…

..................................................………………………………………..………….. [1]

Peter’s acceleration downwards decreases (or velocity increases at a decreasing rate),


as the weight remains the same but the air resistance increases with increasing
velocity, resulting in the net force downwards decreasing with time, till terminal velocity
is reached when the weight is equal to the air resistance.

SRJC 2010 9646/ Prelim/2010 [Turn Over


For
2 Examiner’s
Use
(ii) When Peter opens his parachute, his velocity is 55 m s-1 downwards. His landing
speed is 4 m s-1.

Sketch a velocity- time graph from the time Peter has just started falling at t = 0 s
until he is about to land. The time till which Peter lands is not needed.

Note that when the parachute is first opened, the force by the parachute on Peter
is larger than his weight. [2]
-1
v/ m s

t/s
0

-1
v/ m s

55.0

4.0
t/s
0

(iii) Explain the shape of the graph from the time the parachute opens till the landing.
……………………………………………………………………………………………

……………………………………………………………………………………………

……………………………………………………………………………………………

…………………………………………………………………………………………[2]

The velocity is decreasing at a decreasing rate because the net force is upwards.

However, as the net force upwards is decreasing with time, the deceleration is
decreasing. Hence the gradient is getting less negative, until it is equal to 0 m s-2 where
terminal velocity is reached.

SRJC 2010 9646/ Prelim/2010


For
3 Examiner’s
Use

2 (a) A uniform plank of mass 40.0 kg and length 2.0 m is held horizontally by two identical
supports at points A and B. Point B is 0.5 m away from end of beam at point C as
shown in Fig. 2.1.

plank
A B C

0.5 m

2.0 m

Fig. 2.1

(i) On Fig. 2.1, draw the three forces acting on the plank. [1]

(ii) Calculate the force provided by the support at point A.

Force at A = .......................... N [1]

Taking moments about B,


mg x 0.5 = FA x 1.5
Fa = 130.8 N

(iii) Consider another scenario where a person pushes the support at point A towards
the middle of the plank steadily. With the aid of clear calculations and/or
explanations, state how the force on the plank at point B changes as the support
at point A is shifted.

...............................................................................................................................

...............................................................................................................................

............................................................................................................................[2]

Let the distance of the pivot on the left from the centre of gravity be SA
Let the distance of the pivot on the right from the centre of gravity be SB

Fa + Fb = (40.0) (9.81) = 392.4 N


Fa = 392.4 - Fb

Taking moments about the centre of gravity,

Fa x Sa = Fb x Sb

SRJC 2010 9646/ Prelim/2010 [Turn Over


For
4 Examiner’s
Use
Fa Sb

Fb Sa
392.4  Fb Sb

Fb Sa
392.4 S
1  b
Fb Sa

392.4 S
1 b
Fb Sa
Sb
As the support at point A is shifted right, Sa decreases. So 1 
Sa increases, and

Fb decreases.

(b) Fig. 2.2 shows a variable force acting on a 200 kg object travelling in a straight line
with an initial velocity, in the positive direction along the same line as the line of action
of the force , of 15 m s-1 at t = 0 s.

Calculate the magnitude of the final velocity of the object at t = 30.0 s.

F/N

100.0

0 8.0 10.0
0 12.0 20.0 28.0 30.0 t/s

100.0

Fig. 2.2

Impulse = Change in momentum of object = Area under F-t graph


Impulse = (0.5 x 10.0 x 100.0) = 500 N s
Δp = m Δv
p 500
Δv = m = 200 = 2.5 m s-1

Δv = vf - vi
vf = Δv + vi = 2.5 + 15.0 = 17.5 m s-1

final velocity = __________ m s-1 [2]

SRJC 2010 9646/ Prelim/2010


For
5 Examiner’s
Use

(c) Fig. 2.3 shows a wooden block Z resting on block X. Block X and Z are connected by
an inelastic string which goes around a fixed smooth pulley. The mass of block X is 20
kg and the mass of block Z is 2.0 kg.

The friction between blocks X and Z is 11.0 N. Block X is pulled at a constant velocity
by a 100.0 N force. All strings are of negligible mass.

2.0 kg

Z 100.0 N
X
Fixed 20 kg
block
O
20

Fig. 2.3

Calculate the frictional force between X and the table.

friction = …………….… N [3]

FX,T
Z

T mz g sin 20°

[1]

mzg

F
FX, Z X

T mxg sin 20°

FX,T
mxg

Let the frictional force between X and the table be FX,T.


Let the frictional force between X and Z be FX,Z.

From the free-body diagram for Z,


SRJC 2010 9646/ Prelim/2010 [Turn Over
For
6 Examiner’s
Use
T + mzg sin20° - 11 = mza

Since velocity of Z is constant,


T = 11 - mzg sin20°
= 11 – (2)(9.81)sin20°
= 4.290 N

From the free-body diagram for X,


100 – (T + 11 + mxg sin20° + FX,T ) = mxa

Since velocity of X is constant,


100 – (T + 11 + (20)(9.81) sin20° + FX,T ) = 0
100 – [4.290 + 11 + (20)(9.81) sin20° + FX,T ] = 0
100 – 4.290 – 11 – (20)(9.81) sin20° – FX,T = 0
FX,T = 17.6 N

SRJC 2010 9646/ Prelim/2010


For
7 Examiner’s
Use

3 A pendulum bob is tied to a string as shown in Fig. 3.1. A person swings it in a vertical
circular path. The mass of the bob is 45.0 g. The length of the string is 70.0 cm. The
diameter of the bob is 2.0 cm.

70.0 cm

Pendulum bob
Fig. 3.1

(i) Calculate the range of angular speeds to achieve this motion.


Ans:
T + mg = mr2
For T to be more than zero,
mr2 > mg
 > (g/r)½
> (9.81/0.71)½
> 3.72 rad s-1
range of angular speed ……………………. rad s-1 [2]

(ii) Calculate the maximum tension experienced in the string when the bob is spun at an
angular speed of 5.00 rad s-1.

Ans:
T = mg + mr2
= 0.045(9.81 + 0.71(5.00)2)
= 1.24 N

maximum tension = ……………………. N [2]

(iii) State the assumption made in your calculation in part (ii).

………………………………………………………………......................................…………

……………………………………………………………………………………………..……[1]

Ans: The mass of the string is insignificant.

(iv) Explain what might happen to the pendulum bob if it is spun around at an increasing
angular speed.

………………………………………………………………......................................…………

………………………………………………………………......................................…………

SRJC 2010 9646/ Prelim/2010 [Turn Over


For
8 Examiner’s
Use
……………………………………………………………………………………………..……[2]

Ans:

As  increases, tension in the string increases,


The string may break off.

SRJC 2010 9646/ Prelim/2010


For
9 Examiner’s
Use

4 (a) A vertical peg is fixed to the rim of a horizontal turntable of radius, r, rotating with a
constant angular speed, as shown in Fig. 4.1.

Parallel light
r
G
θ

ω H

Fig. 4.1

Parallel light is incident on the turntable so that the shadow of the peg is observed on a
screen which is normal to the incident light. At time t = 0 s and
θ = 0° the shadow of the peg is seen at G.

(i) Write an expression for the distance GH in terms of r, ω and t. [2]

GH = r sin (ω t)

(ii) The peg is now placed midway between the centre and the edge of the disc.
Describe what will be the effects on the shadow in terms of its

1. period;

………………………………………………………………………………………………

………………………………………………………………………………………………

…………………………………………………………………………………….…… [1]

No change to its period since the angular velocity of the peg, ω,thus angular
frequency of the shadow does not change.

2. maximum speed; and

………………………………………………………………………………………………

………………………………………………………………………………………………

…………………………………………………………………………………….…… [1]

The maximum speed of the shadow is directly proportional to the amplitude,


hence it is halved. i.e v0 = ωx0

SRJC 2010 9646/ Prelim/2010 [Turn Over


For
10 Examiner’s
Use
3. maximum acceleration.

………………………………………………………………………………………………

………………………………………………………………………………………………

…………………………………………………………………………………….…… [1]

The maximum acceleration of the shadow is directly proportional to the amplitude,


hence it is halved. i.e a0 = ω2x0

(b) (i) Explain the meaning of the word resonance.

......……………………………………………………………….………………………………...

......……………………………………………………………….………………………………...

......……………………………………………………………….…………………………… [1]

Resonance is a phenomenon where an oscillating system responds to an external


driving force such that when the frequency of the driving force matches the natural
frequency of itself (i.e. the driven system) the system oscillates with maximum
amplitude.

(b) (ii) A driver of constant amplitude and variable frequency, f, causes forced
oscillations of an object. The amplitude a0 of the object’s oscillations depends on
f.

1. Sketch on Fig. 4.2 a graph to show how a0 varies with f over a wide range
of frequencies which includes the natural frequency f0 of the object.

Label this graph U [1]

a0

f
SRJC 2010 9646/ Prelim/2010
Fig. 4.2
For
11 Examiner’s
Use

2. Add to your sketch in Fig. 4.2 a second line which shows the effects of
increased damping.

Label this line D. [1]

y0
U

f0 f

4(b)(ii)1 Marking points: Shape of graph is correct. Y-intercept non-zero.


Natural frequency f0 must be labeled.

4(b)(ii)2 Marking points: Shape of graph is correct. Y-intercept non-zero.


Amplitude lies to the right of undamped natural frequency f and amplitude
must be of lesser magnitude.

(b) (iii) A washing machine has a drum which rotates at high speed. Suggest why a large
mass, fixed to the inside of the machine helps to reduce the possibility of
resonance.

......……………………………………………………………….………………………………...

......……………………………………………………………….………………………………...

......……………………………………………………………….…………………………… [1]

The large mass will change natural frequency of oscillation of washing machine
Hence driving frequency from motor and natural frequency no longer matches,
reducing possibility of maching experiencing resonance (violent shaking).

SRJC 2010 9646/ Prelim/2010 [Turn Over


For
12 Examiner’s
Use
5 Two equally charged solid conducting spheres with small radii, of equal uniform mass m are
hanged from the top plate of two parallel charged plates, with insulated strings of negligible
mass, as shown in Fig. 5.1.

Each sphere has a charge of 3.20 x 10-5 pC.

-20 V

10.0 cm 3o 3o

4.0 cm
+4 V

Fig. 5.1

(i) Determine the magnitude of the force acting on each sphere due to the parallel plates
only.

force = ...................... N [1]


dV 4  ( 20)
E   240 N C-1
dr 0.100
F  qE  3.2  1012  240  7.68  1010 N

(ii) Determine the force between the charged spheres due to the charges on the spheres
only.

force = ...................... N [1]


12 2
Qq (3.2  10 )
F   2.33  109 N
4 0r 2
4 0 (2  0.06 tan 3)2

(iii) Hence, determine the mass of the spheres.

mass = ...................... kg [3]

Fplate

Tension, T

Fsphere

mg

For object in equilibrium, resultant force = 0


mg  7.68  1010  T cos3 ___(1)
T sin3  2.33  109 ___(2)
SRJC 2010 9646/ Prelim/2010
For
13 Examiner’s
Use

Solving simultaneously,
m = 4.53 x 10-8 kg

(iv) Describe the effect on the positions of the spheres when the charge of one of the
spheres is reduced while the charge on the other is reduced by an even greater
magnitude, with all other factors remaining the same.

………………………………………………………………......................................…………

………………………………………………………………......................................…………

……………………………………………………………………………………………..……[1]

SRJC 2010 9646/ Prelim/2010 [Turn Over


For
14 Examiner’s
Use
6 Fig. 6.1 shows a circuit in which PR is a 20.0 Ω slide wire, 75.0 cm long. E2 is a 40.0 V cell.
Both E1 and E2 have non-negligible internal resistance. R1 and R2 are resistances of 8.0 Ω and
5.0 Ω respectively.

The balance length is 60.0 cm when S1 and S2 are open as shown in Fig 6.1.

E2=40 V

25.0 Ω S2 10.0 Ω

P Q
R
E1

R2 R1

S1
5.0 Ω 8.0 Ω

Fig. 6.1

(i) When E1 is replaced with a 12 V cell with negligible internal resistance, the balance
length becomes 45.0 cm.

Calculate the internal resistance of the E2.

internal resistance of E2 = ……………………..Ω [2]

lPR
VPR  VPQ
lPQ
When E1 is replaced with a 12 V cell with negligible internal resistance,
0.75
VPR   12 = 20 V
0.45
RPR
VPR   40
RPR  10.0  rE2
20.0
20   40
20.0  10.0  rE2
rE2 = 10.0 Ω

(ii) Calculate the electromotive force of cell E1

E1 = …………………….. V [2]
lPR
VPR  VPQ
lPQ

SRJC 2010 9646/ Prelim/2010


For
15 Examiner’s
Use

0.75
20  VPQ
0.60
VPQ = 16 V
E1 = 16 V

(iii) Calculate the balance length when S1 and S2 are closed. It is given that the internal
resistance of E1 is 3.0 Ω

Balance length = ……………………..m [3]

58
VPQ =  16
583
= 13 V [1]
When S1 and S2 are closed,
 (20)(25) 
 
VPR =  20  25   40
 (20)(25) 
   10  10
 20  25 
11.11
=  40
11.11 20
= 14.28 V
l
VPR  PR VPQ
lPQ
0.75
14.28   13
lPQ
lPQ = 0.683 m

SRJC 2010 9646/ Prelim/2010 [Turn Over


For
16 Examiner’s
Use
7 A cathode ray oscilloscope is connected across the input of a transformer. Fig 7.1 below
shows the screen. Each of the main squares in the diagram has a length of 1 cm on each
side (as shown in the figure).

1 cm

1 cm

Fig. 7.1

The Y-plate sensitivity is set at 10 V cm-1. The timebase is set so that the horizontal
sensitivity is 1.5 ms cm-1.

(a) For the alternating potential difference applied to the Y-plates, calculate the following:
(i) Frequency

frequency = ……………… Hz [1]

Period = 3.0 x 10-3 s


1 1
Frequency = = = 333 Hz
T 3.0  103

(ii) Root-mean-square value of potential difference

r.m.s. value of potential difference = ……………… V [1]

Peak value = 10 x 2 = 20.0V


20
Vrms = = 14.1 V
2

SRJC 2010 9646/ Prelim/2010


For
17 Examiner’s
Use

(b) The transformer has 110 turns on its primary winding and 750 turns on its secondary
winding.

Determine the output r.m.s. potential difference

Output r.m.s of potential difference = ……………… V [2]

750
Vs =  14.1
110
= 96.1 V

SRJC 2010 9646/ Prelim/2010 [Turn Over


For
18 Examiner’s
Use
8 A small boat is powered by an outboard motor of variable power output P. Fig. 8.1 shows the
variation with speed v of P when the boat is carrying different loads.

P / kW

5.0

4.5 400
kg
4.0

3.5

3.0
350
2.5 kg

2.0

1.5 300
kg
1.0 250
kg
0.5

0.0
0.0 0.5 1.0 1.5 2.0 2.5 3.0 3.5 4.0
v / m s-1
Fig. 8.1

The masses shown are the total mass of the boat plus passengers.

(a) For the boat having a steady speed of 2.00 m s−1 and with a total mass of 250 kg,

(i) use the graph in Fig. 8.1 to determine the power of the engine.

power = ………………… kW [1]

power = 0.40 kW

SRJC 2010 9646/ Prelim/2010


For
19 Examiner’s
Use

(ii) calculate the resistive force acting on the boat.

resistive force = …………………. N [1]

Using P = Fv (where F = resistive force)


0.40 x 103 = F x 2.00
F = 200 N

(b) Consider the case of the boat of mass 350 kg moving with a speed of 2.50 m s-1.

(i) By using data from the graph, estimate the total amount of energy which the
motor provides in order for the boat to travel for a total of 30 minutes.

Total amount of energy = ……………. MJ [2]

From graph,
Power for 350 kg boat to travel at 2.50 m s-1, P = 1.90 kW
Energy required = Pt
= (1.90 x 103) x (30 x 60)
= 3.42 MJ

(ii) The amount of energy given off when 1.00 kg of fuel is mixed with oxygen in the
air is 45.0 MJ. The efficiency of the motor in converting the energy released by
the combination of oxygen and the fuel is 40.0%.

Determine for the case in (i) the total amount of fuel which would be expended.

Total amount of fuel = …………………… kg [2]

Output Energy
Efficiency of motor =  100%
Input Energy

40.0 3.42  106


=
100 Input Energy
Input Energy = 8.55 MJ

8.55
Total amount of fuel burnt =
45.0
= 0.190 kg

SRJC 2010 9646/ Prelim/2010 [Turn Over


For
20 Examiner’s
Use
(iii) Hence, explain if the estimate in (i) is a reasonable one.

………………………………………………………………......................................…………

………………………………………………………………......................................…………

……………………………………………………………………………………………..……[2]

The estimate was made based on the assumption that the mass of the boat is constant
at 350 kg. The amount of fuel expended is 0.190 kg, which is a very small fraction of
350 kg.
The estimate should be a reasonable one.

(c) Fig. 8.2 shows how the speeds of 2 boats of equal mass vary with respect to time. Boat
A starts from rest while boat B travels at a constant speed.

Speed / m s-1
boat A
V

1.50 boat B

900 t / s
Fig. 8.2

(i) Boat A and boat B both travel the same distance at t = 900 s. State the velocity V
of boat A at t = 900 s.

……………………………………………………………………………………………..……[1]

Since the distance travelled by boat A and B are equal, the area under both graphs
should be equal.

V = 3.00 m s-1

(ii) Explain which boat, if any, would expend a greater amount of petrol at the end of
900 s.

………………………………………………………………......................................…………

………………………………………………………………......................................…………

………………………………………………………………......................................…………

………………………………………………………………......................................…………
SRJC 2010 9646/ Prelim/2010
For
21 Examiner’s
Use

……………………………………………………………………………………………..……[3]

Boat B is travelling at a constant speed, the energy output from the engine is used to
do work against resistive forces. [1]
Boat A is travelling with a constant acceleration. The energy output from the engine is
used to increase the kinetic energy of the boat as well as do work against the resistive
forces. [1]
Therefore Boat A will expend more petrol. [1]

SRJC 2010 9646/ Prelim/2010 [Turn Over


SERANGOON JUNIOR COLLEGE
General Certificate of Education Advanced Level
Higher 2

NAME

CG INDEX NO.

PHYSICS 9646
Preliminary Examination 24 August 2010
Paper 3 Longer Structured Questions 2 hours

Candidates answer on the Question Paper.


No additional Materials are required.

READ THIS INSTRUCTIONS FIRST

Write your name, civics group and index number in the spaces at the top of this page.
Write in dark blue or black pen on both sides of the paper.
You may use a soft pencil for any diagrams, graphs or rough working.
Do not use staples, paper clips, highlighters, glue or correction fluid.

Section A
Answer all questions.

Section B For Examiner’s Use


Answer any two questions.
Section A
At the end of the examination, fasten all your work securely
together. 1
The number of marks is given in bracket [ ] at the end of
each question or part question. 2

Section B

Total

This document consist of 24 printed pages and no blank page


SRJC 2010 9646/Prelim/2010
2

DATA AND FORMULAE


Data
speed of light in free space, c = 3.00 x 108 m s1
permeability of free space, 0 = 4 x 107 H m1
permittivity of free space, 0 = 8.85 x 1012 F m1
(1 / (36π)) x 109 F m1
elementary charge, e = 1.60 x 1019 C
the Planck constant, h = 6.63 x 1034 J s
unified atomic mass constant, u = 1.66 x 1027 kg
rest mass of electron, me = 9.11 x 1031 kg
rest mass of proton, mp = 1.67 x 1027 kg
molar gas constant, R = 8.31 J K1 mol1
the Avogadro constant, NA = 6.02 x 1023 mol1
the Boltzmann constant, k = 1.38 x 1023 J K1
gravitational constant, G = 6.67 x 1011 N m2 kg2
acceleration of free fall, g = 9.81 m s2

Formulae
uniformly accelerated motion, s = ut + ½ at2
v2 = u2 + 2as
work done on/by a gas, W = pV
hydrostatic pressure, p = gh
Gm
gravitational potential,  = –
r
displacement of particle in s.h.m., x = x0 sin t
velocity of particle in s.h.m., v = vo cost
v = ω x 2
0  x2 
resistors in series, R = R1 + R2 + …
resistors in parallel, 1/R = 1/R1 + 1/R2 + …
electric potential, V = Q / 4or
alternating current/voltage, x = x0sin t
transmission coefficient, T α exp(2kd)
8 π 2 m(U  E)
where k =
h2
radioactive decay, x = x0 exp(t)
0.693
decay constant,  =
t1
2

SRJC 2010 9646/Prelim/2010


For
3 Examiner’s
Use

Section A
Answer all the questions in this section

1 (a) State what is meant by a progressive wave.

..…………………………………………………………………………………………..……………….

..……………………………………………………………………………………………………..…….

..………………………………………………………………………………………….…………… [2]

A progressive wave is one which energy can be transferred.


Its particles does not propagate in the direction of the wave but vibrates either perpendicular
or parallel to the direction of wave propagation.

(b) Two speakers S1 and S2 produce waves of the same frequency.

(i) State three conditions that must be satisfied for waves from the two sources to
produce detectable interference pattern.

1. ….…………………………………...…………………………………….…………………..

………………………………………………………………..………...………………...…

2. ……………………………………...………………………………………………….……..

………………………………………………………………..………...…………….……

3. ……………………………………...………………………………………………….........

………………………………………………………………..………...…………………[3]

1. Sources must be coherent.


2. Waves must interfere.
3. Waves must have roughly the same amplitude.

(ii) One of the speakers, S1, is connected to a signal generator. It is then oriented to
face a wall as shown in Fig. 1.1.

Signal generator wall


Fig. 1.1

SRJC 2010 9646/Prelim/2010 [Turn Over


For
4 Examiner’s
Use
Sound waves are produced between the speaker and the wall to form a stationary wave.

1. Outline how you would use this setup to obtain the speed of sound waves
in air.

………………………………………………………………………………………...…….

………………………………………………………………………………………...…….

………………………………………………………………………………………...…….

………………………………………………………………………………………...…….

………………………………………………………………………………………...…….

………………………………………………………………………………………...…….

………………………………………………………………………………………...…….

…………………………………………………………………………………………....[3]

Measure separation between (adjacent) nodes / antinodes and double to get λ/this is ½λ
[not between peaks and troughs]
Frequency known from/produced by signal generator OR measured on CRO / by digital
frequency meter.
Detail on measurement of wavelength OR frequency e.g. measure several [if a number is
specified then ≥3] node spacings and divide by the number [not one several times]
OR measure several (≥3) periods on CRO and divide by the number
OR adjust CRO so only one full wave on screen
Use v = fλ

2. In principle, stationary waves produced in this way could cause problems


for listeners in a concert hall. Explain why.

………………………………………………………………………………………...…….

…………………………………………………………………………………………....[1]

Little or no sound /amplitude OR you may be sat at a pressure node

3. In practice this problem is not serious. Suggest a reason why.

………………………………………………………………………………………...…….

…………………………………………………………………………………………....[1]

Reflected wave not as strong as incident wave.


OR walls are covered to reduce reflections/Absorbs incident waves.
OR waves arrive from elsewhere [reflections/different speakers].
OR such positions depend on wavelength / frequency.
SRJC 2010 9646/Prelim/2010
For
5 Examiner’s
Use

2 Three electron energy levels in atomic hydrogen are represented in Fig. 2.1.

1880 nm

Increasing 486 nm
energy

656 nm

Fig. 2.1

The wavelengths of the emission spectral lines produced by electron transitions between
these three energy levels are 486 nm, 656 nm and 1880 nm.

(a) On Fig. 2.1, draw arrows to show the electron transitions between the energy levels
that would give rise to these wavelengths. Label each arrow with the wavelength of the
emitted photon. [2]

Award 2 marks when all arrows and labels are correct.


Deduct 1 mark for each wrong label or direction.

(b) Calculate the minimum change in energy of an electron in eV when making transitions
between these levels.

change in energy = …………………… eV [3]

Minimum change in energy corresponds to the emission of photon with the longest
wavelength.

hc
Using E= ,

6.63  10 34  3.00  108
E=
1880  10 9
E = 1.058 x 10-19 J

1.058  1019
E=  0.661 eV
1.60  10 19

SRJC 2010 9646/Prelim/2010 [Turn Over


For
6 Examiner’s
Use
3 (a) To produce X-rays, high energy electrons are fired at a particular target metal. Fig. 3.1
is the X-ray spectra which shows how the intensity of the X-ray radiation varies with the
x-ray photon energy.
Intensity

80 200 Photon energy / keV


Fig. 3.1

(i) Explain how the photons of 200 keV energy are generated.

..…………………………………………………………………………………………………….

..…………………………………………………………………………………………………….

..…………………………………………………………………………………………………[1]

The photons of 200 keV energy are generated when all of the kinetic energy of the
accelerated electron is lost by the electron in the first collision with the target metal and
the kinetic energy is transformed into electromagnetic radiation.

(ii) Sketch in Fig. 3.2 the how the intensity of the X-ray radiation varies with the
wavelength of the X-ray. (Ensure that values should be shown where applicable.)
[3]
Intensity

0.00622 0.0155 Wavelength /nm


Fig. 3.2

SRJC 2010 9646/Prelim/2010


For
7 Examiner’s
Use

(b) Radioactive decay is known to be a random process. It is impossible for one to know
exactly when a particular nucleus will decay. Explain using Quantum theory why this is
so.

..…………………………………………………………………………………………..……………….

..……………………………………………………………………………………………………..…….

..…………………………………………………………………………………………..……………….

..……………………………………………………………………………………………………..…….

..…………………………………………………………………………………………..……………….

..……………………………………………………………………………………………………..…….

..………………………………………………………………………………………….…………… [3]

The particles within in the nucleus are actually held tightly by strong nuclear forces. From the
classical point of view, it is impossible for any of the particles to escape from the nucleus.
but in Quantum theory, the particle is described by a wave function ψ. The wave function is
continuous and continues even outside of the nucleus.
2
The  which is known as the probability density function of the particle is therefore non-
zero outside of the nucleus. This means that there is a finite probability of finding the particle
outside of the nucleus, hence it is a random process.

SRJC 2010 9646/Prelim/2010 [Turn Over


For
8 Examiner’s
Use
4 (a) Laser is an acronym for Light by Amplification by Stimulated Emission of Radiation.
The properties of Laser lights are very different from the light from an incandescent
lamp which occurs due to spontaneous emission.

(i) Distinguish between the terms spontaneous emission and stimulated emission.

..…………………………………………………………………………………………………….

..…………………………………………………………………………………………………….

..…………………………………………………………………………………………………….

..…………………………………………………………………………………………………….

..…………………………………………………………………………………………………[2]

Spontaneous emission occurs when an electron within an atom de-excites naturally


giving off a photon.
Stimulated emission occurs when an electron within an atom is disturbed to de-excite
to a lower level because a photon with an energy equal to the energy gap is incident on
it. The resulting photon is in phase and travels in the same direction as the incident
photon.

(ii) Fig. 4.1 shows some of the energy levels of a particular element used as a lasing
material in a gas laser. The wavelength of the laser emitted is 517 nm.

E∞ = 0 eV
E3 = - 1.5 eV
E2 = - 1.8 eV

E1 = -2.4 eV

E0 = -4.2 eV

Fig. 4.1

State and explain which energy level is the metastable state.

SRJC 2010 9646/Prelim/2010


For
9 Examiner’s
Use

..…………………………………………………………………………………………………….

..…………………………………………………………………………………………………….

..…………………………………………………………………………………………………[2]

hc
Using E= ,

6.63  10 34  3.00  108
=
517  10 9
= 3.847 x 10-19 J
= 2.40 eV

The energy of a photon of laser is 2.40 eV.


This implies that the lasing process should occur between E2 and E0 which therefore
means that E2 must be the metastable state.

(b) Using band theory, explain the reason why certain solids act as

(i) insulators.

..…………………………………………………………………………………………………….

..…………………………………………………………………………………………………….

..…………………………………………………………………………………………………….

..…………………………………………………………………………………………………….

..…………………………………………………………………………………………………[2]

For solids which act as insulators, the solid has valence bands which are completely
filled and there are no electrons found in the conduction band therefore there are no
free charge carriers.
Moreover, energy gap between the valence band and the conduction band is very
large, therefore it is very difficult to excite an electron from the valence band to the
conduction band.

(ii) intrinsic semiconductors.

..…………………………………………………………………………………………………….

..…………………………………………………………………………………………………….

..…………………………………………………………………………………………………….

..…………………………………………………………………………………………………….

SRJC 2010 9646/Prelim/2010 [Turn Over


For
10 Examiner’s
Use
..…………………………………………………………………………………………………[2]

For semiconductors, the energy gap between the valence band and conduction band is
very small (in the orders of 1 eV).
At room temperature, some electrons from the valence band would have enough
energy to be excited to the conduction band, leaving behind ‘holes’ in the valence band
and electrons in the conduction band which are mobile.

SRJC 2010 9646/Prelim/2010


For
11 Examiner’s
Use
55 56
5 In a nuclear reaction, whereby stable Mn are irradiated with neutrons to produce
25 Mn .
25
56 56
25Mn decays subsequently to a stable nucleus of Fe .
26

55
Atomic mass of 25 Mn =54.93805 u
56
Atomic mass of 25 Mn =55.93890 u
56
Atomic mass of 26 Fe =55.93494 u
Mass of proton = 1.007276 u
Mass of neutron = 1.008665 u
Mass of electron = 0.000549 u

(a) (i) Write down the equation for these nuclear transformations.

55
25 Mn  01n  56
25 Mn
56
25 Mn  56
26 Fe  0
1 

56
(ii) Determine the total energy liberated when a Mn nucleus decays.
25

Difference in mass = (55.93890-55.93494) x 1.66 x 10-27 kg


Energy Released
= mc2
= [(55.93890-55.93494) x 1.66 x 10-27](3.0 x 108)2
=5.92 x 10-13 J

(b) (i) Explain what is meant by binding energy.

Binding energy is the amount of energy needed to completely separate a nucleus into its
constituent nucleons.

55
(ii) Determine the binding energy per nucleon of 25 Mn nucleus.

Total mass of neutrons and protons individually


= 25 x 1.007276 + 30 x 1.008665) u
= 55.44185 u

Mass defect
= (55.44175 – 54.93805) u
= 0.5038 u

Binding energy
= 0.5038 x x 1.66 x 10-27 x (3.0 x 108)2
= 7.527 x 10-11 J
= 0.470 MeV

Binding energy per nucleon = 0.470/55 = 8.55 keV

SRJC 2010 9646/Prelim/2010 [Turn Over


For
12 Examiner’s
Use

(iii) Explain why the binding energy per nucleon is associated with the stability of the
nucleus and not the total binding energy.

Stability refers to the ability to resist fusion or fission.

Fusion and fission refers to the processes of forming larger or smaller nuclei, which
involves loss or gain of nucleons.

A larger binding energy per nucleon implies a greater difficulty to lose or gain additional
nucleons as more energy is needed.

Total binding energy increases even though the mass number increases beyond that of
iron.

SRJC 2010 9646/Prelim/2010


For
13 Examiner’s
Use

Section B

Answer two questions in this section

6 (a) (i) The gravitational force is significant only when we deal with celestial objects like
stars and planets but not with atoms or molecules. Explain the rationale for this
statement.

..…………………………………………………………………………………………………….

..…………………………………………………………………………………………………….

..…………………………………………………………………………………………………….

..…………………………………………………………………………………………………….

..…………………………………………………………………………………………………[2]

The gravitational force between two isolated masses is given by the equation,
GMm
F= .
r2
Gravitational forces are only significant when the masses involved are massive. This is
because of the magnitude of Newton’s universal gravitational constant, G, whose
magnitude is very small.
Hence atoms and molecules whose masses are very small would result in insignificant
magnitude of gravitational forces.

(ii) It is often said that astronauts experience weightlessness only because they are
beyond the pull of the Earth’s gravity. Comment on the validity of this statement.

..…………………………………………………………………………………………………….

..…………………………………………………………………………………………………….

..…………………………………………………………………………………………………….

..…………………………………………………………………………………………………….

..…………………………………………………………………………………………………[2]

The statement is not the only that is correct.

Astronauts can also be under the pull of gravity but still be weightless. This occurs when
the space vehicle that the astronaut is in, does circular orbit such that the centripetal
acceleration is equal to the acceleration due to gravity at the position of the space vehicle.

SRJC 2010 9646/Prelim/2010 [Turn Over


For
14 Examiner’s
Use
(iii) Rockets are usually launched at locations near the Equator in an easterly
direction. Explain whether there is any advantage in launching a rocket to the
east versus launching to the west.

..…………………………………………………………………………………………………….

..…………………………………………………………………………………………………….

..…………………………………………………………………………………………………….

..…………………………………………………………………………………………………….

..…………………………………………………………………………………………………[2]

There is an advantage in launching a rocket at the equator towards the east.


As the Earth rotates west to east, the rocket can take advantage of the Earth’s
rotational speed during the launch.

(b) An Earth satellite in a circular orbit has a period of 12 hours about the Earth’s centre.
The radius of the Earth is 6.40 x 103 km.

Calculate

(i) the mass of the Earth given that the acceleration of free fall at its surface is
9.81 m s-2.

mass = …………………… kg [2]

GME
Using g =
 rE 2

9.81 =
 6.67 x10  M 11
E

 6.40 x 10  3 2

ME = 6.02 x 1024 kg

(ii) the height of the satellite above the Earth’s surface.

height = …………………… m [3]

GMEms
Since msrω2 =
r2
 2 
2
GMEms
ms r   =
 T  r2

SRJC 2010 9646/Prelim/2010


For
15 Examiner’s
Use

1
 GME T 2  3
r = 2 
 4 
1

  
 6.67 x1011 6.02 x 1024 12 x 36002  3
=  
 4 2 
 
= 2.67 x 107 m [1]

Height of satellite above earth’s surface = 2.67 x 107 – 6.40 x 103


= 2.03 x 107 m

(iii) the orbital speed of the satellite.

speed = …………………… m s-1 [2]


v2 GMEms
Since ms =
r r2
GME
v2 =
r

v =
 6.67 x 10  6.02 x 10 
-11 24

2.67 x 107
v = 3.88 x 103 m s-1.

(c) (i) The satellite is directly above an observer located at the Equator at certain
instant of time. The satellite moves in the same direction of rotation as the Earth.
Describe the motion of the satellite relative to the observer over the next 24 hours.
Consider two 12 hour periods.

..…………………………………………………………………………………………………….

..…………………………………………………………………………………………………….

..…………………………………………………………………………………………………….

..…………………………………………………………………………………………………….

..…………………………………………………………………………………………………[2]

In first 12 hours, satellite completed one full revolution. Observer would have only
completed half. Therefore satellite and observer would be on opposite sides.
In the next 12 hours, satellite would be directly above observer, exactly at its original
position.

SRJC 2010 9646/Prelim/2010 [Turn Over


For
16 Examiner’s
Use
(ii) Determine the orbital period for the satellite to be geostationary.

period = ……………………. hr [1]


24 hours.

(iii) List one advantage and one disadvantage to the observer if the satellite was
geostationary.

advantage:

..…………………………………………………………………………………………………….

..…………………………………………………………………………………………………….

..…………………………………………………………………………………………………[1]

disadvantage:

..…………………………………………………………………………………………………….

..…………………………………………………………………………………………………….

..…………………………………………………………………………………………………[1]

advantage:
The observer is able to transmit / receive data from the satellite at any time of the day.
There is no need to wait 24 hours to be in line of sight with the satellite.
OR
The observer is able to gather real time data of activities occurring in his geographical
location rather than have snapshots of the activities happening at certain hours of the
day.
disadvantage
The signals would take a longer time to transmit and receive since the satellite would
now be at a higher altitude in space.

(d) Gravitational and electric fields have analogous characteristics. List one similarity and
one difference between these two fields.

similarity:

..…………………………………………………………………………………………………….

..…………………………………………………………………………………………………….

..…………………………………………………………………………………………………[1]

SRJC 2010 9646/Prelim/2010


For
17 Examiner’s
Use

difference:

..…………………………………………………………………………………………………….

..…………………………………………………………………………………………………….

..…………………………………………………………………………………………………[1]
similarity:
Force follows the inverse square law.
The derivative of the potential gives the magnitude of the field strength.
The derivative of the potential energy gives the magnitude of the force.
difference
Gravitational force is always attractive but electric forces is either attractive/repulsive.

SRJC 2010 9646/Prelim/2010 [Turn Over


For
18 Examiner’s
Use
7 (a) The following statements were made with regard to various thermal processes.

Using kinetic theory, comment on the validity of the statements and elaborate on the
reasoning.

(i) Energy must be supplied to a sample of pure liquid to maintain constant


temperature during boiling because it is needed to maintain the amount of
internal energy of the sample.

..…………………………………………………………………………………………………….

..…………………………………………………………………………………………………….

..…………………………………………………………………………………………………….

..…………………………………………………………………………………………………….

..…………………………………………………………………………………………………….

..…………………………………………………………………………………………………….

..…………………………………………………………………………………………………[3]

It is increasing the internal energy and not maintaining it.


It is to increase the potential energy due to intermolecular forces
And not the average random translational energy which is directly proportional to the
temperature.

(ii) More energy is needed for boiling of pure water than melting of pure ice of the
same mass under the same environmental conditions because ice has a lower
density than water and therefore less energy is needed.

..…………………………………………………………………………………………………….

..…………………………………………………………………………………………………….

..…………………………………………………………………………………………………[2]

Less energy is needed not because of the lower density.


More energy is needed to do work against the environment for the case of boiling
because of large increase in volume as compared to melting.

SRJC 2010 9646/Prelim/2010


For
19 Examiner’s
Use

(b) Using kinetic theory, explain why evaporation occurs at all temperature and is
accompanied by cooling.

..…………………………………………………………………………………………………….

..…………………………………………………………………………………………………….

..…………………………………………………………………………………………………….

..…………………………………………………………………………………………………….

..…………………………………………………………………………………………………….

..…………………………………………………………………………………………………….

..…………………………………………………………………………………………………[3]

Water at a particular temperature other than the boiling temperature has particles of different
amount of kinetic energy which is not equal to the average random translational kinetic
energy.
During evaporation, the more energetic particles possess sufficient energy to escape while
the less energetic particles remain behind.
This leads to lower average random translational energy, which is proportional to
temperature.
Hence, temperature decreases.

(c) The setup in Fig. 7.1 is used for the electrical method of determining the specific heat
capacity, c of a solid material X of mass m1.

V
A

Heater Thermometer

Material X Insulating
layer

Fig. 7.1

The temperature of the solid increased from θ1 to θ2 in time t1 when the voltmeter and
ammeter reading remained constant at V1 and I1.

SRJC 2010 9646/Prelim/2010 [Turn Over


For
20 Examiner’s
Use

The following equation was used to determine c.

V1I1t1  m1c ( 2  1 )

(i) State and explain whether the calculated c value, based on the above equation,
is an underestimate, overestimate or accurate reflection of the true value of c.

..…………………………………………………………………………………………………….

..…………………………………………………………………………………………………….

..…………………………………………………………………………………………………….

..…………………………………………………………………………………………………[2]

Overestimation.
Part of the heat supplied, as calculated by VIt may be lost to the environment and not
used to increase the temperature, hence, actual amount of heat needed should be
lesser.

(ii) Describe and explain how the accuracy of c can be improved with the same
setup, with accompanying equations and defined symbols, if necessary.

..…………………………………………………………………………………………………….

..…………………………………………………………………………………………………….

..…………………………………………………………………………………………………….

..…………………………………………………………………………………………………….

..…………………………………………………………………………………………………….

..…………………………………………………………………………………………………….

..…………………………………………………………………………………………………….

..…………………………………………………………………………………………………….

..…………………………………………………………………………………………………….

..…………………………………………………………………………………………………….

..…………………………………………………………………………………………………….

..…………………………………………………………………………………………………….

..…………………………………………………………………………………………………….

SRJC 2010 9646/Prelim/2010


For
21 Examiner’s
Use

..…………………………………………………………………………………………………[6]
Repeat experiment with the same temperature changes from θ 1 to θ2 in the same
period of time t1.

Vary mass and adjust variable resistor to vary potential difference across heater and
current across heater to get same temperature change in same period of time despite
change in mass.

Equation of repeated experiment:


V2 I 2t1  m2c (1   2 )  heat loss, h
Where V2 and I2 are the new reading of voltmeter and ammeter respectively and m 2 is
the new mass.
Equation of original experiment, taking into account of heat loss:

V1I1t1  m1c (1   2 )  heat loss, h


As time and temperature remains the same, rate of heat loss remains the same.

Solve simultaneously to obtain c.

(d) An ideal monatomic gas in an enclosed space obeys the two equations

1
pV = nRT and pV = Nm < c 2 >
3

(i) Derive an expression for the relationship between average random translational
kinetic energy of the gas atom and the temperature of the gas. [1]

1
nRT = Nm < c 2 >
3
1 nRT
m < c 2 >=
3 N
1 3nRT
m < c 2 >=
2 2N

(ii) Two samples of the same gas, X and Y of 3.0 mol and 4.5 mol respectively are at
temperature 100 °C and 200 °C.

Determine the ratio of the root mean square speeds of X to Y.

ratio = ..................... [1]


< c 2 >x 373.15
2
= = 0.888
< c >y 473.15

SRJC 2010 9646/Prelim/2010 [Turn Over


For
22 Examiner’s
Use

(iii) Two samples of ideal gas P and Q of 1.0 mol each are at the same temperature.

Determine the ratio of root mean square speeds of P to Q, when the ratio of the
relative molecular mass of P to Q is 3:2.

ratio = ………………… [2]


1
2
< c >x mP mQ 2
= =  =0.816
< c 2 >y 1 mP 3
mQ

SRJC 2010 9646/Prelim/2010


For
23 Examiner’s
Use

8 (a) Fig. 8.1 shows part of an experimental setup. Plates A and B are connected to an
e.m.f. source. Electron beam is projected equidistant from plates A and B along
x = 0 cm at a constant speed of 5 x 105 m s-1.

A very small fluorescent plate P can move left and right but not vertically. The current
position of centre of plate P is at x = 3 cm.

The magnetic field strength B of the magnetic region is 5.0 x 10-4 T. The potential
difference between A and B is V and the electric field strength generated between the
plates is E.

Electron

A B Magnetic
field into
page

x = -10 cm x = 0 cm x = 3 cm x = 10 cm

Fig. 8.1

(i) Derive the general simplified expression of the speed of the electron such that it
will not be deflected when it passes through plates A and B.

general expression of speed = ……………………. [2]

BeV=eE
V= E/B

(ii) The electron was not deflected. The plate P was moved at constant speed to the
left.

Determine the displacement of detector along the x direction from the initial
position such that maximum intensity will be observed at the centre of the
fluorescent plate for the second time.

SRJC 2010 9646/Prelim/2010 [Turn Over


For
24 Examiner’s
Use

displacement = …………………… m [3]

Calculate radius of curvature


BQv = mv2/r
r = mv/(BQ)
= 9.11 x 10-31x 5 x 105/(0.0005 x 1.60 x 10-19)
= 5.69 x 10-3 m
Displacement = - (2r + 0.03) = -0.0414 m

(iii) After the plate P is at the new location as determined in (a) (ii), the e.m.f. source
malfunctiones.

State and explain any difference in the observation on plate P.

..…………………………………………………………………………………………………….

..…………………………………………………………………………………………………….

..…………………………………………………………………………………………………….

..…………………………………………………………………………………………………[2]

The electron beam will no longer travel straight between plates A and B, and thus will
not reach plate P at its new position.
Maximum intensity will no longer be observed.

(b) A 6.00 m long conductor with 3.50 A current flowing upwards in an upright position is
placed in a magnetic field of 2.00 x 10-2 T, such that the magnetic field passes through
the conductor at an inclined angle of 50° to the horizontal as shown in Fig. 8.2.

conductor

50o

Directon of
magnetic field
Direction of current

Fig. 8.2
Determine the magnitude and direction of the force acting on the conductor.

magnitude of force = ……………………. N

direction of force = ……………………. [3]

SRJC 2010 9646/Prelim/2010


For
25 Examiner’s
Use

F = BILsin
= 0.0200 x 3.50 x 6.00 x sin 40°
= 0.270 N

The direction of the force is out of the page.

(c) Fig. 8.3 shows the plan view of an experimental setup. A magnetic field of 15.0 T
consists of semicircular and rectangular regions, with the field lines pointing
perpendicularly into the page. A copper rod slides on the iron bar with a constant speed
0.50 cm s-1 towards C.

Cylindrical
copper rod
Iron bar Region of
magnetic field

G 7.0 cm 12.0 cm

C B A
7.0 cm 1.0 cm 4.0 cm

Fig 8.3

(i) Calculate the average induced e.m.f.

e.m.f. = ……………………. V [3]

time = distance/ speed = (1.0 + 3.5)/0.5 = 9.0 s

area = 0.5r2 + LB = 0.5(3.5)2 + (1.0)(7.0) = 26.24 cm2


= 2.624 x 10-3 m2

E = d(BA)/dt where A is area of magnetic regions.


= (BA)/t
= 15(2.624 x 10-3)/9.0
= 4.37 x 10-3 V

SRJC 2010 9646/Prelim/2010 [Turn Over


For
26 Examiner’s
Use
(ii) Describe and explain the observation on a galvanometer connected to the iron
bar as shown in Fig. 8.3, as the copper rod moves from the current position until
it reaches point C.

..…………………………………………………………………………………………………….

..…………………………………………………………………………………………………….

..…………………………………………………………………………………………………….

..…………………………………………………………………………………………………….

..…………………………………………………………………………………………………….

..…………………………………………………………………………………………………….

..…………………………………………………………………………………………………….

..…………………………………………………………………………………………………….

..…………………………………………………………………………………………………….

..…………………………………………………………………………………………………….

..…………………………………………………………………………………………………….

..…………………………………………………………………………………………………….

..…………………………………………………………………………………………………….

..…………………………………………………………………………………………………[6]

According to Faraday’s Law, there is an induced e.m.f. when the conductor moves
through a magnetic field.
An induced current flows when there is a complete circuit
Initially, there is no deflection as the copper rod does not pass through any magnetic
field
From A to B, there is a deflection as the copper rod passes through magnetic field
From B to C, the deflection becomes smaller because the area of magnetic field
decreases, until there is no deflection

(iii) Describe and explain whether there is any difference in the answer to part (ii) if a
straight copper rod of the same cross-sectional area but twice its original length
is used.

..…………………………………………………………………………………………………….

..…………………………………………………………………………………………………….

..…………………………………………………………………………………………………….
SRJC 2010 9646/Prelim/2010
For
27 Examiner’s
Use

..…………………………………………………………………………………………………[2]

the effective resistance experienced by current flow in the process is the same as in
part (ii).
There is no difference in the observation.

SRJC 2010 9646/Prelim/2010 [Turn Over


1

TAMPINES JUNIOR COLLEGE


Preliminary Examination 2010

PHYSICS 9646/01
Higher 2
PAPER 1 Multiple Choice

Monday 20 September 2010

1 hour 15 minutes

Time 0800 – 0915 hr

Additional Materials: Multiple Choice Answer Sheet

READ THESE INSTRUCTIONS FIRST

Write in soft pencil.


Do not use staples, paper clips, highlighters, glue or correction fluid.
Write your name and class on the Answer Sheet in the spaces provided.

There are forty questions on this paper. Answer all questions.


For each question there are four possible answers A, B, C and D. Choose the one you
consider correct and record your choice in soft pencil on the separate Answer Sheet.

Read the instructions on the Answer Sheet very carefully.

Each correct answer will score one mark. A mark will not be deducted for a wrong answer.
Any rough working should be done in this booklet.

This document consists of 15 printed pages and 1 blank page.


2

9646 H2 PHYSICS (2010)


3

1 The viscous force experienced by an object in a fluid under turbulent conditions can be
given by the expression F  kv 2 , where  is the density of the fluid and v is the
velocity of the object in the fluid.

The unit of the constant k, expressed in SI base units, is

A m3 s-1 B m-3 s-1 C m2 D m-4

2 A student makes measurements from which he calculates the acceleration of free fall
as 9.7823 m s–2. He estimates that his result is accurate to  3%.
How should he present his result?

A (10  3) m s–2
B (9.8  0.3) m s–2
C (9.78  0.29) m s–2
D (9.78  0.03) m s–2

3 The period T of a simple pendulum of length L is given by the expression:

T  2 L
g
A student carries out an experiment with a particular simple pendulum and measured
the time for 20 oscillations for various lengths of pendulum L. He plots a graph of T2
against L. The student measured the length of the pendulum by measuring the length
of the string, and did not include the short distance between the end of the string and
the centre of mass of the pendulum.

What effect does this have on the graph he plotted?

A The gradient of the graph will be smaller than the correct value.
B The gradient of the graph will be larger than the correct value.
C The y-intercept of the graph will occur above the origin.
D The y-intercept of the graph will occur below the origin.

4 Two thermometers X and Y are placed inside an enclosure whose walls are maintained
at a steady temperature T.

When equilibrium is established, X and Y will give the same reading T

A under all circumstances.


B only if they absorb radiation at equal rates.
C only if they have equal thermal capacities.
D only if the enclosure is evacuated.
4

5 The graph in Fig. 5 shows the variation of temperature change  with time t for 1 kg of
a substance, initially solid at room temperature. The substance receives heat at a
uniform rate of 2000 J min-1.

Fig. 5

What can be deduced from this graph?

A The specific heat capacity of the substance is greater when liquid than when solid.
B The specific latent heat of fusion of the substance is 6000 J kg-1.
C After 4 minutes of heating the substance is all liquid.
D After 10 minutes the substance is all gaseous.

6 Fig. 6 shows a ball being thrown horizontally from a tower and lands 20 m away. At
what speed is the ball thrown? (Ignore air resistance)

Fig. 6

A 10 m s-1 B 15 m s-1 C 20 m s-1 D 28 m s-1


5

7 Fig. 7 shows a trolley traveling at a constant speed of 10 m s-1 to the left. A steel ball is
held by an electromagnet attached to trolley.

Fig. 7

The ball is released and stroboscopic photographs (a series of exposures on the same
film at equal intervals) are taken of the path of the ball.

Which of the following diagrams best represents what is seen on the photograph?

A B C D

8 A wire is stretched by a force F which causes an extension x. The energy stored in the
wire is ½ F x only if

A the extension of the wire is proportional to the force applied.


B the weight of the wire is negligible.
C the wire is not stretched beyond its elastic limit.
D the cross-sectional area of the wire remains a constant.

9 A trailer of weight 30 kN is hitched to a cab at the point X as shown in Fig. 9.

Fig. 9

If the trailer carries a weight of 20 kN at the position shown in the diagram, what
upward force is exerted by the cab on the trailer at the point X?

A 15 kN B 20 kN C 30 kN D 40 kN
6

10 Which one of the following pairs of forces is a valid example of action and reaction?

A the gravitational force on a parachutist and the viscous force of the air on the
parachutist and his parachute.
B the forces of repulsion between an atom in the surface of a table and an atom in
the surface of a book resting on the table.
C the centripetal force keeping a satellite in orbit round the Earth and the weight of
the satellite.
D the upthrust on a boat and the weight of seawater displaced by the boat.

11 Particles X (of mass 4 units) and Y (of mass 9 units) move directly towards each other,
collide and then separate.

If Δv x is the change of velocity of X and Δv y is the change of velocity of Y, the


magnitude of the ratio Δv x / Δv y is

A 9/4 B 3/2 C 2/3 D 4/9

12 A trolley moves along a track from P to Q, as shown in Fig. 12. The trolley has a
kinetic energy of 60 kJ at P. Its potential energy at Q is 40 kJ less than that at P. The
work it does against friction from P to Q is 10 kJ.

trolley
P
Q

Fig. 12

The kinetic energy of the trolley at Q is

A 10 kJ B 50 kJ C 90 kJ D 100 kJ

13 An electric motor is required to haul a cage of mass 200 kg up a mine shaft through a
vertical height of 800 m in 4.0 minutes.

What will be its electrical power required if its overall efficiency is 75%?

A 0.89 kW B 4.9 kW C 5.2 kW D 8.7 kW


7

14 Which graph correctly represents the variation of centripetal acceleration a of a point on


a disc with the radial distance r from the axis of rotation when the disc is rotating with a
constant angular velocity?

a A a B

r r

a C a D

r r

15 When an aircraft is moving in a horizontal circle at a constant speed, it tilts at an angle


of 20 to the vertical, as shown in Fig. 15.

20

Fig. 15

What is the ratio of the centripetal force to the weight of the aircraft?

A 0.342 B 0.364 C 0.940 D 2.75

16 The gravitational field strength at a point on the surface of spherical planet of radius r
and uniform density  is g. What is the corresponding value on the surface of another
planet of radius 2r and density 1.5?

A 0.38 g B 0.75 g C 1.5 g D 3.0 g

17 The weight of a satellite at the surface of the Earth is W. What is the gravitational force
on the satellite when it is orbiting at a height of ½R, where R is the radius of the Earth?

A 0 B W C 4 W D 4W
9
8

18 Charges –5 C and +5 C are situated at points P and Q respectively as shown in Fig.


18 in a vertical plane and point P is vertically above point Q. X lies midway between P
and Q while Y is at the same vertical position as X but displaced to the right.

Fig. 18

Which of the following correctly describes the electric potentials and the directions of
the electric field at point X and Y?

at point X at point Y
Electrical potential Electric field Electrical potential Electric field
A zero towards Q zero towards Q
B negative towards Q negative downwards
C zero towards P zero upwards
D negative towards P negative towards P

19 N small conductors, on the edge of an insulating disc of radius r, are each given a
charge of Q, as shown in Fig. 19. The frequency of rotation of the disc is f.

r
Q Fig. 19
Q

What is the equivalent electric current at the edge of the disc?

A NQf B NQ/f C NrQf D NQf/r


9

20 Fig. 20 shows the dimensions of a metal block used as a resistor of resistance R, with
the current along the 20.0 mm direction as shown. The resistivity of the metal is 3.0 x
10–4  m.
20.0 mm

10.0 mm

Fig. 20
2.0 mm

Current Direction

What is the value of R?

A 3.0 x 10–4  B 3.0 x 10–3  C 3.0 x 10–1  D 7.5 x 10–2 

21 Three parallel conductors, carrying equal currents in the directions shown in Fig. 21,
pass vertically through the corners of an equilateral triangle PQR.

Fig. 21

What is the direction of the resultant force F on the conductor at Q?


10

22 A light and flexible conducting loop is hung freely on a smooth horizontal rail as shown.

Fig. 22

A bar magnet is moved away from the loop to the right. Which of the following
statements about the motion and area of the loop is true?
Motion Area
A stationary no change
B to the right no change
C to the right increase
D to the left decrease

23 Fig. 23 (a) shows the initial position of a rectangular coil of wire which is being rotated
with constant angular velocity. A magnetic field acts in the direction X X’.

Fig. 23

Fig. 23 (b) shows the sinusoidal e.m.f. produced across the ends of the coils.

Which of the following correctly describes the movement that would have given this
result?

A Rotation of a quarter revolution about axis Y Y’.


B Rotation of a half revolution about axis X X’.
C Rotation of a half revolution about axis Z Z'.
D Rotation of a quarter revolution about axis X X’.
11

24 An airplane with a wing span of 26.0 m flies at a speed v = 250 m s-1 parallel to the
Earth's surface at a location where the Earth's magnetic field is 70° to the vertical as
shown in Fig. 24.

Fig. 24

If the magnetic flux density, B, is 1.7 x 10-4 T what is the e.m.f. induced between the
wing tips?

A 1.78 V B 1.11 V
C 1.04 V D 0.38 V

25 A mains transformer, as shown in Fig. 25, has a 240 V r.m.s. ac input and a 12 V r.m.s.
output. It is used to light three 12V, 24 W lamps in parallel.

240 V
~ a.c.

Fig. 25

Assume that there are no power losses in the transformer. The current, in ampere,
drawn from the mains is

A 0.10 B 0.21 C 0.30 D 0.42

26 A half rectified square voltage of amplitude 100 V gives the waveform as shown in Fig.
26. The r.m.s. value of the rectified voltage is

Fig. 26

A 100 V B 71 V
C 50 V D 25 V
12


27 A body performs simple harmonic motion, which of the following is/are radian out of
2
phase with displacement?

(i) Velocity
(ii) Acceleration
(iii) Resultant force

A (i) only
B (iii) only
C (i) & (ii) only
D (ii) & (iii) only

28 The graph in Fig. 28 below shows the kinetic energy of a particle which is undergoing
simple harmonic motion, with amplitude of 0.60 cm.

KE/ J
0.01 Fig. 28

0 0.2 0.4 t/ s

What is the maximum acceleration of the particle?

A 1.48 m s-2 B 2.47 m s-2


C 5.92 m s-2 D 9.87 m s-2

29 Data transmitted along glass-fibre cables is in the form of pulses of monochromatic red
light each of duration 2.5 ns. Which of the following is the best estimate of the number of
wavelengths in each pulse?

A 103 B 106 C 109 D 1012

30 When a two-slit arrangement was set up to produce interference fringes on a screen,


using a monochromatic source of green light, the fringes were found to be too close
together for convenient observation. It would be possible to increase the separation of
the fringes by

A decreasing the distance between the slits and the screen.


B increasing the distance between the source and the slits.
C increasing the distance between the two slits.
D replacing the light source with a monochromatic source of red light.
13

31 A water wave P is incident on a wall. A reflected wave Q moves away from the wall.
Fig. 31 illustrates the position of P and Q at time zero.

Fig. 31

X, Y and Z represent three positions of the resulting stationary wave.

In which order does the stationary wave have these positions, beginning at time zero?

A X Y Z Y B Y X Y Z
C Y Z Y X D Z Y X Y

32 In a diffraction grating experiment, the first order image of the 435.8 nm blue light from a
commercial mercury vapour discharge lamp occurred at an angle of 15.8o. A first order
red line was also observed at 23.7o, thought to be produced by an impurity in the
mercury.

The wavelengths of red lines of various elements are listed below. Which element is the
impurity in the mercury lamp?

Element Wavelength/nm
A zinc 636.0
B cadmium 643.3
C hydrogen 656.3
D neon 670.3

33 In a photoelectric experiment, the saturation current produced by shining light of


frequency f on a particular piece of metal is 6.0 nA. What would be the value of this
current if f is increased to 1.5 f while keeping the rate of incidence of photons on the
metal constant?

A 6.0 nA B 8.0 nA C 9.0 nA D 12 nA


14

34 What is the wavelength of electrons that are accelerated from rest across a potential
difference of V? h is the Planck constant, e the electronic charge and m is the mass of
electron.

A hm B eV C eV D h
h 2h
eV m m 2meV

35 Fig. 35 shows the three lowest energy levels of an atom.

Fig. 35

What is the highest frequency of radiation possible from transition within these levels?

A 2.10 x 1015 Hz B 1.17 x 1016 Hz C 7.21 x 1016 Hz D 7.75 x 1016 Hz

36 In an X ray tube, electrons of charge e are accelerated through a potential difference V.


The target is cooled by water of specific heat capacity c with a mass m flowing per unit
time. If n electrons strike the target per unit time, what is the maximum possible rise in
the temperature of the water?

A n(eV + mc) B nmeV/c C eV/mnc D eVn/mc

37 Fig. 37 shows the energy levels of a substance used to produce laser.

E3

E2 Metastable state Fig. 37

E1 Ground state

Which of the following statements about the process of laser production is correct?

A Population inversion is achieved when there are less atoms in the metastable state
than in the ground state.
B Electrons in the ground state are pumped to energy level 3 so that they may
undergo transition to the metastable state to achieve population inversion.
C Laser is produced by coherent photons of energy equal to the difference between
E 3 and E 2 .
D A metastable state is necessary so that electrons may return immediately from
level E 2 to the ground state.
15

38 The diagram below shows the energy band structure of three materials P, Q and R.

Energy

Conduction
band (empty) Conduction
band (empty) Conduction
band
Valence
Valence band Valence
band (filled) band
(filled) (filled)

P Q R

Which of the following states correctly the types of material that P, Q and R belong to?

Conductor Semi-conductor Insulator

A R Q P
B P R Q
C Q P R
D P Q R

39 A piece of wood from a recently cut tree shows 12.4 decays per minute of 14C. A sample
of the same size from a tree cut many years ago shows 3.5 decays per minute. 14C has
a half life of 5700 years. What is the age of the sample?

A 9,600 years B 10,400 years C 11,100 years D 12,300 years

40 The speed of an electron is measured to within an uncertainty of 2.0 x 104 m s-1. What is
the minimum uncertainty in the position of the electron?

A 2.6 x 10-39 m B 3.3 x 10-25 m C 2.9 x 10-9 m D 3.6 x 10-8 m

End of Paper
Candidate Name _________________ ______________ Civics Class ___________

TAMPINES JUNIOR COLLEGE


Preliminary Examination 2010

PHYSICS 9646/02
Higher 2

PAPER 2
THURSDAY 2 SEPTEMBER 2010
1 hour 45 minutes

Time 0800 – 0945 hr

Candidates answer on the Question Paper.


No additional materials are required.

READ THESE INSTRUCTIONS FIRST

Write your name and class on all the work you hand in.
Write in dark blue or black pen in the spaces provided on the Question Paper.
You may use a soft pencil for any diagrams, graphs or rough working.
Do not use paper clips, highlighters, glue or correction fluid.

Section A
Answer all questions.
It is recommended that you spend about 1 hour 15 minutes on this section.

Section B
Answer Question 7. For Examiner's Use
It is recommended that you spend about 30 minutes on this
section. 1 9

2 8
At the end of the examination, fasten all your work securely
together. 3 13
The number of marks is given in brackets [ ] at the end of each
question or part question. 4 9

5 4

6 17

7 12

Total 72

This document consists of 18 printed pages and 0 blank page.


2

9646 H2 PHYSICS (2010)


3
Section A

Answer all questions.


It is recommended that you spend about 1 hour 15 minutes on this section.

1 (a) Galileo’s famous demonstration at the Tower of Pisa showed that falling objects
accelerate equally, regardless of their masses. This is strictly true if air resistance is
negligible.

Using Newton’s Second Law, show that a 10 kg canon ball and a 1 kg stone, when
dropped together from the top of the tower, can strike the ground at the same time.

[2]

(b) A body is released in a fluid. With the aid of a free body diagram, explain how the
body falling through a fluid can reach terminal velocity.

.....................................................................................................................................

.....................................................................................................................................

.....................................................................................................................................
[4]

(c) A parachutist has a mass of 80 kg. When he falls with his parachute open, the air
resistance R he encounters is given by the equation R = k v2, where v is the
parachutist’s velocity and k has the value of 35 N s2 m-2.
Determine the magnitude and direction of the acceleration of the parachutist when
his velocity is 5.0 m s-1.

magnitude of acceleration = ……………………………….. m s-2

direction of acceleration = ………………………………… [3]


4

2 Sphere P of mass 2.0 kg and sphere Q of mass 1.0 kg are moving towards each other
with speeds 2.0 m s–1 and 1.0 m s–1 respectively, as shown in Fig. 2.1.

2.0 m s–1 1.0 m s–1

P Q Fig. 2.1

2.0 kg 1.0 kg

The spheres have a head-on, inelastic collision. The force that P exerts on Q during the
collision varies with time as shown in Fig. 2.2.

F/N

150
Fig. 2.2

0
0 20 40 t / ms

(a) State the principle of conservation of momentum.

.....................................................................................................................................

.....................................................................................................................................

.....................................................................................................................................
[1]

(b) Determine the momentum change of mass Q after the collision.

change of momentum = ……………………………….. kg m s-1 [1]


5

(c) Sketch, with clear labeling of values, a graph of the force that Q exerts on P using
the axes provided.

F/N

0 t / ms

[2]
(d) Calculate the velocities of P and Q after collision.

velocity of P = ……………………………….. m s-1

velocity of Q = ……………………………….. m s-1 [2]

(e) Calculate percentage loss in total kinetic energy of P and Q after the collision.

percentage loss in total kinetic energy = ……………………………….. % [2]


6

3 (a) Define potential difference and the volt.

………………………………………………………………………………………..............

………………………………………………………………………………………………...

………………………………………………………………………………………………...

………………………………………………………………………………………………...
[2]

(b) The variation of resistance R of a thermistor with temperature T is shown in Fig.


3.1.

R / kΩ

T/K

Fig. 3.1

The above thermistor is connected in a potential divider circuit as shown in Fig. 3.2
with a battery of e.m.f. 12.0 V and negligible internal resistors. The thermistor is
placed in the freezer of a meat handling factory. It functioned as a temperature
probe to activate a switch to power the freezer and the switch will be on if the
potential at point P is at 4.5 V.
7

12.0 V

P
0V

5.0 kΩ

Fig. 3.2

(i) What is meant by the expression an e.m.f. of 12 V ?

…………………………………………………………………………………………

…………………………………………………………………………………………
[1]

(ii) State and explain the effect of a decrease in surrounding temperature in the
freezer on the potential at point P.

…………………………………………………………………………………………

…………………………………………………………………………………………

…………………………………………………………………………………………

…………………………………………………………………………………………
[3]

(iii) Use Fig. 3.1 to determine the temperature that would trigger the switch.

temperature = …………………………….. K [3]


8

(iv) Suggest why it is reasonable to choose a value of 4.5 V as a trigger potential


in this context.

…………………………………………………………………………………………

…………………………………………………………………………………………
[1]

(v) Without changing the thermistor and keeping the trigger potential at 4.5 V,
suggest one way that the circuit could be modified if a different trigger
temperature is desired.

…………………………………………………………………………………………

…………………………………………………………………………………………
[1]

(vi) Noting the usefulness of such a temperature probe, it was suggested that a
similar circuit in Fig. 3.2 to be used in a device for controlling a boiler. It is
desired that when the temperature is 78oC, the switch will be activated to boil
the liquid. Discuss whether this proposal is feasible.

…………………………………………………………………………………………

…………………………………………………………………………………………

…………………………………………………………………………………………

…………………………………………………………………………………………
[2]

4 (a) What is meant by the photoelectric effect?


.....................................................................................................................................

.....................................................................................................................................

.....................................................................................................................................
[1]
9

(b) The circuit as shown in Fig. 4 is used to study the photoelectric effect with sodium.
When a piece of sodium metal is irradiated by monochromatic light of wavelength
420 nm, the stopping potential is found to be 0.65 V. When light of wavelength 310
nm is used, the stopping potential is 1.69 V.

light

Variable voltage source


Fig. 4

(i) What is meant by stopping potential?

………………………………………………………………………………………….

………………………………………………………………………………………….

………………………………………………………………………………………….
[1]

(ii) Explain how the stopping potential may obtained experimentally using the
circuit shown.

………………………………………………………………………………………….

………………………………………………………………………………………….

………………………………………………………………………………………….

………………………………………………………………………………………….

………………………………………………………………………………………….

…………………………………………………………………………………………

………………………………………………………………………………………….

[3]
10

(iii) Determine the work function of sodium metal and the value of Planck
constant.

work function of sodium metal = ……………………………….. J

value of Planck constant = ……………………………….. J s [4]

5 A semi-conductor diode is formed by joining a p-type and an n-type semiconductor.

(a) What is meant by a depletion region at the p-n junction?

.....................................................................................................................................

.....................................................................................................................................
[1]

(b) Describe the formation of a depletion region at the p-n junction.

.....................................................................................................................................

.....................................................................................................................................

.....................................................................................................................................

.....................................................................................................................................

.....................................................................................................................................

.....................................................................................................................................
[3]
11

6 Capacitors are used in virtually every electronics circuit that is built today. A capacitor is an
electrical device that is able to store electrical energy. It is basically made up of two
conducting sheets or plates which are separated by an insulator, such as mica, ceramic or
paper. It is charged by using direct current, which will result in the capacitor having a
potential difference and the two conductors carrying charges of opposite sign. A charged
capacitor is able to deliver electrical energy to a resistor in a way similar to a cell.

A particular capacitor C is connected in a circuit as shown in Fig. 6.1, to a cell with an emf
of 12.0 V and a resistor of 30.0 kΩ. A current sensor is used to measure the variation of
current in the circuit with time, and a part of the graph is shown in Fig. 6.2 below.

12.0 V 30.0 kΩ

A
Current sensor
Fig. 6.1

Fig. 6.2
12

At any instant, the emf of the cell E, the potential difference across the capacitor Vc and
the potential difference across the resistor VR are related by the following equation:

E = VC + VR

(a) As the capacitor is charged by the current flow, indicate on Fig. 6.1 the plate that is
positively charged.
[1]

(b) Suggest why the current decreases with time.

...................................................................................................................................

...................................................................................................................................

...................................................................................................................................
[2]

(c) If the circuit remains closed for a sufficiently long time, state the final value of

(i) the current in the circuit,

current = ……………………………….. A [1]

(ii) the potential difference across the capacitor

potential difference across capacitor = ……………………………….. V [1]

(d) At time t = 10 s, deduce the potential difference across the capacitor.

potential difference across capacitor = ……………………………….. V [2]

(e) Use Fig. 6.2 to estimate the charge stored on the capacitor after 10 s.

charge stored on the capacitor = ……………………………….. C [2]


13

The capacitance of a capacitor is defined as the ratio of the charge stored to the potential
difference across it.

(f) Use your answers above to determine the capacitance of C.

capacitance of C = ……………………………….. C V-1 [1]

(g) On the axes below, sketch how the potential difference across capacitor C will vary
with time.
Pd across C

time
0

[2]

It is suggested that the relation between the current and time is

where I0 is the current at time t = 0 s, and k is a constant for this circuit.

Some data from Fig. 6.1 are used to plot the graph of ln I with time as shown in Fig. 6.3.
14

Fig. 6.3

(h) Use Fig. 6.3 to deduce the value of k.

Value of k = ……………………………….. s [2]

The value of k is known as the time constant for this circuit, and it depends only on the
resistance of R and the capacitance of C.

(i) If the cell in the circuit in Fig. 6.1 is replaced by another cell of emf 24.0 V and
capacitor C is replaced by an identical one that is uncharged, draw the
corresponding graph for this circuit in Fig. 6.3.

[3]
15
Section B

It is recommended that you spend about 30 minutes on this section.

7 A particular radioactive source emits alpha α, beta β and gamma γ radiation. Information
relating to the penetrating properties of each type of radiation from this source is given in
Fig. 7.1.

radiation type penetrating properties


Complete absorption by a sheet of paper or by a
α
sheet of aluminium 5 mm thick
Negligible absorption by paper; 99% absorption by
β
a sheet of aluminium 5 mm thick
Negligible absorption by paper or by a sheet of
γ
aluminium 5 mm thick
Fig. 7.1

A scientist wishes to know how well a beam of beta radiation is absorbed by air, but the
only radioactive source available is a radioisotope which emits all three types of radiation
as detailed above.

Design a laboratory experiment to investigate how the absorption of beta radiation only
depends on the distance which the radiation has travelled through the air.

The information in Fig. 7.1 may be helpful in some aspects of the design. You may
assume that all the equipment listed below is available, together with any other standard
laboratory apparatus which may be found in a school or college science laboratory.

A radioactive source emitting all three types of Ammeter


radiation (i.e α-radiation, β-radiation and γ-
radiation)
Ratemeter (this measures output from Geiger Voltmeter
Muller tub in terms of counts per unit time)
Long glass tank filled with water Stopwatch

Oscilloscope Small plate of lead (2 mm thick)


Small sheet of aluminium (5 mm
Geiger-Müller tube
thick)
Signal generator Sheet of paper

High voltage power supply unit Metre Rule

Tongs
16

You should draw a diagram of the arrangement of the apparatus you would use, and in
your account you should pay particular attention to

(a) the method by which the count rate due to beta radiation only is measured,
(b) the procedure to be followed,
(c) any safety precautions which you would take. [12]

Diagram:

……………………………………………………………………………………………………………

……………………………………………………………………………………………………………

……………………………………………………………………………………………………………

……………………………………………………………………………………………………………

……………………………………………………………………………………………………………

……………………………………………………………………………………………………………

……………………………………………………………………………………………………………
17

……………………………………………………………………………………………………………

……………………………………………………………………………………………………………

……………………………………………………………………………………………………………

……………………………………………………………………………………………………………

……………………………………………………………………………………………………………

……………………………………………………………………………………………………………

……………………………………………………………………………………………………………

……………………………………………………………………………………………………………

……………………………………………………………………………………………………………

……………………………………………………………………………………………………………

……………………………………………………………………………………………………………

……………………………………………………………………………………………………………

……………………………………………………………………………………………………………

……………………………………………………………………………………………………………

……………………………………………………………………………………………………………

……………………………………………………………………………………………………………

……………………………………………………………………………………………………………

……………………………………………………………………………………………………………

……………………………………………………………………………………………………………

……………………………………………………………………………………………………………

……………………………………………………………………………………………………………

……………………………………………………………………………………………………………
18
……………………………………………………………………………………………………………

……………………………………………………………………………………………………………

……………………………………………………………………………………………………………

……………………………………………………………………………………………………………

……………………………………………………………………………………………………………

……………………………………………………………………………………………………………

……………………………………………………………………………………………………………

……………………………………………………………………………………………………………

……………………………………………………………………………………………………………

……………………………………………………………………………………………………………

……………………………………………………………………………………………………………

……………………………………………………………………………………………………………

……………………………………………………………………………………………………………

……………………………………………………………………………………………………………

……………………………………………………………………………………………………………

……………………………………………………………………………………………………………

……………………………………………………………………………………………………………

……………………………………………………………………………………………………………

……………………………………………………………………………………………………………

……………………………………………………………………………………………………………

……………………………………………………………………………………………………………

End of Paper
Candidate Name ____________________________________ Civics Class ___________

TAMPINES JUNIOR COLLEGE


Preliminary Examination 2010

PHYSICS 9646/03
Higher 2
PAPER 3
MONDAY 13 SEPTEMBER 2010
2 hours

Time 0800 - 1000 hr

Candidates answer on the Question Paper.


No additional materials are required.

READ THESE INSTRUCTIONS FIRST


Write your name and class on all the work you hand in.
Write in dark blue or black pen in the spaces provided on the Question Paper.
You may use a soft pencil for any diagrams, graphs or rough working.
Do not use paper clips, highlighters, glue or correction fluid.

Section A
Answer all questions. For Examiner's Use

Section B 1 9
Answer any two questions.
2 8
You are advised to spend about one hour on each section.
3 7
At the end of the examination, fasten all your work securely
together. 4 8
The number of marks is given in brackets [ ] at the end of each
question or part question. 5 8

Section B

6 20

7 20

8 20

Total 80

This document consists of 17 printed pages and 1 blank page.


2

9646 H2 PHYSICS (2010)


3
Section A

Answer all the questions in this section.

1 In a heat engine, the working substance is an ideal monatomic gas with 3.0 moles of
molecules. The gas undergoes a cycle of thermodynamic processes ABCDA as it drives
the engine, as shown in Fig. 1.

p / 106 Pa

Fig. 1

V / 10-3 m3

Fig. 1

(a) Determine the thermodynamic temperature of the gas at B.

temperature = ……………………………….. K [2]

(b) If the average kinetic energy of a molecule of a gas at temperature T is given by


3kT/2, determine the change in internal energy of the gas in the process BC.

change in internal energy = ……………………………….. J [3]

(c) Determine the work done by the gas in the process BC.

work done by gas = ……………………………….. J [2]


4
(d) Determine the heat absorbed by the gas in the process in BC.

heat absorbed by gas = ……………………………….. J [2]

2 The movement of an α-particle in vacuum in the (x, y) plane (indicated in Fig. 2.1) is
observed at two instants, t = 0.0 s and t = 2.0 x 10-7 s and the vectors are shown in Fig.
2.2 and Fig. 2.3 respectively.

60 o

Fig. 2.1 Fig. 2.2 at Fig. 2.3 at


t = 0.0 s, t = 2.0 x 10-7 s
v = 5.0 x 104 m s -1 v = 10.0 x 104 m s -1

(a) By drawing a vector diagram to scale, or otherwise, find the change in velocity.

change in velocity = ……………………………….. m s-1 [3]

direction of change = …………………………………

(b) Hence determine the magnitude of the acceleration of the α-particle.

magnitude of acceleration = ……………………………….. m s-2 [1]


5
(c) Assuming that the acceleration of the α-particle has been caused by the application
of a uniform electric field of field strength E. Determine the magnitude of E.
(Neglect the effect of gravity)

magnitude of E = ……………………………….. V m-1 [2]

(d) Suggest quantitatively why it is reasonable to neglect the effect of gravity in this
case.

[2]

3 A small delivery truck can be thought of as a box supported by four springs, one at each
wheel (the suspension of the truck).

On a particular road, speed bumps are put on the road to slow down the traffic. After
passing rapidly over one of these speed bumps, a delivery truck experiences rapid
vertical oscillations.

Speed bumps

Fig. 3.1
Fig. 3.2 shows a graph of acceleration, a, against displacement (from equilibrium), x, for
the motion of the truck.
a / m s-2

7.6

x/m
-0.10 0.10

- 7.6

Fig. 3.2
6
(a) Calculate the angular velocity  of the truck.

 = ……………………………….. rad s-1 [2]

(b) Calculate the shortest time taken t for the truck to oscillate from its lowest point to a
point 0.025 m below its equilibrium position.

t = ……………………………….. s [3]

(c) If the truck travels at a certain speed over the series of speed bumps, the vertical
oscillations can be very large. Explain why this is so.

.....................................................................................................................................

.....................................................................................................................................

.....................................................................................................................................
[2]

4 In Fig. 4 below, a negatively charged particle of mass m and charge q = - 5.0 x 10-6 C
performs uniform circular motion horizontally in a clockwise direction (when viewed from
the top). This motion is performed in a region with a uniform magnetic field and a
uniform electric field. Both fields point downwards and have the strengths B = 0.50 T
and E = 150 N C-1 respectively.

Fig. 4

(a) Draw a free body diagram to show all forces acting on the charged particle.

[2]
7
(b) Show that the period of revolution of the charged particle does not depend on the
radius of the circular path.

[3]

(c) Suppose that the electric field is removed, state and explain what will happen to
the path of the charged particle.

.....................................................................................................................................

.....................................................................................................................................

.....................................................................................................................................

.....................................................................................................................................
[3]

5 There are several types of fusion reactions. One of them involved the isotopes of
hydrogen called deuterium ( 12 H ) and tritium( 13 H ):

2
1 H + 2
1 H → 3
1 H + 1
1 H

(a) Calculate the energy released from this reaction, given the following:

Mass of deuterium = 3.34358 x 10-27 kg

Mass of proton = 1.67262 x 10-27 kg

Mass of tritium = 5.00823 x 10-27 kg

energy released = ……………………………….. J [3]


8
(b) If two deuterium nuclei are to approach each other for this reaction to occur, they
must overcome the electrostatic repulsion between them. Given that the radius of
deuterium nucleus is 1.5 x 10-15 m, estimate the electrostatic potential energy of the
two nuclei when they are just touching.

electrostatic potential energy = ……………………………….. J [2]

(c) If the average kinetic energy of a molecule of a gas at temperature T is given by


3kT/2, show that the minimum temperature of the deuterium gas required for the
reaction to proceed is of the order 109 K.

minimum temperature = ……………………………….. K [2]

(d) The temperature inside the Sun is about 107 K. Suggest why nuclear fusion is able
to occur inside the Sun.

.....................................................................................................................................

.....................................................................................................................................

.....................................................................................................................................
[1]
9
Section B

Answer two questions for this section.

6 (a) The gravitational field strength of the Earth at a point P that is 2100 km above its
surface is 5.56 N kg-1.

(i) Show that the acceleration of free fall at P is 5.56 m s-2.

[2]

(ii) Determine the gravitational force exerted on a 5.0 kg mass at point P.

gravitational force = ……………………………….. N [2]

(iii) Show that N kg-1 is equivalent to m s-2 in base units.

[1]

(b) (i) Define gravitational potential at a point.

.....................................................................................................................................

.....................................................................................................................................

.....................................................................................................................................
[1]
10
(ii) Given that the Earth has a mass of 5.98 x 1024 kg and a radius of 6.37 x 106 m,
determine the gravitational potential at

1. point P, which is at a height of 2100 km above the surface.

2. point Q, which is at a height of 4200 km above the surface.

gravitational potential at point P = ……………………………….. J kg-1

gravitational potential at point Q = ……………………………….. J kg-1 [3]

(iii) Hence determine the work done in bringing a 5.0 kg mass from point P to point Q
without changing its kinetic energy.

work done = ……………………………….. J [2]

(c) Explain whether the answer to (b)(iii) can be calculated by using the equation

Work done = force x displacement,

where the force is the value calculated in (a)(ii) and the displacement is 2100 km.

..............................................................................................................................................

..............................................................................................................................................

..............................................................................................................................................
[2]
11
(d) A weather satellite is sent up to a polar orbit that contains point P.

(i) Determine the period of the satellite.

period = ……………………………….. s [3]

(ii) Determine the linear velocity of the satellite.

linear velocity = ……………………………….. m s-1 [2]

(iii) State one advantage of having the weather satellite in this polar orbit as
compared to the geostationary satellite which has an orbital radius of 42 000 km.

.....................................................................................................................................

.....................................................................................................................................

.....................................................................................................................................
[1]

(iii) Explain why despite being attracted by Earth’s gravitational force, the orbiting
weather satellite will not fall down to the Earth.

.....................................................................................................................................

.....................................................................................................................................

.....................................................................................................................................
[1]
12
7 A rectangular coil ABCD of N turns has its plane set parallel to a uniform magnetic field B
as shown in Fig. 7.1. The coil of sides AB and CD are of length L and side BC is of length y.
A current I is flowing through the coil from P to Q.

B C

N A D S

P Q

Fig. 7.1

(a) Define the term torque of a couple.

..............................................................................................................................................

..............................................................................................................................................

..............................................................................................................................................
[1]

(b) Indicate in Fig. 7.1, the direction of the force acting on sides AB and CD respectively.
Label them F.
[2]

(c) Write an equation for the torque acting on the coil ABCD at the instant shown in Fig.
7.1.

..............................................................................................................................................
[1]

(d) The current I is then switched off and the coil is rotated about an axis as shown in Fig.
7.1 at a constant angular velocity  . Using the law(s) of electromagnetic induction,
explain why an e.m.f. is induced in the coil.

..............................................................................................................................................

..............................................................................................................................................

..............................................................................................................................................

..............................................................................................................................................

..............................................................................................................................................

..............................................................................................................................................

..............................................................................................................................................

[3]
13
(e) Draw sketch graphs to show the variation of

(i) the magnetic flux linkage through the coil with respect to time for a complete
revolution, where 0  t  T, in Fig. 7.2. Mark the peak values on the vertical
axes of your graph.

(Assuming that Fig. 7.1 shows the position of the coil ABCD at t = 0 and the
rotating coil is always within the magnetic field B.)

Fig. 7.2

[2]

(ii) the induced e.m.f. in the coil with respect to time for a complete revolution,
where 0  t  T, in Fig. 7.3.

Fig. 7.3

[3]

(f) State the effects on your sketch in (e) (ii) when the angular velocity  is increased.

..............................................................................................................................................

..............................................................................................................................................

..............................................................................................................................................

..............................................................................................................................................
[2]
14
(g) Given that the terminals at the side PQ is now connected across a transformer, where
the secondary coil of the transformer is connected to a diode and a calculator circuitry
as shown in the Fig. 7.4.

Fig. 7.4
(i) Assuming that the primary coil has a peak voltage of 240 V and the turns ratio of
the transformer is 20:1, calculate the r.m.s voltage in the secondary coil of the
transformer.

r.m.s voltage = ……………………………….. V [3]

(ii) Using Fig 7.5, sketch a graph to show how the voltage varies with time across
the calculator circuitry.

Voltage

Fig. 7.5
[1]

(h) State and explain one disadvantage of using a diode in the above application.

..............................................................................................................................................

..............................................................................................................................................

..............................................................................................................................................
[2]
15

8 (a) The following short passage is based on extracts from the catalogues of suppliers of
college scientific equipment. Read the passage and answer the questions that follow.

HELIUM-NEON LASER AND ACCESSORY SET

This laser emits an intense, coherent monochromatic light of wavelength 633 nm


and power output 1.0 mW. The beam is plane-polarised. Also available is a range
of slits and gratings to illustrate the phenomena of interference and diffraction.

(i) Explain the following terms in the passage.

1. coherent

.....................................................................................................................................

.....................................................................................................................................
2. diffraction

.....................................................................................................................................

.....................................................................................................................................
[2]

(ii) The beam divergence is 1.2 milliradians. The laser beam is directed normally at a
wall 6.0 m away. Estimate the diameter of the spot of light on the wall.

diameter = ……………………………….. m [2]

(b) A double slit system is illuminated by the laser mentioned in (a) and produces an
interference pattern on the screen as shown in Fig. 8.1. The diagram is not drawn
to scale.

R
S fringe pattern

dark
bright

laser
beam

screen
Fig. 8.1
16
(i) Calculate the path difference between the laser beams emerging from the two
slits at

1. point R, and

path difference = ……………………………….. m [2]

2. point S.

path difference = ……………………………….. m [2]

(c) In a Young’s double-slit experiment, the fringe separation observed using yellow light
was found to be 0.275 mm.

The yellow lamp, giving a wavelength of 5.50 x 10-7 m is replaced by a purple light
source that is made of 2 wavelengths; violet light with wavelength 4.00 x 10-7 m and red
light with wavelength 6.00 x 10-7 m. The remainder of the apparatus is undisturbed.

(i) Calculate the distance between the fringes formed by the violet light.

distance between violet fringes = ……………………………….. m [2]

(ii) Calculate the distance between the fringes formed by the red light.

distance between red fringes = ……………………………….. m [2]


17
(iii) Calculate the distance from the purple fringe on the axis to the next purple fringe
observed.

distance between purple fringes = ……………………………….. m [3]

(iv) Hence, draw a diagram of the appearance of the new fringe system, indicating the
colours and extending as far as 1mm from the axis.

[3]

(v) In what way(s) would the fringe pattern change if the purple light source is
replaced by a source producing white light assuming source is coherent? Briefly
explain.

.....................................................................................................................................

.....................................................................................................................................

.....................................................................................................................................

.....................................................................................................................................
[2]

End of Paper
VICTORIA JUNIOR COLLEGE
2010 JC2 PRELIMINARY EXAMINATIONS

PHYSICS 9646/01
Higher 2

Paper 1 Multiple Choice

23/9/2010 1400 h – 1515 h


THURSDAY (1 h 15 min)

READ THESE INSTRUCTIONS FIRST


Write in soft pencil.

Do not use staples, paper clips, highlighters, glue or correction fluid.

Write your name and NRIC number on the Answer Sheet in the spaces provided.

There are forty questions on this paper. Answer all questions. For each question
there are four possible answers A, B, C and D.
Choose the one you consider correct and record your choice in soft pencil on the
separate Answer Sheet.

Read the instructions on the Answer Sheet very carefully. Please shade the
ovals on the Answer Sheet correctly.

Each correct answer will score one mark. A mark will not be deducted for a wrong
answer.

Any rough working should be done in this booklet.


 

This document consists of 18 printed pages.


VJC Preliminary Exams 2010 Physics H2/P1/9646/1
2

Data

speed of light in free space, c = 3.00 x 108 m s-1

permeability of free space, μo = 4π x 10-7 H m-1

permittivity of free space, εo = 8.85 x 10-12 F m-1

(1/(36π)) x 10-9 F m-1

elementary charge, e = 1.60 x 10-19 C

the Planck constant, h = 6.63 x 10-34 J s

unified atomic mass constant, u = 1.66 x 10-27 kg

rest mass of electron, me = 9.11 10-31 kg

rest mass of proton, mp = 1.67 x 10-27 kg

molar gas constant, R = 8.31 J mol-1 K-1

the Avogadro constant, NA = 6.02 x 1023 mol-1

the Boltzmann constant, k = 1.38 x 10-23 J K-1

gravitational constant, G = 6.67 x 10-11 N m2 kg-2

acceleration of free fall, g = 9.81 m s-2

VJC Preliminary Exams 2010 Physics H2/P1/9646/1


3

Formulae

uniformly accelerated motion 1


s = ut + at2
2

v2 = u2 + 2as

work done on/by a gas W = p ΔV

hydrostatic pressure p = ρgh


gravitational potential φ = -GM/r

displacement of particle in s.h.m. x = xo sin ωt

velocity of particle in s.h.m. v = vo cos ωt

= ± ω xo 2 - x 2

resistors in series  R = R1 + R2 + … 

resistors in parallel  1/R = 1/R1 + 1/R2 + …

electric potential  V = Q/4πεor

alternating current/voltage  x = xo sin ωt 

transmission coefficient T = exp(-2kd)

  8π m(U − E )
2

= 2
 
where k h

radioactive decay x = xo exp(-λt )


decay constant  0.693
 
λ = t1
2

VJC Preliminary Exams 2010 Physics H2/P1/9646/1


4

1 Which of the following gives the estimated number of atoms in your body?
A 1024 B 1027 C 1030 D 1033

2 The ideal gas equation can be given as pVm = RT, where Vm is the volume per mole
of gas. The behaviour of many real gases deviates from the ideal gas equation but
can be represented quite closely by an equation of the form
⎛ a ⎞
⎜ p + 2 ⎟(Vm − b) = RT
⎝ Vm ⎠
in which the values of a and b are characteristic of the particular gas.
What are the units of a and b?

a b
A Pa m6 mol-2 m3 mol-1
B Pa m6 mol-2 m-3 mol
C Pa m-6 mol2 m-3 mol-1
D Pa m-6 mol2 m3 mol-1

3 Experimental data taken of a child sliding down a playground slide provided the
following data.

Time/s 0 1.0 2.0 3.0 4.0 5.0 6.0


Speed/m s-1 0 1.0 2.0 3.0 3.5 4.0 4.5

Which diagram represents the slope of the playground?

A B

C D

4 An artillery gun sited at the top of a cliff fires a shell horizontally so as to hit a target
2.0 km away from the bottom of the cliff. If the cliff is 180 m high, the initial velocity of
the shell is

A 54 m s-1 B 99 m s-1 C 110 m s-1 D 330 m s-1

VJC Preliminary Exams 2010 Physics H2/P1/9646/1


5

5 A sphere of mass 3.00 kg rests on a frictionless slope as shown.

Wall
30˚

The spring obeys Hooke’s Law. The spring constant is 500 N m-1. What is the
compression in mm of the spring?
A 29.4 B 34.3 C 51.0 D 58.9

6 The figure below represents the various forces acting on a car moving towards the
right. The driving force, D acts on the front wheels and the total resistive force is
represented by the force, R. The weight W of the car is 12000 N and it acts on the
centre of mass, G which is 90 cm above the ground.

1.5 m 0.50 m

R G
A B
D
B A
W
Given that the values of D and R are both 7000 N, what are the values of the normal
reaction forces at A and at B acting on the wheels?

Normal reaction force at A Normal reaction force at B


A 8100 N 3900 N
B 6000 N 6000 N
C 6150 N 5850 N
D 5850 N 6150 N

7 A helicopter rises vertically with a constant speed. According to Newton’s third law,
there is a force which makes an action-reaction pair with the weight of the helicopter.

Which of the following is this force?


A The lift force created by the engine of the helicopter.
B The gravitational force on the Earth due to the helicopter.
C The air resistance on the helicopter as it rises up.
D The upthrust acting on the helicopter due to air around it.

VJC Preliminary Exams 2010 Physics H2/P1/9646/1


6

8 The graph below shows how the force acting on a 2.0 kg body varies with time.

F/N
6.0

2.0

t/s
0
1.0 3.0
Assuming that the body is initially moving in a straight line at 3.0 m s-1, what is its
final velocity?
A 8.0 m s-1 B 10.0 m s-1 C 13.0 m s-1 D 16.0 m s-1

9 Two identical blocks are released from rest from the tops of two ramps as shown
below.

60°
30°
 
 
Assuming  no  friction,  what  is  the  ratio  of  their  speeds  if  the  ratio  of  the  length 
X/Y is 2.0? 
A   1.1      B   1.9      C   2.0      D  3.5 
 
             
10 The speed of a vehicle of total mass 1.60 x 103 kg was brought down to 14.0 m s-1 on
a level road by applying brakes. On braking, 550 kJ of heat was produced.
What is the speed of the vehicle just before the brakes were applied?
A 40.2 m s-1 B 29.7 m s-1 C 26.2 m s-1 D 22.2 m s-1

11 A ball of mass 0.10 kg is attached to a string and swung in a vertical circle of radius
0.50 m, at a constant frequency of 1.9 revolutions per second. The string snaps at
the position when the tension is greatest in its circular path.
What was the tension in the string when it snapped?
A 6.1 N B 7.1 N C 8.1 N D 9.1 N

VJC Preliminary Exams 2010 Physics H2/P1/9646/1


7

12 A bird is soaring in a horizontal circular path of radius 2.0 m. Its bank angle relative
to the horizontal is 24° as shown in the diagram below.

24°

What is the speed of the bird?


A 1.5 m s-1 B 3.0 m s-1 C 6.6 m s-1 D 8.7 m s-1

Questions 13 and 14 refers to data given in Table 13.

13 Ekapluto is a hypothetical planet that has two moons in circular orbits. Table 13
below summarizes the hypothetical data about the moons:

Table 13
Mass / kg Radius / m Orbital radius / m Orbital period / s
Moon A 4.0 × 1020 not available 2.0 × 108 4.0 × 106
Moon B 1.5 × 1020 2.0 × 105 3.0 × 108 not available

Using the data from Table 13 above, the mass of Ekapluto is closest to
A 1.5 × 1015 kg
B 9.4 × 1022 kg
C 3.0 × 1023 kg
D 1.9 × 1029 kg

14 Using the data from Table 13 above, the maximum gravitational force between the
two moons is estimated to be
A 1.6 × 1013 N
B 4.4 × 1013 N
C 1.0 × 1014 N
D 4.0 × 1014 N

15 A mass attached to a very light spring executes simple harmonic motion. Which of
the following options will NOT change the total energy of the system?
A halve the force constant of the spring while changing nothing else.
B double the amplitude of vibration while changing nothing else.
C double the mass while changing nothing else.
D double both the amplitude and force constant.

VJC Preliminary Exams 2010 Physics H2/P1/9646/1


8

16 A car driver notices that her rear view mirror shakes a lot at a particular speed. To try
to stop it she sticks a big lump of chewing gum on the back of the mirror. Which one
of the following outcomes is most likely to result?
A The mirror no longer shakes a lot because it is heavily damped.
B The mirror shakes a lot at a different speed because the chewing gum has
changed the resonant frequency of the mirror.
C The mirror stills shakes a lot at the same speed as before because the
chewing gum does not change the damping.
D The mirror shakes a lot at a different speed because the chewing gum
changes the damping.

17 In Fig 17 below, heat is added to a pure substance in a closed container at a


constant rate. A graph of the temperature of the substance as a function of time is
shown here.

Fig 17

If Lf is the latent heat of fusion and Lv is the latent heat of vaporization of the
Lv
substance, what is the value of the ratio for this substance?
Lf
A 1.0 B 2.4 C 3.5 D 5.0

18 An ideal gas is held in a container of volume V at pressure P. The root-mean-


squared (rms) speed of a gas molecule under these conditions is u. If the volume
and pressure are now changed to 2V and 2P, the rms speed of a molecule will be
A u B 2u C 2u D 4u

VJC Preliminary Exams 2010 Physics H2/P1/9646/1


9

19 In a progressive transverse wave set-up in a string, Fig 19.1 below shows the shape
of the string at time t = 0. X and Y are two points on the string.

Displacement

position
X Y

Fig 19.1
Displacement

time
0

Fig 19.2

Which of the following statements is true if Fig 19.2 represents the displacement-time
graph for point X?
A Point Y will have a similar displacement-time graph as it is of the same phase
as X in oscillation.
B Point X has maximum kinetic energy while Y has maximum potential energy.
C The progressive wave is travelling towards the left.
D The point Y is moving upward with maximum speed at time t = 0.

20 Two wave generators S1 and S2 produce water waves of wavelength 2.0 m. They are
placed 6.0 m apart as shown and are operated in phase. A sensor D which
measures the amplitude of water waves is 7.0 m away from S1 as shown in the
diagram below.
S1 6.0 m S2

7.0 m

D
The shortest distance D could be moved along the straight line S1D in order to detect
large amplitude of the resultant wave motion is
A 1.0 m towards S1 B 3.0 m towards S1
C 1.0 m away from S1 D 3.0 m away from S1

VJC Preliminary Exams 2010 Physics H2/P1/9646/1


10

21 Light from a laser is directed normally at a diffraction grating as shown in the figure
below. The diffraction grating is situated at the centre of the circular scale, marked in
degrees. The readings on the scale for the second order diffracted beams are 140o
and 166o. The wavelength of the laser light is 500 nm.

166o

laser

140o

What is the spacings of the slits of the diffraction grating?


A 1.14 × 10-6 m
B 2.22 × 10-6 m
C 2.28 × 10-6 m
D 4.45 × 10-6 m

22 Which of the following statements about an electric field is incorrect?

A The electric field strength due to a point charge varies as 1/r2 where r is
the distance from the charge.

B Electric field strength is a vector quantity.

C The electric field strength at a point is a measure of the potential gradient


at that point.

D The electric field strength is zero at all points where the potential is zero.

VJC Preliminary Exams 2010 Physics H2/P1/9646/1


11

23 The diagram shows equipotential lines spaced 2.0 cm apart.


80 V 20 V -40 V -100V

equipotential line
2.0 cm  2.0 cm  2.0 cm 
What is the electric force which is exerted on a charge of +5.0 µC when placed
at X as shown?

A 3.0 x 10-2 N to the right


B 3.0 x 10-2 N to the left
C 1.5 x 10-2 N to the right
D 1.5 x 10-2 N to the left

24 Two wires X and Y, each of the same length and the same material, are connected
in parallel to a battery. The diameter of X is half that of Y.
What fraction of the total current passes through X?

A 0.20 B 0.25 C 0.33 D 0.50

25 The current I flowing through a component varies with the potential difference V
across it as shown.

Which graph best represents how the resistance R varies with V?

A B
A

C D
A

VJC Preliminary Exams 2010 Physics H2/P1/9646/1


12

26 A p-n junction diode with the forward characteristic shown in Fig 26.1 is connected in
series with a variable, low voltage d.c. power supply, a meter of negligible internal
resistance and a 50 Ω resistor as shown in Fig 26.2.
current / mA
20

15

10

5 Fig 26.1

0
p.d. / V
0.20 0.40 0.60 0.80 1.00

Fig 26.2

When the meter reads 5 mA, the potential difference across the supply is about
A 0.25 V B 0.75 V C 1.05 V D 1.25 V

VJC Preliminary Exams 2010 Physics H2/P1/9646/1


13

27 A potentiometer is to be calibrated with a standard cell using the circuit shown in the
diagram below.

The balance point is found to be near L. To improve accuracy the balance point
should be nearer M. This may be achieved by

A replacing the galvanometer with one of lower resistance. .


B replacing the potentiometer wire with one of higher resistance per unit
length.
C reducing the resistance R.
D increasing the resistance R.

28 A flat circular coil of 120 turns, each of area 0.070 m2, is placed with its plane
perpendicular to a uniform magnetic field. The flux density of the field is changed
steadily from 80 mT to 20 mT without a change in direction, over a period of 2.0s.

What is the e.m.f. induced in the coil during this time?

A 0.13 V B 0.25 V C 0.42 V D 0.50 V

VJC Preliminary Exams 2010 Physics H2/P1/9646/1


14

29 When a coil is rotated in a magnetic field, the induced e.m.f. E varies as shown.

Which of the following graphs, drawn to the same scale, would be obtained if the
speed of rotation of the coil were doubled?

VJC Preliminary Exams 2010 Physics H2/P1/9646/1


15

30 A small horse shoe magnet is placed on a smooth horizontal table. A wire carrying a
current is inserted into the space between the poles of a magnet as shown in the
figure. The wire is held fixed by two insulating stands on the ground.

What happens to the magnet?


A It remains stationary
B It accelerates towards the North
C It accelerates towards the East
D It accelerates towards the West

31 A 240 V mains voltage is applied to the primary coil of a step-down transformer. A


resistor is placed across the secondary coil, drawing a current of 2.5 A from the
secondary coil, which has a voltage of 72 V across it:

2.5 A
240 V 72 V

What is the current flowing in the primary coil?

A 0.53 A B 0.75 A C 1.1 A D 8.3 A

32 The voltage V of an a.c. power supply, measured in volts, varies with time t,
measured in seconds, as follows:
V = 3.0 sin 380t
What is the r.m.s. voltage and frequency of the power supply?

r.m.s. voltage / V Frequency / Hz


A 2.1 60
B 2.1 380
C 3.0 60
D 3.0 380

VJC Preliminary Exams 2010 Physics H2/P1/9646/1


16

33 A blue laser beam is incident on a metallic surface, causing electrons to be ejected


from the metal. If the frequency of the laser beam is increased while the intensity of
the beam is held fixed,
A the rate of ejected electrons will decrease and the maximum kinetic energy
will increase.
B the rate of ejected electrons will remain the same but the maximum kinetic
energy will increase.
C the rate of ejected electrons will increase and the maximum kinetic energy will
increase.
D the rate of ejected electrons will remain the same but the maximum kinetic
energy will decrease.

34 Part of the energy level diagram of a certain atom is shown in Fig 34 below. The
energy spacing between levels 1 and 2 is twice that between 2 and 3. If an electron
makes a transition from level 3 to level 2, the radiation of wavelength λ is emitted.

Fig 34

What possible radiation wavelengths might be produced by other transitions between


the three energy levels?
λ
A Only
2
λ λ
B Both and
2 3
C Only 2λ
D Both 2λ and 3λ

35 To decrease the cutoff wavelength of the continuous x-ray spectrum, you should
A increase the electric potential difference used to accelerate the electrons.
B decrease the electric potential difference used to accelerate the electrons.
C allow the electrons to strike a thin foil instead of a thick block of the target
material.
D change the target to an element of higher atomic number.

VJC Preliminary Exams 2010 Physics H2/P1/9646/1


17

36 The conductivity of a semiconductor can be increased by the addition of impurities.


The impurity atoms can be donors or acceptors of electrons. Which of the following
diagrams show the energy level of a donor atom relative to the conduction and
valence bands of the semiconductor?
A C

Conduction Conduction
band band
Energy level of
Energy level of donor atom
donor atom
Valence Valence
band band

B D

Valence Valence
band band
Energy level of
Energy level of donor atom
donor atom
Conduction Conduction
band band

37 The following diagrams show the conduction and valence bands of three different
materials:

Material X Material Y Material Z

Which of the following options correctly identify the three materials?

Material X Material Y Material Z


A Semiconductor Insulator Metal
B Metal Semiconductor Insulator
C Insulator Semiconductor Metal
D Metal Insulator Semiconductor

VJC Preliminary Exams 2010 Physics H2/P1/9646/1


18

38 Which of the following graphs shows how the electric potential varies across the p-n
junction of a semiconductor diode?

A C

Potential Potential
n-type p-type n-type p-type
semiconductor semiconductor semiconductor semiconductor

Distance Distance
Junction Junction

B D

Potential Potential
n-type p-type n-type p-type
semiconductor semiconductor semiconductor semiconductor

Distance Distance
Junction Junction

39 α-particles are deviated by thin metal foils through angles that ranges from 0° to
180°. Which of the following is the explanation?

A Diffraction of α-particle by the crystal lattice.


B Scattering of α-particle by small but heavy regions of positive charge.
C Scattering of α-particle by free electrons.
D Scattering of α-particle by bound electrons.

40 A radioactive source contains the nuclide 187


74W which has a half life of 24 hours. In
-1
the absence of this source, a constant average count-rate of 10 min is recorded.
Immediately after the source is placed in a fixed position near the counter, the
average count-rate rises to 90 min-1.

What average count-rate is expected with the source still in place 12 hours
later?
A 45 min-1 B 50 min-1 C 57 min1 D 67 min-1

VJC Preliminary Exams 2010 Physics H2/P1/9646/1


Name Class
09S

VICTORIA JUNIOR COLLEGE


2010 Preliminary Examination
PHYSICS 9646 / 02
Higher 2

Paper 2

16 September 2010 Thursday 1 hour 45 minutes


Candidates answer on the Question Paper.
No Additional Materials are required.

READ THESE INSTRUCTIONS FIRST

Write your name and class in the spaces provided at the top of this page.
Write in dark blue or black pen.
You may use a soft pencil for any diagrams, graphs or rough working.
Do not use staples, paper clips, highlighters, glue or correction fluid.

Answer all questions.


Write your answers in the spaces provided in this booklet.
The number of marks is given in brackets [ ] at the end of each question or part question.

For Examiner’s Use


1
2
3
4
5
6
7
s.f.
unit
Total /72

This booklet consists of 14 printed pages including the cover page.


2

Data

speed of light in free space, c = 3.00 x 108 m s-1

permeability of free space, μo = 4π x 10-7 H m-1

permittivity of free space, εo = 8.85 x 10-12 Fm-1


(1 / (36 π)) x 10-9 Fm-1

elementary charge, e = 1.60 x 10-19 C

the Planck constant, h = 6.63 x 10-34 J s

unified atomic mass constant, u = 1.66 x 10-27 kg

rest mass of electron, me = 9.11 x 10-31 kg

rest mass of proton, mp = 1.67 x 10-27 kg

molar gas constant, R = 8.31 J K-1 mol-1

the Avogadro constant, NA = 6.02 x 1023 mol-1

the Boltzmann constant, k = 1.38 x 10-23 J K-1

gravitational constant, G = 6.67 x 10-11 N m2 kg-2

acceleration of free fall, g = 9.81 m s-2


3

Formulae

uniformly accelerated motion, s = ut + ½at2

v2 = u2 + 2as

work done on/by a gas, W = pΔV

hydrostatic pressure, p = ρgh

gravitational potential, Gm
φ = −
r

displacement of particle in s.h.m. x = xo sin ωt

velocity of particle in s.h.m. v = vo cos ωt

= ±ω (x o
2
− x2 )
resistors in series, R = R1 + R2 + …

resistors in parallel, 1/R = 1/R1 + 1/R2 + …

electric potential, V = Q / 4πεor

alternating current/voltage, x = xo sin ωt

transmission coefficient, T = exp(-2kd)

8π 2 m (U − E )
where k =
h2

radioactive decay, x = xo exp (-λt)

decay constant 0.693


λ =
t1
2
4

Answer all questions in the spaces provided.

1(a) Define electric potential energy at a point. [1]

(b) The figure below shows the path of a proton ( p ) travelling head-on towards a
1
1
235
uranium U nucleus.
92

1.5x10-14 m

path of proton

uranium nucleus

By considering only the electrostatic repulsion, the distance of closest approach of


the proton to the uranium nucleus is 1.5x10-14 m.

(i) Name the form of the energy to which the kinetic energy of the proton has
been converted when it is at the distance of closest approach. [1]

(ii) Calculate the initial speed of the proton, and state the assumption that you
have made in the calculation. [4]

(iii) In nuclear fission reactions, protons are not normally used to induce the
reactions. Suggest an alternative particle that you would use to penetrate the
nucleus and explain your choice. [2]
5

2(a) State Faraday’s law of electromagnetic induction. [1]

(b) A plane coil of wire containing 50 turns each of diameter 5.0 cm and total resistance
5.0 Ω is placed so that the plane of the coil is normal to a uniform magnetic field B
of flux density 2.0 x 10-3 T.

The coil is moved with a speed of 15.0 m s-1, at right angles to the magnetic field
from P to the positions Q and R shown dotted in the figure below.

40.0 cm

P
B
.

15.0 m s-1
40.0 cm
Fig.2.1

Q
15.0 m s-1 R

(i) Calculate the average induced e.m.f. in the coil as it travels from

1 P to Q;

2 Q to R. [4]
6

2(b) (ii) Calculate the work done on the coil as it travels from

1 P to Q;

2 Q to R. [3]

(c) The coil is now replaced by a semi-circular wire MN with the same diameter of
5.0 cm and moved with a speed of 15.0 m s-1at right angles to the same magnetic
field as shown in Fig.2.2.

M N
B
15.0 m s-1

Fig.2.2

(i) Find the magnitude of the induced e.m.f. generated between M and N. [2]

(ii) State whether M or N is at a higher potential. [1]


7

3(a) A 240 V r.m.s. voltage is applied across the primary coil of a step-down transformer
with a primary:secondary turns ratio of 20:1. Calculate the r.m.s. voltage induced
across the secondary coils. [2]

(b) A 6.0 Ω resistor is placed across the secondary coil. Calculate the maximum power
dissipated by the resistor. [3]

(c) It is desired that the voltage across the resistor be converted into a d.c. voltage.
Complete the circuit in Fig. 3 to show how this could be achieved. Include the
resistor in your circuit. [1]

B
Secondary
coil
Fig. 3

(d) Sketch a graph of the voltage across the resistor in (c) against time. [1]

(e) Suggest a disadvantage of the circuit in (c). [1]

(f) Indicate on Fig. 3 the polarity of A and B when a current is flowing through the
resistor. [1]
8

4(a) An electron has kinetic energy 5.00 eV and its momentum is accurate to within
0.0100%. Find the minimum uncertainty in determining the position of this electron.
[4]

(b) The electron described in (a) is now incident on a potential barrier with thickness
0.200 nm and height 10.0 eV (see Figure 4 below).

Figure 4

(i) Calculate the probability that the electron will tunnel through the barrier.

(ii) Calculate the probability that the electron will be reflected. [3]

(c) Suppose that the barrier height is now reduced. Discuss how the amplitude of the
reflected wave function of the electron changes. [2]
9

5(a) Explain the basic principles behind the production of laser light. [3]

(b) In the traditional setup for the Young’s double slit experiment, a single slit must be
placed between the light source and the double slits in order to produce an
observable fringe pattern. However, if a laser is used as the light source, the single
slit is not needed.

(i) State the property of laser light that makes the single slit unnecessary.

(ii) Explain how laser light acquires the property that makes the single slit
unnecessary. [3]

(c) In a certain laser medium, laser light is produced when electrons de-excite from the
metastable state of energy 3.98 x 10-19 J to the ground state, of energy 0 J. At
steady state, 2.51 x 1015 electrons de-excite per second. The cross-sectional area
of the laser beam is 7.07 x 10-6 m2. Calculate:

(i) the energy of one photon in the laser beam.

(ii) the power of the laser beam.

(iii) the intensity of the laser beam. [3]

(d) It can be seen that the power of the laser beam is rather low; however, the intensity
of the beam is quite high. Explain how this is possible. [1]
10

6 A wire-wound resistor is manufactured by winding resistance wire on an insulating


former. A commonly used material for the wire is an alloy of nickel and chromium
called nichrome. The wire is produced by pulling the nichrome through a suitable
sized hole. Nichrome is sufficiently ductile to be drawn into a wire without danger of
it cracking or breaking after winding. It resists corrosion and has a fairly high
resistivity. The wire itself must be uniform and thin, and is covered with an insulating
material.

A manufacturer of resistors of this type supplies information concerning them in the


form of a family of lines shown in the graph of Fig. 6. Resistors of different
resistance R1, R2, … R5 etc are shown by the separate lines.

(a) By choosing some values of potential difference and current from Fig. 6, complete
the table showing the resistances R1, R2, … R5. [2]

Fig. 6

R1 =

R2 =

R3 = 1000 Ω

R4 =

R5 =

(b) Draw two additional lines on Fig. 6:


(i) one line for a resistance of 2000 Ω,
(ii) one line for a resistance of 47 Ω. [3]
11

6(c) This particular set of resistors is manufactured so that the resistors can safely be
used with power dissipation up to 1 W. Complete the following table to show the
maximum safe current in the resistors for the potential differences given.

Potential difference / V Maximum current / A


1000

100

10

1
[2]

(d) Plot the points in (c) on the graph of Fig. 6. On the graph, indicate the region of safe
use for all these resistors. [4]

(e) The lines on Fig.6 represent ideal behaviour. Suggest, with a reason, how the line
for a real resistor might differ from the ideal. [2]
12

7 Solar power is energy from the sun. The sun is 150 million kilometres away, but it is
over 1 million degrees Celsius at the core. If we could harness it, there is enough
solar power reaching the earth to provide all of our energy needs 10,000 times over.

Since almost the beginning of man, solar power has been used for drying clothes and
food. It was not until 1954 however, that scientists in the United States produced
electricity from the sun - to power satellites in space. They invented photoelectric
cells, which capture the sun's energy and turn it into electricity. A photoelectric cell is
an electric cell made from two layers of different materials that can produce an
electric current when light of certain frequencies shines on the cell.

Design an experiment to find how the power generated by a solar panel depends on
the frequency of the light falling on it.

The equipment available includes the following:

Connecting wires Rule


Voltmeter Micrometer screw gauge
Ammeter Variable power supply unit
Galvanometer Switch
Resistors Stopwatch
Solar panel Colour filters
Light intensity meter Spectrometer
Diffraction grating Lamp

You should draw diagrams to show the arrangement of your apparatus. In your
account you should pay particular attention to

(a) the equipment you would use for the investigation,

(b) the procedure to be followed,

(c) how the frequency of the light used is to be measured,

(d) the control of variables,

(e) any safety precautions,

(f) any precautions that you would take to improve the accuracy of the experiment.
[12]

(Write your answer in the following two blank pages.)


13
14

End of paper
Name: ________________________ Class: 09S ___________

VICTORIA JUNIOR COLLEGE


2010 JC2 PRELIMINARY EXAMINATION

PHYSICS 9646/3
Higher 2
Paper 3

21/9/2010 0800h – 1000h


TUESDAY (2 Hours)

This paper consists of two sections: For marker’s use


Section A
Section A (40 marks) consists of 4 short structured
questions. Write your answers in the spaces provided Q1
for each question.
Q2
Section B (40 marks) consists of 3 long structured Q3
questions. Answer any two questions. Write your
answers in the spaces provided for each of the chosen Q4
questions.

The intended marks for each question or part question in Section B


sections A and B are given in brackets [ ].
Q5
N.B. You will hand in the whole question set issued to Q6
you at the end of the examination. Do not separate the
question set into parts. Q7
s.f.
unit
Total (80)

This question set consists of a total of 16 printed pages.


2

Data

speed of light in free space, c = 3.00 x 108 m s-1

permeability of free space, µo = 4π x 10-7 H m-1

permittivity of free space, εo = 8.85 x 10-12 F m-1


(1/(36π)) x 10-9 F m-1

elementary charge, e = 1.60 x 10-19 C

the Planck constant, h = 6.63 x 10-34 J s

unified atomic mass constant, u = 1.66 x 10-27 kg

rest mass of electron, me = 9.11 10-31 kg

rest mass of proton, mp = 1.67 x 10-27 kg

molar gas constant, R = 8.31 J mol-1 K-1

the Avogadro constant, NA = 6.02 x 1023 mol-1

the Boltzmann constant, k = 1.38 x 10-23 J K-1

gravitational constant, G = 6.67 x 10-11 N m2 kg-2

acceleration of free fall, g = 9.81 m s-2


3

Formulae

uniformly accelerated motion, s = ut + ½ at2


v2 = u2 + 2as

work done on/by a gas, W = pΔV

hydrostatic pressure, p = hρ g

gravitational potential, GM
φ =−
r

displacement of particle in s.h.m., x = xo sin ωt

velocity of particle in s.h.m., v = vo cos ωt


= ±ω ( xo2 − x 2 )

resistors in series, R = R1 + R2 + …

resistors in parallel, 1/R = 1/R1 + 1/R2+ …

electric potential, V = Q/4πεor

alternating current/voltage, x = xo sin ωt

transmission coefficient, T = exp(-2kd)


8π 2 m(U − E )
where k =
h2

radioactive decay, x = xoexp(-λt)

decay constant, 0.693


λ=
t1
2
4

Section A (40 marks)


Answer all questions in the spaces provided.

1 (a) Distinguish between a random error and a systematic error in the


measurement of a physical quantity. [2]

(b) A student set up the apparatus shown in Fig 1 in order to determine the
spring constant k of a spring by finding the extension of the spring when
additional mass is loaded.

metre rule

Fig 1
scale reading

mass

The following readings with their errors were recorded in a particular


experiment:

mass added initial scale reading final scale reading


(20 ± 1) g (32.00 ± 0.05) cm (36.30 ± 0.05) cm

Using the readings above, calculate the spring constant k with its
associated uncertainty and present your answer in SI units of N m-1. [4]
5

(c) A second student repeated the experiment in (b) with the same spring. In
this new experiment, the additional masses were loaded and the
corresponding extension readings were tabulated. A graph showing the
variation of the extension and loaded masses was then plotted. Discuss
three advantages of this procedure for the determination of the spring
constant as compared to that used in (b). [3]

2 (a) The figure below shows a thin taut wire held horizontally by two supports
placed 0.40 m apart.
0.40 m

Fixed Movable

weights
When the wire is plucked at its centre, a standing wave is formed and the
wire vibrates in its fundamental mode of frequency 50 Hz.
(i) Explain why a standing wave is formed between the supports. [2]

(ii) Determine the speed of the wave in the wire. [1]


6

(iii) Sketch the next 2 higher modes which the string can vibrate in and
[3]
hence determine their corresponding frequencies.

(b) The wire is then connected to an a.c. source in a closed circuit and a
magnet is brought near to the wire as shown in the next figure below. This
causes the wire to vibrate in its fundamental mode with a large amplitude.
When the movable support is shifted from its position, the amplitude of
vibration decreases abruptly.

0.40 m B-field due to magnet

Fixed Movable support

a.c. supply weights


(i) Explain the change in amplitude of the wire’s vibration when the
movable support is shifted. Hence, deduce the frequency of the a.c.
source. [3]
7

(ii) Suggest two ways that the same wire can be made to resonate with
a fundamental frequency of 100 Hz.
[2]

3 (a) State the Law of Conservation of Momentum. [1]

(b) Explain what is meant by an elastic collision. [1]

(c) A 200g rubber ball is tied to a 1.0 m long string and released from rest at
angle θ. It swings down and at the very bottom has a elastic collision with
a 1.0 kg block. The block is resting on a frictionless surface and is
connected to a 20 cm long spring of spring constant 2.0 kN m-1. After
collision, the spring compressed a maximum distance of 2.0 cm.

1.0 m θ

1.0 kg spring

200 g

20 cm
(i) Determine the strain energy stored in the spring. [2]
8

(ii) Determine the speed of the block after collision with the ball. [2]

(iii) Given that the collision is elastic, determine the speed of the ball
before collision with the block. [2]

(iv) Hence, determine from what angle was the rubber ball released. [2]

4 (a) Define resistance and the ohm. [2]

(b) A wire with a resistance of 6.0 Ω is stretched so that its new length is three
times its original length. Assuming that the resistivity and density of the
material are not changed during the stretching process, calculate the
resistance of the longer wire. [3]
9

(c) The circuit shown in Fig. 4 is constructed of resistors, each of which has a
maximum safe power rating of 0.40 W.

160Ω

Fig.4

1000Ω 1000Ω

(i) Find the maximum potential difference that can be applied between
X and Y without damage to any of the resistors. [3]

(ii) If this potential difference were exceeded, explain which resistor


would be most likely to fail. [2]
10

Section B [40 marks]


Answer any two questions for this section. Each question carries 20 marks. Write your
answers to each part of the questions in the spaces provided.

5 (a) (i) Define magnetic flux density and state its SI unit. [2]

(ii) Express the unit of magnetic flux density in terms of its base units. [2]

(b) Fig 5 shows the arrangement of a mass spectrometer, which is an


instrument to measure the masses of ions. An ion of mass m and charge
+q is produced at rest in an ion source S. The ion is accelerated through a
pair of parallel plates P1 and P2 of potential difference V. The ion is then
allowed to enter a region of uniform magnetic field of flux density B, which
is directed perpendicularly to the plane of the paper. In the field it moves
in a semicircle, striking a photographic plate at a distance x from the entry
slit.

Region of
magnetic field
directed
vertically to
plane of paper

v x photographic
q+ plate
P2 -V
P1 0
V
S
Fig 5
11

(i) Show that the ion exit from plate P2 and enter the magnetic field with
2qV [1]
a velocity v = .
m

(ii) Describe and explain the effects of the magnetic field on the velocity
of the ion upon its entry into the magnetic field. [2]

(iii) If the ion is positively-charged, state the direction of the magnetic


field. [1]

(iv) For an ion that enters the magnetic field with velocity v, show that
mv
the radius r of its semicircular path is r = . [2]
Bq

(v) Hence find an expression for x in terms of m, B, q and the potential


difference V of the parallel plates. [2]
12

(c) (i) 12
C and C are isotopes of carbon. Ions of these two isotopes
14

were introduced into the mass spectrometer described in (b) and two
spots were observed on the photographic plate. Explain why there
would be two spots. [2]

(ii) The singly and positively charged ions from the two isotopes were
accelerated through a potential difference V of 4.0 kV and enter the
magnetic field of flux density B of 0.50 T. If the masses of the ions
are 12u and 14u respectively, calculate the distance Δx between the
spots on the photographic plate. [3]

(d) If an electron were to be introduced into the mass spectrometer, describe


and explain what would be observed if the accelerating potential and the
magnetic field remained unchanged. [3]
13

6 (a) (i) Define the term binding energy. [1]

(ii) Use the data below to calculate the binding energy in MeV of a
nucleus of 168 O .
Data: mass of proton = 1.007 276 u
mass of neutron = 1.008 665 u
[3]
mass of oxygen nucleus = 15.990 527 u

(iv) The binding energy of 178 O is 126.43 MeV.


State which of these two isotopes of oxygen would be more stable.
[4]
Explain your answer.

(b) Scientists have worked out the age of the Moon by dating rocks brought
back by the Apollo missions. They use the decay of potassium 40 19 K to

argon 40
18 Ar , which is stable. The decay constant of potassium-40 is
5.3 × 10–10 per year.
(i) Write a full nuclear equation for this decay. [2]

(ii) Explain what is meant by the decay constant of potassium-40 being


5.3 × 10–10 per year. [1]
14

(iii) Sketch a labelled graph to show how the activity of 40


K changes with
19

time. Mark on your graph the half life of 40


19 K. [2]

(iv) In one rock sample the scientists found 0.84 μg of argon-40 and
0.10 μg of potassium-40. Calculate the age of the rock sample in
years. [3]

(v) State any assumption that you have made for the calculation in (iv). [1]

(vi) Calculate the activity of the potassium-40 in the rock sample. Hence,
explain if it is necessary for the scientists who handled the rock
sample to take special safety precautions. [3]
15

7 (a) The photoelectric equation may be written in the form


.
Explain the physical meaning of the three terms in this equation. [3]

(b) Light of wavelength 540 nm is incident normally on a metal surface having


a work function of 3.38 eV. The light energy is totally absorbed by the
surface. Calculate the threshold wavelength for this metallic material and
discuss whether the photoelectric emission of electrons occurs. [3]

(c) The light has intensity 1.2 mW m-2 and the area of the metal surface is
1.4 cm2. Calculate
(i) the momentum of a photon of the incident light, [1]

(ii) the rate at which photons are incident on the metal surface, [3]
16

(iii) the force exerted by the light on the surface, assuming that all the
light is absorbed. [2]

(iv) Suppose that the light is now incident on a piece of thin metal foil.
Suggest whether the force calculated in (c)(iii) is a practicable
means of moving the piece of foil. [2]

(d) Although electrons, protons and neutrons are usually treated as particles,
they also possess “wave” characteristics, which can be exploited by
Transmission Microscopes to obtain high-resolution images of extremely
small objects. For instance, electrons with a de Broglie wavelength of
5.0 nm can be used by such microscopes to image the structure of viruses.
(i) Determine the kinetic energy of an electron which has a de Broglie
wavelength of 5.0 nm.

(ii) The resolution of an image can be improved by using particles with


shorter de Broglie wavelengths. Suggest two ways to decrease the de
Broglie wavelength further, and explain your answer. [6]
Answers to VJC Prelim 2010 - H2 Physics Paper 2

1(a) Electric potential energy of a charge at a point is defined as the work done in
bringing the charge from infinity to that point.

(b) (i) Electric potential energy


(ii) I assumed that the uranium nucleus remains at rest.

By conservation of energy,

k.e. of proton = electric potential energy


1 2 qq
mv = 1 2
2 4πε 0 r
1
( )
1.67 × 10 −27 v 2 =
( )[ (
1.60 × 10 −19 92 × 1.60 × 10 −19 )]
2 ( )(
4π 8.85 × 10 −12 1.5 × 10 −14)
−1
v = 4.1× 10 m s
7

Initial speed of proton of proton = 4.1 × 10 7 m s −1

(iii) The alternative particle would be the neutron.


The neutron has no charge, thus it can penetrate the nucleus easily without
being repelled.

2 (a) Faradays’s law of electromagnetic induction states that the magnitude of


induced emf E is proportional to the rate of change of magnetic flux
linkage.

(b) Given that B = 2.0 × 10-3 T

(i) 1. From P to Q, there is no change in flux linkage of the coil,

average induced e.m.f. = 0

2. Time for coil to move out of the magnetic field,


diameter of coil 5.0 × 10−2
t= = = 3.3 × 10− 3 s
v 15.0

1
Δφ NBA − 0
Average induced e.m. f . E = =
Δt Δt
⎡ ⎛ 5.0 ×10 − 2 ⎞ 2 ⎤
( )
50 2.0 ×10 T ⎢π ⎜⎜
-3
⎟⎟ ⎥
⎢⎣ ⎝ 2 ⎠ ⎥⎦
= −3
3.3 ×10
= 0.059 V

(ii)

1 Work done on coil from P to Q, W = (average power dissipated) x time


E2
= (t )
R
= 0 (since induced e.m.f. is zero)

E2
2 Work done on coil from Q to R, W = (t )
R
0.059 2
= (3.3 × 10 −3 )
5.0
2.3 × 10 -6 J

(c)(i) E = Blv
= (2.0 × 10-3)(5.0 × 10-2)(15.0)
= 1.5 × 10-3 V

(ii) N is at a higher potential.

2
Ns
3(a) (Vms)s = (Vrms)p
Np
1
= × 240
20
= 12 V

(V )
peak
2

(b) Maximum power dissipated =


R

=
( 2 Vrms ) 2

=
( 2 × 12 )
2

6.0
= 48 W

(c)
A

(d) V

0 t
T

(e) Half of the power supplied is not transferred to the resistor.

(f)
+
A

B
-

3
p2
4(a) Kinetic energy K of the electron can be expressed as: K =
2m

So, its momentum, p = 2mK = 2(9.11 × 10 −31 )(5.00)(1.60 × 10 −19 )

p = 1.207 × 10 −24 kg m s −1
Uncertainty in p, Δp = 1.207 × 10 −28 kg m s −1
h
According to the uncertainty principle, ΔxΔp ≥

So, the minimum uncertainty in the electron’s position is
h 6.63 × 10 −34
(Δx) min = = = 4.4 × 10 − 7 ≈ 4 × 10−7 m
4πΔp 4π (1.207 × 10 )− 28

(b)(i)
Tunneling probability, T = exp(−2kd)

8π 2 m(U − E ) 8π 2 (9.11 × 10 −31 )(10.0 − 5.0)(1.60 × 10 −19 )


where k = =
h2 (6.63 × 10 − 34 ) 2

k = 1.144 × 1010 m −1
So, T = exp[−2(1.144 × 1010)(0.200× 10−9)] = 0.0103

(b)(ii) Since R + T = 1, the reflection probability is R = 1 − 0.0103 = 0.99

(c) If the barrier height is reduced, the tunneling probability T will increase.
The reflection probability R is reduced. Since R is proportional to the square of the
amplitude of the reflected wavefunction, the amplitude will decrease.

4
5(a)
• Atoms in a laser medium are excited to a metastable state, which is an excited
state with a longer than usual lifetime.
• This continues until the number of excited atoms in the metastable state is more
than that in the lower energy state. Thus population inversion is achieved.
• Eventually, one of the atoms in the metastable state de-excites to the lower energy
level, producing a photon, which triggers other atoms in the metastable state to
de-excite to the same energy level, producing photons which travel in the same
direction and are in the same phase as the triggering photon. This is called
stimulated emission.
• All the photons produced in this way form the laser light.

(b)(i) Laser light is coherent.

(ii) The photons which are produced by stimulated emission have the same phase and
frequency as the triggering photon.

(c)(i) Energy of 1 photon, E = 3.98 x 10-19 J

(ii) Power = (Energy of 1 photon) x (no. of photons per second)


= 3.98 x 10-19 x 2.51 x 1015
= 9.99 x 10-4 W

power
(iii) Intensity =
cross sectional area of beam
9.99 × 10 −4
=
7.07 × 10 −6
= 1.41 x 102 W m-2

(e) This is because the cross sectional area of the beam is very narrow.

1
6(a) R1 =
0.10
= 10 Ω

1
R2 =
0.01
= 100 Ω

10
R4 =
0.001
= 10 000 Ω

5
100
R5 =
0.001
= 100 000 Ω

(b)
47 Ω

2000 Ω

(d) Region of safe use

(d)

(c)
Potential difference / V Maximum current / A
1000 0.001
100 0.01
10 0.1
1 1

(d) The lower half of the graph is the region of safe use, as the values of V x I are
below 1 W.

(e) In real life, as the current increases, the heating effect on the resistor increases.
Thus the resistance will increase, and the graph will curve downwards.

6
7.
Fixed distance
Lamp Solar
panel

Variable Light meter


Colour filter
power
supply

Solar Panel

Resistor
Switch

Procedure:
(a) Set up as shown in the diagram.
(b) The lamp and filter is placed at a fixed distance measured using a ruler from the
solar panel.
(c) Place a light meter next to the solar panel.
(d) With the room lights switched off, switch on the power supply to the lamp and
adjust so that the light meter registers a reading say x. (this should be adjusted to
the same value x for the other filters)
(e) With the circuit connected to the solar panel, record the current I and the p.d. V.
(f) Calculate the power generated by the solar panel, P = IV.
(g) The frequency f of the light through each filter is determined by using the
spectrometer and diffraction grating by finding the angle of diffraction θ and its
wavelength. (or any other method). By taking note of the spacing d of the lines on
the grating and the order n of the diffracted light being observed, f can be
nc
calculated using dsinθ = , where c is the speed of light.
f
(h) Replace the filter with another colour and repeat steps (d) to (f).
(i) Plot power generated P against the frequency f of light used.

7
Control of variables:
1) Intensity of light of different colours (frequencies) is kept fixed by
adjusting the power supply to the lamp and that the light meter registering
reading x throughout.
2) Solar panel is placed perpendicular to axis of lamp so that incident angle
of light does not affect intensity.
3) Ensure that there is no surrounding light (room light), so that all the light
falling on the solar panel is from the lamp.

Accuracy / Safety
(See marking scheme below)

Diagram shows
Setup for lamp to shine light through filter onto the solar panel D1
Circuit connected with ammeter in series and voltmeter in parallel D2
across the solar panel, with resistor. [2]
Basic procedure
Direct light through filters at solar panel P1
Repeat for different coloured filters to change the frequency of light P2 [2]
Measurements
Measure the current and potential difference across the solar panel
Frequency of light through each filter is determined by using the M1
spectrometer and diffraction grating. (Measure angle of diffraction)
Control of variables M2 [2]
Intensity of light on solar panel constant by ensuring that the light meter
registers a constant reading OR adjusting the power supply to the lamp
OR Keeping distance between lamp and solar panel constant
OR Keeping orientation of solar panel constant (perpendicular to axis of
lamp).
Room lights switched off (Dark Room) C1
Switching off lamp between readings to prevent solar panel from C2
heating to maintain constant conductivity.
Any further detail C3 [max 2]
Calculate the power generated by the solar panel, P = IV.
Equation to determine frequency from dsinθ= nc/f A1
Room lights switched off (Dark Room) A2
Light meter placed next to solar panel. A3
Adjust position of lamp to get maximum intensity on solar panel A4
Ensure that all light on solar panel is through the coloured filters A5
Safety (1) concerning electrical power supply to lamp – handle with care A6
if voltage is high.
Safety (2) concerning lamp – handle with care if lamp gets too hot. A7
Safety (3) concerning light intensity – looking directly at lamp over a A8 [max 4]
long period of time.

8
Suggested Answer to VJC Prelim 2010 H2 P3  

Section A 

1(a)  Random errors are errors with different magnitudes and signs in repeated measurements, and 
can be reduced by taking the average of many readings.          
     
Systematic errors are errors of measurements that occur according to some 'fixed rule or 
pattern' such that they yield a consistent over‐estimation or under‐estimation of the true value. 
       
 
  F mg
(b) k= =    
e (x2 − x1 )
 
0.020 x 9.81
    =
  (36.30− 32.00)
    = 4.563 N m −1
 
  Δk Δm Δe
  = +
k m e
  1 0.1
  = + = 0.073    
20 4.3
  Δk = 0.073 x 4.563 = 0.3 N m (to 1 sf)
-1

 
  Hence k = (4.6 ± 0.3) N m‐1     
 
(c)  Advantages of many sets of readings and drawing graph: (state any 3)     
  1.  By drawing best fit line, random errors are reduced. 
  2.  Systematic error can be spotted eg. if the F‐e graph does not pass  
through origin. 
  3.  Can spot erroneous points that are out of the trend. 
  4.  Can check if the proportionality limit is exceeded eg. if the F‐e  
graph turns into a curve instead of a straight line. 
 

2 (a)  (i)  Incident waves travel along the wire to the ends are reflected.   

    The incident and reflected waves travelling in opposite directions have same frequency  
    and amplitude. They superimpose to form standing waves.   

(ii) v = fλ = 50 (0.80)= 40 m s‐1   

  1
         
0.40 m 
  (iii)                 v = fλ  

              40 = f (0.40) 

                 f = 100 Hz     
0.40 m 
                 v = fλ  

              40 = f (2/3 x 0.40) 

                 f = 150 Hz     

     

   (b)  (i)  When the support is shifted, the natural frequency of the wire is changed and no longer  


    matches that of the driving frequency of the periodic force produced by the alternating  
    current in the wire and magnetic field.            

    The system no longer resonates and hence its amplitude decreases.   

    The frequency of the a.c. source must therefore be the natural frequency of the wire  
    which is 50 Hz.                 

  (ii)  The length l can be reduced to 0.20 m to double the fundamental frequency.   

    The weights attached to the wire can be increased to increase the speed of the waves to 
    80 m s‐1.                   

3   (a)  In a system of interacting bodies, the total momentum of the      


    system remains constant provided no external force acts on the system.    
 
  (b)  An elastic collision is a collision in which the kinetic energy of the system    
    is conserved.                          

(c)   (i) Strain energy stored = .
1 2 1
2
( )(
kx = 2.0 × 10 3 2.0 × 10 − 2
2
)
2
= 0.40 J        

        (ii) By Conservation of Energy, 

    KE of block after collision = Strain energy stored in spring 

1 1
∴ mv 2 = 0.40 ⇒ (1.0)v 2 = 0.40
      2 2                                          
−1
v = 0.894 m s

  2
  (iii) By conservation of momentum,  m1u1 + m2 u 2 = m1v1 + m2 v2       
                (0.2)u1 + 0 = (0.2)v1 + (1.0)v2           

         For elastic collision,   u1 − u 2 = v2 − v1  

u1 − 0 = v2 − v1
                                      
⇒ v1 = v2 − u1
     

∴ 0.2u1 = 0.2(v2 − u1 ) + v2
        1.2  
0.4u1 = 1.2v2 ⇒ u1 = (0.894) = 2.68 m s −1
0.4

       

        Speed of ball = 2.68 m s‐1 

    

  (iv) By Conservation of Energy, 

    Gain in KE of ball = Loss in GPE of ball 

1
∴ mv 2 − 0 = mg (l − l cosθ )
2
v2 2.68 2
      ⇒ (1 − cosθ ) = =        
2 gl 2(9.81)(1.0)
θ = 50.7°
     

4  (a)   The resistance of a conductor is the potential difference across the      
V
      conductor per unit  current flowing through it.  ( R = )     
I

The ohm is defined as the resistance of a conductor such that it experiences a voltage 
per unit current of one volt per ampere.    

  3
  (b)   

 
ρl for same density of material, 
Using R = = 6 - - - - - - - - - - - - - (1)
A
m
ρ (3l ) original area, A=
 
R' =
(A 3 ) - - - - - - - - - - - - - (2)
 
ρl
m A
     

R' new area, A =


'
=
(2) =9 ρ (3l ) 3
(1) R
R ' = 9 x 6 .0 = 54 Ω

(c)   
C

 
160Ω 
 
B
 
1000Ω  1000Ω 
 

  Y  A

(i)  If the safe power rating is 0.40 W, 

V2
  Using  P = , the maximum safe voltage for  
R

  4
                                                      
V2
1000 Ω : 0.40 =
1000
V = 20 V
             
V2
160 Ω : 0.40 =
160
V = 8V
1000 V XY
V XY = I ( + 160) , I =
2 500 + 160

V AB = IR AB
⎛ V XY ⎞
if VAB = 20V , 20 = ⎜ ⎟ × 500 ,
⎝ 500 + 160 ⎠
  V XY = 26.4 V  

⎛ V XY ⎞
if VBC = 8V , 8=⎜ ⎟ ×160 ,
⎝ 500 + 160 ⎠
V XY = 33 V

∴ Max safe voltage would be 26.4V

 
Or   By Potential divider principle  
 
1000
V AB = 2 ×V
   
500 + 160
XY

  500
if VAB = 20 V , 20 = × V XY
500 + 160
  VXY = 26.4V
(ii)    
160
if VAB = 8V , 8= × V XY
500 + 160
VXY = 33V

(ii) If this potential difference were exceeded, one of the 1000Ω resistor would most likely 
fail.  This is because when VXY exceeds 26.4 V , the max safe power for the 1000 Ω would 
be exceeded  first.  

  5
 

                     

Section B 

5(a)(i)  Magnetic flux density is defined as the force per unit current per unit length acting on a straight 
conductor that is placed perpendicularly to the field.     
  SI units:  tesla (T)     
 
(ii)    F
B=
  IL
  N
 
Units of B =
Am
  kg m s -2
  = = kg s -2 A -1
Am
 
 
(b)(i)  Gain in KE = Loss in EPE
 
2 mv = qV
1 2

 
2qV
  v= (shown)
  m
 
(ii)  The ion will experience a force that acts perpendicularly to its velocity.     
This force will cause the velocity to change its direction but not its speed.     
Consequently, the ion moves in a circular path. 
 
(iii)  The direction of the B‐field is out of the plane of the paper.       
 
(iv)  Magnetic force on ion provides the centripetal force  
 
mv 2
  Bqv =
r
 
mv
  r= (shown)
Bq

  6
(v)  Since x is the diameter, ie. x =2r, then from (iv) 
     
2mv
  x = 2r =
Bq
 
  From b(i), substitute for v we get 
     
  2m 2qV
x=
  Bq m
  8mV
=
  B2q
 
c(i)  Since 14C is more massive than 12C, then from the equation in b(v) 14C would have a larger 
diameter in path than 12C.  Thus it would hit the photographic plate at two spots. 
 
(ii)  8 x 14 x 1.66 x 10 -27 x 4000
For 14C , x1 =
  0.50 2 x 1.6 x 10 -19
  = 0.136 m [1]
  8 x 12 x 1.66 x 10 -27 x 4000
  For 12C , x 2 =
0.50 2 x 1.6 x 10 -19
 
= 0.126 m
 
 
  Hence, Δx = 0.136 – 0.126 = 0.010 m   
 
(d)  If an electron were to be introduced, it would deflect to the left because electron is negative 
charge.  
  The diameter of its path would also be much smaller since the mass of electron is only 
9.11 x  10‐31 kg.   
 
6  (a)(i)  Binding energy is the energy released in forming the nucleus from its    
    component nucleons.                               
 
(ii) Mass defect, Δm = 8(1.007276) + 8(1.008665) – 15.990527 
                = 0.137001 u 

  ( )(
      Binding energy,  Δmc 2 = 0.137001 × 1.66 × 10 − 27 3.00 × 10 8 ) 
2

= 2.04 × 10 −11 J
                = 127.6 MeV          
= 128 MeV

  7
126.43
(iii) Binding energy per nucleon for  178 O = = 7.43 MeV    
17
127.6
        Binding energy per nucleon for  168 O = = 7.98 MeV    
16

        Since  168 O  has a higher binding energy per nucleon, it would be more          


       stable.                 

40
(b)(i)   19 K →18
40
Ar + 10β                

  (ii) The probability that a particular potassium‐40 nucleus will decay           
within a year is 5.3 x 10‐10.                   

 
A
  (iii) 
Ao

  Ao/2

  0 t1/2 t

                   

  (iv)    M = M 0 e − λt                       
−10
0.1 = (0.1 + 0.84)e − (5.3×10 )t
         
⇒ t = 4.23 ×10 9 years

  (v) All the argon‐40 is formed from the decay of potassium‐40.       

5.3 × 10 −10 ⎛ 0.1×10 −6 ⎞


  (vi) Activity,    A = λN = ⎜⎜ × 6.02 × 10 23 ⎟⎟  
365 × 24 × 60 × 60 ⎝ 40 ⎠

        = 0.025 Bq                    

       Since the activity is quite low, simple safety precautions would suffice.  

  8
7  (a)  hf is the energy of the incident photon             
    Φ is the work function, which is the minimum energy required to cause photoelectric  
    emission                   

1 2
    mv  is the maximum kinetic energy of the emitted electrons       
2

hc
  (b)  Work function,  Φ = , where λth is the threshold wavelength.      
λth
hc (6.63 × 10−34 )(3.00 × 108 )
    So,  λth = = = 3.68 × 10−7 m = 368 nm     
Φ 3.38(1.60 × 10−19 )

    Since 540 nm is above λth, the incident photons do not possess sufficient energy to  
    cause photoelectric emission.               

  (c)  (i)  Momentum of an incident photon,  

h 6.63 × 10−34
            p= = = 1.23 × 10−27 kg m s−1          
λ 540 × 10− 9

    (ii)   Incident power on the metal surface, P = Intensity × Area 

     = (1.2 mW m−2)(1.4 × 10−4 m2) 

               = 1.68 × 10−7 W          
−34
hc (6.63 × 10 )(3.00 × 10 ) 8
  Energy of a single incident photon,  E = =  
λ 540 × 10− 9

     = 3.683 × 10−19 J       

P 1.68 × 10−7
    No. of photons incident per unit time, n =  = −19
= 4.56 × 1011 s−1   
E 3.683 × 10

    (iii)   Magnitude of the change in momentum of each photon as it is absorbed, 

      Δp = | pf − pi | = | 0 − 1.23 × 10−27 | = 1.23 × 10−27 kg m s−1     
⎛ Δp ⎞
    Force exerted on the surface, F =  (nΔt )⎜ ⎟  =  n × Δp  
⎝ Δt ⎠

       = (4.56 × 1011)(1.23 × 10−27) 

               = 5.61 × 10−16 N         

  9
  (iv)  Suppose the piece of foil has a mass of 1 g, the acceleration resulting from F would be 

F 5.61 × 10−16
      a= = = 5.61 × 10−13 m s−2 
m 1 × 10− 3

    This is extremely small, so the effect on the foil would be negligible. Hence, the force  
    would not be a practicable means to move the piece of foil. 

h 6.63 × 10−34
(d)  (i)  Momentum of the electron,  p = = = 1.326 × 10−25 kg m s−1   
λ 5.0 × 10 −9

    Kinetic energy of the electron,  K =
p2
=
(
1.326 × 10− 25 )2

= 9.65 × 10−21 J   
2m 2(9.11 × 10− 31 )

h h
  (ii)  The de Broglie wavelength of a particle may be expressed as:  λ = =  
p 2mK

    If we increase the kinetic energy K of the particle, the above relation tells us that λ  
    would be reduced.                 

    So, the electrons can be accelerated through a larger potential difference to achieve a  
    greater K, so that λ is smaller.               

    If we use a more massive particle (larger m), then λ would also be reduced.   

    So, instead of electrons, we can accelerate more massive particles like protons to  
    achieve a shorter λ.                 

  10
Candidate’s Name ………………………………….……… CTG ……………….…

YISHUN JUNIOR COLLEGE


JC 2 Preliminary Examinations 2010

PHYSICS 9745,9646/1
HIGHER 2
27 August 2010
Paper 1 Multiple Choice Friday
1 hour 15 minutes
Additional Materials:
Optical Mark Sheet

YISHUN JUNIOR COLLEGE YISHUN JUNIOR COLLEGE YISHUN JUNIOR COLLEGE YISHUN JUNIOR COLLEGE YISHUN JUNIOR COLLEGE
YISHUN JUNIOR COLLEGE YISHUN JUNIOR COLLEGE YISHUN JUNIOR COLLEGE YISHUN JUNIOR COLLEGE YISHUN JUNIOR COLLEGE
YISHUN JUNIOR COLLEGE YISHUN JUNIOR COLLEGE YISHUN JUNIOR COLLEGE YISHUN JUNIOR COLLEGE YISHUN JUNIOR COLLEGE
YISHUN JUNIOR COLLEGE YISHUN JUNIOR COLLEGE YISHUN JUNIOR COLLEGE YISHUN JUNIOR COLLEGE YISHUN JUNIOR COLLEGE
YISHUN JUNIOR COLLEGE YISHUN JUNIOR COLLEGE YISHUN JUNIOR COLLEGE YISHUN JUNIOR COLLEGE YISHUN JUNIOR COLLEGE
YISHUN JUNIOR COLLEGE YISHUN JUNIOR COLLEGE YISHUN JUNIOR COLLEGE YISHUN JUNIOR COLLEGE YISHUN JUNIOR COLLEGE
YISHUN JUNIOR COLLEGE YISHUN JUNIOR COLLEGE YISHUN JUNIOR COLLEGE YISHUN JUNIOR COLLEGE YISHUN JUNIOR COLLEGE
YISHUN JUNIOR COLLEGE YISHUN JUNIOR COLLEGE YISHUN JUNIOR COLLEGE YISHUN JUNIOR COLLEGE YISHUN JUNIOR COLLEGE
YISHUN JUNIOR COLLEGE YISHUN JUNIOR COLLEGE YISHUN JUNIOR COLLEGE YISHUN JUNIOR COLLEGE YISHUN JUNIOR COLLEGE
YISHUN JUNIOR COLLEGE YISHUN JUNIOR COLLEGE YISHUN JUNIOR COLLEGE YISHUN JUNIOR COLLEGE YISHUN JUNIOR COLLEGE
YISHUN JUNIOR COLLEGE YISHUN JUNIOR COLLEGE YISHUN JUNIOR COLLEGE YISHUN JUNIOR COLLEGE YISHUN JUNIOR COLLEGE
YISHUN JUNIOR COLLEGE YISHUN JUNIOR COLLEGE YISHUN JUNIOR COLLEGE YISHUN JUNIOR COLLEGE YISHUN JUNIOR COLLEGE
YISHUN JUNIOR COLLEGE YISHUN JUNIOR COLLEGE YISHUN JUNIOR COLLEGE YISHUN JUNIOR COLLEGE YISHUN JUNIOR COLLEGE
YISHUN JUNIOR COLLEGE YISHUN JUNIOR COLLEGE YISHUN JUNIOR COLLEGE YISHUN JUNIOR COLLEGE YISHUN JUNIOR COLLEGE
YISHUN JUNIOR COLLEGE YISHUN JUNIOR COLLEGE YISHUN JUNIOR COLLEGE YISHUN JUNIOR COLLEGE YISHUN JUNIOR COLLEGE

INSTRUCTIONS TO CANDIDATES

Do not open this booklet until you are told to do so.

Write your name and CTG on the Optical Mark Sheet in the spaces provided.

Shade your CTG and Register Number in the space provided.

There are forty questions in this paper. Answer all questions. For each question there are four
possible answers A, B, C and D.
Choose the one you consider correct and record your choice in soft pencil on the separate
Optical Mark Sheet.

Read the instructions on the Optical Mark Sheet carefully.

INFORMATION FOR CANDIDATES

Each correct answer will score one mark. A mark will not be deducted for a wrong answer.
Any rough working should be done in this booklet.

This question paper consists of 20 printed pages.


2

Data

speed of light in free space, c = 3.00 × 108 m s–1


permeability of free space, μo = 4π × 10–7 H m–1
permittivity of free space, εo = 8.85 × 10–12 F m–1
(1/(36π)) × 10–9 F m–1
elementary charge, e = 1.60 × 10–19 C
the Planck constant, h = 6.63 × 10–34 J s
unified atomic mass constant, u = 1.66 × 10–27 kg
rest mass of electron, me = 9.11 × 10–31 kg
rest mass of proton, mp = 1.67 × 10–27 kg
molar gas constant, R = 8.31 J K–1 mol–1
the Avogadro constant, NA = 6.02 × 1023 mol–1
the Boltzmann constant, k = 1.38 × 10–23 J K–1
gravitational constant, G = 6.67 × 10–11 N m2 kg–2
acceleration of free fall, g = 9.81 m s–2

Formulae

1 2
uniformly accelerated motion, s = ut + at
2
v2 = u2 + 2as
work done on/by a gas, W = pΔV
hydrostatic pressure, p = ρgh
gravitational potential, Gm
φ = −
r
Displacement of particle in s.h.m. x = xo sin ω t
velocity of particle in s.h.m., v = vo cos ω t
=
±ω ( x o2 − x 2 )
resistors in series, R = R1 + R2+……….
resistors in parallel, 1 1 1
= + + ........
R R1 R 2
electric potential, Q
V = 4πε o r
alternating current/voltage, x = xo sin ω t
transmission coefficient T = exp(−2kd)
8π 2 m(U − E )
where k =
h2
radioactive decay, x = xo exp(–λt)

0.693
decay constant, λ = t1
2

9745,9646/1/JC2Prelims/YJC2010
3

1 What are the base units of specific latent heat of fusion?

A m2 s-2
B kg-1 m2 s2
C m2 s2 K-1
D kg-1 m2 s2 K-1

2 The residents of a certain town would like the passenger train service that
passes through the town to make an additional stop at their station.

Graph A (in solid line) shows the time variation of the speed of a train that stops
at the station whereas graph B (in dotted line) is for one that does not stop.

Speed / m s−1

Graph B
50.0

Graph A

train leaves

train just arrives

Time / s
0
70.0 190.0 320.0

The total delay in making this additional stop is

A 320.0 s
B 220.0 s
C 190.0 s
D 120.0 s

9745,9646/1/JC2Prelims/YJC2010
4

3 The time variation of the acceleration of an object is as shown in the graph


below.

At which point is the velocity the greatest?

acceleration

C
B
time
D

4 The figure below shows the path of a ball leaving the ground at point X and about
to hit the ground at point Z.

initial
velocity Y

θ
X Z
ground

If air resistance is negligible, the vertical component of

A velocity is maximum at Y
B displacement is minimum at Y
C velocity is the same at X and Z
D acceleration is the same at Y as at Z

9745,9646/1/JC2Prelims/YJC2010
5

5 A paratrooper jumps down from an aircraft and falls without significant drag force
for 3 s before opening his parachute.
Which of the following best shows the time t variation of his vertical acceleration
a?

a a

t/s t/s
3 3

A B

a a

t/s t/s
3 3

C D

9745,9646/1/JC2Prelims/YJC2010
6

6 A trolley X of total mass 0.50 kg and speed 6.0 m s−1 approaches, head-on,
another trolley Y of total mass 0.60 kg and speed 5.0 m s−1 moving in the same
direction along a level track. The collision is elastic with each trolley having a
magnet with the like poles facing each other as shown below.

6.0 m s−1 5.0 m s−1

X N S S N Y

At some instant during the collision, X is at rest. The speed of Y then is

A 3.0 m s−1
B 5.0 m s−1
C 6.0 m s−1
D 10 m s−1

7 A uniform horizontal beam is acted upon by an upward push force, as shown


below, whose magnitude is equal to its weight.

X Y

wall weight wall


push

The ends of the beam fit into sockets X and Y of two rigid vertical walls which
keep the beam stationary.

Which of the following correctly states the directions of the forces acting on the
beam at X and Y?

Socket X Socket Y
A downwards upwards
B upwards upwards
C downwards downwards
D upwards downwards

9745,9646/1/JC2Prelims/YJC2010
7

8 Two rigid beams, 1 and 2, are fixed to a vertical wall. A stationary load of weight
W is hung from point X where the two beams are joined as shown.

X
Beam 1

Beam 2

W
Which diagram shows the forces acting at X?

Force in Beam 1 Force in Beam 1

W W
Force in Beam 2 Force in Beam 2

A B

Force in Beam 1 Force in Beam 1

W W
Force in Beam 2 Force in Beam 2

C D

9745,9646/1/JC2Prelims/YJC2010
8

9 The base area of a barge is 80 m2 and the sides of the barge are vertical. The
depth h to which it rests in fresh water of density 1.0 × 103 kg m−3 is as shown on
the left figure.

barge

h
Δh

80 m2
Before After

When further loaded, as shown on the right figure, with 5.0 × 103 kg of cargo, the
extra depth Δh to which the barge will rest is

A 0.063 m
B 0.0064 m
C 5.0 m
D 610 m

10 A car of mass 1000 kg, traveling at 20 m s−1 on a level road comes to rest in a
distance of 25 m. What is the work done against friction?

A 0.50 kJ
B 8.0 kJ
C 200 kJ
D 250 kJ

9745,9646/1/JC2Prelims/YJC2010
9

11 A ball of mass m and speed v, rolls up a slope from P to Q, a horizontal distance


d away as shown. The distance PQ measured vertically is r and along the slope
is s .
What is the work done against the weight during the journey? Q

A mgs
r
B mgr s
C mgd
P
D mg d 2 + r 2
d

12 A car moves around a horizontal circular track at a constant angular speed ω.

ω
Car

Top view

Which of the following statements is true about the car’s motion?

A Its linear velocity is constant


B Its linear velocity is changing
C Its linear acceleration is zero
D Its linear acceleration is constant

9745,9646/1/JC2Prelims/YJC2010
10

13 A race car moves at a high speed along a banked circular track, as shown below.

Centre of Race car


circular motion
Side view

Which of the followings best represents the free-body diagram of the race car?

A B

C D

14 Planet X of mass M and planet Y of mass 3M revolve around a common centre.


The two planets are constantly separated by a distance of D.

Where is this common centre of their rotation?

A D from planet X
3
B D from planet Y
3
C D from planet X
4
D D from planet Y
4

9745,9646/1/JC2Prelims/YJC2010
11

15 A spring oscillates with a period of 1.0 s at a place where the acceleration of free
fall is g. If it were to oscillate in another place where the acceleration of free fall is
2g, what would be the value of its new period?

A 0.5 s
B 0.7 s
C 1.0 s
D 2.0 s

16 A particle P performs simple harmonic motion between points X and Y which are
4.0 cm apart . The time taken to move from X to Y is 0.8 s. What is the maximum
speed of particle P?

A 5.0 x 10−2 m s−1


B 8.0 x 10−2 m s−1
C 16 x 10−2 m s−1
D 34 x 10−2 m s−1

17 Which one of the following is essential for the equation pV = nRT to be obeyed
by a real gas?

A Changes should be at constant temperature


B Changes should take place at constant volume
C Pressures should be low
D Temperatures must be higher than 273.15 K

18 Two coherent waves of intensities I and amplitude Y, meet in phase at a point.


Calculate, in terms of I and Y, the intensity and amplitude of the resultant wave at
that point.

Intensity Amplitude
A 0 0
B I Y
C 2I 2Y
D 4I 2Y

9745,9646/1/JC2Prelims/YJC2010
12

19 Coherent monochromatic light illuminates two narrow parallel slits and the
interference pattern which results is observed on a screen some distance beyond
the slits. Which of the following will not affect the fringe separation of the
interference pattern?

A Using monochromatic light of higher frequency


B Increasing the distance between the screen and the slits
C Increasing the width of each slit
D Increasing the distance between the slits

20 A thin copper rod is clamped at one end and made to vibrate by a driving force of
variable frequency applied to the free end. At specific frequencies, it is found that
the rod resonates. Which of the following diagram is incorrect, where N and A
represent a nodal and antinodal position respectively?

clamp

A N A

clamp

B N A N

clamp

C N A N A

clamp

D N A N A N A

9745,9646/1/JC2Prelims/YJC2010
13

21 Which of the following statements about an electric field is incorrect?

A The electric field strength due to a point charge proportionately with 1/r2
where r is the distance from the charge.

B The electric field strength at a point is a measure of the force exerted on a


unit positive charge at that point.
dV
C The negative sign in the formula E = − , implies that the field points
dr
in the direction of decreasing potential.

D The electric field strength at a point is equal to the potential gradient at


that point.

22 A point charge is placed at a point Y in front of an earthed metal sheet X. P and


Q are two points between X and Y as shown in the diagram. Which of the
following mathematical relations between the electric field strengths E at different
points is correct?

P Q Y

point charge

A EP = EQ
B EP = 0
C EP > EQ
D EP < EQ

23 A 10 Ω resistor is connected across a cell of e.m.f 6.0 V and internal resistance


2.0 Ω. How much heat energy is dissipated by the resistor when 2 C of charge
flows through it?

A 2.0 J
B 10 J
C 12 J
D 40 J

9745,9646/1/JC2Prelims/YJC2010
14

24 Five identical resistors of resistance 2.0 Ω are connected as shown below.

R
R
P Q
R
R
R

If the resistor R in the middle is removed from the circuit, what is the change in
the effective resistance across PQ?

A Increase by 0.1 Ω
B Increase by 0.4 Ω
C Decrease by 0.4 Ω
D Decrease by 2.0 Ω

25 In the circuit below, four resistors are connected in series with a 12 V battery.
One point along the circuit is earthed as indicated.
12 V

Point Z

2.5 Ω 3.0 Ω 1.0 Ω 2.5 Ω

What is the potential at point Z?

A − 5.3 V
B − 8.6 V
C + 3.3 V
D + 5.3 V

9745,9646/1/JC2Prelims/YJC2010
15

26 Four identical bulbs labeled 1 to 4 are connected with a cell E in the circuit shown
below.

2
1 4

3
Which of the followings will increase the brightness of bulb 1?

A Replace the cell E with another cell of lower e.m.f


B Remove bulb 2
C Connect another bulb in parallel with bulb 2
D Connect another bulb in series with bulb 4

27 A thermistor is connected in series with a fixed resistor of resistance R and a cell


of e.m.f 10 V, as shown in the diagram below.

R
10 V

VT

When the temperature of thermistor is 20°C, its resistance is 5.3 Ω and the
potential difference VT across it is 4.5 V.

What is the value of VT if the temperature of thermistor increases to 60°C and the
resistance drops to 3.1 Ω?

A 1.5 V
B 2.6 V
C 3.2 V
D 3.5 V

9745,9646/1/JC2Prelims/YJC2010
16

28 Two identical magnets A and B are dropped from rest through the middle of a
conducting ring. The ring for B has a small break in it as shown. Which of the
following statements is correct?

N N

Magnet A Magnet B
S S

A Magnet A has a smaller acceleration.


B Magnet B has a smaller acceleration.
C Both magnets fall through with the same acceleration
D Both magnets fall through at constant velocity.

29 A rectangular coil with 200 turns has dimensions 5.0 cm by 4.0 cm. When the coil
is placed in a magnetic field B of 0.35 T and the current, I in the coil is 1.6 A,
what is the value and direction of the maximum torque?

4.0 cm

B = 0.35 T
I = 1.6 A
5.0 cm

A 0.22 N m with the left edge moving into plane of paper


B 5.6 N m with right edge moving into plane of paper
C 0.22 N m with right edge moving into plane of paper
D 11.2 N m with left edge moving into plane of paper

9745,9646/1/JC2Prelims/YJC2010
17

30 The figure below shows the circuit diagram for a half-wave rectifier. The supply to
the rectifier is rated as 50 Hz , 6.0 V r.m.s. What is the average power dissipated in
the load resistor, R = 2.0 Ω.

50 Hz R
6.0 Vr.m.s

A 9.0 W
B 18 W
C 25 W
D 36 W

31 Transitions between the lowest three energy levels in a particular atom give rise
to three spectral lines of wavelength, in order of increasing magnitude, λ1, λ2 and
λ3. Which of the following correctly relates λ1, λ2 and λ3?

A λ1 + λ 2 = λ3
B λ1 = λ 2 + λ3
1 1 1
C = +
λ1 λ2 λ3
1 1 1
D + =
λ1 λ2 λ3

32 White light from a hot source is passed through sodium vapour and viewed
through a diffraction grating.

Which of the following best describes the spectrum seen?

A Dark lines on a white background


B Coloured lines on a white background
C Coloured lines on a dark background
D Dark lines on a coloured background

9745,9646/1/JC2Prelims/YJC2010
18

33 A beam of electrons are accelerated to hit a metal target and the corresponding
X-ray spectrum is as shown below.

Intensity

I3

I2

0 Wavelength
λ1 λ2 λ3
Which of the following statements is true of the features of the spectrum?

A Increasing the accelerating voltage increases λ1


B Changing the type of metal changes λ1
C Both I2 and λ2 decrease when the electron beam is made less intense
D I3 includes the number of X-ray photons of wavelength λ3 emitted by the
‘braking radiation’ process.

34 The uncertainty in the de Broglie wavelength of an electron was found to be 1 ×


10−10 m. Its uncertainty in position is thus

A 1 × 10−9 m
B 1 × 10−10 m
C 8 × 10−12 m
D 5 × 10−45 m

35 In terms of band theory, which of the following is one of the main features which
explains the electrical properties of intrinsic semiconductors?

A More valence electrons


B Smaller energy band gap
C More holes
D More energy levels

36 Which of the following terms is not applicable to photons produced by stimulated


emission?

A Coherent
B Parallel
C In Phase
D Polarised

9745,9646/1/JC2Prelims/YJC2010
19

37 Which of the following is true about extrinsic semiconductors?

A Extrinsic semiconductors can only be made by doping silicon with either


arsenic or boron.
B p-type extrinsic semiconductors are positively charged.
C Extrinsic semiconductors have higher conductivity compared to intrinsic
semiconductors.
D Extrinsic semiconductors have larger energy band gaps compared to
intrinsic semiconductors.

38 For the same given p-n junction, which circuit below gives the widest depletion
region?

p n

p n

p n

p n

9745,9646/1/JC2Prelims/YJC2010
20

39 A Thorium-234 nucleus decays and undergoes two beta-particle emissions, one


alpha-particle emission and one unknown emission to form the Radium-226
nucleus.

What could be the unknown emission?

A Alpha-particle emission
B Beta-particle emission
C Gamma emission
D Neutron emission

238
40 A stationary uranium nucleus, 92 U undergoes radioactive decay with emission
4
of a helium nucleus, He of kinetic energy E. What is the kinetic energy of the
2
daughter nucleus?

4
A E
234

4
B E
238

C E

238
D E
4

End Of Paper

9745,9646/1/JC2Prelims/YJC2010
21

Answers to Paper 1 of Preliminary H2 Physics Exam 2010

1 A 11 B 21 D 31 C
2 B 12 B 22 D 32 D
3 B 13 B 23 B 33 D
4 D 14 D 24 A 34 C
5 A 15 B 25 A 35 B
6 D 16 B 26 C 36 D
7 A 17 C 27 C 37 C
8 C 18 D 28 A 38 D
9 A 19 C 29 C 39 A
10 C 20 B 30 A 40 A

9745,9646/1/JC2Prelims/YJC2010
Candidate’s Name ……………………………… CTG ……….…

YISHUN JUNIOR COLLEGE


JC 2 PRELIMINARY EXAMINATION 2010

PHYSICS 9646/2
HIGHER 2
19 August 2010
Paper 2 Thursday
1 hour 45 minutes
YISHUN JUNIOR COLLEGE YISHUN JUNIOR COLLEGE YISHUN JUNIOR COLLEGE YISHUN JUNIOR COLLEGE YISHUN JUNIOR COLLEGE
YISHUN JUNIOR COLLEGE YISHUN JUNIOR COLLEGE YISHUN JUNIOR COLLEGE YISHUN JUNIOR COLLEGE YISHUN JUNIOR COLLEGE
YISHUN JUNIOR COLLEGE YISHUN JUNIOR COLLEGE YISHUN JUNIOR COLLEGE YISHUN JUNIOR COLLEGE YISHUN JUNIOR COLLEGE
YISHUN JUNIOR COLLEGE YISHUN JUNIOR COLLEGE YISHUN JUNIOR COLLEGE YISHUN JUNIOR COLLEGE YISHUN JUNIOR COLLEGE
YISHUN JUNIOR COLLEGE YISHUN JUNIOR COLLEGE YISHUN JUNIOR COLLEGE YISHUN JUNIOR COLLEGE YISHUN JUNIOR COLLEGE
YISHUN JUNIOR COLLEGE YISHUN JUNIOR COLLEGE YISHUN JUNIOR COLLEGE YISHUN JUNIOR COLLEGE YISHUN JUNIOR COLLEGE
YISHUN JUNIOR COLLEGE YISHUN JUNIOR COLLEGE YISHUN JUNIOR COLLEGE YISHUN JUNIOR COLLEGE YISHUN JUNIOR COLLEGE
YISHUN JUNIOR COLLEGE YISHUN JUNIOR COLLEGE YISHUN JUNIOR COLLEGE YISHUN JUNIOR COLLEGE YISHUN JUNIOR COLLEGE
YISHUN JUNIOR COLLEGE YISHUN JUNIOR COLLEGE YISHUN JUNIOR COLLEGE YISHUN JUNIOR COLLEGE YISHUN JUNIOR COLLEGE
YISHUN JUNIOR COLLEGE YISHUN JUNIOR COLLEGE YISHUN JUNIOR COLLEGE YISHUN JUNIOR COLLEGE YISHUN JUNIOR COLLEGE
YISHUN JUNIOR COLLEGE YISHUN JUNIOR COLLEGE YISHUN JUNIOR COLLEGE YISHUN JUNIOR COLLEGE YISHUN JUNIOR COLLEGE
YISHUN JUNIOR COLLEGE YISHUN JUNIOR COLLEGE YISHUN JUNIOR COLLEGE YISHUN JUNIOR COLLEGE YISHUN JUNIOR COLLEGE

For Examiner’s Use


INSTRUCTIONS TO CANDIDATES
Paper 2
Write your name and CTG in the spaces at the
top of this page. 1 /7
Write your answers in the spaces provided on 2
the question paper. /7
You must use a soft pencil for any diagrams, 3 /7
graphs or rough working. 4
Do not use staples, paper clips, highlighters, glue /7
or correction fluid. 5 /9
6 /5
Section A 7 /18
Answer all questions.
8 /12
It is recommended that you spend about 1 hour
15 minutes on this section. Total
/72
Section B
Answer Question 8.
It is recommended that you spend about 30 minutes on this section.

The number of marks is given in brackets [ ] at the end of each question or


part question.

This question paper consists of 17 printed pages


2
Data
speed of light in free space, c = 3.00 × 108 m s-1
permeability of free space, μo = 4π × 10-7 H m-1
permittivity of free space, εo = 8.85 × 10-12 F m-1
= (1/(36π)) × 10−9 F m-1
elementary charge, e = 1.60 × 10-19 C
the Planck constant, h = 6.63 × 10-34 J s
unified atomic mass constant, u = 1.66 × 10-27 kg
rest mass of electron, me = 9.11 × 10-31 kg
rest mass of proton, mp = 1.67 × 10-27 kg
molar gas constant, R = 8.31 J K-1 mol-1
the Avogadro constant, NA = 6.02 × 1023 mol-1
the Boltzmann constant, k = 1.38 × 10-23 J K-1
gravitational constant, G = 6.67 × 10-11 N m2 kg-2
Acceleration of free fall g = 9.81 m s-2

Formulae
uniformly accelerated motion, s = ut + ½at2
v2 = u2 + 2as
work done on/by a gas, W = pΔV
hydrostatic pressure, p = ρgh
Gm
gravitational potential, φ = −
r
Displacement of particle in s.h.m. x = xo sin ω t
velocity of particle in s.h.m., v = vo cos ω t
= ±ω ( x o2 − x 2 )
resistors in series, R = R1 + R2+……….
1 1 1
Resistors in parallel, R
= R + R + ........
1 2

Q
electric potential, V =
4πε o r
alternating current/voltage, x = xo sin ω t
8π 2 m(U − E)
transmission coefficient T = exp(−2kd), where k = 2
h
radioactive decay, x = xo exp(-λ t)
0.693
decay constant, λ = t1 / 2
9646(New Syllabus)/2/JC2Prelims/YJC2010
3
Section A
Answer all questions.
It is recommended that you spend about 1 hour 15 minutes on this section.

1 (a) State Newton’s law of gravitation. [2]

..…………………………………………………………………………………

..…………………………………………………………………………………

..…………………………………………………………………………………

(b) A source reported that Singapore plans to launch a satellite that will
orbit around the Earth at 2.5 × 103 m above its surface in the year
2020. Take the radius of Earth to be 6.38 × 106 m and mass of
Earth to be 5.97 × 1024 kg.

(i) Calculate the linear velocity of the satellite when in orbit. [2]

Linear velocity = …………….. m s−1

(ii) Deduce whether the satellite is geostationary. [2]

(iii) If the satellite were to orbit above the equator, state the
direction of launch, in order to minimize energy required. [1]

………………………………………………………………………………

9646(New Syllabus)/2/JC2Prelims/YJC2010
4

2 Fig. 2.1 shows a potentiometer circuit that can be used to determine the
unknown e.m.f. of a test cell. The driver cell has an e.m.f. of 12 V and
internal resistance of 1.5 Ω. The resistance of the rheostat can vary
between 0.0 Ω and 5.0 Ω and the resistance wire has a length of 1.2 m.

12 V, 1.5 Ω 0.0 – 5.0 Ω

1.2 m long
resistance wire

Fig. 2.1

Test cell

(a) When the resistance of rheostat is 2.3 Ω, the balance length is 0.57 m.
When the resistance of rheostat is changed to 3.5 Ω, the balance length
becomes 0.68 m. Calculate the e.m.f. of the test cell and the resistance of
the 1.2 m long resistance wire. [4]

Emf of test cell = ……………….. V

Resistance of 1.2 m long resistance wire = ……………….. Ω

(b) State what will happen to the balance length if the internal resistance of
the test cell is doubled. [1]

………………………………………………………………………………………

9646(New Syllabus)/2/JC2Prelims/YJC2010
5

(c) Explain why the resistance of the rheostat cannot be higher than a
particular value, if the potentiometer is to be able to determine the
unknown e.m.f. [2]

………………………………………………………………………………………

………………………………………………………………………………………

3 A narrow beam of electrons at a speed of 3.2 × 107 m s─1 travels along a


circular path in a uniform magnetic field of flux density, B, as shown in Fig.
3.1 below.

incident beam of
electrons

Fig. 3.1

(a) (i) Explain why the electrons undergo uniform circular motion. [3]

..………………………………………………………………………………...…

..…………………………………………………………………………………...

..………………………………………………………………………………...…

..…………………………………………………………………………………

9646(New Syllabus)/2/JC2Prelims/YJC2010
6

(ii) Show that the speed, v, of the electrons in the field is given by

Ber
v=
m

where r is the radius of the circular path of the beam in the field.

[2]

(iii) The radius of the circular path of the beam in the field was found to be 25
mm. Determine the magnetic flux density of the field.

Flux density = ….................... T [2]

9646(New Syllabus)/2/JC2Prelims/YJC2010
7
4 A circular coil of diameter 140 mm has 850 turns. It is oriented so that its
plane is perpendicular to a horizontal magnetic field of uniform flux density
45 mT, as shown in Fig. 4.1.
vertical axis

uniform
magnetic
field coil of 850 turns

Fig. 4.1

(a) Calculate the magnetic flux passing through the coil in this position.

Magnetic flux = ….................... Wb [2]

(b) The coil is rotated through 90° about the vertical axis shown in a time of
120 ms.

(i) Calculate
1. the change of magnetic flux linkage produced by this rotation, and

change of magnetic flux linkage produced = ….................... Wb [2]


9646(New Syllabus)/2/JC2Prelims/YJC2010
8

2. the average e.m.f induced in the coil during this rotation.

Average e.m.f. induced = ….................... V [1]

(ii) State and explain what will happen to the value of the average e.m.f
induced if the coil is rotated through 360°.

..…………………………………………………………………………………

..…………………………………………………………………………………

..…………………………………………………………………………………

…....………………………………………………..……………………..………[2]

5 (a) A heating device is designed to operate on either an a.c. or d.c. power supply.
The device has a resistance of 6.0 Ω. Calculate the average power dissipated
in the device when operating at

(i) an a.c. supply of voltage 12.0 V, 50 Hz

average power dissipated = …………. W [2]

9646(New Syllabus)/2/JC2Prelims/YJC2010
9

(ii) a d.c. supply of voltage 12.0 V

average power dissipated = …………. W [1]

(b) Draw the time t variation of the power P dissipated in the device for both the
a.c. and d.c. supply on the same axes below. Mark values on both axes.

P/W

t/s

[4]

(c) The alternating supply of voltage 12.0 V, 50 Hz is derived from the mains
supply of voltage 230 V, 50 Hz using a transformer, assumed to have 100%
efficiency.

Calculate the primary r.m.s. current when the heating device is in use.

primary r.m.s. current = …………….. A [2]

9646(New Syllabus)/2/JC2Prelims/YJC2010
10

6 In a three-level ruby laser, light of wavelength 550 nm from a flash lamp is


used to excite the atoms in the ruby from ground state E1 to state E3. In
subsequent de-excitations, laser light is emitted. The energy levels are as
shown in Fig. 6.1.

E3

E2*

E1 Fig. 6.1

(a) Draw, on Fig. 6.1, the transition that produces the laser light. [1]

(b) (i) Explain why a metastable state is required for population inversion.

..…………………………………………………………………………………

..…………………………………………………………………………………
[1]

(ii) Explain why population inversion is necessary for lasing to work.

..…………………………………………………………………………………

..…………………………………………………………………………………
[1]

(c) Explain the function of the reflective surfaces in the laser.

..…………………………………………………………………………………

..…………………………………………………………………………………

..…………………………………………………………………………………

[2]

9646(New Syllabus)/2/JC2Prelims/YJC2010
11

7 Jupiter has many moons with different orbital period T (in days) and
average orbital radius r (in × 109 m). Data for six of them are shown in the
table of Fig. 7.1 below.

Moon T / days r / × 109 m


Sinope 758 23.7
Leda 239 11.1
Callisto 16.7 1.88
Europa 3.55 0.671
Io 1.77 0.422
Metis 0.295 0.128

Fig. 7.1

(a) Suggest why the values of r are averages. [1]

………………………………………………………………………………………

………………………………………………………………………………………

(b) It is expected that the moons obey the relation

T rn = k

where n and k are constants.

Explain how the relation may be tested by plotting a graph of lg T against


lg r. [3]

………………………………………………………………………………………

………………………………………………………………………………………

………………………………………………………………………………………

9646(New Syllabus)/2/JC2Prelims/YJC2010
12
(c) Some data from Fig. 7.1 are used to plot the graph of Fig. 7.2.

lg (T / days)

4.0

3.5

3.0

2.5

2.0

1.5

1.0

0.5

0.0 lg (r / m)
8.0 8.5 9.0 9.5 10.0 10.5 11.0

−0.5

−1.0

Fig. 7.2

(i) On Fig. 7.2,

1. Plot the point corresponding to Callisto. Label it C. [2]

2. Draw the line of best fit for the six points. [1]

9646(New Syllabus)/2/JC2Prelims/YJC2010
13
(ii) From (i), determine the magnitudes of the constants n and k. [4]

n = ……………..

k = ……………..

(iii) Explain whether the answer to (ii) is in agreement with Kepler’s


Third Law. [2]

………………………………………………………………………………………

………………………………………………………………………………………

(iv)Another moon, Thermisto has a period of 130 days. Use Fig. 7.2 to
estimate the orbital radius of Thermisto. [2]

radius = ………………….. m

(v) Earth’s moon has an orbital radius of 0.384 × 109 m.

1. Plot its corresponding point. Label it E. [2]

2. Suggest why point E deviates from the line of best fit. [1]

………………………………………………………………………………………

………………………………………………………………………………………
9646(New Syllabus)/2/JC2Prelims/YJC2010
14
Section B
It is recommended that you spend about 30 minutes on this section.

8 Students are shown a demonstration illustrating some principles of


electromagnetic induction.

A coil is wrapped around the full length of a vertical Perspex tube through
which a soft iron rod is inserted. An aluminium ring is placed over the
upper end of the rod. When released from rest, the ring falls freely down
the gap between the rod and the Perspex tube. The time taken for the
aluminium ring to fall from the top to the bottom of the Perspex tube is
noted. When an alternating current is passed through the coil, the time
taken for the aluminium ring to fall from the top to the bottom of the
Perspex tube is seen to increase slightly.

aluminium ring
gap
Perspex tube

coil

soft iron rod

Design an experiment to investigate how the time for the aluminium ring to fall
from the top to the bottom of the Perspex tube is affected by a chosen factor of
the experimental arrangement.

You should assume that the normal laboratory apparatus used in schools and
colleges is available. You may wish to draw a diagram to illustrate your answer.

9646(New Syllabus)/2/JC2Prelims/YJC2010
15
Your answer should contain details of

(a) the procedure to be followed including which measurements would be


taken,

(b) how you propose to use your measurements to obtain reliable results for
the investigation.

(c) any factors you will need to control and how you will do this.

(d) any particular features of your design which may improve the accuracy of
your experiment.
[12]

Diagram

9646(New Syllabus)/2/JC2Prelims/YJC2010
16
………………………………………………………………………………………

………………………………………………………………………………………

………………………………………………………………………………………

………………………………………………………………………………………

………………………………………………………………………………………

………………………………………………………………………………………

………………………………………………………………………………………

………………………………………………………………………………………

………………………………………………………………………………………

………………………………………………………………………………………

………………………………………………………………………………………

………………………………………………………………………………………

………………………………………………………………………………………

………………………………………………………………………………………

………………………………………………………………………………………

………………………………………………………………………………………

………………………………………………………………………………………

………………………………………………………………………………………

………………………………………………………………………………………

………………………………………………………………………………………

………………………………………………………………………………………

………………………………………………………………………………………

………………………………………………………………………………………

………………………………………………………………………………………
………………………………………………………………………………………
9646(New Syllabus)/2/JC2Prelims/YJC2010
17

………………………………………………………………………………………

………………………………………………………………………………………

………………………………………………………………………………………

………………………………………………………………………………………

………………………………………………………………………………………

………………………………………………………………………………………

………………………………………………………………………………………

………………………………………………………………………………………

………………………………………………………………………………………

………………………………………………………………………………………

………………………………………………………………………………………

………………………………………………………………………………………

………………………………………………………………………………………

………………………………………………………………………………………

………………………………………………………………………………………

………………………………………………………………………………………

………………………………………………………………………………………

………………………………………………………………………………………

………………………………………………………………………………………

………………………………………………………………………………………

………………………………………………………………………………………

………………………………………………………………………………………

END OF PAPER

9646(New Syllabus)/2/JC2Prelims/YJC2010
Candidate’s name ………………………………. CTG ……….…

YISHUN JUNIOR COLLEGE


JC 2 Preliminary Examinations 2010

PHYSICS 9745, 9646/3


HIGHER 2
26 August 2010
Paper 3
Candidates answer on the Question Paper. Thursday
No Additional Materials are required. 2 hours

YISHUN JUNIOR COLLEGE YISHUN JUNIOR COLLEGE YISHUN JUNIOR COLLEGE YISHUN JUNIOR COLLEGE YISHUN JUNIOR COLLEGE
YISHUN JUNIOR COLLEGE YISHUN JUNIOR COLLEGE YISHUN JUNIOR COLLEGE YISHUN JUNIOR COLLEGE YISHUN JUNIOR COLLEGE
YISHUN JUNIOR COLLEGE YISHUN JUNIOR COLLEGE YISHUN JUNIOR COLLEGE YISHUN JUNIOR COLLEGE YISHUN JUNIOR COLLEGE
YISHUN JUNIOR COLLEGE YISHUN JUNIOR COLLEGE YISHUN JUNIOR COLLEGE YISHUN JUNIOR COLLEGE YISHUN JUNIOR COLLEGE
YISHUN JUNIOR COLLEGE YISHUN JUNIOR COLLEGE YISHUN JUNIOR COLLEGE YISHUN JUNIOR COLLEGE YISHUN JUNIOR COLLEGE
YISHUN JUNIOR COLLEGE YISHUN JUNIOR COLLEGE YISHUN JUNIOR COLLEGE YISHUN JUNIOR COLLEGE YISHUN JUNIOR COLLEGE
YISHUN JUNIOR COLLEGE YISHUN JUNIOR COLLEGE YISHUN JUNIOR COLLEGE YISHUN JUNIOR COLLEGE YISHUN JUNIOR COLLEGE
YISHUN JUNIOR COLLEGE YISHUN JUNIOR COLLEGE YISHUN JUNIOR COLLEGE YISHUN JUNIOR COLLEGE YISHUN JUNIOR COLLEGE
YISHUN JUNIOR COLLEGE YISHUN JUNIOR COLLEGE YISHUN JUNIOR COLLEGE YISHUN JUNIOR COLLEGE YISHUN JUNIOR COLLEGE
YISHUN JUNIOR COLLEGE YISHUN JUNIOR COLLEGE YISHUN JUNIOR COLLEGE YISHUN JUNIOR COLLEGE YISHUN JUNIOR COLLEGE
YISHUN JUNIOR COLLEGE YISHUN JUNIOR COLLEGE YISHUN JUNIOR COLLEGE YISHUN JUNIOR COLLEGE YISHUN JUNIOR COLLEGE

INSTRUCTIONS TO CANDIDATES
Write your name and CTG in the spaces at the For Examiner’s Use
top of this page.
Section A
Write your answers in the spaces provided on
the question paper. 1 /8
For numerical answers, all working should be
shown clearly. 2 /8
3 /8
Section A
Answer all questions. 4 /8

Section B 5 /8
Answer any two questions.
Section B
6 /20
INFORMATION FOR CANDIDATES
The number of marks is given in brackets [ ] at 7 /20
the end of each question or part question.
8 /20
Penalty

Total /80

This question paper consists of 19 printed pages


2
For
Examiner’s
Data Use

speed of light in free space, c = 3.00 × 108 m s–1


permeability of free space, μo = 4π × 10–7 H m–1
permittivity of free space, εo = 8.85 × 10–12 F m–1
(1/(36π)) × 10–9 F m–1
elementary charge, e = 1.60 × 10–19 C
the Planck constant, h = 6.63 × 10–34 J s
unified atomic mass constant, u = 1.66 × 10–27 kg
rest mass of electron, me = 9.11 × 10–31 kg
rest mass of proton, mp = 1.67 × 10–27 kg
molar gas constant, R = 8.31 J K–1 mol–1
the Avogadro constant, NA = 6.02 × 1023 mol–1
the Boltzmann constant, k = 1.38 × 10–23 J K–1
gravitational constant, G = 6.67 × 10–11 N m2 kg–2
acceleration of free fall, g = 9.81 m s–2

Formulae
1 2
uniformly accelerated motion, s = ut + at
2
v2 = u2 + 2as
work done on/by a gas, W = pΔV
hydrostatic pressure, p = ρgh
gravitational potential, Gm
φ = −
r
Displacement of particle in s.h.m. x = xo sin ω t
velocity of particle in s.h.m., v = vo cos ω t
= ± ω ( x o2 − x 2 )
resistors in series, R = R1 + R2+……….
resistors in parallel, 1 1 1
= + + ........
R R1 R 2
electric potential, Q
V =
4πε o r
alternating current/voltage, x = xo sin ω t
transmission coefficient T = exp(−2kd)
8π 2 m(U − E )
where k =
h2
radioactive decay, x = xo exp(–λt)
0.693
decay constant, λ = t1
2

9745, 9646/3/JC2Prelims/YJC2010
[Turn over
3
For
Examiner’s
Section A Use
Answer all the questions in this section.

1 A car that is moving along a horizontal road may be considered to have three forces
acting on it as shown in Fig. 1.1 below.

Resultant force Z of road on car

Resultant force Y of
air on car

Fig. 1.1

Resultant force X of Earth on car

(a) Explain why X and Z are resultant forces. [2]

………………………………………………………………………………………………………

………………………………………………………………………………………………………

………………………………………………………………………………………………………

(b) The car and its contents have a total mass of 1200 kg. Force Y is horizontal and
has magnitude 2000 N. If the car is accelerating at 8 m s−2, calculate
1. the magnitude of force Z
2. the angle that Z makes with the road
[6]

magnitude of Z = ………..…….. N

angle = ………….. °

9745, 9646/3/JC2Prelims/YJC2010
[Turn over
4
For
Examiner’s
2 (a) (i) Derive the equation Use

Ep = mgh

where Ep is the change in potential energy of a mass m moved through a


vertical distance h near the Earth’s surface. [2]

(ii) Explain why the above equation is not valid for masses placed at very
large distances away from the Earth’s surface. [2]

......…………………………………………………………………………………………

………………………………………………………………………………………………

………………………………………………………………………………………………

(b) (i) Fig. 2.1 shows data for ethanol.

Density 0.79 g cm-3


Specific heat capacity of liquid ethanol 2.4 J g-1 K-1
Specific latent heat of fusion 110 J g-1
Specific latent heat of vaporisation 840 J g-1
Melting point -120 oC
Boiling point 78 oC

Fig 2.1

Use the data in Fig. 2.1 to calculate the thermal energy required to convert
1.0 cm3 of ethanol at 20 oC into vapour at its normal boiling point. [3]

9745, 9646/3/JC2Prelims/YJC2010
[Turn over
5
For
Examiner’s
Use

Thermal energy required = ………..…….. J

(ii) Suggest why there is a considerable difference in magnitude between its


specific latent heat of fusion and vaporization. [1]

………..……………………………………………………………………………………

………..……………………………………………………………………………………

………..……………………………………………………………………………………

3 In the recently opened Universal Studios, one of the roller coaster sections includes a
loop-a-loop that looks like the one shown in Fig. 3.1. The radius of the loop is 18.0 m and
the centre of the loop is 20.0 m from the ground.

18.0 m

Coaster car
20.0 m
Fig. 3.1

(a) If the mass of a coaster car is 250 kg and there is no support system holding the
car to the track, calculate

(i) the minimum speed at the top of the loop required for the car to stay in
contact with the track, [2]

Minimum speed at the top of loop = ………………….. m s−1


9745, 9646/3/JC2Prelims/YJC2010
[Turn over
6
For
Examiner’s
Use
(ii) the minimum speed the car needs to have when it enters the loop, if the
car loses 15.0 J of energy per unit length of track travelled, and [3]

Minimum speed when entering loop = …………………… m s−1

(iii) the vertical force exerted by the track on the car when the car just enters
the loop. [2]

Vertical force exerted by track = ………………….. N

(b) Explain why is the magnitude of the force calculated in (a) (iii) not equal to the
weight of the car. [1]

……….………..……………………………………………………………………………………

……….………..……………………………………………………………………………………

9745, 9646/3/JC2Prelims/YJC2010
[Turn over
7
For
Examiner’s
4 (a) Explain, using one of the observations of the photoelectric effect experiment, how Use
the effect illustrates the particulate nature of electromagnetic radiation. [3]

……….………..……………………………………………………………………………………

……….………..……………………………………………………………………………………

……….………..……………………………………………………………………………………

……….………..……………………………………………………………………………………

……….………..……………………………………………………………………………………

(b) The lifetime of an electron in the ground state of an atom is very long. Explain why
this implies that the energy of the ground state is well-defined. [2]

……….………..……………………………………………………………………………………

……….………..……………………………………………………………………………………

……….………..……………………………………………………………………………………

……….………..……………………………………………………………………………………

(c) Fig. 4.1 below shows how the potential energy Ep of an α-particle varies with
distance r along a line from the centre of a nucleus where Ro is the nuclear radius.
Using this graph, suggest why nuclei that emit high energy α-particles have short
half-lives. [3]

Ep

0 r
Ro
Fig. 4.1

……….………..……………………………………………………………………………………

……….………..……………………………………………………………………………………

……….………..……………………………………………………………………………………

……….………..……………………………………………………………………………………

……….………..……………………………………………………………………………………

……….………..……………………………………………………………………………………

9745, 9646/3/JC2Prelims/YJC2010
[Turn over
8
For
Examiner’s
Use
5 (a) As temperature increases, a semiconductor has greater conductivity whereas a
resistance wire has greater resistivity. Explain why this is so, in terms of charge
carriers. [2]

……….………..……………………………………………………………………………………

……….………..……………………………………………………………………………………

……….………..……………………………………………………………………………………

……….………..……………………………………………………………………………………

(b) Fig. 5.1 shows the possible energy band structure of an intrinsic semiconductor.
The valence band is completely filled and there is no charge carrier in the
conduction band. Explain in terms of charge carriers and energy band structure
how conduction is possible. [2]

Conduction band

Valance band Fig. 5.1

……….………..……………………………………………………………………………………

……….………..……………………………………………………………………………………

……….………..……………………………………………………………………………………

……….………..……………………………………………………………………………………

(c) Explain the origin of the depletion region in a p-n junction. [2]

……….………..……………………………………………………………………………………

……….………..……………………………………………………………………………………

……….………..……………………………………………………………………………………

……….………..……………………………………………………………………………………

9745, 9646/3/JC2Prelims/YJC2010
[Turn over
9
For
Examiner’s
(d) Draw a labelled circuit diagram to show how the depletion region of a p-n junction Use
can be reduced or removed. [2]

9745, 9646/3/JC2Prelims/YJC2010
[Turn over
10
For
Examiner’s
Section B Use
Answer two questions in this section.

6 (a) Explain what is meant by simple harmonic motion. [2]

……….………..……………………………………………………………………………………

……….………..……………………………………………………………………………………

……….………..……………………………………………………………………………………

……….………..……………………………………………………………………………………

(b)
Fig. 6.1
5.0 cm Q
P
water

Fig. 6.1 shows floating beads P and Q, 5.0 cm apart, on the surface of the water. They
will oscillate vertically when a wave passes through from P to Q. The displacement x
versus time t graph of P is shown in Fig. 6.2. The speed of the wave is 2.5 cm s−1.

x /cm

2.5−

1.0 2.0 3.0 t /s

−2.5 −
Fig. 6.2

(i) Write an equation to describe the variation of x of P with t. [1]

(ii) Calculate the maximum speed of P. [2]

Maximum speed = ………………….. m s−1

9745, 9646/3/JC2Prelims/YJC2010
[Turn over
11
For
Examiner’s
Use
(iii) The amplitude of Q is 2.0 cm. Calculate its maximum speed. [1]

Maximum speed = ……….………… m s−1

(iv) Calculate the phase difference between P and Q. [2]

Phase difference = …………………. radian

(v) Draw a displacement versus distance graph to show the damping of the
wave from P to Q and beyond. [2]

9745, 9646/3/JC2Prelims/YJC2010
[Turn over
12
For
Examiner’s
(c) White light has a wavelength range from 400 nm to 750 nm. A diffraction grating Use
with 6 × 105 lines per metre is placed at right angles to a ray of white light and
produces the first and second order spectra as shown in Fig. 6.3.

Second order spectrum


β

Fig. 6.3
First order spectrum
α

White light A

(i) Show, by calculation, that the angle β is greater than α. [4]

(ii) Show, by calculation, whether the second order spectrum overlaps with the
third order spectrum. [3]

9745, 9646/3/JC2Prelims/YJC2010
[Turn over
13
For
Examiner’s
(iii) State two advantages of analysing the light in the first order spectrum. [2] Use

………..……………………………………………………………………………………

………..……………………………………………………………………………………

………..……………………………………………………………………………………

………..……………………………………………………………………………………

(iv) State what would be seen at A. [1]

………..……………………………………………………………………………………

7 (a) (i) Define electric field strength and state the SI unit in which it is measured.
[2]

………..……………………………………………………………………………………

………..……………………………………………………………………………………

………..……………………………………………………………………………………

………..……………………………………………………………………………………

(ii) Two charged parallel plates are separated by a distance d and have a
potential difference V between them. Write down an expression for the
electric field strength of the uniform field between the plates. [1]

9745, 9646/3/JC2Prelims/YJC2010
[Turn over
14
For
Examiner’s
(b) Fig. 7.1 illustrates two plates A and B, at a distance 30 mm apart in a vacuum, Use
with plate A at a potential of − 4.2 V and plate B at a potential of − 2.0 V.

Plate A − 4.2 V

v2
v1
30 mm
θ Fig. 7.1
β2 β1

Plate B − 2.0 V

(i) Draw arrows to represent the electric field between the plates. [1]

(ii) Calculate the magnitude of this electric field strength. [2]

Magnitude of electric field strength = ………………

(iii) An electron, β1 is emitted from plate B in a direction normal to its surface


and moves directly towards plate A. Calculate the minimum velocity, v1
with which the electron needs to be emitted in order to reach plate A. [2]

Minimum velocity = ………………… m s-1

(iv) State, with a reason, how your answer in (b) (iii) may be affected if

1. the distance between the plates had been halved to 15 mm, while
keeping the potential difference the same. [2]

.……………………………………………………………………………………

……….……………………………………………………………………………

….…………………………………………………………………………………

..……………………………………………………………………………………

9745, 9646/3/JC2Prelims/YJC2010
[Turn over
15
For
Examiner’s
2. the potential of plate A is changed to + 4.2 V and potential of plate Use
B is changed to + 2.0 V while keeping the distance between the
plates at 30 mm. [2]

.……………………………………………………………………………………

……….……………………………………………………………………………

….…………………………………………………………………………………

..……………………………………………………………………………………

(v) Another electron, β2 is emitted from plate B with a velocity v2 =


1.1×106 m s−1 at an angle θ to the normal as shown in Fig 7.1. Determine
the largest possible angle θ which would allow the electron to just reach
plate A. [2]

Largest possible angle = ……….. °

9745, 9646/3/JC2Prelims/YJC2010
[Turn over
16
For
Examiner’s
(c) The potentials of plates A and B are now changed to + 0.29 V and 0 V Use
respectively. A charged micro-particle is suspended between the two plates as
shown in Fig. 7.2 below.

Plate A + 0.29 V

30 mm
Fig. 7.2

Plate B 0V

(i) State the sign of charge present on the particle. [1]

………..……………………………………………………………………………………

(ii) Find the charge to mass ratio, q/m of the particle. [2]

q/m = ………………… C kg-1

(iii) A student claims that the mass of the particle is 72 × 10−19 kg. Based on
your answer to part (c) (ii), explain why his answer is not valid. [3]

………..……………………………………………………………………………………

………..……………………………………………………………………………………

………..……………………………………………………………………………………

………..……………………………………………………………………………………

………..……………………………………………………………………………………

………..……………………………………………………………………………………

9745, 9646/3/JC2Prelims/YJC2010
[Turn over
17
For
Examiner’s
8 (a) Explain what is meant by nuclear fission. [2] Use

……….………..……………………………………………………………………………………

……….………..……………………………………………………………………………………

(b) A typical nuclear fission reaction that involves uranium-235 is represented by the
equation

235
92 U + n → 92
36 Kr + 141
56 Ba + 3 n

Data:
Nucleus Mass in u
235
92 U 235.044
92
36 Kr 91.910
141
Ba
56 140.916
n 1.009

92
(i) Deduce the number of protons and neutrons in the 36 Kr nucleus.
[2]

Number of protons = ………………….

Number of neutrons = ………………….

(ii) Calculate the energy released, in joules, in the above reaction. [3]

Energy released = ……………………… J

9745, 9646/3/JC2Prelims/YJC2010
[Turn over
18
For
235 −3 −1 Examiner’s
(iii) If a nuclear power station uses up U at a rate of 3.5 × 10
92 kg s and Use

has an efficiency of 23%, estimate the useful power output. [3]

Useful power output = ……………………… W

(iv) State two forms of energy of the product particles. [2]

……….………..……………………………………………………………………………………

……….………..……………………………………………………………………………………

……….………..……………………………………………………………………………………

(c) Uranium-234 is another isotope of uranium that is radioactive and has a half-life of
2.4 × 105 years. The daughter nuclei from the decay is Thorium-230 with the
emission of another particle X.

(i) Define half-life. [1]

………..……………………………………………………………………………………

………..……………………………………………………………………………………

(ii) Suggest what X is. [1]

………..……………………………………………………………………………………

(iii) Calculate the decay constant of Uranium-234. [1]

Decay constant = ……………………… year−1

9745, 9646/3/JC2Prelims/YJC2010
[Turn over
19
For
4 Examiner’s
(iv) Calculate the activity of a U-234 source after 8.7 × 10 years if it initially Use
has 5.5 × 1026 atoms. [3]

Activity = …………………… year−1

(v) Describe two applications of radioisotopes. [2]

………..……………………………………………………………………………………

………..……………………………………………………………………………………

………..……………………………………………………………………………………

………..……………………………………………………………………………………

………..……………………………………………………………………………………

~ END OF PAPER ~

9745, 9646/3/JC2Prelims/YJC2010
[Turn over
Suggested Answers to Paper 2 of Preliminary H2 Physics Exam 2010

1a) Every mass attracts every other mass with a force that is directly
proportional to the product of the two masses and inversely proportional to
the square of the distance between them.

GMm mv 2
1bi) =
r2 r

(6.67 × 10 )(5.97 × 10 )m =
−11 24
mv 2

(2.5 × 10 + 6.38 × 10 ) (2.5 × 10 + 6.38 × 10 )


3 6 2 3 6

v = 7900 m s-1

1bii) v = rω
⎛ 2π ⎞
( )
7900 = 2.5 × 10 3 + 6.38 × 10 6 ⎜ ⎟
⎝ T ⎠
T = 5076 s
= 1.41 hr
Since the period of satellite is less than 24 hours, it is not geostationary.

1biii) Towards the east direction.

2a)
⎛ 0.57 ⎞⎛ R ⎞
V =⎜ ⎟⎜ × 12 ⎟ ------- equation 1
⎝ 1.2 ⎠⎝ R + 1.5 + 2.3 ⎠
⎛ 0.68 ⎞⎛ R ⎞
V =⎜ ⎟⎜ × 12 ⎟ ------- equation 2
⎝ 1.2 ⎠⎝ R + 1.5 + 3.5 ⎠

Solving equation 1 & 2:

R = 2.42 Ω and V = 2.22 V

2b) There will be no change to the balance length

2c) If the resistance of rheostat is too high, the pd across the resistance wire
will be too low.
If the p.d. across the resistance wire is lower than the e.m.f. of test cell,
there will be no balance point.
3 a)i)
• magnetic force perpendicular to (direction of) motion/velocity
• force (or acceleration) is centripetal/ acts towards centre of curvature
• force does not change speed but causes direction of motion to change

ii) magnetic force = Bev


mv 2
centripetal force =
r

Ber
iii) v =
m
mv 9.11 ×10 -31 × 3.2 ×10 7
B= =
er 1.60 × 10 -19 × 25 ×10 −3

= 7.3 x 10−3 T

4a) Φ (= BA) = 45 × 10−3 × π × (70 × 10−3)2


= 6.9 × 10−4 Wb (6.93 × 10−4 Wb)

b)i)1. NΔΦ (= NBA − 0) = 850 × 6.93 × 10−4


= 0.59 Wb
2. Induced emf = 0.59 / 120 x 10-3 = 4.9 V

ii) Zero emf induced because the overall change in magnetic flux is zero

V 2 12 2
5a) i) and ii) Power = = = 24 W
R 6.0
P/W
b)
a.c.
48

24 d.c.

0.02 t/s
c) Power input = Power output

V1 I1 =V2 I2
24
I1 = = 0.104 A
230

6a) E2* to E1

6bi The electrons remain at n = 2 for a relatively long time so that population at n
= 2 is greater than at n = 1
b)ii) So that upon the arrival of a photon, the probability of stimulated emission
is greater than stimulated absorption.
6c) The light photons are reflected back and forth through the laser medium
using the reflective surfaces so that more atoms can be stimulated to emit
photons.

7(a) Orbits of the moons are not exact circles.

(b) Taking log, we will have lg T = −n lg r + lg k

If the graph of lg T against lg r is linear,

then the relation is valid .

(c) (i)1. Point C is plotted at (9.27, 1.22)


2. Best-fit line

(c) (ii) n = − 1.46 ; k = 5.01 × 10−13

(c) (iii) Yes, because n ≈ −1.5


⇒ T2 ∝ r3 which is Kepler’s Third Law.

(c) (iv) lg (130) = 2.11 ⇒ from graph, lg r = 9.85


⇒ r = 7.08 × 10 9 m

(c) (v)1. Tmoon = 27.3 days


⇒ lg Tmoon = 1.44 ⇒ point E is plotted at (8.58, 1.44)

(c) (v)2. The value of k is different for a different planet OR the Earth and Jupiter
has different mass values
lg (T / days)

4.0

3.5

3.0

2.5

2.0

1.5 E
C
1.0

0.5

0.0 lg (r / m)
8.0 8.5 9.0 9.5 10.0 10.5 11.0

−0.5

−1.0
8
Defining the problem

• control amplitude of current or pd and number of turns [P1] for current or


pd frequency variation [P1] OR
• control frequency of ac output and number of turns [P1] for variation of
current or pd amplitude variation [P1]
OR
• control frequency/ amplitude of ac output [P1] for variation in number of
turns [P1]

Total : 2 marks
Methods of data collection
• determine transit time for falling ring
- use suitable automatic data capture method triggered by signals from two
suitable sensors (photo gates allowed, ‘stopclock’ not allowed) M1
- connect sensors to data logger (not ‘computer’) M2

• To vary/maintain the amplitude of the ac supply


• accept (ac) ammeter, voltmeter or cro, or
• measure or maintain frequency of ac supply ( accept cro) M3
• use same start and stop positions for timing M4

Total : 4 marks

Method of analysis
• Propose t= axn
t – transit time
x – variable
a and n – constants A1
• Plot lg(t/s) against lg (x) an inspect results for mathematical link/plot graph
to determine whether any relationship exists between transit time and
variable
A2

Total : 2 marks

Additional details
any two of the following
1. reduce the uncertainty (change) in transit time because the ring may not
fall through with its axis vertical at all times. D1
• repeat timing and average results, or
• repeat timing to check for anomalous results , or
• increase length of coil (to maximise transit time) [Maximum 1 mark
for any given method to reduce uncertainty in timing ] D2

2. reduce the uncertainty in measurement made with cro D1


• sensible detail, e.g. use of large gain (y-sensitivity) to measure
amplitude or time-base to measure period D2

3. ensure ring falls smoothly D1


• by making tube and ring coaxial (vertical) D2

Safety precaution

1. To avoid the toppling of the retort stand, place bricks at the base of the
retort stand.

Total : 4 marks
Suggested Answers to Paper 3 of Preliminary H2 Physics Exam 2010

Section A

1a) X is the resultant of the weights of all parts of the car


Z is the resultant of the normal force on the car and frictional force on the tyres.

1b) 1. Zx − 2000 = 1200 (8) ⇒ Zx = 11600 N


Zy = 1200 (9.81) = 11772 N
Thus, Z = √(116002 + 117722) = 16.5 kN

2. Angle = tan−1 (11772/11600) = 45.4°

2ai)
Consider a body of mass m lifted by a distance h above its initial position at constant
velocity by an upward force.

F = mg
W = (F) h
Î Ep = mgh

2aii)
Gravitational field strength is not constant if object is moved over long distances.

2bi)
Q = thermal energy to raise temp to boiling point + thermal energy required to
vaporize the ethanol

Q = mc∆θ + m Lv
= ρ V (c ∆θ + Lv)
= (0.79 g cm-3) (1.0 cm3) [(2.4 Jg-1 K-1) (78-20 K) + 840 Jg-1]
= 774 J
2bii)
The change in volume/potential energy when ethanol vaporizes is much greater than the
change in volume/potential energy during melting.
OR
There is additional work done against the atmospheric pressure during vaporization.
mv 2 mv 2
3ai) At the top, N + mg = ⇒N= − mg
r r
To remain in contact with track, N > 0
mv 2
⇒ > mg
r
v2
> 9.81
18
Vmin = 13.3 m s-1

3aii) By COME, Ek(bottom) + Ep(bottom) = Ek(top) + Ep(top) + Elost


1
2 mvbottom =
2 1
2 (250)(13.3)2 + (250)(9.81)(36)
+ 15(π )(18)

Vbottom = 29.8 m s−1

mv 2
3aiii) At bottom, N − W =
r

N − (250)(9.81) =
(250)(29.8)
2

18
N = 14.8 × 103 N

3b) There must be a non-zero net force pointing towards the centre of loop to change
the direction of car.

4a) Description and explanation of any one of the following observations:


- no photoelectrons emitted if frequency is below a certain minimum
- maximum kinetic energy of photoelectrons is independent of the intensity of the
incident light
- graph of KEmax / stopping potential versus frequency of incident light is a
straight line
- photoelectric emission is almost instantaneous, even if intensity of light is very
low

4b) Long lifetime ⇒ Δt is large ⇒ ΔE is small ⇒ energy of level is well-defined

4c) High energy → barrier width shorter


→ transmission coefficient greater
→ shorter average waiting time before tunnelling
→ shorter half-life

5a) Semiconductor: At higher temperature, more charge carriers are released as


many electrons have enough energy to overcome band gap.
Resistance wire: its free electrons encounters more frequent collisions with lattice
reducing its ease to flow through.
5b) Given sufficient energy, the electrons in the valence band will be able to transit
up to conduction band.
The electrons in the CB and holes in the VB both contribute to the flow of current
when a voltage is applied.

5c) When the P type and N type semiconductors are brought together, the electrons
will diffuse into the P type and the holes diffuse into the N type.
Positive and negative core ions set up internal electric field that would prevent
further diffusion.
Hence depletion region is formed.

5d)

p n

Section B

6a) The acceleration is directly proportional to its displacement


and it is directed towards a fixed point.

6bi) X = 0.025 cos 2π t


6bii) Vmax = ωxo
= 2π(0.025) = 0.157 m s-1

6biii) Vmax = ωxo = 2π(0.020) = 0.126 m s-1

6biv) time taken for wave to go from P to Q = d/v = 5.0/2.5 = 2.0s


From the graph, points that are 2.0 s apart are in phase. Hence phase difference
is zero.

6bv) 1 mark for reduction in amplitude as d increases


1 mark for correct number of cycles (i.e 2) for d = 5 cm .

6ci)
wavelength Angle (n=1) Angle (n=2)
400 nm 13.89 28.69
750 nm 26.74 64.16
α 12.85
β 35.47

β is greater than α

6cii) At n=3, the 400 nm light is at an angle of 46.1 degrees.


It is before n =2 of 750 nm at an angle of 64.2 degrees.
Hence overlap occurs since n=3 occur at a smaller angle than n=2.
6ciii) No overlap
Brighter

6civ) White light

7ai) Electric field strength is the electric force per unit charge on a positive test charge
placed at that point. The SI unit is N C−1 or V m−1

Electric field strength = V / d

7bi)

Plate A −4.2 V

v2
30 mm v1

θ
β2 β1

Plate B −2.0 V

7bii) |E| = (4.2 – 2.0)/(30 x 10−3)


= 73 N C−1

7biii) loss in KE = gain in PE


½ mev12 = e(2.2)
v1 = 8.8 x 105 m s−1

7biv) 1. Even though the distance has been halved, as long as the p.d. remains
the same, the velocity v1 will remain the same .

2. As plate A is now at a higher potential than plate B, the electron emitted


from plate B will always be attracted to plate A. Thus the minimum
velocity required will be zero.

7bv) ½ me (1.1 x 106 cos θ)2 = 1.6.x 10−19(2.2)

cos θ = 0.7992
θ = 36.90.

7c) (i) Negative charge

(ii) Electric force upwards = gravitational force downwards


qE= mg
q/m = g/E
= 9.81/9.67 = 1.01448 = 1.01 C kg−1
(iii) If the mass is 72 x 10−19 kg, the amount of charge on the particle
=72 x 10−19 x 1.001448
=7.3 x 10−18 C
This is 7.3 x 10-18 / 1.6 x 10-19 = 45.6 times the charge of an electron
This is not valid as charge is quantized in sizes of 1.6 x 10-19 C.

8a) It is a nuclear reaction that involves the splitting of a heavy nucleus into smaller
parts.
Neutrons and gamma rays are usually produced OR energy is usually released
in the process.

8bi) There are 36 protons.


There are (92 – 36 =) 56 neutrons.

8bii) mass defect = (235.044 + 1.009) – (91.910 + 140.916 + 3×1.009)


= 0.2 u
Energy released = 0.2 (1.66 × 10-27) (3 × 108)2
= 2.99 × 10-11 J

8biii) rate of mass of U used that contribute to useful output = 0.23 × (3.5 × 10-3)
= 8.05 × 10-4 kg s-1
8.05 × 10 −4
useful power output = − 27
× 2.99 × 10 −11
235.044 × 1.66 × 10
= 6.17 × 1010 W

8biv) The energy is usually released as radiation and kinetic energy of fragments.

8ci) It is the time taken for the number of undecayed nuclei to decay by half.

8cii) Since the mass no of the particle is 234 – 230 = 4, it is most likely alpha particle /
helium nucleus.

ln 2
8ciii) decay constant =
240000
= 2.89 × 10-6 year-1

8civ) N = 5.5 × 1026 e-(2.89 × 10-6) (87000)


= 4.28 × 1026
A = (2.89 × 10-6) (4.28 × 1026)
= 1.24 × 1021 year-1

8cv) - Used as radioactive tracers in either medicine, agriculture or pipe leakage


- Used to monitor thickness of papers or sheets produced in factory
- Used of ionizing radiation in radiotherapy to treat cancer/tumor
- Used in smoke detectors to trigger alarm
- Used in carbon-dating to measure age of material

You might also like